Authors: Van Kleunen, Jonathan P. Title: Step-Up to USMLE Step 3, 1st Edition Copyright ©2009 Lippincott Williams & Wilkins > Front of Book > Editors
Author Jonathan P. Van Kleunen MD Resident, Department of Orthopaedic Surgery, The Hospital of the University of Pennsylvania, University of Pennsylvania Health System, Philadelphia, PA
Step-Up Series Editors Samir Mehta MD Resident, Department of Orthopaedic Surgery, University of Pennsylvania Health System, Philadelphia, Pennsylvania
Adam J. Mirarchi MD Resident, Department of Orthopaedics, Case Western Reserve University, Cleveland, Ohio
Lieutenant Edmund A. Milder MD Staff Pediatrician and General Medical Officer Naval Branch Health Clinic, La Maddalena, Italy
Authors: Van Kleunen, Jonathan P. Title: Step-Up to USMLE Step 3, 1st Edition Copyright ©2009 Lippincott Williams & Wilkins > Front of Book > Preface
Preface The U.S. Medical Licensing Examination (USMLE) Step 3 examination serves as the final hurdle in the transition from medical school to graduate medical education. This test builds on the principles of the Step 1 and Step 2 examinations and places them in a format focused on clinical decision making. The purpose of this test is to determine if the new physician is capable of applying the vast amount of information that he or she has learned from medical school into the treatment of patients. Unlike the prior two components of the USMLE examination, this test not only requires the examinees to recall information that they have learned but also requires them to be able to apply this basic knowledge to realistic clinical scenarios and to make the appropriate decisions based on their knowledge. Also unlike the previous USMLE tests, the Step 3 examination is taken following graduation from medical school and typically during the intern year of residency. This schedule can make preparation for the examination fairly difficult. Unlike during medical school, it is difficult or nearly impossible to block off large amounts of time to study for the test. Test takers must rely on their knowledge gained from the experiences of their intern year, supplemented by a concise, yet high-yield, review of testable information. Step-Up to USMLE Step 3 has been designed with both the test and the test taker in mind to provide a high-yield review for the USMLE Step 3 examination. Because of the emphasis on patient management in the examination, this book has been designed to provide a realistic clinical scenario for the many tested diagnoses. The organization of this book is unlike other reviews for the USMLE Step 3 in that nearly all of the information is presented in a case-based format. Each case consists of an extensive history and physical for a presenting patient, multiple diagnostic studies performed during the work-up, the diagnosis made due to this work-up, the treatment administered, and the follow-up of the patient following therapy. Following each case, each of the most likely conditions in the differential diagnosis is reviewed, and the reason why each diagnosis is correct or incorrect is explained. This type of review is a better reproduction of the thought processes of the new resident physician than a simple iteration of facts. It seeks to reproduce the clinical decisions that a resident physician is required to make on a daily basis. In addition, the limited amount of time available for test preparation has been considered during the writing of this book. The reviews of each diagnosis are concise and designed to include only the high-yield information that is vital on test day. It is not intended to be a self-assessment of testable information, but rather a presentation of how certain conditions may present and how they are appropriately managed. Please feel free to forward any comments to me at
[email protected]. I wish you all the best of luck in taking and passing this final stage of the USMLE examination series. Although the demands of residency are significant, the completion of this test is yet another hurdle that you will have cleared on the road to completing medical education and beginning a complete practice of medicine.
Authors: Van Kleunen, Jonathan P. Title: Step-Up to USMLE Step 3, 1st Edition Copyright ©2009 Lippincott Williams & Wilkins > Table of Contents > Chapter 1 - Cardiology
Chapter 1 Cardiology Basic clinical primer Cardiac Anatomy (Figure 1-1) Cardiac ischemia develops in the distribution of an occluded vessel.
The left anterior descending artery (LAD) is the most common site of coronary artery occlusion.
Cardiac Cycle (Figure 1-2) The pressure and volume of each heart chamber vary during the cardiac cycle depending on inflow, outflow, and the activity of contraction Pressure is equal between two adjacent chambers when the valve between them is open
Coronary arteries fill during diastole, while systemic arteries fill during systole. Conditions or drugs that reduce diastolic filling decrease coronary perfusion during a given period of time.
Cardiac Output (CO) The volume of cardiac outflow to the systemic vasculature over a given period of time CO is dependent on the rate of contraction (i.e., heart rate) and the volume of blood forced out of the left ventricle per contraction (i.e., stroke volume) Heart rate (HR) is number of contractions per unit of time and is expressed as beats per minute (bpm) Stroke volume (SV) is the change in volume from immediately before a contraction to the completion of the contraction SV = (end-diastolic volume) - (end-systolic volume) Dependent on contractility (i.e., force of heart's contraction), preload (i.e., the amount of stretching force on cardiac muscle fibers at the end of diastole), and afterload (i.e., the vascular resistance which ventricles must overcome to produce outflow) Increases with catecholamine release (e.g., epinephrine), increased intracellular calcium, decreased intravascular sodium, digoxin use, and stressful events (e.g., anxiety, exercise) Decreases with β-blocker use, heart failure, and hypoxia with acidosis
As heart rate increases, the time available for diastolic filling decreases
During exercise, cardiac output initially increases due to an increased stroke volume and then due to an increased heart rate.
Vascular Pressures Systolic blood pressure (SBP) is the maximum vascular pressure experienced during heart contraction; diastolic blood pressure (DBP) is the baseline vascular pressure between contractions Pulse pressure is the increase in blood pressure (BP) attributed to cardiac outflow during contraction Pulse pressure = (SBP) - (DBP) Mean arterial pressure (MAP) is the average BP considering that unequal amounts of time are spent in systole and diastole MAP = (DBP) + (1/3 × SBP) P.2
Figure 1-1 (A) Anterior and posterior views of the heart. LA, left atrium; LV, left ventricle; RA, right atrium; RV, right ventricle; SVC, superior vena cava. (B) Coronary artery hierarchy and regions of the heart supplied by branches. AV, atrioventricular; SA, sinoatrial. (See color image.)
Physiology of Heart Contraction Key principles: Increasing the end-diastolic ventricular volume causes an increased stretch on cardiac muscle fibers; this leads to an increase in the force of contraction (i.e., Frank-Starling relationship) The end-systolic volume and the pressure generated by the ventricles are dependent on afterload Increased contractility (i.e., force of contraction independent of preload and afterload) leads to increased muscle fiber tension during isometric contraction
Changes in volume-pressure relationships determine heart compliance; persistent high demands will lead to heart failure Contractile dysfunction Systolic dysfunction is caused by decreased contractility, increased preload, increased afterload, HR abnormalities, or chronic high output conditions (e.g., anemia, hyperthyroidism) and leads to an insufficient cardiac output for the systemic demand P.3
Figure 1-2 (A) Pressure relationships between left-sided heart chambers and the timing between normal heart sounds and the electrocardiogram for one full cardiac cycle. AV, aortic valve; ECG, electrocardiogram; LA, left atrium; LV, left ventricle; MV, mitral valve. (B) Normal left ventricular pressure-volume loop for one full cardiac cycle.
Diastolic dysfunction is caused by cardiac hypertrophy or restrictive cardiomyopathy and leads to decreased ventricular compliance, decreased ventricular filling, increased DBP, and decreased cardiac output
Heart failure may be considered an insufficient cardiac output for a given systemic demand, and it results from progressive contractile dysfunction.
Electrocardiogram (ECG) Recorded tracing of electrical impulses through the heart that is used to provide information regarding cardiac function (Figure 1-3) A systematic review increases the sensitivity to discerning abnormal patterns Confirm calibration on tracing Rhythm (i.e., regularity, reproduced patterns) Rate Intervals (e.g., PR, QRS, ST) P wave, QRS complex, ST segment, and T wave morphology Normal tracings P wave: atrial depolarization PR interval: conduction through the atrioventricular (AV) node (<0.2 sec) QRS interval: ventricular depolarization (<0.12 sec) ST interval: isoelectric ventricular contraction T wave: ventricular repolarization U wave: relative hypokalemia
Normal Cholesterol Function Cholesterols and triglycerides are carried by lipoproteins Increased low-density lipoprotein (LDL) levels lead to an increased coronary artery disease (CAD) risk P.4
Figure 1-3 General structure of the electrocardiogram tracing and the significance of specific regions.
Increased high-density lipoprotein (HDL) is protective Increased LDL and decreased HDL result from a diet high in fatty foods, tobacco use, obesity, alcohol use, diabetes mellitus (DM), and certain medications (e.g., oral contraceptive pills [OCPs], diuretics)
Preoperative Cardiac Risk Assessment Risk assessment for a surgical patient estimating the likelihood of an undesired cardiac event occurring as a result of surgery or anesthesia Considers measurable cardiac function, preexisting cardiac disease, age, and important comorbidities Young, healthy patients may be cleared for surgery with a normal ECG Older patients and/or those with comorbidities require a more extensive workup by a cardiologist and/or cardiac function testing Factors suggesting an increased risk of an adverse cardiac event Age: >70 years old Pulmonary function: forced expiratory volume in 1 sec/functional vital capacity (FEV 1/FVC) <70% expected, partial pressure of carbon dioxide (pCO2) >45 mm Hg, pulmonary edema Cardiac disease: MI within past 30 days, poorly controlled nonsinus arrhythmia, pathologic Q waves on the preoperative ECG, severe valvular disease, decompensated congestive heart failure with poor ejection fraction Renal insufficiency: creatinine (Cr) >2.0 or a 50% increase from baseline Surgery type: cardiac/vascular surgery or anticipated high blood loss High-risk patients should have their cardiac function optimized prior to elective surgery and should be made aware
of the increased risks if the surgery is emergent Perioperative β-blockers and postoperative noninvasive cardiac monitoring are frequently recommended for patients determined to have increased cardiac risk
The greatest risk for postoperative myocardial infarction (MI) is within the initial 48 hours after surgery.
Invasive Cardiac Monitoring Arterial line: constant access to artery (e.g., radial, femoral, axillary, brachial, dorsalis pedis) that allows accurate measurement of arterial BP and allows easy access to arterial blood for blood gas measurements Pulmonary artery catheter (i.e., Swan-Ganz catheter): catheter inserted through the (usually left) subclavian or (usually right) internal jugular vein that runs through the P.5 heart to the pulmonary artery; a transducer in the catheter allows the measurement of cardiac output, mixed venous oxygen (O2) saturation, systemic vascular resistance, and pressures in the right atrium and pulmonary artery; a balloon may be inflated at the catheter tip to fill the pulmonary artery lumen and to measure the wedge pressure (equivalent to left atrium pressure)
Fetal Circulation Gas exchange occurs in the uteroplacental circulation Fetal hemoglobin (Hgb) has a greater O2 affinity than adult Hgb and pulls O2 from maternal blood Umbilical arteries carry deoxygenated blood to placenta; umbilical veins carry oxygenated blood from placenta to portal system (Figure 1-4)
Figure 1-4 Diagram of fetal circulation. Arrows indicate the direction of blood flow; three shunts (ductus venosus, foramen ovale, ductus arteriosus) exist in utero but close shortly after birth.
P.6 Changes following birth Lung expansion causes an increased pulmonary blood flow leading to an increase in relative blood oxygenation
A decreasing serum level of prostaglandin E2 results in ductus arteriosus closure; umbilical cord clamping results in the end of placental circulation and an increase in systemic vascular resistance This increased vascular resistance, in turn, induces ductus venosus closure and umbilical artery and vein constriction Left atrial pressure increases (due to increased pulmonary blood flow), and umbilical circulation decreases, causing a decrease in inferior vena cava pressure Decrease in inferior vena cava and right atrial pressures leads to foramen ovale closure
Cardiac Transplantation Indicated for end-stage cardiac disease (e.g., CAD, congenital disease, cardiomyopathy) with an estimated survival of <2 years Contraindicated in patients with pulmonary hypertension, renal insufficiency, chronic obstructive pulmonary disease, or other terminal illnesses; smokers and patients >70 years old are also excluded Acute rejection is common Most deaths occur in the initial 6 months after transplant; 5-year survival is 70%
Case 1-1: “My father died when he was young” A 25-year-old man presents for the first time to a primary care provider (PCP) for a wellness check-up. He says that he has been in good health for several years and has only seen a doctor a few times since he was a child for the occasional illness. His only complaint is recurrent Achilles tendon and hamstring pain that occurs following significant exertion. He has noticed some small bumps in the back of his heels and knees in the past year in the regions of pain. He denies any medical conditions but states that his father died last year at a young age (i.e., 50 years old) from a heart attack. He has been thinking about this recently and decided he should probably find a regular internist to follow him. He denies any substance use. A review of systems and the remainder of his family history are negative. On examination, he appears to be a healthy individual of normal weight. Auscultation of his lungs and heart is normal. He has multiple small hard nodules behind his knees and heels. The following vital signs are measured: Temperature (T): 98.7°F, HR: 90 bpm, BP: 130/85 mm Hg, Respiratory rate (RR): 16 breaths/min
Differential Diagnosis Chronic tendonitis, crystal arthropathy (e.g., gout, pseudogout), hypercholesterolemia
Laboratory Data and Other Study Results Complete blood cell count (CBC): white blood cells (WBC) 8.2, Hgb 15.1, platelets (Plt) 320 Chemistry panel (Chem7): sodium (Na) 142 mEq/L, potassium (K) 4.1 mEq/L, chloride (Cl) 107 mEq/L, carbon dioxide (CO2) 26 mEq/L, blood urea nitrogen (BUN) 20 mg/dL, creatinine (Cr) 0.9 mg/dL, glucose (Glu) 85 mg/dL Lipid panel: Total cholesterol 350 mg/dL, LDL 290 mg/dL, HDL 45 mg/dL, triglycerides (Trig) 120 mg/dL Coagulation panel (Coags): protime (PT) 12 sec, International normalized ratio (INR) 1.0, partial thromboplastin time (PTT) 40 sec P.7 Urinalysis (UA): straw-colored, pH 5.0, specific gravity 1.010, no glucose/ketones/ nitrites/leukocyte esterase/hematuria/proteinuria Following these findings, the additional studies are performed:
ECG: normal sinus rhythm; no abnormal wave morphology Biopsy of heel lesion: large collections of cholesterol-laden material
Diagnosis Familial hypercholesterolemia (heterozygote)
Treatment Administered The patient was placed on a regular exercise regimen and a low-fat, low-cholesterol diet The patient was prescribed a regimen of simvastatin and ezetimibe
Follow-up A repeat lipid analysis in 1 month found the LDL to be 155 mg/dL The tendon xanthomas gradually regressed The patient was followed regularly to confirm an adequate reduction of his lipid levels and for cardiac screening
Steps to the Diagnosis Congenital hypercholesterolemia Inherited form of hypercholesterolemia in which a genetic defect causes abnormally high levels of total cholesterol, LDL, and/or triglycerides Patients have a significantly increased risk of ischemic heart disease Common types:
The majority of cases of hypercholesterolemia are acquired. Familial hypercholesterolemia (FH): autosomal dominant defect in LDL receptors with an associated increased total cholesterol and LDL; the disease is much more severe in homozygotes than heterozygotes Familial combined hyperlipidemia: hypercholesterolemia and hypertriglyceridemia associated with an increased hepatic production of apolipoprotein B-100 protein Familial defective apolipoprotein B-100: similar to FH except that the defect is in the LDL particle and not LDL receptor History: tendonitis around xanthomas, possible symptoms of ischemic heart disease (homozygotes experience symptoms in childhood, heterozygotes are usually asymptomatic until adulthood) Physical examination: xanthomas (i.e., cholesterol deposits in tendon or skin), xanthelasmas (i.e., deposits in eyelids), cholesterol emboli are seen in the retina on fundoscopic examination Tests: Total cholesterol and LDL >250 mg/dL in heterozygotes and >600 mg/dL in homozygotes Triglycerides are elevated (>200 mg/dL) in familial combined hyperlipidemia A biopsy of xanthomas will detect collections of cholesterol; genetic testing is available but typically unneeded for the diagnosis
Treatment: Healthy diet, exercise, and smoking cessation will decrease the risk for an ischemic event Patients should be educated about the role of risk factors Lipid-lowering agents are administered (consisting of one statin and at least one other drug) to achieve LDL levels below 160 mg/dL or lower depending on the number of cardiac risk factors (Figure 1-5, Table 1-1) Triglyceride-lowering drugs may be added if required P.8
Figure 1-5 Decision tree for screening for hypercholesterolemia. BP, blood pressure; CAD, coronary artery disease; DM, diabetes mellitus; HDL, high-density lipoprotein; HTN, hypertension; LDL, low-density lipoprotein; PVD, peripheral vascular disease.
Outcomes: Significantly increased risk for ischemic heart disease Prognosis is heavily dependent on the ability to control LDL levels Patients with homozygous disease have poorer prognosis Clues to the diagnosis: History: death of father at a young age Physical: xanthomatous nodules behind the knee and ankle Tests: increased total cholesterol and LDL, biopsy results
Blood for serum cholesterol levels should be collected from a fasting patient (12 to 14 hours) to minimize postprandial influence.
Acquired hypercholesterolemia Most common variant of hypercholesterolemia Typified by high LDL levels and/or low HDL levels with an associated increased risk of ischemic heart disease History: usually asymptomatic
Table 1-1 Lipid-Lowering Agents
Drug
HMG-CoA reductase inhibitors
Site of Ac tion
Liver
Effec t
Effec t
Effec t on
on LDL
on HDL
Triglyc erides
↓↓↓
↑
↓
Side Effec ts
Myositis, increased LFTs (must monitor)
(e.g., lovastatin, pravastatin, simvastatin)
Cholesterol absorption
Intestines
↓↓
—
—
Myalgias, possible increased LFTs
Fibric acids (e.g., gemfibrozil,
Blood (all
↓↓
↑
↓↓↓
Myositis, increased LFTs (must
fenofibrate)
stimulate
inhibitors (e.g., ezetimibe)
monitor)
lipoprotein lipase)
Bile acid sequestrants (e.g.,
GI tract
↓↓
—
—/↑
Bad taste, GI upset
Liver
↓↓
↑↑
↓
Facial flushing, nausea, paresthesias,
cholestyramine, colestipol, colesevelam)
Niacin
pruritus, increased LFTs, insulin resistance, exacerbates gout
HMG-CoA, 3-hydroxy-3-methyl-glutaryl-CoA; GI, gastrointestinal; HDL, high-density lipoprotein; LDL, low-density lipoprotein; LFTs, liver function tests; ↑, increased; ↑↑, more increased; ↑↑↑, most increased; —/↑, normal or increased.
P.9 Physical examination: the xanthomas seen in the congenital form are rare in the acquired form Tests: increased total cholesterol and LDL (typically <300 mg/dL), decreased HDL Treatment: Focus is initially on the prevention of cardiac disease and the reduction of cholesterol levels through exercise, low-fat and low-cholesterol diet, and smoking cessation Cholesterol-lowering medication is used in patients with increased cardiac risks or in patients unable to achieve ideal cholesterol levels with nonpharmacologic treatment alone (Table 1-1); the goal LDL level should be below 160 mg/dL in patients with less than two CAD risk factors, below 130 mg/dL in patients with two or more risk factors, and below 100 mg/dL in patients at a high risk for CAD (e.g., multiple risk factors, advanced age, very high LDL, peripheral vascular disease, diabetes mellitus, aortic aneurysms) Outcomes: the prevention of cardiac disease is contingent on the patient's ability to follow nonpharmacologic recommendations and the ability of medications to optimize LDL levels Why eliminated from differential: the LDL levels and presence of tendon xanthomas seen in this patient are
higher than what would be expected for the acquired form and are more consistent with congenital disease Chronic tendonitis More thorough discussion in Chapter 9 Why eliminated from differential: the presence of calcified tendon masses may be seen in chronic tendonitis, but this diagnosis cannot explain the patient's cholesterol levels Crystal arthropathy More thorough discussion in Chapter 9 Why eliminated from differential: tophi may be confused with xanthomas, but the biopsy confirms the identity of the nodules as xanthomas, and this diagnosis would not explain the patient's cholesterol levels
Case 1-2: “I get pains in my chest when I work in the yard” A 61-year-old man presents to his PCP with a complaint of occasional chest pain. He describes the pain as a dull ache near the center of his chest that radiates to his left shoulder. He says that the pain has occurred during the past 2 months when he was cutting down a couple of trees in his yard and chopping them into firewood (about four times). He has a sedentary desk job and denies pain at work or rest. He does not exercise and denies pain with walking or stair climbing. He denies nausea and vomiting and is unsure if the pain tends to follow eating. He says that he has had some mild dyspnea during the episodes that was slightly worse in the most recent episode during the previous weekend. He denies any paresthesias or paralysis. In each case, the pain has resolved within several minutes after resting. He denies any drug use. His past medical history is only significant for hypertension (HTN) that he has controlled with a diuretic. He notes that his brother had a heart attack 2 years ago. On examination, he appears to be comfortable and breathing normally. Chest auscultation finds normal breath sounds bilaterally, no murmurs, and an accentuated second heart sound. He has no abdominal pain on palpation. He remains asymptomatic during walking in the hallway. His neurologic examination is normal. The following vital signs are measured: T: 98.2°F, HR: 84 bpm, BP: 140/88 mm Hg, RR: 16 breaths/min
Differential Diagnosis Angina pectoris, panic disorder, esophagitis/gastritis/reflux disease, pulmonary embolism, pericarditis, aortic dissection P.10
Laboratory Data and Other Study Results Pulse oximetry: 99% on room air at rest and during ambulation Chem7: Na: 141 mEq/L, K: 4.0 mEq/L, Cl: 105 mEq/L, CO2: 27 mEq/L, BUN: 20 mg/dL, Cr: 1.0 mg/dL, Glu: 90 mg/dL Lipid panel: total cholesterol: 230 mg/dL, LDL: 165 mg/dL, HDL: 40 mg/dL, Trig: 125 mg/dL Chest x-ray (CXR): clear lung fields; normal cardiac shape and size ECG: normal sinus rhythm; normal wave and interval morphology Given the benign results above, the following tests are ordered on an outpatient basis: Exercise stress test: development of chest pain at 75% predicted maximum heart rate; 1 mm ST interval depression during this time; chest pain and ECG abnormality resolve following cessation of the test and a brief rest Nuclear perfusion test: technetium-99m-sestamibi is injected during the exercise stress test; scintigraphy
demonstrates a perfusion defect in the distribution of the left anterior descending coronary artery Echocardiography: 45% ejection fraction, mildly hypokinetic lateral wall, no effusions
Diagnosis Stable angina pectoris subsequent to ischemic heart disease
Treatment Administered The patient was placed on low-dose aspirin, atorvastatin, lisinopril, and atenolol The patient was prescribed sublingual nitroglycerin to be used as needed during any recurrent chest pain episodes The patient was provided education for healthy lifestyles
Follow-up Following the initiation of medicines, patient had an improved lipid profile and BP control A cardiac catheterization demonstrated a 40% blockage of the left anterior descending coronary artery The patient reported rare episodes of chest pain (one to two per year) that responded well to nitroglycerin The patient continued his cardiology follow-up to track any worsening of his condition
Steps to the Diagnosis Angina pectoris (stable) Temporary myocardial ischemia during exertion that causes chest pain Most commonly due to ischemic heart disease (a.k.a. CAD)
Common risk factors for atherosclerosis may be remembered by the mnemonic SHIFT MAID: Smoking, HTN, Insulin resistance (NIDDM), Family history, Triglycerides and cholesterol (high), Male, Age (increased), Inactivity, Diet.
High serum levels of homocysteine have been associated with a threefold risk of significant atherosclerosis. An inadequate supply of O2 for a given myocardial demand leads to myocardial hypoxia and an accumulation of waste products The vast majority of cases of ischemic heart disease arise from atherosclerosis of the coronary arteries (i.e., CAD), but arterial vasospasm (i.e., Prinzmetal angina) or valvular pathology are less common causes Atherosclerosis A gradual narrowing of arteries due to endothelial dysfunction, progressive formation of plaques (which consist of lipids and smooth muscle), and the associated inflammatory response Plaques may calcify, rupture, and thrombose, leading to further narrowing of arteries and progressive occlusion of blood flow The workup consists of identifying the associated degree of CAD P 11
Treatment focuses on the control of cardiac risk factors (e.g., control of HTN, hyperglycemia, and hypercholesterolemia), healthy diet, exercise, and smoking cessation History: substernal chest pain that may radiate to the left shoulder, arm, jaw, or back and lasts 1 to 5 minutes Physical examination: may be benign between episodes, an extra heart sound may be heard during episodes Tests: Severity is determined by quantifying the degree of cardiac ischemia Exercise stress test: The patient exercises on an aerobic fitness machine at increasingly strenuous workloads while the heart rate and ECG are constantly monitored The test is continued until the patient achieves 85% of the predicted maximum heart rate (i.e., approximately equal to [220 – patient age]) or the patient develops angina or signs of ischemia as seen on ECG ST segment depression of at least 1 mm is indicative of ischemia ST segment depression of at least 2 mm that is persistent for 5 minutes after cessation of the activity and is accompanied by a minimal increase in BP is indicative of severe disease Ischemic heart disease is diagnosed with signs of reproducible angina or obvious signs of ischemia at low workloads Nuclear exercise test: thallium-201 or technetium-99m-sestamibi is injected during exercise testing, and scintigraphy (planar or single-positron emission computed tomography [SPECT]) is performed to assess myocardial perfusion; it is used in cases of suspected ischemic heart disease in which the results of a regular exercise stress test are equivocal or if the performance of exercise is considered too risky for the patient (Color Figure 1-1) Exercise stress test with echocardiography: exercise stress testing is performed in conjunction with echocardiography to increase the sensitivity of detecting myocardial ischemia Pharmacologic stress testing: a cardiac inotrope (e.g., adenosine, dobutamine if there is comorbid pulmonary disease) is administered in place of exercise to increase myocardial demand; it is frequently performed in conjunction with a SPECT or performed in patients for whom comorbidities interfere with ability to perform exercise Positron emission tomography (PET) myocardial imaging: injection of positron-emitting isotopes with subsequent three-dimensional detection imaging is performed to evaluate the heart for perfusion defects and tissue viability Cardiac catheterization with coronary angiography: gold standard for identifying CAD, but more invasive than other techniques Treatment: Control of comorbid risk factors (e.g., hypercholesterolemia, diabetes mellitus, HTN, smoking) is important to reduce the risk of worsening cardiac ischemia β-blockers, angiotensin-converting enzyme inhibitors (ACE-I), and calcium channel blockers are all useful to decrease the workload of the heart (Table 1-2) Low-dose ASA decreases the risk of progressive coronary thrombosis Sublingual nitroglycerin may be taken as needed during episodes to reduce symptoms
Patients with multiple vessel disease or >50% occlusion of the left main coronary artery may require catheterized or surgical intervention Outcomes: prognosis is related to the ability to prevent progression of atherosclerosis and to control comorbid factors; left ventricle function and the degree of left coronary artery occlusion are the strongest predictors of long-term survival Clues to the diagnosis: History: exertion-induced chest pain with radiation to the left side that resolves with rest Physical: noncontributory Tests: increased cholesterol, exercise stress test, and nuclear perfusion test results P.12
Table 1-2 Common Medications Used in Ischemic Disease
Drug
ASA
Indic ations
MI prevention;
Cardiovasc ular Benefits
Contraindic ations
Decreases thrombosis risk
High risk of GI bleeding
Decreases thrombosis risk
High risk of GI bleeding
Decreases thrombosis risk
High risk of GI bleeding,
during and after MI
Clopidogrel
During angina and MI; after PTCA
GP IIb/IIIa inhibitor
During angina or
(e.g., abciximab,
MI; after PTCA or
eptifibatide)
thrombolysis
Nitroglycerin
During angina and
Decreases venous pressure causing decrease in
MI
preload and end-diastolic volume; as a result, blood
thrombocytopenia
Significant hypotension
pressure, ejection time, and O 2 consumption decrease while contractility and heart rate increase
β-Blocker (e.g.,
MI prevention;
Decreases blood pressure, contractility, heart rate,
Long-term use with PVD, asthma,
metoprolol, atenolol)
during angina;
and O 2 consumption; increases end-diastolic volume
COPD, DM (may mask hypoglycemia),
during and post-MI
and ejection time; dec reases mortality following MI
and depression (may worsen symptoms)
ACE-I (e.g., lisinopril,
Post-MI
enalapril)
Decreases afterload leading to decreased O 2
Pregnancy
consumption and blood pressure; dec reases mortality following MI; particularly helpful with comorbid CHF or DM
HMG-CoA reductase
Post-MI
inhibitors (e.g.,
Decreases risk of atherosclerosis progression by
Use of multiple lipid-lowering
lowering LDL
medications
Decreases risk of thrombus formation
Active hemorrhage
Decreases risk of thrombus formation
Pregnancy, active hemorrhage
simvastatin, atorvastatin)
Heparin
Immediately post-MI, inpatient setting
Warfarin
Post-MI
Morphine
During and
No direct cardiac benefit, but decreases pain during
immediately post-MI
MI leading to decreased heart rate, blood pressure,
Respiratory distress
and O 2 consumption
Thrombolytics (e.g.,
Immediately
Breaks up thrombus; dec reases mortality if used
t-PA, urokinase)
post-MI, inpatient
within 12 hr post-MI
High bleeding risk
setting
ACE-I, angiotensin converting enzyme inhibitors; ASA, acetylsalicylic acid; CHF, congestive heart failure; COPD, chronic obstructive pulmonary disease; DM, diabetes mellitus; ECG, electrocardiogram; GI, gastrointestinal; LDL, low-density lipoprotein; MI, myocardial infarction; PTCA, percutaneous transluminal coronary angioplasty; PVD, peripheral vascular disease.
Unstable angina pectoris Worsening angina that occurs at rest and is due to plaque rupture, hemorrhage, or thrombosis in the coronary arteries History: angina with any changes from the prior symptoms (e.g., worse pain, increased frequency, symptoms at rest, symptoms less responsive to prior treatment regimens) Physical examination: tachycardia or bradycardia, extra heart sounds, murmurs, or rales may be heard on auscultation Tests: ECG typically demonstrates ST interval depression and T wave flattening or inversion (similar to episodes of stable angina but more pronounced) Cardiac enzymes will likely show signs of cardiac distress (Figure 1-6) Creatine phosphokinase myocardial fraction (CK-MB) increases 2 to 12 hours post-MI, peaks in 12 to 40 hours, and decreases in 24 to 72 hours P.13
Figure 1-6 Changes in cardiac enzymes following the onset of chest pain in unstable angina and ST-elevation myocardial infarction. The rise and fall of CK-MB (solid line), LDH (dashed line), and troponin-I (dotted line) follow different patterns. CK-MB, creatine phosphokinase myocardial fraction; LDH, lactate dehydrogenase.
Lactate dehydrogenase (LDH) increases in 6 to 24 hours and peaks in 3 to 6 days (rarely used for diagnosis) Troponin-I increases in 3 hours, peaks in 6 hours, and gradually decreases over 7 days Treatment: Unstable angina is an indication for admission to the hospital Antiplatelet therapy consisting of ASA and either clopidogrel (if no intervention is planned) or a glycoprotein (GP) IIb/IIIA inhibitor (if catheterized intervention is planned) Heparin or low-molecular-weight heparin (LMWH) should be started to prevent thrombosis O2, nitroglycerin, β-blockers, ACE-I, and lipid-lowering drugs are administered to reduce cardiac workload Percutaneous transluminal coronary angioplasty (PTCA) Suggested in cases that are nonresponsive to medications A catheter is inserted through a femoral or brachial artery and maneuvered through the heart to the stenotic vessel A balloon on the catheter is inflated to dilate the stenosis Catheters may also be used for arthrectomy (i.e., plaque is shaved by a burr on the catheter) or stent placement (i.e., intravascular support structure) Coronary artery bypass grafting (CABG)
Considered for left main stenosis >50%, three-vessel disease, or a history of CAD and DM A donor vessel is grafted to the coronary artery to bypass the obstruction P.14
Table 1-3 Thrombolysis in Myocardial Infarction (TIMI) Risk Score
Risk Criteria1
Age ≥65 years
3+ risk factors for CAD (Figure 1-5)
Prior coronary occlusion ≥50%
ST interval depression or elevation on admission
2+ episodes of angina in past 24 hr
Elevated cardiac enzymes
Cumulative Sc ore
Rate of A dverse Event (%)2
0 or 1
4.7
2
8.3
3
13.2
4
19.9
5
26.2
6 or 7
40.9
Use of ASA with past 7 days
ASA, aspirin; CAD, coronary artery disease. 1
Value of 1 given if criteria present, 0 if criteria absent.
2
Defined as death, new or recurrent MI, or recurrent ischemia requiring revascularization.
The saphenous vein and internal mammary artery are most commonly used Outcomes: Short-term prognosis is frequently predicted by the thrombolysis in myocardial infarction (TIMI) risk score (Table 1-3) Long-term prognosis is best predicted by EF and the degree of residual ischemia detected on posttherapy stress testing Up to 10% of patients will have a myocardial infarction or will die within 6 months following the onset of new unstable angina Why eliminated from differential: the reversible nature of symptoms and the ECG changes and the normal cardiac enzymes are most consistent with stable angina and do not meet the definition of unstable angina
Myocardial ischemia is occasionally asymptomatic in patients with DM due to sensory neuropathy.
The differential diagnosis for chest pain may be remembered by the mnemonic CHEST PAIN: Cocaine/Costochondritis, Hyperventilation/ Herpes zoster, Esophagitis/ Esophageal spasm, Stenosis of aorta, Trauma, Pulmonary embolism/ Pneumonia/ Pericarditis/Pancreatitis, Angina/ Aortic dissection/ Aortic aneurysm, Infarction (myocardial), Neuropsychiatric disease (depression).
You must rule out a cardiac cause for chest pain with a formal stress test before considering alternative diagnoses.
An inconclusive noninvasive perfusion workup in the presence of reversible myocardial ischemia is an indication for cardiac catheterization.
The only pharmacologic treatments that have been shown to reduce the mortality of ischemic heart disease are aspirin (ASA), ACE-I, and β-blockers.
Any patient admitted to the hospital with suspected unstable angina or myocardial infarction must be worked up with serial ECGs and cardiac enzymes to elucidate the evolution of cardiac ischemia or to rule out the process.
Tight glycemic control is important to improving outcomes in diabetic patients following unstable angina.
One third of patients will have a myocardial infarction within 3 years after a new onset of unstable angina. Panic disorder More thorough discussion in Chapter 13 Why eliminated from differential: the direct relation of symptoms to exertion is more suggestive of a cardiac nature, and stress testing confirms the cardiac cause; the symptomatology does not meet the Diagnostic and Statistical Manual of Mental Disorders 4th edition (DSM-IV) criteria for panic disorder Esophagitis/gastritis/reflux disease More thorough discussion in Chapter 3 Why eliminated from differential: substernal chest pain may be seen in all of these conditions, and reflux may worsen with exertion, but the reversible nature of the symptoms is less consistent with a gastrointestinal (GI) etiology, and the stress test results further rule out these diagnoses
Symptoms of gastrointestinal reflux disease (GERD) or esophageal spasm may also improve with the administration of sublingual nitroglycerin. Pulmonary embolism More thorough discussion in Chapter 2 Why eliminated from differential: the normal pulse oximetry, normal CXR, and reversible nature of the
symptoms and ECG changes argue against this diagnosis Pericarditis More thorough discussion in later case Why eliminated from differential: although both conditions feature anterior chest pain that may be associated with dyspnea, the absence of diffuse ST elevations (ST depression is seen in this case), absence of pericardial effusion on the echocardiogram, absence of pericardial signs (e.g., friction rub, improvement in pain with leaning forward), and the stress test findings all argue against this diagnosis Aortic dissection More thorough discussion in later case Why eliminated from differential: for this diagnosis one would expect to find sudden constant “ripping” chest pain and mediastinal widening on the CXR, neither of which were present in this case P.15
Case 1-3: “I have severe tightness in my chest!” A 79-year-old woman is brought to an emergency department by her son with a complaint of significant tightness in her chest for a half hour. The patient says she was watching TV when the tightness started. She says the pain is in the center of her chest and feels like she is being squeezed too tightly. She says the pain radiates into her face. She feels very worried and is having some problems catching her breath. She feels slightly sick to her stomach. She denies any vomiting, syncope, paresthesias, or paralysis. She says that she has a history of high BP and heart disease (diagnosed from pharmacologic stress test 3 years ago) for which she takes an ACE-I and nitroglycerin as needed. She has had a few episodes of milder chest pain in the past month that responded to sublingual nitroglycerin. She has taken three nitroglycerin tablets on this occasion but has had no relief of her symptoms. On examination, she appears to be in moderate distress and is diaphoretic. Auscultation detects tachypneic breathing with faint bibasilar rales, tachycardia, and an extra heart sound. Her abdomen is nontender and soft. Her neurologic examination is normal. The following vital signs are measured: T: 97.9°F, HR: 110 bpm, BP: 90/70 mm Hg, RR: 24 breaths/min
Differential Diagnosis Myocardial infarction, unstable angina, pericarditis, aortic dissection, pneumothorax, pulmonary embolism, upper GI pathology
Laboratory Data and Other Study Results Pulse oximetry: 95% on room air, 99% on 2L O2 via nasal cannula CBC: WBC: 9.9, Hgb: 13.1, Plt: 240 Chem10: Na: 143 mEq/L, K: 3.4 mEq/L, Cl: 103 mEq/L, CO2: 32 mEq/L, BUN: 25 mg/dL, Cr: 1.1 mg/dL, Glu: 90 mg/dL, magnesium (Mg): 1.9 mg/dL, Calcium (Ca): 10.2 mg/dL, Phosphorus (Phos): 3.0 mg/dL Cardiac enzymes: creatine kinase (CK): 290 U/L, CK-MB: 4.5 ng/mL, troponin-I: 0.3 ng/mL ECG: sinus tachycardia; 3 mm ST interval elevation, Q waves, and T wave inversions in leads I, aVL, and V 2 to V6 CXR: normal heart shadow, slight midlung and bibasilar infiltrates
Diagnosis Myocardial infarction, anterolateral wall
Treatment Administered O2 via nasal cannula was continued, and intravenous (IV) fluids with K were initiated ASA, eptifibatide (a GP IIb/IIIa inhibitor), metoprolol, captopril, morphine, and IV heparin were administered One additional dose of sublingual nitroglycerin was administered The patient was rushed to cardiac catheterization and found to have an 80% occlusion of the left anterior descending artery; angioplasty was performed (heparin stopped afterward)
Follow-up The patient had symptomatic improvement following intervention Repeat cardiac enzymes in 8 and 16 hours demonstrated progressive increases in CK-MB and troponin levels P.16 Repeat ECGs over several days demonstrated the gradual normalization of ST intervals with maintenance of the abnormal Q waves The patient stabilized and gradually was weaned from IV medications A nuclear stress test performed prior to her discharge demonstrated a limited irreversible defect in the anterolateral cardiac wall The patient's outpatient medication regimen on discharge included ASA, clopidogrel, metoprolol, lisinopril, and atorvastatin The patient experienced a recurrent severe episode of chest pain 3 months after the episode described ab1ove, cardiac arrest occurred, and patient was unable to be resuscitated
Steps to the Diagnosis Myocardial infarction (MI) Cardiac tissue death resulting from ischemia caused by the occlusion of coronary arteries or vasospasm It is often secondary to thrombus formation following plaque rupture Risk factors: increased age, HTN, hypercholesterolemia, family history of CAD, DM, and tobacco use, male gender, postmenopausal status History: chest pain (tightness or “elephant on chest”) in the same distribution as prior anginal pain, dyspnea, diaphoresis, nausea, and vomiting Physical examination: tachycardia, hypotension, pulmonary rales, new S 4 heart sound, new systolic murmur Tests: Cardiac enzymes (typically performed three times, every 6 to 8 hours) will show progressive increases and follow general trends (Figure 1-6) ECG shows ST elevation, T wave changes, and possible new arrhythmia, left bundle branch block (LBBB), or pathologic Q waves (Figure 1-7 and Table 1-4) CXR may show pulmonary edema Treatment:
Acute
Figure 1-7 Acute myocardial infarction shown on an electrocardiogram. Note the ST elevation in leads V 2 to V5 suggesting anterior wall involvement.
P.17
Table 1-4 Relation of ECG Changes to Location of Infarct
ECG Leads W ith Changes
Area of Infarc t
Coronary Artery Branc h
V 2, V 3, V 4
Anterior
Left anterior descending
V 1, V 2, V 3
Septal
Left anterior descending
II, III, aVF
Inferior
Posterior descending or marginal branch
I, aVL, V 4, V 5, V 6
Lateral
Left anterior descending or circumflex
V 1, V 2 (frequent comorbid inferior MI)
Posterior
Posterior descending
ECG, electrocardiogram; MI, myocardial infarction.
Give O2, ASA, clopidogrel or a GP IIb/IIIa inhibitor, β-blockers, ACE-I, nitroglycerin, morphine (pain control may decrease cardiac demand); achieve tight glycemic control in diabetics with insulin; administer potassium and magnesium to keep levels above 4.0 mEq/L and 2.0 mEq/L, respectively Consider thrombolysis (e.g., tissue plasminogen activator (t-PA), urokinase) or angioplasty if the patient presents in the initial 12 hours after the start of an MI Heparin should be administered for up to 48 hours to reduce thrombus risk (if angioplasty is performed, stop heparin after the procedure) If the patient is hypotensive, stop nitroglycerin and give IV fluids Give antiarrhythmics for frequent premature ventricular complex (PVCs) contractions or ventricular tachycardia (Vtach) Subacute Perform a stress test 5 days post-MI to assess future risk; if the test is suggestive of an increased risk for repeat MI, perform cardiac catheterization to measure vessel patency and consider possible PTCA or CABG if significant occlusion is found Long term Risk reduction medications should include low-dose ASA or clopidogrel, β-blockers, ACE-I, K-sparing diuretic, and a beta-hydroxy-beta-methylglutaryl-coenzyme A (HMG-CoA) reductase Exercise, smoking cessation, and dietary modifications are also important for risk reduction Outcomes: there is a 30% acute mortality with 50% of survivors being rehospitalized within 1 year and 10% dying within 1 year; complications include infarct extension, arrhythmias, myocardial dysfunction, papillary muscle necrosis, wall rupture, aneurysm, mural thrombus, pericarditis, and Dressler syndrome (i.e., fever, pericarditis, and increased erythrocyte sedimentation rate [ESR] 2 to 4 weeks post-MI) Clues to the diagnosis: History: severe sudden chest pain radiating to the face, history of heart disease Physical: tachycardia, extra heart sound Tests: ECG findings
CK-MB is considered by some to be the first positive test in the initial 24 hours post-MI, but troponin I is considered the most overall sensitive test within 7 days following an MI.
Because general CK will be increased with significant muscular trauma or degradation, CK-MB is a better indicator of cardiac muscle damage.
Treatment for MI may be remembered by the mnemonic BeMOAN: Beta-blocker, Morphine, O2, ASA/ ACE-I, Nitroglycerin.
The greatest risk of sudden cardiac death is in the first few hours post-MI due to Vtach, Vfib, or cardiogenic shock.
The greatest risk of ventricular wall rupture is 4 to 8 days post-MI.
Unstable angina More thorough discussion in prior case Why eliminated from differential: the presentations may be similar, but the greater severity of symptoms, changes in cardiac enzymes and ECG findings (e.g., ST elevation, not depression), and the presence of an irreversible defect on follow-up nuclear stress testing all suggest that MI is the diagnosis Pericarditis More thorough discussion in later case Why eliminated from differential: ST elevation may be seen in both, but the severity of symptoms, positive cardiac enzymes, and nuclear stress test findings are all more consistent with MI P.18 Aortic dissection More thorough discussion in later case Why eliminated from differential: both will have chest pain, but aortic dissection will have a normal ECG and will not feature the cardiac enzyme and stress test changes seen in this case Pneumothorax More thorough discussion in Chapter 2 Why eliminated from differential: the CXR in this case does not show collapsed lung fields, and the ECG, cardiac enzyme, and stress test findings in this case would not be expected for this diagnosis Pulmonary embolism More thorough discussion in Chapter 2 Why eliminated from differential: some degree of desaturation may be seen in both conditions, but the ECG, cardiac enzyme, and stress test findings would not be expected for this diagnosis Upper GI pathology (e.g., gastritis, esophagitis) More thorough discussion in Chapter 3 Why eliminated from differential: chest pain may be common to both, but the ECG, cardiac enzyme, and stress test findings would not be expected for this diagnosis
Case 1-4: “I'm following up for my heart medication” A 58-year-old man presents to his cardiologist in follow-up for his ischemic heart disease. He has a history of stable angina that has been treated with a regimen of ASA, atenolol, enalapril, and simvastatin, and nitroglycerin as needed. He had experienced a couple of anginal episodes in the previous month that were treated by increasing the dose of his atenolol. He is following up at this time to make sure that he has had no new episodes since his last appointment. He reports no new episodes of angina and no new problems. On examination, he appears comfortable. His lung fields sound clear. Chest auscultation finds a slightly irregular, slow-sounding heart rate and no extra heart sounds. His peripheral pulses are strong with a similar irregularity. His abdomen is nontender with normal bowel sounds. A neurovascular examination is normal and includes palpable pulses in all four extremities. The following vital signs are measured: T: 98.6°F, HR: 60 bpm, BP: 105/80 mm Hg, RR: 14 breaths/min
Differential Diagnosis
Bradycardia, heart block (first, second, third degree)
Laboratory Data and Other Study Results Chem10: Na: 142 mEq/L, K: 3.6 mEq/L, Cl: 103 mEq/L, CO2: 28 mEq/L, BUN: 15 mg/dL, Cr: 0.8 mg/dL, Glu: 90 mg/dL, Mg: 2.1 mg/dL, Ca: 10.0 mg/dL, Phos: 3.0 mg/dL ECG: HR 60 bpm; progressive lengthening of PR interval with occasional skipped QRS intervals
Diagnosis Second-degree heart block, Mobitz I (i.e., Wenckebach block)
Treatment Administered The patient's dose of atenolol was decreased slightly P.19
Follow-up The patient was seen 1 week after starting the new dosing of atenolol The patient continued to have no symptoms, and a repeat ECG demonstrated normal sinus rhythm
Steps to the Diagnosis Heart block Impaired myocardial contraction that occurs when electrical impulses encounter tissue that is electronically unexcitable and result in arrhythmia Variants First degree: caused by increased vagal tone or functional conduction impairment Second degree, Mobitz I (i.e., Wenckebach): caused by intranodal or His bundle conduction defect, drug effects (e.g., β-blockers, digoxin, calcium channel blockers), or increased vagal tone Second degree, Mobitz II: cause is an infranodal conduction problem in the bundle of His or Purkinje fibers Third degree (i.e., complete): cause is an absence of conduction between the atria and ventricles History: all first- and second-degree variants are generally asymptomatic, but patients with third-degree block have syncope and dizziness Physical examination: some irregularity in the heart rate may be detected, but third-degree block is more irregular and also is associated with hypotension Tests: First degree – ECG shows PR longer than 0.2 sec (Figure 1-8A) Second degree, Mobitz I (i.e., Wenckebach): progressive PR lengthening until a skipped QRS occurs; PR progression then resets and begins again (Figure 1-8B) Second degree, Mobitz II: randomly skipped QRS without changes in the PR interval (Figure 1-8C) Third degree (i.e., complete): no relationship between P waves and QRS (Figure 1-8D)
Treatment: First degree: none required Second degree, Mobitz I (i.e., Wenckebach): adjust doses of medication associated with heart block; no other treatment required unless symptomatic bradycardia (treat with pacemaker) Second degree, Mobitz II: ventricular pacemaker Third degree (i.e., complete): avoid medications affecting atrioventricular conduction; insert ventricular pacemaker Outcomes: reversible first-degree and second-degree block are benign; patients with irreversible second- or third-degree block are at an increased risk of developing ventricular arrhythmias but do very well if treated with a pacemaker Clues to the diagnosis: History: recent increase in atenolol dose Physical: slow heart rate Tests: ECG findings
Common causes of syncope include cardiac dysfunction (e.g., aortic stenosis, bradycardia, arrhythmia), vasovagal response, hypotension, hypoglycemia, seizures, and cerebrovascular ischemia.
Common presentations of syncope may be remembered by the mnemonic SUCH DROPS: Seizures, Unexplained (50% presentations), Cardiac, Hypoglycemia, Drugs, Reflex mechanisms (vasovagal response), Orthostasis (hypotension), Psychogenic, Stroke. Sinus bradycardia Heart rate below 50 bpm due to an increased vagal tone or nodal disease Risk factors: history of CAD, advanced age History: typically asymptomatic with occasional weakness or syncope Physical examination: noticeably slow heart rate on auscultation Tests: ECG demonstrates sinus rhythm slower than 50 bpm Treatment: stop precipitating medications; pacemaker required if irreversible and symptomatic Outcomes: irreversible cases carry a predisposition to the development of other arrhythmias Why eliminated from differential: the ECG in this case was pathognomonic for second-degree block Mobitz I and was not a sinus rhythm P.20
Figure 1-8 (A) Primary heart block: regular PR interval prolongation without skipped QRS. (B) Secondary Mobitz I heart block: progressive lengthening of the PR interval until QRS is skipped. (C) Secondary Mobitz II heart block: regular PR interval with random skipped QRS. (D) Tertiary heart block: no relationship between P waves and QRS.
Case 1-5: “My mother feels that her heart is racing” A 75-year-old woman is an inpatient at a hospital 2 days after a successful femoral-popliteal artery bypass. This surgery was performed because of significant atherosclerotic peripheral vascular disease causing severely impeded blood flow to her right leg. The patient has done well since the completion of surgery. While her daughter is visiting her, the patient describes the sudden feeling that her heart is racing. She denies chest pain, dizziness, or difficulty breathing. She says that this experience has never happened previously. She has a history of peripheral vascular disease, HTN,
and mild CAD. Her medications include ASA, clopidogrel (started postoperatively), metoprolol, and enalapril. Prior to surgery she was taking pentoxifylline but stopped postoperatively. She was a heavy smoker for several years but quit 2 years ago. On examination, she appears to be slightly nervous. Her lung fields P.21 sound clear. Auscultation of the chest reveals a rapid heart beat with significant irregularity; there are no extra heart sounds. The remainder of her examination is normal. The following vital signs are measured: T: 98.9°F, HR: 120 bpm, BP: 145/83 mm Hg, RR: 18 breaths/min
Differential Diagnosis Atrial fibrillation, paroxysmal supraventricular tachycardia, multifocal atrial tachycardia, atrial flutter
Laboratory Data and Other Study Results CBC: WBC: 10.1, Hgb: 9.8, Plt: 310 Chem10: Na: 139 mEq/L, K: 3.8 mEq/L, Cl: 108 mEq/L, CO2: 24 mEq/L, BUN: 17 mg/dL, Cr: 1.1 mg/dL, Glu: 93 mg/dL, Mg: 1.9 mg/dL, Ca: 10.1 mg/dL, Phos: 3.1 mg/dL Coags: PT: 13 sec, INR: 1.1, PTT: 43 sec ECG: HR 120 bpm but highly irregular rate; no discernible P waves in tracing
Diagnosis Atrial fibrillation
Treatment Administered 20 mEq K, 1 mg Mg, and 5 mg metoprolol IV were all administered following review of the ECG Transfusion of one unit of packed red blood cells (PRBCs) was started An additional 5 mg IV metoprolol was administered when patient failed to convert to sinus rhythm following first dose
Follow-up The patient converted into sinus rhythm following the second dose of metoprolol The patient's K was maintained at 4.0 mEq/L, Mg was maintained at 2.0 mg/dL, and Hgb was maintained at 10.0 The patient's dose of metoprolol was titrated to maintain regular sinus rhythm and reduce BP The patient remained in sinus rhythm for the remainder of the admission and was discharged home on an appropriate dose of β-blocker The patient was followed closely postoperatively to confirm no recurrent episodes of atrial fibrillation
Common risk factors for Afib are remembered by the mnemonic CHAPTERS: Coronary artery disease, Hypertension, Anemia, Pulmonary disease, hyperThyroid, Ethanol, Rheumatic heart disease, and Sepsis.
Steps to the Diagnosis Atrial fibrillation (Afib)
A lack of coordinated atrial contractions with independent sporadic ventricular contractions Due to rapid, disorderly firing from a second atrial focus Risk factors: left atrial enlargement, CAD (particularly MI), HTN, anemia, valvular disease, pericarditis, chronic obstructive pulmonary disease, pulmonary embolism hyperthyroidism, rheumatic heart disease (RHD), sepsis, alcohol use, electrolyte abnormalities (e.g., Mg, phosphorus, K, calcium) History: possibly asymptomatic or dyspnea, chest pain, and palpitations Physical examination: irregularly irregular pulse and heart beat Tests: ECG shows no discernible P waves and an irregular QRS rate (Figure 1-9) Treatment: Rate control via calcium channel blockers, β-blockers, or digoxin (Table 1-5) P.22
Figure 1-9 Atrial fibrillation—irregular QRS rate and no discernible P waves.
The initial attempt at rate control and cardioversion is 5 mg IV metoprolol IV metoprolol may be repeated twice before attempting to control the rate with IV diltiazem Anticoagulation (i.e., heparin, then warfarin) is frequently required unless the arrhythmia is identified and treated within 24 hours of onset Electric cardioversion Electric cardioversion may be performed if the presentation is within the initial 2 days Cardioversion may be performed in delayed presentations if an absence of thrombi is confirmed by transesophageal echocardiogram (TEE) If the patient presents after 2 days or if thrombus is seen on echocardiogram, then anticoagulate and wait 3 to 4 weeks before attempting cardioversion AV nodal ablation may be considered for recurrent cases Outcomes:
Complications include increased risk of MI, heart failure, and mural thrombus formation due to atrial blood stasis Dislodgement of a mural thrombus may cause stroke The risk of complications correlates with the amount of time not spent in normal sinus rhythm Clues to the diagnosis: History: sudden palpitations Physical: irregularly irregular heart rate Tests: ECG findings
In a patient with Afib, echocardiogram should be performed before cardioversion to rule out mural thrombus formation.
Table 1-5 Classes of Antiarrhythmic Medications
Class
General Mec hanism of Ac tion
Examples
Potential Uses
IA
Na channel blockers (prolong action potential)
Quinidine, procainamide
PSVT, Afib, Aflutter, Vtach
IB
Na channel blockers (shorten action potential)
Lidocaine, tocainide
Vtach
IC
Na channel blockers (no effect on action potential)
Flecainide, propafenone
PSVT, Afib, Aflutter, PSVT
II
β-blockers
Propanolol, esmolol, metoprolol
PVC, PSVT, Afib, Aflutter, Vtach
III
K channel blockers
Amiodarone, sotalol, bretylium
Afib, Aflutter, Vtach (not bretylium)
IV
Calcium-channel blockers
Verapamil, diltiazem
PSVT, MAT, Afib, Aflutter
Other
K channel activation, decrease in intracellular cAMP
Adenosine
PSVT
Afib, atrial fibrillation; Aflutter, atrial flutter; cAMP, cyclicadenosine monophosphate; K, potassium; MAT, multifocal atrial tachycardia; Na, sodium; PSVT, paroxysmal supraventricular tachycardia; PVC, premature ventricular contraction; Vtach, ventricular tachycardia.
P.23
Figure 1-10 Mechanism of atrioventricular nodal reentry tachycardia. (A) Action potential reaches division in conduction pathway with both fast and slow fibers. (B) Conduction proceeds quickly down fast pathway to reach distal fibers and also proceeds up slow pathway in retrograde fashion. (C) Impulse returns to original division point after fibers have repolarized allowing a reentry conduction loop and resultant tachycardia. AV, atrioventricular.
Paroxysmal supraventricular tachycardia (PSVT) Tachycardia (i.e., HR >100 bpm) arising in the atria or atrioventricular (AV) junction Occurs most commonly in young patients with otherwise healthy hearts Cause frequently is reentry anomaly AV nodal reentry: presence of both slow and fast conduction pathways in AV node; conduction proceeds quickly through the fast pathway and progresses up slow pathway in retrograde fashion to create a conduction loop and reentrant tachycardia (Figure 1-10) AV reentry as found in Wolff-Parkinson-White (WPW) syndrome: similar to AV nodal reentry, but instead of fast and slow pathways existing in the AV node, a separate accessory conduction pathway exists between the atria and ventricles that returns a conduction impulse to the AV node to set up a reentry loop (Figure 1-11) History: sudden palpitations, possible dyspnea or syncope Physical examination: sudden onset of tachycardia Tests: ECG shows P waves hidden in T waves, normal QRS morphology, and rate 150 to 250 bpm; ECG shows a delta wave (i.e., slurred upstroke of the QRS) and a shortened PR interval in cases of WPW syndrome (Figure 1-12) Treatment: carotid massage or Valsalva maneuver may halt an acute arrhythmia, but cardioversion (frequently using a calcium channel blocker) is required in cases of hemodynamic instability; pharmacologic therapy (e.g., adenosine or calcium channel P.24 blocker for AV nodal reentrant tachycardia and adenosine or type IA or IC antiarrhythmic for WPW syndrome) or catheter ablation of accessory conduction pathways is frequently used for long-term control in symptomatic patients (Table 1-5)
Figure 1-11 Mechanism of atrioventricular reentry tachycardia as seen for Wolff-Parkinson-White syndrome. (A) Action potential passes through the AV node and encounters an accessory pathway during conduction to the ventricles. (B) Accessory pathway conducts action potential back to AV node. (C) Return of secondary action potential to AV node completes reentry loop and results in tachycardia. AV, atrioventricular node; AP, accessory pathway.
Figure 1-12 Wolff-Parkinson-White syndrome on electrocardiogram. Note the presence of delta waves, slurred upstrokes preceding each QRS that are characteristic of the condition.
Outcomes: structural cardiac anomalies may contribute to the development of other arrhythmias; risk of sudden cardiac death is minimal Why eliminated from differential: the ECG in the case is highly suggestive of Afib, and the clinical scenario (i.e., elderly patient with HTN, anemia, and CAD) is more consistent with AFib than PSVT
Antiarrhythmics other than class IA or IC are contraindicated for WPW syndrome because they may speed up conduction through the accessory pathway. Multifocal atrial tachycardia (MAT)
Tachycardia caused by several ectopic foci in the atria that discharge automatic impulses (i.e., multiple pacemakers), which causes tachycardia History: usually asymptomatic Physical examination: tachycardia detected on pulse monitoring and heart auscultation Tests: ECG shows tachycardia with a HR >100 bpm and a variable morphology of P waves (at least three varieties) (Figure 1-13) Treatment: calcium channel blockers or β-blockers acutely; catheter ablation or surgery may be performed to eliminate abnormal pacemakers Outcomes: persistent untreated tachycardia may lead to cardiomyopathy Why eliminated from differential: the ECG in the case is diagnostic for Afib and does not show the P wave findings expected for MAT P.25
Figure 1-13 Multifocal atrial tachycardia (MAT). Note the variety in the shape of P waves and PR intervals and the irregular ventricular rate.
Figure 1-14 Atrial flutter—rapid sawtooth P waves preceding QRS.
Atrial flutter (Aflutter) Tachycardia due to the rapid firing of an ectopic focus in the atria in a distinct pattern Risk factors: CAD, congestive heart failure (CHF), chronic obstructive pulmonary disease (COPD), valvular disease, pericarditis History: possibly asymptomatic, possible palpitations or syncope Physical examination: rapid regular tachycardia Tests: ECG shows regular tachycardia >150 bpm with a set ratio of P waves to QRS and a sawtooth pattern of P waves (Figure 1-14) Treatment: Rate control with calcium channel blockers or β-blockers Electrical or chemical (class IA, IC, or III antiarrhythmics) cardioversion should be performed if unable to control the rate with medication Catheter ablation may be performed to remove an ectopic focus in some cases Outcomes: cases in which successful catheter ablation is performed have rare recurrence; stroke is a possible complication when anticoagulation is not prescribed in cases with delayed treatment Why eliminated from differential: ECG in this case is diagnostic for Afib and does not show the characteristics typical of Aflutter
Case 1-6: “My husband just collapsed!” A 68-year-old man is an inpatient on the medicine service following a moderate MI. He initially presented to the emergency department with chest pain and was found to have mild ST interval elevation in his anterior ECG leads. He was treated pharmacologically and was stabilized on a regimen of ASA, metoprolol, captopril, and nitroglycerin. A nuclear stress test performed 5 days following admission demonstrated a new area of irreversible ischemia in the anterior cardiac wall. Subsequent cardiac catheterization detected a 65% occlusion of the main left coronary artery that was treated with angioplasty and stenting. His telemetry has been stable for several days, and his cardiac enzymes have begun to trend downward. He was scheduled to be discharged to a cardiac rehabilitation facility today or
tomorrow. During a visit, his wife runs to the nurses' station to tell them that her husband had gotten out of bed to go to the bathroom when he suddenly collapsed. When the response team enters the room, they find the patient lying on the ground next to his bed. On examination, he is anxious-appearing, pale, and interacts poorly with the medical team. Chest auscultation detects shallow, rapid breathing and a rapid heart rate. A faint pulse is detectable, and his fingers have minimal capillary refill. Following this initial assessment, the patient becomes unresponsive without respiration or a palpable pulse. The following vital signs are measured: T: 97.9°F, HR: 150 bpm, BP: 90/60 mm Hg, RR: 24 breaths/min initially then 0 breaths/min P.26
Differential Diagnosis Ventricular tachycardia, premature ventricular complexes, ventricular fibrillation, pulseless electrical activity, asystole, PSVT, Afib, Aflutter, MAT
Laboratory Data and Other Study Results STAT arterial blood gas with electrolytes (Super Gas): pH: 7.37, partial pressure of oxygen (pO2): 80 mm Hg, pCO2: 48 mm Hg, Bicarb: 23 mEq/L, O2 sat: 86%, Hgb: 13.1, Na: 135 mEq/L, K: 3.4 mEq/L, Cl: 108 mEq/L, Glu: 85 mg/dL, Ca: 9.8 mg/dL ECG: HR 150; regular rate; wide QRS intervals; no apparent relation between P waves and QRS intervals
Diagnosis Pulseless ventricular tachycardia
Treatment Administered The cardiac arrest protocol was initiated, and the patient was intubated and bag ventilated Defibrillation was attempted with 200 J shock; no response Defibrillation was repeated with 300 J shock leading to the return of spontaneous contractions and circulation The patient was transferred to the cardiac intensive care unit (CICU) to continue close cardiac monitoring, K repletion, and titration of antiarrhythmic medications
Follow-up Cardiac enzymes demonstrated continued elevation of cardiac enzymes with slow trend downward An internal defibrillator was implanted to prevent recurrent episodes of ventricular arrhythmia The patient was eventually able to be discharged to cardiac rehabilitation
Torsades de pointes is Vtach with a sine wave morphology that carries a poor prognosis and may rapidly convert to Vfib; Mg may be useful in its treatment.
Steps to the Diagnosis Ventricular tachycardia (Vtach) Series of three or more premature ventricular beats with HR 160 to 240 bpm Risk factors: CAD, prior MI
History: possibly asymptomatic if brief or palpitations, syncope, hypotension, impaired consciousness, and dyspnea Physical examination: tachycardia, tachypnea, loss of pulse as cardiac output worsens Tests: ECG shows series of regular, wide QRS intervals independent of P waves (Figure 1-15)
Figure 1-15 Ventricular tachycardia—wide, rapid QRS with no discernible P waves.
P.27
Figure 1-16 Assessment and initial treatment protocol for the unresponsive patient. CPR, cardiopulmonary resuscitation; EMS, emergency medical services; PEA, pulseless electrical activity; Vfib, ventricular fibrillation; Vtach, ventricular tachycardia.
Treatment: Any minimally responsive or unresponsive patient necessitates initiation of the advanced cardiac life support (ACLS) protocol (Figure 1-16) Pulseless Vtach requires defibrillation and/or antiarrhythmic medications (Figure 1-17) Vtach with a pulse is treated with antiarrhythmic medications alone Conscious patients should be treated with antiarrhythmic medications (class IA, IB, II, or III) Recurrent Vtach requires ablation of an ectopic focus of signal or the implantation of an internal defibrillator (senses ventricular arrhythmia and automatically releases electric pulse to restore normal rhythm) Outcomes: prognosis correlates with the degree of cardiac infarction at the time of onset; approximately 30% rate of sudden cardiac death within 2 years of onset
Clues to the diagnosis: History: recent MI, sudden collapse Physical: tachycardia, tachypnea progressing to no breathing, pulselessness Tests: ECG findings
All cardiovascular emergency protocols are based on the ABC concept: Airway, Breathing, Circulation. Ventricular fibrillation (Vfib) Lack of ordered ventricular contraction that leads to no cardiac output and is rapidly fatal Risk factors: CAD, recent MI, recent Vtach History: chest pain and palpitations followed by syncope and impaired consciousness Physical examination: hypotension, pulselessness, lack of respirations, poorly discernible heart sounds P.28
Figure 1-17 Treatment protocol for ventricular fibrillation or pulseless ventricular tachycardia. ABC, airway, breathing, circulation; CPR, cardiopulmonary resuscitation; IV, intravenous; Vfib, ventricular fibrillation; Vtach, ventricular tachycardia.
Tests: ECG shows totally erratic tracing with no P waves or QRS intervals (Figure 1-18) Treatment: immediate cardiopulmonary resuscitation with electric (+/- pharmacologic) defibrillation (Figure 1-17); implantation of an internal defibrillator is needed by patients at a high risk for recurrence Outcomes: poorer prognosis with high recurrence rate if occurring more than 48 hours after a MI; acute survivorship decreases with delays to treatment or an inability to restore a stable cardiac output
Figure 1-18 Depicts initial torsades de pointes with degeneration into ventricular fibrillation.
P.29 Why eliminated from differential: the ECG in this case demonstrated intact QRS intervals and was not erratic as would be expected for Vfib; given the pulseless nature of the patient in this case, it is quite possible that a transition to Vfib would soon occur
The treatment protocol for Vfib or Vtach may be remembered by the mnemonic “Shock, Shock, Shock, Everybody Shock, Anybody Shock, Little Shock, Big Shock, Mama Shock, Papa Shock, Baby Shock”: Shock (200 J)→Shock (300 J)→Shock (360 J)→ Epinephrine→Shock (360 J)→ Amiodarone→Shock→ Lidocaine→Shock→Bretylium→Shock→Magnesium→Shock→ Procainamide→Shock→Bicarbonate (sodium)→Shock. Premature ventricular complexes (PVCs) Isolated ectopic beats from a ventricular origin Frequently benign; may be caused by hypoxia, abnormal serum electrolyte levels, hyperthyroidism, caffeine use History: usually asymptomatic, but possible palpitations or syncope Physical examination: ectopic beat may be heard if auscultation is performed at time of ectopy
Tests: ECG shows early and wide QRS without a preceding P wave that is followed by a brief pause in conduction Treatment: None is necessary if the patient is healthy Replenish low K or Mg levels Consider β-blockers in patients with CAD Outcomes: may be a precursor to more severe arrhythmias in patients with CAD Why eliminated from differential: the maintained series of wide QRS intervals seen on the ECG in this case defines the difference between PVCs (only a few isolated ectopic beats) and Vtach (a series of ectopic beats)
PVCs become concerning for the development of other ventricular arrhythmias if there are more than three PVC/min.
Common causes of PEA may be remembered by the 6 Hs and 4 Ts: Hypovolemia, Hypoxia, Hyperkalemia, Hypokalemia, Hypomagnesemia, Hydrogen ions (acidosis), Tension PTX, Thrombosis (CAD or PE), Tablets (drugs) Tamponade (cardiac). Pulseless electrical activity (PEA) Loss of a pulse and no signs of systemic circulation in the presence of recordable cardiac electrical activity History: syncope, unresponsiveness Physical examination: absent pulse, respiratory arrest, no discernable heart beats, hypotension Tests: ECG will demonstrate some form of electrical activity (e.g., several morphologies of QRS intervals and T waves) Treatment: pharmacologic cardioversion (Figure 1-19)
Figure 1-19 Treatment protocol for pulseless electrical activity. ABC, airway, breathing, circulation; CPR, cardiopulmonary resuscitation; IV, intravenous.
P.30
Figure 1-20 Treatment protocol for asystole. ABC, airway, breathing, circulation; CPR, cardiopulmonary resuscitation; IV, intravenous.
Outcomes: poor prognosis without rapid correction Why eliminated from differential: despite the pulseless nature of the patient in this case, the ECG displaying regular wide QRS intervals is indicative of Vtach (i.e., more organized than PEA)
For pulseless electrical activity, think PEA: Pulseless→Epinephrine and Atropine. Asystole Absence of any electric cardiac activity History: syncope, unresponsiveness Physical examination: no signs of circulation, respiratory arrest, no heart sounds, hypotension Tests: ECG shows absence of any electrical activity Treatment: transcutaneous pacing or pharmacologic stimulus of contraction (Figure 1-20) Outcomes: poor prognosis without immediate restoration of cardiac output Why eliminated from differential: the presence of electric activity on the ECG in this case rules out this diagnosis
Cardiac arrest lasting more than ten minutes without any cardiac output is generally considered consistent with severe brain injury or brain death.
Paroxysmal supraventricular tachycardia, atrial fibrillation, atrial flutter, multifocal atrial tachycardia More thorough discussion in prior case Why eliminated from differential: the ECG in this case shows wide QRS intervals indicative of a ventricular arrhythmia as opposed to what would be expected for PSVT (i.e., narrow QRS intervals), Afib (i.e., normal QRS intervals with a completely irregular rate), Aflutter (i.e., pattern of multiple sawtooth P waves for every QRS), and MAT (i.e., variable P waves with normal QRS intervals)
Case 1-7: “I've been having a hard time breathing since 2 days ago” A 63-year-old man presents to the emergency department with a complaint of progressive dyspnea over the past 2 days. He says that he first noticed this symptom at night when trying to sleep and required extra pillows to help him breathe easier. He also reports P.31 feeling more tired than usual and coughing frequently. He denies recent illness, fevers, dizziness, chest pain, nausea, vomiting, or neurologic symptoms. He says that he has a “bad heart” for which he takes a couple of medications. He ran out of his medications 1 week ago and did not refill them because “they make [him] go to the bathroom too much.” He does not remember the names of his medications. He drinks two alcoholic beverages about 4 days per week. On examination, he is in moderate distress. He pauses often during the history to catch his breath. He has notable distension of his neck veins to the level of his mandibular angle. Auscultation detects decreased breath sounds bilaterally with diffuse rales that are more pronounced in the lung bases. He has an extra heart sound and a palpable impulse near his left axillary line. Slight hepatomegaly is palpable. He has 2+ edema in both legs. His neurologic examination is normal. The following vital signs are measured: T: 98.4°F, HR: 90 bpm, BP: 160/95 mm Hg, RR: 26 breaths/min
Differential Diagnosis Heart failure, unstable angina, MI, chronic obstructive pulmonary disease, pneumonia, idiopathic pulmonary fibrosis
Laboratory Data and Other Study Results Pulse oximetry: 92% on room air, 98% on 4L O2 via nasal cannula CBC: WBC: 7.6, Hgb: 13.5, Plt: 320 Chem10: Na: 137 mEq/L, K: 4.2 mEq/L, Cl: 107 mEq/L, CO2: 22 mEq/L, BUN: 30 mg/dL, Cr: 1.3 mg/dL, Glu: 102 mg/dL, Mg: 1.9 mg/dL, Ca: 9.5 mg/dL, Phos: 3.3 mg/dL Cardiac enzymes: CK: 235 U/L, CK-MB: 2.1 ng/mL, troponin-I: 0.2 ng/mL Liver function tests (LFTs): alkaline phosphatase (AlkPhos): 130 U/L, alanine aminotransferase (ALT): 65 U/L, aspartate aminotransferase (AST): 50 U/L, total bilirubin (TBili): 1.3 mg/dL, Direct bilirubin (DBili): 0.3 mg/dL, Indirect bilirubin (IBili): 1.0 mg/dL Brain natriuretic peptide (BNP): 610 pg/mL ECG: HR: 90 bpm; QRS intervals in I and V 1 to V5 with >30 mm amplitude; no ST interval changes CXR: cardiomegaly; bilateral hilar and basilar fluffy infiltrates; increased marking of pulmonary vessels
Diagnosis Congestive heart failure with acute exacerbation due to medical noncompliance
Treatment Administered The patient was admitted for respiratory support and diuresis O2 was administered to keep his O2 saturation above 97% Intravenous furosemide was administered to induce diuresis A review of medical records notes the current pharmacologic regimen of furosemide, enalapril, atenolol, ASA, and digoxin; the patient was restarted on his outpatient medications
Follow-up The patient experienced significant diuresis following the initiation of furosemide and experienced symptomatic improvement The patient was discharged to home following symptomatic improvement and improvements in BNP level, CXR appearance, and O2 saturation The patient was educated about the importance of medications, low-salt diet, and recognition of exacerbations P.32
Steps to the Diagnosis Congestive heart failure (CHF) Progressive decrease in cardiac output due to systolic and/or diastolic dysfunction Left-sided pathology Left ventricle unable to produce adequate CO Blood backs up leading to pulmonary edema that eventually causes pulmonary HTN Progressive left ventricular hypertrophy (LVH) to compensate for poor output causes eventual failure because the heart is unable to keep pace with systemic need for CO Right-sided pathology Increased pulmonary vascular resistance leads to right ventricular hypertrophy (RVH), hepatojugular reflux, and systemic venous stasis with associated edema Most commonly due to left-sided failure (Eisenmenger syndrome); also may be due to unrelated pulmonary HTN, valvular disease, or congenital defects Risk factors: CAD, HTN, valvular disease, cardiomyopathy, COPD, drug toxicity, alcohol use History: fatigue, dyspnea on exertion, orthopnea, paroxysmal nocturnal dyspnea, nocturia, cough; symptoms are more severe during exacerbations Physical examination: displaced point of maximum impulse, S3 heart sound, jugular vein distention (JVD), peripheral edema, hepatomegaly Tests: Plasma brain natriuretic peptide (BNP) and N-terminal pro-BNP will be increased with left ventricle dysfunction Liver enzymes (e.g., ALT, AST) are frequently increased and are a sign of congestive hepatomegaly and cardiac-induced cirrhosis
BUN and Cr are slightly elevated with a BUN: Cr ratio >20 and suggest chronic decreased cardiac output and poor renal perfusion ECG may show signs of ventricular hypertrophy (e.g., increased QRS amplitude, heart block, low baseline voltage) CXR shows cardiac enlargement, Kerley B lines (i.e., increased marking of lung interlobular septa due to pulmonary edema), and cephalization of pulmonary vessels (i.e., increased marking of superior pulmonary vessels due to congestion and stasis) Echocardiogram is useful to assess heart chamber size and function (i.e., can help differentiate systolic and diastolic dysfunction) Treatment: CHF due to systolic dysfunction First medications given are typically loop diuretics (decrease heart preload) and either ACE-I or angiotensin receptor blockers (ARB) (decrease preload and afterload and increase cardiac output) β-blockers may be added once a stable ACE-I dose is prescribed and should not be administered during acute exacerbations Digoxin (increases contractility) may be added to the regimen to improve symptoms but has not demonstrated an ability to improve mortality Spironolactone or vasodilators may be added for persistent symptoms CHF due to diastolic dysfunction BP is controlled with calcium channel blockers, ACE-I, or ARB β-blockers are useful to control the heart rate and to decrease cardiac workload in stabilized patients K-sparing diuretics should be given to reduce cardiac hypertrophy caused by aldosterone Recombinant BNP (i.e., nesiritide) has been shown to improve outcomes during severe acute exacerbations Underlying conditions should be treated The patient should adhere to a low-salt diet Progressive cases may require cardiac transplant or implantation of an assistive device (indicated for an ejection fraction below 35%) P.33 Outcomes: Approximately half of patients with CHF will eventually die from a ventricular arrhythmia Hospital admission is associated with up to a 20% mortality rate Mortality increases to 40% in the presence of a comorbid MI and 80% in the presence of hypotension Clues to the diagnosis: History: dyspnea, nocturnal dyspnea, orthopnea, recent medication noncompliance Physical: tachypnea, rales and decreased breath sounds on auscultation, jugular venous distention, hepatomegaly, peripheral edema Tests: mildly elevated LFTs, increased BNP, CXR appearance, increased lead amplitude on ECG
COPD may lead to right-side hypertrophy that ends in right-sided failure (i.e., cor pulmonale)
The presence of an S3 extra heart sound is the most consistent finding of CHF
ACE-I and spironolactone have been shown to decrease mortality in CHF and should be incorporated into the treatment plan when appropriate, as other medications have not been proven to reduce mortality.
Frequent causes of CHF exacerbations are remembered by the mnemonic A SMITH PEAR: Anemia, Salt, MI, Infection, Thyroid (high or low), HTN, Pericarditis, Endocarditis, Arrhythmia, Rx (not taking medications). Unstable angina/MI More thorough discussion in prior case Why eliminated from differential: the absence of suggestive ECG changes and normal cardiac enzymes despite the 2-day history of symptoms rules out these diagnoses Chronic obstructive pulmonary disease (COPD) More thorough discussion in Chapter 2 Why eliminated from differential: the patient does not have the characteristic appearance of a COPD patient, CXR is not consistent with COPD, and BNP levels suggest a heart failure etiology Pneumonia More thorough discussion in Chapter 2 Why eliminated from differential: the lack of fevers or an elevated WBC count argue against this diagnosis, and the BNP level and ECG appearance suggest a cardiac etiology Idiopathic pulmonary fibrosis (IPF) More thorough discussion in Chapter 2 Why eliminated from differential: the CXR in the case is less consistent with IPF than CHF, the onset of symptoms is too rapid for progression of IPF, and the BNP level and ECG appearance suggest a cardiac etiology
Case 1-8: “I was advised to see you by my insurance company” A 30-year-old woman presents to a cardiologist for assessment of a heart murmur. As part of the application for a life insurance policy, she was required to see a general internist for a history and physical. The only abnormal finding on the examination was the presence of a blowing murmur during chest auscultation. The insurance company doctor advised her to follow-up with her regular physician or a cardiologist. Because she has been in good health and has not seen a physician for several years, she elected to see a cardiologist to assess the cause of the murmur. She describes herself as a healthy individual and has had no medical problems besides an occasional cold for the past 10 years. She sees a gynecologist on a yearly basis but has not seen an internist for over 10 years. She denies any dizziness, syncope, chest pain, palpitations, gastrointestinal symptoms, or neurologic symptoms. She exercises about three times per week and has not noticed any significant dyspnea beyond what she considers normal. She takes no medications besides a multivitamin. She drinks alcohol socially. On examination, she is a healthy appearing, normal weight patient in no distress. Her lung fields are clear bilaterally. Chest auscultation detects an audible blowing holosystolic murmur that
radiates from the cardiac apex to the axilla, a faint extra heart sound, and appreciable splitting of her second heart sound. She has no edema or abnormal findings in any of her extremities. Her neurologic examination is normal. The following vital signs are measured: T: 98.4°F, HR: 72 bpm, BP: 110/80 mm Hg, RR: 14 breaths/min P.34
Differential Diagnosis Mitral regurgitation, mitral stenosis, aortic regurgitation, aortic stenosis, pulmonary stenosis
Laboratory Data and Other Study Results ECG: normal sinus rhythm; normal wave morphology and amplitude CXR: possible left atrial and ventricular enlargement but otherwise normal findings Echocardiogram: prolapse of the mitral valve leaflets with associated mild mitral regurgitation; left atrium and ventricle size within upper half of normal range; normal EF (65%)
Diagnosis Mitral regurgitation due to mitral valve prolapse
Treatment Administered The patient was placed on daily low-dose ASA The patient was prescribed prophylactic antibiotics to be taken prior to any intervention with a risk of bleeding (e.g., dental procedures) She was permitted unrestricted activity
Follow-up The patient was reassessed in 6 months and then every year to follow for any worsening in her condition
Steps to the Diagnosis Mitral regurgitation (MR) Incompetency of the mitral valve due to degeneration or local myocardial dysfunction that allows backflow of blood into the left atrium Common causes include mitral valve prolapse (i.e., “floppy” valve), RHD, papillary muscle dysfunction, endocarditis, and left ventricle dilation Most commonly a chronic condition History: asymptomatic in early and mild cases, palpitations, dyspnea on exertion, orthopnea, and paroxysmal nocturnal dyspnea seen in worse cases Physical examination: harsh blowing holosystolic murmur radiating from apex to axilla, S 3, widely split S2, midsystolic click (Figure 1-21)
Figure 1-21 Common murmurs associated with valvular diseases.
P.35
Figure 1-22 Apical view echocardiogram demonstrating mitral valve prolapse with associated mitral regurgitation. Both mitral valve leaflets (arrows) are prolapsed into the left atrium and are well beyond the normal level of the mitral annulus (dashed line). LA, left atrium; LV, left ventricle; RA, right atrium; RV, right ventricle.
Tests: CXR may detect LVH and left atrial enlargement; echocardiogram demonstrates regurgitation and is useful to determine the cause of MR, the size of the left heart chambers, and the ejection fraction (Figure 1-22) Treatment: Prophylactic antibiotics are given for any intervention with risk of bleeding to reduce risk of accidental
valve infection (risk is higher with any valvular disease) Anticoagulation is prescribed to reduce the risk of thrombus formation (ASA if healthy, warfarin with a history of Afib or stroke) Vasodilators are given if the patient is symptomatic at rest Severe or acute cases require surgical repair or reconstruction Outcomes: mild cases have few complications; more severe cases are associated with the development of CHF and pulmonary disease Clues to the diagnosis: History: history of heart murmur Physical: systolic murmur radiating to the axilla, split S 2, midsystolic click Tests: echocardiogram results
An echocardiogram should be performed anytime you suspect a valvular lesion. Mitral stenosis (MS) Decreased mitral valve motion or inability to open fully resulting in an obstruction of blood flow from the left atrium to the left ventricle Most commonly a result of RHD History: asymptomatic for initial approximately 10 years of condition followed by dyspnea on exertion, orthopnea, and paroxysmal nocturnal dyspnea Physical examination: opening snap after second heart sound, diastolic rumble, loud S 1 heart sound, possible peripheral edema, and hepatomegaly Tests: CXR shows RVH, left atrial enlargement, and mitral valve calcifications (Figure 1-23); echocardiogram shows a thickened and calcified mitral valve with decreased mobility and allows measurement of the decreased mitral opening Treatment: Prophylactic antibiotics are given for any intervention with a risk of bleeding to reduce the risk of accidental valve infection Diuretics are given to reduce preload P.36
Figure 1-23 Chest x-ray in a patient with mitral stenosis due to rheumatic heart disease and secondary tricuspid regurgitation. Note the increased left atrial size (arrows) due to mitral regurgitation and the significantly increased right atrial size (arrow heads) due to tricuspid regurgitation.
Antiarrhythmics and anticoagulants are required with the development of Afib Surgical repair, replacement, or balloon valvotomy is required prior to progression Outcomes: progressive CHF and pulmonary disease; poor prognosis without surgical treatment in advanced disease Why eliminated from differential: the murmur heard in the case is systolic (diastolic murmur expected in MS), and the echocardiogram is diagnostic of mitral valve prolapse
Successful treatment and surgical correction of valvular diseases depend on diagnosing the disorder before severe symptoms develop. Aortic regurgitation (AR) Aortic valve incompetency causing backflow of blood into the left ventricle Common causes include congenital defects, endocarditis, RHD, tertiary syphilis, aortic root dilation, and aortic dissection
History: initially asymptomatic with eventual development of dyspnea on exertion, orthopnea, and chest pain Physical examination: bounding pulses, widened pulse pressure, diastolic decrescendo murmur at right second intercostals space, late diastolic rumble (i.e., Austin-Flint murmur), capillary pulsations in the nail beds that become more visible with application of pressure (i.e., Quincke sign) Tests: CXR shows aortic dilation and LVH; echocardiogram demonstrates abnormality of aortic valves or root and a backflow of blood Treatment: Prophylactic antibiotics are given for any intervention with a risk of bleeding to reduce the risk of accidental valve infection ACE-I, calcium channel blockers, or nitroglycerin are used to decrease afterload Valve replacement is eventually required P.37 Outcomes: Increased risk of CHF, arrhythmias, or sudden cardiac death Yearly risk of mortality in symptomatic patients is up to 5% This risk increases to 10% with angina and to 20% with comorbid CHF Why eliminated from differential: the murmur heard in the case is systolic (diastolic murmur expected in MS), and the echocardiogram is diagnostic of mitral valve prolapse Aortic stenosis (AS) Narrowing of the aortic valve causing obstruction of the flow of blood from the left ventricle to the aorta Common causes include congenital defects, RHD, and valvular calcification (elderly patients) History: syncope, chest pain, dyspnea on exertion Physical examination: weak prolonged pulse, systolic crescendo-decrescendo murmur radiating from the right upper sternal border to the carotid arteries (murmur lessens during Valsalva), weak S 2 heart sound Tests: CXR may show calcified valves; echocardiogram shows the valvular abnormality and detects obstruction of blood flow into the aorta (Figure 1-24) Treatment: Prophylactic antibiotics are given for any intervention with risk of bleeding to reduce risk of accidental valve infection β-blockers help to reduce cardiac workload against the obstruction Valve replacement is indicated for severe symptomatic AS or for patients with moderate to severe AS requiring other cardiac or aortic surgery Outcomes: The development of symptoms is correlated with an increased risk for CHF, arrhythmias, and sudden cardiac death The development of CHF is a poor prognostic sign Without surgical correction, mortality in symptomatic patients commonly occurs within 3 years
Why eliminated from differential: the patient's normal exercise tolerance and the echocardiogram findings rule out this diagnosis
Figure 1-24 Parasternal long-axis view echocardiogram in a patient with significant aortic stenosis. Note the thickened, calcified aortic valve with impaired opening. AV, aortic valve; LA, left atrium; LV, left ventricle; RV, right ventricle; VS, ventricular septum.
In aortic stenosis, Valsalva will decrease the murmur; in hypertrophic obstructive cardiomyopathy (HOCM), Valsalva will increase the murmur. P.38 Pulmonary stenosis Obstruction of blood flow from the right ventricle to the pulmonary arteries due to a diseased valve Most commonly due to a congenital defect History: initially asymptomatic with eventual development of fatigue and dyspnea on exertion Physical examination: peripheral edema, cyanosis, systolic crescendo-decrescendo murmur, midsystolic click Tests: CXR shows dilation of the pulmonary arteries and RVH; echocardiogram detects an abnormal pulmonary valve with impaired blood flow Treatment: Prophylactic antibiotics are given for any intervention with a risk of bleeding to reduce the risk of accidental valve infection Valvuloplasty is the preferred treatment
Outcomes: good prognosis unless stenosis is severe (right-sided heart failure develops) Why eliminated from differential: the CXR in the case does not display pulmonary dilation; the echocardiogram is confirmatory of a mitral pathology
Case 1-9: “I get short of breath whenever I exercise” A 24-year-old man presents to his PCP with a complaint of exertional dyspnea and mild chest pain that has worsened over the past few months. He says that he has become mildly short of breath during exercise for several years, but that this symptom has seemed to worsen over the past few months. He has also started to feel slight chest discomfort during exercise over this time. He states that his childhood pediatrician reasoned that his dyspnea was due to asthma and provided him with an albuterol inhaler. He rarely uses his inhaler because he feels that it does not help his symptoms. He denies any past medical history but says that his brother died a few years ago at the age of 16 when he had a heart attack while playing basketball. Besides the albuterol inhaler, which he does not use, he takes no other medications. He drinks alcohol socially. A review of systems finds a history of dizziness and fainting episodes that have occurred a couple of times each year for several years. On examination, he is found to be in no distress. He has no jugular venous distention. Auscultation reveals clear lung fields, a harsh holosystolic murmur, and both S 3 and S4 extra heart sounds. He is found to have two apical impulses per heart cycle on chest palpation. Palpation of his carotid pulse finds a similar double pulse. His abdominal and neurologic examinations are normal. The following vital signs are measured: T: 98.6°F, HR: 80 bpm, BP: 115/80 mm Hg, RR: 16 breaths/min
Differential Diagnosis Aortic stenosis, mitral regurgitation, hypertrophic obstructive cardiomyopathy, dilated cardiomyopathy, restrictive cardiomyopathy
Laboratory Data and Other Study Results Pulse oximetry: 100% on room air ECG: normal sinus rhythm; increased amplitude in leads V 1 to V4 CXR: clear lung fields; cardiomegaly with a “boot-shaped” appearance Echocardiogram: mild mitral regurgitation with abnormal systolic leaflet motion; increased left ventricular outflow velocity; septal hypertrophy; decreased diastolic ventricular wall motion P.39
Diagnosis Hypertrophic obstructive cardiomyopathy
Treatment Administered The patient was advised to refrain from strenuous activities Metoprolol was prescribed at a low dose Partial septal ablation was performed via cardiac catheterization
Follow-up The patient experienced improvement in his symptoms following the catheter ablation procedure
A follow-up echocardiogram demonstrated improved parameters of ventricular outflow The patient was allowed to gradually return to physical activities and was followed to observe for signs of recurrent obstruction
Steps to the Diagnosis Hypertrophic obstructive cardiomyopathy (HOCM) Septal and/or ventricular thickening causing decreased diastolic filling, left ventricular outflow obstruction, and diastolic dysfunction (Figure 1-25A) Congenital anomaly due to autosomal dominant genetic defect History: dyspnea, orthopnea, palpitations, chest pain, dizziness, or syncope that become worse with exertion; sudden cardiac death during exertion may be first presentation of disease Physical examination: holosystolic murmur, S3 and S4 extra heart sounds, double or triple apical impulse, double carotid pulse Tests: echocardiogram shows thickening of septum and ventricular wall, increased ventricular outflow velocity, mitral regurgitation, diastolic dysfunction and decreased wall compliance, and abnormal systolic movement of the anterior leaflet of the mitral valve Treatment: Lifestyle adjustment to avoid strenuous activities β-blockers or calcium channel blockers decrease cardiac workload Left ventricular myomectomy or catheter septal ablation is used to decrease the size of outflow obstruction Mitral valve replacement may be required in cases of severe regurgitation Internal defibrillator or pacemaker implantation may be required to prevent fatal arrhythmias Outcomes: complications include CHF, arrhythmias, and sudden death; 4% mortality per year (due to sudden death) Clues to the diagnosis: History: death of brother at a young age, exertional dyspnea and chest pain, syncopal episodes Physical: holosystolic murmur, extra heart sounds, double apical and carotid pulses Tests: CXR and echocardiogram appearances
Squatting may relieve symptoms in HOCM.
The murmur in HOCM will decrease in severity when preload is increased (e.g., squatting) or afterload is increased (e.g., tensing of muscles). The murmur increases in severity if the preload decreases (e.g., Valsalva maneuver) or the afterload decreases (e.g., vasodilators).
HOCM is the most common cause of sudden death in the young athlete.
Dilated cardiomyopathy accounts for 90% of all cardiomyopathies. Dilated cardiomyopathy Dilation of the ventricles causing valve incompetence and allowing significant regurgitation of blood flow, decreased cardiac output, and pulmonary congestion (Figure 1-25B) May be idiopathic or caused by chronic alcohol use, beriberi, coxsackie virus B, cocaine use, doxorubicin administration, human immunodeficiency virus (HIV), or pregnancy History: dyspnea, orthopnea, palpitations, chest pain P.40
Figure 1-25 Diagrams of most common variations of cardiomyopathy. (A) Hypertrophic—note the ventricular wall and septal thickening leading to outlet obstruction. (B) Dilated—note the decreased wall thickness and increased ventricular size. (C) Restrictive—note the increased ventricular wall thickness and decreased chamber size.
P.41 Physical examination: jugular venous distention, hepatomegaly, ascites, decreased breath sounds, rales, displaced point of maximal impulse, holosystolic and diastolic murmurs, S3 and S4 extra heart sounds, peripheral edema Tests: Increased creatinine and liver enzymes suggest chronic overload CXR shows cardiomegaly (“water bottle” heart), pulmonary congestion, pulmonary effusions, and Kerley B lines Echocardiogram shows dilated ventricular walls, multiple valve regurgitations, and wall motion abnormalities Treatment: Salt and water restrictions in diet Diuretics, ACE-I, β-blockers, and vasodilators help to reduce preload and afterload and decrease regurgitation Anticoagulants may be required to decrease thrombus risk Implantation of a ventricular assist device or heart transplantation is frequently required for long-term treatment Outcomes: prognosis is related to underlying cause (i.e., reversible causes carry a better prognosis; patients with severe heart failure due to regurgitation carry a yearly 50% risk of mortality Why eliminated from differential: the absence of findings suggestive of a right-sided cardiac pathology and the echocardiogram findings argue against this diagnosis Restrictive cardiomyopathy Uncommon disease of the myocardium in which decreased wall compliance causes impaired diastolic filling (Figure 1-25C) May be due to sarcoidosis or amyloidosis History: progressive fatigue, weakness, dyspnea, chest pain, palpitations, or syncope Physical examination: jugular venous distension, ascites, peripheral edema, hepatomegaly, holosystolic murmur, loud S3 Tests: Eosinophilia may be seen in the CBC CXR may show cardiomegaly and pleural effusions Echocardiogram demonstrates diastolic dysfunction, dilation of both atria, and a decrease in wall compliance with inspiration Cardiac catheterization demonstrates increased chamber pressures with a further increase in pressure
during diastole Treatment: pacemaker implantation may be required to prevent arrhythmias; heart transplantation is the only definitive cure Outcomes: with few treatment options, long-term prognosis is often poor; complications follow those for the underlying cause and include arrhythmias, mural thrombosis, and sudden death Why eliminated from differential: differences between the expected echocardiograms for these diagnoses help differentiate them; the predominance of right-sided heart symptoms (e.g., hepatomegaly, jugular venous distention) would be expected for restrictive cardiomyopathy Aortic stenosis More thorough discussion in prior case Why eliminated from differential: the echocardiogram is able to rule out AS by showing a process involving the myocardium and mitral valve and not limited to the aortic valve Mitral regurgitation More thorough discussion in prior case Why eliminated from differential: although it is apparent that MR is occurring in this patient, the history, physical, and echocardiogram findings are indicative of a more global cardiac process P.42
Case 1-10: “No one can figure out why I'm short of breath” A 55-year-old man is referred to a cardiologist to discuss his chronic dyspnea. He describes gradually worsening dyspnea and lower extremity edema over the past 5 years. He says the dyspnea is fairly consistent but becomes worse with exertion. He also notes fairly constant mild abdominal pain, chest pain, and fatigue. He is most comfortable when sitting or standing, so he sleeps in a chair at night. He denies any episodes of syncope, productive cough, palpitations, nausea or vomiting, numbness, or weakness. He denies any significant illnesses for as long as he can remember or any past medical history. He is prescribed inhaled albuterol, furosemide, and lisinopril by his PCP for his current symptoms. He drinks alcohol occasionally and smokes cigars on rare occasions. He says that he has seen his PCP about his symptoms and was told he most likely had CHF. Because he has not improved significantly on his medical regimen, he was told to see a pulmonologist (who does not believe that this is a primary respiratory condition) and a cardiologist in hopes of determining the source of his dyspnea. He reports having had lab work, which showed slight elevations of ESR and BNP, an ECG, which was normal, and a CXR and chest computed tomography (CT) that found diffuse mediastinal calcifications and no intrapulmonary processes. On examination, he is in minimal distress. He has moderate JVD that increases with inspirations. His lungs sound minimally congested but no rales or focal areas of consolidation are detectable. Auscultation finds faint heart sounds, tachycardia, and an indistinct extra heart beat. No murmurs are heard. An apical impulse is unable to be detected. He has mild hepatomegaly but no abdominal tenderness. He has mild edema in both lower extremities. His neurologic examination is normal. The following vital signs are measured: T: 98.3°F, HR: 105 bpm, BP: 110/75 mm Hg, RR: 18 breaths/min
Differential Diagnosis Restrictive cardiomyopathy, dilated cardiomyopathy, chronic constrictive pericarditis, cardiac tamponade, acute pericarditis, sarcoidosis, tricuspid regurgitation, CHF, COPD
Laboratory Data and Other Study Results The following results are forwarded from the patient's PCP:
CBC: WBC: 7.5, Hgb: 14.3, Plt: 360 Chem10: Na: 139 mEq/L, K: 4.1 mEq/L, Cl: 107 mEq/L, CO2: 26 mEq/L, BUN: 20 mg/dL, Cr: 1.1 mg/dL, Glu: 84 mg/dL, Mg: 2.1 mg/dL, Ca: 10.1 mg/dL, Phos: 3.0 mg/dL ESR: 20 mm/h BNP: 110 pg/mL PPD: negative ECG: sinus tachycardia, no abnormal waveforms CXR: clear lung fields, diffuse calcifications within mediastinum These additional studies are ordered: LFTs: AlkPhos: 135 U/L, ALT: 70 U/L, AST: 53 U/L, Tbili: 1.6 mg/dL, Dbili: 0.6 mg/dL Antinuclear antibodies (ANA): negative Rheumatoid factor (RF): negative Echocardiogram: rapid early diastolic filling that slows significantly with inspiration, no valvular disease, normal myocardial constriction and relaxation Chest CT: pericardial calcifications; 6-mm thick pericardium (normal ≤2 mm); no focal cardiac or pulmonary lesions or lymphadenopathy
Diagnosis Chronic constrictive pericarditis (likely idiopathic)
Treatment Administered Furosemide was continued, but albuterol and the ACE-I were discontinued The patient was started on prednisone to attempt to decrease pericardial inflammation
Follow-up The patient did not experience significant improvement on prednisone, which was subsequently discontinued Based on the risks of surgical pericardectomy, the patient elected to delay surgery until further worsening of his symptoms His symptoms became significantly worse in 3 years, and the patient elected to undergo total pericardectomy, which was successful The patient improved to the extent of only having dyspnea during significant exertion and having no symptoms at rest
Steps to the Diagnosis Chronic constrictive pericarditis Diffuse thickening and calcification of the pericardium leading to impaired diastolic filling and decreased cardiac output May be idiopathic or due to a viral or bacterial infection, radiation exposure, or prior cardiac surgery
History: dyspnea on exertion, orthopnea, chest pain, palpitations, abdominal pain and distention, fatigue Physical examination: JVD that worsens during inspiration (Kussmaul sign), peripheral edema, hepatomegaly, tachycardia, normal or low BP, pericardial knock (sounds similar to S3), no murmur, apical impulse may be undetectable Tests: ESR and BNP will be mildly increased; increased WBCs with bacterial infection LFTs may be mildly increased with hepatic congestion ECG is frequently normal except for tachycardia Echocardiogram will show rapid early diastolic filling that plateaus, decreased filling with inspiration, and normal myocardial relaxation CXR will show pericardial calcifications and possible superior vena cava dilation (Figure 1-26A) High-resolution chest CT or magnetic resonance imaging (MRI) will show pericardial calcifications and may be used to measure pericardial thickness (normal ≤2 mm) (Figure 1-26B) Cardiac catheterization may be used for difficult-to-diagnose cases and will show equal end-diastolic pressure in all chambers and similar flow patterns as seen with echocardiography Pericardial biopsy may be useful to detect chronic inflammation and fibrosis Treatment: Loop diuretics help reduce vascular congestion Prednisone may decrease inflammation in subacute cases and improve symptoms Total pericardectomy is the only definitive treatment but carries up to 15% mortality Outcomes: prognosis worsens as signs of right-sided heart failure (due to vascular congestion) worsen and in general is poor with medical treatments only; prognosis is best for idiopathic and postsurgical conditions Clues to the diagnosis: History: dyspnea that worsens with exertion, chest and abdominal pain Physical: faint heart sounds, no apical impulse, edema Tests: mildly elevated LFTs, diastolic filling progression on echocardiogram, pericardial calcifications on chest CT
Perform an echocardiogram and either MRI or high resolution CT to help differentiate constrictive pericarditis from restrictive cardiomyopathy. Acute pericarditis Acute inflammation of the pericardial sac accompanied by a pericardial effusion
Figure 1-26 Radiographic examples of typical findings seen in constrictive pericarditis. (A) Lateral chest x-ray displaying calcification of the pericardial sac (arrows). (B) Chest computed tomography of a patient with chronic pericarditis demonstrating substantial pericardial thickening (arrow) around the ventricles. LV, left ventricle; RV, right ventricle.
Caused by viral infection, tuberculosis, systemic lupus erythematosus (SLE), uremia, neoplasm, drug toxicity (e.g., isoniazid, hydralazine), post-MI inflammation (i.e., Dressler syndrome), radiation exposure, or recent heart surgery History: anterior chest pain with inspiration (i.e., pleuritic chest pain) that improves with leaning forward, dyspnea, cough Physical examination: fever, pericardial friction rub (best heard when patient leans forward), pulsus paradoxus (fall in SBP >10 mm Hg during inspiration that occurs because the physiologic increased right ventricle filling during inspiration and the pathologic left ventricle compression by the pericardial effusion cause impaired left ventricle filling, decreased stroke volume, and decreased inspiratory SBP) Tests: ECG shows PR depression and global ST elevation CXR helps to rule out a pulmonary process and may detect a pericardial effusion Echocardiogram may be used to measure a pericardial effusion and examine heart function Treatment: Treat the underlying cause Pericardiocentesis is performed to drain large effusions Nonsteroidal anti-inflammatory drugs (NSAIDs) decrease pain and inflammation and used prior to the consideration of corticosteroids Colchicine may be useful for preventing recurrences due to viral or idiopathic causes Outcomes: may evolve into chronic pericarditis or cardiac tamponade if untreated
Why eliminated from differential: the history in this case suggests a more chronic process than acute pericarditis; the absence of ECG changes or a pericardial effusion also argues against this diagnosis
ST elevation is seen in acute pericarditis as well as in MI, but acute pericarditis also demonstrates PR depression and ST elevation in most leads (MI frequently does not show PR depression, and ST elevation is focal).
If ST elevation is seen on ECG, a full workup is required to rule out MI.
Pericardial effusions are usually transudates (i.e., low in proteins, specific gravity <1.012); if exudates (i.e., rich in proteins, specific gravity >1.020) are collected during pericardiocentesis, perform workup for neoplasm or fibrotic disease. Cardiac tamponade Large pericardial effusion that causes compression of the heart and a greatly decreased cardiac output Caused by progressive acute pericarditis, chest trauma, left ventricle rupture following MI, or a dissecting aortic aneurysm History: sudden dyspnea, chest pain, syncope, weakness Physical examination: tachycardia, tachypnea, increased JVD, pulsus paradoxus, hypotension, faint heart sounds Tests: ECG will be low voltage with sinus tachycardia CXR will show an enlarged heart shadow Echocardiogram will detect a large effusion Treatment: Immediate pericardiocentesis IV hydration and supplemental O2 A pericardial window may need to be performed for recurrent tamponade Outcomes: rapidly fatal without emergent treatment Why eliminated from differential: the chronic nature of this cases makes this an unlikely diagnosis; this is confirmed by the absence of a pericardial collection on the echocardiogram
If you find Beck's triad—hypotension, distant heart sounds, and distended neck veins—think of cardiac tamponade and perform an urgent pericardiocentesis! Restrictive cardiomyopathy More thorough discussion in prior case Why eliminated from differential: this condition will present in a very similar fashion to constrictive pericarditis; pericardial thickness (as measured on MRI or CT) will be increased (normal in restrictive
cardiomyopathy), and myocardial relaxation will be normal (abnormal in restrictive cardiomyopathy) in constrictive pericarditis Dilated cardiomyopathy More thorough discussion in prior case Why eliminated from differential: the echocardiogram did not detect a dilated heart with multivalve regurgitation as would be expected for this diagnosis Sarcoidosis More thorough discussion in Chapter 2 Why eliminated from differential: the expected CXR and chest CT findings (e.g., hilar adenopathy, pulmonary processes, ground-glass appearance) and absence of erythema nodosum make this diagnosis highly unlikely Tricuspid regurgitation More thorough discussion in prior case Why eliminated from differential: the echocardiogram findings rule out this diagnosis Congestive heart failure More thorough discussion in prior case Why eliminated from differential: the minimally elevated BNP and the absence of significant lung infiltrates on the CXR make this diagnosis unlikely Chronic obstructive pulmonary disease More thorough discussion in Chapter 2 Why eliminated from differential: the appearance of the CXR is not typical for COPD; in the absence of the finding of pericardial calcifications, pulmonary function tests should be performed to determine if this diagnosis is the cause of the patient's symptoms
Case 1-11: “I think I really got myself sick this time” A 38-year-old man is an inpatient on the medicine service after being admitted from the emergency department the previous evening for a fever workup. He is a recalcitrant IV drug user who is well known to both the emergency medicine and internal medicine teams for his frequent presentations for infections and complications due to his drug use. When he presented last night, he described a 2-day history of shaking chills, intense sweating episodes, myalgias, painful fingers and toes, and general malaise. On examination in the emergency department, he was found to have clear lung fields, a soft diastolic murmur, painful nodules on his finger and toe pads, and a normal neurologic examination. The following vital signs were measured last night: T: 105.1°F, HR: 110 bpm, BP: 110/73 mm Hg, RR: 20 breaths/min Labs collected at that time were suggestive of an infectious process. Following admission, multiple cultures of his blood and urine were collected and he was started on empiric vancomycin and gentamicin. Today he feels somewhat better, although he is still highly fatigued. He says that the chills and sweating episodes have stopped, and the digit pain is slightly better. On examination today, he is found to have clear lung fields without any abnormal sounds. Auscultation of his chest detects a subtle diastolic murmur but no other abnormal heart sounds. His abdomen is nontender with no organomegaly. Examination of his extremities finds multiple painful nodules on some of his fingers and toes, mild clubbing of his fingers, and multiple hemorrhages under his fingernails. His neurologic examination is normal. The latest set of vital signs is:
T: 99.5°F, HR: 86 bpm, BP: 115/80 mm Hg, RR: 17 breaths/min
Differential Diagnosis Infective endocarditis, rheumatic heart disease, myocarditis, Lyme disease, SLE
Laboratory Data and Other Study Results Results of labs performed in emergency department: CBC: WBC: 20.4, Hgb: 13.1, Plt: 278 Chem7: Na: 140 mEq/L, K: 4.1 mEq/L, Cl: 107 mEq/L, CO2: 28 mEq/L, BUN: 13 mg/dL, Cr: 0.5 mg/dL, Glu: 80 mg/dL Blood gram stain: many polymorphonuclear cells (PMNs), many gram-positive cocci in clusters UA: straw-colored, pH: 5.0, specific gravity: 1.010, no glucose/ketones/nitrites/leukocyte esterase/hematuria /proteinuria C-reactive protein (CRP): 15.2 mg/dL ESR: 125 mm/h ECG: sinus tachycardia; no abnormal wave morphology CXR: clear lung fields; slight cephalization of vessels; normal cardiac silhouette The following are results reported this morning on new tests ordered today: Blood culture: Staphylococcus aureus, sensitivities pending Echocardiogram: EF 65%; normal myocardial motion; 5 mm growth on leaflet of tricuspid valve
Diagnosis Infective endocarditis
Treatment Administered The patient was maintained on an empiric antibiotic regimen pending the culture sensitivities The patient was prescribed a 6-week course of IV nafcillin when blood cultures grew methicillin-sensitive S. aureus (MSSA)
Follow-up The patient had symptomatic improvement and no febrile recurrences with the continued antibiotic therapy The patient was transferred to a skilled nursing facility (SNF) to complete his antibiotic course because he was judged to not be a candidate for a peripheral indwelling central catheter (PICC) placement and outpatient treatment (history of IV drug use) Repeat blood cultures performed 3 days after the initiation of antibiotics and surveillance cultures performed at 2 week intervals remained negative Following discharge from the SNF, the patient did not return for scheduled appointments and was lost to follow-up
Steps to the Diagnosis
Infective endocarditis Bacterial or fungal infection of the endocardium (i.e., inner lining of heart) with or without valve involvement Rates are increased in patients with congenital heart defects, prosthetic heart valves, or a history of IV drug use Noninfectious, inflammatory forms may be seen in patients with SLE (Libman-Sacks endocarditis) or cancer (marantic endocarditis) Presentation variants: Acute
Endocarditis due to IV drug use most commonly involves the chambers and valves on the right side of the heart.
Prosthetic valves are particularly susceptible to Staphylococcus epidermidis infection.
Negative-culture endocarditis can result from the HACEK bacteria: Haemophilus, Actinobacillus, Cardiobacterium, Eikenella, and Kingella. Caused by Staphylococcus aureus, Streptococcus pneumoniae, Staphylococcus pyogenes, and Neisseria gonorrhoeae Onset is sudden with frequently severe symptoms Bacterial vegetations tend to occur on previously healthy valves Subacute Caused by viridans streptococcus, Enterococcus, fungi, and Staphylococcus epidermidis Onset is more insidious Bacterial vegetations tend to occur on valves that have some preexisting abnormality (e.g., prior infection, congenital abnormality) History: chills, night sweats, fatigue, arthralgias, myalgias Physical examination: fever (may be very high in the acute form), new murmur (due to valvular pathology), small tender nodules on finger and toe pads (i.e., Osler nodes), peripheral petechiae (i.e., Janeway lesions), subungual petechiae (i.e., splinter hemorrhages), retinal hemorrhages (i.e., Roth spots) Tests: Increased WBC, ESR, and CRP; serial blood cultures will grow the same pathogen (except in HACEK [Haemophilus species, Actinobacillus actinomycetemcomitans, Cardiobacterium hominis, Eikenella corrodens, and Kingella species (group of fastidious, gram-negative, aerobic bacilli)] infections) prior to appropriate treatment CXR may reveal congestion in cases of septic emboli and right-sided heart failure Echocardiogram (preferably transesophageal) may show the growth of vegetations on valves Diagnosis is determined by the Duke criteria (Table 1-6)
Treatment: Long-term (4 to 6 weeks) IV antibiotics (initially broad spectrum, then bug-specific); a β-lactam and aminoglycoside regimen is the most commonly used pending blood culture sensitivities and in cases of culture-negative endocarditis Antibiotic prophylaxis before any invasive procedure is required if any valves are damaged
Table 1-6 Duke Criteria for Diagnosis of Infective Endocarditis
Definitive diagnosis of infective endocarditis requires→
Direct histologic evidence of infective endocarditis
Positive gram stain or culture from surgical debridement of cardiac abscess
OR
OR
or autopsy specimen
2 major criteria
2 major and 3 minor criteria
5 minor criteria
Major Criteria
OR
OR
OR
Minor Criteria
Serial blood cultures positive for
Predisposing heart condition or intravenous drug use
organisms associated with infective
Fever >38°C
endocarditis
Vascular phenomenon (e.g., arterial emboli, septic pulmonary infarcts,
Presence of vegetations or cardiac
mycotic aneurysm, intracranial hemorrhage, conjunctival hemorrhage, or
abscess seen on echocardiogram
Janeway lesions)
Evidence of new onset valvular
Immunologic phenomenon (e.g., glomerulonephritis, Osler nodes, Roth spots,
regurgitation
positive rheumatoid factor)
Blood culture positive for Coxiella
Positive cultures not meeting requirements for major criteria or serologic
burnetii
evidence of infection without positive culture
Valve replacement may be necessary for severely damaged valves and in patients who develop significant valvular dysfunction, CHF, or an inability to clear the pathogen from their bloodstream Any abscesses will require incision and drainage Outcomes: Best prognosis occurs in patients who become afebrile after initiation of treatment and who maintain negative cultures Recurrence is most common within 2 months of having completed an antibiotic course and occurs more frequently in IV drug users and for cases due to S. aureus, enterococcus, and gram-negative organisms Complications include valvular dysfunction, CHF, myocardial abscesses, metastatic infection, septic embolization, and organ dysfunction Clues to the diagnosis: History: IV drug use, recurrent infections, shaking chills with diaphoresis, digital pain, symptomatic improvement with antibiotics
Physical: high fever, Osler nodes, splinter hemorrhages, digital clubbing Tests: increased WBCs, increased ESR and CRP, MSSA on blood cultures, valvular vegetation seen on echocardiogram
RHD only occurs in 3% of untreated streptococcal infections, and the overall incidence is significantly lower in the United States due to standardized antibiotic treatment for suspected streptococcal infections. Rheumatic heart disease An uncommon sequela of untreated or inadequately treated group A streptococcus infection Streptococcus infection may induce the production of autoantibodies that attack the heart valves and joints (order of decreasing frequency is mitral, then aortic, then right-sided valves) History: migratory arthralgias (i.e., aching joints), polyarthritis (i.e., painful joints due to degeneration that worsen with motion or weight-bearing), prior streptococcal infection, possible chest pain, dyspnea, cough, and orthopnea Physical examination: hot and swollen joints, fever, subcutaneous nodules on extensor surfaces, Sydenham chorea (i.e., purposeless involuntary movements), erythema marginatum (i.e., painless rash), joint crepitus or tenderness, murmur, tachycardia
Figure 1-27 JONES criteria mnemonic for diagnosis of rheumatic heart disease (RHD). CRP, C-reactive protein; ECG, electrocardiogram; ESR, erythrocyte sedimentation rate; PR, PR interval; RHD, rheumatic heart disease.
Tests: Increased ESR, CRP, and WBC; positive antistreptococcal antibodies in 90% of cases ECG demonstrates a prolonged PR interval Diagnosis is made using the Jones criteria (Figure 1-27) Treatment: NSAIDs are used for joint pain; corticosteroids may be started for carditis symptoms
A β-lactam is frequently selected as an antibiotic for the streptococcal infection If valve damage has occurred, prophylactic antibiotics will be required prior to any invasive procedure Outcomes: valvular damage may progress without treatment; carditis can be fatal if antibiotics are not prescribed (rare in industrialized nations) Why eliminated from differential: the sudden severe onset of symptoms and the high fever are more consistent with endocarditis; the positive cultures for S. aureus in the case are indicative of endocarditis
Myocarditis in South and Central America is commonly due to Trypanosoma cruzi (i.e., Chagas disease), and in these cases may be associated with achalasia. Myocarditis An inflammatory reaction in the heart limited to cardiac muscle involvement Most commonly due to infection (e.g.,coxsackie virus, bacteria, rickettsiae, fungi, parasites) May also be caused by drug toxicity (e.g., doxorubicin, chloroquine, penicillins, sulfonamides, cocaine), radiation exposure, chemical toxins, or systemic rheumatic conditions History: possible recent upper respiratory infection, pleuritic chest pain, dyspnea, diaphoresis, chills Physical examination: extra heart sound, diastolic murmur, possible friction rub, fever Tests: WBC, ESR, and CRP may be increased; viral titers or serology or cultures for other pathogens may identify an infectious cause ECG demonstrates variable conduction abnormalities including ST interval and T wave changes CXR may show cardiomegaly Echocardiogram may be used to rule out other pathologies Treatment: treat amenable infections (e.g., nonviral) or stop offending agents; the development of heart failure requires symptomatic treatment Outcomes: complications include heart failure and arrhythmias; poor prognosis is associated with syncope, new bundle branch block, poor ejection fraction, and new cardiomyopathy Why eliminated from differential: the absence of ECG changes, the presence of a valvular vegetation, and the positive cultures seen in this case make this diagnosis unlikely Lyme disease More thorough discussion in Chapter 9 Why eliminated from differential: the presentation of progressive Lyme disease may be similar to that seen in this case, but the absence of neurologic symptoms and the blood cultures positive for S. aureus make endocarditis a more likely diagnosis Systemic lupus erythematosus More thorough discussion in Chapter 9 Why eliminated from differential: given the multiple findings in this case suggesting an infectious etiology, this diagnosis is unlikely; any workup for endocarditis with negative cultures should prompt testing for ANA and
rheumatoid factor
Case 1-12: “My wife says I should cut down on the salt” A 53-year-old man presents to his PCP to undergo a wellness check-up. He says he scheduled this appointment because his wife was concerned about his diet and weight and thought that it was important for him to see a doctor regularly. He has no current complaints other than acknowledging that he is overweight. He denies any medical problems and only takes low-dose ASA daily. When he was last seen 2 years ago, he was found to have borderline hypertension (135/85) and was encouraged to lose weight, minimize salt and fats in his diet, and exercise. He says that he has not done well in following these recommendations. He has a job that requires much driving, and he eats fast foods at least once per day. One of his friends at work was recently diagnosed with diabetes mellitus, and he is concerned about this issue. He drinks about ten beers per week and smokes a half pack of cigarettes per day (for 25 years). His review of systems is only notable for a dry cough in the morning. On examination, he is an obese man in no acute distress. Auscultation of his lungs and heart demonstrates clear lung fields and a regular heart rate with no extra sounds. He has no abdominal tenderness, masses, or abnormal sounds. He has equal palpable pulse in all extremities. His neurologic examination is normal. The following vital signs are measured: T: 98.8°F, HR: 80 bpm, BP: 155/100 mm Hg (confirmed in both arms and left leg), RR: 16 breaths/min
Differential Diagnosis Primary hypertension, renal artery stenosis, pheochromocytoma, hyperaldosteronism, coarctation of the aorta, impending hypertensive emergency
Laboratory Data and Other Study Results Height/weight: 6﹐ 4", 305 lbs, body mass index (BMI) 37 CBC: WBC: 7.1, Hgb: 15.3, Plt 400 Chem7: Na: 143 mEq/L, K: 4.2 mEq/L, Cl: 106 mEq/L, CO2: 26 mEq/L, BUN: 15 mg/dL, Cr: 0.8 mg/dL, Glu: 95 mg/dL Coags: PT: 11 sec, INR: 1.0, PTT: 38 sec Lipid panel: Total cholesterol: 220 mg/dL, LDL: 170 mg/dL, HDL: 48 mg/dL, Trig: 135 mg/dL UA: straw-colored, pH: 5.0, specific gravity: 1.010, no glucose/ketones/nitrites/leukocyte esterase/hematuria /proteinuria
Diagnosis Primary hypertension
Treatment Administered The patient was encouraged to reduce salt and fats in diet, lose weight, exercise, moderate his alcohol intake, and stop smoking The patient was scheduled for two separate follow-up visits in the next 2 months to repeat BP measurements (measured at 150/95 and 155/95 on repeat measurements) The patient was started on atorvastatin to reduce cholesterol levels and hydrochlorothiazide (HCTZ) to lower BP
Follow-up Patient had an improvement in his total cholesterol to 180 mg/dL and LDL to 120 mg/dL
The patient's BP improved to 130/80 on the next follow-up visit After maintaining a stable BP for 4 years, patient developed recurrent hypertension that was controlled by adding losartan to his drug regimen
Steps to the Diagnosis Primary (essential) hypertension Idiopathic HTN that accounts for >95% of all cases of high BP Defined as a SBP >140 mm Hg and/or a DBP >90 mm Hg consistently measured on three separate occasions Risk factors: family history of HTN, high-salt diet (especially if salt-sensitive), tobacco use, obesity, increased age, African American heritage History: asymptomatic for most of the course until well progressed, and then headache may be the only symptom other than those associated with its complications Physical examination: BP >140/90 (consistently); chronic cases are notable for a loud S 2 or S4 on chest auscultation and arteriovenous nicking (i.e., apparent retinal vein narrowing secondary to arterial wall thickening), cotton-wool spots, or retinal hemorrhages (i.e., flame hemorrhages) on fundoscopic examination Tests: no tests are required for the diagnosis of primary HTN, but tests may be appropriate to rule out secondary causes of HTN Treatment: Encourage weight loss, exercise, healthy eating, salt restriction, alcohol reduction, and smoking cessation A thiazide diuretic is typically the first drug prescribed unless comorbid conditions indicate otherwise (e.g., calcium channel blocker, β-blocker; Tables 1-7 and 1-8) An ACE-I, ARB, or calcium channel blocker is frequently the second prescribed drug if additional control is required Both thiazide and either an ACE-I or ARB should be started for pressures with a SBP >160 mm Hg and a DBP >100 mm Hg Outcomes: Untreated or poorly treated disease increases the risk of CAD, stroke, aortic aneurysm, aortic dissection, CHF, kidney disease, and ophthalmologic disease Long-term prognosis is directly related to the ability to control BP, ability to reduce unhealthy behaviors, and the avoidance of complications The behaviors that are risk factors for HTN are also associated with diabetes mellitus
The patient should be sitting quietly for 5 minutes before BP is measured to minimize false high readings.
Do not start a new antihypertensive medication unless three consecutive readings demonstrate a high BP.
Secondary causes of HTN are far less common and are due to an identifiable cause.
Renal diseases are the most common pathology.
An ACE-I is contraindicated in cases of bilateral renal artery stenosis since it may accelerate renal failure by impeding sufficient renal perfusion and lowering the glomerular filtration rate.
Keep secondary causes of HTN in the back of your mind during a hypertensive workup. If the patient exhibits certain pathognomonic signs or does not respond as expected to regular treatments, begin to consider a secondary cause. Renal artery stenosis See Table 1-9 for a more thorough discussion Why eliminated from differential: this diagnosis is less common in the patient's age group; in addition, the absence of a renal artery bruit makes this an unlikely diagnosis Pheochromocytoma See Table 1-9 for a more thorough discussion Why eliminated from differential: the patient does not exhibit the classic signs of this diagnosis (e.g., sudden episodic headaches and diaphoresis) and responds well to basic antihypertensive therapies Hyperaldosteronism See Table 1-9 for a more thorough discussion Why eliminated from differential: the patient has normal sodium and potassium levels and does not exhibit the characteristic physical findings for this diagnosis (e.g., polyuria, headaches, neurologic abnormalities) Coarctation of the aorta See Table 1-9 for a more thorough discussion Why eliminated from differential: consistent BP are demonstrated in multiple limbs, the ECG finding are normal, and the patient responds well to basic antihypertensive therapies Hypertensive emergency Excessive BP in the presence of an acute onset of end organ dysfunction (e.g., renal failure, encephalopathy, papilledema)
Table 1-7 Common Antihypertensive Agents
Class of Medic ation
Diuretics
Examples
Mec hanism of Ac tion
Presc ription Strategy
Side Effec ts
Thiazides (e.g., HCTZ);
Reduce circulatory volume
Early; particularly effective
Increased serum glucose,
K-sparing (e.g.,
to decrease CO and mean
in African Americans and
cholesterol, or
spironolactone); loop
arterial pressure
salt-sensitive patients
triglycerides; hypokalemia
diuretics too potent for regular anti-HTN use
α-blockers
Prazosin, doxazosin,
Block α-adrenergic
Adjunct to other
Postural hypotension,
terazosin
receptors (primary
medications; less commonly
headache, rebound HTN if
controllers of vascular
used
stopped
tone) to decrease total peripheral resistance
β-Blockers
Nonselective (e.g.,
Decrease HR,
Early; many important
Sexual dysfunction in
propranolol, timolol);
contractility, CO, and
cardiac uses; more effective
males,
β1-selective (e.g.,
decrease renin secretion
in Caucasian patients
bronchoconstriction if
metoprolol, atenolol,
to decrease total
non-β1 selective, HDL
esmolol)
peripheral resistance
reduction, increased triglycerides
Calcium
Nondihydropyridines
Reduce influx of calcium
Second line;
Hypotension, headache,
channel
(e.g., diltiazem,
during cardiac and
nondihydropyridines mainly
constipation, increased
blockers
verapamil);
vascular smooth muscle
effect the myocardium;
GI reflux
dihydropyridines (e.g.,
contraction to cause
dihydropyridines mainly
nifedipine, amlodipine)
vasodilation
effect vascular smooth muscle and are utilized more often for HTN
Vasodilators
ACE-I
Hydralazine, minoxidil
Direct relaxation of
Adjunct to other
Reflex tachycardia,
vascular smooth muscle
medications; less commonly
possible adverse
used
cardiovascular incidents
Lisinopril, captopril,
Block conversion of
Early or second line;
Dry cough, azotemia,
enalapril
angiotensin I to
important cardiac and renal
hyperkalemia,
angiotensin II and
uses; more effective in
teratogenicity
increase circulating
young Caucasian patients
bradykinin to decrease angiotensin II vasopressor activity and aldosterone secretion causing decrease in total peripheral resistance
ARBs
Irbesartan, losartan
Block binding of
Second line
angiotensin II to
Few side effects; high cost; teratogenicity
receptors to inhibit vasopressor activity and decrease aldosterone secretion
ACE-I, angiotensin converting enzyme inhibitors; ARBs, angiotensin receptor blockers; CO, cardiac output; GI, gastrointestinal; HCTZ, hydrochlorothiazide; HDL, high-density lipoprotein; HR, heart rate; HTN, hypertension; K, potassium.
History: headache, chest or abdominal pain, confusion, fatigue, nausea, oliguria, possibly asymptomatic in patients with chronic HTN Physical examination: change of mental status, hematuria, retinal vascular changes, congested lung fields Tests: ECG may indicate cardiac ischemia or a MI; labs will reflect end organ damage Treatment: Reduce the diastolic BP to 100 mm Hg using IV nitroprusside, labetalol, or fenoldopam After the initial reduction use oral β-blocker and ACE-I to maintain mean DBP at 100 to 105 mm Hg
Table 1-8 Recommendations and Contraindications for Antihypertensive Drug Selection
Comorbid Condition
DM
Rec ommended
Rec ommended
Contraindic ated
Antihy pertensive
Reason
A ntihy pertensive
ACE-I
CHF
Post-MI
Benign prostatic
Delays renal damage
Contraindic ated Reason
Thiazide diuretic
Impaired glucose tolerance
β-blocker
May mask signs of hypoglycemia
ACE-I
Improves mortality
Calcium channel
Reduced rate/contractility may
Diuretic
Improves mortality
blocker
exacerbate heart failure
β-blocker
Improves mortality
ACE-I
Improves mortality
Selective α1-blocker
Improves symptoms
β-blocker
May improve
Nonselective
Exacerbates bronchoconstriction
hypertrophy
Migraine headache
symptoms
Osteoporosis
Thiazide diuretic
Maintains normal/high serum calcium
Asthma/COPD
β-blocker
Pregnancy
Thiazide diuretic
Increased blood volume during
ACE-I
pregnancy should be maintained
ARB
Teratogenic Teratogenic
Gout
Diuretic
Increase serum uric acid
Depression
β-blocker
May worsen symptoms
ACE-I, angiotensin converting enzyme inhibitors; ARB, angiotensin receptor blocker; CHF, congestive heart failure; COPD, chronic obstructive pulmonary disease; DM, diabetes mellitus; MI, myocardial infarction.
Table 1-9 Causes of Secondary Hypertension
Condition
Common Patient Group
Signs/Symptoms
Diagnosis
Treatment
Renal diseases
Depends on disease
Depends on disease
ACE-I (delays
(various)
identity
identity
progression)
Renal artery bruit
Arteriography; renal
Angioplasty; stent
vein renin ratio
placement; ACE-I if
Renal artery stenosis
<25 years old (fibromuscular dysplasia) or >50 years
one-sided; surgical
old (atherosclerosis)
repair
OCPs (combination pill)
History
Women >35 years old;
Stop use; change to
obese women;
progestin-only pill or
long-term OCP use
intramuscular medroxyprogesterone
Pheochromocytoma
Young patients;
Episodic HTN,
Increased 24-hr urinary
Surgical removal of
patients with history
diaphoresis,
catecholamines or
tumor with
of endocrine tumors
headaches;
vanillylmandelic acid; CT,
pharmacologic control of
symptoms occur
MRI
HTN up until time of surgery
suddenly
Primary
Patients with an
Polyuria,
Increased sodium,
Surgical resection of
hyperaldosteronism
adrenal adenoma or
headache,
decreased potassium,
tumor, spironolactone,
(i.e., Conn disease)
low renal blood flow
weakness,
metabolic alkalosis,
improve renal blood flow
paresthesias,
increased 24-hr urine
by treating underlying
tetany, resistant
aldosterone
cause
HTN
Excess corticosteroid
Central obesity,
Decreased potassium,
Titration of
use, pituitary
hirsutism, buffalo
increased glucose
corticosteroid dosing,
adenoma, adrenal
hump, striae,
tumor,
glucose intolerance
Coarctation of the
Male > female; Turner
HTN in arm but not
Possible LVH on ECG;
aorta
syndrome, aortic valve
in legs, weak
echocardiogram can
pathology, PDA
femoral pulse
localize defect
Cushing syndrome
resection of tumors
Surgical repair
ACE-I, angiotensin converting enzyme inhibitors; CT, computed tomography scan; ECG, electrocardiogram; HTN, hypertension; LVH, left ventricular hypertrophy; MRI, magnetic resonance imaging; OCP, oral contraceptive pill; PDA, patent ductus arteriosus.
More severe signs of end organ damage should prompt a more rapid reduction in the DBP Diuretics may be required to relieve pulmonary edema Outcomes: prolonged excessive BPs are associated with high rates of CAD, stroke, CHF, and renal failure; excessive decreases in BP can lead to organ hypoperfusion Why eliminated from differential: the patient in this case does not exhibit any signs of end organ damage, so this diagnosis is ruled out
In a hypertensive emergency the initial decrease in DBP should not exceed 25% of the presenting pressure to avoid triggering an ischemic event.
Case 1-13: “I think he's really sick” A 38-year-old man (the same patient from Case 1-11 above) is brought into the emergency department by a friend after several days of a worsening illness. The patient is stuporous and is not able to provide a history, so the patient's friend relates what he knows to the medical staff. He says that the patient had been complaining about feeling sick for the past week. He has been having chills, sweating spells, and significant weakness for the past few days. The friend brought the patient in to the emergency department when he found him poorly responsive this morning. He does not know of any traumatic injuries. The patient had not returned to clinic since his discharge from the hospital 6 months ago. His friend admits that he resumed his IV drug use shortly after his discharge. On examination, the patient is stuporous and only mumbles unintelligible responses to questions. He has cervical and axillary lymphadenopathy. He has faint tachypneic breath sounds in both lower lung fields. Chest auscultation reveals tachycardia and a harsh diastolic murmur. No abdominal tenderness, guarding, or organomegaly is appreciable. All pulses are palpable but weak. He has good capillary refill and warm extremities. Given the patient's mental state a full neurologic examination
cannot be completed, but he does withdrawal from painful stimuli. The following vital signs are measured: T: 104.9°F, HR: 125 bpm, BP: 90/60 mm Hg, RR: 24 breaths/min
Differential Diagnosis Septic shock, cardiogenic shock, hypovolemic shock, anaphylactic shock, acute renal failure, MI, pulmonary embolism, cerebrovascular accident, acute respiratory distress syndrome (ARDS), pneumonia
Laboratory Data and Other Study Results Pulse oximetry: 96% on room air, 99% on 2L nasal cannula CBC: WBC: 28.8, Hgb: 12.3, Plt: 175 Chem7: Na: 135 mEq/L, K: 3.8 mEq/L, Cl: 106 mEq/L, CO2: 23 mEq/L, BUN: 29 mg/dL, Cr: 1.3 mg/dL, Glu: 80 mg/dL Coags: PT: 16 sec, INR: 1.2, PTT: 45 sec LFTs: AlkPhos: 180 U/L, ALT: 73 U/L, AST: 65 U/L, Tbili: 1.5 mg/dL, Dbili: 0.5 mg/dL Lactate: 4.0 mEq/L Cardiac enzymes: CK: 300 U/L, CK-MB: 1.1 ng/mL, troponin-I: 0.1 ng/mL Arterial blood gas (ABG) on room air: pH: 7.41, pO2: 96 mm Hg, pCO2: 36 mm Hg, Bicarb: 23 mEq/L, O2 sat: 96% UA: amber-colored, pH: 4.7, specific gravity: 1.05, no glucose/ketones/nitrites/leukocyte esterase/hematuria /proteinuria Blood cultures: gram stain with gram-positive cocci in clusters and gram-negative rods; culture pending ECG: sinus tachycardia; no abnormal wave morphologies CXR: bibasilar infiltrates; normal cardiac shadow Head CT: no evidence of hemorrhage or ischemia; normal cortical and ventricular volume Abdominal/pelvic CT: no hemorrhage or masses; no abnormal visceral appearance
Diagnosis Septic shock
Treatment Administered The patient was started on vancomycin and gentamicin as empiric antibiotics with the culture results pending The patient was admitted to the medical intensive care unit (MICU)
Follow-up The patient developed worsening hypotension despite antibiotic treatment and was started on phenylephrine and vasopressin His condition worsened with evidence of progressive renal failure and disseminated intravascular coagulation (DIC) He continued to deteriorate, became asystolic, and was unable to be revived
The common presentation of shock is described by the mnemonic CHORD ITEM:
Clammy skin, Hypotension, Oliguria, Rapid breathing, Drowsiness/mental status changes, Irritability, Tachycardia, Elevated/reduced central venous pressure, Multiorgan dysfunction.
Steps to the Diagnosis Shock Circulatory collapse in which the blood delivery to tissues is inadequate for perfusion demands Variants differ by their pathology that causes circulatory collapse (Table 11-10) History: fatigue, malaise, diaphoresis, chills, confusion, and other symptoms related to exact etiology of shock Physical examination: high fever, tachycardia, tachypnea, warm extremities early, cold extremities later, and other signs related to the exact etiology
Table 1-10 Variations of Shock
Ty pe of Shoc k
Mec hanism
Common Causes
Charac teristic s
Treatment
Anaphylactic
Massive release of
Type I
Sudden onset, severe
Maintain airway,
mast cells and
hypersensitivity
upper airway edema,
epinephrine,
basophils resulting
reaction (i.e., allergic
prior history of allergic
diphenhydramine,
in vasodilation and
response)
reactions
cimetidine, IV fluids
Severe heart
MI, arrhythmias,
History of a cardiac
IV fluids, vasoactive
failure with
cardiac tamponade,
event, initially adequate
medications,
significantly
pulmonary embolism
Mortality
2%
increased vascular permeability
Cardiogenic
Hypovolemic
systemic pressure,
ventricular assistive
impaired cardiac
cardiac enzymes and
device, intra-aortic
output
echocardiography are
balloon pump, PTCA,
useful for diagnosis
CABG
Significant
Hemorrhage, trauma,
History of trauma or
IV fluids, transfusions,
intravascular
severe burns
hemorrhagic event,
wound care, surgery
volume loss leading
documented blood loss,
frequently required to
to insufficient
anemia, CT is useful to
control source of
tissue perfusion
locate sites of internal
bleeding, tissue
despite attempts
hemorrhage
transfers for severe
Septic
Variable1
burns
at compensation
Neurogenic
55%–65%
Widespread
Brain or spinal cord
History of neurologic
IV fluid, vasoactive
Not
peripheral
injury
injury, other neurologic
medications
defined
35%–50%
vasodilation due to
findings frequently
dysfunctional
present, response to
neural signaling
therapies is variable
Global peripheral
Infection; most
History of infection,
Appropriate
vasodilation due to
common sources are
compensatory high
antibiotics, vasoactive
an exaggerated
lower respiratory
cardiac output, high
medications
systemic
tract, urinary tract,
fever, high WBCs
inflammatory
GI tract, soft tissues,
response
and reproductive systems
CABG, coronary artery bypass grafting; CT, computed tomography; GI, gastrointestinal; IV, intravenous; MI, myocardial infarction; PTCA, percutaneous transluminal coronary angioplasty; WBCs, white blood cells. 1
Dependent on ability to replace lost volume, not on occurrence of shock itself.
Tests: Initial tests may include CBC, basic metabolic panel (i.e., serum chemistries), coagulation studies, cardiac enzymes, LFTs, UA, toxicology screens, and cultures Increased lactate is an indicator of tissue ischemia ECG, x-rays, CT, MRI, or ultrasound (US) are useful to help determine a cause for the condition Additional studies may be ordered to confirm the diagnosis as suggested by the clinical picture Treatment: Treat underlying conditions Additional therapies depend on the type of shock
IV hydration and vasoactive medications may be required to maintain adequate organ perfusion (Table 1-11) Outcomes: Prognosis is grim with mortality rates frequently >50% Good prognosis is linked to successful maintenance of tissue perfusion and ability to treat underlying disease Complications include ARDS, DIC, and multiorgan dysfunction Clues to the diagnosis: History: prior endocarditis, chills and diaphoresis, found unresponsive, IV drug use Physical: faint breath sounds, tachypnea, fever, hypotension Tests: increased WBCs, increased LFTs, increased Coags, positive blood cultures, increased lactate Reasons to exclude nonseptic shock variants from the differential: Anaphylactic: insidious nature of this case makes this diagnosis unlikely; the intact airway in the patient also helps to rule out this diagnosis Cardiogenic: the patient has a history of a valvular lesion, but the findings of an infectious process (e.g., CBC, fever) make sepsis a better explanation Hypovolemic: the lack of a history of trauma and the CT ruling out an intraabdominal hemorrhage make this diagnosis unlikely
Table 1-11 Vasopressors and Inotropes Commonly Used in the Intensive Care Unit
Drug
Mec hanism
Effec ts
Indic ation
Phenylephrine
Agonist for α-adrenergic receptors (α1 > α2)
Vasoconstriction, reflex bradycardia
Sepsis, shock
Norepinephrine
Agonist for α1 and β1-adrenergic receptors
Vasoconstriction, mildly increased
Shock
contractility
Epinephrine
Dopamine
Agonist for primarily β1- and to lesser extent
Increased contractility (increased CO),
Anaphy lac tic shock,
α1- and β2-adrenergic receptors; α effects
vasodilation at low doses; increased
septic shock,
(i.e., vasoconstriction) predominate at high
contractility and vasoconstriction at
postbypass
doses
higher doses
hypotension
Agonist for β1-adrenergic receptors (low
Increased heart rate and contractility
Shoc k (renal sparing)
dose) and α-adrenergic receptors (high dose);
(increased CO), vasoconstriction (high
binding with dopaminergic receptors causes
dose only), increased renal blood flow
renal vasc ular vasodilation
Dobutamine
Isoproterenol
Agonist for β1-adrenergic receptors
Agonist for β1- and β2-adrenergic receptors
Increased heart rate and contractility
CHF, cardiogenic
(increased CO), mild reflex vasodilation
shock
Increased heart rate and contractility
Contractility stimulant
(increased CO), vasodilation
in cardiac arrest
Vasopressin
ADH analogue with weak pressor effect
Vasoconstriction
Resistant septic shock, second vasopressor
ADH, antidiuretic hormone; CHF, congestive heart failure; CO, cardiac output.
Neurogenic: the lack of a history of spinal trauma and the findings of an infectious process make this diagnosis an unlikely explanation
Acute renal failure is the first sign of multiorgan dysfunction seen with shock. Acute renal failure More thorough discussion in Chapter 5 Why eliminated from differential: some mild renal insufficiency is present in this case, but it is insufficient to account for this patient's hypotension, mental status changes, and high fever Myocardial infarction More thorough discussion in prior case Why eliminated from differential: the relatively normal ECG and cardiac enzymes rule out this diagnosis Pulmonary embolism More thorough discussion in Chapter 2 Why eliminated from differential: pulse oximetry and the ABG do not suggest this diagnosis; the insidious nature of the presentation is also less consistent with this diagnosis Acute respiratory distress syndrome/pneumonia More thorough discussion in Chapter 2 Why eliminated from differential: ARDS is very common in septic shock, and the bibasilar infiltrates seen on the CXR may be the early stages of a pulmonary process, but the full spectrum of findings is more consistent with a systemic response due to the bacteremia Cerebrovascular accident (CVA) More thorough discussion in Chapter 7 Why eliminated from differential: the mental status changes may be seen with a CVA, but the normal head CT makes this diagnosis unlikely
Case 1-14: “My heart is ripping out of my chest!” A 32-year-old man is brought to the emergency department with a complaint of sharp chest pain that began suddenly an hour ago. He says that he was moving some boxes of clothing into his basement when the pain started. The pain is sharp and tearing in nature, is focused behind his sternum, and radiates to his back. He also had mild dizziness and dysphagia since the pain started. He denies losing consciousness or other new symptoms. He denies any previous episodes of chest pain. He says that he has mitral valve prolapse but denies any other past medical history and only takes daily low-dose ASA for his condition. He denies any drug use. He is an only child and says that his father died when the patient was 2 years old from an unclear heart problem. On examination, the patient is a tall, thin male who is
very anxious. He is diaphoretic. Auscultation reveals clear lung fields, a blowing late systolic murmur that radiates from the apex to the axilla, no extra heart beats, and regular tachycardia. An abdominal examination does not detect any tenderness or palpable masses. His pulses are palpable and are not strong, but he has good capillary refill. He has very long arms and legs and long, thin digits. His neurologic examination is normal. The following vital signs are measured: T: 98.4°F, HR: 105 bpm, BP: 135/85 mm Hg, RR: 18 breaths/min
Differential Diagnosis Aortic regurgitation, aortic stenosis, MI, aortic dissection, aortic aneurysm, reflux disease, pancreatitis, acute pericarditis, pulmonary embolism
Laboratory Data and Other Study Results Pulse oximetry: 100% on room air CBC: WBC: 8.1, Hgb: 14.7, Plt: 240 Chem7: Na: 140 mEq/L, K: 4.1 mEq/L, Cl: 107 mEq/L, CO2: 25 mEq/L, BUN: 14 mg/dL, Cr: 0.8 mg/dL, Glu: 83 mg/dL Coags: PT: 12 sec, INR: 1.0, PTT: 35 sec Amylase: 75 U/dL Lipase: 50 U/L Cardiac enzymes: CK: 355 U/L, CK-MB: 4.5 ng/mL, troponin-I: 0.3 ng/mL ECG: sinus tachycardia; mildly increased amplitude in leads I, aVL, andV 4 to V6; 2 mm ST depression in all leads CXR: clear lung fields; widening of superior mediastinum CT angiography of chest: intimal tear of ascending aorta with false lumen from above the aortic root to the aortic arch; no extravasation of contrast; no intrapulmonary abnormalities
Diagnosis Aortic dissection (Stanford A)
Treatment Administered The patient's mild HTN was controlled with IV labetalol The patient was rushed to surgery for emergent reconstruction of the ascending aorta and arch with a synthetic Dacron graft The patient was admitted to the CICU postoperatively
Follow-up The patient recovered successfully following surgery and eventually returned home He was later diagnosed with Marfan syndrome The patient was maintained on a regimen of ASA and metoprolol with prohibitions on strenuous activity The patient followed-up closely with cardiology and cardiothoracic surgery to monitor the development of any further complications of his condition
Steps to the Diagnosis Aortic dissection Tear within the aorta extending from the intima to the media that creates a false lumen within the aorta Stanford classification Type A: involves the ascending aorta Type B: dissection that begins distal to the left subclavian artery Risk factors: HTN, trauma, coarctation of the aorta, syphilis, Ehlers-Danlos syndrome, Marfan syndrome History: acute “tearing” chest pain that extends to sternum (ascending aortic dissection), jaw (aortic arch), or scapula (descending aorta); hoarseness, dysphagia, paresthesia, paralysis, dyspnea, or syncope may occur depending on location and severity Physical examination: normal or increased BP, decreased peripheral pulses, neurologic deficits, Horner syndrome (i.e., ptosis, miosis, anhydrosis), heart murmurs, pulsus paradoxus, and decreased breath sounds are all possible findings Tests: Cardiac enzymes will be elevated if cardiac ischemia occurs due to dissection (e.g., ascending dissections); BUN and Cr will be elevated if the renal arteries are involved (e.g., low descending dissections) ECG is frequently normal or may show increased amplitude due to left ventricular hypertrophy CXR will demonstrate superior mediastinal widening for ascending dissections Echocardiogram (TEE preferred) can detect proximal dissections and aortic regurgitation resulting from extension into the aortic root CT angiography and MRI are highly sensitive for detecting intimal tears and false lumens and have displaced traditional angiography as a diagnostic tool (Figure 1-28) Treatment: Stabilize HTN with nitroprusside, calcium channel blockers, or β-blockers and reduce cardiac contractility with β-blockers Stanford A dissections require emergent cardiothoracic surgery to repair or reconstruct the ascending aorta Stanford B dissections may be treated medically unless rupture or occlusion develops Life-long β-blocker therapy and close cardiology follow-up is required Outcomes: Complications include MI, aortic regurgitations, CHF, pericardial tamponade, neurologic deficits, renal or mesenteric ischemia, and recurrent dissection Surgical treatment for any type of dissection is associated with a 30% risk of in-hospital mortality; in-hospital mortality with medical treatment alone is 60% for Stanford A dissections and 10% for Stanford B dissections The average 10-year survival is 60% and is influenced by the presence of comorbid conditions Clues to the diagnosis:
History: sudden tearing chest pain radiating to the back, sudden dizziness, and dysphagia Physical: tachycardia, weak pulses Tests: ECG appearance, widened mediastinum on CXR, CT angiogram appearance
Normal aortic diameter is 1.5 to 2.5 cm; increase to twice this size or more is considered aneurysmal. Aortic aneurysm Abdominal aortic aneurysm (AAA) Account for the majority of aortic aneurysms May be infrarenal (more common) or suprarenal and involving the renal arteries (less common, worse complications) Risk factors: atherosclerosis, HTN, tobacco, family history, age >55 years
Figure 1-28 Abdominal computed tomography angiogram demonstrating aortic dissection in a patient. (A) An abdominal type B aortic dissection is evident with the intimal wall bisecting the true lumen (arrowhead) from the false one. The celiac artery can be seen originating from the false lumen (arrow). (B) View of the same aortic dissection at a more distal level. The intimal wall separating the true and false lumens is still apparent. The superior mesenteric artery is originating from the true
lumen (arrow). K, kidney; L, liver; S, spleen.
Thoracic aortic aneurysm May involve the ascending aorta, aortic arch, or descending aorta Same risk factors as AAAs but also more common in patients with Ehlers-Danlos syndrome, Marfan syndrome, and syphilis Thoracoabdominal aortic aneurysm Extension of an aneurysm into both the thoracic and abdominal cavities History: Usually asymptomatic until progression or rupture AAAs are associated with lower back pain, groin pain, lower extremity numbness Thoracic aneurysms are associated with scapular or neck pain dysphagia, dyspnea, hoarseness, hemoptysis Physical examination: AAAs are associated with a pulsatile abdominal mass and abdominal bruits Thoracic aneurysms are associated with upper body edema, JVD, and neurologic deficits The development of hypotension and severe pain is suggestive of rupture Tests: X-rays may demonstrate a widened aortic shadow; the mediastinum may be wide for thoracic aneurysms Echocardiogram or abdominal US can be used to detect and measure aneurysms in the thoracic and abdominal regions, respectively; US screening should be performed once in any male over the age of 65 years with a smoking history CT angiography is able to provide the most accurate localization and size determination of aneurysms Treatment: Smoking cessation and the treatment of HTN are important to reduce the risk of rupture AAAs are monitored if <5 cm in diameter; surgical repair (open surgery or endovascular stent grafting) is performed for aneurysms >5 cm in diameter Thoracic aneurysms are repaired if >6 cm in diameter or if they are rapidly enlarging Outcomes: complications include rupture, renal insufficiency, mesenteric ischemia, CVA, neurologic deficits, MI, and spinal cord ischemia Why eliminated from differential: the symptoms for aortic dissection and rupture of an aortic aneurysm may be similar, but the appearance of the CT angiograph confirms the presence of the former diagnosis in this case
Rupture of an aortic aneurysm is typically fatal unless it occurs while the patient is in the hospital and immediate surgical control of the bleeding can be performed.
Aortic regurgitation and stenosis More thorough discussion in prior case Why eliminated from differential: chest pain may result from either of these conditions, and an ascending aortic dissection may cause secondary aortic regurgitation, but the identification of the intimal tear of the ascending aorta on CT confirms that this is the primary pathology in this case Myocardial infarction More thorough discussion in prior case Why eliminated from differential: chest pain and elevated cardiac enzymes (high normal in this case) are seen in MI, but identification of the intimal tear of the ascending aorta on CT confirms that this is the primary pathology in this case (although secondary myocardial ischemia would be common) Reflux disease More thorough discussion in Chapter 3 Why eliminated from differential: substernal chest pain is common for this condition, but the remainder of the workup is confirmatory of an aortic process Acute pericarditis More thorough discussion in prior case Why eliminated from differential: ST interval elevation is not seen on the ECG, and the CT confirms the presence of the aortic pathology Pancreatitis More thorough discussion in Chapter 3 Why eliminated from differential: severe epigastric may be seen in this diagnosis, but the normal amylase and lipase levels rule it out Pulmonary embolism More thorough discussion in Chapter 2 Why eliminated from differential: normal pulse oximetry, the absence of significant respiratory symptoms, and the results of the CT rule out this diagnosis
Case 1-15: “My legs ache so much” A 68-year-old woman presents to her PCP with the complaint of bilateral leg pain with activity. She says that she has had occasional leg pain (predominantly in her calves) for the past 6 years. She attributed this pain initially to getting older and standing all day at her job with the postal service, but she continued to have the same pain even after she retired at age 65. Although the pain has been predominantly during activity, she notes that it has gotten significantly worse in the past year and now sometimes occurs at rest. She can walk one block before the pain starts. She denies any swelling or redness in her legs. She does not feel that the pain occurs in her knees. She says the pain has typically improved after a period of rest and is helped slightly by taking ibuprofen. She admits to a history of hypercholesterolemia and HTN. She takes ASA, enalapril, HCTZ, and simvastatin on a daily basis. She has smoked a half pack of cigarettes per day for 40 years. She does not exercise due to her leg pain. Review of systems is significant for poor exercise tolerance and dry skin on her legs. On examination, she appears to be an overweight woman in no distress. Auscultation finds clear lungs, a regular heart rate, and no abnormal heart sounds. Abdominal examination detects no tenderness or masses. Examination of her extremities detects increased pigmentation in both of her shins, several small tortuous veins, and dry skin. It is difficult to palpate her dorsalis pedis and posterior tibial pulses, but
both are detectable by Doppler sonography. Her neurologic examination is normal. When the doctor has her walk in the hallway for 2 minutes, the pain is reproduced in her calves. The pain does not become worse with straight leg raises while lying supine. The following vital signs are measured: T: 98.3°F, HR: 84 bpm, BP: 125/80 mm Hg, RR: 15 breaths/min
Differential Diagnosis Peripheral artery disease, varicose veins, deep vein thrombosis (DVT), osteoarthritis, spinal radiculopathy, diabetes neuropathy
Laboratory Data and Other Study Results Chem10: Na: 143 mEq/L, K: 3.7 mEq/L, Cl: 102 mEq/L, CO2: 28 mEq/L, BUN: 30 mg/dL, Cr: 1.3 mg/dL, Glu: 95 mg/dL, Mg: 1.9 mg/dL, Ca: 10.3 mg/dL, Phos: 3.0 mg/dL Lipid panel: Total cholesterol: 190 mg/dL, LDL: 125 mg/dL, HDL: 38 mg/dL, Trig: 150 mg/dL D-dimer: 157 ng/mL Ankle-brachial index (ABI): 0.6 for right leg, 0.5 for left leg Bilateral knee and lower leg x-rays: minimally decreased joint spaces; no fractures or dislocations Based on these results, the following tests are performed: Bilateral lower extremity duplex US with venous Doppler study: bilateral popliteal artery stenoses; no venous occlusions Lower extremity magnetic resonance angiography (MRA): bilateral patent iliac and femoral arteries; bilateral popliteal stenoses with moderately decreased perfusion of distal vessels
Diagnosis Peripheral artery disease
Treatment Administered The patient was started on pentoxifylline to improve claudication symptoms She was encouraged to exercise as tolerated
Follow-up The patient had 3 months of improvement in symptoms before gradual worsening Percutaneous transluminal angioplasty (PTA) and stenting was performed for both popliteal arteries The patient had significant improvement in her symptoms following this procedure and was able to tolerate activities for much longer periods of time
Steps to the Diagnosis Peripheral artery disease Occlusion the peripheral arterial blood supply secondary to atherosclerosis and progressive endovascular damage
Risk factors: HTN, diabetes mellitus, CAD, smoking History: leg pain with activity that improves with rest (i.e., intermittent claudication), resting leg pain in severe disease, male impotence with aortoiliac involvement Physical examination: dry skin, skin ulcers, decreased hair growth, and increased skin pigmentation in affected area, diminished pulses Tests: The ankle-brachial index (ABI) is the ratio of SBP at the ankle to that at the brachial artery
Remember the six Ps to grade PVD severity: Pain, Pallor, Poikilothermia, Pulselessness, Paresthesia, and Paralysis. ABI <1 indicates vascular insufficiency at the ankle ABI <0.4 indicates severe disease (frequently seen with resting pain) Duplex US may be used to measure the waveforms in arteries and detect areas of abnormal blood flow and stenosis MRA is useful to localize the extent of the disease and to determine the severity in affected regions and has largely supplanted angiography as a diagnostic tool Treatment: Treat any comorbid conditions that contribute to worsening of the disease (e.g., CAD, HTN) Exercise helps to increase blood flow; smoking cessation is the most effective treatment for preventing further progression Patients should be instructed to examine their feet for signs of ulcers from vascular insufficiency ASA and pentoxifylline or cilostazol are useful to help to slow occlusion and provide some symptomatic relief Percutaneous transluminal angioplasty (PTA) is indicated for failed nonoperative treatment, significant disability due to claudication, or a predictable benefit and improvement in prognosis Bypass grafting is indicated if incapacitating claudication, resting pain, or necrotic foot lesions develop or when PTA fails to improve blood flow Prolonged ischemia may require limb amputation Outcomes: The number of risk factors correlates with the severity of disease 12% of patients with a 10-year history of intermittent claudication will require some type of amputation Long-term survival of patients with claudication (in the absence of successful treatment) is 70% at 5 years, 50% at 10 years, and 30% at 15 years Varicose veins Incompetent venous valves cause elongation, dilation, and tortuosity of veins History: usually asymptomatic; possible pain and fatigue that improves with leg elevation Physical examination: possible visible or palpable veins, increased skin pigmentation, edema, or ulceration
around involved veins Tests: Duplex US and contrast venography (sometimes performed with MRI) are used to map involved veins and locate sites of insufficiency Treatment: exercise, compression hosiery, and leg elevation improve symptoms; surgical removal or injection sclerotherapy may be performed for cosmetic improvement or symptomatic varicosities Outcomes: patients with severe disease are at a higher risk to develop venous ulcers; majority of patients have good outcomes with treatment Why eliminated from differential: the patient likely has some degree of varicose veins based on the physical examination, but varicosities cannot account for the impaired lower extremity perfusion in this case or the significant degree of the patient's pain
Removal or sclerotherapy of the saphenous vein is discouraged because of its potential use in bypass grafting. Deep vein thrombosis Development of a thrombosis in the large veins, which is at risk to dislodge and travel to another site of occlusion (i.e., embolization) Most common in the lower extremities (in order of increasing frequency: iliac, popliteal, femoral, calf veins) Involved veins may become significantly inflamed (i.e., thrombophlebitis) Risk factors: prolonged inactivity (travel, immobilization, recent surgery), heart failure, disorders of coagulation, neoplasm, pregnancy, OCP use, tobacco use, vascular trauma History: asymptomatic or deep pain Physical examination: tenderness, warmth, redness, or swelling over the area of involvement, pain with foot dorsiflexion (i.e., Homan sign; not very reliable) Tests: D-dimer is increased with DVT formation but has higher sensitivity than specificity; Doppler US is used for detection Treatment: Anticoagulation for at least 3 months either by heparin with a transition to warfarin or by use of LMWH Patients with contraindications to anticoagulation should have a venous filter placed in the inferior vena cava Direct thrombolysis of clots may be considered in DVTs located at the site of catheter placement Outcomes: complications include recurrent thrombosis, chronic venous insufficiency, and pulmonary embolism Why eliminated from differential: the chronicity of the patient's leg pain make this diagnosis unlikely; this is further confirmed by the absence of thrombosis on the US Osteoarthritis More thorough discussion in Chapter 9 Why eliminated from differential: the x-rays in the workup show only minimal joint degeneration; this diagnosis would not explain the perfusion defects in the legs
Spinal radiculopathy (disc herniation or spinal stenosis) More thorough discussion in Chapter 9 Why eliminated from differential: the patient does not have an exacerbation of leg pain with straight leg raises and lacks neurologic symptoms; this diagnosis also would not explain the perfusion deficits in the case Diabetes neuropathy More thorough discussion in Chapter 4 Why eliminated from differential: diabetic neuropathy may be a cause of leg pain but alone is not responsible for the perfusion deficits; however, DM is a common comorbid condition in these patients
Case 1-16: “What can you do to stop my headache?” A 75-year-old woman presents to her PCP with the complaint of a severe headache that has lasted for 2 days. The headache is unilateral, started suddenly while she was reading, and is focused on the left side of her head anterior and superior to her ear. She says that the headache has been so bad that she has had problems seeing out of her left eye and that it radiates into her jaw. She has never had a headache similar to this one in the past. She denies syncope or numbness and weakness in any extremities. She has a history of osteoarthritis and HTN for which she takes celecoxib and HCTZ. The celecoxib has helped her headache very little. She denies any substance use. Review of systems is significant for myalgias in her thighs and arms for the past year and arthralgias in both knees, her left hip, and her right shoulder. On examination, she is a thin elderly woman obviously bothered by pain. She is tender to palpation of her left temporal region. A fundoscopic examination appears normal. Auscultation finds no abnormal lung or heart sounds. Her abdomen is nontender. She has mild pain on palpation of her upper arms and hips. Her neurologic examination is normal, although a visual acuity test detects worse vision from her left eye compared to her right side. The following vital signs are measured: T: 100.3°F, HR: 76 bpm, BP: 124/76 mm Hg, RR: 15 breaths/min
Differential Diagnosis Temporal arteritis, Takayasu arteritis, CVA, trigeminal neuralgia, migraine headache, arteriovenous malformation, cerebral aneurysm, SLE, rheumatoid arthritis, polymyalgia rheumatica
Laboratory Data and Other Study Results CBC: WBC: 6.8, Hgb: 12.5, Plt: 627 ESR: 125 mm/h CRP: 5.2 mg/dL ANA: negative RF: negative Head MRI: normal cerebral volume; no lesions; no sites of hemorrhage; normal ventricles Scalp duplex US: no fluid collections; halo around the lumen of the left temporal artery
Diagnosis Temporal arteritis
Treatment Administered
The patient was place on high-dose prednisone, low-dose ASA, and vitamin D and calcium supplements Temporal artery biopsy was performed, which demonstrated inflammation throughout the temporal artery wall and the presence of multinucleated giant cells in the vessel adventitia
Follow-up The patient had improvement in her headache pain following the initiation of prednisone The patient's eyesight on the left improved but never fully returned to normal The patient was tapered off prednisone after 2 months without any recurrence of headache or any complications The patient was referred to ophthalmology to continue follow-up of her vision
Steps to the Diagnosis Temporal arteritis (a.k.a. giant cell arteritis) See Table 1-12 for a more thorough discussion of the most common vasculitides Clues to the diagnosis: History: severe unilateral temporal headache, acute vision deficits Physical: temporal tenderness, impaired unilateral visual acuity Tests: increased ESR and CRP, scalp US appearance
If you have a clinical suspicion for temporal arteritis, start corticosteroid therapy immediately and do not wait for a temporal artery biopsy to start therapy. Takayasu arteritis See Table 1-12 for a more thorough description of the most common vasculitides Why eliminated from differential: Takayasu and temporal arteritis may have very similar clinical and histologic presentations, but Takayasu arteritis is associated more commonly with the aorta and its branches rather than cranial vessels; biopsy in Takayasu arteritis may show inflammation focused on the vaso vasorum with extension to the media and adventitia Cerebrovascular accident More thorough discussion in Chapter 7 Why eliminated from differential: the absence of neurologic symptoms besides impaired left eye vision and the normal head MRI rule out this diagnosis Trigeminal neuralgia More thorough discussion in Chapter 7 Why eliminated from differential: both conditions may cause scalp and facial pain, but the high ESR and visual changes would not be seen in trigeminal neuralgia Migraine headache More thorough discussion in Chapter 7 Why eliminated from differential: unilateral temporal pain may be seen in both conditions, but the high ESR
and US findings would not be found in the case of a migraine headache Arteriovenous malformation (AVM) Abnormal communications between arteries and veins May be congenital or may develop later in life Large AVMs may cause local ischemia and have an increased risk of thrombosis Cerebral AVMs may cause hemorrhagic stroke if they bleed History: may be painful if mass effect compresses local structures Physical examination: warm, pulsating, palpable mass if superficial, possible neurologic deficits with hemorrhagic stroke Tests: MRA or angiography is used for detection of intracranial or deep lesions Treatment: endovascular embolization, focal radiation sclerosis, or surgical excision is performed for symptomatic or intracranial AVMs Outcomes: bleeding is the key problem and may cause visceral complications depending on the location; intracranial AVMs have a 4% risk of hemorrhaging per year and account for 2% of all hemorrhagic strokes Why eliminated from differential: the normal head MRI findings make this diagnosis unlikely Cerebral aneurysm (also refer to Chapter 7 for discussion of CVA) Saccular (e.g., berry aneurysm) or fusiform dilation of intracranial vessels Rupture can have dire consequences (e.g., hemorrhagic stroke, subarachnoid hemorrhage) History: typically asymptomatic until rupture Physical examination: neurologic deficits dependent on location Tests: high-resolution CT angiography, MRA, or cerebral angiography are used to detect aneurysms and determine their size Treatment: Observation may be chosen for unruptured aneurysms, but it carries the risk of rupture occurring Surgical clipping or endovascular embolization is frequently prophylactically performed in unruptured aneurysms
Table 1-12 Common Vasculitides
Disease
Pathology
Risk Fac tors
H/P
Tests
Treatment
Outc ome
Temporal
Subacute
Female
New onset of
Inc reased ESR,
Corticosteroids
Complications
arteritis
granulomatous
gender,
severe
increased
for one to two
include
(giant cell
inflammation of
age >50
headac he with
platelets,
months followed by
blindness,
arteritis)
carotid and
years
scalp pain,
cranial US
taper, low-dose
facial
temporal region
shows halo
ASA to reduce risk
ischemia,
tenderness, jaw
surrounding
of vessel occlusion,
frequent
claudication,
vessel lumen,
vitamin D and
recurrence,
transient or
temporal artery
calcium
frequent
permanent
biopsy shows
supplementation to
association
cranial arteries
monocular
inflammation in
reduce risk of
with
blindness,
vessel media
steroid-induced
polymyalgia
weight loss,
and adventitia
osteoporosis, close
rheumatica
myalgias,
and presence of
ophthalmology
arthralgias,
multinucleated
follow-up to track
fever, possible
giant cells
visual improvement
thrombosis of ophthalmic or ciliary arteries on fundoscopic examination
Takayasu
Inflammation of
Asian
Malaise, vertigo,
Increased ESR,
Corticosteroids,
15-year
arteritis
aorta and its
heritage,
syncope, fever,
MRA or
immunosuppressive
survival is
branc hes that
women
reduced carotid
angiography
agents, bypass
90%–95%
may result in
10–40
and extremity
shows multiple
grafting of
myocardial or
years old
pulses
stenoses and
obstructed vessels
cerebrovascular
aneurysms of
ischemia
involved vessels, biopsy shows plasma cell and lymphocyte infiltration and fibrosis of vessel vaso vasorum, media, and adventitia
Polyarteritis
Inflammation of
Hepatitis
Weight loss,
Increased WBC,
Corticosteroids,
Complications
nodosa
small and
B or C,
arthralgias,
decreased Hgb,
immunosuppressive
include tissue
medium arteries
increased
fever, HTN,
increased ESR,
agents
necrosis and
leading to
age, male
hematuria,
hematuria and
visceral
ischemia of
gender
neuropathy,
proteinuria on
hemorrhage;
palpable
UA, positive
5-year
purpura or
p-ANCA
mortality
ulc ers
(50%–80%
depends on
patients),
number of
numerous
systems
aneurysm on
involved and
angiography
is 12%–46%
supplied tissues
Churg-
Allergic
Asthmalike
Inc reased
Corticosteroids,
5-year
Strauss
reac tion of
symptoms,
serum
immunosuppressive
survival is
disease
inflammation of
fatigue, malaise,
eosinophils,
agents
25% without
(allergic
small and
mononeuropathy
increased ESR,
treatment
angiitis)
medium arteries
(e.g., pain,
positive ANCA,
and 62% with
paresthesias, or
inc reased
treatment
weakness),
serum IgE, lung
(usually due
erythematous or
biopsy shows
to
popular rash
eosinophilic
myocarditis
granulomas
or MI)
Asthma
Henoch-
IgA immune
Children,
Rec ent upper
Increased WBC,
Frequently
Recurrence
Schönlein
complement-
Caucasians
respiratory
increased ESR,
self-limited;
in 50% of
purpura
mediated
infec tion,
inc reased
significant GI
patients,
inflammation of
polyarticular
serum IgA ,
symptoms suggest
rare renal
arterioles,
arthritis, fever,
proteinuria and
use of
insufficiency
capillaries, and
hemorrhagic
hematuria on
corticosteroids
venules
urtic aria,
UA, biopsy of
palpable
purpura or
purpura,
kidneys shows
abdominal pain,
IgA deposition
possible GI bleeding
Kawasaki
Necrotizing
Y oung
Fever,
Endothelial
ASA, IV gamma
Complications
disease
inflammation of
c hildren,
lymphadenopathy,
autoantibodies,
globulin
include heart
large, medium,
Asian
conjunctival
increased ESR
failure,
and small
heritage
lesions,
and CRP,
myocarditis,
maculopapular
increased WBC,
valvular
arteries
rash, edema,
decreased Hgb,
desquamation of
echocardiogram
hands and feet
or angiography
pathology
shows coronary artery aneurysms
ANCA,; ASA, aspirin; CRP, C-reactive protein; ESR, erythrocyte sedimentation rate; GI, gastrointestinal; Hgb, hemoglobin; H/P,; HTN, hypertension; IgA, immunoglobulin A; IgE, immunoglobulin E; IV, intravenous; MI, myocardial infarction; MRA, magnetic resonance angiography; UA, urinalysis; US, ultrasound; WBC, white blood cells.
Ruptured aneurysms require treatment to avoid rebleeding episodes Outcomes: complications include subarachnoid hemorrhage, severe neurologic impairment from hemorrhagic stroke, and death Why eliminated from differential: the absence of intracranial hemorrhage, the high ESR, and the US findings help to rule out this diagnosis Systemic lupus erythematosus More thorough discussion in Chapter 9 Why eliminated from differential: the normal ANA and lack of other characteristic findings make this diagnosis unlikely Rheumatoid arthritis (RA) More thorough discussion in Chapter 9 Why eliminated from differential: the negative rheumatoid factor, focus of symptoms on cranial symptoms, and characteristic US findings rule out this diagnosis Polymyalgia rheumatica (PMR) More thorough discussion in Chapter 9 Why eliminated from differential: while this condition frequently occurs in association with temporal arteritis, it is mainly a condition of polyarthralgia and myalgia, and severe headaches are not a common component
Case 1-17: “The pediatrician says my baby has a heart murmur” A 2-month-old girl is brought to a pediatric cardiologist on referral by her pediatrician. The patient's mother says that the pediatrician detected a heart murmur on the initial well-baby appointment that has remained consistent on the follow-up visits. Given the age of the child and the volume of the murmur, the pediatrician felt that it may be nonphysiologic. The mother says that her baby has been healthy without any serious problems. She was born at 39 weeks of gestation through a routine vaginal delivery and was 3.6 kg birth weight (65th percentile) and 48 cm birth length (60th percentile). She has eaten well during breastfeeding with normal weight maintenance. The mother denies any illnesses since birth or episodes of distress. The mother denies any complications during her pregnancy. She has one other child, who is 3 years old and healthy. On examination, the infant is well appearing and active. She is light pink in color with strong pulses throughout and good capillary refill. Auscultation reveals clear lung sounds, an easily audible (grade III/VI) pansystolic murmur that is loudest at the lower sternal border, and a loud second heart sound with normal splitting. A minimal systolic thrill is palpable. The infant has no abdominal masses and normal bowel sounds. She is actively moving all extremities, which appear to be symmetric. The following vital signs are measured: T: 98.8°F, HR: 125 bpm, BP: 90/70 mm Hg, RR: 45 breaths/min
(Normal for 2-month-old infant: HR: 100 to 170 bpm, BP: 65 to 115/42 to 80, RR: 30 to 50 breaths/min)
Differential Diagnosis Ventricular septal defect, atrial septal defect, patent ductus arteriosus, physiologic murmur, aortic or pulmonary stenosis
Laboratory Data and Other Study Results ECG: age-appropriate sinus rhythm and rate; normal wave morphology CXR: normal cardiac size; normal aortic and pulmonary vascular shadows Echocardiogram: small membranous ventricular septal defect with widest dimension slightly less than half the diameter of the aortic annulus; small degree of left-to-right flow; normal valve appearance and function
Diagnosis Ventricular septal defect
Treatment Administered Given the small size of defect, the decision was made to observe the anomaly with regular cardiologic follow-up
Follow-up The infant grew normally on the growth curve without development of respiratory symptoms The size of the defect decreased over the following 2 years until a minimal opening existed that remained stable in size
VSDs are the most common type of congenital heart defect.
Steps to the Diagnosis Ventricular septal defect (VSD) An opening in membranous or muscular portion of the interventricular septum that allows shunting of blood (Figure 1-29A)
Figure 1-29 Diagrams of congenital heart defects. (A) Ventricular septal defect (VSD)—depicts shunting of blood predominantly from the left to right ventricle (arrows). (B) Atrial septal defect (ASD)—depicts shunting of blood from the left to right atrium (arrows). (C) Patent ductus arteriosus (PDA)—depicts shunting of blood from the aorta to the pulmonary arteries (arrows). (D) Transposition of the great vessels—the aorta arises from the right ventricle, and the pulmonary artery arises from the left ventricle. Shunting may occur between the great vessels via a PDA. (E) Persistent truncus arteriosus—single vessel exits both ventricles (with VSD) and gives rise to both systemic and pulmonary circulation. (F) Complete endocardial cushion defect—ASD, VSD, and single atrioventricular canal. (G) Tetralogy of Fallot—combination of ventricular septal defect, right ventricular outflow obstruction, aorta overriding the ventricular septum, and right ventricular hypertrophy.
History: asymptomatic if small, but larger defects may be associated with frequent respiratory infections, failure to thrive, dyspnea, orthopnea, cough, and edema Physical examination: pansystolic murmur at lower left sternal border, systolic thrill, loud pulmonic S2 heart sound Tests: ECG may demonstrate left and right ventricular hypertrophy in chronic cases but is usually normal; echocardiogram demonstrates the septal defect and may be used to measure the velocity of shunted flow
Treatment: Small defects (i.e., less than half the diameter of the aortic annulus) may be observed for the development of signs of heart failure Larger shunts that cause symptoms may be treated with ACE-Is or diuretics to decrease intravascular fluid volumes and resistance Symptomatic large defects that do not respond to medical treatment should be surgically repaired before Eisenmenger syndrome develops Outcomes: small VSDs will spontaneously close in 50% of cases; the prognosis in large VSDs correlates with the initiation of treatment prior to the development of heart failure Clues to the diagnosis: History: noncontributory Physical: pansystolic murmur, systolic thrill Tests: echocardiogram appearance
All congenital heart defects are associated with a higher risk of endocarditis, so patient should always take prophylactic antibiotics prior to any invasive procedure. Atrial septal defect (ASD) An opening in the atrial septum allowing shunting of blood between the atria (Figure 1-29B) Left-to-right shunting is the initial pattern of abnormal blood flow Right-to-left shunting will develop in chronic cases that develop heart failure History: frequently asymptomatic, but symptoms of heart failure (e.g., dyspnea, orthopnea, edema), fatigue, failure to thrive, and rarely cyanosis may be seen in large defects Physical examination: strong palpable impulse at the lower left sternal border, S2 heart sound with fixed wide splitting (not heard in VSD), systolic ejection murmur at upper left sternal border Tests: ECG shows right axis deviation CXR shows increased pulmonary vascular markings due to relative pulmonary HTN Echocardiogram shows atrial septal defect, shunting of blood flow, dilated right ventricle, and cardiomegaly Treatment: small defects are observed for signs of worsening shunting; surgical closure is performed for symptomatic infants or when the pulmonary blood flow becomes twice that of systemic blood flow Outcomes: although the majority of children will require eventual surgical closure, a minority (i.e., 15%) will undergo spontaneous closure; potential complications include CHF, pulmonary HTN, endocarditis, and recurrent respiratory infections Why eliminated from differential: the fixed wide splitting of the second heart sound expected for an ASD is not apparent in this case; the results of the echocardiogram confirm the diagnosis of VSD
Patients with ASDs are more susceptible to oxygen desaturation at high altitudes and
decompression sickness during deep sea diving. Patent ductus arteriosus (PDA) Failure of the ductus arteriosus to close at the time of birth resulting in a left-to-right aorta-to-pulmonary artery shunt (Figure 1-29C) Risk factors: prematurity, high altitude, first-trimester maternal rubella, maternal prostaglandin administration, female gender History: may be asymptomatic, or symptoms of heart failure may be experienced Physical examination: wide pulse pressure, bounding pulses, continuous “machinery” murmur at second intercostals space, loud S2 heart sound Tests: ECG may show signs of left ventricular hypertrophy CXR may demonstrate cardiomegaly Echocardiogram shows a large left atrium and ventricle Angiography can confirm the presence of an abnormal vascular conduit Treatment: indomethacin induces closure of the ductus arteriosus; surgical closure is required for closure if patency is maintained following medical treatment or in case of large defects Outcomes: no complications develop following closure; untreated cases are at a higher risk for CHF, pulmonary vascular abnormalities, and endocarditis Why eliminated from differential: the murmur in this case is not consistent with what would be expected for a PDA; the echocardiogram confirms the diagnosis
Treatment for a PDA may be remembered with the mnemonic “Come In and Close the Door”: (indomethacin closes a PDA).
A PDA or VSD is necessary for survival with transposition of the great vessels. Physiologic murmur Faint murmurs in normal, healthy hearts that are caused by vibration of the valve leaflets or pulmonary trunk or by the flow of blood through the heart More common in children, and may disappear with age Why eliminated from differential: physiologic murmurs are not louder than grade II to VI; the echocardiogram confirms that the murmur is due to a cardiac defect and is not physiologic Aortic or pulmonary stenosis More thorough discussion in prior case Why eliminated from differential: systolic murmurs would be expected for these conditions, but the echocardiogram demonstrates no valvular pathology
Case 1-18: “Your baby is cyanotic” A full-term girl neonate is delivered by an uncomplicated caesarian section following concerning fetal heart tracings
during the initial stage of labor. Apgar scores for the child are 7 at 1 minute and 9 at 5 minutes after delivery. The mother is doing well after the procedure. Because of the multiple late decelerations on the fetal heart tracing prior to delivery, the decision is made to closely observe the neonate in the neonatal intensive care unit during the immediate postpartum period. Several hours following the delivery, the neonatal care staff notices that the child is becoming mildly cyanotic. The child has remained active during this time, but the cyanosis worsens during crying episodes. The neonatal pediatrician is called to evaluate the child. On examination, the child is found to have mild cyanosis that is worse around the eyes and mouth. No obvious deformities are notable. Auscultation detects clear lungs, a continuous murmur that worsens during systole, and a galloping heart rhythm with a third heart beat. Abdominal examination detects mild hepatomegaly but no other masses. The extremity examination detects no abnormalities other than cyanosis. The neurologic examination appears to be normal. The following vital signs are measured: T: 99.6°F, HR: 160 bpm, BP: 75/50 mm Hg, RR: 45 breaths/min (Normal for neonate: HR: 100 to 170 bpm, BP 65 to 95/30 to 60, RR: 30 to 50 breaths/min)
Differential Diagnosis Transposition of the great arteries, tricuspid atresia, persistent truncus arteriosus, endocardial cushion defect, tetralogy of Fallot
Laboratory Data and Other Study Results Pulse oximetry: 76% on room air, 86% on 100% O2 ABG (on 100% O2): pH: 7.38, pO2: 90 mm Hg, pCO2: 46 mm Hg, Bicarb: 23 mEq/L, O2 sat: 89% CXR: cardiomegaly; significantly increased pulmonary vascular markings Echocardiogram: reversal of aorta and pulmonary artery origins with the former arising from the right ventricle and the latter arising from the left ventricle; moderate-sized VSD with significant mixing of circulations; patent ductus arteriosus; normal origin of coronary vessels from aorta
Diagnosis Transposition of the great arteries
Treatment Administered The patient was kept in a high-oxygen environment Prostaglandin E1 was administered continuously to maintain ductus arteriosus patency The patient was scheduled for surgical correction of the defect within the first week following birth; reversal of arterial origins and closure of the VSD and PDA were performed
Follow-up The patient survived the surgical correction and was readmitted to the neonatal intensive care unit following procedure The patient was maintained on furosemide while in intensive care to reduce the intravascular fluid volume The patient was eventually able to be discharged to home with her parents
Steps to the Diagnosis Transposition of the great arteries
Reversal of the systemic and pulmonary circulations in which the aorta originates from the right ventricle and the pulmonary artery originates from the left ventricle (Figure 1-29D) Venous connections to the heart remain normal High frequency of other congenital heart defects Cause is likely related to the development of the cardiac septum Risk factors: Apert syndrome, Down syndrome, cri-du-chat syndrome, trisomy 13 or 18 History: cyanosis at the time of birth or within hours of birth, cyanosis worsens with crying, dyspnea develops without early treatment Physical examination: murmurs consistent with comorbid cardiac anomalies, possible extra heart sounds or a galloping heart rhythm, tachypnea, tachycardia, hepatomegaly Tests: Pulse oximetry will demonstrate decreased O2 saturation ABG on 100% O2 will demonstrate pO2 <150 mm Hg CXR will show cardiomegaly and prominent pulmonary vascular markings Echocardiogram will demonstrate the reversal of the arterial origins and can be used to detect any other cardiac defects Treatment: The infant is kept in a high O2 environment Prostaglandin E1 is administered continuously to prevent closure of a patent ductus arteriosus Surgical reversal of the great arteries is required to avoid permanent cardiac complications Antibiotic prophylaxis is required prior to any invasive procedure following treatment Outcomes: poor prognosis in untreated children with 30% mortality in the first week, 50% in the first month, and 90% in the first year; 90% of children who undergo arterial reversal survive and are able to lead normal lives Clues to the diagnosis: History: predelivery fetal distress Physical: cyanosis, continuous murmur Tests: hypoxemia responding poorly to supplemental oxygen, cardiomegaly on CXR, echocardiogram findings
Cyanotic congenital heart diseases may be remembered by the 5 Ts: Tetralogy of Fallot, Total anomalous pulmonary venous return, Transposition of the great vessels, Tricuspid atresia, and Truncus arteriosus.
Significant hypoxemia in an infant that improves minimally with supplemental oxygen is highly suggestive of transposition of the great vessels. Tricuspid atresia
Failure of the tricuspid valve to develop Blood flow into the pulmonary artery is impeded An anomalous connection between the systemic and pulmonary circulations is required for survival History: cyanosis that develops soon after birth, frequent respiratory infections Physical examination: tachypnea, tachycardia, single S 2 heart beat, holosystolic murmur Tests: Pulse oximetry will indicate low O2 saturation and ABG will demonstrate hypoxemia CXR shows a normal-sized heart with decreased pulmonary markings Echocardiogram shows a small right ventricle, large right atrium, absence of a tricuspid valve, and any comorbid anomalies Treatment: prostaglandin E1 is administered if a PDA is present; surgical reconstruction or atrial balloon septostomy is attempted to allow adequate blood delivery to the pulmonary vasculature Outcomes: Untreated disease has a poor prognosis and is associated with severe heart failure and pulmonary collapse Several complications may follow surgical correction including arrhythmias, cardiac thrombosis, heart failure, and endocarditis Postoperative patients have a significant rate of mortality (30%) and do not achieve a level of function similar to people without the condition Why eliminated from differential: the echocardiogram is able to rule out this diagnosis and confirm the arterial reversal Persistent truncus arteriosus Failure of the aorta and pulmonary arteries to separate during development, resulting in a single vessel that supplies both the systemic and pulmonary circulations (Figure 1-29E) History: cyanosis shortly after birth, dyspnea, fatigue, failure to thrive Physical examination: harsh systolic murmur at lower left sternal border, loud S1 and S2 heart sounds, hepatomegaly, possible sudden circulatory collapse Tests: ECG may show left and right ventricular hypertrophy CXR shows a boot-shaped heart, absence of a pulmonary artery shadow, and a large outflow vessel arching to the right side Echocardiogram demonstrates a single vessel serving as the outflow for both ventricles and can be used to measure the flow to each branch of circulation Treatment: Digoxin and diuretics are frequently used to treat heart failure Prostaglandin E1 is administered to maintain patency of any open conduits between the circulations Surgical reconstruction is required to develop separate circulatory systems
Outcomes: 90% of patients who undergo surgical reconstruction survive to be discharged to home; reoperation rates are extremely high during growth to maintain the reconstructed circulations Why eliminated from differential: the appearances of the CXR and echocardiogram in this case are able to rule out this diagnosis
20% of children with Down syndrome have some type of endocardial cushion defect. Endocardial cushion defect Malformation of the atrioventricular valves, atrial septum, and/or ventricular septum during fetal development causing a variety of valvular and septal defects Complete defects have an ASD, a VSD, and a single atrioventricular canal (Figure 1-29F) Incomplete defects have an ASD and minor atrioventricular valve abnormalities History: presentation for incomplete forms resembles that for an ASD, and complete defects are characterized by heart failure symptoms Physical examination: murmurs appropriate for each particular defect, hepatomegaly, edema Tests: ECG shows left axis deviation; echocardiogram will detect the various congenital defects Treatment: Digoxin and diuretics are frequently used to treat heart failure Prostaglandin E1 is administered to maintain patency of any open conduits between the circulations
Authors: Van Kleunen, Jonathan P. Title: Step-Up to USMLE Step 3, 1st Edition Copyright ©2009 Lippincott Williams & Wilkins > Table of Contents > Chapter 2 - Pulmonary Medicine
Chapter 2 Pulmonary Medicine Basic clinical primer Lung Volumes Air volume varies with effort of breathing (Figure 2-1, Table 2-1)
Measurements of Airflow and Air Exchange Airflow FEV 1/FVC is the ratio of forced expiratory volume over 1 second (FEV1) to functional vital capacity (FVC) FEF25%–75% is the forced expiratory flow rate between 25% and 75% of FVC Quantities describe how much air the lungs are able to expire over a finite period of time Air exchange Diffusing capacity of lungs (D Lco) measures the relative success of the lungs in transferring gas from the pulmonary alveoli to pulmonary capillaries Expressed as a percentage of the expected value
Normal FEV1/FVC is 80%; <80% suggests an obstructive pathology; >110% suggests a restrictive pattern. Alveolar-arterial gradient (A-a gradient) Compares the oxygenation status of alveoli (PAo2) to that for arterial blood (Pao2) Normal gradient is 5 to 15 mm Hg Increases in pulmonary embolism (PE), pulmonary edema, right-to-left cardiac shunting, or in any case in which resistance against right ventricular cardiac outflow increases Facetious normal values may be seen with hypoventilation or at high altitudes PAo2 is calculated in the following manner:
for room air, this equation becomes:
Pao2 is measured directly from an arterial blood gas (normal is 90 to 100 mm Hg at room air) Paco2 is measured directly from an arterial blood gas (normal is 40 mm Hg at room air) Fraction of oxygen (O2) in inspired air (Fio2) is considered 0.21 for room air A-a gradient is calculated using the variables listed above:
P.44
Figure 2-1 Diagram of healthy lung volumes and their variation with the effort of respiration. ERV, expiratory reserve volume; FRC, functional reserve capacity; FVC, functional vital capacity; IC, inspiratory capacity; IRV, inspiratory reserve volume; RV, residual volume; TV, tidal volume; Vol, volume. (See color image.)
Pulmonary Function Tests (PFTs) Measure variations in lung volumes and compare them to normal values (Table 2-2) Useful to categorize lung processes as obstructive or restrictive Useful to assess disease severity and success of treatment
Intubation and Mechanical Ventilation Intubation Placement of an endotracheal tube to maintain airway patency and to allow mechanical ventilation during general anesthesia and in times of respiratory distress Tube placement Nearly all are performed orally (nasal intubation is indicated for oral surgery and when vocal cords cannot
be visualized by a laryngoscope)
Table 2-1 Definitions of Lung Volume Terms and Formulas
Lung V olume
Definition
TV
Inspiratory volume during normal respiration
IRV
Air volume beyond normal tidal volume that is filled during maximum inspiration
IC
Total inspiratory air volume considering both tidal volume and inspiratory reserve volume (IC = TV + IRV )
ERV
Air volume beyond tidal volume that can be expired during normal respiration
RV
Remaining air volume left in lung following maximum expiration
FRC
Air volume remaining in lungs after expiration of tidal volume (FRC = RV + ERV )
FVC
Maximum air volume that can be inspired and expired (FV C = IC + ERV )
TLC
Total air volume of lungs (TLC = FV C + RV )
ERV, expiratory reserve volume; FRC, functional reserve capacity; FVC, functional vital capacity; IC, inspiratory capacity; IRV, inspiratory reserve volume; RV, residual volume; TLC, total lung capacity; TV, tidal volume.
P.45
Table 2-2 Changes in Pulmonary Function Tests from Normal Lung to Obstructive and Restrictive Disease States
Measurement
Obstruc tive
Restric tive
TLC
↑
↓
FVC
↑
↓
RV
↑
↓
FRC
↓↓
↓
FEV 1
↓
↓
FEV 1/FVC
↓
Normal or ↑
FEV 1, one-second forced expiratory volume; FRC, functional reserve capacity; FVC, functional vital capacity; RV, residual volume; TLC, total lung capacity; ↑, increase; ↓, decrease; ↓↓, large decrease.
Anesthesia and muscle relaxants are first administered The patient is placed into mild cervical flexion while in a supine position A laryngoscope is inserted into the mouth to lift the jaw and to visualize the vocal cords (cricoid pressure may ease visualization) An endotracheal tube is inserted past the vocal cords (must be visually confirmed!) to 21- to 23-cm depth (measured at the lips) Proper placement is confirmed with end-tidal carbon dioxide (CO2) measurement and by confirming bilateral lung expansion during inspiration The endotracheal tube is adjusted as needed, the cuff is inflated, and the tube is secured to the mouth with tape Complications of placement include dental injury, esophageal placement, and increased infection risk Consider conversion from an endotracheal tube to a tracheostomy if ventilation will last longer than 3 weeks Ventilation Assisted respiration is required when a patient is unable to breathe under his or her own power (e.g., anesthesia, respiratory distress or collapse, severe hypoxia, decreased respiratory drive, or neurologic injury) (Table 2-3 presents a description of general settings)
Table 2-3 Modes of Mechanical Ventilation
Mode
CMV
Mac hine Ac tions
Determines and automatically delivers tidal
Patient Ac tions
Uses
No effort
General anesthesia, overdose
Determines and automatically delivers tidal
Can breathe spontaneously
Weaning patient from ventilator
volume and rate
between mechanical breaths
Machine tries to synchronize rate with
Can breathe spontaneously
More comfortable for patient because of
patient-initiated breaths; automatically
between mechanical breaths
attempted synchronization; frequently
volume and rate
IMV
SIMV
used in place of IMV
delivers tidal volume and rate
AC
CPAP
Machine senses patient's attempt to breathe
Patient driven unless no
Used when patient is more awake and in
and delivers full preset tidal volume; backup
attempts to breathe
progressive weaning
rate if no spontaneous breaths
(backup rate)
Machine maintains airway patency to decrease
Patient does all breathing
work of breathing
Used when patient relies less on ventilator; intubation not required
AC, assist-control ventilation; CMV, controlled mechanical ventilation; CPAP, continuous positive airway pressure; IMV, intermittent mandatory ventilation; SIMV, synchronized intermittent mandatory ventilation.
P.46 The ventilator controls inspiration while expiration occurs through natural recoil of the lungs Tidal volume, respiratory rate, Fio2, and inspiratory pressure may be adjusted for variability in respiratory drive, compliance, and oxygenation status Positive end-expiratory pressure (PEEP) helps maintain alveolar patency during expiration Ventilator weaning protocols involve changing modes from patient-independent to more patientdependent modes; extubation is performed when a patient is able to breathe by him- or herself on minimal dependent settings Predictors of weaning success Pao2 >60 mm Hg with an Fio2 <0.35 Pao2/Fio2 >200 Vital capacity >10 mL/kg Maximum negative inspiratory pressure <30 cm water (H2O) Minute ventilation <10 L/min
It is important to visualize the insertion of the endotracheal tube between the vocal cords to reduce the risk of esophageal placement.
Preoperative Cardiac Risk Assessment Risk assessment of a preoperative patient to determine the likelihood of pulmonary complications including the inability to extubate following surgery Smoking Increases the risk of infection and the need for postoperative ventilation Cessation should occur prior to surgery (i.e., preferably 8 weeks) Nicotine replacement may be useful to help patients quit smoking Preoperative testing A CXR is routinely performed in any patient older than 50 years or with a history of pulmonary disease or prior to a surgery with an anticipated length beyond 3 hours PFTs are indicated in patients with pulmonary disease or respiratory concerns (e.g., smokers, chronic obstructive pulmonary disease [COPD], myasthenia gravis) to assess their respiratory capacity and to anticipate the need for prolonged ventilation Preoperative antibiotics Given to any patient with a significantly increased risk of developing a respiratory infection following intubation (e.g., COPD) Anticipated postoperative care Incentive spirometry, deep breathing exercises, pain control, and physical therapy are all very important postoperatively to help prevent atelectasis, pneumonia, and pulmonary embolism Bronchodilators and inhaled corticosteroids may be beneficial in postoperative patients with preexisting
obstructive disease
LUNG TRANSPLANTATION Indicated for COPD (particularly patients with α1-antitrypsin deficiency), primary pulmonary hypertension (HTN), cystic fibrosis, or idiopathic pulmonary fibrosis with an estimated survival of <2 years Contraindicated in patients with poor cardiac function, renal or hepatic insufficiency, or other terminal illnesses; smokers (any use within 6 months) and patients older than 65 years are also excluded Acute rejection is common, and most patients have at least one episode; chronic rejection is also common The risk of pneumonia in the postoperative period is fairly high Three-year survival is 56% P.47
Case 2-1: “I have a sore throat and a cold” A 25-year-old man presents to his primary care provider (PCP) with a 4-day history of fatigue, sore throat, nasal congestion, and a nonproductive cough. He states that he has not taken his temperature at home and has had a few episodes of chills. He is unable to describe any inciting event for his symptoms but describes similar symptoms when he has had a “cold” in the past. He has had some relief of his symptoms with an over-the-counter cold medicine. He denies any past medical history or substance use. On examination, he is found to have pharyngeal erythema without purulent drainage and mild cervical lymphadenopathy. Auscultation detects slightly congested breath sounds and normal heart sounds. The abdominal examination finds no tenderness or masses. The following vital signs are measured: Temperature (T): 100.2°F, heart rate (HR): 84 beats per minute (bpm), blood pressure (BP): 110/70 mm Hg, respiratory rate (RR) 16 breaths/min
Differential Diagnosis Viral pharyngitis, bacterial pharyngitis, tonsillitis, viral rhinitis, influenza
Laboratory Data and Other Study Results Complete blood cell count (CBC): white blood cells (WBC): 9.0, hemoglobin (Hgb): 14.5, platelets (Plt): 300 Rapid streptococcal antigen test: negative Throat swab culture: negative (results reported after the initiation of therapy)
Diagnosis Viral pharyngitis
Treatment Administered Supportive therapy including hydration, expectorant/cough suppression medication, analgesia, and rest was prescribed No antibiotics were given
Follow-up The patient's symptoms gradually improved and resolved over the next few days
Steps to the Diagnosis Viral pharyngitis A common viral upper respiratory infection with symptoms due to predominance of inflammation in the oropharynx (Figure 2-2) History: sore throat, nasal congestion Physical examination: pharyngeal erythema, absence of tonsillar exudates, variable degree of lymphadenopathy, possible low-grade temperatures or fever (above 101.5°F) Tests: normal WBC; negative rapid streptococcal antigen test and culture Treatment: supportive therapy only (e.g., hydration rest, over-the-counter cold medicines, analgesia) Outcomes: symptoms should resolve over 7 to 10 days with no long-term effects; usually the diagnosis made and treatment is initiated before study results are reported P.48
Figure 2-2 Division of the respiratory tree into upper and lower regions and the appropriate sites of infection. (See color image.)
Clues to the diagnosis: History: fatigue, nasal congestion, nonproductive cough, sore throat Physical: pharyngeal erythema, cervical lymphadenopathy, congested breath sounds Tests: noncontributory
Antibiotic therapy is of no benefit in the treatment of viral upper respiratory infections and may contribute to the development of resistant strains of bacteria. Bacterial pharyngitis and tonsillitis Bacterial infection of the pharynx and possible involvement of the palatine tonsils that is most commonly due to group A β-hemolytic streptococcus History: sore throat, malaise, headache Physical examination: pharyngeal erythema, tonsillar exudates, variable lymphadenopathy, fever (more common than in the viral form) Tests: normal or mildly increased WBC; rapid streptococcal antigen test and culture will be positive Treatment: Supportive therapy β-lactam antibiotics reduce the infection length Tonsillectomy may be performed in the case of recurrent infections Outcomes: Symptoms will likely self-resolve even without treatment Untreated infection may cause rheumatic heart disease (3% of cases) or scarlet fever (i.e., high fevers and a rash during a streptococcal infection) Severe tonsillitis may cause airway obstruction or abscess formation requiring surgical drainage Why eliminated from differential: the absence of tonsillar exudates and only a low-grade fever are seen in this case; the presence of a cough and nasal congestion make this diagnosis unlikely
Nasal congestion and a cough are typically not seen in bacterial pharyngitis and are more indicative of a viral pathology.
Completing a prescribed course of antibiotics for bacterial pharyngitis is important to preventing relapse and the development of antibiotic resistance.
Respiratory infections above the level of the thyroid cartilage are considered upper respiratory infections; those below the thyroid cartilage are lower respiratory infections. Viral rhinitis Inflammation of the upper airways most commonly caused by rhinovirus, coronavirus, or adenovirus History: mild sore throat, prevalent nasal congestion and increased secretions (i.e., rhinorrhea), cough P.49 Physical examination: mild pharyngeal erythema, possible lymphadenopathy, or low-grade fevers Tests: negative rapid streptococcal antigen test and culture
Treatment: supportive therapy Outcomes: self-limited Why eliminated from differential: pharyngeal involvement predominates in this case and is more consistent with pharyngitis Viral influenza Systemic infection with upper respiratory infection (URI) symptoms caused by one of several influenza viruses History: arthralgias, myalgias, sore throat, nonproductive cough, nausea, vomiting and diarrhea Physical examination: high fevers, lymphadenopathy Tests: serologic tests are confirmatory of etiology (rarely ordered) Treatment: Supportive care (particularly hydration) Amantadine may shorten the disease course Health-care workers, elderly patients, immunocompromised patients, and patients with respiratory diseases should receive annual vaccination Outcomes: self-limited; complications may arise from dehydration without an adequate fluid intake Why eliminated from differential: the absence of systemic symptoms including myalgias and only low-grade fevers are seen in the case; pharyngeal erythema is more suggestive of a local pharyngeal process
Remember the symptoms for viral influenza with the mnemonic Having Flu Symptoms Can Make Moaning Children A Nightmare: Headache, Fever, Sore throat, Chills, Myalgias, Malaise, Cough, Anorexia, Nasal congestion.
Case 2-2: “I have a sore throat and can't eat” A 19-year-old man presents to the emergency department with a 5-day history of a worsening sore throat. He describes an increasing difficulty with eating because of throat pain and having a “lump in his throat.” He feels the pain becomes worse when trying to open his mouth. He says that he has had chills intermittently over the past few days and measured his temperature at home at 102°F. He denies any nasal congestion, nausea, vomiting, or diarrhea. He denies any past medical history or substance use. On examination, he is a moderately sick-appearing individual with a mildly hoarse voice. Examination of his oropharynx finds significant erythema with some streaky whitish material in the posterior pharynx. His pharynx appears slightly asymmetrical with apparent uvular deviation to the right side and a focal area of swelling near the left palatine tonsil. He has palpable lymphadenopathy in his cervical nodes. Auscultation finds mildly rhonchial breath sounds and normal heart sounds. The following vital signs are measured: T: 102.7°F, HR: 90 bpm, BP: 115/75 mm Hg, RR: 20 breaths/min
Differential Diagnosis Viral pharyngitis, bacterial pharyngitis/tonsillitis, peritonsillar abscess, influenza
Laboratory Data and Other Study Results CBC: WBC: 13.0, Hgb: 14.0, Plt: 280 Rapid streptococcal antigen test: positive
Throat swab culture: positive for group A streptococcus (results reported after initiation of therapy)
Diagnosis Peritonsillar abscess P.50
Treatment Administered Urgent incision and drainage (I + D) of the peritonsillar abscess was performed; intraoperative cultures were sent to determine the species of bacteria Intravenous (IV) ampicillin/sulbactam was initiated
Follow-up The patient had a significant improvement in his pain after the I + D His symptoms gradually improved after the initiation of treatment Group A streptococcus was confirmed in operative cultures with a sensitivity to ampicillin A peripheral indwelling catheter was placed that allowed the patient to return home to complete a 14-day course of ampicillin/sulbactam The patient was scheduled for elective tonsillectomy
Steps to the Diagnosis Peritonsillar abscess Progression of bacterial tonsillitis causing the formation of an abscess in the peritonsillar or retropharyngeal spaces History: sore throat, difficulty opening mouth (i.e., trismus), hoarseness, chills, drooling, inadequate treatment of a recent case of bacterial pharyngitis Physical examination: significantly inflamed pharynx, peritonsillar or retropharyngeal mass, uvular deviation, tonsillar exudates Tests: increased WBC, positive rapid streptococcal antigen, positive throat culture, positive operative abscess fluid culture (used to confirm pathogen identity); ultrasound (US) may be helpful in locating abscess and measuring its size Treatment: IV β-lactam or cephalosporin antibiotics Urgent I + D is performed unless the abscess is extremely small (i.e., microabscess) Planned tonsillectomy is recommended to prevent recurrence Outcomes: Respiratory complications and systemic infection can be avoided with prompt drainage Long-term risks are similar to bacterial pharyngitis (e.g., rheumatic heart disease, glomerulonephritis) The recurrence risk argues in favor of tonsillectomy
Clues to the diagnosis: History: worsening sore throat, trismus, feeling of lump in throat Physical: pharyngeal erythema and purulent exudates, tonsillar mass, uvular deviation, fever, hoarse voice, lymphadenopathy Tests: positive streptococcal screen and culture
With a clinical diagnosis of peritonsillar abscess, proceed to operative treatment and do not wait for culture results to be reported. Viral pharyngitis More thorough discussion in prior case Why eliminated from differential: the signs are consistent with a bacterial infection (e.g., exudates, high fever), and the presence of an abscess eliminates a viral cause from the differential Bacterial pharyngitis/tonsillitis More thorough discussion in prior case Why eliminated from differential: a cellulitic pharyngeal process will precede abscess formation, but the presence of trismus and uvular deviation are highly suggestive of an actual abscess Viral influenza More thorough discussion in prior case Why eliminated from differential: the signs and symptoms seen in the case are not systemic and are limited to the pharynx; abscess formation is not seen in influenza P.51
Case 2-3: “I have a cold and a bad toothache” A 35-year-old woman presents to her PCP's office with 10 days of rhinorrhea and a progressive toothache in the area of both of her upper incisors (with the right side being worse than the left). She feels general malaise and has not taken her temperature. She thought that she was getting better earlier in the week, but her symptoms have been worsening over the past few days. Over the past 3 days she began to notice dark brown discharge when blowing her nose and became concerned. Some of her symptoms have improved after taking a cold medicine containing acetaminophen. She admits to having had “colds” previously, but none have featured this type of toothache pain. She denies any medical problems, and the only medicine she uses is an oral contraceptive. She drinks alcohol and smokes tobacco socially but not on a regular basis. On examination, she is notable for mild tenderness along her “cheek bones” and mild cervical lymphadenopathy. Her pharynx is nonerythematous with no exudates. Auscultation detects minimally rhonchial breathing and normal heart sounds. Her abdominal examination is normal. The following vital signs are measured: T: 101.2°F, HR: 80 bpm, BP: 113/85 mm Hg, RR: 20 breaths/min
Differential Diagnosis Dental abscess, viral rhinitis, acute sinusitis, chronic sinusitis
Laboratory Data and Other Study Results CBC: WBC: 11.0, Hgb: 13.0, Plt: 315
Cranial x-ray: questionable opacification in right maxillary sinus
Diagnosis Acute sinusitis, most likely bacterial
Treatment A 2-week prescription of oral amoxicillin was provided to the patient Supportive therapy was continued to help improve the patient's symptoms
Follow-up The patient experienced improvement and resolution of her symptoms and fever over the next few days
Patients with asthma, allergic rhinitis, recent barotrauma, or prolonged nasogastric tube placement are at higher risk for developing sinusitis.
Steps to the Diagnosis Acute sinusitis Sinus infection (most commonly in maxillary sinuses) due to viral infection, Streptococcus pneumoniae, Haemophilus influenzae, or Moraxella catarrhalis The bacterial variant is much less common than viral sinusitis but most frequently occurs as a complication of the latter History: pain over the involved sinuses (one side usually worse than other), thick or purulent nasal discharge (more common with bacterial infection), foul-smelling nasal discharge, referred toothache pain; symptoms lasting a week or longer with a brief period of improvement followed by worsening are more consistent with a bacterial infection Physical examination: pain on palpation of the involved sinuses, purulent drainage between nasal turbinates on otoscopic examination, illumination test (i.e., light held close to frontal or maxillary sinuses) may detect congestion but is unreliable, possible fever P.52
Figure 2-3 Maxillofacial computed tomography (coronal view) demonstrating complete opacification of the left ethmoid sinuses and congestion of the left maxillary sinus consistent with acute sinusitis.
Tests: Usually a clinical diagnosis, so imaging is not recommended unless there is difficulty in making the diagnosis X-ray may show opacification of the sinuses (not highly sensitive) Computed tomography (CT) will detect opacification of the involved sinuses and is superior to x-ray (Figure 2-3) Sinus aspiration and culture is the definitive test but is rarely performed Treatment: supportive care; a 2-week course of either β-lactam antibiotics, sulfonamide, or erythromycin is indicated if a bacterial cause is suspected Outcomes: Viral disease will self-resolve Bacterial disease frequently resolves with an adequate antibiotic course Subsequent meningitis is a rare complication Clues to the diagnosis: History: rhinorrhea and toothache pain that initially improved then worsened and has lasted 10 days, dark nasal discharge Physical: sinus tenderness, lymphadenopathy, low-grade fever Tests: appearance of cranial x-ray
Sinusitis is a common culprit for a persistent fever for which an infectious work-up has found no source. Perform a sinus CT to look for sinus opacification if another source has not been detected.
Diabetic patients have a higher incidence of chronic sinusitis due to mucormycosis. Chronic sinusitis Clinical sinusitis lasting >3 months and most commonly related to sinus obstruction or anaerobic infection History: similar to that for acute sinusitis but symptoms are frequently milder and last >3 months Physical examination: similar to that for acute sinusitis; deviated septum or sinus obstruction may be apparent Tests: CT is useful to image the sinuses that are likely the source of symptoms; nasal endoscopy may be needed to locate obstructions Treatment: Antibiotic therapy for 6 to 12 weeks (may include quinolones in regimen) Surgical correction of an obstruction may be required for a definitive cure Prophylactic treatment of allergic rhinitis may help reduce the risk of occurrence Outcomes: acute-on-chronic sinusitis is the most frequent complication; cases resolve with adequate antibiotic treatment and removal of obstructions that predispose the patient to recurrences Why eliminated from differential: the symptoms in the case were not present for a sufficient time to consider the condition to be a chronic process P.53 Dental abscess The proliferation of bacteria colonized on teeth resulting in extension into gingival tissue or tooth pulp and the development of an abscess Commonly associated with poor dental hygiene History: significant pain in an affected tooth that may be exacerbated by changes in temperature, general malaise Physical examination: pain on palpation of the involved tooth, possible purulent drainage from around tooth, fever Tests: usually a clinical diagnosis; dental x-rays may show the degree of tooth destruction Treatment: debridement of infected tissue (frequently includes tooth extraction); β-lactamase resistant penicillin or third-generation cephalosporin with or without metronidazole Outcomes: Adequate surgical debridement and antibiotics are usually successful to achieve a cure Root canal is usually required in an affected tooth that is not extracted Untreated abscesses may result in the further local extension of infection Why eliminated from differential: upper respiratory symptoms (e.g., rhinorrhea) are not typical of dental abscesses Viral rhinitis More thorough discussion in prior case Why eliminated from differential: toothache pain and purulent nasal discharge are more consistent with
sinusitis than this diagnosis
Case 2-4: “Mom is having problems breathing” A 75-year-old woman is brought to her PCP's office by her daughter who feels that she has been having worsening shortness of breath over the course of the past week. The patient has a significant smoking history, but her respiratory condition has deteriorated rapidly in the afore-mentioned time. The patient reports having frequent chills for several days, and when her daughter took her temperature the morning before the office visit, it was found to be 102.1°F. The patient notes that she has been coughing up copious amounts of thick mucuslike material and has pain on the right side of her chest when coughing. She denies dizziness, syncope, or palpitations. She has a past medical history of uterine fibroids for which she had a hysterectomy 5 years ago. She takes a daily aspirin (ASA), a multivitamin, alendronate, and a calcium supplement. She has smoked close to a pack of cigarettes per day for the past 50 years. On examination, she appears to be an elderly female in mild respiratory distress. She is found to have decreased breath sounds in the base of her right lung field and dullness to percussion in the same region. Her breathing is tachypneic. Auscultation of her heart detects no abnormal sounds and a regular rate. Her abdomen is nontender with no masses. Her neurologic examination is normal. The following vital signs are measured: T: 101.5°F, HR: 90 bpm, BP: 125/90 mm Hg, RR: 26 breaths/min
Differential Diagnosis Pneumonia (viral/bacterial), acute bronchitis, viral rhinitis, chronic obstructive pulmonary disease, pulmonary edema, myocardial ischemia
Laboratory Data and Other Study Results Pulse oximetry: 89% on room air, 96% on 4L O2 via nasal cannula CBC: WBC: 17.3, Hgb: 12.3, Plt: 250 P.54 Chest x-ray (CXR): consolidation of right lower lobe; no masses or lymphadenopathy; small right-sided plural effusion at the base of the lung Electrocardiogram (ECG): regular rate (90 bpm) and rhythm; no abnormal wave morphology Sputum culture: Gram stain demonstrates Gram-positive cocci in pairs; culture pending
Diagnosis Bacterial pneumonia, most likely pneumococcal
Treatment Administered The patient was admitted to the hospital for respiratory care (e.g., oxygen, pulse oximetry, respiratory therapy), initiation of azithromycin and IV penicillin G, and observation
Follow-up The patient began to have gradual improvement in her respiratory function and overall condition following the initiation of treatment Streptococcus pneumoniae was isolated from the cultures and found to be sensitive to penicillin G; IV penicillin G was continued during her admission The patient was eventually able to return home following symptomatic recovery on oral amoxicillin prescribed for
a total of 14 days of antibiotic coverage
Gram-positive bacteria typically cause community acquired pneumonia; Gram-negative bacteria typically cause nosocomial pneumonia.
Steps to the Diagnosis Pneumonia Infection of the bronchoalveolar tree by host respiratory bacteria (typical pneumonia) or bacteria, viruses, or fungi from the surrounding environment (atypical pneumonia) (Table 2-4) Common pathogens vary with patient age (Table 2-5) History: productive or nonproductive cough, dyspnea, chills, night sweats, pleuritic chest pain Physical examination: tachypnea, decreased breath sounds, rales, wheezing, dullness to percussion, egophony (i.e., change in resonated sound quality over affected region), tactile fremitus Tests: Increased serum WBCs (less notable for viral or fungal pneumonia) Positive sputum culture; blood cultures may be positive for respiratory pathogens (Color Figure 2-1) Pulse oximetry will show decreased O2 saturation CXR demonstrates infiltrates or lobar consolidation and a possible effusion; similar findings are seen on chest CT (Figure 2-4) Treatment: Supportive care (only treatment necessary for viral pneumonia) Appropriate antibiotics for bacterial or fungal cases; β-lactams and macrolides are frequently used for empiric therapy until sensitivities are determined The decision to treat as an outpatient or to admit for inpatient care is based on multiple patient factors Younger, healthy patients with uncomplicated cases are more amenable to outpatient treatment Age >65 years, multiple comorbidities (e.g., respiratory disease, diabetes mellitus, renal failure, cardiac disease, alcoholism, prior stroke, etc.), significant test abnormalities (e.g., significantly abnormal WBC, anemia, hypoxemia, acidosis), multilobar involvement on x-ray, or signs of sepsis are findings more consistent with a need for inpatient care P.55
Table 2-4 Overview of Etiologies of Pneumonia
Pathogen
V iral Pneumonia
Patients A ffec ted
Charac teristic Symptoms
Treatment
Viral (e.g., influenza,
Most c ommon pneumonia in
Classic symptoms 1;
Self-limited; amantadine
parainfluenza, adenovirus,
c hildren; common in adults
nonproduc tive c ough
may be used for influenza A virus
cytomegalovirus, respiratory syncytial virus)
Typic al Bac terial Pneumonia
Streptococcus pneumoniae
Haemophilus influenzae
Staphylococcus aureus
β-lactams, macrolides
Most c ommon pneumonia in adults;
Classic symptoms; high
higher risk of infection in sickle cell
fevers, pleuritic pain,
patients
produc tive c ough
COPD patients; higher risk of
Classic symptoms;
infection in sickle cell patients
slow er onset
Nosocomial pneumonia,
Classic symptoms;
immunocompromised patients
abscess formation
Alcoholics, patients with high risk of
“Currant-jelly”
Both cephalosporins and
aspiration, patients staying in the
sputum; classic
aminoglycosides (e.g.,
hospital for extended amounts of
symptoms
gentamicin, tobramycin)
Less severe symptoms;
Macrolides (e.g.,
possible rash; positive
azithromycin,
c old-agglutinin test
clarithromycin,
β-lactams, TMP-SMX
β-lactams
Atypic al Bac terial Pneumonia
Klebsiella pneumoniae
time, sickle cell patients
Mycoplasma pneumoniae
Y oung adults
erythromycin)
Pseudomonas aeruginosa
Legionella pneumophila
Chronically ill and
Classic symptoms; rapid
Fluoroquinolones (e.g.,
immunocompromised patients,
onset
ciprofloxacin),
patients w ith c ystic fibrosis,
aminoglycosides, third-
nosocomial pneumonia
generation cephalosporins
Associated with aerosolized w ater
Slow onset of classic
Macrolides,
(e.g., air conditioners)
symptoms; nausea,
fluoroquinolones
diarrhea, confusion, or ataxia
Chlamydia pneumoniae
More common in very young and
Slow onset of classic
elderly
symptoms; frequent
Doxycycline, macrolides
sinusitis
Group B Streptococcus
Neonates and infants
Respiratory distress,
β-lactams
lethargy
Enterobacter sp.
Nosocomial pneumonia, elderly
Classical symptoms
TMP-SMX
Travelers to southw est U.S.
Less severe symptoms;
Antifungal agents
(coccidioidomycosis), c aves
subacute disease for
(amphotericin B,
(histoplasmosis), or Central America
initial history
ketoconazole)
patients
Fungal Pneumonia
Fungi
(blastomycosis)
Pneumocystis carinii
Immunocompromised patients (e.g.,
Slow onset of classic
(fungi-like)
AIDS)
symptoms; GI symptoms
TMP-SMX
AIDS, acquired immune deficiency syndrome; COPD, chronic obstructive pulmonary disease; GI, gastrointestinal; TMP-SMX, trimethoprimsulfamethoxazole. 1
Classic symptoms are productive or nonproductive cough, dyspnea, chills, night sweats, pleuritic chest pain.
Outcomes: risk stratification (considering above factors) is used to determine the need for inpatient treatment; patients able to be treated as outpatients have less morbidities and mortalities than those requiring admission Clues to the diagnosis: History: 1-week history of dyspnea, pleuritic chest pain, productive cough, chills Physical: unilateral decreased breath sounds, dullness to percussion, tachypnea, fever Tests: increased WBCs, CXR appearance, positive sputum Gram stain and culture P.56
Table 2-5 Most Common Etiologies of Pneumonia by Age Group
Age Group
Neonatal
Community Ac quired
Nosoc omial
Group B streptococcus
Staphylococcus aureus
Escherichia coli
Group B streptococcus
Klebsiella pneumoniae
Klebsiella pneumoniae
Staphylococcus aureus
Respiratory syncytial virus
Streptococcus pneumoniae
Infant to 5 years of age
Respiratory syncytial virus
Staphylococcus aureus
Streptococcus pneumoniae
Klebsiella pneumoniae
Staphylococcus aureus
Respiratory syncytial virus
Mycoplasma pneumoniae
Chlamydia pneumoniae
5 to 20 years of age
Streptococcus pneumoniae
Staphylococcus aureus
Mycoplasma pneumoniae
Klebsiella pneumoniae
Chlamydia pneumoniae
Respiratory syncytial virus
Respiratory syncytial virus
20 to 40 years of age
Mycoplasma pneumoniae
Streptococcus pneumoniae
Streptococcus pneumoniae
Viruses (various)
Viruses (various)
Staphylococcus aureus
Chlamydia pneumoniae
40 to 60 years of age
Streptococcus pneumoniae
Streptococcus pneumoniae
Mycoplasma pneumoniae
Haemophilus influenzae
Staphylococcus aureus
Enterobacter species
60 + years of age
Streptococcus pneumoniae
Streptococcus pneumoniae
Haemophilus influenzae
Haemophilus influenzae
Chlamydia pneumoniae
Staphylococcus aureus
Staphylococcus aureus
Enterobacter species
Respiratory syncytial virus
(Modified from Mehta S, Milder EA, Mirachi AJ, Milder E. Step-up: a high-yield, systems-based review for the USMLE step 1. 2nd ed. Philadelphia: Lippincott Williams & Wilkins, 2003. Used with permission of Lippincott Williams & Wilkins.)
Acute bronchitis Inflammation of the trachea and bronchi by local extension of an URI (usually viral) or from exposure to inhaled irritants The majority of cases are of a viral nature, but smokers and patients with other respiratory conditions are at risk for a bacterial infection History: sore throat, productive cough Physical examination: fever, wheezing, rhonchi
Tests: WBCs are frequently normal CXR does not demonstrate lobar consolidation but may show some mild congestion Sputum culture is usually only performed in persistent cases and is negative except for the uncommon cases with a bacterial cause (typically Mycoplasma pneumoniae or Chlamydia pneumoniae) Treatment: supportive care only if due to a viral cause; antibiotics are given for bacterial cases Outcomes: typically self-limited; smokers, elderly patients, or patients with preexisting lung diseases may be at risk for a superimposed secondary respiratory infection Why eliminated from differential: the examination findings and x-ray appearance are much more consistent with pneumonia; the positive sputum culture is indicative of pneumonia
Smokers are at risk for developing acute bronchitis due to infection by Streptococcus pneumoniae and Haemophilus influenzae. P.57
Figure 2-4 Radiograph demonstrating right upper lobe pneumonia. Note the consolidation of the right upper lobe with an associated volume loss. The division between the right upper and middle and lower lobes is well demarcated by fluid (white arrows).
Viral rhinitis More thorough discussion in prior case
Why eliminated from differential: the presentation, examination, and tests of the case are indicative of a lower respiratory infection Chronic obstructive pulmonary disease More thorough discussion in later case Why eliminated from differential: the patient would be expected to have more chronic dyspnea and cough for this diagnosis; CXR findings are consistent with an acute lobar process Pulmonary edema More thorough discussion in later case Why eliminated from differential: the CXR in the case demonstrates a lobar process and not diffuse infiltrates as seen in pulmonary edema; the patient also does not have a history of cardiac disease Myocardial ischemia More thorough discussion in Chapter 1 Why eliminated from differential: chest pain in the case is more pleuritic in nature than cardiac and the normal ECG decreases the concern for a cardiac cause; cardiac enzymes could be ordered to further rule out cardiac processes if concern still exists
Case 2-5: “My cousin has had a cough for a very long time” A 57-year-old woman is brought to an internist's office by her cousin because of a persistent cough. The patient immigrated from Southeast Asia several months ago, and her cousin, who grew up in the United States, reports that she has had a cough ever since she arrived. The patient says that she has had this cough for many months and that while it initially occurred in the morning it now occurs more often. The patient notes that the cough occasionally is productive of yellowish sputum that is also sometimes rusty in appearance. She occasionally P.58 wakes in the middle of the night and finds herself wet from sweating. She feels that she tires easily but denies any fevers or shortness or breath. She says that she is about 10 pounds lighter than she was at this time last year but has not tried to intentionally lose weight. She has never been diagnosed with any medical problems. She does not use any medications or illicit substances. On examination, she does not appear to be in any distress but coughs occasionally. She does not have any significant lymphadenopathy. On auscultation, she has slightly decreased breath sounds in the left upper lung field but no rales or rhonchi. Her heart sounds are normal. Her abdomen is nontender with no masses. She is notable for clubbing of her fingernails. Her neurologic examination is normal. The following vital signs are measured: T: 99.6°F, HR: 73 bpm, BP: 135/85 mm Hg, RR: 18 breaths/min
Differential Diagnosis COPD, lung cancer, gastroesophageal reflux, asthma, chronic lower respiratory infection, avian flu
Laboratory Data and Other Study Results CBC: WBC: 8.3, Hgb: 12.2, Plt: 190 CXR: infiltrates in the left upper lung field and apex; some small pockets of air-fluid levels Following these findings, the following additional studies are performed: PPD: 20 mm induration
Sputum culture: moderate acid-fact bacilli seen on initial Gram stain
Diagnosis Pulmonary tuberculosis, early to intermediate stage of reactivation
Treatment Administered The patient was admitted to the hospital and placed in respiratory isolation Multidrug chemotherapy was initiated using isoniazid (INH), rifampin, pyrazinamide, and ethambutol Close contacts were given purified protein derivatives (PPDs) to rule out infection and were given 6 months prophylactic INH The case was reported to local and state health agencies
Follow-up The patient was able to be discharged to home after 4 weeks of treatment when her symptoms improved and her sputum no longer displayed acid-fast bacilli After the initial 8 weeks of therapy, the regimen was changed to INH and rifampin only for a total of 6 months of therapy Human immunodefiency virus (HIV) testing was performed to rule out infection in the patient Repeat sputum testing performed at monthly intervals to confirm adequate treatment indicated successful therapy
Steps to the Diagnosis Tuberculosis (TB) Pulmonary infection caused by Mycobacterium tuberculosis There has been a slow increase in the incidence of cases in the United States in the past 20 years due to the HIV epidemic P.59 Begins as primary infection and typically enters an inactive state Untreated infection may enter reactivation (majority of active cases) and progress to involve a wider region of the lungs or extrapulmonary sites (i.e., miliary TB) Risk factors: immunocompromised patients (e.g., HIV), alcoholism, preexisting lung disease, diabetes mellitus, homelessness, malnutrition, crowded living conditions, close contact with infected patients (e.g., relatives, health care workers), immigrants (much higher incidence in developing nations) History: Primary: rarely chest pain or cough Reactivated: productive (more common) or nonproductive cough, fatigue, night sweats, weight loss, and possible chest pain, dyspnea, or hemoptysis Physical examination: Primary: occasional fever Reactivated: decreased breath sounds or rales, fever, clubbing; signs become more prominent as disease
progresses Tests: WBCs are typically normal PPD test is useful to screen for exposure (Table 2-6) Acid-fast bacilli are seen on a sputum acid-fast stain; culture may take several weeks and is rarely useful in making the diagnosis (Color Figure 2-2) CXR should be performed in any patient with a positive PPD and will show lower lobe calcified granulomas (i.e., Ghon complex) or infiltrates in primary or inactive disease and apical infiltrates in reactivated disease (Figure 2-5) Treatment: Patients requiring admission for respiratory symptoms or current inpatients require respiratory isolation All cases must be reported to local and state health agencies For patients with diagnosed infection, multiple drug regimens exist, but all consist of an initial period of multidrug treatment (i.e., INH, rifampin, pyrazinamide, and ethambutol) followed by only INH and rifampin for a total of 6 to 9 months High-risk individuals (e.g., close contacts of infected individual, immunocompromised patients, indigents, patients <35 years old, intravenous drug users) with an asymptomatic positive PPD should be given a 6-month course of prophylactic INH Patients should have monthly sputum acid-fast tests to confirm adequate treatment Relapse is treated by introducing multiple drugs not utilized in the initial regimen Patients who have received the BCG vaccine should be treated in the same manner as unvaccinated patients if they have been vaccinated more than 1 year prior to presentation or if they have a reactive PPD >10 mm in diameter Outcomes: Up to 90% of patients respond well to treatment and have a documented cure Recurrent cases are most commonly due to new infection Untreated cases may be complicated by TB meningitis, bone involvement (i.e., Pott disease), or widespread dissemination to multiple organ systems (i.e., miliary TB)
Table 2-6 Criteria Used to Determine Positive Purified Protein Derivative Skin Test for Tuberculosis
Size of Induration 1
W hen Considered Positive
5 mm
HIV-positive, close contact with TB-infected patient, signs of TB seen on CXR
10 mm
Homeless patients, immigrants from developing nations, IVDA patients, chronically ill patients, healthcare workers, patients with recent incarceration
15 mm
Always considered positive
CXR, chest x-ray; HIV, human immunodeficiency virus; IVDA, intravenous drug abuse; TB, tuberculosis. 1
Induration is considered the firm cutaneous region and not the region of erythema.
P.60
Figure 2-5 Chest radiograph demonstrating right lung apical infiltrates and cavitation consistent with reactivated pulmonary tuberculosis.
Clues to the diagnosis: History: chronic productive cough with possible hemoptysis, recent immigration from Southeast Asia, weight loss, night sweats Physical: decreased upper lobe breath sounds, digital clubbing Tests: positive PPD, CXR appearance, sputum Gram stain
Although patients born in other countries may have received the BCG vaccine and may have a false-positive PPD if they have received the vaccine within the past few years, they should be treated as if they had not been vaccinated unless they have received the vaccine within the past year.
Immunocompromised patients should be given an anergy test (e.g., subcutaneous
Candida injection) in addition to a PPD to check for an appropriate immune response in TB infection.
Remember the multidrug regimen for TB with the acronym RIPE: Rifampin, INH, Pyrazinamide, Ethambutol.
Vitamin B6 is administered during INH therapy to reduce the risk of drug-related peripheral neuritis. Chronic obstructive pulmonary disease More thorough discussion in later case Why eliminated from differential: although initially a distinct possibility for a diagnosis, the occurrence of night sweats and fever, positive PPD, characteristic appearance on the CXR, and positive acid-fast sputum stain are confirmatory of TB Lung cancer More thorough discussion in later case Why eliminated from differential: should be definitely considered for any case of chronic cough; the sputum findings are confirmatory of TB, but in the absence of this finding, an extensive work-up would be performed to rule out neoplasm Gastroesophageal reflux More thorough discussion in Chapter 3 Why eliminated from differential: the absence of a history of heartburn and the presence of night sweats, weight loss, and decreased breath sounds rule out this diagnosis Asthma More thorough discussion in later case Why eliminated from differential: the presence of night sweats, weight loss, and decreased breath sounds are not consistent with this diagnosis, and the characteristic appearance on the CXR and positive acid-fast sputum stain are confirmatory of TB Avian flu Influenza virus affecting birds that has shown the capacity to undergo genetic reassortment and become transmissible to humans At least one of the severe influenza pandemics in history has been due to transmission of a strain of avian flu to humans History: arthralgias, myalgias, sore throat, productive or nonproductive cough, nasal congestion, nausea, vomiting, and diarrhea Physical examination: fever, lymphadenopathy, possible conjunctivitis P.61 Tests: CXR may show either patchy or diffuse infiltrates; serologic testing confirms the viral strain but is not useful in the acute setting
Treatment: Supportive care A combination of amantadine or rimantadine with a neuraminidase inhibitor (e.g., oseltamivir, zanamivir) is used as an antiviral regimen Oseltamivir may be given prophylactically to those exposed to an infected individual Outcomes: significantly higher mortality rate than other strains of influenza (due to severe respiratory infection or encephalopathy) Why eliminated from differential: the time course of symptoms in the case is much longer than that expected for avian flu; as above, the CXR appearance and positive acid-fast stain confirm the true diagnosis
Case 2-6: “This intensive care unit (ICU) patient is having more problems breathing” A 38-year-old man is an inpatient in the ICU due to pneumonia and sepsis associated with end-stage acquired immune deficiency syndrome (AIDS). Overnight the patient becomes increasingly dyspneic and tachypneic. During this time his O2 requirement has increased, and he is currently on 6L O2 via nasal cannula. His nurse denies any sudden events in which his condition has worsened and says that this deterioration has been steady but gradual over the past several hours. The critical care team evaluates him in hopes of stopping this downward spiral. On examination, he is stuporous and minimally interactive. He does not have palpable lymphadenopathy. On auscultation he is found to be significantly tachypneic with considerable rales throughout both lung fields. He is tachycardic but lacks any abnormal heart sounds. His abdomen is soft, and no masses are detectable. He is unable to follow commands but withdraws from painful stimuli. He is found to be mildly cyanotic. He has palpable pulses and 2-second capillary refill. The following vital signs are measured: T: 97.2°F, HR: 110 bpm, BP: 106/70 mm Hg, RR: 26 breaths/min
Differential Diagnosis Acute respiratory distress syndrome, pneumonia, pulmonary embolism, septic shock, cardiac pulmonary edema
Laboratory Data and Other Study Results ABG (6L O2): pH: 7.48, pO2: 68 mm Hg, pCO2: 23 mm Hg, Bicarb: 20 mEq/L, O2 sat: 81% CBC: WBC: 2.1, Hgb: 10.1, Plt: 130 Chem7 (7-electrolyte chemistry panel): sodium (Na): 145 mEq/L, potassium (K): 4.5 mEq/L, chloride (Cl): 105 mEq/L, CO2: 20 mEq/L, blood urea nitrogen (BUN): 30 mg/dL, creatine (Cr): 1.1 mg/dL, glucose (Glu): 85 mg/dL Coagulation panel (Coags): protime (PT): 14 s, international normalized ratio (INR): 1.1, partial thromboplastin time (PTT): 40 s, fibrinogen: 190 mg/dL, D-dimer: 0.6 µg/mL B-type natriuretic peptide (BNP): 90 pg/mL ECG: sinus tachycardia; no abnormal wave morphology CXR: diffuse fluffy infiltrates throughout both lungs; no pleural effusions Swan-Ganz catheterization: wedge pressure 12 mm Hg
Diagnosis Acute respiratory distress syndrome, secondary to sepsis P.62
Treatment Administered The patient required intubation and ventilation to improve his respiratory status Antibiotic therapy was continued to treat the underlying sepsis
Follow-up The patient initially had minimal improvement in his arterial blood gases (ABGs) following intubation Over the next 48 hours the patient developed worsening coagulopathy consistent with disseminated intravascular coagulation (DIC), and his respiratory status began to worsen despite the adjustment of ventilatory parameters Patient began to exhibit signs of multiorgan dysfunction and died 3 days after the initial decompensation
Common causes of ARDS can be remembered with the mnemonic ARDS: Aspiration/Acute pancreatitis/Air or Amniotic embolism, Radiation, Drug overdose/Diffuse lung disease/DIC/Drowning, Shock/Sepsis/Smoke inhalation.
Steps to the Diagnosis Acute respiratory distress syndrome (ARDS) Acute respiratory failure secondary to sepsis, trauma, aspiration, near drowning, drug overdose, shock, or lung infection that is characterized by refractory hypoxemia, decreased lung compliance, and pulmonary edema History: acute dyspnea and general decompensation in the setting of an underlying illness (as listed above), possible cough or chest pain Physical examination: cyanosis, tachypnea, wheezing, rales, rhonchi Tests: ABG demonstrates respiratory alkalosis with low pO2 (due to impaired alveolar to arterial gas transfer) and low CO2 (due to hyperventilation) Other lab tests should reflect the underlying pathology (e.g., CBC, toxin screens) CXR will show bilateral infiltrates consistent with pulmonary edema Swan-Ganz catheterization typically shows wedge pressure <18 mm Hg Pao2/Fio2 ratio will be <200 during mechanical ventilation Treatment: Treatment in the ICU and intubation/mechanical ventilation are frequently required Ventilation should utilize PEEP, increased inspiratory times, and modulation of Fio 2 to aim for a goal O2 saturation >90% The underlying cause must be treated appropriately Extracorporeal membrane oxygenation (ECMO) may be required in severe cases to help maintain an adequate O2 supply to tissues Outcomes: despite improvements in critical care management mortality remains high (40%–50%); surviving patients frequently suffer lifelong pulmonary impairment to various degrees
Throughout the differential diagnosis, differentiating ARDS from other diagnoses is more of an issue of semantics than clinical differentiation because ARDS incorporates characteristics of several other pulmonary processes; the important keys are realizing that ARDS (and not a single pulmonary process) is developing in a susceptible patient group and that these patients require aggressive intensive treatment Clues to the diagnosis: History: existing pneumonia and sepsis, worsening respiratory function, increasing oxygen requirements Physical: cyanosis, tachypnea, diffuse rales Tests: diffuse infiltrates on CXR, hypoxemia and hypocapnia on ABG Pneumonia More thorough discussion in prior case Why eliminated from differential: although underlying pneumonia is a large component leading to the development of ARDS in this patient, the patient's hypoxemia that fails to improve despite ventilation indicates the progression to fulminant ARDS and that pneumonia is not the solitary cause of his deterioration P.63 Cardiac pulmonary edema Increased fluid in the lungs caused by increased pulmonary venous pressure and hydrostatic leak of fluid from vessels Most commonly occurs due to left-sided heart failure, myocardial infarction (MI), valvular disease, arrhythmias, or in association with ARDS History: dyspnea, orthopnea, paroxysmal nocturnal dyspnea Physical examination: tachycardia, frothy sputum, wheezing, rhonchi, rales, dullness to percussion, peripheral edema Tests: CXR shows fluid throughout lungs, cephalization of vessels (i.e., increased vascular markings in upper lung fields), and Kerley B lines (i.e., prominent horizontal interstitial markings in lower lung fields) (Figure 2-6) Treatment: treat underlying cause; diuretics, salt restriction, O2, morphine, vasodilators can decrease fluid overload, improve blood oxygenation, and decrease cardiac workload Outcomes: Correction of the underlying cause is the key to long-term resolution Patients with cardiac causes may have repeat exacerbations if optimal control of fluid status and cardiac function is not maintained Patients with pulmonary edema associated with ARDS must have the underlying condition treated to avoid continued pulmonary issues Why eliminated from differential: the underlying pneumonia and sepsis is more suggestive of ARDS due to these infectious causes; the normal range BNP and relatively benign ECG rule out sudden cardiac decompensation as an inciting factor Pulmonary embolism More thorough discussion in later case Why eliminated from differential: an increased D-dimer and a sudden onset of hypoxemia would be expected
for deep vein thrombosis or pulmonary embolism; if pneumonia/sepsis was not already present and the patient was stable enough to leave the ICU, spiral CT would be a useful study to rule out a suspected pulmonary embolism Septic shock More thorough discussion in Chapter 1 Why eliminated from differential: although sepsis is the underlying cause of ARDS in this patient, the patient had not yet entered septic shock at the time of P.64 initial decompensation (i.e., has mild hypotension and does not require pressors to maintain pressure); although this initial worsening is primarily a pulmonary process, the later development of DIC and the general deterioration of the patient leading to his demise is likely due in part to the eventual development of septic shock
Figure 2-6 Chest radiograph demonstrating diffuse pulmonary edema with generalized opacification that is worse near the hilar regions.
Case 2-7: “I'm short of breath and don't feel right” A 64-year-old man is admitted to the surgical floor following an uncomplicated total knee replacement. During his immediate postoperative stay he has inadequate pain control and is resistant to participating in his postoperative physical therapy. He complains of pain in his operative knee radiating to his thigh and calf. On the third postoperative day he experiences sudden shortness of breath and chest pain. He becomes extremely anxious with the onset of these symptoms and exclaims that he has never been through such an experience. He only has a history of HTN, for which he takes lisinopril, and no history of pulmonary disease. He drinks alcohol occasionally but used no other substances prior to his surgery. On examination, he is in moderate distress and is very anxious. He has no swelling in his neck or
lymphadenopathy. He is found to be tachypneic and tachycardic. Auscultation of his lungs and heart finds soft rales in his left lung field and a loud second heart sound. His abdomen is soft, nontender, and without masses. His operative leg is painful on palpation of the knee and calf. There is some moderate ecchymoses around his knee. It is swollen to a degree consistent with his recent surgery. All of his pulses are strong and uniform. The following vital signs are measured: T: 100.6°F, HR: 125 bpm, BP: 145/95 mm Hg, RR: 26 breaths/min
Differential Diagnosis Pulmonary embolism, myocardial ischemia, atelectasis, pneumonia, pulmonary hypertension
Laboratory Data and Other Study Results CBC: WBC: 8.4, Hgb: 12.5, Plt: 350 ABG (room air): pH: 7.45, pO2: 78 mm Hg, pCO2: 31 mm Hg, Bicarb: 20 mEq/L, O2 sat: 90% BNP: 45 pg/mL ECG: sinus tachycardia; T wave and ST segment changes without distinct pattern CXR: mild bibasilar infiltrates; no focal consolidation or effusions; no masses or lymphadenopathy Upon receipt of these results, the following tests are ordered: Cardiac enzymes: Creatine kinase (CK) 500 U/L, Creatine kinase myocardial component (CK-MB) 3.4 ng/mL, troponin-I 0.2 ng/mL Spiral chest CT: probable occlusion of arterial vessels supplying part of the left lower lobe
Diagnosis Pulmonary embolism, likely affecting part of the left lower lobe
Treatment Administered The patient was transferred to the ICU for closer observation Supplemental O2 was administered to keep O2 saturation above 92% Subcutaneous enoxaparin was administered every 12 hours started at 1 mg/kg dosing, and he was started on oral warfarin P.65
Follow-up The patient was able to return to the surgical floor after remaining stable overnight with no signs of further deterioration The patient was discharged to a rehabilitation hospital when he was able to be weaned from oxygen, his pain was adequately controlled, and he was able to participate fully in physical therapy The patient remained on enoxaparin therapy until his INR was >2.0; then he was kept on warfarin alone for 6 months without any further complication
Ninety-five percent of PEs arise from a deep vein thrombosis (DVT) in the lower extremity.
Risk factors for PE may be remembered by the seven H's: Heredity (genetic hypercoagulability), History (prior PE or DVT), Hypomobility (fracture, surgery, obesity, travel), Hypovolemia (dehydration), Hypercoagulability (cancer, smoking), Hormones (pregnancy, oral contraceptive pills [OCPs]), Hyperhomocysteinemia.
Common causes of dyspnea may be remembered by the mnemonic AAAAPPPP: Airway obstruction, Angina, Anxiety, Asthma, Pneumonia, Pneumothorax, Pulmonary edema, Pulmonary embolism.
A positive or negative V/Q scan is diagnostic or rules out PE, but an equivocal result indicates a need for additional studies (e.g., CT with contrast, angiography) to provide a definite answer.
LMWH does not require monitoring of the PT or PTT.
Steps to the Diagnosis Pulmonary embolism Occlusion of the pulmonary vasculature by a dislodged thrombus An increased pulmonary artery pressure due to the occlusion increases the risks for right-sided heart failure and pulmonary infarction Risk factors: immobilization, cancer, prolonged travel, recent surgery, pregnancy, oral contraceptive use, hypercoagulability, obesity, fractures, prior DVT, or severe burns History: sudden dyspnea, pleuritic chest pain, nonproductive cough, possible syncope or hemoptysis, feeling of impending doom Physical examination: fever, tachypnea, tachycardia, cyanosis, loud S 2 heart sound, rales Tests: D-dimer is typically increased with any thrombotic or embolic process ABG is inconsistent as a diagnostic test but may show decreased O2 and CO2 and an increased A-a gradient ECG shows tachycardia and possible ST segment and T wave abnormalities CXR may be normal or may show a pleural effusion or wedge-shaped infarct Ventilation-perfusion scan (V/Q scan) may demonstrate areas of ventilation-perfusion mismatch and is a fairly specific study Spiral CT is a quick and sensitive study that may detect more proximal PEs and has become the standard test at hospitals with good CT experience (Figure 2-7) US is useful for detecting lower extremity DVTs and may be beneficial to making a diagnosis in cases with equivocal results in other studies Pulmonary angiography is the gold standard test but carries higher risks than other diagnostic studies and is used less commonly today Treatment:
Supplemental oxygen is administered to maximize O2 saturation IV fluids or pressors are used as needed for patients who develop hypotension Anticoagulate the patient with either low molecular weight heparin (LMWH, e.g., enoxaparin) or unfractionated heparin (IV infusion titrated for PTT 1.5 to 2.5 times normal) Patients treated with unfractionated heparin will need to be converted to either LMWH or warfarin (dosed to keep INR 2.0 to 3.0) prior to discharge Anticoagulation is continued 3 to 6 months An inferior vena cava filter may be placed in patients with contraindications for anticoagulation Thrombolysis may be considered for patients with a massive PE or those with no cardiac contraindications, recent trauma, or surgery Outcomes: as long as lung infarction does not occur, the body will gradually breakdown and reorganize the embolus; patients without cardiac arrhythmias or significant pulmonary compromise are more likely to have better outcomes Clues to the diagnosis: History: sudden dyspnea and chest pain, recent surgery, immobility, calf pain Physical: tachycardia, tachypnea, rales Tests: hypoxemia and hypocapnia on ABG, spiral CT results P.66
Figure 2-7 Chest computed tomography with contrast demonstrating multiple sites of pulmonary embolization. (A) Large emboli are present in both pulmonary arteries (black arrows) causing intraluminal filling defects. (B) Filling defects are also seen in the left lower lobe pulmonary artery and a right lower lobe segmental arteries indicating further emboli (white arrows).
Myocardial ischemia More thorough discussion in Chapter 1 Why eliminated from differential: the ECG is concerning for some type of cardiac processes, but the normal cardiac markers rule out acute myocardial ischemia, and the normal BNP rules out an exacerbation of heart failure
Pneumonia More thorough discussion in prior case Why eliminated from differential: the history of dyspnea and recent hospitalization do create concern for this diagnosis, but the benign appearance of the CXR, lack of fever, and normal WBC rule out this possibility
Atelectasis from airway obstruction tends to be more serious than that from surgery or anesthesia.
Atelectasis is frequently blamed for a postoperative fever, but the relationship is more likely coincidental than causal. Atelectasis Localized alveolar collapse that is common after surgery (particularly abdominal) and anesthesia; also may occur in asthmatics, following foreign body aspiration, or from mass effect (e.g., tumors, pulmonary lesions, lymphadenopathy) History: frequently asymptomatic, but possible pleuritic chest pain or dyspnea may occur Physical examination: decreased breath sounds, dullness to percussion Tests: CXR will show infiltrates in mild cases or lobar collapse in cases of obstruction Treatment: inspiratory spirometry, ambulation, and physical therapy are important prophylactic measures used in the hospital and postoperatively; bronchoscopy with suctioning or obstruction relief are required for more severe cases Outcomes: generally not clinically serious following surgery; prolonged atelectasis (beyond 72 hours) may predispose patients to developing pneumonia Why eliminated from differential: the bibasilar infiltrates seen on the CXR in this case are due to atelectasis, and they may confuse the diagnosis somewhat, but the sudden onset of hypoxemia, dyspnea, tachypnea, and tachycardia suggests a more severe process than what is seen on the CXR Pulmonary hypertension Increased pulmonary artery pressure due to valvular disease, PE, left-to-right cardiac shunts, congestive heart failure (CHF), chronic obstructive pulmonary disease, or idiopathic causes History: dyspnea, fatigue, deep chest pain, cough, syncope Physical examination: cyanosis, digital clubbing, jugular venous distension, loud S2, hepatomegaly P.67 Tests: ECG findings are consistent with right ventricular hypertrophy CXR shows large pulmonary artery and right ventricle Echocardiogram is useful for measuring pulmonary artery pressure noninvasively and detecting any valvular abnormalities Cardiac catheterization is the gold standard test but is riskier than noninvasive studies Treatment:
Treat underlying conditions to lessen pulmonary artery pressure Supplemental O2 is given if hypoxemia develops Vasodilators decrease pulmonary vascular resistance Anticoagulants may be indicated to reduce the risk of pulmonary thrombus formation Outcomes: prognosis worsens with development of right-sided heart failure; idiopathic form has a high mortality rate within a few years of diagnosis Why eliminated from differential: primary pulmonary hypertension is rare, and the condition generally develops secondary to another process; the clinical, laboratory, and radiographic findings suggestive of PE make it a much more likely diagnosis
Case 2-8: “I get short of breath sometimes” A 10-year-old boy is brought to his family practitioner by his mother because of a chronic cough and occasional dyspnea. The patient describes episodes of dyspnea that are accompanied by chest tightness that are not constant but last several minutes and occur once or twice a week. He feels that these episodes are worse during “allergy season.” His mother describes him as having had “allergy attacks” when he was younger, but the current symptoms have developed over the past few years. He is occasionally forced to take a break during exertional activity because he becomes short of breath. He denies heartburn or symptoms following eating. He has not taken any medications for this problem. He does not have a history of any medical disorders. His mother denies having any allergies besides allergic rhinitis in the fall and spring. His mother denies any similar symptoms in his parents, and he has no siblings. He denies tobacco use. On examination, he appears to be in no distress and is breathing easily. On auscultation of his lungs and heart he is notable for faint wheezing throughout both lung fields. He has normal heart sounds. His abdominal and neurologic examinations are normal. The following vital signs are measured: T: 98.6°F, HR: 72 bpm, BP: 115/80 mm Hg, RR: 14 breaths/min
Differential Diagnosis Asthma, gastroesophageal reflux disease, sarcoidosis, cystic fibrosis
Laboratory Data and Other Study Results Height and weight: 5﹐ 10", 170 lb, body mass index (BMI) 24.4 Peak expiratory flow rate (PEFR): 480 L/min CXR: clear lung fields; no masses, lymphadenopathy, effusions, or infiltrates Chem7: Na: 140 mEq/L, K: 4.0 mEq/L, Cl: 106 mEq/L, CO2: 30 mEq/L, BUN: 10 mg/dL, Cr: 0.7 mg/dL, Glu: 95 mg/dL Sweat test: Cl: 25 mEq/L, Na: 30 mEq/L
Diagnosis Asthma P.68
Treatment Administered The patient was prescribed a short-acting inhaled β2-agonist to be used during exacerbations The patient was educated regarding common exacerbating factors (e.g., allergens, respiratory irritants),
avoidance of precipitating factors, recognition of symptoms, and recognition of a worsening condition requiring emergent care
Follow-up The patient was successfully able to use an inhaler during the recurrence of symptoms with a reasonable relief of dyspnea; he noted inhaler use approximately five times per week The patient was started on a low-dose corticosteroid inhaler with a continued prescription for a short-acting inhaled β2-agonist inhaler for breakthrough dyspnea; control improved with minimal exacerbations Patient was able to be weaned to a short-acting inhaled β2-agonist inhaler only as needed over the course of 1 year
Asthma severity may be worse during childhood and improve with age.
Status asthmaticus is a prolonged, nonresponsive asthma attack that can be fatal and should be treated with aggressive bronchodilator therapy, corticosteroids, O2, and possible intubation.
Steps to the Diagnosis Asthma Reversible airway obstruction secondary to bronchial hyperactivity, acute airway inflammation, mucous plugging, and smooth muscle hypertrophy Characteristic exacerbations with sudden airway inflammation and bronchoconstriction may be triggered by allergens (e.g., dust, smoke, pollen, fumes), URIs, exercise, stress, β-antagonistic medicines, and rarely aspirin or sulfites PEFRs, clinical symptoms, and frequency of medication use are used to classify the disease as mild intermittent, mild persistent, moderate persistent, or severe Risk factors: family history of asthma, allergies, atopic dermatitis, low socioeconomic status History: cough, dyspnea, chest tightness with a worsening of symptoms during exacerbations Physical examination: tachypnea, tachycardia, wheezing, prolonged expiratory duration, decreased breath sounds, increased accessory respiratory muscle use, possible pulsus paradoxus, cyanosis Tests: Decreased O2 saturation on pulse oximetry Decreased PEFR; PFTs also will demonstrate changes consistent with obstructive disease including decreased FEV 1 (Table 2-2) and normal or increased DLco ABG less practical except during severe exacerbations (a normalized CO2 level signals an impending respiratory collapse and indicates need for aggressive treatment) Treatment: Treatment follows an algorithm depending on the classification and uses a combination of medications (Tables 2-7 and 2-8) Patients with gradually worsening symptoms will need to be advanced to the next higher treatment level Patients who achieve control of symptoms may have their medications tapered as needed
Patient education is important in achieving long-term control Outcomes: although some patients may have a gradual resolution of symptoms, others may have consistent or worsening disease that requires long-term therapy with possible hospitalization for severe exacerbations Clues to the diagnosis: History: chronic cough, dyspnea on exertion, chest tightness Physical: expiratory wheezing Tests: decreased PEFR P.69
Table 2-7 Commonly Used Medications for Treatment of Asthma
Medic ation
Mec hanism of Ac tion
Role
Rapid-acting β2-agonists (e.g.,
Bronchodilators that relax airway smooth
First-line therapy in mild intermittent cases and during
albuterol, pirbuterol, bitolterol)
muscle; have rapid onset of action
exacerbations
Regular use in patients with moderate persistent or severe asthma
Long-acting β2-agonists (e.g.,
Bronchodilators that relax airway smooth
salmeterol, formoterol, sustained-
muscle; have gradual onset and sustained
release albuterol)
activity
Inhaled corticosteroids (e.g.,
Decrease number and activity of cells
Mild persistent or worse cases; frequently combined with β2-agonist
beclomethasone, flunisolide)
involved with airway inflammation
use
Leukotriene inhibitors (e.g.,
Block activity or production of
Oral agents; adjunctive therapy in mild persistent or worse cases
zafirlukast, zileuton)
leukotrienes that are involved in inflammation and bronchospasm
Mast cell stabilizing agents (e.g.,
Stabilizes mast cells; anti-inflammatory
Not useful acutely; anti-inflammatory prophylaxis in mild persistent
cromolyn, nedocromil)
prophylaxis
cases
Theophylline
Bronchodilator
Former first-line therapy but now replaced by β2-agonists because of side effects and interactions with other drugs; may be useful as adjunct in mild persistent or worse cases
Anticholinergic agents (e.g.,
Blocks vagal-mediated smooth muscle
ipratropium)
contraction
Systemic steroids (e.g.,
Similar action to inhaled steroids; stronger
methylprednisolone, prednisone)
effect than inhaled preparation
Adjunctive therapy in moderate to severe cases
Adjunctive therapy in severe, refractory cases
Table 2-8 Classification of Asthma Severity and Treatment Algorithms
Ty pe
Symptoms
PEFR
Treatment of Exac erbations
Long-term Control
Mild intermittent
≤2 times/week
When asymptomatic,
Inhaled short-acting
No daily medications needed
Nocturnal awakening
>80% predicted value
β2-agonist
May use mast cell stabilizers
≤2 times/month
IV corticosteroids if
if known trigger
May only occur
persistent symptoms
during exercise
Mild persistent
Bronchodilator use
>20% fluctuations
Inhaled short-acting
Inhaled low dose
>2 times/week
over time
β2-agonist
corticosteroid
Nocturnal awakening
IV corticosteroids if
Consider mast cell stabilizer,
> every 2 weeks
persistent symptoms
leukotriene inhibitor, or theophylline
Moderate persistent
Daily symptoms
• 60%–80% predicted value
Inhaled short-acting
Inhaled low to medium dose
Daily bronchodilator
β2-agonist
corticosteroids and
use
IV corticosteroids if
long-acting β2-agonist
Symptoms interfere
persistent symptoms
Consider leukotriene
with activity
inhibitor or theophylline
Nocturnal awakening >1 time/week
Severe
Symptoms with
Wide variations
Inhaled short-acting
IInhaled high dose
minimal activity
Rarely >70%
β2-agonist
corticosteroids and
Awake multiple
predicted value
IV corticosteroids if
long-acting β2-agonist
times/night
Associated FEV 1 <60%
persistent symptoms
Consider systemic
Require multiple
predicted value
corticosteroids
medications on daily basis
FEV 1, forced expiratory volume in 1 second; IV, intravenous; PEFR, peak expiratory flow rate.
P.70 Gastroesophageal reflux disease More discussion in Chapter 3 Why eliminated from differential: the absence of dyspnea following eating or of symptoms consistent with reflux (e.g., heartburn, dysphagia) make reflux as a cause of dyspnea less likely; the response of symptoms to bronchodilator use also makes a gastric cause of the symptoms less likely Sarcoidosis Systemic disease of an unknown etiology characterized by noncaseating granuloma formation in the lungs Classic patient group is African American females in their 30s or 40s History: cough, malaise, weight loss, dyspnea, arthralgias (e.g., knees, ankles), vision loss Physical examination: fever, erythema nodosum (i.e., tender red nodules on shins and arms), lymphadenopathy, possible cranial nerve palsies Tests: Hypercalcemia and hypercalciuria, increased alkaline phosphatase, increased angiotensin-converting enzyme, increased erthrocyte sedimentation rate (ESR), decreased WBC Skin testing (i.e., PPD) frequently demonstrates anergy PFTs show restrictive pattern and decreased DLco
CXR shows bilateral hilar adenopathy and ground-glass pulmonary infiltrates Treatment: occasionally will self-resolve; systemic corticosteroids, azathioprine, methotrexate, cyclophosphamide, or cyclosporine may be useful in cases with significant pulmonary symptoms Outcomes: highly variable disease course; two thirds of patients will have spontaneous resolution, while others have persistent symptoms of variable severity; <5% of patients will have fatal disease Why eliminated from differential: sarcoidosis tends to be a diagnosis of exclusion when other processes cannot explain a constellation of symptoms; more systemic symptoms and signs would be expected in sarcoidosis (e.g., malaise, arthralgias, erythema nodosum, neurologic changes, adenopathy apparent on CXR)
CF is the most common fatal autosomal recessive disease in the United States and whites are at higher risk for the disorder. Cystic fibrosis (CF) Autosomal recessive disorder due to a defect in the chloride-pumping channels in exocrine glands Ducts of exocrine glands (e.g., lungs, pancreas, reproductive glands) become clogged with thick secretions Causes both pulmonary (e.g., recurrent infections, chronic sinusitis) and gastrointestinal (e.g., malabsorption, pancreatic enzyme deficiency) complications History: recurrent pulmonary infections (particularly Staphylococcus aureus and Pseudomonas aeruginosa), dyspnea, hemoptysis, chronic sinusitis, chronic cough, history of meconium ileus at birth, steatorrhea, failure to thrive Physical examination: cyanosis, digital clubbing, esophageal varices, rectal prolapse Tests: Decreased serum sodium Sweat test shows increased sodium and chloride (>60 mEq/L in children, >80 mEq/L in adults) Genetic testing can locate a mutation in the cystic fibrosis transmembrane conductance regulator (CFTR) gene in affected patients or carriers Treatment: DNase aids in decreasing the viscosity of secretions Aggressive chest physical therapy helps in clearing the airways of secretions Bronchodilators, nonsteroidal anti-inflammatory drugs (NSAIDs), and antibiotics frequently are used for pulmonary exacerbations or suspected infections Supplemental pancreatic enzymes and vitamins A, D, E, and K are given for malabsorption Outcomes: typically presents in childhood and is universally fatal, but improved therapies allow patients to live into late 20s or 30s P.71 Why eliminated from differential: the patient is older than expected for an initial diagnosis, and there is no failure to thrive; the normal sweat test and electrolyte panel rules out this diagnosis
Case 2-9: “I'm a big smoker” A 75-year-old man presents to his PCP to discuss chronic, worsening dyspnea. He describes having seen a commercial
on television discussing the risks of smoking tobacco and became concerned about his smoking history. He states that he has smoked about two packs per day since he was 16 years old. He has had shortness of breath to some degree for several years that he feels has gotten progressively worse over the past few years. He says that when he gets out of his chair to walk down the hall to the bathroom he becomes out of breath. He feels that he catches colds about four times each year. He has a productive cough and headache for the first few hours after waking daily. He denies any weight loss, night sweats, or hemoptysis. He denies ever having to go to an emergency department because of worsening of his symptoms. He has a history of HTN and mild coronary artery disease (CAD), for which he takes ASA, hydrochlorothiazide (HCTZ), and losartan. On examination, he does not appear to be in distress but is found to be mildly tachypneic and uses his accessory muscles during respiration. His breath sounds are faint throughout both lungs with a consistent wheezing heard during expiration. It is difficult to hear his heart sounds because of the rounded shape of his chest, but no abnormal sounds are detected. His liver is palpable 2 cm below his rib cage, and he has jugular venous distention to the angle of his mandible. Examination of his extremities and neurologic function are normal. The following vital signs are measured: T: 98.6°F, HR: 90 bpm, BP: 140/85 mm Hg, RR: 22 breaths/min
Differential Diagnosis COPD (chronic bronchitis vs. emphysema), asthma, bronchiectasis, cardiac pulmonary edema, TB
Laboratory Data and Other Study Results CBC: WBC: 8.4, Hgb: 12.5, Plt: 250 BNP: 81 pg/mL CXR: bilateral lung hyperinflation; diaphragm flattening; multiple subpleural blebs; generalized decreased vascular markings; no masses or lymphadenopathy PFTs: FEV1 50% expected; FEV 1/FVC 65% expected; increased TLC; PEFR decreased; resting O2 sat 88% on room air; DLco 55% predicted PPD: no reaction
Diagnosis COPD, emphysema type (moderate to severe)
Treatment Administered The patient was started on a daily short-acting β2-agonist and ipratropium inhalers The patient was provided with portable and home O2 to use as needed to maximize his activity tolerance The patient was referred to a respiratory therapy program The patient was highly encouraged to stop smoking and was enrolled into a smoking cessation program Pneumococcal and influenza vaccinations were administered
Follow-up The patient was able to improve function and mobility at home due to a decreased degree of dyspnea following the initiation of treatment The patient began to have acute exacerbations requiring brief hospital stays and titration of his medicines about every 4 to 6 months
The common form of emphysema has a centrilobular distribution, while the variant due to α1-antitrypsin deficiency has a panlobular distribution.
Steps to the Diagnosis COPD, emphysema variant Destruction of alveoli and bronchioles with associated panacinar airspace enlargement and decreased size of the pulmonary capillary bed due to macrophage and neutrophil proteolytic enzymes Occurs as a result of long-term tobacco use A less common form occurs in young nonsmokers due to α1-antitrypsin deficiency History: progressive dyspnea, possible productive cough, morning headache (due to relative hypercapnia), worsening of symptoms during exacerbations Physical examination: barrel chested, pursed-lip breathing, prolonged expiratory duration, wheezing, rhonchi, decreased breath sounds, decreased heart sounds, accessory muscle use, jugular venous distention (JVD), and possible palpable liver edge (due to right-sided heart failure), more prominent findings during exacerbations Tests: PFTs demonstrate decreased FEV1, FEV1/FVC, and PEFR and increased total lung capacity (TLC) (Figure 2-8) CXR demonstrates flattened diaphragm, hyperinflated lungs, subpleural blebs and bullae, and decreased vascular markings (Figure 2-9) ABG will show decreased O2 and increased CO2 (more pronounced during exacerbations)
Figure 2-8 Spirometry tracings and FEV1 measurements for normal lung function, obstructive disease, and restrictive disease. (A) Normal lung function. (B) Obstructive disease: note the increased lung volumes, decreased FEV 1, and decreased FEV1/FVC. (C) Restrictive disease: note the decreased lung volumes, decreased FEV1, but maintained FEV1/FVC. FEV1, forced expiratory volume in one second; FVC, functional vital capacity.
Figure 2-9 Anteroposterior (A) and lateral (B) chest x-ray views in a patient with chronic obstructive pulmonary disease. Note the hyperinflated lungs, flattened diaphragm, and decreased vascular markings.
Treatment: Smoking cessation is important to help prevent further progression Respiratory therapy is helpful to improve function Inhaled short-acting β 2-agonists and anticholinergics (e.g., ipratropium) decrease bronchoconstriction; inhaled long-acting β2-agonists and corticosteroids have less-defined benefits but may be useful for severe cases Supplemental O2 may improve dyspnea and activity tolerance (indicated for resting O2 saturation ≤88%) Antibiotics are given during respiratory infections because of the increased susceptibility to bacterial infections Enzyme replacement may be beneficial in cases of a1-antitrypsin deficiency Lung transplant may be considered in end-stage disease Outcomes: This is an irreversible condition, but smoking cessation and pharmacologic therapy help to slow progression The condition is complicated by chronic respiratory decompensation, right-sided heart failure (i.e., cor pulmonale), increased frequency of respiratory infections, and acute exacerbations that may require hospitalization
An increased frequency of lung cancer occurs in the same patient population Clues to the diagnosis: History: chronic dyspnea that worsens with exertion, significant smoking history, frequent respiratory infections Physical: tachypnea, barrel chest, accessory breathing muscle use, faint breath sounds, wheezing, JVD, hepatomegaly Tests: hypoxemia, CXR appearance, PFT results (e.g., decreased FEV 1, FEV1/FVC, DLco, PEFR, and increased TLC) COPD, chronic bronchitis variant Chronic bronchial inflammation associated with prolonged tobacco use (most common) or chronic asthma (less common) Occurs in a continuum with emphysema History: productive cough (must be present for at least 3 months of the year for more than 2 years in order to meet the criteria for diagnosis), dyspnea, recurrent respiratory infections Physical examination: wheezing, mild tachypnea, rhonchi Tests: PFTs may be within the normal range but gradually demonstrate more significant abnormalities consistent with obstructive disease; CXR may be normal or show early emphysematous changes Treatment: Smoking cessation is important to prevent further progression of the disease Antibiotics are given for URIs due to a higher risk of bacterial infection Bronchodilators are useful during exacerbations to reduce symptoms Outcomes: smoking cessation is the key to preventing further progression of COPD; continued tobacco use leads to the development of cor pulmonale and later emphysema Why eliminated from differential: although COPD is a continuum including both chronic bronchitis and emphysema, the severe pulmonary degeneration and symptoms are more consistent with progressive disease; the significant changes in the PFTs including the decreased DLco are more consistent with emphysema than chronic bronchitis
Patients with chronic bronchitis are “blue bloaters” because the secondary development of cor pulmonale causes cyanosis and peripheral edema; patients with emphysema are “pink puffers” because of their pursed lip breathing, dyspnea, and barrel chests.
DLco is useful to differentiate between chronic bronchitis (normal range) and emphysema (decreased). Asthma More thorough discussion in prior case Why eliminated from differential: the case describes an irreversible pulmonary condition (asthma is reversible); asthma typically starts at an earlier age, and a long smoking history is more typical of COPD
Bronchiectasis Permanent dilation of small and medium bronchi due to destruction of bronchial elastic components Occurs secondary to TB, fungal pulmonary infections, severe pneumonia, or cystic fibrosis History: persistent productive cough, hemoptysis, prior frequent respiratory infections, copious sputum production Physical examination: wheezing, rales, consistent hypoxemia Tests: CXR shows multiple cysts and bronchial crowding; CT shows dilation of bronchi and bronchial wall thickening Treatment: Inhaled β2-agonists may ameliorate symptoms Antibiotics are given when the sputum production worsens to reduce the risk of respiratory infection Respiratory therapy helps to optimize function Outcomes: poor prognosis and typically occurs as end-stage pulmonary disease; associated with cor pulmonale, massive hemoptysis, and pulmonary abscess formation Why eliminated from differential: the absence of constant copious sputum production and the lack of characteristic CXR findings in the case rules out this diagnosis Cardiac pulmonary edema More thorough discussion in prior case Why eliminated from differential: the CXR in the case demonstrates hyperinflation and diaphragm flattening and not diffuse infiltrates, as would be expected for pulmonary edema, the BNP is a normal value in this case, and the PFTs in pulmonary edema would not demonstrate an obstructive pattern Tuberculosis More thorough discussion in prior case Why eliminated from differential: CXR in the case is not characteristic for TB; the negative PPD helps to rule out this diagnosis
Case 2-10: “I'm losing weight and feel tired” The patient from the previous case, now 78 years old, returns to his PCP's office for routine follow-up for his diagnosis of COPD. He has occasional exacerbations of his COPD and has been hospitalized on a few occasions to stabilize his pulmonary function and to titrate his medications. In between hospitalizations, his COPD has been relatively controlled. He has had difficulty quitting smoking but has decreased his usage to a few cigarettes per day. On this visit he notes that he has been feeling increasingly tired over the past 2 months. He also describes losing about 15 pounds over this time. On his current regimen of an inhaled short-acting β2-agonist and ipratropium and consistent supplemental O2, he has minimal dyspnea and is able to tolerate light activity. He denies other new symptoms besides the weight loss and fatigue. He denies any nausea, vomiting, diarrhea, abdominal pain, or urinary problems. On examination he is not in any distress and appears to be breathing comfortably on his supplemental O 2. He has mild JVD. On auscultation, he has decreased heart and breath sounds. He has no audible wheezing and his accessory muscle use has lessened. Abdominal examination detects mild hepatomegaly. His neurologic examination is normal. The following vital signs are measured: T: 98.3°F, HR: 80 bpm, BP: 140/85 mm Hg, RR: 17 breaths/min
Differential Diagnosis COPD (severe vs. end-stage), lung neoplasm, CHF
Laboratory Data and Other Study Results CBC: WBC: 7.2, Hgb: 12.0, Plt: 217 CXR: a 5-cm nodule without calcifications localized to the right middle lobe; hilar adenopathy; no effusions or lobar consolidation PEFR: decreased from normal but improved from the previous case Pulse oximetry: O2 sat 94% on 1L O2 Following report of these studies, the tests below are performed: Chest CT with contrast: solitary mass in right middle lobe (approximately 6 cm from carina); right hilar and mediastinal lymphadenopathy Fluorodeoxyglucose positron emission tomography (FDG-PET): increased uptake in the region of the lesion and around the right hilar and mediastinal lymph nodes; no distant areas of increased uptake Head magnetic resonance imaging (MRI): no lesions, normal cerebral size, normal ventricles, no hemorrhage Decision is made to perform biopsy to confirm diagnosis: Bronchoscopy: mass visualized with extension into the lumen of the right posterior basal segmental bronchus; biopsy of mass and transbronchial biopsy of hilar nodes performed Lesion brushings and biopsy cytology: consistent with poorly differentiated squamous cell carcinoma
Diagnosis Lung neoplasm, squamous cell carcinoma
Treatment Administered Surgery was deferred given the mediastinal lymph node involvement and the impaired pulmonary function due to COPD Chemotherapy with cisplatin and paclitaxel and radiation therapy were initiated
Follow-up The patient was closely monitored for a response to therapy; he was found to have a moderate response to chemotherapy and radiation with an initial decrease in tumor size The patient was not judged to be amenable to surgical resection given his pulmonary function and the partial response to nonoperative therapy; the chemotherapy regimen was modified to continue nonoperative treatment The patient developed distant metastases 4 months after his initial diagnosis and died an additional 2 months later due to complications related to metastatic disease
Breast, colon, prostate, endometrial, and cervical cancers are the most common causes of metastases to the lungs and account for 10% of solitary cancerous lesions.
Adenocarcinoma is the most common lung cancer in nonsmokers.
Steps to the Diagnosis Lung cancer Lung neoplasms are most frequently associated with tobacco use (90% of cases) and occupational exposures (e.g., smoke, asbestos) (Table 2-9) History: Frequently asymptomatic Cough, dyspnea, pleuritic chest pain, fatigue, weight loss, frequent respiratory infections Additional symptoms may result from local extension of the tumor or tumor biologic activity Horner syndrome: invasion of cervical ganglia causing miosis, ptosis, and anhidrosis of the eye Pancoast syndrome: Horner syndrome and brachial plexopathy
Table 2-9 Common Types of Primary Lung Cancer
Primary Primary Lung
Malignanc ies
Canc er Ty pe
(%)
Squamous cell
25–35
Assoc iated Paraneoplastic Loc ation
Central
carcinoma
Adenocarcinoma
Charac teristic s
Cavitary lesions; direct extension to hilar lymph nodes
25–35
Peripheral
Wide metastases; may be caused by asbestos; pleural effusions show increased hyaluronidase levels; bronchiolar cancer is subtype that is low grade and occurs in single nodules
Syndromes
Hypercalcemia Dermatomyositis
DIC Thrombophlebitis Microangiopathic hemolytic anemia Dermatomyositis
Small cell
20–25
Central
carcinoma
Rapidly growing; early distant metastases; several paraneo-plastic syndromes
Cushing syndrome Syndrome of inappropriate ADH secretion (SIADH) Ectopic growth hormone and ACTH secretion Peripheral neuropathy Subacute cerebellar degeneration Eaton-Lambert syndrome (similar presentation to myasthenia gravis) Subacute sensory neuropathy Limbic encephalitis Dermatomyositis
Large cell
5–15
Peripheral
Late distant metastases, early cavitation
carcinoma
Gynecomastia Dermatomyositis
ACTH, adrenocorticotropic hormone; ADH, antidiuretic hormone; DIC, disseminated intravascular coagulation; SIADH, syndrome of inappropriate antidiuretic hormone secretion.
Superior vena cava syndrome: mass effect of the tumor causes compression of the superior vena cava leading to impaired cervical and cranial venous drainage, head swelling, and central nervous system (CNS) symptoms Paraneoplastic syndromes: extrapulmonary syndromes due to biologic products of tumor (Table 2-9) Physical examination: decreased breath sounds with bronchial obstruction, lymphadenopathy, possible JVD or hepatomegaly with SVC syndrome or liver metastases, other findings consistent with paraneoplastic syndromes or local extension (findings depend on the size of tumor, extent of local expansion, and presence of metastases) Tests: Any solitary pulmonary nodule seen on CXR requires a systematic work-up to determine its significance (Figure 2-10) Concerning lesions require a CT of the chest to determine the size and extent of involvement (Figure 2-11) FDG-PET or bone scan should be performed to look for distant metastases Head MRI may be performed to look for brain metastases
Biopsy is performed to determine the final tissue diagnosis Treatment: Therapy varies according to tumor type and staging and consists of surgery, chemotherapy, and/or radiation therapy Staging to guide therapy is determined considering local extension, lymph node involvement, and the presence or absence of metastases (Table 2-10) Palliative chemotherapy or radiation may be used for unresectable tumors, associated tumor syndromes, or in patients not medically appropriate for surgical treatment Outcomes: Generally poor prognosis Small cell lung cancer carries up to a 15% 5-year survival rate for limited disease and <2% 5-year survival for extensive disease
Figure 2-10 Work-up of the solitary pulmonary nodule. CT, computed tomography; FDG-PET, fluorine-18-2-deoxy-2-fluoro-D-glucose positron emission tomography.
Figure 2-11 Imaging of a pulmonary mass. (A) Chest x-ray demonstrates a mass (black arrow) adjacent to the right hilum. Because the mass does not obscure the border of the ascending aorta (arrowheads), it must be located either anterior or posterior to this structure. (B) Chest computed tomography of the same patient localizing the mass to the anterior region of the right upper lobe. Further testing confirmed the mass to be a malignancy.
Non–small cell lung cancer carries up to a 75% 5-year survival rate for disease with minimal local extension and no nodal involvement beyond ipsilateral hilar lymph nodes and <15% 5-year survival for more extensive disease Clues to the diagnosis: History: history of COPD, weight loss, fatigue
Physical: noncontributory Tests: nodule on CXR and chest CT, PET scan appearance, visualization of a mass on bronchoscopy, biopsy results
Forty percent of all solitary pulmonary nodules are malignant.
Smoking cessation is the only therapy found to reduce the risk of lung cancer in active smokers.
The most common sites of distal metastases for lung cancer may be remembered by the acronym BLAB: Bone, Liver, Adrenals, Brain.
Table 2-10 Treatment for Lung Cancer Based on Staging Algorithm
Neoplasm Ty pe
Staging
Surgery
Chemotherapy
Radiation Therapy
Non–small cell (e.g.,
No mediastinal invasion, no
Surgic al resec tion
Adjuvant therapy to
Primary therapy (medically
squamous cell,
lymph node involvement
(e.g., lobectomy,
surgery
inappropriate for surgery) or
adenocarcinoma, large
beyond ipsilateral hilar
video-assisted
postoperative adjuvant
cell)
nodes, no metastases
thorascopic surgery)
therapy to surgery
No mediastinal invasion or
Consider if significant
Induction therapy if
Primary therapy; continue
metastases, has extension
decrease in tumor size
considering surgery
postoperatively if surgery
to ipsilateral mediastinal
following radiation
or adjuvant to
performed
radiation
nodes
Mediastinal invasion, distant
None
Palliative
Palliative
Small lesion, no nodal spread,
Consider for very small
Primary therapy
Adjuvant therapy to
no metastases
lesions
All other lesions
None
nodes, and/or metastases
Small cell
chemotherapy
Primary therapy
Adjuvant therapy after chemotherapy
Chronic obstructive pulmonary disease More thorough discussion in prior case Why eliminated from differential: the previous symptoms of COPD in the case have stabilized other than the new-onset fatigue and weight loss, and pulmonary measurements do not demonstrate the worsening that would be expected for progressive disease; this diagnosis is excluded through the work-up of the lesion Congestive heart failure More thorough discussion in Chapter 1 Why eliminated from differential: CXR in the case demonstrates a pulmonary nodule and not pulmonary edema as would be expected for CHF; the diagnosis is excluded through work-up of the lesion
Case 2-11: “I was a smoker years ago, and now I'm out of breath” A 75-year-old man presents to his PCP with a complaint of dyspnea that has been worsening over the past 2 years. He says that he can walk only about one block before becoming short of breath and having to take a break. He denies any acute worsening of his condition and feels that this has been a gradual deterioration. He denies recent weight loss. He says that he started smoking when he was 18 and smoked about half a pack per day, but quit a little more than 20 years ago. He worked in construction for many years, primarily in the demolition of buildings. He has a history of mild HTN that has been controlled with diet and a history of benign prostatic hypertrophy that is being treated with tamsulosin. He denies a family history of respiratory disorders. On examination, he is mildly tachypneic but in no distress. Auscultation detects bilateral dry rales and some faint wheezing. Otherwise, his lung and cardiac examinations are normal. Examination of his abdomen finds no organomegaly or masses. He is found to have clubbing of his fingers. The following vital signs are measured: T: 97.9°F, HR: 78 bpm, BP: 128/75 mm Hg, RR: 20 breaths/min
Differential Diagnosis COPD, lung cancer, pneumoconiosis, other interstitial pulmonary disease (e.g., idiopathic pulmonary fibrosis, Goodpasture syndrome, Wegener granulomatosis, etc.), sarcoidosis, TB, malignant mesothelioma
Laboratory Data and Other Study Results CBC: WBC: 8.2, Hgb: 13.5, Plt: 350 Chem7: Na: 138 mEq/L, K: 4.1 mEq/L, Cl: 104 mEq/L, CO2: 22 mEq/L, BUN: 14 mg/dL, Cr: 0.9 mg/dL, Glu: 101 mg/dL ESR: 15 mm/hr PPD: nonreactive CXR: multinodular opacities throughout both lungs; bilateral costophrenic angle pleural effusions; blurring of the heart border Following the report of these studies the following additional tests are ordered: Chest CT: plaque formation along pleural borders with linear densities; diffuse parenchymal scarring PFTs: decreased TLC; decreased residual volume (RV); decreased FVC; decreased FEV 1; normal FEV1/FVC; decreased DLco; O2 sat: 94% To confirm the diagnosis, the following test is performed: Pleural biopsy via thoracoscopy: multiple short brown fibers in pleural tissue
Diagnosis Pneumoconiosis asbestosis
Treatment Administered The patient was educated regarding the risks of smoking and was encouraged to remain tobacco free The patient was provided influenza and pneumococcal vaccines
Follow-up
The patient was followed periodically to track his respiratory symptoms He was placed on supplemental O2 5 years after his diagnosis as his dyspnea worsened and as he began to develop hypoxemia Seven years after the initial diagnosis, the patient was diagnosed with small cell carcinoma of the lung and despite chemotherapy died in 6 months
Steps to the Diagnosis Pneumoconioses Interstitial lung diseases that result from long-term occupational exposure to substances that cause pulmonary inflammation (Table 2-11) History: cough, dyspnea on exertion, possible heavy sputum production; symptoms begin when significant pulmonary fibrosis has occurred, and a lag time of several years from the last exposure to the onset of symptoms is common Physical examination: coarse rales, possible wheezing, clubbing of digits Tests: ESR may be normal or minimally increased PFTs demonstrate a restrictive pattern with decreased lung volumes, normal FEV 1/FVC, and decreased DLco CXR generally shows multinodular opacities (Figure 2-12) Chest CT shows parenchymal fibrosis Treatment: No successful curative treatments Supportive care as needed Prevention is the key for avoiding disease (e.g., air filters, masks, safe-handling guidelines) Outcomes: gradual worsening of pulmonary fibrosis at variable rates and the development of progressive respiratory failure occurs as the condition worsens; may be associated with an increased risk for malignancy Clues to the diagnosis: History: former work in building demolition, dyspnea that worsens during exertion Physical: dry rales, wheezing, digital clubbing Tests: appearance of CXR and chest CT, PFTs with a restrictive pattern, pleural biopsy results Chronic obstructive pulmonary disease More thorough discussion in prior case Why eliminated from differential: the PFTs in the case demonstrate a restrictive, not obstructive pattern; radiographic findings in the case suggest parenchymal fibrosis and not obstructive disease
Table 2-11 Common Pneumoconioses and How to Diagnose Them
Disease
Asbestosis
Silicosis
Exposure
Labs
Radiology
Working with insulation,
PFTs show
Multinodular opacities, pleural
Increased risk of malignant
construction,
restrictive
effusions, blurring of
mesothelioma and lung
demolition, building
pattern; asbestos
heart/diaphragm; chest CT shows
cancer; synergistic effect
maintenance,
fibers seen in
linear pleural/parenchymal fibrosis
with tobacco
automobiles
pleural biopsy 1
Mining, pottery making,
PFTs show
Small apical nodular opacities; hilar
Inc rease risk of TB
sandblasting, cutting
restrictive pattern
adenopathy
infec tion; progressive fibrosis
granite
Coal
Complic ations
Coal mining
PFTs show
Small apical nodular opacities
Progressive fibrosis
restrictive pattern
worker's disease
Berylliosis
Elec tronic s, ceramics,
Pulmonary edema,
Diffuse infiltrates; hilar
Increased risk of lung
tool, die manufacturing
diffuse granuloma
adenopathy
cancer; may need chronic
formation
corticosteroid treatment to maintain respiratory function
CT, computed tomography; PFTs, pulmonary function tests; TB, tuberculosis. 1
Not needed for diagnosis with known exposure to asbestos and suggestive radiographic work-up.
Figure 2-12 Patient with radiographic signs of asbestosis. (A) Chest radiograph demonstrating pleural thickening, blurring of the heart and diaphragm borders, and nodular opacities. (B) Chest computed tomography from same patient demonstrating pleural thickening and linear parenchymal fibrosis.
Lung cancer More thorough discussion in prior case Why eliminated from differential: radiographic findings are most consistent with parenchymal fibrosis and not the development of neoplastic lesions (patient is still at increased risk for lung cancer as evidenced by the
development of small cell carcinoma later during the disease course); the presence of asbestos bodies on biopsy is confirmatory of the underlying disease Idiopathic pulmonary fibrosis (IPF) Inflammatory lung disease of an unknown origin that causes progressive parenchymal fibrosis History: age >50 years, progressive exercise intolerance, dyspnea on exertion Physical examination: dry rales, JVD, tachypnea, digital clubbing Tests: PFTs will show a restrictive pattern CXR shows a reticulonodular pattern and “honeycomb” lung in advanced cases Chest CT shows diffuse ground-glass appearance of parenchyma Bronchioalveolar lavage shows increased pulmonary polymorphonuclear cells (PMNs) Treatment: supplemental O2; corticosteroids may ease symptoms Outcomes: progressive fibrosis with mortality occurring frequently within 5 years Why eliminated from differential: the case presentation is similar to IPF, but the pleural biopsy and exposure history reveal the true diagnosis; numerous PMNs are not seen on the biopsy in the case as would be expected for IPF Goodpasture syndrome Rare progressive autoimmune disease of the lungs and kidneys characterized by intra-alveolar hemorrhage and glomerulonephritis Caused by antibodies to the glomerular basement membrane History: dyspnea, recurrent respiratory infections, hemoptysis (possibly massive), chills, weight loss, arthralgias, chest pain, possible gross hematuria Physical examination: fever, tachypnea, rales, cyanosis, possible hepatosplenomegaly and HTN Tests: BUN and creatinine will be increased Urinalysis (UA) will demonstrate hematuria and proteinuria PFTs show restrictive pattern CXR shows diffuse interstitial disease Serologic testing detects anti-glioblastoma multiforme (GBM) antibodies Treatment: corticosteroids or immunosuppressants may ease symptoms; plasmapheresis may be required to remove autoantibodies Outcomes: prognosis is generally poor without aggressive treatment (usually due to renal failure) Why eliminated from differential: the absence of hemoptysis or renal insufficiency makes this diagnosis unlikely Wegener granulomatosis
Rare disease characterized by granulomatous inflammation and necrosis of pulmonary and other organ systems Due to systemic vasculitis (affects mainly lung and kidney) with the resultant formation of noncaseating granulomas and parenchymal necrosis History: dyspnea, myalgias, chronic sinusitis, hemoptysis Physical examination: nasopharynx ulcerations, fever, other vague renal (e.g., mild hematuria), neurologic (e.g., sensory neuropathy, hearing loss), ophthalmologic (e.g., corneal ulceration, diplopia), or cardiac (e.g., arrhythmia) signs Tests: Positive cytoplasmic antineutrophil cytoplasmic antibodies (c-ANCA) Renal biopsy detects a vasculitic process Biopsy of lesions detects noncaseating granulomas Treatment: corticosteroids and cyclophosphamide may slow disease progression Outcomes: rapidly fatal if untreated; gradual multiorgan deterioration is typical with 30% 5-year survival Why eliminated from differential: the absence of renal findings and pleural granulomas on the biopsy in this case make this diagnosis less likely; serologic testing for c-ANCA may help provide a definite rule out Pulmonary Langerhans cell histiocytosis Autoimmune overproliferation of pulmonary Langerhans cells (differentiated from macrophages) leading to an inflammation of bronchioles, intraluminal fibrosis, and interstitial pneumonia Possible association with tobacco use History: young adult 30 to 40 years old, dyspnea, nonproductive cough, possible weight loss and fatigue, possible spontaneous pneumothorax Physical examination: possible rales or digital clubbing Tests: CXR shows reticulonodular opacities in upper lung fields and possible pneumothorax Chest CT shows multiple cysts and nodules in the upper lung fields and interstitial thickening PFTs show decreased DLco and likely a restrictive pattern Transbronchial biopsy will detect Langerhans cells Treatment: Smoking cessation Corticosteroids may be helpful in improving symptoms Consider lung transplant in progressive cases Outcomes: Variable course with some remissions and some progressing to end-stage disease Seventy-five percent 5-year survival Regular follow-up is needed to detect neoplasms or disease progression
Why eliminated from differential: the patient in the case is older than expected for this diagnosis, and the pleural biopsy does not detect Langerhans cells Sarcoidosis More thorough discussion in prior case Why eliminated from differential: the lack of systemic findings in case, and no hilar adenopathy on the CXR in the case make this diagnosis less likely Tuberculosis More thorough discussion in prior case Why eliminated from differential: the PPD is negative in the case, and the CXR is not typical of pulmonary TB Malignant mesothelioma Rare pleural tumor most commonly due to asbestos exposure History: chest pain, dyspnea Physical examination: dullness to percussion over lung bases, palpable chest wall mass, scoliosis toward lesion Tests: CXR shows pleural thickening and pleural effusion; pleural biopsy is usually diagnostic Treatment: chemotherapy and radiation therapy Outcomes: poor prognosis with typical life expectancy following diagnosis of 4 to 18 months Why eliminated from differential: the pleural biopsy in the case does not detect a malignancy, no effusion is seen on the CXR, and a chest wall mass or scoliosis is not apparent on examination
Case 2-12: “I keep falling asleep at work” A 35-year-old man presents to the office of an internist for a medical consultation by request of his employer. He works as a computer programmer and has been caught falling asleep at his desk on numerous occasions. He reports that he gets about 7 hours of sleep each night but rarely awakens feeling refreshed. He says that his wife often complains of his loud snoring. He drinks several cups of coffee each day but still feels tired. He denies dyspnea, syncope, or depression. He denies any medical problems other than obesity. He reports that he drinks a few beers while watching TV on the weekends but otherwise does not use any substances. On examination, he is a morbidly obese male who appears sleepy but comfortable. He does not appear to have any abnormalities in his oropharynx. All cardiac, pulmonary, and neurologic examinations are normal. The following vital signs are measured: T: 98.7°F, HR: 85 bpm, BP: 140/85 mm Hg, RR: 16 breaths/min
Differential Diagnosis Sleep apnea, narcolepsy, substance abuse, asthma (poorly controlled)
Laboratory Data and Other Study Results Height and weight: 5﹐ 6″, 280 lb, BMI: 45.2 PEFR: 640 L/min Multidrug serum and urine toxicology: no detectable illicit substances Epworth sleepiness scale: 12
Home polysomnography is scheduled to assess sleep: Home polysomnography: Apnea Index (AI) 21; continued respiratory effort during periods of apnea; desaturation to 85% during apneic periods; frequent arousals
Diagnosis Obstructive sleep apnea
Treatment Administered The patient was placed on a weight loss program The patient was provided educational information instructing the avoidance of alcohol and depressant medications before bedtime Continuous positive airway pressure (CPAP) was prescribed for use at night
Follow-up Follow-up home polysomnography after the initiation of treatment demonstrated a significant improvement in the measurements of apnea The patient's performance at work improved The patient had difficulty maintaining weight loss and was eventually scheduled for bariatric surgery
Steps to the Diagnosis Sleep apnea Episodic cessation of airflow during sleep leading to desaturation and frequent arousals May be either obstructive or central in nature Obstructive: obstruction of the upper airway during sleep with continued respiratory effort; most frequently due to obesity or an anatomic abnormality in the pharynx Central: loss of central respiratory drive, leading to the cessation of airflow in the absence of respiratory effort Mixed: combines characteristics of both types above The etiology may be linked to an abnormal feedback control during sleep or a decreased sensitivity of upper airway muscles to stimulation History: fatigue, daytime sleepiness, snoring, morning headaches or confusion, impaired function during daytime due to sleepiness Physical examination: frequent obesity in obstructive type, possible visible abnormalities of the soft palate, uvula, tonsils, or nasopharynx Tests: Epworth sleepiness scale Series of questions directed to determine relative sleepiness in a patient during various activities Useful for predicting the likelihood of sleep apnea as a cause of daytime somnolence
A score >10 is common in sleep apnea Polysomnography (advantage of home vs. lab uncertain) is the definitive test that measures the AI (average apneic episodes per hour), O2 saturation, and the number of arousals Treatment: depends on the cause of apnea Obstructive: Weight loss, limitation of sedating substances CPAP helps to maintain airway patency Surgical tonsil or polyp removal, correction of soft palate deformity, or tracheostomy may be necessary in severe cases Bariatric surgery for morbid obesity may help to relieve obstruction Central: Respiratory stimulants Phrenic nerve pacemaking may be needed in severe cases Outcomes: The untreated condition is associated with the development of several pulmonary and cardiac complications (e.g., cor pulmonale, HTN) The degree of compliance with CPAP affects the success of treatment Significant weight loss due to bariatric surgery is associated with resolution of the disease in 86% of cases In patients with documented obstructive sleep apnea the 8-year survival is >95% in patients with an AI below 20 and 63% in patients with an AI above 20 Clues to the diagnosis: History: sleepiness despite adequate time devoted to sleeping at night, daytime somnolence, snoring Physical: obesity Tests: increased Epworth score, home polysonography results (e.g., apneic episodes with desaturation, frequent arousals, high AI)
Unlike sleep apnea, a short nap will temporarily relieve the feeling of sleepiness in patients with narcolepsy.
Remember that “narcolepsy occurs in CHAPs” (young adults): Cataplexy, Hallucinations, Attacks of sleep, Paralysis upon waking. Narcolepsy Neurologic disorder characterized by persistent excessive daytime sleepiness regardless of prior sleep quality More common in young adults History: chronic sleepiness, falling asleep at inappropriate times, possible sleep attacks (i.e., sudden falling asleep regardless of the current activity), hallucinations while falling asleep or during waking, sleep paralysis
(i.e., inability to move when first waking), cataplexy (i.e., sudden episodes of weakness triggered by strong emotions), possible insomnia Physical examination: cataplexy may be observed during laughter or crying Tests: Epworth score is frequently >15; polysomnography may be normal or may show multiple arousals and delayed latency until rapid eye movement (REM) sleep Treatment: Modafinil or amphetamines help to decrease sleepiness and prevent cataplexy Venlafaxine or fluoxetine may help to reduce cataplexy Gamma hydroxybutyrate (GHB) helps reduce sleepiness and prevent cataplexy but has become infamous for its abuse potential and use as a “date rape” drug A regular sleep schedule including short naps is helpful for maintaining improved wakefulness Outcomes: the condition can place significant limitations on daily function and work; a treatment plan of pharmacologic and nonpharmacologic therapies can give these patients normalcy in their daily lives Why eliminated from differential: cataplexy does not occur in this case; polysomnography in the case demonstrates apneic episodes and is not consistent with narcolepsy Substance abuse More thorough discussion in Chapter 13 Why eliminated from differential: the negative toxicology testing rules out substance use as a cause of sleepiness Asthma More thorough discussion in prior case Why eliminated from differential: poorly controlled asthma may worsen at night and cause frequent awakenings, but the normal PEFR and absence of history for asthmatic symptoms make this diagnosis unlikely
Case 2-13: “I can barely breathe and my chest hurts” A 19-year-old man is brought into the trauma bay by emergency medical technicians (EMTs) after he was hit by a car. He reports riding a skateboard and being hit from the side by a car making a turn. He tried to jump on to the car right before it hit him but landed on the windshield. He was able to get up and walk immediately afterward but had to sit down because of the pain on the right side of his chest and shortness of breath. He denies pain in any other region of his body and denies losing consciousness. He denies prior episodes of chest pain. The standardized trauma evaluation was performed upon his arrival focusing on the Airway, Breathing, Circulation (ABCs) followed by a primary survey. He is in mild-to-moderate respiratory distress. He has tenderness to palpation along the right fifth and sixth ribs in the midaxillary line. He has decreased breath sounds over the right side of his chest and asymmetrical chest movement between the left and right sides. He has no open wounds detectable on examination but has an abrasion on the right side of his chest and axilla. He has no pain to palpation in his abdomen, pelvis, extremities, neck, or back. He has no pain with movement of his extremities and has a normal neurologic examination. The following vital signs are measured: T: 98.9°F, HR: 100 bpm, BP: 120/75 mm Hg, RR: 22 breaths/min
Differential Diagnosis Pneumothorax (PTX; simple vs. tension), hemothorax, pleural effusion, pulmonary contusion
Laboratory Data and Other Study Results Pulse oximetry: O2 sat 88% on room air CBC: WBC: 7.5, Hgb: 13.7, Plt: 400 Upright CXR: visible pleural line of right lung with 50% loss of lung field; absent lung markings and greater picture clarity lateral to the pleural line; mildly displaced fractures of the midlateral right fifth and sixth ribs; no tracheal deviation
Diagnosis Simple PTX, 50% involvement of right lung
Treatment Administered A chest tube was placed in sterile fashion under local anesthesia in the right side of the chest Supplemental O2 was administered
Follow-up Repeat CXR showed near complete resolution of the pneumothorax with an adequate chest tube placement The secondary survey detected no other areas of concern The patient was admitted to the trauma service for observation and chest tube management The chest tube was successfully removed in 3 days without any residual PTX The patient was able to be discharged to home in stable condition
Common causes of PTX may be remembered by the acronym A CHEST IN: Asthma, Cystic fibrosis, HIV (AIDS), Emphysema, Spontaneous, Trauma, Iatrogenic, Neoplasm.
Steps to the Diagnosis Pneumothorax Collection of air in the pleural space causing parenchymal collapse and predisposing the patient to respiratory failure (Table 2-12)
Table 2-12 Types of Pneumothorax and Their Causes
Type of Pneumothorax
Mec hanism
Causes
Closed
Internal rupture of respiratory system; chest wall intact
Spontaneous, COPD, TB, blunt trauma
Open
Passage of air through opening in chest wall
Penetrating trauma, iatrogenic (e.g., central line placement, thoracocentesis, biopsy)
Tension
Open pneumothorax; “ball-valve” condition allows air to
Trauma
enter but not leave pleural space
COPD, chronic obstructive pulmonary disease; TB, tuberculosis.
History: unilateral chest pain, dyspnea (worse symptoms with open or tension types) Physical examination: decreased chest wall movement, unilateral decreased breath sounds, increased resonance to percussion, decreased tactile fremitus; significant respiratory distress, hypotension, JVD, tracheal deviation suggest a tension PTX Tests: CXR shows lung retraction with a visible pleural line and mediastinal shift away from affected side (Figure 2-13) On CXR tension PTX will show tracheal deviation in addition to the findings seen with a simple PTX Pulse oximetry may show severe hypoxemia with a tension PTX and milder hypoxemia with a simple PTX Treatment: A small (<15%) PTX may resolve with supplemental O2 only A simple PTX >15% requires placement of a chest tube to decompress intrapleural air An open PTX with a small open wound is treated with an occlusive dressing and a chest tube; a larger open PTX should be treated with attempted closure and a low clinical threshold for intubation Tension PTX requires immediate needle decompression (fourth or fifth intercostal space in the maxillary line) and chest tube placement Recurrent PTX may require pleurodesis
Figure 2-13 Chest radiograph demonstrating a right-sided pneumothorax with a visible pleural line along the lateral border of the lung
(arrows).
Outcomes: the majority of patients treated will rapidly recover well and may be gradually weaned from the chest tube as the air output decreases; patients with significant open or tension PTX are at high risk for severe respiratory distress and may require intubation and ventilation Clues to the diagnosis: History: blunt chest trauma, dyspnea, chest pain Physical: unilateral decreased breath sounds, chest tenderness, tachypnea, mild respiratory distress, asymmetric chest wall motion Tests: CXR appearance
The classic patient for a spontaneous simple PTX is a young, thin, and tall male.
Do not wait for a CXR in cases of suspected tension PTX; immediately perform needle decompression! Hemothorax Collection of blood in the pleural space due to trauma, malignancy, TB, or pulmonary infarction History: dyspnea, pleuritic chest pain, weakness Physical examination: decreased breath sounds, dullness to percussion, decreased tactile fremitus, egophony Tests: CXR shows lung retraction, mediastinal shift away from the affected side, and opacification of the involved pleural space; thoracocentesis demonstrates a bloody effusion Treatment: Chest tube placement Supplemental O2 Intravenous fluids (IVF) and possible blood transfusions are required if a significant blood volume is lost into the chest cavity Treat the underlying medical cause if nontraumatic to help prevent recurrence Outcomes: may be complicated by thrombus formation or pleural fibrosis if blood is not drained from the pleural space; hypotension, anemia, and tissue ischemia may result from significant blood loss Why eliminated from differential: the CXR in the case shows greater clarity in the pleural space and not greater opacity from collected blood
A high triglyceride level is a characteristic finding in pleural effusions due to lymphatic duct rupture. Pleural effusion Serous or lymphatic fluid collection in the pleural space due to changes in hydrostatic and oncotic pressure
(i.e., transudative), inflammation (i.e., exudative), or lymphatic duct rupture (i.e., lymphatic) Classified according to the protein and lactate dehydrogenase (LDH) content (Table 2-13) History: possibly asymptomatic, but dyspnea, pleuritic chest pain, and weakness are frequently present Physical examination: decreased breath sounds, dullness to percussion, decreased tactile fremitus, egophony Tests: CXR shows blunting of costophrenic angles; comparison of upright and decubitus CXR can differentiate between free-flowing and loculated effusions Pleural fluid analysis from thoracocentesis is analyzed for protein and LDH levels, cell count, Gram stain, and cytology to determine the etiology
Table 2-13 Distinctive Characteristics and Causes of Types of Pleural Effusions
Pleural/Serum Protein
Pleural/Serum LDH
Total Pleural
Ratio
Ratio
Protein
Effusion
Transudate
<0.5
<0.6
<3 g/dL
Causes
CHF, cirrhosis, kidney diseases (e.g., nephrotic syndrome)
Exudate
>0.5
>0.6
>3 g/dL
Infection, cancer, vasculitis
CHF, congestive heart failure; LDH, lactate dehydrogenase.
Treatment: Thoracocentesis and possible chest tube placement is used to relieve compressive forces on the lung; a chest tube must be placed for an empyema (i.e., purulent effusion) to adequately drain the pleural space Underlying medical conditions should be treated to reduce the risk of recurrence Multiply recurrent effusions may require pleurodesis to prevent additional recurrences Outcomes: Can cause gradual respiratory collapse if not recognized and treated Since typically due to a medical (nontraumatic) cause, treatment of the underlying cause is the key to prevention of recurrence One quarter of all effusions are malignant in nature, so any effusion without a known cause requires a cancer work-up Why eliminated from differential: the CXR in the case does not demonstrate pleural space opacity; trauma is typically not the cause of this diagnosis
Pulmonary contusion is the most common lung injury encountered as a result of blunt trauma.
Pulmonary contusion Lung parenchymal injury due to blunt trauma that may result in significant respiratory compromise History: dyspnea, pleuritic chest pain, hemoptysis Physical examination: tachypnea, rhonchi, wheezing, signs of blunt chest trauma (e.g., seat belt sign, chest wall contusions) Tests: Pulse oximetry will show hypoxemia ABG will show increased CO2 and an increased A-a gradient CXR shows diffuse opacification of the lungs as a result of inflammation (may not be seen for 4 to 6 hours after injury) Chest CT demonstrates extensive parenchymal contusion and inflammation but is typically not required for diagnosis Treatment: supportive care consisting of careful fluid management, respiratory therapy, and adequate analgesia (consider regional nerve blocks); multitrauma patients and those with significant contusions frequently require ICU care with mechanical ventilation Outcomes: symptoms can resolve within a week if treated appropriately; some patients will develop chronic dyspnea and decreased respiratory capacity Why eliminated from differential: diffuse opacification is not seen on CXR in this case; follow-up CXR would be performed during the hospital stay in this case to monitor PTX resolution and pleural stability, and any sign of progressive lung field opacification should raise immediate concern for an evolving pulmonary contusion
Case 2-14: “My little boy has a horrible cough” A mother brings her 3-year-old boy to the pediatrician because he has had a barking cough for 2 days. She says that he goes to day care 5 days per week and that some of the other children have had similar symptoms. She became very concerned when two mothers said that their children were hospitalized for observation because they were having problems breathing. The boy's mother says that her son does not seem to be having a problem breathing, but that his cough is very loud. He has a runny nose and has had a couple of axillary temperatures up to 101°F in the past 2 days. His voice is a little hoarse, but he has no problem talking to her. His mother says that he has had colds in the past, but none have had this type of cough. She has given him acetaminophen, which has seemed to keep him comfortable. She denies any other past medical history or medication use besides children's vitamins. On examination, he is found to have a red nose and is sniffing often. He has minimal pharyngeal erythema. There are no pharyngeal exudates. He has mild cervical lymphadenopathy. Auscultation of his lungs finds some mildly coarse breath sounds but no wheezing or rales. His heart sounds are normal. His abdomen is nontender with no masses. The following vital signs are measured: T: 100.7°F, HR: 100 bpm, BP: 95/45 mm Hg, RR: 20 breaths/min
Differential Diagnosis Bacterial pharyngitis, tonsillitis, croup, epiglottitis, bronchiolitis, aspirated foreign body
Laboratory Data and Other Study Results Rapid streptococcal antigen test: negative Throat swab culture: negative (results reported after initiation of therapy) CXR: mild subglottic narrowing of airway; lungs clear without infiltrates; no epiglottic swelling
Diagnosis Croup
Treatment Administered Supportive care consisting of hydration, expectorant/cough suppression medication, analgesia, rest, and humidified air was prescribed
Follow-up The patient continued to have similar upper respiratory symptoms without worsening for an additional 2 days but then had a rapid improvement The child had one other similar episode when he was 3 years old that was managed in a similar fashion but no other episodes after that time
Stridor at rest in children is an indication for hospitalization and inpatient observation.
Symptoms of severe cases of croup are remembered by the three Ss: Seal-bark cough, Subglottic swelling, and Stridor.
Steps to the Diagnosis Croup Acute inflammation of the larynx due to parainfluenzae virus type 1 (most common), parainfluenzae virus 2 or 3, respiratory syncytial virus (RSV), influenza virus, rubeola, adenovirus, or Mycoplasma pneumoniae Most common between 3 months to 5 years of age History: barking cough, nasal congestion, dyspnea, possible inspiratory stridor in worse cases
Figure 2-14 Chest x-ray of a child with croup demonstrating subglottic narrowing of the airway, which is reminiscent of the shape of a steeple (steeple sign).
Physical examination: fever, mild pharyngeal erythema, lymphadenopathy, possible respiratory distress in severe cases Tests: x-ray of the neck may show a “steeple sign” (i.e., subglottic narrowing of the airway) but is typically unneeded for the diagnosis unless concern exists for a more serious condition (Figure 2-14) Treatment: Supportive care (e.g., hydration, analgesia, humidified air, rest) Inhaled epinephrine or corticosteroids may be indicated for severe cases Patients typically are treated as an outpatient with expected self-resolution, but a concern for respiratory distress should prompt a hospital admission for observation Outcomes: Children with moderate to severe respiratory symptoms are at risk for developing hypoxemia and respiratory distress Patients may be predisposed to developing secondary bacterial infection Recurrent cases are possible but cease following 5 years of age Clues to the diagnosis: History: barking cough, nasal congestion, hoarseness Physical: mildly coarse breath sounds, lymphadenopathy Tests: subglottic narrowing on CXR
The most common causes of stridor in children are listed in the acronym ABCDEFGH: Abscess of tonsil, Bacterial pharyngitis/tracheitis, Croup, Diphtheria, Epiglottitis (all with fever), Foreign body, Gas poisoning, Hypersensitivity reaction (all without fever).
Great care must be taken during throat examination in suspected epiglottitis because additional throat irritation or augmentation of anxiety may cause complete respiratory occlusion; throat examination should be avoided in patients with significant symptoms. Bacterial pharyngitis/tonsillitis More thorough discussion in prior case Why eliminated from differential: absence of significant pharyngeal erythema, tonsillar exudates, or high fever in the case make this diagnosis less likely Epiglottitis Rapidly progressive infection of the epiglottis and surrounding pharyngeal tissue that may cause airway obstruction Most commonly due to Haemophilus influenzae type B infection but also caused by other types of H. influenzae or streptococcus Most common from 2 to 7 years of age History: dysphagia, drooling, mild stridor, muffled voice, sore throat, anxiety from symptoms Physical examination: fever, inspiratory retractions, leaning forward to help with breathing, “sniffing” neck flexion posture, erythematous, and swollen epiglottis
Figure 2-15 Lateral chest x-ray of a child with epiglottitis demonstrating a swollen epiglottis that resembles a thumb print (thumb print sign).
Tests: Neck x-ray will show a swollen, opacified epiglottis with partial airway obstruction on the lateral view (i.e., thumbprint sign) (Figure 2-15) Use of a laryngoscope may allow better visualization of the epiglottis but is frequently contraindicated because of the risk of worsening airway obstruction Culture from swab of the epiglottis (only after intubation) can detect causative bacteria Treatment: Keep the child calm and avoid increasing anxiety Admit the child for close observation and respiratory monitoring Unless airway obstruction is very mild, the child should be intubated (nasotracheal) to control the airway (typically for 3 to 5 days) Antibiotics as directed by throat cultures for 7 to 10 days Severe airway obstruction preventing intubation requires emergent tracheostomy Outcomes: most children recover without long-term airway complications; most mortalities are a result of delayed presentation or cardiopulmonary arrest during inappropriate pharyngeal examination Why eliminated from differential: no thumbprint sign is seen on x-rays in the case; the child in this case lacks the muffled voice, anxiety with breathing, and dysphagia that would be expected for this diagnosis
Widespread use of the Hib vaccine has vastly decreased the incidence of epiglottitis.
Bronchiolitis Viral infection of the bronchioles due to RSV (most common) or parainfluenzae virus type 3 Most common in children younger than 2 years old in spring and winter History: nasal congestion, cough, various degrees of respiratory distress depending on the severity Physical examination: low grade temperatures, tachypnea, wheezing, rales, prolonged expiratory phase, hyperresonance to percussion Tests: CXR shows hyperinflation of the lungs and patchy infiltrates; virologic testing is available but rarely utilized Treatment: Supportive care (e.g., hydration, humidified air, rest) Inhaled bronchodilators and corticosteroids may improve symptoms but have not been proven to be of benefit Ribavirin may be used in severe cases to shorten the disease course Children with respiratory distress or significant hypoxemia should be admitted to the hospital for observation and respiratory therapy Outcomes: typically self-limited; recurrent cases may predispose a child to the development of asthma Why eliminated from differential: the child in the case is on the older end of the age range for bronchiolitis; the absence of wheezing on examination and the absence of infiltrates on the CXR further help to rule out this diagnosis
The right main bronchus is the most common site of aspiration given its greater vertical orientation off of the trachea compared to the left side. Aspirated foreign body Accidental aspiration of an object that becomes lodged in the lower respiratory tract and obstructs the airway Food is common in all age groups; toys, coins, and other small objects are common in children History: possible witnessed aspiration, cough, choking, stridor (progressing to respiratory distress with worsening obstruction), hoarseness Physical examination: decreased breath sounds, wheezing, cyanosis Tests: Neck x-ray may show foreign body if it is radiopaque CXR may show atelectasis or the foreign body Flexible bronchoscopy may be used to visualize object Treatment: Actively coughing patients should be encouraged to remain calm and continue coughing to dislodge the object The Heimlich maneuver should be used to attempt to dislodge an aspirated object in a child or adult in
respiratory distress Rigid bronchoscopy with forceps should be used to remove the object in patients not in distress; the administration of IV corticosteroids prior to extraction may ease removal by decreasing bronchial inflammation Bronchoscopy should be repeated following removal of the object to check for retained particles or a second foreign body Emergent tracheostomy may be required in a patient with a prolonged obstruction Outcomes: symptoms resolve, and complications are avoided with prompt removal; foreign bodies left in place for extended periods of time (e.g., weeks) may lead to local infection, atelectasis, postobstructive pneumonia, or bronchiectasis Why eliminated from differential: the patient presentation is more consistent with an infectious cause (e.g., URI symptoms, fever); a more sudden onset of symptoms in a healthy child would be expected for a foreign body aspiration
Case 2-15: “Newborn in respiratory distress” A 26-year-old woman is in her 29th week of gestation in an uncomplicated pregnancy. She is involved in a motor vehicle crash in which she is a restrained driver. She is brought to the emergency department for evaluation and found to have bruising across her abdomen and vaginal bleeding. An ultrasound performed by an obstetrician consultant confirms a diagnosis of abruptio placentae, and an urgent caesarian delivery is scheduled because of fluctuations in the mother's vital signs and signs of fetal distress demonstrated by ultrasound and fetal monitoring findings. An uncomplicated caesarian delivery is performed for the 29-week gestation girl, who is initially in stable condition. The neonate is admitted to the neonatal ICU (NICU) for observation given her significant prematurity. Six hours following the delivery the child begins to develop progressive tachypnea and cyanosis. On further examination, the neonate appears to be in respiratory distress. Auscultation detects diffuse rales and faint breath sounds. A soft systolic murmur is faintly audible. The child is moving all extremities and withdraws to stimuli. The following vital signs are measured: T: 99.0°F, HR: 125 bpm, BP: 70/40 mm Hg, RR: 65 breaths/min (Normal for neonate: HR: 100–170 bpm, BP: 65–95/30–60, RR: 30–50 breaths/min)
Differential Diagnosis Respiratory distress of the newborn, meconium aspiration syndrome, transient tachypnea of the newborn, congenital cardiopulmonary malformation
Laboratory Data and Other Study Results Pulse oximetry: 86% on room air, 91% on air hood ABG (room air): pH: 7.30, pO2: 86 mm Hg, pCO2: 52 mm Hg, Bicarb: 20 mEq/L, O2 sat: 85% Blood cultures: negative Gram stain CXR: small lung volumes; bilateral infiltrates; diffuse ground-glass appearance of the parenchyma; air bronchograms Echocardiogram: no patent foramen ovale or ductus arteriosus; normal valves; normal wall motion; normal vasculature
Diagnosis Respiratory distress of the newborn
Treatment Administered Supplemental O2 was administered via air hood to keep O2 saturation as high as possible Surfactant was administered via a nebulizer Close cardiopulmonary monitoring and parenteral nutrition were continued while the child remained in the NICU Empiric ampicillin and gentamicin were started but discontinued upon negative blood culture results
Follow-up The newborn was stabilized in the NICU with a significant improvement in pulmonary function by 48 hours after the delivery The newborn remained in the NICU for an additional 10 weeks until reaching the predicted term age The child was transferred to the pediatrics floor and eventually was able to be discharged to home with her parents
Steps to the Diagnosis Respiratory distress of the newborn Respiratory distress in the neonate that presents soon after birth In almost all cases it is due to surfactant deficiency in preterm neonates (24 to 34 weeks gestation) causing lung immaturity with an associated impaired lung compliance, atelectasis, and respiratory collapse History: respiratory distress within the initial 48 hours following birth Physical examination: cyanosis, nasal flaring, intercostal retraction, tachypnea (>60 breaths/min), rales, decreased breath sounds Tests: ABG shows increased CO2 and decreased O2 CXR shows bilateral atelectasis, reticulogranular infiltrates (ground-glass appearance), and decreased lung volumes Amniotic fluid testing performed between 32 and 34 weeks gestation to measure amniotic lecithin/sphingomyelin ratio (should be >2) can guide treatment Treatment: NICU admission required for close observation Supplemental O2 to maximize O2 saturation Surfactant replacement therapy Intubation should be performed for any neonate not responding to therapy and requiring high levels of supplemental O2 to maintain O2 saturation Maternal corticosteroid administration should always be performed prior to 34 weeks of gestation (lung immaturity is assumed) and is not typically performed after 34 weeks of gestation (lung maturity is assumed) Outcomes: Neonates given prompt treatment in the NICU setting tend to have good outcomes
Neonates are at increased risk to develop pneumonia until their lungs mature Children with a history of respiratory distress of the newborn are more likely to develop asthma than other children Clues to the diagnosis: History: preterm delivery Physical: tachypnea and respiratory distress developing after the delivery, rales, cyanosis Tests: hypoxemia and hypercapnia, appearance of the lungs on CXR
If a tachypneic neonate does not respond to supplemental O2 within hours of initiation, a more extensive work-up should be performed to look for another diagnosis. Meconium aspiration syndrome Aspiration of meconium (i.e., fetal stool passed into the amniotic sac) prior to birth resulting in airway obstruction and the development of pneumonia History: meconium-stained amniotic fluid seen during delivery, postterm delivery Physical examination: cyanosis, intercostals retractions, chest distention, tachypnea Tests: CXR shows atelectasis and lung hyperinflation; blood culture should be considered to rule out sepsis Treatment: Supplemental O2 Intubation and ventilation for increasing respiratory distress Surfactant therapy may be beneficial Consider empiric antibiotic therapy if concerned for neonatal pneumonia Outcomes: typically good outcomes; children may be at a higher risk to develop asthma later in life Why eliminated from differential: the gestational age in the case is inconsistent with that typically seen in this diagnosis (preterm vs. postterm); the CXR in the case shows decreased lung volumes instead of hyperinflation Transient tachypnea of the newborn Self-limited pulmonary edema that occurs soon after birth as a result of a delayed clearance of fetal lung fluid History: preterm infants or those delivered by caesarian section Physical examination: tachypnea with an onset within 2 hours of birth, cyanosis, intercostals retractions, normal breath sounds Tests: ABG may demonstrate mildly decreased O2 and increased CO2; CXR will show hyperinflation with flat diaphragms and possible fluffy infiltrates Treatment: supplemental O2 Outcomes: symptoms tend to resolve within hours of the initiation of treatment Why eliminated from differential: the abnormal breath sounds, CXR with decreased lung volumes, and gradual response to O2 therapy make this diagnosis less likely than respiratory distress of the newborn
Pneumonia More thorough discussion in prior case Why eliminated from differential: negative culture results make this diagnosis much less likely; the ground-glass appearance of the CXR is more consistent with respiratory distress of the newborn than with neonatal pneumonia Congenital cardiopulmonary malformation More thorough discussion in Chapter 1 Why eliminated from differential: the normal echocardiogram rules out a congenital defect as the cause of the symptoms
Case 2-16: “Something happened while he was swimming” A 16-year-old adolescent boy is brought to the emergency department by EMTs after becoming confused while swimming. The boy's parents state that their son was swimming in the middle of a large lake bordering their property while training for a triathlon. The boy's father was watching him from their old motorboat in the lake. When the patient decided to return to shore, he chose to swim behind the boat for the trip. The boy's father says that he was swimming comfortably 20 feet behind the boat at first, but about 10 minutes into his return, he began to struggle. When his father turned the boat around to see what was wrong, he found his son to be confused. After being helped into the boat the boy complained about a bad headache and feeling very dizzy. The father called the local emergency medical service from the boat, and they were brought directly to the hospital. The patient is slightly sleepy but is able to answer questions. He is complaining about a diffuse headache, feeling dizzy, feeling nauseous, and aching everywhere. He has a difficult time recalling what happened while he was swimming. His parents say that their son is a strong swimmer and that this has never happened previously. They deny any medical problems or medication use. They are unaware of any substance use. On examination, the patient is mildly somnolent but is in no distress. His skin is pink without cyanosis. His neck is supple without lymphadenopathy. Auscultation finds tachypneic breathing with slightly coarse breath sounds. No abnormal heart sounds are present besides tachycardia. His abdomen is nontender without masses. Neurologic examination detects very brisk reflexes but no weakness or paresthesias. Funduscopic examination finds bright red retinal vessels and mild papilledema. The following vital signs are measured: T: 98.9°F, HR: 116 bpm, BP: 100/70 mm Hg, RR: 24 breaths/min
Differential Diagnosis Near drowning, diabetic ketoacidosis, labyrinthitis, carbon monoxide poisoning, meningitis, substance overdose, migraine or cluster headache
Laboratory Data and Other Study Results Pulse oximetry: 96% on room air CBC: WBC: 10.5, Hgb: 15.4, Plt: 410 Chem7: Na: 137 mEq/L, K: 3.5 mEq/L, Cl: 107 mEq/L, CO2: 20 mEq/L, BUN: 28 mg/dL, Cr: 1.4 mg/dL, Glu: 105 mg/dL Liver function test (LFT): alkaline phosphate (AlkPhos) 130 U/L, alanine aminotransferase (ALT): 50 U/L, aspartate aminotransferase (AST) 40 U/L, total bilirubin (TBili): 1.1 mg/dL, direct bilirubin (DBili) 0.4 mg/dL UA: straw-colored, pH: 5.0, specific gravity: 1.010, no glucose/ketones/nitrites/leukocyte esterase/hematuria /proteinuria Toxicology screen: negative for any illicit substances
ABG (room air): pH: 7.32, pO2: 98 mm Hg, pCO2: 33 mm Hg, Bicarb: 18 mEq/L, O2 sat: 99% Carboxyhemoglobin (HgbCO): 28% CXR: minimal pulmonary congestion; no effusions or consolidated infiltrates Head CT: no focal lesions; normal cerebral size; no hemorrhages
Diagnosis Carbon monoxide poisoning
Treatment Administered 100% O2 was initiated via face mask The patient was transferred to the hyperbaric medicine department and placed in a hyperbaric oxygen chamber to continue his O2 therapy
Follow-up The patient experienced a significant improvement in his mental status after the initiation of O2 therapy The patient was transferred to the observation unit of the emergency department A repeat ABG with HgbCO analysis demonstrated a decrease in HgbCO to 5% and improvement of his metabolic acidosis The patient was able to be discharged to home
Steps to the Diagnosis Carbon monoxide poisoning Carbon monoxide intoxication resulting in hypoxemia and tissue ischemia Carbon monoxide displaces O2 on Hgb molecules and prevents O2 delivery to tissues May be caused by prolonged exposure to car or boat engine fumes, smoke, furnace or heater exhaust, or chemicals (e.g., paint thinner, solvents) History: sufficient exposure to carbon monoxide, headache, dizziness, confusion, nausea, myalgias, dyspnea, chest pain, anxiety, amnesia, visual disturbances Physical examination: mental status changes, tachycardia, tachypnea, possible hypotension, possible cherry-red lips, abnormal retinal findings on funduscopic examination (e.g., papilledema, flame hemorrhages, red retinal veins), brisk reflexes, decreased vision acuity Tests: Mild increased WBC, possible decreased K, increased glucose, increased BUN and Cr, increased CK ABG shows normal pO2, mildly decreased CO2, mild metabolic acidosis, and minimally decreased O2 saturation Carboxyhemoglobin levels are increased CXR is typically normal Head CT may show cerebral edema in severe cases
Treatment: 100% O2 speeds the displacement of carbon monoxide from Hgb Hyperbaric O2 therapy is useful in patients who exhibit mental status changes or signs of severe intoxication Patients with severe intoxication or smoke inhalation may require intubation to facilitate O2 delivery Outcomes: cardiac ischemia, significant metabolic acidosis, coma, and very high HgbCO levels follow significant poisoning and are associated with a poor outcome; abnormalities on head CT are associated with permanent neurologic injury Clues to the diagnosis: History: exposure to boat exhaust, confusion, dizziness, headache, myalgias, amnesia Physical: tachycardia, tachypnea, papilledema, red retinal vessels, hyperreflexia Tests: increased BUN and creatinine, hyperglycemia, metabolic acidosis, increased HgbCO
Any patient with significant thermal burns, burns of the face, or exposure to large quantities of smoke should be worked-up for carbon monoxide poisoning and thermal airway injury.
Pulse oximetry and measurement of pO2 on an ABG suggest O2 levels that are much higher than the true levels in carbon monoxide poisoning.
Drowning is most common in children younger than 5 years and in males between 15 to 25 years old. Near drowning Hypoxemia resulting from submersion in some type of fluid (most commonly water) Aspiration of any type of water causes pulmonary damage (e.g., decreased lung compliance, ventilationperfusion mismatch, shunting) and cerebral hypoxia Pathophysiology of late-stage drowning varies by fluid type Fresh water: hypotonic fluid is absorbed from the alveoli into the vasculature, resulting in decreased electrolyte concentrations and red blood cell lysis Salt water: hypertonic fluid creates an osmotic gradient that draws fluid from the pulmonary capillaries into alveoli and causes pulmonary edema and increased serum electrolyte concentrations History: prolonged submersion, dyspnea, vomiting, coughing Physical examination: decreased consciousness, cyanosis, tachypnea Tests: Pulse oximetry shows decreased O2 saturation ABG demonstrates hypoxemia CXR shows significant pulmonary infiltrates and edema
Treatment: Secure the airway and initiate resuscitation Supplemental O2 should be administered A nasogastric tube should be placed to prevent further aspiration The body temperature should be maintained at normal levels Aggressive diuresis and bronchodilator therapy may be required to decrease water load and to improve oxygenation Outcomes: complications include pneumonia, ARDS, neurologic injury, and multiorgan failure; up to 60% of patients requiring cardiopulmonary resuscitation on arrival to the emergency department will die Why eliminated from differential: the pulse oximetry and ABG findings are not consistent with near-drowning in this case, and the CXR does not show pulmonary congestion Diabetic ketoacidosis (DKA) More thorough discussion in Chapter 5 Why eliminated from differential: serum glucose is minimally elevated, and the urinalysis does not demonstrate glucose or ketones Labyrinthitis More thorough discussion in Chapter 7 Why eliminated from differential: this diagnosis features dizziness but cannot explain the mental status changes or increased HgbCO levels Meningitis More thorough discussion in Chapter 7 Why eliminated from differential: a similar clinical presentation may be seen as for carbon monoxide poisoning, but the lack of a fever and a normal WBC count make this diagnosis less likely; a lumbar puncture could be performed to further rule out the diagnosis if concern still exists Substance overdose More thorough discussion in Chapter 13 Why eliminated from differential: the negative toxicology studies rule out this diagnosis Headache syndrome More thorough discussion in Chapter 7 Why eliminated from differential: these diagnoses do not account for the change in mental status, tachypnea, funduscopic findings, or the increased HgbCO
Authors: Van Kleunen, Jonathan P. Title: Step-Up to USMLE Step 3, 1st Edition Copyright ©2009 Lippincott Williams & Wilkins > Table of Contents > Chapter 3 - Gastroenterology
Chapter 3 Gastroenterology Basic clinical primer Gastrointestinal Absorption Digestion and the absorption of nutrients occurs along the digestive tract from the stomach through the colon (Figure 3-1) Digestion is initiated by saliva in the mouth and esophagus The stomach creates an acidic environment to augment initial digestion and to provide a site for gastrin and pepsin to act The majority of nutrients are absorbed in the small bowel The colon is an important site of water reabsorption
Hepatobiliary System Bile production Bile is produced by the liver and stored in the gallbladder Bile is released from the gallbladder into the duodenum to function as a fat emulsifier Bilirubin transport Unconjugated bilirubin is a product of red blood cell (RBC) hemolysis and exists in the venous circulation Once unconjugated bilirubin enters the hepatic circulation, it is conjugated by glucuronosyltransferase in the hepatocytes Conjugated bilirubin reenters the venous circulation Increases in either or both types of bilirubin depend on the site of pathology
Prehepatic conditions cause an increase in indirect bilirubin; posthepatic conditions cause an increase in direct bilirubin; intrahepatic conditions may cause an increase of either or both types of bilirubin.
Preoperative Hepatic Risk Assessment Perioperative and postoperative mortality are greater with increased levels of bilirubin, decreased albumin, hepatic coagulopathy, and liver-related encephalopathy
Attempts should be made to correct biliary and hepatic dysfunction prior to the time of surgery Surgery should be avoided (unless necessary) in patients with significant hepatitis, cirrhosis, or extrahepatic manifestations of liver disease
Liver and Pancreas Transplantation Liver Indicated for chronic hepatitis B or C, alcoholic cirrhosis, primary biliary cirrhosis, primary sclerosing cholangitis, biliary atresia, and progressive hepatic parenchymal disease (e.g., Wilson disease, hemochromatosis) Contraindicated in active alcoholism, multiple suicide attempts, liver cancer, and cirrhosis from chronic hepatitis (uninfected donors) P.73
Figure 3-1 The location of absorption of vitamins, minerals, and nutrients throughout the gastrointestinal tract. (See color image.)
Forty percent of patients experience acute rejection Five-year survival is 60% to 70% and correlates with the preoperative health of the recipient Pancreas Indicated for severe diabetes mellitus (DM) type I or parenchymal pancreatic disease Contraindicated for age >60 years, coronary artery disease (CAD), peripheral vascular disease (PVD), obesity, and DM type II
Acute rejection is common in most patients Three-year survival is 80%
Individuals with hepatitis may be used as donors for patients with existing hepatitis as long as no significant hepatic injury has occurred.
Case 3-1: “I just can't seem to swallow my food” A 43-year-old woman patient presents to a gastroenterologist on referral for progressive difficulty with swallowing. She says that over the past several years she has experienced progressive difficulty with swallowing both solid food and liquids. More recently, she has had episodes of coughing during eating, and she feels like she is aspirating small amounts of food during these episodes. She occasionally gets heartburn. She feels that she had been P.74 losing weight because of the difficulty in consuming food. She denies nausea, vomiting, chest or throat pain, or hoarseness. She had a tubal ligation performed 2 years ago but denies any other medical or surgical history. She takes aspirin (ASA) and a multivitamin daily. She denies any substance use. On examination, she is a thin woman in no distress. No neck masses or lymphadenopathy are detectable. Auscultation detects no abnormal lung or heart sounds. Abdominal examination detects no tenderness or masses. She has normal bowel sounds. Neurologic examination, including commands to swallow, is normal. The following vital signs are measured: Temperature (T): 98.5°F, heart rate (HR): 72 beats per minute (bpm), blood pressure (BP): 105/74 mm Hg, respiratory rate (RR) 16 breaths/min
Differential Diagnosis Esophageal cancer, laryngeal cancer, gastroesophageal reflux disease (GERD), achalasia, diffuse esophageal spasm, Zenker diverticulum, scleroderma
Laboratory Data and Other Study Results Barium swallow: dilated esophagus with acute taper at the lower esophageal sphincter (LES) Esophagogastroduodenoscopy (EGD): no detectable masses; no evident dysplastic tissue; no gastrointestinal (GI) ulcerations or lesions Based on these findings, the following test is performed: Esophageal manometry: increased LES pressure; decreased LES relaxation during swallowing
Diagnosis Achalasia
Treatment Administered Nifedipine was prescribed for the patient but discontinued because of hypotension Endoscopic pneumatic dilation of LES was performed
Follow-up The patient had substantial improvement of her dysphagia and was able to consume solids and liquids with little difficulty
Steps to the Diagnosis Achalasia Neuromuscular disorder of the esophagus characterized by impaired peristalsis and decreased relaxation of the LES resulting in dysphagia Due to dysfunction of intramural esophageal neurons Most commonly an idiopathic condition affecting patients 20 to 60 years old Dysphagia Difficulty swallowing due to dysfunctional pharyngeal or esophageal transport or pain during swallowing (i.e., odynophagia) Causes include neuromuscular disorders (e.g., achalasia, scleroderma) and obstruction (e.g., peptic strictures, esophageal webs or rings, neoplasm, radiation fibrosis) Obstruction leads to difficulty in swallowing solids only; neuromuscular disorders cause a difficulty in swallowing solids and liquids P.75 Barium swallow, esophageal manometry, and EGD are useful tools for making a diagnosis History: progressive dysphagia of solids and liquids, cough, regurgitation, aspiration, heartburn, weight loss from decreased food intake Physical examination: typically noncontributory Tests: Barium swallow will demonstrate a dilated esophagus with a “bird's beak” taper at the site of the LES (Figure 3-2) EGD should be performed to rule out the presence of esophageal masses or obstruction Esophageal manometry will demonstrate increased LES pressure, incomplete relaxation of the LES, and impaired peristalsis Treatment: Nitrates and calcium channel blockers may help to relax the LES but are limited by their vasodilatory effects Endoscopic injection of botulinum toxin or pneumatic dilation provides relief for most patients Myotomy may be performed in refractory cases Outcomes: most patients undergoing pneumatic dilation have good outcomes; esophageal rupture is a risk for EGD or dilation Clues to the diagnosis: History: difficulty swallowing both solids and liquids, microaspirations Physical: noncontributory Tests: barium swallow and esophageal manometry findings
Secondary causes of achalasia include Chagas disease, neoplasm, and scleroderma.
A barium swallow is the first study that should be performed in the diagnosis of dysphagia because of its low risk and diagnostic potential.
Nitrates and calcium channel blockers are effective in only 10% of patients with achalasia but are safer than esophageal dilation.
Figure 3-2 (A) Barium swallow in a patient with achalasia. Note the distended proximal esophagus with distal tapering and “bird's beak” sign (white arrow). (B) Illustration of achalasia demonstrating tapering of the distal esophagus. (See color image.)
P.76
Figure 3-3 (A) Barium swallow in a patient with diffuse esophageal spasm. Note the “corkscrew” pattern throughout the visible esophagus. (B) Illustration of diffuse esophageal spasm demonstrating a twisting “corkscrew” pattern. (See color image.)
Diffuse esophageal spasm (DES) Neuromuscular disorder in which the esophagus contracts in a nonperistaltic fashion History: dysphagia, chest or throat pain Physical examination: typically noncontributory Tests: barium swallow demonstrates a “corkscrew” pattern (Figure 3-3); esophageal manometry demonstrates uncoordinated, nonperistaltic contractions Treatment: nitrates, calcium channel blockers, tricyclic antidepressants, and botulinum injections may help reduce dysphagia and chest pain by relaxing esophageal muscles; myotomy may be considered in refractory cases Outcomes: prognosis is variable and different patient will respond better to different therapies; esophageal rupture is a risk of myotomy Why eliminated from differential: the absence of chest pain and the appearance of the barium swallow help to rule out this diagnosis
Nitrates may relieve pain due to DES but will worsen the symptoms of GERD. Zenker diverticulum An outpouching in the upper posterior esophagus due a region of smooth muscle weakness History: difficulty initiating swallowing, regurgitation of food hours or days after consumption, occasional dysphagia, feelings of aspiration, weight loss due to difficulty eating Physical examination: bad breath, possible neck fullness Tests: barium swallow will demonstrate an outpouching off of the esophagus (Figure 3-4) P.77
Figure 3-4 (A) Barium swallow in a patient with a small Zenker diverticulum (white arrow). (B) Illustration of a Zenker diverticulum demonstrating outpouching of the esophagus. (See color image.)
Treatment: surgical cricopharyngeal myotomy or diverticulectomy Outcomes: surgery has a high rate of successful alleviation of the condition; vocal cord paralysis, mediastinitis, and recurrence are possible complications of surgery Why eliminated from differential: bad breath and delayed regurgitation are not seen in this case, and the barium swallow did not detect a diverticulum
Esophageal cancer Squamous cell carcinoma (most commonly) or adenocarcinoma (less common) of the esophagus Adenocarcinoma is commonly preceded by the development of Barrett esophagus (i.e., columnar metaplasia of the distal esophagus secondary to chronic GERD) Risk factors: alcohol, tobacco, chronic GERD, obesity (adenocarcinoma only) History: progressive dysphagia of solids initially and both solids and liquids later, weight loss, odynophagia, regurgitation, vomiting, weakness, cough, hoarseness, hematemesis Physical examination: possible lymphadenopathy Tests: Barium swallow will show narrowing of the esophagus and the presence of an abnormal mass (Figure 3-5) Magnetic resonance imaging (MRI), computed tomography (CT), or positron emission tomography (PET) scan may be used to determine the extent of the disease EGD is performed to locate the lesion and perform a biopsy Biopsy provides a histologic diagnosis of the lesion Treatment: Photodynamic therapy with optional laser thermal ablation may be used to treat Barrett esophagus and prevent further progression Total esophagectomy is performed for early stage cancers Radiation therapy and chemotherapy are used as adjuncts to surgery or as primary therapies in unresectable cases P.78
Figure 3-5 Barium swallow in a patient with squamous cell carcinoma of the esophagus. Note the irregularity of the left esophageal wall due to the neoplastic mass. (See color image.)
Outcomes: esophageal leakage, myocardial infarction (MI), arrhythmias, atelectasis, and pneumonia are potential surgical complications; nonmetastatic disease is associated with up to a 25% 5-year survival rate, while the 5-year survival for metastatic disease is only 5% Why eliminated from differential: the presentations of esophageal cancer and achalasia are very similar, but the absence of a lesion on EGD and the esophageal manometry findings help to rule out cancer Laryngeal cancer Squamous cell cancer of the larynx associated with tobacco and alcohol use History: progressive hoarseness, dysphagia, ear pain, hemoptysis, weight loss Physical examination: decreased neck mobility, neck mass Tests: Laryngoscopy is used to visualize the lesion Biopsy is used to confirm the histologic diagnosis MRI is used to detect a soft tissue mass and to measure its extent
PET scan may be useful to detect early lesions or metastases Treatment: Partial or total laryngectomy is performed to remove lesions confined to the larynx Radiation therapy may be used as an adjunct to surgery or as a single therapy in cases of extensive disease Advanced lesions may require surgery, radiation, and chemotherapy Outcomes: five-year survival rates are 60% to 85% Why eliminated from differential: although the patient has a history of dysphagia and weight loss, the absence of hoarseness decreases the concern for this condition; in the absence of the barium swallow findings a laryngoscopy should be performed Gastroesophageal reflux disease More thorough discussion in later case Why eliminated from differential: although regurgitation, cough, dysphagia, and aspiration may be seen with GERD, the severity of dysphagia seen in this case is typically P.79 not experienced; esophageal manometry demonstrates an increased LES tone instead of insufficient LES tone Scleroderma More thorough discussion in Chapter 9 Why eliminated from differential: although esophageal dysmotility is seen in scleroderma, the absence of other systemic sclerosis findings rules out this diagnosis
Case 3-2: “My heartburn really bothers me” A 54-year-old man presents to his primary care provider (PCP) with the complaint of chronic heartburn not responding to medication. He says that he has had intermittent heartburn for several years that had previously responded to over-the-counter antacids. These episodes of heartburn had become more frequent over the past year and occasionally were fairly painful, so he was started on omeprazole. He reports that although this medication has helped improve his symptoms somewhat, he is continuing to have occasional moderate heartburn. He describes this sensation as a burning feeling behind his sternum. He says that his symptoms are worse after eating spicy foods, drinking alcohol, or bending over. He denies dysphagia, odynophagia, dyspnea, hematemesis, melena, hematochezia, or radiation of his discomfort away from his sternum. He states that he has a history of hypercholesterolemia and hypertension (HTN) for which he takes ASA, simvastatin, and losartan. He drinks approximately eight beers per week and three cups of coffee every morning. He acknowledges that he has done poorly with following lifestyle recommendations that were made at his previous visit. On examination, he is an obese man in no acute distress. He has no lymphadenopathy and swallows normally. Auscultation detects no abnormal heart or lung sounds. Abdominal examination detects no masses or tenderness. He has normal bowel sounds. The following vital signs are measured: T: 98.8°F, HR: 86 bpm, BP: 126/85 mm Hg, RR: 17 breaths/min
Differential Diagnosis Achalasia, esophageal cancer, gastritis, peptic ulcer disease, GERD, hiatal hernia, MI
Laboratory Data and Other Study Results Urea breath test: negative
Barium swallow: apparent sliding hiatal hernia of moderate size EGD: no detectable esophageal masses, erosions, or dysplastic areas; moderate lower esophageal irritation
Diagnosis Sliding hiatal hernia with secondary GERD
Treatment Administered The patient was encouraged to adopt the lifestyle changes of raising the head of his bed, decreasing his alcohol and caffeine intake, losing weight, and exercising The patient was placed on an aggressive twice daily dose of esomeprazole
Follow-up The patient had further improvements in his symptoms but not complete resolution of his occasional heartburn After 6 months of therapy without complete resolution of his symptoms, the patient underwent a Nissen fundoplication The patient responded well to surgical treatment with significantly improved symptoms P.80
Steps to the Diagnosis Hiatal hernia Herniation of a section of the stomach above the level of the diaphragm Sliding type: gastroesophageal junction and stomach are displaced above the diaphragm (95% of cases) Paraesophageal type: stomach protrudes through the diaphragm, but the gastroesophageal junction remains fixed in the correct location History: most patients are asymptomatic, but some patients with sliding hernias may have symptoms of GERD that are refractory to normal therapy Physical examination: typically noncontributory, but symptoms are more common in obese or pregnant patients Tests: chest x-ray (CXR) or barium swallow is useful for detecting the section of the stomach that has herniated through the diaphragm (Figure 3-6) Treatment: sliding hernias frequently respond to H2 antagonists or protein pump inhibitors (PPIs); paraesophageal hernias and sliding hernias with refractory GERD symptoms may be treated with gastropexy or Nissen fundoplication Outcomes: complications include incarceration of the stomach with paraesophageal hernias and the sequelae of chronic GERD with sliding hernias Clues to the diagnosis: History: heartburn not responding to medication Physical: noncontributory Tests: barium swallow findings
GERD symptoms are usually absent in patients with paraesophageal hiatal hernias. Gastroesophageal reflux disease Insufficiency of the LES leading to an abnormal reflux of gastric contents into the esophagus Risk factors: obesity, hiatal hernia, pregnancy, scleroderma, tobacco use, consumption of alcohol, caffeine, or fatty foods
Figure 3-6 Detection of a sliding type hiatal hernia with x-ray. (A) Anteroposterior view of a hiatal hernia (arrows) with partial protrusion of the stomach above the diaphragm. (B) Lateral view of the same hiatal hernia.
P.81 History: burning chest pain (“heartburn”) approximately 30 minutes after eating, worsening of pain with reclining or bending over, sour taste in the mouth, regurgitation, dysphagia, odynophagia, nausea, cough Physical examination: typically noncontributory Tests: The diagnosis is frequently clinical and does not require invasive testing Esophageal pH monitoring will detect decreased pH if measured over a 24-hour period Esophageal manometry detects decreased LES tone EGD or barium swallow may be performed to rule out esophageal cancer or Barrett esophagus Treatment: Elevation of the head of the bed, weight loss, and decreasing the consumption of substances that
precipitate symptoms (e.g., alcohol, caffeine, fatty foods) are all recommended lifestyle modifications Antacids are initially used on an as-needed basis (Table 3-1) H2 antagonists are prescribed if antacids are insufficient to control symptoms and are effective in 70% to 80% of cases of mild esophagitis PPIs are used in more severe cases and are highly effective in most cases Promotility agents may improve symptoms by speeding gastric emptying Refractory disease may be treated surgically with Nissen fundoplication Outcomes: the vast majority of patients will respond to pharmacologic treatment; complications of chronic disease include esophageal ulceration, esophageal stricture, Barrett esophagus, and adenocarcinoma Why eliminated from differential: GERD does occur in this patient and explains his collection of symptoms, but the failure of pharmacologic therapy to alleviate his symptoms points to an underlying cause of his GERD
Symptoms of GERD may mimic asthma or a MI. Peptic ulcer disease (PUD) More thorough discussion in later case Why eliminated from differential: the predominance of esophageal symptoms, the negative urea breath test, and the negative EGD findings rule out this diagnosis Gastritis More thorough discussion in later case Why eliminated from differential: the predominance of esophageal symptoms, the negative urea breath test, and the negative EGD findings rule out this diagnosis Achalasia More thorough discussion in prior case
Table 3-1 Medications Used in Treatment of Gastroesophageal Reflux Disease
Presc ription Medic ation
Antacids (e.g., calcium
Mec hanism
Neutralize gastric acid
Adverse Effec ts
Constipation, nausea, diarrhea
carbonate, aluminum
Strategy
Initial therapy, as needed
hydroxide)
H2 antagonists (e.g.,
Reversibly block histamine H2
Headache, diarrhea, rare thrombocytopenia;
Patients not
cimetidine, ranitidine)
receptors to inhibit gastric acid
cimetidine may cause gynecomastia and
responding to
secretion
impotence
antacids
PPIs (e.g., omeprazole,
Irreversibly inhibit parietal cell
Well tolerated; may increase effects of warfarin,
Patients not
lansoprazole)
proton pump (H+/K+ ATPase) to
benzodiazepines, phenytoin, digoxin, or
responding to
block gastric acid secretion
carbamazepine in some patients
antacids
Promotility agents (e.g.,
Promote gastric emptying
Headache, diarrhea, cardiac effects
cisapride)
Patients with poor LES function
GERD, gastroesophageal reflux disease; H+ , hydrogen ion; K+ , potassium ion; LES, lower esophageal sphincter; PPIs, proton pump inhibitors.
P.82 Why eliminated from differential: the absence of significant dysphagia makes this diagnosis unlikely, and the barium swallow findings further confirm the true diagnosis Esophageal cancer More thorough discussion in prior case Why eliminated from differential: the negative EGD findings and absence of a mass on barium swallow rule out this diagnosis Myocardial infarction More thorough discussion in Chapter 1 Why eliminated from differential: the quality of chest pain in this case is more consistent with reflux than a cardiac cause; the frequent recurrence of heartburn, relation to eating, and response to antacid drugs further help to suggest a GI pathology
Case 3-3: “I get burning pains every time I eat” A 63-year-old man presents to his PCP with the complaint of burning epigastric pain that occurs soon after eating on a fairly regular basis. He says that this pain begins within an hour after meals. He also describes mild nausea accompanying this pain. He had two episodes during the prior week in which he vomited due to this nausea and noticed some brown discoloration to his emesis. He says that these symptoms have been occurring for the past 2 months. He denies any gross blood during episodes of emesis, any blood in his stool, or weight loss. He has gotten some relief from over-the-counter antacids and ranitidine. He describes a history of significant osteoarthritis in both of his knees and takes naproxen and ASA as medications. He has approximately four alcoholic drinks per week and denies tobacco use. On examination, he is in no distress. He does not exhibit lymphadenopathy. Auscultation detects no abnormal lung or heart sounds. Abdominal examination detects mild tenderness on epigastric palpation, no masses, no guarding, and no rebound tenderness. The following vital signs are measured: T: 98.5°F, HR: 78 bpm, BP: 130/85 mm Hg, RR: 16 breaths/min
Differential Diagnosis Gastritis, peptic ulcer disease, Zollinger-Ellison syndrome, pancreatitis, GERD, gastric cancer
Laboratory Data and Other Study Results Complete blood cell count (CBC): white blood cells (WBC): 7.4, hemoglobin (Hgb): 13.5, platelets (Plt): 290 7-electrolyte chemistry panel (Chem7): sodium (Na): 135 mEq/L, potassium (K): 3.9 mEq/L, chloride (Cl): 106 mEq/L, carbon dioxide (CO2): 26 mEq/L, blood urea nitrogen (BUN): 20 mg/dL, creatine (Cr): 0.5 mg/dL, glucose (Glu): 95 mg/dL Coagulation panel (Coags): protime (PT) 10.9, international normalized ratio (INR): 1.0, partial thromboplastin time (PTT) 24.7
Amylase: 100 U/L Lipase: 90 U/L Urea breath test: negative Helicobacter pylori antibody screen: negative Barium swallow: small uniform focal pooling of contrast within lesser curvature of stomach; no esophageal or duodenal abnormalities Following these tests, the additional following studies are ordered: EGD: small ulceration on lesser curve of stomach; biopsy of ulcer performed; no active bleeding from ulcer; no esophageal or duodenal abnormalities Ulcer biopsy: fibrous tissue with infiltration of monocytes; no dysplastic cells P.83
Diagnosis Gastric ulcer (peptic ulcer disease) secondary to nonsteroidal anti-inflamatory drug (NSAID) overuse
Treatment Administered The patient was prescribed regular ranitidine and lansoprazole Naproxen was discontinued, and the patient was started on celecoxib for arthritis pain
Follow-up The patient experienced a gradual improvement in his symptoms with a complete resolution of epigastric pain in 4 months
Causes of peptic ulcer disease may be remembered by the mnemonic ANGST HAM: Aspirin, NSAIDs, Gastrinoma (ZES), Steroids, Tobacco, H. pylori, Alcohol, MEN type I.
Steps to the Diagnosis Peptic ulcer disease Erosion of the gastric or duodenal mucosal surface secondary to impaired endothelial protection and increased gastric acidity (Color Figure 3-1) Duodenal ulcers are more common and are most often a result of H. pylori infection (Table 3-2) Gastric ulcers are less common and are commonly due to NSAID overuse or H. pylori infection (Table 3-2) Risk factors: H. pylori infection, NSAIDs, tobacco, alcohol, corticosteroids History: periodic burning epigastric pain that may vary in intensity following meals, nausea, hematemesis, melena, hematochezia Physical examination: epigastric tenderness, and acute perforation of an ulcer is associated with abdominal rigidity and rebound tenderness
Tests: CBC is useful for detecting anemia due to active bleeding A positive urea breath test or positive immunoglobulin G (IgG) antibodies to H. pylori helps diagnose the respective infection Abdominal x-ray (AXR) will detect free abdominal air in cases of perforation; a barium swallow may detect collections of barium within ulcerations EGD is performed to visualize ulcers, assess bleeding, and perform biopsies to rule out cancer Treatment: PPIs and H2 antagonists are used to decrease gastric acid levels and prevent further mucosal damage
Table 3-2 Distinguishing between Gastric and Duodenal Ulcers
Charac teristic
Gastric Ulc er
Duodenal Ulc er
Patients
Age >50 years, Helicobacter pylori infection, NSAID users
Younger, H. pylori infection
Frequency
25% cases
75% cases
Timing of pain
Soon after eating
2–4 hours after eating
Gastric acid level
Normal/low
High
Gastrin level
High
Normal
Effect of eating
May w orsen symptoms and cause nausea and vomiting
Initial improvement in symptoms, with later w orsening
NSAIDs, nonsteroidal anti-inflammatory drugs.
P.84 Sucralfate, bismuth subsalicylate, and misoprostol may be useful to aid in mucosal protection H. pylori infection is eliminated by multidrug regimens (e.g., clarithromycin, amoxicillin, and a PPI and metronidazole, tetracycline, bismuth, and a PPI for 14 days are two common regimens) Partial gastrectomy may be required for severe hemorrhage due to erosion of an ulcer into the gastric vasculature; parietal cell vagotomy or antrectomy may be considered in refractory cases Outcomes: complications include significant hemorrhage (the gastroduodenal artery in particular with posterior ulcers), perforation (most common in anterior ulcers), and lymphoproliferative disease; uncomplicated ulcers typically respond very well to medical therapy Clues to the diagnosis: History: epigastric pain and nausea following eating, clotted blood in emesis, response to antacid medication, frequent use of NSAIDs and ASA
Physical: epigastric tenderness Tests: low normal Hgb, barium swallow findings, EGD, and biopsy findings
Peptic ulcers may develop secondary to the systemic inflammatory response seen with severe burns (i.e., Curling ulcers), intracranial injuries (i.e., Cushing ulcers), or multisystem trauma, so all trauma patients with significant injuries should receive prophylactic ranitidine.
Barium swallow radiographic findings that suggest the presence of a malignant lesion associated with an ulcer include abnormal-appearing mucosal folds in the region of the ulcer, the presence of a mass near the ulcer, and irregular filling defects in the base of the ulcer.
Patients younger than 40 years with symptoms suggestive of PUD for a short duration of time may be worked-up with noninvasive testing; an EGD is required to rule out gastric malignancy in older individuals or those with symptoms lasting longer than 2 months.
Patients with refractory PUD that does not respond to PPI therapy should undergo fasting gastrin and secretin-stimulation testing to determine if they have Zollinger-Ellison syndrome. Gastritis Inflammation of the gastric mucosa without full mucosal penetration Acute variant: rapidly developing superficially erosive lesions in any region of the stomach that are related to NSAID use, alcohol, corrosive substance ingestion, or stress from severe illness Chronic variant: nonerosive lesions most common in the antrum or fundus of the stomach (Table 3-3) History: possibly asymptomatic or epigastric pain, indigestion, nausea, vomiting, hematemesis, or melena Physical examination: frequently normal with occasional epigastric tenderness and bad breath Tests: A positive urea breath test or positive IgG antibodies to H. pylori help diagnose this infection Analysis of pepsinogen isoenzymes is useful to the detection of autoimmune disease (decreased pepsinogen I/pepsinogen II ratio in disease) EGD allows visualization of the mucosa and biopsy Gastric biopsy may help differentiate causes of chronic disease Treatment: Acute disease is treated in a similar fashion to PUD (e.g., stopping offending medications, prescribing PPIs and H2 antagonists)
Table 3-3 Variants of Chronic Gastritis
Charac teristic
Ty pe A (Autoimmune)
Ty pe B (Helicobacter pylori)
Ty pe C (Chemic al)
Site
Fundus
Antrum
Antrum
Pathology
Autoantibodies for parietal cells
H. pylori infection
induce gastric atrophy
Labs
Chronic chemical irritation (e.g., NSAIDs) or biliary reflux
Decreased gastric acid level,
Increased gastric acid
decreased gastrin
level, increased gastrin
May be normal
Associated
Pernicious anemia, achlorhydria,
Peptic ulcer disease,
Peptic ulcer disease, incompetent pyloric
conditions
thyroiditis
gastric cancer
sphincter, abnormal intestinal motility
NSAIDs, nonsteroidal anti-inflammatory drugs.
P.85 Chronic gastritis due to H. pylori is treated with regimen similar to that for PUD Chemical chronic gastritis is treated by identifying and stopping use of the offending agent Autoimmune chronic gastritis requires vitamin B12 replacement Sucralfate, bismuth subsalicylate, and misoprostol may be useful for mucosal protection Outcomes: complications include PUD, hemorrhage, pernicious anemia (autoimmune gastritis), and gastric cancer (H. pylori gastritis) Why eliminated from differential: although gastritis and PUD may have similar presentations, the barium swallow and EGD findings are confirmatory of the presence of an ulcer
In pernicious anemia, autoantibodies destroy parietal cells, leading to low levels of intrinsic factor, vitamin B12 malabsorption, and megaloblastic anemia. Zollinger-Ellison syndrome Syndrome of refractory PUD and malabsorption due to the presence of a gastrin-producing tumor History: refractory PUD, abdominal pain, nausea, vomiting, indigestion, diarrhea, steatorrhea Physical examination: possible epigastric tenderness, hepatomegaly, jaundice Tests: Increased fasting gastrin Administration of secretin causes an excessive increase in serum gastrin levels (i.e., secretin-stimulation test) Venous gastrin sampling may be useful to localize the tumor Single-positron emission computed tomography (SPECT) with somatostatin receptor imaging may be useful for tumor localization Angiography may detect tumor hypervascularity
Treatment: Surgical resection is indicated for nonmetastatic disease (easiest for extrapancreatic lesions) PPIs and H2 antagonists may help mollify symptoms Octreotide is used to improve symptoms in metastatic disease Outcomes: Occasionally associated with other endocrine tumors (e.g., multiple endocrine neoplasia 1 [MEN1]) Sixty percent of tumors are malignant Hemorrhage and perforation due to severe PUD Nonmetastatic disease has an excellent prognosis Why eliminated from differential: the good response of the patient to a regimen of lansoprazole and ranitidine without recurrence make this diagnosis unlikely
Stop PPI use prior to gastrin level testing to ensure an accurate measurement. Gastric cancer Adenocarcinoma (more common) or squamous cell carcinoma (rare, due to esophageal invasion) involving the stomach Ulcerating: dysplastic irregular ulcers that may be confused for PUD Polypoid: large intraluminal neoplasms Superficial spreading: mucosal and submucosal involvement only Linitis plastica: all layers of the stomach are involved; impaired stomach elasticity Risk factors: H. pylori infection, family history, Japanese heritage and living in Japan, tobacco, alcohol, vitamin C deficiency, high consumption of preserved foods, male gender History: weight loss, anorexia, early satiety, vomiting, dysphagia, epigastric pain Physical examination: abdominal mass, left supraclavicular lymphadenopathy (i.e., Virchow node), periumbilical lymphadenopathy (i.e., Sister Mary Joseph node) Tests: Increased carcinoembryonic antigen (CEA) Increased 2-glucuronidase in gastric secretion analysis Barium swallow may show a mass or a thickened “leather bottle” stomach in the case of linitis plastica P.86 EGD is used to visualize ulcerations and masses and to perform biopsies Biopsy is required to provide a histologic diagnosis Treatment: subtotal gastrectomy is performed for tumors in the distal third of the stomach, and total gastrectomy is performed for more proximal or invasive tumors; chemotherapy and radiation therapy are performed as adjuncts to surgery or alone for extensive disease
Outcomes: Superficial spreading type has the best prognosis, linitis plastica has the worst Early detection has cure rates up to 70%, but prognosis is poor for any delays in detection (<15% 5-year survival) Why eliminated from differential: the results of the biopsy indicate that the ulcer in this patient is not cancerous
The most common locations for a gastrinoma are the duodenum (70% cases) and the pancreas. Gastroesophageal reflux disease More thorough discussion in prior case Why eliminated from differential: epigastric pain and a response to antacid drugs may be seen in both conditions, but the barium swallow and EGD results are confirmatory of PUD Pancreatitis More thorough discussion in later case Why eliminated from differential: both conditions may cause epigastric pain, but the normal amylase and lipase levels and barium swallow and EGD findings rule out this diagnosis
Case 3-4: “This diarrhea is driving me crazy!” A 43-year-old man presents to his PCP with a complaint of six episodes of bloody diarrhea since the previous evening. He also describes achy abdominal pain. He denies any dizziness, syncope, weakness, numbness, arthralgias, vision symptoms, or hematemesis. He says that he went to a picnic 5 days ago and believes that he must have eaten something that caused these symptoms because some of the other people who attended the picnic have been having the same complaints. He denies previous episodes of bloody diarrhea and any past medical history. He denies any medication or illicit substance use. On examination, he appears to be uncomfortable but not in any distress. He has a pink skin color. He has no palpable lymphadenopathy. Auscultation detects hyperactive bowel sounds but no abnormal lung or heart sounds. He has mild abdominal pain on deep palpation and no masses, rebound tenderness, or rigidity. His neurologic examination is normal. His pulses are bounding, and he has quick capillary refill. The following vital signs are measured: T: 100.2°F, HR: 90 bpm, BP: 110/78 mm Hg, RR: 17 breaths/min
Differential Diagnosis Bacterial gastroenteritis, viral gastroenteritis, parasitic gastroenteritis, celiac sprue, inflammatory bowel disease (IBD), diverticulitis
Laboratory Data and Other Study Results CBC: WBC: 11.2, Hgb: 15.9, Plt: 290 Fecal leukocytes: positive Stool culture: no parasites visible in microscopic examination; culture pending Clostridium difficile stool assay: negative
Diagnosis Bacterial gastroenteritis P.87
Treatment Administered The patient was strongly encouraged to drink liquids to maintain his hydration The patient was started on empiric ciprofloxacin
Follow-up The patient experienced some improvement in the degree of diarrhea after the initiation of antibiotics Stool culture grew Campylobacter jejuni, so the prescribed antibiotic was changed to erythromycin The patient experienced a further improvement in his symptoms, and the diarrhea stopped by two days after beginning treatment
Steps to the Diagnosis Bacterial gastroenteritis Infection of the GI tract due to a bacterial cause that results in some type of GI disturbance (e.g., vomiting, diarrhea) (Table 3-4)
Table 3-4 Common Pathogens in Bacterial Gastroenteritis
Pathogen
Bacillus cereus
Sourc e
Fried rice
Signs and Sy mptoms
V omiting within several hours
Treatment
Self-limited; hydration
of eating, diarrhea later
Campylobacter jejuni
Poultry (second most common
Bloody diarrhea, abdominal
Hydration, erythromycin; generally
food-bourne bacterial GI infection)
pain, fever; rare Guillain-Barré
self-limited
syndrome
Clostridium botulinum
Clostridium difficile
Honey, home-canned foods
Nausea, vomiting, diarrhea,
Botulism antitoxin (not given to
flac c id paraly sis
infants); self-limited
Antibiotic -induc ed suppression
W atery diarrhea; gray
Metronidazole, vancomycin
of normal colonic flora
pseudomembranes seen on colonic mucosa
Escherichia coli
Food/water (i.e., travelers'
W atery diarrhea, vomiting,
(enterotoxigenic)
diarrhea)
fever
Hydration; self-limited
E. coli type O157:H7
Ground beef, indirect fecal
Bloody diarrhea, vomiting,
Hydration; self-limited; antibiotics
(enterohemorrhagic)
contamination
fever, abdominal pain, risk of
may actually worsen symptoms due
HUS
to toxin release
Staphylococcus aureus
Room-temperature food
V omiting, within several hours
Self-limited; hydration
of eating, diarrhea later
Salmonella species
Eggs, poultry, milk, fresh produce
Nausea, abdominal pain, bloody
Hydration; self-limited; treat
(most c ommon food-bourne
diarrhea, fever, vomiting
immunocompromised patients with
bac terial GI infec tion)
Shigella species
Vibrio cholerae
fluoroquinolone
Food/water; associated with
Fever, nausea, vomiting,
Hydration; self-limited;
overcrowding
severe bloody diarrhea,
ciprofloxacin, TMP-SMX in severe
abdominal pain, risk of HUS
cases
Copious w atery diarrhea,
Hy dration; tetracycline or
signs of dehydration
doxycycline decrease disease
Water, seafood
length
Vibrio parahaemolyticus
Seafood (e.g., oysters)
Abdominal pain, w atery
Hydration; self-limited
diarrhea within 24 hours of eating
Yersinia enterocolitica
Pork, produce
Abdominal pain, bloody
Hydration; self-limited
diarrhea, right lower quadrant pain, fever
GI, gastrointestinal; HUS, hemolytic uremic syndrome; TMP-SMX, trimethoprim-sulfamethoxazole.
P.88
Figure 3-7 Diagnostic and treatment pathways for acute diarrhea.
Contaminated food and water are the most common source of infection Diarrhea Frequency of bowel movements (stool production >200 g/day) and increased stool liquidity Acute diarrhea lasts <2 weeks and is usually due to infection (Figure 3-7) Chronic diarrhea lasts longer than 2 weeks and may be due to chronic infection, malabsorption, or dysfunctional GI motility (Figure 3-8)
Figure 3-8 Diagnostic pathway for chronic diarrhea.
P.89 Secretory diarrheas are usually hormone mediated or due to enterotoxic bacterial infections Osmotic diarrheas are due to an increased intraluminal solute concentration that leads to a decreased reabsorption of water Inflammatory diarrheas are due to an autoimmune process or chronic infection Diarrhea is common in immunocompromised patients (e.g., human immunodeficiency virus [HIV], chronic corticosteroid use) and is typically due to opportunistic pathogens History: nausea, diarrhea (watery or bloody), abdominal pain, possible vomiting Physical examination: perianal erythema, signs of dehydration (e.g., decreased skin turgor, dry mucous membranes), increased bowel sounds, rare neurologic deficits Tests:
Fecal leukocytes are present on stool analysis Stool assays exist for the detection of C. difficile Culture will provide an identification of the causative organism Colonoscopy may be useful to visualize pseudomembrane formation in C. difficile colitis Treatment: The majority of cases are self-limited and only require supportive care (particularly rehydration) Patients who are immunosuppressed, significantly dehydrated, have bloody diarrhea, or are very young or old are treated with empiric antibiotics pending culture results (typically fluoroquinolones) Species-appropriate antibiotics are prescribed when the pathogen is identified (14 days of therapy) Outcomes: prognosis is good with administration of the proper therapy; uncommon complications include chronic malabsorption, hepatic damage, and spread of infection to non-GI sites Clues to the diagnosis: History: acute bloody diarrhea, similar symptoms in other picnic attendees Physical: abdominal tenderness, hyperactive bowel sounds Tests: mildly increased WBC, positive stool leukocytes
Common pathogens involved in food poisoning are remembered by the mnemonic “Eating Contaminated Stuff Sometimes Causes Very Big Smelly Vomit”: Escherichia coli (enterotoxigenic, enterohemorrhagic), Campylobacter jejuni, Staphylococcus aureus, Salmonella, Clostridium botulinum, Vibrio cholerae, Bacillus cereus, Shigella, Vibrio parahaemolyticus.
Rotavirus is the most common cause of acute diarrhea in children.
Lactase deficiency is the most common cause of adult chronic diarrhea.
Hemolytic uremic syndrome (HUS) is a complication of E. coli 0157:H7 infection and is characterized by thrombocytopenia, hemolytic anemia, and acute renal failure; it is usually self-limited and occurs more commonly in children. Viral gastroenteritis Infection of the GI tract by a viral pathogen resulting in GI disturbance Common pathogens include Norwalk virus, Coxsackie virus A1, echovirus, adenovirus, and rotavirus History: nausea, vomiting, diarrhea, abdominal pain, myalgias Physical examination: low-grade fever, mild abdominal tenderness Tests: no fecal leukocytes are present in stool analysis; viral cultures may be performed but are rarely necessary
Treatment: supportive care only Outcomes: self-limited Why eliminated from differential: the presence of fecal leukocytes in the stool analysis rules out this diagnosis Parasitic gastroenteritis Infection of the GI tract by a protozoan or parasitic organism (Table 3-5) History: abdominal pain, diarrhea (watery, greasy, or bloody), malaise, myalgias Physical examination: fever, abdominal tenderness, signs of dehydration, rare neurologic deficits Tests: parasites are visualized in a stool sample; antibody tests exist for several pathogens Treatment: Supportive care (especially rehydration) Species-specific antiparasitic therapy (e.g., metronidazole, paromomycin) Surgical decompression and resection of necrotizing colitis or hepatic abscesses Outcomes: excellent prognosis with the appropriate treatment; possible complications include hepatic abscesses, intestinal obstruction, necrotizing colitis, and neurologic infection P.90
Table 3-5 Common Pathogens in Parasitic and Protozoan Gastrointestinal Infections
Pathogen
Sourc e
Signs and Symptoms
Giardia lamblia (Color
Surface water (usually limited to
Greasy , foul-smelling diarrhea; abdominal
Figure 3-2)
wilderness or other countries)
pain, malaise; cysts and trophozoites seen
Treatment
Metronidazole; hydration
in stool sample
Entamoeba histolytica
Water, areas of poor sanitation
Mild to severe bloody diarrhea, abdominal
Metronidazole,
pain; cysts and trophozoites seen in stool
paromomycin
sample
Cryptosporidium
Food/water; immunocompromised
W atery diarrhea, abdominal pain, malaise;
Control immune suppression;
parvum
patients
acid-fast stain of stool shows parasites
nitazoxanide
Trichinella spiralis
Undercooked pork
Fever, my algias, periorbital edema;
Albendazole, mebendazole if
eosinophilia
CNS or cardiac symptoms
Mild diarrhea, CNS sy mptoms
Praziquantel,
Taenia solium
Undercooked pork
corticosteroids if more than five cysts
CNS, central nervous system.
Why eliminated from differential: the absence of visible parasites on microscopic examination makes this diagnosis less likely
Celiac sprue More thorough discussion in later case Why eliminated from differential: diarrhea in this condition would be a more chronic condition, so this diagnosis is less likely for the given case presentation; the diagnosis could be further ruled out by performing testing for antiendomysial and antigliadin antibodies Inflammatory bowel disease More thorough discussion in later case Why eliminated from differential: this diagnosis would be expected to be a more chronic process, to have presented earlier in the patient's life, and also to feature multiple extraintestinal symptoms; colonoscopy and biopsy could be performed to aid in the diagnosis if concern for this condition existed Diverticulitis More thorough discussion in later case Why eliminated from differential: left lower quadrant pain and lower GI bleeding would predominate in this presentation, and diarrhea would not be common; CT could be performed to look for free air, diverticuli, and possible rupture if a strong clinical suspicion remained
Case 3-5: “My bowels just aren't right” A 37-year-old woman presents to a gastroenterologist on referral from her PCP for the work-up of chronic diarrhea and multiple other GI symptoms. The patient says that she has had frequent bouts of loose stools or watery diarrhea for the past 5 years and has become increasingly frustrated with attempts to change her diet in order to improve these symptoms. She also describes frequent abdominal cramping, flatulence, bloating, fatigue, and an itchy rash on her trunk. She has occasionally had problems maintaining her weight. She denies bloody diarrhea, rectal bleeding, nausea, vomiting, or arthralgias. She believes that her symptoms have stemmed from an intolerance of meat and tries to be a strict vegetarian. However, she says that her symptoms have not improved and actually have worsened with some vegetarian meals. She has a history of depression for which she takes fluoxetine but no other known medical conditions. She has undergone allergy testing that P.91 detected no apparent allergies. She denies tobacco, alcohol, and illicit substance use. She traveled to Mexico last year but was forced to cut her trip short because of her symptoms. On examination, she is a thin-appearing white woman in no distress. Chest auscultation detects no abnormal sounds. Her abdomen is mildly distended and hypertympanic, and auscultation detects hyperactive bowel sounds. No masses or organomegaly are present. She has a faint papular rash on her abdomen and anterior legs. A rectal examination detects no gross blood or masses. She has mild weakness on all strength testing but no paresthesias. The following vital signs are measured: T: 98.4°F, HR: 80 bpm, BP: 108/76 mm Hg, RR: 15 breaths/min
Differential Diagnosis Irritable bowel syndrome, celiac sprue, tropical sprue, lactose intolerance, IBD, gastroenteritis, laxative abuse, Whipple disease, pancreatitis
Laboratory Data and Other Study Results CBC: WBC: 6.5, Hgb: 11.5, Plt: 260 10-electrolyte chemistry panel (Chem10): Na: 142 mEq/L, K: 3.7 mEq/L, Cl: 106 mEq/L, CO2: 27 mEq/L, BUN: 20 mg/dL, Cr: 1.1 mg/dL, Glu: 80 mg/dL, magnesium (Mg): 1.7 mg/dL, calcium (Ca): 8.4 mg/dL, phosphorus (Phos):
3.0 mg/dL Parathyroid hormone (PTH): 6.7 pmol/L Amylase: 100 U/L Lipase: 210 U/L Nutrition panel: albumin: 3.6 g/dL, iron (Fe): 34 µg/dL, transferrin: 220 mg/dL, ferritin: 50 ng/mL Lactose tolerance and breath tests: negative D-xylose tolerance test: positive (decreased urinary excretion) Stool pH: 7.1 Stool microscopy: no fecal leukocytes; no parasites Based on these results, the following additional studies are performed: Barium enema: no abnormal masses or defects in colon Stool osmolality gap: 136 mOsm/kg Stool Sudan stain: positive for high fecal fat content Antigliadin antibodies: positive Antiendomysial antibodies: positive
Diagnosis Celiac sprue
Treatment Administered The patient was placed on a gluten-free diet The patient was referred to a nutritionist to help guide her diet planning
Follow-up The patient experienced a significant improvement in her symptoms with institution of the gluten-free diet and had resolution of her GI symptoms and fatigue
Steps to the Diagnosis Celiac sprue Genetic disorder of gluten (e.g., wheat, barley, rye) intolerance in which antiendomysial and antigliadin antibodies cause jejunal mucosal damage P.92 Occurs in a bimodal distribution with presentations within the 1st year or third decade of life Risk factors: European heritage (white) History: failure to thrive, bloating, loose stools or watery diarrhea, steatorrhea, weight loss, flatulence, fatigue, depression, anxiety Physical examination: weakness, abdominal distention, peripheral edema, papular rash on trunk and
extensor surfaces, orthostatic hypotension, possible Chovstek or Trousseau sign Tests: Decreased K, Mg, and Ca Albumin may be decreased Iron is frequently low with an associated decreased Hgb Sudan stain detects fat in stool (Figure 3-9) D-xylose tolerance test will show decreased urinary excretion Presence of antiendomysial and antigliadin antibodies in serum Jejunal biopsy demonstrates loss of duodenal and jejunal villi Small bowel follow through after barium swallow shows dilation of the jejunum and loss of the normal mucosal surface (will not be apparent on barium enema) Treatment: a gluten-free diet is the key to treatment and may be all that is needed to eliminate symptoms; corticosteroids may be required to decrease bowel inflammation in refractory cases Outcomes: patients who respond to a gluten-free diet have an excellent prognosis; patients with refractory disease tend to have chronic symptoms Clues to the diagnosis: History: chronic loose stools and diarrhea, flatulence, bloating, cramping, fatigue Physical: abdominal distention, hyperactive bowel sounds, papular rash Tests: mildly decreased electrolytes, low normal albumin, low iron, positive D-xylose test, anemia, increased stool osmolality gap, positive fecal fat, positive antigliadin, and antiendomysial antibodies
Classic symptoms of malabsorption syndromes include weight loss, bloating, diarrhea, steatorrhea, glossitis, dermatitis, and edema.
Celiac and tropical sprue exhibit the same symptoms, but only celiac sprue responds to removal of gluten from diet, and tropical sprue occurs in patients who have spent time in the tropics. Tropical sprue Malabsorption syndrome similar to celiac sprue with a likely infectious etiology Acquired disorder in patients spending time in the tropics Folate deficiency likely plays some role in the pathology
Figure 3-9 Diagnostic pathway for suspected malabsorption syndrome.
P.93 History: bloating, loose stools or diarrhea, fatigue, weight loss, steatorrhea, flatulence Physical examination: weakness, abdominal distention, peripheral edema, papular rash on trunk and extensor surfaces, orthostatic hypotension, possible Chovstek or Trousseau sign Tests: test profile similar to celiac sprue except no antiendomysial and antigliadin antibodies are detected Treatment: folate replacement and tetracycline for 6 months; gluten-free diet has no effect Outcomes: with folate and antibiotic therapy prognosis is generally good Why eliminated from differential: the patient in the case had symptoms prior to her trip to Mexico; the presence of antiendomysial and antigliadin antibodies confirms celiac sprue as the diagnosis Lactose intolerance Malabsorption syndrome resulting from a deficiency of lactase Undigested lactose causes an osmotic diarrhea A similar syndrome may also occur secondary to bacterial overgrowth or Crohn disease History: abdominal pain, flatulence, diarrhea, and bloating following dairy consumption Physical examination: typically noncontributory Tests: positive lactose breath test (i.e., increased hydrogen concentration in expired air following a lactose meal); positive lactose tolerance test (i.e., minimal increase in serum glucose concentration following a
lactose meal) Treatment: lactose-free or lactose-restricted diet with adequate nutrients; lactase replacement may be beneficial in some patients Outcomes: patients do very well when following the prescribed diet Why eliminated from differential: the negative lactose breath and tolerance tests rule out this diagnosis Irritable bowel syndrome (IBS) Idiopathic disorder of irregular bowel habits and chronic abdominal pain Most common in female patients during the second and third decades of life History: Abdominal pain, diarrhea or loose stools alternating with constipation, bloating, nausea, possible vomiting, and weight loss Manning criteria: the likelihood of a diagnosis of IBS increases with the number of satisfied criteria Abdominal pain relieved with defecation Stool frequency increases following onset of abdominal pain Stools become looser following the onset of abdominal pain Visible abdominal distention Passage of mucus with stool Feeling of incomplete defecation Physical examination: mild abdominal tenderness, abdominal distention Tests: no tests are diagnostic for IBS, but tests should be ordered to rule out other causes of chronic diarrhea and GI distress (e.g., malabsorption, neoplasms, IBD, infection) Treatment: Reassurance from the physician that a serious condition is not the cause of the symptoms A diet high in fiber and low in caffeine, lactose, and legumes may help to improve the symptoms Psychotherapy may be beneficial in patients with comorbid psychiatric conditions Antispasmodics, antidepressants, and serotonin receptor antagonists have demonstrated benefit in some patients Outcomes: IBS is frequently a chronic condition with multiple relapses, but associated with no significant complications Why eliminated from differential: evidence of a malabsorptive process (e.g., increased stool osmolality and presence of fecal fat) and the presence of antiendomysial and antigliadin antibodies rule out this condition
Half of the patients with IBS will also have a comorbid psychiatric condition. P.94 Whipple disease
A malabsorption disorder due to Tropheryma whippelli infection and an associated immunodeficiency that also effects several other organ systems Risk factors: white males with European ancestry History: weight loss, joint pain, abdominal pain, diarrhea, dementia, cough, bloating, steatorrhea Physical examination: fever, vision abnormalities, lymphadenopathy, new heart murmur, progressive wasting Tests: jejunal biopsy will show villous atrophy and foamy macrophages on periodic acid-Schiff stain Treatment: trimethoprim-sulfamethoxazole (TMP-SMX) or ceftriaxone for 12 months Outcomes: seventy percent of patients will have complete resolution of symptoms if treated (30% will have relapses); mortality is 100% within 1 year if untreated Why eliminated from differential: the absence of cardiac, central nervous system (CNS), and visual symptoms and the presence of antiendomysial and antigliadin antibodies rule out this condition
The characteristic features of Whipple disease may be remembered by the mnemonic WAD TAMP: Weight loss, Arthralgias, Diarrhea, Tropheryma whippelli, Abdominal pain, Multiple systems involved, Periodic acid-Schiff used for diagnosis. Inflammatory bowel disease More thorough discussion in later case Why eliminated from differential: the presentations for IBD and sprue may be similar, but the presence of antiendomysial and antigliadin antibodies is pathognomonic for celiac sprue; the normal barium enema also makes IBD an unlikely diagnosis Gastroenteritis More thorough discussion in prior case Why eliminated from differential: the chronicity of the symptoms, normal WBC count, and absence of leukocytes or organisms on microscopy make this diagnosis unlikely Laxative abuse Excessive use of laxatives in the attempt to prevent weight gain or to induce weight loss History: significant concerns over weight and body image, possible bingeing history, frequent stools or diarrhea, depression Physical examination: possible edema Tests: stool pH, osmolality, and fat content will be normal, but 24-hour stool weight will be increased Treatment: cessation of laxative use; psychotherapy or antidepressants may be useful to modify self-image and weight compensation behaviors Outcomes: long-term abuse may lead to chronic constipation, steatorrhea, and protein loss Why eliminated from differential: the increased stool osmolality and presence of fecal fat rule out this diagnosis Pancreatitis More thorough discussion in later case
Why eliminated from differential: although malabsorptive diarrhea may be present in chronic pancreatitis, the normal amylase and lipase levels seen in the case make this diagnosis unlikely
Case 3-6: “Can drinking give you diarrhea?” An 18-year-old man presents to a gastroenterologist on referral from an internist at his college's student health center for the work-up of bloody diarrhea. The patient is a freshman college student who initially sought treatment at the student health center for recurrent bouts of bloody diarrhea, abdominal pain, and painful bowel movements. He is worried that his heavy drinking during weekend parties has caused these symptoms. He says that the abdominal pain is crampy in nature and has occurred occasionally over the past 8 months. The diarrhea has been bloody in nature and occurs three or four times per week. P.95 He has formed bowel movements between these episodes, but they are frequently painful. This altered bowel behavior has developed over the past 6 months. Prior to the past 8 months he never exhibited any of these symptoms. He denies vomiting, constipation, or urinary symptoms. He denies any past medical history or medication use. He denies any similar history in family members. He drinks more than five alcoholic beverages on two to three nights per week but denies tobacco or illicit drug use. On examination, he appears to be a thin, white male in no acute distress. Auscultation of his heart and lungs finds no abnormal sounds. He has diffuse mild abdominal tenderness and no organomegaly. His bowel sounds are normal. A rectal examination detects occult blood. Neurologic examination is normal. The patient has multiple painful nodules on his anterior lower legs. The following vital signs are measured: T: 99.3°F, HR: 90 bpm, BP: 120/75 mm Hg, RR: 16 breaths/min
Differential Diagnosis Ulcerative colitis, Crohn disease, bacterial or parasitic gastroenteritis, celiac sprue, lactose intolerance, Whipple disease, IBD, carcinoid tumor
Laboratory Data and Other Study Results CBC: WBC: 12.5, Hgb: 13.0, Plt: 450 Chem7: Na: 139 mEq/L, K: 3.6 mEq/L, Cl: 106 mEq/L, CO2: 27 mEq/L, BUN: 18 mg/dL, Cr: 0.7 mg/dL, Glu: 84 mg/dL Erythrocyte sedimentation rate (ESR): 33 mm/hr C-reactive protein (CRP): 2.1 mg/dL Liver function tests (LFTs): alkaline phosphatase (AlkPhos): 120 U/L, alanine aminotransferase (ALT): 40 U/L, aspartate aminotransferase (AST): 20 U/L, total bilirubin (TBili): 1.0 mg/dL, direct bilirubin (DBili) 0.3 mg/dL Nutrition panel: albumin: 3.2 g/dL, Fe: 33 µg/dL, transferrin: 432 mg/dL, ferritin: 16 ng/mL Lactose tolerance and breath tests: negative D-xylose tolerance test: negative Stool microscopy: several polymorphonuclear cells (PMNs); no parasites or bacteria Following receipt of these results, the following tests are performed: Abdominal CT: no abscesses, fistulas, masses, or hemorrhage; mild colonic wall thickening Colonoscopy: highly friable colonic mucosa involving the full length of the colon without interruption; no pseudomembranes; biopsy performed
Colonic wall biopsy: inflammation of colonic mucosa with no extension to deeper layers of the bowel wall
Diagnosis Ulcerative colitis
Treatment Administered The patient was prescribed supplemental iron, mesalamine, and a short course of prednisone
Follow-up The patient had a gradual improvement in his symptoms after the initial treatment regimen The patient experienced one to two exacerbations per year requiring short-term corticosteroid therapy P.96 The patient continued to be followed closely for the development of disease sequelae The patient was counseled on responsible alcohol consumption
Characteristics of ulcerative colitis can be remembered by the mnemonic CECAL PLUMB: Continuous involvement, Extraintestinal symptoms (e.g., eyes, joints, skin, liver), Cancer risk, Abscesses in crypts, Large bowel only, Pseudopolyps, “Lead pipe” colon, Ulcerations, Mucosa depth, Bloody diarrhea.
Steps to the Diagnosis Ulcerative colitis (UC) Variant of inflammatory bowel disease (IBD) characterized by continuous involvement of a region of the colon (Table 3-6) Disease begins at the rectum and extends to a proximal end point within the colon Only the mucosal layer of the bowel is involved Typically begins in second or third decade of life Extraintestinal manifestations include arthritis, uveitis, ankylosing spondylitis, primary sclerosing cholangitis, erythema nodosum, and pyoderma gangrenosum Risk factors: white, Ashkenazi Jewish heritage History: abdominal pain, urgency of bowels, bloody diarrhea, painful bowel movements (i.e., tenesmus), nausea, vomiting, weight loss Physical examination: fever, abdominal tenderness, tachycardia, occult blood on rectal examination, orthostatic hypotension Tests: Serum WBCs are mildly increased, mild anemia, increased ESR and CRP, decreased albumin and iron, decreased K, positive perinuclear antineutrophil cytoplasmic antibodies (pANCA), antisaccharomyces cerevisiae antibodies (ASCA) rarely present Stool analysis will frequently detect PMNs
Barium enema shows a “lead pipe” colon without haustral markings and generalized shortening of the colonic length (Figure 3-10)
Table 3-6 Comparison of Crohn Disease and Ulcerative Colitis
Crohn Disease
Site of involvement
Symptoms
Entire GI trac t may be involved with multiple
Ulc erative Colitis
Continuous disease beginning at rec tum and
“skipped” areas; distal ileum most commonly involved;
extending possibly as far as distal ileum; only mucosa
entire bow el w all affec ted
and submucosa affected
Abdominal pain, weight loss, w atery diarrhea
Abdominal pain, urgency, bloody diarrhea, tenesmus, nausea, vomiting, weight loss
Physical
Fever, right lower quadrant abdominal mass, abdominal
Fever, abdominal tenderness, orthostatic hypotension,
examination
tenderness, perianal fissures and fistulas, oral ulcers
tachycardia, gross blood on rectal examination
Extraintestinal
Arthritis, ankylosing spondylitis, uveitis, primary
Arthritis, uveitis, ankylosing spondylitis, primary
manifestations
sclerosing cholangitis, nephrolithiasis
sclerosing cholangitis, erythema nodosum, pyoderma gangrenosum
Labs
Radiology
ASCA frequently positive, pANCA rarely positive; fecal
ASCA rarely positive, pANCA frequently positive; biopsy
occult blood test positive stool; biopsy diagnostic
diagnostic
Colonoscopy shows colonic ulcers, strictures, “c obble-
Colonoscopy shows c ontinuous involvement,
stoning,” fissures, and “skipped” areas of bowel;
pseudopolyps, friable mucosa; barium enema shows
barium enema shows fissures, ulcers, and bowel edema
“lead pipe” colon without haustra and colon shortening
Treatment
Mesalamine, broad-spectrum antibiotics,
Mesalamine, supplemental iron, corticosteroids,
corticosteroids, immunosuppressives; surgical resections
immunosuppressives; total c olec tomy is c urative
of severely affected areas, fistulas, or strictures
Complications
Abscess formation, fistulas, fissures, malabsorption,
Signific antly inc reased risk of c olon c anc er,
toxic megacolon
hemorrhage, toxic megacolon, bowel obstruction
ASCA, antiyeast Saccharomyces cerevisiae antibodies; GI, gastrointestinal; pANCA, perinuclear antineutrophil cytoplasmic antibodies.
P.97
Figure 3-10 Barium enema in a patient with ulcerative colitis. Complete loss of haustra is seen in the descending colon, sigmoid colon, and rectum (i.e., “lead pipe” colon). Haustra are maintained in the ascending and transverse colon.
Colonoscopy demonstrates a friable mucosa and pseudopolyps in continuous fashion Biopsy demonstrates inflammation limited to the mucosal layer of the bowel wall Treatment: Aminosalicylates (e.g., mesalamine, sulfasalazine) are the first line of therapy to suppress disease activity Corticosteroids may be used in patients with disease that is not responsive to aminosalicylates alone Patients not responding to the drugs listed above may benefit from immunomodulators (e.g., mercaptopurine, azathioprine, infliximab) Total colectomy is curative and may be performed in patients with disease that does not respond to medical treatments Outcomes: Exacerbations are common with 50% of patients having such an episode within 2 years of diagnosis Patients with limited disease tend to have progression of the bowel involvement over time After the initial decade of the disease, the risk of developing colon cancer increases by 1% per year Complications include the several extraintestinal manifestations, colon cancer, bowel obstruction, severe hemorrhage, and toxic megacolon
Clues to the diagnosis: History: bloody diarrhea, abdominal pain, age Physical: abdominal tenderness, occult blood on rectal examination, erythema nodosum Tests: mildly increased WBC, anemia, mild hypokalemia, increased ESR and CRP, decreased albumin, colonoscopy results Crohn disease Variant of IBD characterized by multiple sites of involvement throughout the gastrointestinal tract (Table 3-6) P.98 Entire GI tract may be involved with multiple skipped regions (i.e., intermittent normal bowel) The distal ileum is the most commonly involved region The entire bowel wall thickness is involved The disease has a bimodal distribution and typically begins in either the second or third decade of life or the seventh or eighth decade of life Extraintestinal manifestations include arthritis, uveitis, ankylosing spondylitis, primary sclerosing cholangitis, and nephrolithiasis Risk factors: white, Ashkenazi Jewish heritage History: abdominal pain, weight loss, watery diarrhea Physical examination: fever, abdominal pain (worse in the right lower quadrant), perianal fissure and fistulas, oral ulcers Tests: Serum WBCs are mildly increased, mild anemia, increased ESR and CRP, decreased albumin and iron, decreased K, rarely positive pANCA, positive ASCA Fecal occult blood testing of stool is frequently positive Stool analysis will frequently detect PMNs Barium enema displays fissures, ulcers, and edema of the bowel wall in an incomplete distribution (Figure 3-11) Colonoscopy shows colonic ulcers, strictures, “cobble-stoning,” fissures, and normal areas of colon between involved segments Treatment: Aminosalicylates are the first line therapy for the disease Antibiotics are frequently administered to prevent infection via the perianal disease Corticosteroids and immunomodulators are used in resistant disease Surgical resection of strictures, fissures, and fistulas may be required Outcomes: Exacerbations are common over the life of the disease Complications include the extraintestinal manifestations, abscess formation, fistula and fissure formation,
and toxic megacolon The increased risk of colon cancer is similar to that for ulcerative colitis in patients who have colonic involvement Why eliminated from differential: the presence of a bloody diarrhea, the continuous involvement of the colon seen on colonoscopy, and the histologic inflammation limited to the mucosa are more suggestive of UC as the diagnosis
Characteristics of Crohn disease can be remembered by the mnemonic CHRISTMAS: Cobble-stoning, High temperature (fever), Reduced lumen size, Intestinal fistulae, Skip lesions, Transmural involvement, Malabsorption, Abdominal pain, Submucosal fibrosis. Gastroenteritis (bacterial or parasitic) More thorough discussion in prior case Why eliminated from differential: the chronic nature of the condition and colonoscopy findings rule out these diagnoses Celiac sprue More thorough discussion in prior case Why eliminated from differential: the bloody diarrhea, normal D-xylose test, and colonic involvement (as opposed to just small bowel) seen on colonoscopy rule out this diagnosis Lactose intolerance More thorough discussion in prior case Why eliminated from differential: the negative lactose breath and tolerance tests help to rule out this diagnosis Irritable bowel disease More thorough discussion in prior case Why eliminated from differential: the bloody diarrhea, labs suggesting an inflammatory process, and colonoscopy findings rule out this diagnosis Whipple disease More thorough discussion in prior case Why eliminated from differential: the lack of CNS, pulmonary, and cardiac symptoms make this diagnosis unlikely; colonic involvement would not be expected for this diagnosis P.99 Carcinoid tumor Intestinal neoplasm that arises from neuroectodermal cells that function as amine-precursor-uptake and decarboxylation (APUD) cells Most commonly occurs in the appendix, ileum, rectum, and stomach History:
May be asymptomatic, but common symptoms include abdominal pain, fatigue, weight loss, and possible painless rectal bleeding Carcinoid syndrome is seen in 10% of cases Diarrhea, flushing, asthmatic symptoms (e.g., bronchoconstriction), heart murmurs (from tricuspid or pulmonary valve disease) Results from serotonin secretion by the tumor Physical examination: abdominal distension and tenderness are possible findings, but patients with carcinoid syndrome are notable for flushing, facial telangiectasias, wheezing, systolic heart murmurs, hepatomegaly, and lower extremity edema Tests: Increased serum serotonin Increased urine 5-HIAA (5-hydroxyindoleacetic acid) CT or indium-labeled octreotide scintigraphy may be used to localize the tumor Treatment: Tumors <2 cm are excised locally or via appendectomy Tumors >2 cm require a wide resection because of a high risk of metastases Metastatic disease is treated with chemotherapy, α-interferon (α-IFN), octreotide, or embolization Outcomes: full resection carries an excellent prognosis; the 5-year survival for metastatic disease is 67% Why eliminated from differential: the presence of diarrhea and the absence of the other findings seen in carcinoid syndrome make this diagnosis less likely; the colonoscopy results confirm that IBD is the cause of the patient's symptoms
The presentation for a carcinoid tumor is frequently confused with that for IBS, and the diagnosis is commonly delayed.
Case 3-7: “My belly hurts and I keep throwing up” A 36-year-old woman presents to the emergency department with a week-long history of abdominal pain and bilious vomiting. She said that she has had intermittent crampy abdominal pain over the past week that has gradually increased in frequency and intensity. She has had multiple episodes of bilious vomiting over the past few days. She is having difficulty eating because of the vomiting. She has begun to feel fatigued due to poor food consumption. She denies hematemesis, melena, hematochezia, back pain, urinary symptoms, or pelvic pain. She noted that she has not had a bowel movement for 4 days. Her last menstrual period ended 1 week ago. She reports a history of Crohn disease that has been well controlled on a regimen of sulfasalazine and mercaptopurine. She takes no other medications. She notes a history of two prior partial bowel resections related to her Crohn disease, the most recent of which was performed 1 year ago. She denies substance use. She uses condoms as contraception. On examination, she is thin, pale, and looks fatigued. She has no palpable lymphadenopathy. Auscultation of her heart and lungs detects no extra sounds and mild tachycardia. She has diffuse moderate abdominal tenderness that is more pronounced in the central portion of her abdomen. Her bowel sounds are high-pitched and difficult to hear. She has some voluntary guarding but no rebound tenderness. No organomegaly is detected. A rectal examination detects no occult blood. A pelvic examination detects no abnormal fluid, no pelvic masses, and no cervical motion tenderness. The following vital signs are measured:
T: 99.5°F, HR: 110 bpm, BP: 105/70 mm Hg, RR: 18 breaths/min
Differential Diagnosis Small bowel obstruction, large bowel obstruction, ileus, appendicitis, volvulus, intussusception, cholecystitis, diverticulitis, gastroenteritis, mesenteric ischemia, pancreatitis, endometriosis, pregnancy, ectopic pregnancy, pelvic inflammatory disease P.100
Laboratory Data and Other Study Results CBC: WBC: 11.5, Hgb: 13.1, Plt: 200 Chem10: Na: 144 mEq/L, K: 4.1 mEq/L, Cl: 105 mEq/L, CO2: 24 mEq/L, BUN: 25 mg/dL, Cr: 1.2 mg/dL, Glu: 80 mg/dL, Mg: 2.1 mg/dL, Ca: 10.1 mg/dL, Phos: 4.1 mg/dL LFTs: AlkPhos: 96 U/L, ALT: 25 U/L, AST: 21 U/L, TBili: 1.1 mg/dL, DBili: 0.3 mg/dL Lactate: 2.2 mEq/L Amylase: 54 U/L Lipase: 120 U/L ESR: 28 mm/hr CRP: 1.1 mg/dL UA: straw-colored, pH: 5.0, specific gravity: 1.010, no glucose/ketones/nitrites/leukocyte esterase/hematuria /proteinuria Urine pregnancy test: negative Fecal occult blood test (rectal): positive Neisseria gonorrhoeae and Chlamydia trachomatis testing of vaginal fluid: pending AXR: multiple dilated loops of small bowel with air-fluid levels; no free air Abdominal CT: dilation of proximal jejunum to >4 cm with a collapsed middle and distal jejunum; mild proximal jejunal bowel wall thickening; mesenteric fat fibrosis with stranding; no masses, abscesses, biliary tree abnormalities, appendoliths, appendiceal thickening, or hernias
Diagnosis Small bowel obstruction
Treatment Administered A nasogastric tube was inserted, immediate intravenous (IV) fluids were initiated, and the patient was kept NPO (nulla per orem [nothing by mouth]) The patient had continued intermittent abdominal pain and was taken to surgery the following morning for a lysis of adhesions obstructing a section of the jejunum
Follow-up The patient recovered following surgery with resolution of her symptoms and an improvement in her vital signs
The patient was able to eat food normally after resolution of a postoperative ileus The patient continued to be followed for her Crohn disease N. gonorrhoeae and C. trachomatis testing was negative
Small bowel obstruction is most commonly due to adhesion formation, while large bowel obstruction is most commonly due to neoplasm.
Steps to the Diagnosis Some causes of the acute abdomen may be remembered by the mnemonic SAUCED HIPPO: Strangulation of bowel, Appendicitis, Ulcer (peptic), Cholangitis, Ectopic pregnancy, Diverticulitis, Hemorrhage (GI), Ischemia (e.g., mesenteric, splenic, cardiac), Pancreatitis, PID (pelvic inflammatory disease), Obstruction. Small bowel obstruction (SBO) Mechanical or inflammatory obstruction of the bowel preventing the passage of food and leading to progressive local bowel ischemia Common causes include adhesions, incarcerated hernias, neoplasm, intussusception, volvulus, Crohn disease, and congenital strictures Progressive obstruction can present as an acute abdomen (i.e., severe abdominal pain and rigidity requiring emergent intervention) (Table 3-7) History: colicky abdominal pain, nausea, vomiting, constipation, possible history of previous abdominal surgery or intra-abdominal neoplasm Physical examination: abdominal tenderness, abdominal distention, visible peristaltic waves, high-pitched or absent bowel sounds, fever Tests: Increased BUN and Cr suggest dehydration due to obstruction; lactate is increased with strangulation or bowel ischemia P.101
Table 3-7 Causes of an Acute Abdomen
Condition
H/P
Obstruction/strangulation (due
Previous surgery, abdominal
CT or AXR shows distended
Surgic al ly sis of adhesions,
to adhesions, hernias, tumors)
distention, crampy pain,
loops of bowel and air-fluid
hernia repair, surgical
nausea, vomiting,
levels; barium studies may
excision of tumors
high-pitched bowel sounds
locate site of obstruction
Diverticulitis
Diagnosis
Left low er quadrant pain
CT or AXR may show free air
(may progress over several
from perforation; increased
days), blood in stool
WBC
Treatment
Surgical repair
Massive GI hemorrhage (e.g.,
Sudden severe pain,
Colonoscopy or EGD
Octreotide, angiography
perforation)
hematemesis,
visualizes lesion; technetium
w ith embolization, surgical
hematoc hezia, hypotension
scan may detect smaller
repair of detectable site of
bleeding sources
bleeding
Right low er quadrant and
Increased WBC; thickened
Appendec tomy
periumbilic al pain, psoas
appendix or fecalith on CT if
sign, rectal examination
unruptured; free air on CT
tenderness
or AXR if perforated
Severe abdominal pain out of
Bowel wall thickening and air
NPO, antibiotics, resection of
proportion to examination,
within bowel wall on CT;
necrotic bowel
bloody diarrhea
increased WBC and serum
Appendicitis
Mesenteric ischemia
lactate
Pancreatitis
Upper abdominal and bac k
CT shows inflamed pancreas;
Nasogastric tube, NPO,
pain, nausea, vomiting,
increased amylase and lipase
analgesics
Amenorrhea, lower abdominal
US unable to locate
Surgical excision
pain, possible vaginal bleeding,
intrauterine pregnancy in
or palpable pelvic mass
presence of positive urine
history of gallstones or alcoholism
Ruptured ectopic pregnancy
pregnanc y test
Pelvic inflammatory disease
Lower abdominal pain, vaginal
Increased WBC; positive
Antibiotics, treat sexual
discharge, cervical motion
serology for Chlamydia or
partners
pain
Neisseria gonorrhoeae
AXR, abdominal x-ray; CT, computed tomography; EGD, esophagogastroduodenoscopy; GI, gastrointestinal; H/P, history and physical; NPO, nothing by mouth; US, ultrasound; WBC, white blood cell count.
AXR will show multiple ladderlike dilated loops of bowel with air-fluid levels; the use of barium increases the sensitivity of the study (Figures 3-12) Abdominal CT can detect the location of an obstruction based on the presence of dilated bowel proximal to the obstruction and collapsed bowel distal to it; CT is also useful for detecting masses, abscesses, or herniations related to the obstruction Treatment: Keep patient NPO and administer IV fluids Nasogastric suction may decompress the bowel and relieve the obstruction in some simple cases Surgical correction of the obstruction (e.g., lysis of adhesions, hernia repair) is required if the obstruction is unable to be decompressed Outcomes: Prompt treatment is associated with a good prognosis SBOs treated nonsurgically have a higher recurrence rate than those treated surgically Complications include mesenteric ischemia, abscess formation, sepsis, and short-bowel syndrome following surgery Clues to the diagnosis:
History: abdominal pain, bilious vomiting, constipation, history of Crohn disease with previous abdominal surgeries Physical: abdominal tenderness, high-pitched bowel sounds Tests: mildly increased BUN and Cr, high normal lactate, appearance of the AXR and abdominal CT (consistent with Crohn disease) P.102
Figure 3-11 Barium enema in a patient with Crohn disease. Appreciable disease of variable severity is seen throughout the colon and distal ileum. Significant strictures are seen in the distal ileum and descending colon.
Figure 3-12 Abdominal radiograph in a patient with a small bowel obstruction. Note the multiple loops of dilated bowel with a ladderlike appearance.
P.103 Large bowel obstruction (LBO) Mechanical obstruction of the colon leading to constipation and possible bowel ischemia Common causes are neoplasm, diverticulitis, volvulus, and congenital strictures History: abdominal pain, obstipation, nausea, feculent vomiting (progressed stages) Physical examination: abdominal tenderness and distention, palpable abdominal mass, high-pitched or absent bowel sounds, hyperresonance to percussion Tests: Increased BUN and Cr suggest dehydration due to obstruction; lactate is increased with strangulation or bowel ischemia AXR shows significant colonic distention; barium enema may detect distal obstructions (Figure 3-13) Abdominal CT may show a mass causing the obstruction or a thickened bowel wall
Treatment: Keep patient NPO and administer IV fluids Colonoscopy may be used to reduce a LBO due to volvulus Surgical resection of the obstructed segment is frequently required Outcomes: Prognosis is good with early treatment When obstruction is due to neoplasm, the prognosis is dependent on the stage of the cancer Complications include abscess formation and perforation Why eliminated from differential: bilious vomiting is more suggestive of SBO; the abdominal CT demonstrates jejunal wall thickening and no obstructive masses
Figure 3-13 Abdominal radiograph in patient with large bowel obstruction due to sigmoid volvulus. Note the significantly dilated bowel lumen. Of further note is the dense line marking where the walls of two dilated loops of bowel are pressed against each other (open arrow) and the dense marking where a dilated loop of bowel is compressed against the cecum (solid arrow).
P.104
Ileus Paralytic obstruction of the bowel as a sequelae of decreased peristalsis Common causes are recent surgery, infection, ischemia, DM, and opioid use History: vague abdominal pain, nausea, vomiting, bloating, constipation, intolerance of food Physical examination: abdominal distention, decreased bowel sounds, no guarding or rebound tenderness Tests: AXR will show dilated bowel with possible air-fluid levels; barium enema can confirm the absence of a distal obstruction Treatment: Stop opioids and keep patient NPO Prokinetic agents and ambulation may speed recovery of bowel function Cases will self-resolve Outcomes: prognosis is excellent with uncomplicated self-resolution considered the norm Why eliminated from differential: the length of symptoms, severity of signs, and the radiographic findings suggest an actual obstruction and not just impaired peristalsis
Postoperative ileus typically lasts <5 days. Small bowel recovers in 24 hours, the stomach recovers in 48 to 72 hours, and large bowel recovers in 3 to 5 days. Appendicitis Inflammation of the appendix with an increased risk of perforation Due to lymphoid hyperplasia (common in children), fecalith impaction (adults), or fibroid bands (adults) Obstruction of the appendiceal lumen leads to infection and progressive inflammation History: dull periumbilical pain followed later by nausea, vomiting, anorexia, and migration of pain to the right lower quadrant Physical examination: tenderness at McBurney point (i.e., one third of the distance from the right anterior superior iliac spine to the umbilicus), rebound tenderness, psoas sign (i.e., psoas pain on hip extension), fever, Rovsing sign (i.e., right lower quadrant pain with left lower quadrant palpation); signs of an acute abdomen (e.g., severe pain, distention, rebound tenderness, and involuntary guarding) suggest perforation or impending perforation of the appendix Tests: Increased WBC, ESR, and CRP Ultrasound (US) may be useful to detect an enlarged, inflamed appendix Abdominal CT with oral contrast can detect impaired appendiceal filling, wall thickening, an obstructing fecalith, abscess formation, free abdominal fluid, or fat stranding Treatment: appendectomy is the standard of care; antibiotics are generally administered including coverage of anaerobic and Gram-negative organisms Outcomes: prognosis is excellent with no mortality associated with an unruptured appendix and <1% mortality in cases of rupture (5% in elderly patients); complications include abscess formation and perforation without
timely treatment Why eliminated from differential: the classic signs of appendicitis are missing from the physical examination in this case, and the CT does not detect abnormalities of the appendix
Possible causes of right lower quadrant pain may be remembered by the mnemonic APPENDICITIS: Appendicitis, Pelvic issues (e.g., PID, ectopic pregnancy, menstruation, ovarian cyst), Pancreatitis, Endometriosis, Neoplasm, Diverticulitis (rare), Intussusception, Crohn disease, IBD, Torsion (e.g., ovary, testicle), IBS, Stones (e.g., kidney, gallbladder).
Always get a β-hCG (human chorionic gonadotropin) test in a woman of child-bearing age with abdominal pain to rule out pregnancy.
If there is a high clinical suspicion of appendicitis, go right to surgery and do not wait for radiologic examinations. Volvulus Rotation of the bowel upon itself, resulting in obstruction and possible ischemia The cecum and sigmoid colon are the most common sites in the adult Elderly patients and infants are the most common age groups affected History: abdominal pain, vomiting, obstipation Physical examination: abdominal distention and tenderness, possible palpable mass Tests: AXR may show a “double bubble” of air proximal and distal to the site of involvement; barium enema may show a distal “bird's beak” sign Treatment: May be self-limited with automatic derotation Colonoscopic decompression may be performed for a sigmoid volvulus P.105 Surgical reduction is frequently required for a cecal volvulus or following failed colonoscopic detorsion Outcomes: reducible volvuli have a good prognosis but may recur; bowel ischemia is the most significant complication Why eliminated from differential: findings consistent with this diagnosis are not seen on the AXR and abdominal CT in the case Intussusception Telescoping of one segment of bowel into another, leading to obstruction Occurs most commonly proximal to the ileocecal valve Risk factors: Meckel diverticulum, Henoch-Schonlein purpura, adenovirus infection, cystic fibrosis, young age (i.e., more common in children) History: paroxysmal abdominal pain lasting <1 minute, pallor, diaphoresis, vomiting, bloody mucus in stool
(i.e., currant-jelly stool) Physical examination: abdominal tenderness, palpable sausagelike abdominal mass Tests: barium enema will demonstrate the obstruction; US or CT may be able to detect the abnormal section of bowel Treatment: barium enema may reduce the obstruction, but resistant cases require surgical reduction Outcomes: prognosis is excellent; bowel ischemia may occur in cases with delayed presentation Why eliminated from differential: the chronology of the abdominal pain and other symptoms and appearance of the CT make this diagnosis unlikely Cholecystitis More thorough discussion in later case Why eliminated from differential: the presence of bilious vomiting and the normal LFTs rule out this diagnosis, but an abdominal US could be performed if concern for this diagnosis further existed Diverticulitis More thorough discussion in later case Why eliminated from differential: the minimally elevated WBCs and the absence of free air or abscess formation on imaging help to rule out this diagnosis Gastroenteritis More thorough discussion in prior case Why eliminated from differential: the bilious nature of the emesis, length of symptoms, significance of the abdominal examination, and absence of diarrhea help to rule out this diagnosis Mesenteric ischemia Ischemia and necrosis of a section of bowel due to vascular compromise Due to embolization of mesenteric vessels, bowel obstruction, inadequate cardiac output (i.e., impaired systemic perfusion), or iatrogenic vascular compromise (e.g., medication, surgery) Although the left side of the colon is the most common site of involvement, the rectum is typically spared because of collateral circulation Risk factors: DM, atherosclerosis, congestive heart failure (CHF), systemic lupus erythematosus (SLE), peripheral vascular disease History: acute severe abdominal pain, bloody diarrhea, vomiting Physical examination: mild abdominal tenderness, tachycardia, tachypnea, fever, hypotension Tests: WBC and lactate are increased Barium enema will demonstrate mucosal changes in the affected region CT will demonstrate bowel wall thickening and air within the bowel wall Sigmoidoscopy may show a bloody and edematous mucosa
Treatment: Patients should be made NPO and given IV hydration Antibiotics are administered to prevent bacteremia of GI flora Surgical resection of necrotic bowel is required P.106 Outcomes: mortality is 50% for prompt treatment and 90% for missed diagnoses Why eliminated from differential: the absence of a high lactate, bloody diarrhea, and air within the bowel wall make this diagnosis unlikely
Abdominal pain for ischemic colitis is less severe than small bowel ischemia, which is significant and out of proportion to the examination. Pancreatitis More thorough discussion in later case Why eliminated from differential: the normal amylase and lipase levels and the CT findings rule out this diagnosis Pregnancy/ectopic pregnancy More thorough discussion in Chapter 12 Why eliminated from differential: the negative pregnancy test helps to rule out these diagnoses Pelvic inflammatory disease More thorough discussion in Chapter 11 Why eliminated from differential: the normal findings on pelvic examination make this diagnosis unlikely Endometriosis More thorough discussion to follow in Chapter 11 Why eliminated from differential: the absence of pelvic signs and symptoms make this diagnosis unlikely
Case 3-8: “I'm bleeding when I go to the bathroom” A 35-year-old man presents to his PCP with the complaint of painless rectal bleeding during defecation. He says that he does not have bleeding every time he has a bowel movement but has had this finding intermittently over the past 3 months. He says that at times he will have several episodes of rectal bleeding on consecutive bowel movements and has had rectal pruritis during these periods. He describes the volume of blood loss as enough to thoroughly stain the toilet paper but not enough to turn the color of the toilet water pink. He reports occasional constipation and notes that the episodes of bleeding are more common at these times. He denies abdominal or rectal pain, nausea, vomiting, diarrhea, dizziness, joint or back pain, or any rashes. He denies any past medical history or medication use. He smokes a half pack of cigarettes per day and drinks socially. On examination, he appears to be a moderately overweight male in no acute distress. Auscultation of his abdomen detects normal bowel sounds. He has no abdominal tenderness or masses. Rectal examination detects no masses, gross blood, or pus. He does not have any apparent masses or sinus tracts surrounding his rectum. No rashes or lesions are evident on his skin. The following vital signs are measured: T: 98.7°F, HR: 76 bpm, BP: 125/84 mm Hg, RR: 15 breaths/min
Differential Diagnosis Hemorrhoids, diverticulosis, diverticulitis, IBD, anal fissure, anal fistula, anorectal abscess, pilonidal disease, arteriovenous malformation, Meckel diverticulum, colorectal neoplasm
Laboratory Data and Other Study Results Fecal occult blood test: positive CBC: WBC: 6.9, Hgb: 15.7, Plt: 365 ESR: 10 mm/hr CRP: 0.2 mg/dL Following receipt of these results, the following test is performed: Colonoscopy: multiple nonthrombosed internal hemorrhoids; normal-appearing colonic mucosa; no masses or polyps
Diagnosis Internal hemorrhoids
Rectal bleeding may be due to an upper (i.e., proximal to the ligament of Treitz) or lower (i.e., distal to the ligament of Treitz) source.
Treatment Administered The patient was started on fiber supplement to be taken daily The patient was prescribed docusate sodium to be taken as needed during periods of constipation The patient was educated about behaviors likely to exacerbate bleeding
Common causes of upper-GI bleeds are PUD, Mallory-Weiss tears (i.e., longitudinal esophageal tears secondary to violent retching), esophagitis, esophageal varices, and gastritis.
Follow-up The patient reported a good response to treatment with no episodes of bleeding occurring when he was compliant with taking his fiber supplement
Common causes of lower GI bleeds are diverticulosis, neoplasm, ulcerative colitis, mesenteric ischemia, arteriovenous malformations (AVMs), hemorrhoids, and Meckel diverticulum.
Steps to the Diagnosis Hemorrhoids Engorged rectal veins with a predisposition to bleeding following any minor trauma (e.g., straining during
bowel movements) Internal: arise from superior rectal veins above the dentate line and cause painless bright red bleeding External: arise from inferior rectal veins below the dentate line and may be painful (especially if thrombosed) History: painless rectal bleeding, possible pruritis, pain if thrombosed external vein are present Physical examination: external, thrombosed, or prolapsed veins may be apparent on inspection; internal hemorrhoids are frequently undetectable by palpation Tests: CBC is useful to rule out anemia or infection Fecal occult blood testing will detect GI bleeding Colonoscopy or sigmoidoscopy is useful for visualizing internal hemorrhoids (Figure 3-14) Treatment: Avoid activities that increase the risk of bleeding (e.g., straining during bowel movements, sitting on the toilet for prolonged periods of time) A diet high in fiber or with fiber supplementation eases stool passage Sclerotherapy, ligation, or excision may be performed for highly symptomatic or painful hemorrhoids
Figure 3-14 Work-up of acute rectal bleeding. EGD, esophagogastroduodenoscopy; GI, gastrointestinal; IV, intravenous.
Outcomes: despite a high recurrence rate, most patients who follow conservative treatment recommendations do extremely well Clues to the diagnosis: History: painless rectal bleeding during bowel movements, bleeding worse with constipation Physical: noncontributory Tests: internal hemorrhoids visualized on colonoscopy
Bright red blood (i.e., hematochezia) suggests a lower GI bleed or a heavy upper GI bleed; dark blood (i.e., melena) suggests the presence of blood that has moved slowly through much of the GI tract.
Causes of GI bleeding can be remembered by the mnemonic FAMED CHIMP GUM: Fistula, AVM, Mallory-Weiss tear, Esophageal varices, Diverticulosis, Cancer (e.g., gastric, colon), Hemorrhoids, Infectious diarrhea, Mesenteric ischemia, PUD, Gastritis, Ulcerative colitis, Meckel diverticulum.
Patients with painful rectal bleeding or rectal pain require further work-up even if a colonoscopy only detects internal hemorrhoids (consider fistulas or abscesses). Diverticulosis Multiple outpouchings of the colonic mucosa and submucosa that herniate through the muscular layer of the bowel and may cause bleeding by eroding into a colonic vessel Risk factors: low fiber diet, high fat diet, age >60 years History: most commonly asymptomatic, but irregular bowel movements or painless rectal bleeding may occur with erosion into a colonic vessel Physical examination: typically noncontributory unless significant bleeding occurs Tests: fecal occult blood testing is frequently positive; diverticuli are visible on barium enema or colonoscopy (Figure 3-15) Treatment: a high fiber diet may help prevent the development of additional diverticuli Outcomes: complications include progression to diverticulitis or diverticular colitis (i.e., localized colonic inflammation) Why eliminated from differential: the presentation for both diagnoses are similar, but the patient is in an age group more likely to have hemorrhoids, and the colonoscopy is not notable for any significant diverticular disease
Diverticular disease is the most common cause of acute lower GI bleeding in patients over 40 years old. Diverticulitis Obstruction of a colonic diverticulum leading to significant inflammation, bowel wall necrosis, and colonic perforation History: left lower quadrant pain, nausea, vomiting, melena, hematochezia Physical examination: abdominal tenderness, fever, abdominal distention, possible palpable lower abdominal mass Tests: Increased WBCs, ESR, and CRP; possible decreased Hgb CXR or AXR may exhibit free peritoneal air
Figure 3-15 Barium enema in a patient with diverticular disease; numerous diverticuli can be seen in the left colon.
Abdominal CT will show colonic diverticuli, bowel wall thickening, abscess formation or fluid collections, and fat stranding Colonoscopy is contraindicated in cases of suspected diverticulitis because of the increased risk of bowel perforation Treatment: Cases without peritoneal signs (low likelihood of perforation) may be treated by bowel rest (i.e., liquids only for at least 3 days) and oral antibiotics (e.g., fluoroquinolone and metronidazole, trimethoprimsulfamethoxazole and metronidazole, or amoxicillin-clavulanate) Partial colectomy is required in most cases to resect the involved segments of colon and to remove any obstruction or fistula A diverting colostomy performed in cases of peritonitis (reanastomosis is performed in 3 months) Any bowel rupture requires administration of broad-spectrum antibiotics Outcomes: complications include abscess or fistula formation, bowel obstruction, peritonitis, and sepsis; cases with a contained bowel rupture have a good prognosis, but cases with peritonitis carry up to a 20% mortality Why eliminated from differential: the patient's symptoms are not severe enough for this diagnosis to be considered; the CBC and colonoscopy results further rule out this diagnosis
The triad of fever, increased serum WBCs, and left lower quadrant pain is suggestive of diverticulitis. Inflammatory bowel disease More thorough discussion in prior case Why eliminated from differential: the normal ESR and CRP, absence of painful symptoms, absence of extraintestinal symptoms, and colonoscopy findings rule out this spectrum of diagnoses Anal fissures Painful tears in the posterior wall of the anus that occur secondary to trauma during defecation or anal intercourse History: severe pain during every bowel movement, bright red rectal bleeding Physical examination: pain during rectal examination, gross blood may be evident Tests: if fissures are not easily apparent, anoscopy may be useful for visualization Treatment: stool softeners and topical nitroglycerin are useful to decrease pain during defecation; partial sphincterotomy may be performed for recurrent fissures Outcomes: recurrences may occur and should be treated similarly to acute fissures Why eliminated from differential: the absence of rectal pain rules out this diagnosis Anal fistulas Formation of a sinus tract between the rectum and skin May be idiopathic or secondary to IBD or a prior anorectal abscess History: mild pain during defecation, bright red rectal bleeding during bowel movements, pruritis Physical examination: painful rectal examination, tract is frequently visible, pus may be expressed from tract Tests: CBC is used to rule out infection; pelvic CT may be performed to locate any deep abscess Treatment: Acute fistulas will resolve with sitz baths, stool softeners, and a high fiber diet Chronic fistulas require fistulectomy Antibiotics are frequently indicated for underlying abscesses Outcomes: Acute fistulas commonly resolve with supportive care Patients with associated anorectal abscesses or IBD are more likely to develop chronic fistulas Patients undergoing fistulectomy typically have a good prognosis Why eliminated from differential: the patient in the case had no rectal pain, and no fistula was visualized Anorectal abscess Infection of the anal crypts, internal hemorrhoids, or hair follicles leading to abscess formation
History: throbbing rectal pain, worsening of pain with bowel movements or during sitting, pruritis Physical examination: fever, fluctuant mass that is either visible on the anal verge or palpable during rectal examination Tests: typically unneeded if an abscess is located, but CT or MRI may be used to determine the extent of deep abscesses Treatment: surgical incision and drainage (I + D), antibiotics Outcomes: up to 65% of patients will develop a fistula tract; fecal incontinence may result from I + D of deep abscesses Why eliminated from differential: no abscess is visualized or palpated in the case, and the patient is not in pain Pilonidal disease Formation of one or more cutaneous sinus tracts in the superior gluteal midline Sinus tracts do not communicate with the bowel Obstruction of the sinus tracts by hair or debris can lead to cyst and abscess formation History: obstruction causes the formation of painful and pruritic abscesses Physical examination: purulent drainage may be expressed from the sinus tract Tests: noncontributory Treatment: I + D of abscesses; marsupialization of sinus tracts may be performed for recurrent abscesses Outcomes: forty percent of patients will have recurrence of abscesses without sinus tract excision Why eliminated from differential: the patient in the case is not reporting any pain, and no sinus tracts are visualized Arteriovenous malformation More thorough discussion in Chapter 1 Why eliminated from differential: because this is an uncommon cause of painless rectal bleeding, a further work-up (e.g., arteriography, bleeding scan) should be reserved for cases in which another explanation for the bleeding is not provided (internal hemorrhoids in this case) Meckel diverticulum Common remnant of the vitelline duct that exists as an outpouching of the ileum that may contain ectopic tissue History: either asymptomatic or painless rectal bleeding Physical examination: if significant bleeding occurs, hypotension and tachycardia will be seen Tests: a technetium radionucleotide scan may be performed to detect ectopic gastric mucosa (i.e., Meckel scan) Treatment: surgical excision is performed if the patient is symptomatic Outcomes: complications include intussusception, diverticulitis, or abscess formation; prognosis is excellent in asymptomatic cases, but cases that develop complications are associated with a low rate of mortality Why eliminated from differential: the patient in the case is older than the most common age group for
symptomatic presentation of this diagnosis, and the colonoscopy findings (i.e., visible internal hemorrhoids) make this diagnosis unlikely
Meckel diverticulum rule of twos—males two times more common than females, occurs within two feet of ileocecal valve, two types of ectopic tissue (gastric, pancreatic), found in 2% of the population, most complications occur before 2 years of age. Colorectal neoplasm Neoplasm of the colon or rectum Most commonly an adenocarcinoma Risk factors: family history, ulcerative colitis, colonic polyps, hereditary polyposis syndromes, low fiber/high fat diet, previous colon cancer, alcohol, smoking, DM History: change in bowel habits (left-sided disease), weakness, abdominal pain, constipation, hematochezia, melena, malaise, weight loss
Figure 3-16 Barium enema in a patient with colorectal cancer. Acute narrowing of the colon due to the presence of the mass significantly impedes the passage of barium contrast (arrow).
Physical examination: noncontributory in early disease, but occult rectal blood, abdominal tenderness and distention, and a palpable mass may be seen in progressive disease
Tests: Anemia; positive fecal occult blood test CEA is increased in 70% of patients, but it is most useful for measuring responses to treatment and screening for recurrences Barium enema or colonoscopy are useful to determine the location of a lesion (Figure 3-16)
Table 3-8 Familial Polyposis Syndromes
Hereditary Disease
Familial adenomatous polyposis
Charac teristic s
Hundreds of polyps in colon; near-certain development of malignant neoplasm; prophylactic subtotal colectomy recommended
Hereditary nonpolyposis colorectal
Multiple genetic mutations; cancer arises from normal-appearing mucosa; neoplasms tend to form in
cancer
proximal colon
Gardner's syndrome
Similar to familial adenomatous polyposis with addition of common bone and soft tissue tumors
Peutz-Jeghers syndrome
Polyps are hamartomas with low risk of malignancy; mucocutaneous pigmentation of mouth, hands, and genitals
Turcot syndrome
Many colonic adenomas with high malignant potential; comorbid malignant CNS tumors
Juvenile polyposis
Polyps of colon, small bowel, and stomach that frequently are source of GI bleeding; slightly increased risk of malignancy later in life
CNS, central nervous system; GI, gastrointestinal.
Table 3-9 Duke's Classification System for Staging and Corresponding Prognosis of Colorectal Cancer
Class
Equivalent TMN Stage
Desc ription
Cure Rate (%)
A
I
Tumor confined to bowel wall
90
B
II
Penetration of tumor into colonic serosa or perirectal fat
80
C
III
Lymph node involvement
<60
D
IV
Distant metastases
<5
TMN, tumor, metastasis, node.
CT, MRI, or PET is useful for determining the extent of a tumor and for locating metastases Biopsy of a lesion may be performed during colonoscopy and is used to provide a histologic diagnosis Treatment: Surgical resection (e.g., hemicolectomy) is the primary treatment, and regional lymph node dissection is performed to rule out metastases Adjuvant chemotherapy is administered in cases of positive lymph nodes Palliative resections may be performed to relieve obstructions in extensive disease Prophylactic colectomy may be considered for hereditary syndromes with a high risk of developing cancer (Table 3-8) Recommended prevention Regular screening should be performed in all patients older than 50 years Annual fecal occult blood testing should be performed Flexible sigmoidoscopy should be performed every 5 years because the majority of tumors are left sided Colonoscopy should be performed every 10 years More sensitivity than flexible sigmoidoscopy Should be performed more frequently in patients with a hereditary polyposis syndrome There is a 0.1% risk of perforation per procedure Outcomes: prognosis varies with the extent of the disease and corresponds to the Duke classification system (Table 3-9); complications include metastases to the liver and lungs, large bowel obstruction, and hemorrhage Why eliminated from differential: the patient is younger than the most common age group of patients that present with a colorectal neoplasm, and the clean colonoscopy appearance makes this diagnosis unlikely
Iron-deficiency anemia in males older than 40 years is considered colon cancer
until proven otherwise.
Case 3-9: “Just gimme a drink to make my stomach feel okay” A 67-year-old man is brought to the emergency department by his granddaughter for significant abdominal pain that began earlier today. She says that her grandfather is a chronic alcoholic and that she is afraid that this is something due to his drinking. Because of the patient's current state, his granddaughter is assisting him in answering questions during the history. The patient says that he developed mild epigastric pain yesterday that quickly worsened by this morning to continuous moderate pain radiating into his back. He has been controlling the pain with a steady influx of whiskey since yesterday. He denies having had this pain previously or having new pains in other areas of his body. He says that he feels nauseous and has vomited twice since yesterday. He denies diarrhea, constipation, blood in his emesis or stool, numbness, dizziness, or chest pain. His last bowel movement was yesterday. He has a history of hypercholesterolemia for which he takes ASA and atorvastatin inconsistently. He says that he drinks a half bottle of whiskey and a couple of beers every day and has done so for at least 10 years. He denies tobacco or illicit drug use. On examination, he is a diaphoretic, disheveled man who is intoxicated and has difficulty answering questions but is able to do so with encouragement. He is slightly pale. Auscultation of his heart and lungs detects tachycardia and decreased breathing sounds at the lung bases but no extra sounds. The patient has abdominal distention and tenderness that is worse in the epigastric region and voluntary guarding. Auscultation detects few bowel sounds. He has a slight reddish discoloration of both flanks. He has difficulty compiling with the neurologic examination because of his impaired state, but he appears to have no sensory or gross motor deficits. The following vital signs are measured: T: 100.9°F, HR: 120 bpm, BP: 95/60 mm Hg, RR: 20 breaths/min
Differential Diagnosis Acute pancreatitis, chronic pancreatitis, pancreatic cancer, cholecystitis/cholangitis, bowel obstruction, PUD, MI, pneumonia, diabetic ketoacidosis
Laboratory Data and Other Study Results CBC: WBC: 17.9, Hgb: 18.1, Plt: 475 Chem10: Na: 149 mEq/L, K: 3.5 mEq/L, Cl: 101 mEq/L, CO2: 22 mEq/L, BUN:28 mg/dL, Cr: 1.2 mg/dL, Glu: 235 mg/dL, Mg: 1.9 mg/dL, Ca: 8.1 mg/dL, Phos:3.2 mg/dL LFTs: AlkPhos: 135 U/L, ALT: 70 U/L, AST: 85 U/L, TBili: 1.2 mg/dL, DBili: 0.4 mg/dL Amylase: 527 U/L Lipase: 1178 U/L CRP: 11.1 mg/dL ESR: 37 mm/hr LDH: 413 U/L Cardiac enzymes: creatine kinase (CK): 312 U/L, creatine kinase myocardial component (CK-MB): 2.2 ng/mL, troponin-I: 0.2 ng/m Electrocardiogram (ECG): sinus tachycardia; no abnormal wave morphologies CXR: small bilateral inferior pleural effusions; no infiltrates or areas of consolidation; no free air under the diaphragm Abdominal CT: enlarged pancreas with poorly defined borders; no apparent focal lesions; single pancreatic fluid collection measuring approximately 8 cm
EGD: no apparent ulcerations or gastric bleeding; no masses; normal-appearing mucosa
Diagnosis Acute necrotizing pancreatitis
Treatment Administered The patient was admitted to the intensive care unit (ICU) for treatment The patient was made NPO, given a nasogastric tube, started on IV hydration, and administered morphine as needed for pain The patient was started on empiric imipenem/cilastatin pending the infection work-up Drainage of the pancreatic fluid collection was performed by interventional radiology; pancreatic biopsy performed at time of drainage was sent for culture and cytology
Follow-up Despite therapies, the patient developed progressive multisystem organ failure and died 2 days later
Steps to the Diagnosis Acute pancreatitis Acute inflammation, hemorrhage, and necrosis of pancreatic tissue due to chaotic pancreatic enzyme activity Causes include gallstones, alcoholism, trauma, infection, hypercalcemia, hyperlipidemia, abdominal surgery, and certain medications Pseudocysts are collections arising from the pancreas consisting of enzyme-rich fluids enclosed in a sac of inflamed tissue History: acute epigastric pain radiating into the back, nausea, vomiting Physical examination: fever, abdominal tenderness and distention, voluntary guarding, tachycardia, hypoactive bowel sounds, and possible tachypnea and jaundice; severe cases are notable for pallor, diaphoresis, hypotension, hematemesis or melena, Grey-Turner sign (i.e., discoloration of the flanks), and Cullen sign (i.e., bluish periumbilical discoloration) Tests: Increased amylase and lipase (three times normal values), increased WBC and Hgb (due to inflammation and hemoconcentration), increased CRP, increased glucose, increased LDH, and decreased calcium LFTs are increased but especially with disease due to gallstones CXR may show pleural effusions or diaphragm elevation AXR may show a dilated loop of bowel near the pancreas (i.e., sentinel loop) and a distended ascending colon with a sudden end in dilation near the pancreas (i.e., colon cut-off sign) Abdominal CT shows pancreatic enlargement and possible fluid collections (e.g., abscesses, pseudocysts); CT-guided biopsy may be performed to detect infection US may be used to detect gallstones Endoscopic retrograde cholangiopancreatography (ERCP) may be used to detect impacted gallstones in
highly suspicious cases Treatment: Make patient NPO and place a nasogastric tube Give IV hydration and adequate pain control Any offending agents (e.g., alcohol, medications) should be stopped Empiric antibiotics (imipenem/cilastatin most common) are administered for severe cases or cases with concerns about an infectious etiology ERCP may be performed for relief of impacted gallstones Drainage of pseudocysts should be performed (surgical or percutaneous) for collections >7 cm in size or those present for >4 weeks Surgical debridement should be performed for large areas of necrotic or infected tissue Outcomes: Complications include abscess formation, pseudocysts, necrosis, fistula formation, chronic pancreatitis, hemorrhage, shock, DIC, sepsis, and multisystem organ failure
Table 3-10 Ranson Criteria for Determining Prognosis During Acute Pancreatitis
Increased Mortality Associated with Three or More of the Following:
On Admission
During Initial 48 Hours After Admission
>55 years old
Hematocrit decreases >10%
Serum glucose >200 mg/dL
BUN increases >5 mg/dL
Serum LDH >350 IU/L
Serum calcium <8 mg/dL
Serum AST >250 IU/L
Pao2 <60 mm Hg
WBC >16,000/mL
Base deficit >4 mEq/L Fluid sequestration >6 L
AST, aspartate aminotransferase; BUN, blood urea nitrogen; LDH, lactate dehydrogenase; Pao2, partial pressure of arterial oxygen; WBC, white cell count.
Overall mortality is approximately 15%, while severe necrotic disease is associated with a 30% mortality Ranson criteria are a common means of predicting mortality based on several objective measurements at the time of presentation and following admission to the hospital (Table 3-10)
Ranson criteria for increased mortality from acute pancreatitis upon admission may be remembered by the mnemonic GA LAW: Glucose >200 mg/dL, AST >250 IU/L,
LDH >350 IU/L, Age >55 years old, WBC >16,000/mL. Clues to the diagnosis: History: severe epigastric pain radiating to the back, nausea, vomiting, history of alcoholism Physical: diaphoresis, pallor, abdominal distention and tenderness, voluntary guarding, hypoactive bowel sounds, Grey-Turner sign Tests: increased amylase and lipase, increased WBC and Hgb, decreased Ca, mildly increased LFTs, increased CRP and ESR, increased LDH, CT findings
Ranson criteria for increased mortality from acute pancreatitis during the initial 48 hours after admission may be remembered by the mnemonic Calvin & HOBBeS: Calcium <8 mg/dL, Hct decrease >10%, O2 (Pao2) <60 mm Hg, BUN increase >5mg/dL, Base deficit >4 mEq/L, Sequestration of fluid >6 L. Chronic pancreatitis Chronic or recurrent pancreatic inflammation due to long-term inflammation by pancreatic enzymes Most commonly due to alcoholism or congenital defects History: recurrent epigastric pain, nausea, steatorrhea, diarrhea, weight loss Physical examination: abdominal tenderness, epigastric mass or fullness Tests: Mildly increased amylase and lipase; increased glucose Increased fecal fat AXR shows pancreatic calcifications Abdominal CT shows pancreatic enlargement and possible pseudocysts ERCP may be used to visualize pancreatic duct abnormalities Treatment: Stop alcohol use Analgesia for chronic abdominal pain Low fat diet with adequate vitamins and minerals Surgical or endoscopic duct repair or drainage; partial pancreatectomy may be performed for disease in a limited region Outcomes: complications include pseudocyst formation, DM, splenic vein thrombosis, pseudoaneurysm formation, and obstruction of the pancreatic duct, bile ducts, and duodenum; survival is 70% at 10 years and 45% at 20 years after diagnosis Why eliminated from differential: the acuity of symptoms and significantly high amylase and lipase levels seen in this case are more indicative of an acute pancreatic pathology Pancreatic cancer (exocrine) Neoplasms of the exocrine pancreas that are most commonly adenocarcinomas
Most commonly located in the head of the pancreas
Table 3-11 Endocrine Pancreatic Neoplasms
Type
Cells and Effec t
H/P
Tests
Treatment
Outc ome
Gastrinoma
Gastrin-secreting
Refrac tory
Inc reased fasting
PPIs, H2
60% of cases are
(i.e.,
tumor in
PUD, abdominal
gastrin, positive
antagonists;
malignant; may be
Zollinger-
pancreas or
pain, vomiting,
secretin-
surgical resection
associated with
Ellison
duodenum
diarrhea,
stimulation test
for nonmetastatic
MEN1; severe
disease;
PUD
syndrome)
malabsorption
octreotide for metastatic disease
Insulinoma
β-islet c ells;
Headache, vision
Inc reased fasting
Surgical
5% of cases are
exc ess insulin
changes,
insulin,
resection;
malignant; 5%
secretion and
confusion,
inappropriate
diazoxide or
recurrence rate;
hy pogly c emia
weakness,
hypoglycemia,
octreotide in
long survival in
diaphoresis,
positive indium-
unresectable
metastatic
palpitations
labeled octreotide
disease
disease
scintigraphy
Glucagonoma
α-c ells; excess
Abdominal pain,
Hyperglycemia,
Surgical
High malignancy
gluc agon
diarrhea,
increased
resection; octreo
rate with
secretion and
confusion,
glucagon
tide, α-IFN,
frequent poor
hy pergly c emia
weight loss,
chemotherapy, or
prognosis
migratory
embolization in
necrolytic
metastatic
erythema, DM
disease
symptoms
VIPoma
Non-β-islet c ells;
W atery
Increased VIP,
Surgical
50% cure after
excess V IP
diarrhea,
normal stool
resection;
resection; 5-year
weakness,
osmotic gap
octreotide and
survival is 95% in
nausea,
embolization in
nonmetastatic
vomiting,
metastatic
disease and 60%
abdominal pain
disease
in metastatic disease
α-IFN, α-interferon; MEN1, multiple endocrine neoplasia syndrome type 1; PPIs, proton pump inhibitor; PUD, peptic ulcer disease; VIP, vasoactive intestinal peptide.
Endocrine tumors of the pancreas involve a particular type of glandular tissue, and their presentation varies with the glands involved (Table 3-11) Risk factors: chronic pancreatitis, DM, family history, tobacco use, high fat diet, male gender History: abdominal pain radiating to back, anorexia, nausea, vomiting, weight loss, fatigue, steatorrhea Physical examination: painless jaundice (with bile duct obstruction), deep abdominal tenderness, ascites, palpable gallbladder (i.e., Courvoisier sign), splenomegaly (with tail involvement) Tests: Possible hyperglycemia; increased CEA and cancer antigen 19 (CA19) tumor markers Amylase and lipase are typically normal Bilirubin (total and direct) and alkaline phosphatase are increased with bile duct obstruction
Abdominal CT will show a mass, a dilated pancreas, dilated bile ducts, and possible local extension US is an alternative to CT but is not as sensitive ERCP may be used to locate ductal tumors not obvious on CT Biopsy is used to provide a histologic diagnosis Treatment: Nonmetastatic disease limited to the head of the pancreas may be resected with the Whipple procedure (i.e., resection of the head of the pancreas, duodenum, proximal jejunum, common bile duct, gallbladder, and distal stomach) Lesions in the body or tail may be resected via a subtotal pancreatectomy if diagnosed early in their course Adjuvant chemotherapy is beneficial in cases amenable to resection Enzyme replacement therapy may be performed postoperatively for cases in which minimal or no pancreatic tissue is left following resection Stenting of the pancreatic ducts, biliary ducts, or duodenum may be performed for palliation in advanced disease Outcomes: Migratory thrombophlebitis is a common complication Five-year survival following a Whipple resection is approximately 20% Five-year survival for unresectable disease is <2% with a mean survival of approximately 6 months Why eliminated from differential: the elevated amylase and lipase levels and the absence of a mass on the CT help to make this diagnosis unlikely
Insulinomas are almost always solitary; multiple insulinomas may be seen in MEN1.
If Whipple triad is seen (i.e., symptoms of hypoglycemia while fasting, hypoglycemia, and improvement in symptoms with carbohydrate load), perform a work-up for insulinoma. Cholecystitis and cholangitis More thorough discussion in later case Why eliminated from differential: the normal bilirubin levels and absence of dominant right upper quadrant abdominal pain make these diagnoses unlikely Bowel obstruction More thorough discussion in prior case Why eliminated from differential: because the patient is continuing to have bowel movements and the CT is highly suggestive of a pancreatic pathology, this diagnosis is unlikely Peptic ulcer disease More thorough discussion in prior case
Why eliminated from differential: the absence of an ulcer on the EGD rules out this diagnosis Myocardial infarction More thorough discussion in Chapter 1 Why eliminated from differential: the normal ECG and cardiac enzyme findings rule out this diagnosis Pneumonia More thorough discussion in Chapter 2 Why eliminated from differential: the appearance of the CXR rules out this diagnosis Diabetic ketoacidosis More thorough discussion in Chapter 5 Although the glucose level is elevated, it is less than would be expected for diabetic ketoacidosis (DKA), and the electrolytes are not abnormal as would be expected for DKA; fluid replacement is the initial treatment for both this diagnosis and acute pancreatitis, so the initial approach to both diagnoses is similar
Case 3-10: “My right side hurts really bad” A 46-year-old woman presents to the emergency department with the complaint of significant right upper quadrant abdominal pain that has lasted for the past 10 hours. She says the pain radiates to her back and has been fairly continuous during this time. She says that she previously experienced a much milder form of this pain following meals that typically would only last for an hour when it occurred. She has been unable to eat since the current episode started, feels nauseous, and has vomited once today. She denies any chest pain, dyspnea, diarrhea, heartburn, hematemesis or hematochezia, dizziness, weight loss, or chills. She reveals a history of hypercholesterolemia, HTN, and obesity and takes ASA, hydrochlorothiazide (HCTZ), and atorvastatin. She denies any substance use. On examination, she appears to be an obese woman who is quite uncomfortable. Auscultation of her chest and back detects normal breath and hseart sounds and mild tachycardia. She has abdominal tenderness with voluntary guarding to palpation of the right upper quadrant and is unable to breathe inward during palpation. She has fullness of her right upper quadrant, but it is difficult to detect a discrete mass or organomegaly because of her body habitus. She is not jaundiced. Her bowel sounds are mildly hypoactive. Rectal examination does not detect any gross blood. The following vital signs are measured: T: 100.1°F, HR: 109 bpm, BP: 135/86 mm Hg, RR: 14 breaths/min
Differential Diagnosis Cholelithiasis, acute cholecystitis, cholangitis, gallbladder cancer, abdominal aortic aneurysm (AAA), mesenteric ischemia, PUD, gastritis, pneumonia, MI, acute pancreatitis, pyelonephritis, ectopic pregnancy
Laboratory Data and Other Study Results CBC: WBC: 12.1, Hgb: 14.2, Plt: 319 Chem7: Na: 140 mEq/L, K: 4.2 mEq/L, Cl: 109 mEq/L, CO2: 24 mEq/L, BUN: 12 mg/dL, Cr: 0.6 mg/dL, Glu: 100 mg/dL Amylase: 76 U/L Lipase: 123 U/L Lactate: 1.3 mEq/L
LFTs: AlkPhos: 143 U/L, ALT: 49 U/L, AST: 31 U/L, TBili: 1.8 mg/dL, DBili: 0.3 mg/dL UA: straw-colored, pH: 5.0, specific gravity: 1.010, no glucose/ketones/nitrites/leukocyte esterase/hematuria /proteinuria Urine pregnancy test: negative ECG: sinus tachycardia; no abnormal wave morphologies CXR: clear lung fields; no abnormal markings Abdominal CT: gallbladder wall thickening; small pericholecystitic fluid collection; no discrete masses; normal bowel and vascular anatomy Based on these findings, the following test is performed: Abdominal US: pericholecystitic fluid; 6 mm gallbladder wall thickness; multiple gallstone shadows
Diagnosis Acute cholecystitis
Treatment Administered The patient was admitted for analgesia and IV hydration and was made NPO Ampicillin and sulbactam were started The diagnosis was confirmed with a positive hepatic iminodiacetic (HIDA) scan Open cholecystectomy was performed 1 day after admission
Follow-up The patient had successful pain control and an improvement in her symptoms following cholecystectomy The patient was able to be discharged to home successfully following a brief postoperative stay
Steps to the Diagnosis Acute cholecystitis Inflammation of the gallbladder most commonly due to obstruction of the cystic duct by gallstones Acalculous cholecystitis may occur due to conditions that are associated with biliary stasis (e.g., trauma, sepsis, total-parenteral nutrition) History: right upper quadrant abdominal pain radiating to the back and right shoulder (continuous pain >6 hours), nausea, vomiting, anorexia Physical examination: right upper quadrant tenderness, fever, right upper quadrant fullness or a palpable gallbladder, Murphy sign (i.e., inability to inspire during right upper quadrant palpation due to pain) Tests: Increased WBCs, increased bilirubin (total and direct), and increased alkaline phosphatase CT or MRI may show gallbladder wall thickening and pericholecystitic fluid US may detect gallstone shadows, sludge, gallbladder wall thickening, and pericholecystitic fluid
A HIDA scan will detect cystic duct obstruction by demonstrating impaired gallbladder filling Treatment: Hydration, analgesia, and no ingested food following the diagnosis Antibiotics (e.g., β-lactams, cephalosporins) are started to prevent the onset of worsening ductal infection Open or laparoscopic cholecystectomy is the definitive treatment Patients who are inappropriate for surgery may be treated with interventional placement of a percutaneous ductal drainage tube Outcomes: complications include cholangitis, ductal perforation, and gallstone ileus; prognosis is excellent in uncomplicated cases, but mortality is approximately 50% if complications develop Clues to the diagnosis: History: continuous right upper quadrant pain, nausea, vomiting, anorexia, obesity, female, age Physical: right upper quadrant tenderness, positive Murphy sign, right upper quadrant fullness Tests: mildly increased WBC, increased bilirubins, increased alkaline phosphatase, CT and US findings
Only 10% of gallstones are radiopaque.
Remember the five Fs for patients susceptible to gallstone formation: Female, Fertile, Fat, Forty (years old), and Family history. Cholelithiasis Gallstone formation in the bladder that may cause intermittent cystic duct obstruction Most stones are composed of cholesterol Stones composed of calcium bilirubinate (i.e., pigmented stones) occur due to chronic hemolysis Risk factors: age >40 years, obesity, female gender, multiparity, oral contraceptive use, DM, family history, total parenteral nutrition (TPN), recent rapid weight loss History: frequently asymptomatic, but possible symptoms include postprandial right upper quadrant abdominal pain, nausea, vomiting, indigestion, and flatulence Physical examination: right upper quadrant tenderness, palpable gallbladder
Figure 3-17 Ultrasound demonstrating multiple gallstones (white arrows) in the gallbladder. Note the shadow caused by the gallstones, which may be more apparent than the gallstones themselves in several cases.
Tests: US may show gallstones or gallstone shadows (Figure 3-17); AXR will only show pigmented stones with a high iron component Treatment: Dietary reduction of high fat foods Bile salts may help to dissolve small stones Shock wave lithotripsy may be helpful for breaking up stones Cholecystectomy is indicated for worsening symptoms Outcomes: complications include acute cholecystitis, pancreatitis, and stone recurrence; the prognosis is excellent in asymptomatic or mildly symptomatic cases Why eliminated from differential: the length and continuous nature of the patient's symptoms are more indicative of a progression from cholelithiasis to acute cholecystitis Cholangitis Infection of the bile ducts that results due to unrelieved ductal obstruction Risk factors: cholelithiasis, biliary cancer, congenital ductal abnormalities History: right upper quadrant abdominal pain, chills Physical examination: jaundice, fever, right upper quadrant tenderness, change in mental status, hypotension
Tests: Increased WBC, increased total and direct bilirubin, increased alkaline phosphatase, and increased ALT and AST Amylase and lipase may be increased with associated pancreatic inflammation Blood cultures or cultures of drained biliary fluid will indicate the causative pathogen US and HIDA scan may be used to confirm biliary obstruction Treatment: IV hydration and antibiotics Endoscopic biliary drainage followed by delayed cholecystectomy Emergent biliary decompression may need to be performed in severe cases Outcomes: complications include sepsis, hepatic abscesses, and renal failure; prognosis is poor once other organ systems are affected by the disease Why eliminated from differential: the acute nature of the symptoms, near normal WBC, and only mild worsening of the LFTs in this case make this diagnosis less likely than cholecystitis
If you see Charcot triad (i.e., right upper quadrant pain, jaundice, and fever), suspect cholangitis and perform an US or HIDA scan. Gallbladder cancer Adenocarcinoma of the gallbladder wall Risk factors: cholelithiasis, cholecystitis or cholangitis, biliary tract disease History: diffuse abdominal pain, anorexia, weight loss, back pain Physical examination: jaundice, palpable gallbladder, Sister Mary Joseph or Virchow lymphadenopathy Tests: Anemia, increased alkaline phosphatase, increased total and direct bilirubin, and increased CA19-9 tumor marker AXR may show a calcified gallbladder (i.e., porcelain gallbladder) US, CT, or MRI is useful for detection of a mass ERCP is useful to localize and biopsy lesions Treatment: Cholecystectomy, lymph node dissection, and partial removal of adjacent hepatic tissue Adjuvant radiation therapy and chemotherapy may reduce recurrence rates and are used as primary therapies in unresectable disease Outcomes: five-year survival rates for limited and locally expanded disease are 40% and 15%, respectively Why eliminated from differential: the absence of a discrete mass on CT and US rules out this diagnosis
A calcified gallbladder is highly suggestive of cancer, and biopsy should be performed
promptly to confirm diagnosis. Abdominal aortic aneurysm More thorough discussion in Chapter 1 Why eliminated from differential: the absence of a vascular abnormality on the abdominal CT rules out this diagnosis Mesenteric ischemia More thorough discussion in prior case Why eliminated from differential: the normal lactate level, absence of rectal bleeding, and normal bowel appearance on the abdominal CT rule out this diagnosis Peptic ulcer disease More thorough discussion in prior case Why eliminated from differential: similar chronologies of abdominal pain may be seen in both diagnoses, but the pain localized to the right upper quadrant and the imaging findings are more suggestive of cholecystitis Gastritis More thorough discussion in prior case Why eliminated from differential: similar chronologies of abdominal pain may be seen in both cases, but the pain localized to the right upper quadrant and the imaging findings are more suggestive of cholecystitis Pneumonia More thorough discussion in Chapter 2 Why eliminated from differential: the normal CXR rules out this diagnosis Myocardial infarction More thorough discussion in Chapter 1 Why eliminated from differential: the normal ECG rules out this diagnosis Acute pancreatitis More thorough discussion in prior case Why eliminated from differential: the normal amylase and lipase levels and the normal appearance of the pancreas on the CT rule out this diagnosis Ectopic pregnancy More thorough discussion in Chapter 12 Why eliminated from differential: the negative urine pregnancy test rules out this diagnosis Pyelonephritis More thorough discussion in Chapter 4 Why eliminated from differential: the normal UA makes this diagnosis unlikely
Case 3-11: “I just feel terrible” A 32-year-old woman presents to a community health center with significant malaise and fatigue. The woman says that these symptoms have been present for the past month and that she has never experienced them previously. She feels very fatigued, her arms and legs ache, and she feels nauseous. She says that she has vomited multiple types over the past month due to her nausea. Strong-smelling foods seem to make her nausea worse. She also notes intermittent right upper quadrant abdominal pain. She has little appetite and has lost weight over the past month. She denies diarrhea, hematemesis or hematochezia, constipation, dyspnea, coughing, chest pain, weakness, paresthesias, dizziness, frequent illnesses, or urinary symptoms. She denies any past medical history and does not take any medications. She states that she does not receive any regular medical care and only sees doctors when she becomes very sick. She smokes a pack of cigarettes per day, drinks approximately 20 alcoholic beverages per week, and has intravenously injected cocaine on a few occasions (most recently 3 months ago). She frequently uses condoms for contraception but has had unprotected sex on multiple occasions in the past year with two partners. On examination, she is an ill-appearing woman who is able to fully cooperate with the examination. Her skin is mildly jaundiced, and her sclera are icteric. Auscultation of her back and chest finds normal heart and lung sounds. She has mild abdominal tenderness in the right upper quadrant. She has palpable mild hepatosplenomegaly. Her bowel sounds are normal. A rectal examination detects no masses or gross blood. Neurologic examination is normal. The following vital signs are measured: T: 100.5°F, HR: 86 bpm, BP: 118/73 mm Hg, RR: 16 breaths/min
Differential Diagnosis Hepatitis, alcohol-related liver disease, cirrhosis, cholecystitis, cholangitis, acute pancreatitis, Wilson disease, hemochromatosis, primary biliary cirrhosis, primary sclerosing cholangitis, Gilbert syndrome, hepatocellular carcinoma
Laboratory Data and Other Study Results CBC: WBC: 7.8, Hgb: 13.5, Plt: 320 Chem7: Na: 140 mEq/L, K: 3.9 mEq/L, Cl: 105 mEq/L, CO2: 24 mEq/L, BUN: 24 mg/dL, Cr: 0.9 mg/dL, Glu: 78 mg/dL Coags: PT: 11.1, INR: 1.0, PTT: 27.4 LFTs: AlkPhos: 257 U/L, ALT: 1265 U/L, AST: 1176 U/L, TBili: 7.4 mg/dL, DBili: 4.6 mg/dL Amylase: 54 U/L Lipase: 76 U/L Nutrition panel: albumin: 3.6 g/dL, Fe: 148 µg/dL, transferrin: 300 mg/dL, ferritin: 65 ng/mL Serum ceruloplasmin: 30 mg/dL Abdominal CT: generalized increased opacity of the liver; mild hepatosplenomegaly but no other abnormal appearances of abdominal organs; no discrete masses, gallstones, or ductal or bowel dilations Based on these results, the following tests are ordered: Anti-HAV IgM (hepatitis A virus immunoglobulin M antibody): negative HBsAg (hepatitis B surface antigen): positive HBeAg (HBVe antigen): positive Anti-HBc (hepatitis B core antibody) IgM: positive
HCVAg (hepatitis C virus antigen): negative Anti-HCV (hepatitis C virus antibody): negative
Color Figure 3-2 Giardiasis; several trophozoites are seen with characteristic pear shape and paired nuclei resembling owls' eyes.
Diagnosis Acute hepatitis B
Treatment Administered The patient was prescribed α-IFN following the serologic diagnosis The patient was enrolled in a drug and alcohol counseling program
Follow-up α-IFN was discontinued after 3 weeks because of poor side effect tolerance, and lamivudine and adefovir were started The patient's symptoms improved with medical therapy and rest Follow-up serology demonstrated disappearance of HBsAg and HBeAg and the development of anti-HBc IgG, anti-HBs, and anti-HB (suggestive of disease resolution)
Figure 3-18 Trends in serology and symptoms seen in various courses in acute hepatitis B virus infection with resolution of the disease. HBsAg (hepatitis B surface antigen) and HBe Ag (hepatitis B virus e antigen) are detectable from approximately 1 to 4 months and coincide with appearance of symptoms. Antibodies are indicative of previous infection or vaccination. ALT, alanine aminotransferase; Anti-HBc, hepatitis B core antibody; IgM, immunoglobulin M.
Steps to the Diagnosis Viral hepatitis Inflammatory disease of the liver due to viral infection Classified as acute if symptoms are present <6 months Considered chronic if symptoms are present >6 months Risk factors: intravenous drug use (IVDU), blood transfusions, travel to developing nations, multiple sexual partners History: possibly asymptomatic with later development of malaise, fatigue, myalgias, arthralgias, nausea, vomiting, right upper quadrant abdominal pain, sensitivity to food odors, and anorexia Physical examination: jaundice, scleral icterus, right upper quadrant tenderness, hepatomegaly, splenomegaly, lymphadenopathy, fever (Color Figure 3-2) Tests: Increased ALT and AST, increased alkaline phosphatase, and increased total and direct bilirubin Serologic markers are used to differentiate the viral strain and course of the disease (Figures 3-18 and 3-19 and Tables 3-12 and 3-13)
Figure 3-19 Trends in serology and symptoms seen in hepatitis C virus (HCV) infection. (A) Acute resolving infection. (B) Chronic HCV infection with intermittent exacerbations of symptoms. ALT, alanine aminotransferase.
Table 3-12 Characteristics of Viral Hepatitis
Hepatitis V irus
V irus Ty pe
Spread
Prevention
Complic ations
A
Picornavirus
Food (e.g.,
Vaccine
(HAV)
(single-stranded
shellfish),
prior to
RNA)
fecal-oral
travel
B
Hepadnavirus
Blood, other
V ac c ine
(HBV)
(double-stranded
body fluids (e.g.,
cirrhosis, hepatoc ellular c arc inoma (3%– 5% of
DNA)
sexual contact)
cases), persistent carrier state, fulminant hepatic
May occur in epidemics
Chronic hepatitis (5% of adults and 90% of children),
failure (1% of cases), glomerulonephritis, polyarteritis nodosa
C
Flavivirus (single-
Blood, possibly
(HCV)
stranded RNA)
sexual contact
No vaccine
Chronic hepatitis (80% of cases), cirrhosis (50% of chronic cases), slightly increased risk hepatocellular carcinoma, persistent carrier state, glomerulonephritis
D
Delta agent
Blood; requires
Hepatitis B
(HDV)
(in-complete single-
c oexistent
vaccine
stranded RNA)
hepatitis B
Severe hepatitis, cirrhosis, persistent carrier state
infec tion
E
Calicivirus (single-
(HEV)
stranded RNA)
Water, fecal-oral
No vaccine
High infant mortality when occurring in pregnant women
Hepatitis A virus (HAV): anti-HAV IgM antibodies are present during acute infection, and anti-HAV IgG antibodies are present following resolution Hepatitis B virus (HBV): HBsAg and HBeAg are markers for active infection; anti-HBs, anti-HBe, and anti-HBc are markers for immune responses to the virus (Table 3-13) Hepatitis C virus (HCV): HCVAg is a marker for acute infection, and HCV antibody (anti-HCV) is a marker for the immune response to the virus Hepatitis D virus (HDV): detection of HDV RNA by polymerase chain reaction (PCR) or of HDV antibodies (anti-HDV) indicates infection Hepatitis E virus (HEV): detection of HEV RNA by PCR or of HEV antibodies (anti-HEV) indicates infection Treatment: Rest and avoidance of hepatotoxic substances (e.g., alcohol) during the acute stages Medical treatments vary depending on viral type HAV: self-limited, so only supportive care is required HBV: vaccine may be administered immediately after exposure in unvaccinated individuals; α-IFN or antivirals (e.g., lamivudine, adefovir, entecavir) are prescribed HCV: α-IFN and ribivarin are prescribed to attempt to induce suppression HDV: α-IFN is prescribed HEV: self-limited, so only supportive care is required Outcomes: complications vary between strains and include chronic disease, hepatocellular carcinoma, hepatic failure, cirrhosis, glomerulonephritis, and polyarteritis nodosa (Table 3-12) Clues to the diagnosis:
Table 3-13 Serologies Seen in Various Disease States of Hepatitis B Virus Infection
HBV Core Course of Disease
Acute infection (4–12 weeks
HBV Surfac e
HBV e Antigen
HBV Surfac e
HBV e Antibody
Antibody
Antigen (HB sAg)
(HB e Ag)
Antibody (anti-HBs)
(anti-HBe)
(anti-HBc )
Positive
Positive
Negative
Negative
Positive (IgM)
Negative
Negative
Negative
Negative
Positive (IgM)
Positive
Positive
Negative
Negative
Positive (IgG)
Positive
Negative
Negative
Positive
Positive (IgG)
Past infection (recovered)
Negative
Negative
Positive
Positive
Positive (IgG)
Vaccination
Negative
Negative
Positive
Negative
Negative
postexposure)
Acute infection window period (12–20 weeks postexposure)
Chronic infection, active viral replication
Chronic infection, lesser viral replication (good prognosis)
HBV, hepatitis B virus; Ig, immunoglobulin.
History: fatigue, malaise, myalgias, nausea, vomiting, history of IV drug use, unprotected sex Physical: hepatosplenomegaly, jaundice, scleral icterus Tests: increased ALT, AST, bilirubins, and alkaline phosphatase; positive HBV serology
Coincident HBV infection is required for HDV infection to occur.
Multiple agent therapy is becoming more common in the treatment of HBV because even though the rate of cure is found to be similar to monotherapy the rate of drug resistance is much improved. Alcohol-related liver disease Progressive liver damage due to chronic alcoholism The early stage is characterized by fatty deposits in the liver (i.e., hepatic steatosis) Continued alcohol abuse leads to hepatic inflammation, tissue necrosis, and cirrhosis History: initially asymptomatic for several years then eventual development of anorexia, nausea, and vomiting Physical examination: abdominal tenderness, ascites, splenomegaly, hepatomegaly, fever, jaundice, testicular atrophy, gynecomastia, digital clubbing Tests: Increased ALT, AST, gamma glutamyl transpeptidase (GGT), alkaline phosphatase, total and direct
bilirubin, WBCs, and PT Biopsy is diagnostic and will show an increased fatty composition of the liver, areas of necrosis, and many PMNs (Color Figure 3-3) CT, MRI, or US are useful to detect fatty deposits in the liver Treatment: Cessation of alcohol use High calorie diet with thiamine and folate supplementation Liver transplantation is an option in patients with cirrhosis who are able to maintain abstinence from alcohol Outcomes: prognosis is good in patients who are able to maintain alcohol abstinence but is far worse in patients with cirrhosis Why eliminated from differential: the presentation may be similar for these diagnoses given the patient's alcohol history, but the absence of a fatty liver on the CT and the positive HBV serologic markers make this diagnosis unlikely
In viral hepatitis AST and ALT are equally high; in alcohol-related liver disease AST is greater than ALT.
Color Figure 3-3 Hepatic steatosis due to chronic alcohol consumption. Numerous hepatocytes are distended due to cytoplasmic fat collections.
Cirrhosis More thorough discussion in later case Why eliminated from differential: although the patient may develop cirrhosis from chronic alcoholism or hepatitis, the patient's current presentation does not suggest end-stage liver disease, and this diagnosis is ruled out; the serologic markers for acute viral hepatitis are further evidence against this diagnosis Cholecystitis/cholangitis More thorough discussion in prior case
Why eliminated from differential: the absence of abnormal gallbladder findings on the abdominal CT makes this diagnosis unlikely Acute pancreatitis More thorough discussion in prior case Why eliminated from differential: the normal amylase and lipase levels and the normal pancreatic appearance on the CT rule out this diagnosis Wilson disease Autosomal-recessive disorder of impaired copper secretion Excess systemic copper results in deposits in the liver, brain, and corneas Occurs primarily in young adults History: loss of coordination, dysphagia Physical examination: psychiatric disturbances (e.g., depression, neuroses, personality changes), jaundice, tremor, green-brown corneal rings (i.e., Kayser-Fleischer rings), hepatomegaly (Color Figure 3-4) Tests: Decreased serum ceruloplasmin, increased urinary copper, and mildly increased ALT and AST
Color Figure 3-4 Jaundice in a patient with hyperbilirubinemia. Note the yellow sclera and skin compared to the normal hue of the examiner's hand.
Liver biopsy will show copper deposits Cranial CT or MRI will detect focal white matter lesions and signs of decreased cerebral volume Treatment: Trientine or penicillamine is administered for copper chelation Lifelong zinc and vitamin B6 supplementation is required as maintenance therapy Dietary copper restriction is required (e.g., no organ meats, shellfish, chocolate, nuts, or mushrooms)
Cases of liver failure may require transplantation Outcomes: Complications include fulminant hepatic failure and cirrhosis Prognosis is good in patients who receive adequate treatment before significant hepatic injury occurs Patients in which liver transplantation is performed have a better prognosis than patients receiving a transplant for cirrhosis due to alcoholism or hepatitis Why eliminated from differential: the normal serum ceruloplasmin level and the absence of psychiatric signs rule out this diagnosis Hemochromatosis Autosomal-recessive disorder of excess iron absorption Iron deposition occurs in the liver, pancreas, heart, and pituitary and leads to eventual fibrosis in these sites Rarely may develop as a sequela of chronic blood transfusions or alcoholism History: abdominal pain, polydipsia, polyuria, arthralgias, impotence, lethargy Physical examination: bronze pigmented rash, hepatomegaly, testicular atrophy, possible symptoms of DM or CHF Tests: increased serum iron, percentage of iron saturation, ferritin, transferrin, AST, and ALT; liver biopsy will demonstrate an increased hepatic iron content Treatment: Therapeutic phlebotomy (initially weekly or biweekly until serum iron levels normalize and then monthly) Avoidance of excess alcohol use Deferoxamine may be used for iron chelation Outcomes: complications include cirrhosis, hepatocellular carcinoma, CHF, DM, hypopituitarism, and arthrosis; patients without hepatic fibrosis whoundergo regular phlebotomy have an excellent prognosis Why eliminated from differential: the normal iron, ferritin, and transferrin levels in this patient rule out this diagnosis Primary biliary cirrhosis (PBC) An autoimmune condition of bile duct destruction that leads to the hepatic accumulation of cholesterol, bile acids, and bilirubin Risk factors: rheumatoid arthritis, Sjögren syndrome, scleroderma, female gender History: patients are either asymptomatic or report fatigue, pruritis, and arthralgias Physical examination: jaundice, xanthomas, hepatosplenomegaly, skin hyperpigmentation Tests: Increased alkaline phosphatase, GGT, cholesterol, and bilirubin Normal or mildly increased ALT and AST
Positive antinuclear antibodies (ANA) and antimitochondrial antibodies Biopsy of bile ducts demonstrates ductal inflammation and necrosis Treatment: Ursodeoxycholic acid improves liver function and symptoms Colchicine and methotrexate are second-line drugs and are used in severe cases Liver transplantation may be required in progressive cases Outcomes: Complications include hypercholesterolemia, osteoporosis, malabsorption syndrome, esophageal dysfunction, renal tubular acidosis, and hepatocellular carcinoma Prognosis corresponds with the bilirubin level at the time of diagnosis; the mean survival for a total bilirubin >2 mg/dL is 4 years, >6 mg/dL is 2 years, and >10 mg/dL is 1.5 years Why eliminated from differential: the presentation of both diagnoses may be similar, but the viral serology is confirmatory of the diagnosis of hepatitis; negative serology should prompt a liver biopsy to look for PBC
Gender, presence or absence of antimitochondrial antibodies, and ERCP distinguish PBC from PSC.
Prehepatic conditions cause an increase in indirect bilirubin; posthepatic conditions cause an increase in direct bilirubin; intrahepatic conditions may cause an increase of either or both types of bilirubin. Primary sclerosing cholangitis (PSC) Progressive destruction of intrahepatic and extrahepatic bile ducts leading to hepatic fibrosis and cirrhosis Risk factors: ulcerative colitis, male gender History: patients may be asymptomatic or may experience fatigue, pruritis, night sweats, and right upper quadrant abdominal pain Physical examination: fever, jaundice, xanthomas Tests: Increased alkaline phosphatase, GGT, cholesterol, total and direct bilirubin Mildly increased ALT and AST p-ANCA may be positive Biopsy of the bile ducts demonstrates ductal inflammation and necrosis and appears similar to that for PBC ERCP will demonstrate structuring and irregularity of the intrahepatic and extrahepatic bile ducts (i.e., “pearls on a string” appearance) Treatment: Ursodeoxycholic acid, methotrexate, and corticosteroids are the principal medical treatments Endoscopic stenting of strictures may be performed to avoid ductal obstruction
Surgical resection of affected ducts and liver transplantation may be required in progressive cases Outcomes: complications include malabsorption syndromes, cholelithiasis, cholangitis, gallbladder cancer, and colon cancer; progressive liver failure is inevitable with a median time from diagnosis to the need for liver transplantation being 12 years Why eliminated from differential: the presentation of both diagnoses may be similar, but the viral serology is confirmatory of the diagnosis of hepatitis; negative serology should prompt a liver biopsy and ERCP to look for PSC Gilbert syndrome Autosomal (dominant or recessive) disease of mild glucuronosyltransferase deficiency that causes impaired bilirubin conjugation in the liver History: patients may be asymptomatic or may have mild abdominal pain, fatigue, and general malaise Physical examination: mild jaundice following stress (e.g., exercise, fasting, psychologic stressors) Tests: increased indirect bilirubin (<5 mg/dL); CBC and LFTs (besides bilirubin) will be normal (Table 3-14) Treatment: none is required; patient reassurance that the condition is benign Outcomes: patients have an excellent prognosis with no anticipated complications Why eliminated from differential: the abnormalities in the LFTs besides bilirubin rule out this diagnosis Hepatocellular carcinoma (a.k.a. hepatoma) Malignant tumor of the hepatic parenchyma Liver metastases from primary breast, lung, or colon cancers are more common than primary liver cancers Benign tumors (e.g., hepatic adenoma, focal nodular hyperplasia, hepatic cysts) are more common in women using oral contraceptives Risk factors: HBV or HCV infection, cirrhosis, hemochromatosis, Aspergillus infection, schistosomiasis History: weight loss, right upper quadrant abdominal pain, malaise, anorexia, diarrhea, dyspnea, pruritis
Table 3-14 Causes of Conjugated and Unconjugated Bilirubinemia
Increased Total Bilirubin
Hy perbilirubinemia
Unconjugated
Cause
Excess bilirubin production
(indirect)
Examples
Hemolytic anemia Disorders of erythropoiesis Internal hemorrhage resorption
Impaired conjugation
Physiologic jaundice of newborn Deficiency of glucuronosyltransferase (e.g., Gilbert disease, Crigler-Najjar syndrome) Hepatocellular disease (e.g., cirrhosis, hepatitis)
Conjugated (direct)
Decreased hepatic bilirubin
Impaired bilirubin transport (e.g., Dubin-Johnson syndrome, Rotor syndrome)
excretion
Hepatocellular disease (e.g., cirrhosis, hepatitis) Drug impairment
Extrahepatic biliary obstruction
Intrahepatic bile duct disease (e.g., PBC, PSC) Gallstone obstruction of bile ducts (i.e., choledocholithiasis) Pancreatic or biliary cancer Biliary atresia
PBC, primary biliary cirrhosis; PSC, primary sclerosing cholangitis.
Physical examination: jaundice, hepatomegaly, ascites, liver bruits, abdominal distention, asterixis, varices Tests: Mildly increased ALT and AST, increased alkaline phosphatase, increased total and direct bilirubin, and increased α-fetoprotein Biopsy provides a histologic diagnosis CT, MRI, or US is useful for the detection of a hepatic mass Angiography is useful for mapping the vascularity of the tumor PET scan may help determine the presence of local extension and distant metastases Treatment: Lobectomy or partial hepatectomy is performed for small tumors Chemotherapy may be combined with resection of the tumor but is not highly effective against these cancers Liver transplantation is a possible option for localized disease without extrahepatic spread Radiofrequency ablation and chemoembolization may be performed for unresectable tumors to decrease their mass Outcomes: Complications include cirrhosis, portal vein obstruction, fulminant hepatic failure, and Budd-Chiari syndrome Paraneoplastic syndromes include hypoglycemia, excessive RBC production, refractory watery diarrhea, hypercalcemia, and variable skin lesions Prognosis is poor with a median survival of 4 years even following resection The median survival of unresectable disease is 3 months Why eliminated from differential: both diagnoses may have a similar presentation, but the absence of any liver lesions on the CT make this diagnosis unlikely
Case 3-12: “I keep getting bigger” A 75-year-old man is brought to his PCP by his son because of worsening malaise and increasing abdominal girth. The patient is known to be a recurrent alcoholic with a previous diagnosis of alcohol-related hepatic steatosis, and his son
is concerned that his condition is worsening due to his continued heavy alcohol consumption. The patient says that he feels weak and tired all of the time. He has occasional mild abdominal pain. He says that his abdomen has grown in size over the past month and that his slacks no longer fit him. He feels that he does not have much of an appetite because he is often nauseous. He denies vomiting, diarrhea, hematemesis, melena, hematochezia, back pain, or urinary symptoms. The patient says that he was told that he has a liver problem from drinking and some other medical problems for which he takes several medications whose names he cannot remember. The patient's son tells the doctor that while his father's list of symptoms is accurate the time course over which they have developed is slightly over a year. His father sometimes becomes confused and has occasional mood swings. He also says that in addition to the history of hepatic steatosis, his father has a history of hyperlipidemia, HTN, and CAD and takes ASA, simvastatin, enalapril, nifedipine, and atenolol. He has been an alcoholic for many years with occasional periods of abstinence but relapsed approximately 1 month ago. He does not smoke tobacco or use any illicit substances. There is a family history of HTN and CAD but no history of liver disease. On examination, the patient is an elderly man who is able to follow simple commands well and answer questions appropriately. He has mild scleral icterus and is faintly jaundiced. He has no detectable lymphadenopathy. He is notable for gynecomastia. Auscultation of his chest and back detects mildly decreased breath sounds at the base of the right lung field but no other abnormalities. His abdomen is distended and is notable for hepatosplenomegaly and a small fluid wave. Some faint varicose veins are visible in the periumbilical region. Examination of his extremities detects mild digital clubbing in his hands. His neurologic examination is notable for a slight delay in intentional movements but is otherwise normal. The following vital signs are measured: T: 99.1°F, HR: 70 bpm, BP: 123/79 mm Hg, RR: 20 breaths/min
Differential Diagnosis Cirrhosis, hepatic steatosis, pancreatitis, pancreatic cancer, hepatitis, CHF, cholecystitis, cholangitis, colorectal cancer, Wilson disease, PBC, PSC, hepatocellular carcinoma
Laboratory Data and Other Study Results CBC: WBC: 5.8, Hgb: 13.9, Plt: 240 Chem7: Na: 145 mEq/L, K: 3.7 mEq/L, Cl: 108 mEq/L, CO2: 22 mEq/L, BUN: 35 mg/dL, Cr: 1.4 mg/dL, Glu: 81 mg/dL Coags: PT: 17.9, INR: 1.9, PTT: 33.1 LFTs: AlkPhos: 189 U/L, ALT: 130 U/L, AST: 288 U/L, TBili: 2.9 mg/dL, DBili: 1.8 mg/dL Amylase: 114 U/L Lipase: 125 U/L Ammonia: 91 µmol/L Brain natriuretic peptide (BNP): 51 pg/mL Nutrition panel: albumin: 2.9 g/dL, Fe: 48 µg/dL, transferrin: 205 mg/dL, ferritin: 15 ng/mL Serum ceruloplasmin: 31 mg/dL HBsAg: negative HBeAg: negative Anti-HBc IgM/IgG: negative HCVAg: negative Anti-HCV: negative CXR: small pleural effusion in right diaphragmatic gutter; enlarged liver; lung fields clear; normal heart
appearance Abdominal CT: moderate ascites; hepatosplenomegaly; tortuous hepatic arteries; patchy fibrosis in all hepatic lobes; no focal masses of any abdominal organs detected Based on these results, the following studies are performed: Paracentesis of ascites: serum-ascites albumin gradient: 1.3 g/dL, WBC: 45 cells/mL, Glu: 81 mg/dL CT-guided liver biopsy: moderate hepatocyte necrosis and interspersed fibrosis
Diagnosis Cirrhosis (alcohol related), mild portal hypertension, mild to moderate hepatic encephalopathy
Treatment Administered The patient was encouraged to cease alcohol use and enrolled in a drinking cessation program to aid efforts The patient was started on a low protein and low salt diet with zinc supplementation to aid in the treatment of his ascites and encephalopathy The patient was started on lactulose and furosemide to treat his encephalopathy and ascites
Follow-up The patient experienced some symptomatic improvement following the initiation of therapy The patient was able to maintain his dietary restrictions and remained compliant with his medical regimen but continued to be intermittently noncompliant with alcohol abstinence Two years after his initial diagnosis, the patient developed progressive worsening of his cirrhosis Because of the patient's continued alcohol use, he was deemed ineligible for liver transplantation and died 18 months after his initial diagnosis
Steps to the Diagnosis Cirrhosis Persistent liver damage leading to parenchymal fibrosis and necrosis Causes include chronic alcoholism, chronic hepatitis (HBV or HCV), chronic bile duct obstruction, biliary and parenchymal hepatic disease, drugs, and toxins Intrahepatic fibrosis causes the portal vein pressure to exceed that of the inferior vena cava and results in portal hypertension Other less common intrahepatic causes include schistosomiasis, parenchymal liver disease, and granulomatous diseases Portal vein thrombosis is a prehepatic cause Posthepatic causes include right-sided heart failure, hepatic vein thrombosis, and Budd-Chiari syndrome (i.e., hepatic vein thrombosis due to coagulopathy) Hepatic encephalopathy results from the shunting of high ammonia-laden blood (not filtered in the liver due to hepatic dysfunction) into the systemic circulation
Causes of cirrhosis may be remembered by the mnemonic HEPATIC: Hemochromatosis, Enzyme deficiency (α1-antitrypsin), Primary biliary cirrhosis/sclerosing cholangitis, Alcoholism, Tumor (hepatoma), Infection (hepatitis), Chronic cholecystitis/copper (Wilson disease). History: Impaired hepatic function results in progressive weakness, weight loss, anorexia, and dyspnea Portal hypertension causes abdominal pain and hematemesis or hematochezia due to varices (Table 3-15) Hepatic encephalopathy causes confusion, drowsiness, and mood changes Physical examination: Impaired hepatic function results in hepatomegaly, splenomegaly, jaundice, palmer erythema, digital clubbing, Dupuytren hand contractures, testicular atrophy, gynecomastia
Table 3-15 Sites of Varices Due to Portal Hypertension
V aric es
Causative V eins
Gastroesophageal
Gastric and esophageal veins
Hemorrhoids
Superior, middle, and inferior rectal veins
Caput medusa
Periumbilical veins
Renal varices
Left renal, inferior phrenic, and adrenal veins
Paravertebral varices
Retroperitoneal paravertebral veins
Portal hypertension causes ascites, varices, spider telangiectasia Hepatic encephalopathy causes mental status changes and asterixis (i.e., asynchronous flapping of the hands) Tests: Increased ALT, AST, GGT, alkaline phosphatase, ammonia, and PT; decreased albumin, Hgb, and platelets Paracentesis of ascites shows a fluid with normal glucose, <300 WBC/µL, and an albumin level at least 1.1 g/dL greater than the serum level (i.e., serum-ascites gradient) Hepatic biopsy shows fibrosis and hepatocyte necrosis Abdominal CT is useful for detecting ascites and any hepatic masses EGD may be used to visualize esophageal varices Treatment: Stop all offending agents (e.g., alcohol)
Antivirals or α-IFN help slow progression due to hepatitis β-blockers or sclerotherapy may be used to reduce the bleeding risk from varices Lactulose, neomycin, and a low protein diet may help improve encephalopathy Salt restriction and diuretics are useful for decreasing ascites IV antibiotics are indicated for the development of spontaneous bacterial peritonitis Hepatic shunting via laparotomy or transjugular intrahepatic portal-caval shunting (TIPS) is a short-term solution for severe disease Liver transplantation may be required in progressive cases Outcomes: prognosis is generally poor because the condition is irreversible; severe disease carries a 50% mortality rate within 1 year Clues to the diagnosis: History: malaise, increased abdominal girth, weakness, fatigue, anorexia, nausea, abdominal pain, confusion, mood swings, history of hepatic steatosis Physical: scleral icterus, jaundice, gynecomastia, abdominal distention, hepatosplenomegaly, caput medusa, digital clubbing, impaired coordination Tests: increased PT and INR, increased LFTs, increased ammonia, decreased albumin, appearance of abdominal CT
Spontaneous bacterial peritonitis may result from systemic infection and comorbid portal hypertension; paracentesis will show >250 PMN/µL, total protein >1 g/dL, glucose <50 mg/dL, and LDH greater than normal serum LDH.
If paracentesis detects a very high albumin and LDH equal to 60% of the serum LDH, worry about a neoplastic etiology and perform a full work-up for cancer. Hepatic steatosis More thorough discussion in prior case Why eliminated from differential: this was the patient's diagnosis at some earlier point, but the multiple signs and symptoms of progressive hepatic injury is indicative of a progression to cirrhosis Pancreatitis More thorough discussion in prior case Why eliminated from differential: the normal amylase and lipase levels and the normal appearance of the pancreas on the CT rule out this diagnosis Pancreatic cancer More thorough discussion in prior case Why eliminated from differential: the normal pancreatic appearance on CT makes this diagnosis unlikely Viral hepatitis More thorough discussion in prior case
Why eliminated from differential: the negative serologic markers for HBV and HCV rule out these diagnoses as the cause of the patient's presentation Congestive heart failure More thorough discussion in Chapter 1 Why eliminated from differential: the normal BNP level and the nearly normal appearance of the CXR rule out this diagnosis Cholecystitis/cholangitis More thorough discussion in prior case Why eliminated from differential: the normal gallbladder appearance on the CT makes these diagnoses unlikely, but a HIDA scan could be performed to further rule them out as potential causes of cirrhosis related to biliary obstruction Colorectal cancer More thorough discussion in prior case Why eliminated from differential: the absence of any colonic abnormalities on the CT and the absence of any abnormal bowel habits on the history make this diagnosis unlikely Wilson disease More thorough discussion in prior case Why eliminated from differential: the normal serum ceruloplasmin rules out this diagnosis Primary sclerosing cholangitis/primary biliary cirrhosis More thorough discussion in prior case Why eliminated from differential: these diagnoses could be a cause of cirrhosis, but the patient's history of heavy alcohol use makes that a more likely explanation; in addition the biopsy findings help to rule out these diagnoses Hepatocellular carcinoma More thorough discussion in prior case Why eliminated from differential: the absence of any focal hepatic masses on the CT makes this diagnosis unlikely
Case 3-13: “My little boy keeps throwing up” An 8-week-old boy is brought to a pediatrician by his mother for projectile vomiting. She says that her son ate well without any difficulties initially after birth and had a normal 1-month well-baby visit. Shortly after he turned 1 month old he began to vomit following formula feeding. This vomiting became worse over the subsequent week and has been projectile in nature for the past 2 weeks. She says that she thought he did not tolerate the formula well, so she has tried changing the brand of formula several times. None of these changes have improved the vomiting. He is not gaining weight well, and she is concerned. She says that the emesis is pale colored and not bloody or bilious. He is having small pale brown stools several days per week. She denies that he coughs or has difficulty breathing. He did not develop jaundice in the first month of life. He was the product of an uncomplicated vaginal delivery at 39 weeks gestation. He has two siblings who are healthy without a similar history of vomiting. The mother had normal prenatal care without significant exposure to any teratogens. On examination, the infant is thin-appearing but hydrated.
Auscultation detects tachycardia but no abnormal heart and lung sounds. Active bowel sounds are detected. The child's abdomen is mildly distended, a 2 cm mass is palpable in the right upper quadrant of the abdomen. The child is a little lethargic but still actively moving all extremities. The following vital signs are measured: T: 98.7°F, HR: 175 bpm, BP: 70/45 mm Hg, RR: 35 breaths/min (Normal for 2-month infant: HR: 100–170 bpm, BP: 65–115/42–80, RR: 30–50 breaths/min)
Differential Diagnosis Pyloric stenosis, gastroenteritis, bowel obstruction, necrotizing enterocolitis, Hirschsprung disease, failure to thrive, esophageal atresia, duodenal atresia, GERD
Laboratory Data and Other Study Results Length, weight, and head circumference: 23.25" (65th percentile), 9.5 lb (8th percentile), 15.5" (49th percentile) Chem7: Na: 142 mEq/L, K: 3.3 mEq/L, Cl: 94 mEq/L, CO2: 22 mEq/L, BUN: 14 mg/dL, Cr: 0.4 mg/dL, Glu: 95 mg/dL AXR: mildly distended stomach; no bowel distention; no air-fluid levels Abdominal US: no liver or spleen abnormalities; enlarged pyloric channel length and wall thickness
Diagnosis Pyloric stenosis
Treatment Administered The infant was admitted to the hospital for IV hydration to improve his electrolyte balance Surgical pyloromyotomy was performed on the following day The child was discharged to home 1 day later when able to tolerate feedings
Follow-up Vomiting ceased following the procedure The infant was able to eat normally, and he gained weight
Steps to the Diagnosis Pyloric stenosis
Figure 3-20 Abdominal ultrasound demonstrating pyloric stenosis. Note the thin pyloric lumen (L) and the thickened pyloric musculature (defined by the region between x's and +x's).
Hypertrophy of the pyloric sphincter, causing partial or complete obstruction of the gastric outlet History: normal feeding for the first month of life, nonbilious emesis begins after 1 month of life and is frequently projectile in nature Physical examination: palpable olive-sized mass in the upper right quadrant of the abdomen, possible poor weight gain, possible signs of dehydration (e.g., dry mucus membranes, decreased skin turgor) Tests: Hypokalemia and hypochloremia develop with a delayed diagnosis US shows a pylorus with increased channel length and muscle thickness (Figure 3-20) Barium swallow may show a thin pyloric channel (i.e., string sign) Treatment: correction of electrolyte abnormalities with IV hydration; pyloromyotomy is curative Outcomes: prognosis is excellent with surgical correction; possible complications include dehydration, malnutrition, and iatrogenic bowel perforation during surgery Clues to the diagnosis: History: projectile vomiting, nonbilious and nonbloody emesis Physical: palpable hypertrophic pylorus, poor weight gain Tests: electrolyte abnormalities, pyloric appearance on US Gastroenteritis More thorough discussion in prior case
Why eliminated from differential: the chronicity of the vomiting, absence of diarrhea, and normal temperature make this diagnosis unlikely Bowel obstruction More thorough discussion in prior case Why eliminated from differential: the normal bowel appearance on AXR and history of bowel movements rule out this diagnosis Necrotizing enterocolitis Idiopathic mucosal necrosis and epithelial cell sloughing in neonates and young infants Risk factors: preterm birth, low birth weight History: bilious vomiting, lethargy, poor feeding, diarrhea, hematochezia Physical examination: abdominal distention, abdominal tenderness, possible hypotension Tests: metabolic acidosis, hyponatremia, increased WBC, and anemia; AXR will show bowel distention, air in the bowel wall and portal vein, and free air under the diaphragm Treatment: The infant is made NPO, a nasogastric tube is placed, and TPN is initiated Broad spectrum antibiotics are started Surgical resection of necrotic bowel is required Outcomes: mortality is approximately 25%; 50% of surviving patients will develop short-gut syndrome or bowel strictures Why eliminated from differential: the absence of bilious vomiting, diarrhea, and hematochezia and the normal-appearing AXR rule out this diagnosis Hirschsprung disease An absence of bowel autonomic innervation, resulting in bowel spasms and obstruction History: vomiting, obstipation, lack of a bowel movement within the first day of life Physical examination: abdominal distention, tympanic percussion of the abdomen Tests: AXR demonstrates bowel dilation Barium enema shows significant proximal colonic dilation (i.e., megacolon) with distal tapering Bowel biopsy will show an absence of ganglia Treatment: decompression through placement of a nasogastric or rectal tube; surgical resection of the affected bowel with the creation of a colostomy and a delayed reconstruction Outcomes: most children are able to achieve fecal continence after surgical reconstruction; complications include enterocolitis, bowel obstruction, and postoperative incontinence Why eliminated from differential: the absence of obstipation and the AXR appearance rule out this diagnosis Failure to thrive
Children below the third percentile in weight for a given age or the failure to gain weight appropriate for a given age Abuse (e.g., neglect) and illness are both possible etiologies History: failure of a child to maintain or gain weight; consider the full presentation to differentiate an organic from an abusive cause Physical examination: very low weight for age, possible signs of abuse, possible signs of another disease process Tests: tests that may be useful to determine the etiology of the condition include CBC, Chem10, UA, blood and urine cultures, cystic fibrosis testing, and food intake records Treatment: Treat underlying disorders Contact social services in cases of suspected abuse High calorie diet Educate parents in proper nutrition and feeding Outcomes: prognosis worsens as malnutrition worsens Why eliminated from differential: although this child's history and weight do not currently meet the criteria for this diagnosis, failure to thrive would be an appropriate diagnosis if there was a greater delay until presentation and further impaired weight gain Esophageal atresia Malformation of the trachea and esophagus, resulting in either blind pouches or fistula formation between the structures (i.e., tracheoesophageal fistula) (Figure 3-21) Typically diagnosed soon after birth History: coughing and cyanosis during feeding, excessive drooling, possible history of aspiration pneumonia Physical examination: abdominal distention, difficulty in placing a nasogastric tube Tests: CXR or AXR following attempted nasogastric tube placement may show the tube coiled in a blind pouch or in a bronchi
Figure 3-21 Variations of esophageal atresia and tracheoesophageal fistulas. The directions of airflow and ingestion are marked with arrows. (A) Esophageal atresia with a distal tracheoesophageal fistula; 90% of cases. (B) Blind pouches of both the proximal and distal esophagus; 5% to 8% of cases. (C) Complete esophagus with a tracheoesophageal fistula; 2% to 3% of cases. (D) Atresia of the distal esophagus with a tracheoesophageal fistula involving the proximal esophagus; 1% of cases. (E) Incomplete esophagus with tracheoesophageal fistulas involving both the proximal and distal pouches; 1% of cases.
Treatment: surgical reconstruction of the trachea and esophagus and resection of any fistula Outcomes: complications include anastomotic leaks, strictures, GERD, and esophageal dysmotility; the presence of low birth weight, pulmonary compromise, and secondary cardiac complications correlate with increased mortality Why eliminated from differential: the absence of coughing, cyanosis, or respiratory distress makes this diagnosis less likely Duodenal atresia Complete (i.e., blind pouch) or incomplete (i.e., stricture) obstruction of the duodenum due to a congenital malformation Found with an increased frequency in patients with Down syndrome History: vomiting soon after birth (most commonly bilious), poor feeding Physical examination: epigastric distention, weight loss, dehydration Tests: Hypokalemia, hypochloremia AXR will demonstrate an air-filled stomach and proximal end of the distal duodenal portion (i.e., doublebubble sign) US can detect the anatomic abnormality and air-filled pouches of the stomach and duodenum Barium swallow will show contrast pooling proximal to the obstruction Treatment: surgical reconstruction of the duodenum with resection of the obstruction Outcomes: mortality is <5% acutely, and prognosis is generally very good Why eliminated from differential: the appearance of the AXR and US rule out this diagnosis Gastroesophageal reflux disease More thorough discussion in prior case Why eliminated from differential: infants with severe GERD may vomit occasionally, but the progressive and projectile vomiting, presence of a palpable upper abdominal mass, and US appearance rule out this diagnosis
Case 3-14: “My baby is yellow” A 6-day-old infant girl is brought to the pediatrician by her parents because of jaundice. They say that their daughter became jaundiced 4 days after birth and has remained so since that time. She was a normal color when they left the hospital on the second day after her birth. The child is being formula-fed and seems to be eating normally. She is sleeping the majority of the day and is active when she is awake. She does not seen to be in any pain. She was the product of a full-term pregnancy with an uncomplicated vaginal delivery. No significant hereditary diseases exist in the family as far as the parents are aware. The mother underwent the standard recommended prenatal care and had
no significant teratogenic exposures during her pregnancy. On examination, the infant appears to be in no distress and actively moves all extremities while on the examination table. She is moderately jaundiced. Auscultation detects no abnormal heart, lung, or bowel sounds. She has normal primitive reflexes. The following vital signs are measured: T: 98.7°F, HR: 140 bpm, BP: 78/47 mm Hg, RR: 40 breaths/min (Normal for neonate: HR: 100–170 bpm, BP: 65–95/30–60, RR: 30–50 breaths/min)
Differential Diagnosis Physiologic jaundice, breast-milk jaundice, biliary atresia, Crigler-Najjar syndrome, hemolytic disease of the newborn, neonatal sepsis, glucose-6-phosphate dehydrogenase deficiency
Laboratory Data and Other Study Results Length, weight, and head circumference: 20.00" (72nd percentile), 7.8 lb (62nd percentile), 13.9" (64th percentile) (birth weight 8.1 lb) CBC: WBC: 9.4, Hgb: 15.5, Plt: 230 Reticulocyte count: 1.5% Bilirubin: TBili: 19.4 mg/dL, DBili: 0.4 mg/dL Maternal blood type: A+ Neonatal blood type: A+
Diagnosis Physiologic jaundice
Treatment Administered Home phototherapy was started
Follow-up Jaundice resolved in the infant by 12 days after birth
Steps to the Diagnosis Physiologic jaundice Jaundice in neonates that occurs due to increased hemolysis and decreased bilirubin excretion in the first week of life History: neonates are asymptomatic Physical examination: jaundice that most commonly develops between the 1st and 3rd days of life Tests: Transcutaneous bilirubinometry is typically sufficient to rule out extremely high levels of bilirubin Increased total and indirect bilirubin Cases that present before 24 hours or after 3 days may suggest a work-up including CBC, reticulocyte
count, serum bilirubin levels, blood typing, LFTs, albumin level, blood cultures, and US Treatment: phototherapy is frequently successful at helping to decrease bilirubin levels and is indicated for bilirubin levels >18 mg/dL (with some variation to account for the general medical health of the infant); exchange transfusion is reserved for cases of bilirubin >22 mg/dL (with some variation per the infant's health) Outcomes: prognosis is excellent, and kernicterus (i.e., bilirubin encephalopathy) is very rare with the appropriate treatment Clues to the diagnosis: History: bilirubin within the 1st week of life, no family history of bilirubinemia Physical: jaundice is an otherwise healthy appearing child Tests: bilirubin level Breast milk jaundice Benign jaundice seen in breast-fed neonates that occurs due to the inhibition of a hepatic enzyme by a metabolite of progesterone found in breast milk History: neonates are asymptomatic Physical examination: jaundice in the 2nd week of life in breast-fed neonates Tests: the same work-up is performed as for physiologic jaundice Treatment: reassurance by the physician that the jaundice is benign; phototherapy may be prescribed for high bilirubin levels Outcomes: prognosis is excellent, and kernicterus is extremely rare Why eliminated from differential: the neonate in the case is not being breast fed Biliary atresia Discontinuity of the extrahepatic bile ducts, resulting in the obstruction of normal bile flow History: light-colored stools and dark urine are seen in diapers Physical examination: persistent jaundice, hepatomegaly Tests: increased total and direct bilirubin; US will detect abnormalities of the biliary tree Treatment: surgical reconstruction of the biliary tree to allow drainage; liver transplantation is frequently eventually required Outcomes: Complications include portal hypertension, cirrhosis, hepatocellular carcinoma, and kernicterus Five-year survival is up to 60% following surgical reconstruction Liver transplantation is required once hepatic failure progresses Why eliminated from differential: this diagnosis would be considered if the jaundice had persisted past 2 weeks and if there an excess of direct bilirubin in addition to indirect bilirubin Crigler-Najjar syndrome An autosomal-recessive disease of a severe deficiency in glucuronosyltransferase
History: progressive multiple neurologic abnormalities that develop soon after birth Physical examination: jaundice at birth that remains persistent, hypotonia, deafness, poor vision, lethargy Tests: total bilirubin frequently >17 mg/dL that mainly consists of the indirect component Treatment: Exchange transfusion Long-term phototherapy Phenobarbital and heme oxygenase inhibitor may decrease the CNS effects Outcomes: kernicterus is frequently present at the time of birth, and death usually occurs within the first few years after birth Why eliminated from differential: the several-day delay in the appearance of jaundice and the absence of signs of kernicterus make this diagnosis unlikely Hemolytic disease of the newborn More thorough discussion in Chapter 6 Why eliminated from differential: the mother and child share the same blood type Neonatal sepsis More thorough discussion in Chapter 1 (i.e., septic shock) and Chapter 6 Why eliminated from differential: the healthy appearance of the neonate, normal vital signs, and normal CBC rule out this diagnosis Glucose-6-phosphate dehydrogenase deficiency More thorough discussion in Chapter 6 Why eliminated from differential: this relatively benign condition may present similarly to physiologic jaundice; should recurrent jaundice occur in childhood, a measurement of glucose-6-phosphate dehydrogenase activity may be warranted
Authors: Van Kleunen, Jonathan P. Title: Step-Up to USMLE Step 3, 1st Edition Copyright ©2009 Lippincott Williams & Wilkins > Table of Contents > Chapter 4 - Nephrology and Urology
Chapter 4 Nephrology and Urology Basic clinical primer Renal Physiology The key functions of the kidneys are to filter plasma (20% of all plasma flow), regulate intravascular fluid volume and electrolyte composition, aid in the maintenance of homeostasis between intravascular and extravascular fluids, and secrete several hormones that are important to systemic hemodynamics (Figure 4-1, Table 4-1)
Carbonic anhydrase is the catalyst for HCO3- resorption in the proximal convoluted tubule.
Diuretics Medications that affect the resorption of electrolytes and fluids function at distinct locations along the renal tubular system to modulate the composition and volume of body fluids (Table 4-2)
Dialysis Therapeutic filtering of intravascular serum through either artificial or induced means Indicated for severe hyperkalemia or metabolic acidosis, intravascular fluid overload, uremic syndrome, or chronic kidney disease with a creatinine >12 mg/dL and a blood urea nitrogen >100 mg/Dl Hemodialysis The creation of an arteriovenous fistula or the placement of a synthetic graft is performed to allow consistent vascular access Blood is filtered by a machine and returned to the systemic circulation via the selected conduit Peritoneal dialysis Dialysis fluid is temporarily pumped into the peritoneal cavity via an access catheter The osmotic gradient drives the diffusion of substances from the intravascular serum to the peritoneum with the peritoneal membrane serving as a filter Following dialysis, the fluid is pumped out of the peritoneal cavity
Acid-Base Physiology In healthy individuals, serum pH is regulated by the reabsorption of bicarbonate (HCO3-) and the partial pressure of carbon dioxide (pCO2) of blood pH, partial pressure of oxygen (pO2), and pCO2 are measured directly by an arterial blood gas, and bicarbonate may be measured by the Henderson-Hasselbach equation:
P.108
Figure 4-1 Anatomy of the nephron and major sites of ion, water, and molecule exchange. Ca, calcium; Cl, chloride; H, hydrogen; H 2O, water; HCO3,; K, potassium; Mg, magnesium; Na, sodium. (See color image.)
Disturbances due to HCO3- abnormalities are metabolic; disturbances due to pCO2 levels are respiratory Compensatory mechanisms attempt to keep the serum pH normalized (Table 4-3) Anion gap Difference between the sodium positive ion concentration and chloride and bicarbonate negative ion concentrations Anion gap = [Na] - [Cl] - [HCO3-] (normal = 8–12)
Table 4-1 Functions of Regions of the Nephron
Region
Proximal convoluted
Loc ation
Absorption
Cortex
Glucose, bicarbonate, amino acids, metabolites, Na; Cl and water
tubule
Sec retions
Notes
Organic acids and bases
—
—
Descending limb is permeable
(passively)
Descending loop of
Medulla
Water (due to increasing interstitial osmotic gradient)
Henle
to water
Ascending loop of
Medulla
Na, Cl, K (Na/K/Cl cotransporter); Mg, Ca, K (paracellular
Henle
—
—
diffusion)
Distal convoluted
Cortex
Na and Cl (Na/Cl transporter); Ca (PTH activity)
—
Cells impermeable to water
Collecting tubule and
Cortex,
Na (aldosterone activity on principal cells); water (ADH activity);
K (aldosterone activity); H+
—
duct
medulla
K (inter-calated cells)
(intercalated cells)
tubule
ADH, antidiuretic hormone; Ca, calcium; Cl, chloride; H+, hydrogen ions; K, potassium; Mg, magnesium; Na, sodium; PTH, parathyroid hormone
P.109
Table 4-2 Common Diuretics and Their Effects Within the Nephron
Diuretic
Site of Ac tion
Mec hanism of A c tion
Indic ations
A dverse Effec ts
Carbonic anhydrase inhibitors
Proximal convoluted
Inhibition of carbonic anhydrase causes mild
Glaucoma, epilepsy, altitude
Mild metabolic acidosis,
(e.g, acetazolamide)
tubule
diuresis and prevents HCO 3- reabsorption
sickness, metabolic alkalosis
hypokalemia, nephrolithiasis
Osmotic agents (e.g.,
Proximal convoluted
Increased tubular osmotic gradient increases
Increased intracranial pressure,
No effect on Na excretion,
mannitol, urea)
tubule, loop of Henle,
H2 O excretion
acute renal failure (due to shock or
relative hypernatremia
collecting tubule
Loop diuretics (e.g.,
Ascending loop of Henle
furosemide)
Thiazides (e.g.,
Distal convoluted tubule
hydrochlorothiazide)
drug toxicity)
Inhibit Na/Cl/K cotransporter to decrease
CHF, pulmonary edema,
Ototoxicity,
reabsorption and indirectly inhibit Ca
hypercalcemia; rapid onset useful in
hyperuricemia,
reabsorption
emergent situations
hypokalemia, hypocalcemia
Inhibit Na/Cl cotransporter to decrease
HTN, CHF, hypercalciuria, diabetes
Hypokalemia,
reabsorption and indirectly increase K
insipidus
hyperuricemia,
excretion and increase Ca reabsorption
K-sparing (e.g.,
Collecting tubules
spironolactone)
hypercalcemia
Aldosterone antagonist that inhibits Na-K
Secondary hyper-aldosteronism,
Gynecomastia, menstrual
exchange
CHF, K-preserving diuresis
irregularity, hyperkalemia
Ca, calcium; CHF, congestive heart failure; HCO 3, bicarbonate; H2O, water; HTN, hypertension; K, potassium; Na, sodium.
Metabolic acidosis with a normal anion gap suggests bicarbonate loss Metabolic acidosis with an increased anion gap suggests hydrogen ion excess Mixed acid-base disturbances Apparent if the corrected bicarbonate is different than the measured value: Corrected HCO3- = measured anion gap - normal anion gap + measured HCO3The normal anion gap is considered to be 12 A normal corrected bicarbonate suggests a high anion gap acidosis alone An increased corrected bicarbonate suggests a metabolic alkalosis with a high anion gap acidosis
A decreased corrected bicarbonate suggests a normal anion gap acidosis and a high anion gap acidosis through separate mechanisms
Causes of high-anion gap metabolic acidosis can be remembered by the mnemonic MUD PILES: Methanol, Uremia, Diabetic ketoacidosis, Paraldehyde, Isoniazid (INH), Lactic acidosis, Ethanol, Salicylates.
Preoperative Renal Risk Assessment Electrolyte abnormalities, anemia, and impaired immune function are more common in patients with renal insufficiency Preoperative dialysis is frequently indicated in patients with renal insufficiency to decrease the risk of complications due to serum composition Acetylcysteine may be used as a preoperative renal protectant in patients with renal insufficiency who are scheduled to receive intravascular contrast during surgery
Kidney Transplantation Indicated for end-stage renal disease requiring dialysis for survival Contraindicated in patients with significant comorbidities that carry a poor prognosis Living donor transplants are associated with a 20% risk of acute rejection and a 5-year survival of 91% Allografts carry a 40% acute rejection rate and a 5-year survival rate of 85% P.110
Table 4-3 Acid-Base Disturbances and Compensatory Mechanisms
Case 4-1: “It hurts when I go to the bathroom” A 26-year-old woman presents to her primary care physician (PCP) with complaints of multiple urinary symptoms and back pain that have not responded to therapy. She was seen in the same clinic 2 weeks prior to the current presentation with pain during urination, increased urinary frequency, and urgency prior to urination. She was given a 3-day prescription for trimethoprim-sulfamethoxazole at that visit. The patient did not fill the prescription until 2 days ago because of her busy schedule, although she was continuing to have symptoms. Today she reports nausea, vomiting, chills, and flank pain in addition to continued dysuria, urgency, and urinary frequency. She denies diarrhea, abdominal pain, hematochezia, dyspareunia, or dizziness. She denies any past medical history and uses oral contraceptive pills. She is in a monogamous relationship, and her last menstrual period was 1 week ago. She drinks alcohol socially but denies any other substance use. On examination, she is an uncomfortable-appearing thin woman. She has no palpable lymphadenopathy. Auscultation finds no abnormal sounds of her heart, lungs, or bowels. P.111 She does not have any abdominal masses but has mild suprapubic tenderness. She is tender to palpation along the posterior aspects of her lower ribs. Pelvic examination reveals no tenderness, vaginal discharge, or masses. Neurologic examination is normal. The following vital signs are measured: Temperature (T): 102.7°F, heart rate (HR): 96 beats per minute (bpm), blood pressure (BP): 115/80 mm Hg, respiration rate (RR): 16 breaths/min
Differential Diagnosis Urinary tract infection, pyelonephritis, nephrolithiasis, appendicitis, cervicitis, endometriosis, ectopic pregnancy, gastroenteritis
Laboratory Data and Other Study Results Complete blood cell count (CBC): white blood cells (WBC): 20.1, hemoglobin (Hgb): 14.9, platelets (Plt): 320 Urinalysis (UA): cloudy, pH: 7.5, specific gravity: 1.030, positive nitrites and leukocyte esterase; no glucose/ketones/hematuria/proteinuria; 2 RBC/hpf (red blood cells/high-power field), 35 WBC/hpf Urine culture: Gram stain shows multiple Gram-negative rods and leukocyte casts; culture pending Blood culture: Gram stain and culture pending Vaginal fluid microscopy (saline): no mobile organisms or clue cells Vaginal fluid microscopy potassium hydroxide (KOH): no hyphae or pseudohyphae Vaginal fluid culture: culture pending Abdominal CT with contrast: no bowel or vascular abnormalities; no renal masses or stones Urine pregnancy test: negative
Diagnosis Pyelonephritis
Treatment Administered The patient was admitted to the hospital for intravenous (IV) antibiotics because of her prior noncompliance with prescribed treatment and concern over her ability to take oral medication (she had reported new episodes of emesis) The patient was started on empiric levofloxacin while awaiting the culture results
Follow-up The urine culture grew >10,000 colonies of Escherichia coli The patient was continued on IV levofloxacin for 2 additional days until her symptoms improved The patient was discharged to home to complete a 14-day course of oral levofloxacin The patient had full resolution of her symptoms and completed the full antibiotic course
Steps to the Diagnosis Acute pyelonephritis Infection of the renal parenchyma E. coli, Staphylococcus saprophyticus, Klebsiella pneumoniae, Proteus mirabilis, and Candida albicans are the most common causes Most commonly occurs as a sequela of an ascending urinary tract infection (UTI) P.112 Risk factors: urinary obstruction, immunocompromised, previous pyelonephritis, diabetes mellitus (DM), frequent sexual intercourse (more than
3 times per week), spermicide use, and intercourse with a new partner History: flank pain, chills, nausea, vomiting, urinary frequency, dysuria, urinary urgency Physical examination: fever, costovertebral tenderness, suprapubic tenderness Tests: Increased WBCs, erythrocyte sedimentation rate (ESR), and C-reactive protein (CRP) UA will be positive for nitrites, leukocyte esterase, and WBCs Urine Gram stain will show white cell casts, and culture will grow more than 10,000 bacteria/mL of urine Treatment: IV fluoroquinolones, aminoglycosides, or third-generation cephalosporins are given empirically until an organism is identified Patient may be changed to oral medications after a few days and may be allowed to complete a 14-day course as an outpatient Severe cases may require up to 14 days of IV antibiotics Reliable patients who are hemodynamically stable, lack other medical comorbidities, and are capable of taking oral medications may be amenable to oral antibiotics alone in an outpatient setting Outcomes: complications include renal abscess formation, acute renal failure, chronic kidney disease, and increased risks for preterm labor and low neonatal birth weight in pregnant females; the prognosis for uncomplicated cases is good Clues to the diagnosis: History: multiple urinary symptoms, flank pain, noncompliance with treatment for a diagnosis of UTI Physical: fever, flank tenderness Tests: UA and urine Gram stain suggestive of infection
Fluoroquinolones have comparable bioavailability for the oral and IV formulations. Urinary tract infection Ascending infection of the urethra, bladder, and ureters Most commonly due to direct inoculation of the lower urinary tract; rarely caused by hematogenous spread E. coli, S. saprophyticus, K. pneumoniae, P. mirabilis, or Enterobacter faecalis are the most common causes Risk factors: obstruction, catheterization, vesicoureteral reflux, pregnancy, DM, sexual intercourse, immunocompromise, female gender History: urinary frequency and urgency, dysuria, suprapubic pain Physical examination: suprapubic tenderness Tests: UA will show nitrites, leukocyte esterase, and WBCs; urine culture will show more than 10,000 colony-forming units/mL Treatment: oral amoxicillin, trimethoprim-sulfamethoxazole (TMP-SMX), or fluoroquinolones for 3 days; the treatment time increases to 14 days for relapsing infecting Outcomes: complications include abscess formation and pyelonephritis; prognosis is excellent unless patient is elderly, immunocompromised, or has underlying kidney disease Why eliminated from differential: the patient likely initially had a UTI that ascended and progressed to pyelonephritis, but the worsening of her symptoms and the onset of fever and back pain are more suggestive of a diagnosis of pyelonephritis
Cultured urine should be from a midstream sample (i.e., clean catch) to avoid contamination from skin flora.
Perform a work-up for sexually transmitted urethritis in any male with a suspected UTI because the symptoms may appear similar. Nephrolithiasis Formation of concentrated masses of minerals (i.e., kidney stones) in the ductal system of the urinary tract (Table 4-4) Risk factors: hypercalcemia, hyperparathyroidism, prior nephrolithiasis, family history, low fluid intake, frequent UTIs, hypertension (HTN), DM, gout, renal tubular acidosis, allopurinol, acetazolamide, loop diuretics, and male gender History: stones remain asymptomatic until obstruction of the urinary outflow tract occurs and then acute severe colicky flank pain, nausea,
vomiting, and dysuria occur P.113
Table 4-4 Types of Nephrolithiasis (Renal Stones)
Ty pe
Frequenc y (%)
Cause
Radiology
Notes
Calcium oxalate
72
Idiopathic hy perc alc iuria, small bowel diseases
Radiopaque
Most patients have no identifiable cause
Struvite (Mg-NH4-PO 4)
12
Urinary trac t infec tion
Radiopaque
More common in w omen; may form staghorn calculi
Calcium phosphate
8
Hy perparathy roidism, renal tubular acidosis
Radiopaque
—
Uric acid
7
Chronic acidic/concentrated urine, chemotherapeutic drugs, gout
Radioluc ent
Treat by alkalinizing urine
Cystine
1
Cystinuria, amino acid transport defects
Radiopaque
May form staghorn calculi
Physical examination: acute costovertebral tenderness Tests: UA will show hematuria in most cases (Table 4-5) Serum electrolytes (including calcium and phosphorus), uric acid level, and parathyroid hormone level are useful for determining the etiology of stones Twenty-four-hour urine collection is useful for determining the composition of stones and their etiology Abdominal x-ray (AXR), computed tomography (CT), or ultrasound (US) will detect the majority of stones Intravenous pyelogram (IVP) is the gold standard for detecting stones and measuring their size but is performed uncommonly because of the time requirements and contrast dye load (Figure 4-2) Water-soluble dye is injected intravenously and excreted by the kidneys Appropriate timing of postinjection imaging will demonstrate filling defects associated with impacted stones and hydronephrosis (i.e., dilation) of the renal calyces
Table 4-5 Common Causes of Hematuria
A ge
Below 20 years old
Temporary Hematuria
Idiopathic
Persistent Hematuria
Glomerular disease
UTI
Exercise
Trauma
Endometriosis (women)
20–50 years old
Idiopathic
Adult polycystic kidney disease
UTI
Neoplasm (e.g., bladder, kidney, prostate)
Nephrolithiasis
Glomerular disease
Exercise
Trauma
Endometriosis (women)
Beyond 50 years old
Idiopathic
Adult polycystic kidney disease
UTI
BPH (men)
Nephrolithiasis
Neoplasm (e.g., bladder, kidney, prostate)
Trauma
Glomerular disease
BPH, benign prostatic hyperplasia; UTI, urinary tract infection.
P.114
Figure 4-2 Intravenous pyelogram demonstrating hydronephrosis in the right kidney (asterisk). Renal pelvis dilation is evident as is a radiopaque stone in the right ureter (arrow). The left kidney appears normal. (See color image.)
Treatment: Hydration and analgesics for pain control Shockwave lithotripsy may be useful to break up stones <3 cm and allow them to pass through the ureters Surgical removal is indicated for large impacted stones Chronic cases with the development of hydronephrosis may require drainage with a nephrostomy tube and placement of a double-J stent to allow continued urine flow Outcomes: potential complications include hydronephrosis, renal failure, recurrent stones, fistula formation, and ureteral stricture formation; recurrences occur in 50% of cases within 5 years of the first episode Why eliminated from differential: the absence of stones seen on the CT and the multiple signs suggestive of an infectious process (e.g., fever, WBCs) make this diagnosis unlikely
Patients with impacted stones will be in pain and will shift position frequently in unsuccessful attempts to
find a comfortable position; patients with peritonitis will remain rigid.
Rule out bladder or urethral obstruction in an anuric patient by attempting bladder catheterization.
The differential diagnosis for adult hematuria can be remembered by mnemonic INEPT GUN: Idiopathic, Neoplasm (e.g., bladder, kidney, prostate), Exercise, Polycystic kidney disease, Trauma, Glomerular disease (nephritic, nephrotic), UTI, Nephrolithiasis. Appendicitis More thorough discussion in Chapter 3 Why eliminated from differential: the appearance of the CT makes this diagnosis less likely; although nausea and abdominal pain may be seen in each diagnosis, the presence of flank pain and urinary symptoms are more suggestive of pyelonephritis Cervicitis More thorough discussion in Chapter 11 Why eliminated from differential: the normal pelvic examination rules out this diagnosis Endometriosis More thorough discussion in Chapter 11 Why eliminated from differential: the normal pelvic examination makes this diagnosis unlikely; likewise, urinary frequency and urgency are not typically seen in this diagnosis Ectopic pregnancy More thorough discussion in Chapter 12 Why eliminated from differential: the negative pregnancy test rules out this diagnosis Gastroenteritis More thorough discussion in Chapter 3 Why eliminated from differential: the absence of epigastric pain and diarrhea makes this diagnosis less likely P.115
Case 4-2: “My sides are lumpy” A 35-year-old man presents to his PCP with flank pain and palpable flank masses. He says that a few weeks prior to his appointment he began to experience mild pain in his right flank. When he went to rub the area that was sore, he realized that he had a palpable lumpy mass in that location. He also detected a smaller lumpy mass on the left flank that was painless. He does not recall noticing these lumps prior to this episode of back pain. He denies any weight loss, weakness, nausea, vomiting, diarrhea, hematemesis, hematochezia, constipation, dysuria, or urinary frequency. He does admit to occasional headaches and infrequent episodes of brown urine upon questioning. He denies any past medical history but has not seen a physician for 5 years. He drinks alcohol socially and denies other substance use. On examination, he is a healthy appearing man in no distress. Head and neck, lung, and heart examinations are normal. Auscultation of his abdomen detects normal bowel sounds. He has no abdominal tenderness or hepatosplenomegaly. He has palpable irregular masses on both posterior flanks that are mildly tender to palpation. The masses feel deep. There is no overlying erythema or swelling. Neurologic examination is normal. The following vital signs are measured: T: 98.5°F, HR: 74 bpm, BP: 142/88 mm Hg, RR: 16 breaths/min
Differential Diagnosis Renal cell carcinoma, polycystic kidney disease, Wilms tumor, neuroblastoma, nephrolithiasis, pyelonephritis
Laboratory Data and Other Study Results CBC: WBC: 7.5, Hgb: 16.7, Plt: 279 10-electrolyte chemistry panel (Chem10): sodium (Na): 141 mEq/L, potassium (K): 4.2 mEq/L, chloride (Cl): 106 mEq/L, carbon dioxide (CO2): 27 mEq/L, blood urea nitrogen (BUN): 33 mg/dL, creatinine (Cr): 1.6 mg/dL, glucose (Glu): 87 mg/dL, magnesium (Mg): 1.9 mg/dL, calcium (Ca): 8.1 mg/dL, phosphorus (Phos): 3.2 mg/dL UA: light brown colored, pH: 6.0, specific gravity: 1.030, 1+ proteinuria, 1+ hematuria, no glucose/ketones/nitrites/leukocyte esterase Urine culture: Gram stain negative; culture pending Renal US: significantly enlarged multicystic kidneys bilaterally; no renal stones
Diagnosis Adult polycystic kidney disease
Treatment Administered The patient was started on enalapril to control his HTN The patient was given acetaminophen for pain control
Follow-up The patient continued to be followed closely to trace any worsening of his renal function The patient was able to maintain sufficient renal function for another 15 years before reaching end-stage renal disease and requiring dialysis The patient eventually received a renal transplant
Steps to the Diagnosis Polycystic kidney disease (adult) Hereditary syndrome characterized by the formation of multiple cysts in the kidneys and possibly the liver or spleen (Color Figure 4-1) P.116 Multiple cysts cause impaired renal function and eventual renal failure Types: Autosomal dominant: most common form and is diagnosed in adults; characterized by large multicystic kidneys with impaired function Autosomal recessive: rare form that is diagnosed in children; fatal in the initial years of life without a renal transplant History: typically asymptomatic until adulthood and then characterized by flank pain, possible chronic UTIs, and gross hematuria Physical examination: palpable kidneys, HTN, flank tenderness that becomes worse with cyst rupture Tests: Increased creatinine and BUN; increased Hgb (due to increased erythropoietin activity) UA will show proteinuria and hematuria Renal US or CT will show large multicystic kidneys Treatment: Controlling HTN (angiotensin receptor blockers [ARBs] or angiotensin-converting enzyme inhibitors [ACE-Is]) and treating UTIs will help to preserve renal function Vasopressin receptor antagonists or amiloride may help prevent the accumulation of fluid in cysts Therapeutic drainage of large cysts may improve pain Dialysis or renal transplant is required when renal function deteriorates Outcomes: End-stage renal disease will occur in 50% of patients before 60 years of age, and renal transplant is a common need in these patients Hepatic and splenic cysts compromise the function of those respective organs Intracranial aneurysm may occur in up to 10% of patients Other complications include subarachnoid hemorrhage, mitral valve prolapse, diverticulosis, and nephrolithiasis Clues to the diagnosis: History: flank pain, palpable flank masses, hematuria Physical: palpable flank masses, HTN Tests: increased creatinine and BUN, hematuria and proteinuria on UA, renal US findings Renal cell carcinoma
Most common primary malignant neoplasm of the renal parenchyma Risk factors: tobacco use, cadmium or asbestos exposure History: weight loss, flank pain, hematuria, malaise Physical examination: palpable abdominal or flank mass, HTN, fever Tests: Increased Hgb UA shows hematuria US, magnetic resonance imaging (MRI), or CT with contrast may show a renal mass (cystic or solid) Biopsy provides a histologic diagnosis but is rarely performed because the diagnosis is frequently made with imaging alone Treatment: nephrectomy or a renal-sparing resection with lymph node dissection is routinely performed for localized disease; immunotherapy, radiation therapy, or chemotherapy are utilized in unresectable disease Outcomes: localized tumors have at least an 88% 5-year survival rate with timely resection; regional disease and disease with metastases carry 5-year survival rates of 59% and 20%, respectively Why eliminated from differential: although renal cell carcinoma may be cystic, a multicystic kidney is more characteristic of polycystic kidney disease
Paraneoplastic syndromes associated with renal cell carcinoma include anemia, hepatic dysfunction, hypercalcemia, cachexia, erythrocytosis, thrombocytosis, and polymyalgia rheumatica. Wilms tumor A malignant renal tumor that is most commonly diagnosed in young children (average age is 4 years old) P.117 Risk factors: family history, neurofibromatosis, congenital genitourinary abnormalities History: weight loss, nausea, vomiting, dysuria, urinary frequency Physical examination: palpable abdominal or flank mass, HTN, fever Tests: Increased creatinine and BUN A renal mass will be apparent on CT, MRI, or US Biopsy of the tumor will demonstrate a combination of epithelial cells, blast cells, and stromal cells Treatment: surgical resection or nephrectomy, chemotherapy, and radiation therapy Outcomes: survivorship is 90% in cases without regional or metastatic spread Why eliminated from differential: the age of the patient in this case is greater than the typical age for this cancer Neuroblastoma Tumors of a neural crest cell origin that may arise in the adrenal glands or sympathetic ganglia The majority of patients are younger than 2 years old Risk factors: neurofibromatosis, tuberous sclerosis, pheochromocytoma, Beckwith-Wiedemann syndrome, Turner syndrome, low maternal folate consumption History: abdominal or flank distention and pain, weight loss, malaise, bone pain, diarrhea Physical examination: fever, palpable abdominal or flank mass, HTN, possible Horner syndrome, proptosis, movement disorders, hepatomegaly, periorbital bruising Tests: possible increased vanillylmandelic and homovanillic acids in a 24-hour urine collection; CT may detect an adrenal or ganglion tumor Treatment: surgical resection of tumors followed by postoperative chemotherapy and radiation therapy Outcomes: poor prognosis if presenting after one year of age with metastases to the bone and brain Why eliminated from differential: the patient's age makes this an unlikely diagnosis Nephrolithiasis
More thorough discussion in prior case Why eliminated from differential: the presence of the flank masses makes this diagnosis unlikely, and the US helps to rule it out Pyelonephritis More thorough discussion in prior case Why eliminated from differential: the absence of findings suggesting an infectious process makes this diagnosis unlikely
Case 4-3: “I just don't want to eat anything” A 75-year-old man is brought to the emergency department by his wife because of worsening fatigue and anorexia over the past week. His wife says that he had gastroenteritis 2 weeks ago and was having frequent diarrhea for several days before the condition self-resolved. He says that he never got his appetite back after the illness and has eaten little over the past week. He says that he has felt progressively worse over this time. He also reports headaches and nausea over this same time. He has had difficulty moving his bowels and has urinated only a couple of times in the past week. He denies chest or abdominal pain, dyspnea, chills, current vomiting or diarrhea, hematemesis or hematochezia, paresthesias, or chronic weight loss. He notes a history of HTN, coronary artery disease (CAD), and type II DM and takes aspirin (ASA), hydrochlorothiazide (HCTZ), atenolol, and metformin. He says that his blood pressure and blood sugar levels have been well controlled on this regimen. He denies substance use. On examination, he is a tiredappearing man in no significant distress. He has dry mucous membranes and decreased skin turgor. Auscultation of his heart and lungs finds clear breathing sounds, tachycardia, and a mild friction rub. Examination of his abdomen finds no abdominal tenderness or masses, mild flank P.118 tenderness, and decreased bowel sounds. Rectal examination detects no gross blood or masses. Neurologic examination is normal. The following vital signs are measured: T: 100.1°F, HR: 117 bpm, BP: 90/75 mm Hg, RR: 16 breaths/min
Differential Diagnosis Acute renal failure, interstitial nephropathy, chronic kidney disease, diabetic ketoacidosis, pyelonephritis, congestive heart failure (CHF)
Laboratory Data and Other Study Results CBC: WBC: 7.2, Hgb: 15.7, Plt: 278 Chem10: Na: 146 mEq/L, K: 5.1 mEq/L, Cl: 108 mEq/L, CO2: 23 mEq/L, BUN: 93 mg/dL, Cr: 3.9 mg/dL, Glu: 96 mg/dL, Mg: 2.1 mg/dL, Ca: 8.7 mg/dL, Phos: 4.1 mg/dL BNP: 34 pg/mL UA: dark yellow colored, pH: 6.0, specific gravity: 1.041, no glucose/ketones/nitrites/leukocyte esterase/hematuria/proteinuria Urine culture: Gram stain negative; culture pending Based on these results, the following tests are performed following admission to the hospital: Urine electrolytes: Na: 20 mEq/L, K: 18 mEq/L, Cr: 70 mg/dL Fractional excretion of sodium: 0.8 Renal US: normal size kidneys; no hydronephrosis; no detectable obstruction
Diagnosis Acute renal failure secondary to volume depletion
Treatment Administered HCTZ and metformin were held following his admission; insulin was used to regulate his glucose levels IV hydration was administered to gradually improve his volume depletion
Follow-up After 2 days of supportive care, the patient's BUN and creatinine began to trend downward Patient was discharged to home once his BUN and creatinine reached 35 mg/dL and 2.0 mg/dL, respectively The patient was kept on a low salt and low protein diet as an outpatient The patient had further improvement in his renal function but maintained a new baseline creatinine of 1.4 mg/dL
Causes of acute renal failure may be remembered by the mnemonic “Patients with ARF can't VOID RIGHT”:
Vasculitis, Obstruction (e.g., calyces, bladder, or ureters), Infection, Drugs (i.e., acute tubular necrosis [ATN]), Renal artery stenosis, Interstitial nephropathy, Glomerular disease, Hypovolemia, Thromboembolism.
Steps to the Diagnosis Acute renal failure (ARF) Acute worsening in renal function (e.g., impaired glomerular filtration, urine production, and chemical excretion) due to either a prerenal, intrarenal, or postrenal cause Prerenal causes include hypovolemia, sepsis, and renal artery stenosis Intrarenal causes include ATN (e.g., drugs, toxins), glomerular disease, and renal vascular disease Postrenal disease is due to the obstruction (e.g., stones, tumor, adhesions) of the renal calyces, ureters, or the bladder History: Patients are asymptomatic until a significant degree of glomerular filtration (approximately 50%) is lost P.119 Early symptoms include fatigue, anorexia, nausea, oliguria or anuria, and flank pain Gross hematuria and mental status changes develop with progressive renal injury and the development of uremia Physical examination: possible pericardial friction rub, hypotension, fever, diffuse rash, or edema Tests: Increased BUN and creatinine UA findings vary with the etiology Normal results may be seen with hypovolemia Hematuria and red cell casts are seen in glomerular or vasculitic diseases Granular casts are seen with ATN Pyuria is seen in infection Pyuria with waxy casts are seen in interstitial disease and in obstruction Fractional excretion of sodium (FENa) is useful to differentiate hypovolemia from other causes FENa <1% suggests hypovolemia, and FENa >1% is consistent with ATN:
Renal US, CT, IVP, or renal angiography may be used to detect hydronephrosis, obstruction, renal masses, abnormal blood flow, or vasculitis Renal biopsy is frequently indicated to diagnose intrarenal pathology Treatment: Intravascular volume should be carefully regulated with IV hydration and furosemide Use of any potentially nephrotoxic substances should be stopped Dietary salt and protein restriction helps reduce the risks of volume and protein overload Dialysis is indicated for severe disease with hyperkalemia, metabolic acidosis, or uremia Outcomes: Uremia and multiorgan dysfunction are possible complications Negative prognostic factors include advanced age, oliguria, need for transfusions, recent surgery, hypotension, need for vasoactive medications, and multiorgan dysfunction Mortality varies with the cause of the disease and may be as high as 70% in severe cases Clues to the diagnosis: History: fatigue, anorexia, oliguria
Physical: dehydration, friction rub, flank pain Tests: increased BUN and Cr, FENa <1%
ATN via drug toxicity and hypovolemia are the most common causes of acute renal failure.
The BUN/creatinine ratio is a quick way to help determine the etiology of ARF (i.e., ratio >20 if a prerenal cause). Chronic kidney disease (CKD) Progressive damage of the renal parenchyma taking place over multiple years Greater than 90% of the renal parenchyma is sclerotic or necrotic at the time of diagnosis Chronic HTN and DM are the most common causes History: patients are initially asymptomatic and gradually develop fatigue, nausea, anorexia, vomiting or diarrhea, paresthesias, weakness, and hematochezia or melena Physical examination: changes in mental status, hypertension, brownish discoloration of the skin Tests: Increased BUN, creatinine, potassium, and phosphate; decreased sodium and calcium Metabolic acidosis is common Anemia occurs due to impaired erythropoietin production Urine osmolality will be similar to serum osmolality Renal US will show hydronephrosis or small, sclerotic kidneys P.120 Treatment: Dietary salt and protein restriction helps to prevent additional taxing of the residual kidney function Any underlying conditions should be treated to prevent further secondary degeneration Dialysis or renal transplantation are required for survival as kidney function deteriorates (i.e., end-stage renal disease) Outcomes: complications include uremia with encephalopathy, dangerous electrolyte imbalances, renal osteodystrophy, and severe anemia; survival is longer with renal transplantation than chronic dialysis Why eliminated from differential: the length of symptoms, pattern of electrolyte abnormalities, and findings on the renal US are more indicative of acute renal injury Interstitial nephropathy/acute tubular necrosis Damage of renal tubules or parenchyma due to an acute toxic insult Drugs are the most common cause, but industrial toxins (e.g., cadmium, lead, copper, mercury), infection, or autoimmune conditions (e.g., sarcoidosis, amyloidosis) may also be etiologies The most common culprit drugs are β-lactam antibiotics, sulfonamides, aminoglycosides, nonsteroidal anti-inflammatory drugs (NSAIDs), allopurinol, proton pump inhibitors (PPIs), and diuretics History: fatigue, malaise, nausea, vomiting Physical examination: fever, rash, pericardial friction rub Tests: Increased BUN and creatinine; eosinophilia UA may show granular casts Toxicology screens may be useful to detect a causative drug Renal biopsy will demonstrate numerous inflammatory cells and renal tubular necrosis Treatment:
Stop the offending agent Supportive care with hydration and furosemide Corticosteroids may be beneficial to recovery in refractory cases Outcomes: complications include acute and chronic renal failure, renal papillary necrosis, uremia, electrolyte imbalances, and anemia; the mortality for full ATN with histologic evidence of renal necrosis is 50% Why eliminated from differential: the UA results and FENa rule out this diagnosis in favor of ARF Diabetic ketoacidosis (DKA) More thorough discussion in Chapter 5 Why eliminated from differential: the normal serum and urine glucose levels and the absence of urinary ketones rule out this diagnosis Pyelonephritis More thorough discussion in prior case Why eliminated from differential: the absence of urinary leukocytes and the normal WBC count make this diagnosis unlikely Congestive heart failure More thorough discussion in Chapter 1 Why eliminated from differential: the benign lung examination and the normal brain natriuretic peptide (BNP) make this diagnosis unlikely
Case 4-4: “My urine is brown” A 12-year-old boy is brought to his pediatrician by his mother because he has had brown urine for the past 3 days. The boy says that he has had no pain or urgency during urination and that he first noticed the unusual color incidentally 3 days ago. He reports feeling tired and achy for the past 5 days. His mother says that he had a bad cold 3 weeks ago that P.121 resolved and that they initially attributed these constitutional symptoms to a recurrence of the illness. The boy denies brown urine during his recent illness. He denies flank pain, recent trauma, dyspnea, nausea, vomiting or diarrhea, and hematemesis or hematochezia. His mother says that he is a healthy boy with no previous medical problems and takes a children's vitamin on a daily basis. The patient denies using any illicit substances. He plays organized soccer but has had no sudden changes in his activity level or recent trauma. On examination, he is a healthy appearing boy in no distress. He has some mild puffy swelling of his facial characteristics. No lymphadenopathy is palpable. Tests of his hearing are normal. Auscultation of his heart and lungs finds no abnormalities. Abdominal examination detects no tenderness or masses. A genital examination detects no penile abnormalities and normally descended testes. Neurologic examination is normal. The following vital signs are measured: T: 98.7°F, HR: 72 bpm, BP: 140/85 mm Hg, RR: 16 breaths/min
Differential Diagnosis Acute renal failure, polycystic kidney disease, poststreptococcal glomerulonephritis, immunoglobulin A (IgA) nephropathy, Goodpasture syndrome, Alport syndrome, nephrotic syndrome, cold-antibody hemolytic anemia, rhabdomyolysis
Laboratory Data and Other Study Results CBC: WBC: 6.5, Hgb: 15.5, Plt: 345 Chem7: Na: 141 mEq/L, K: 3.9 mEq/L, Cl: 105 mEq/L, CO2: 27 mEq/L, BUN: 32 mg/dL, Cr: 1.9 mg/dL, Glu: 79 mg/dL UA: brown colored, pH: 6.3, specific gravity: 1.020, multiple red cell casts, 3+ hematuria, 1+ proteinuria, no glucose/ketones/nitrites/leukocyte esterase Twenty-four-hour urine protein: 1,025 mg/d Antistreptolysin O titer: positive Antibasement membrane IgG antibodies: negative Renal US: normal-sized kidneys; no hydronephrosis or obstruction
Diagnosis Poststreptococcal glomerulonephritis
Treatment Administered The patient was put on a temporary salt-restricted diet with no excessive water consumption
The patient and his mother were reassured that the disease would likely be self-limited
Follow-up The patient had symptomatic improvement over the following few days His hypertension was normalized 1 week later on a follow-up visit Proteinuria and hematuria resolved over the following 2 months
The list of nephritic glomerular diseases may be remembered by the mnemonic PIG WAIL: Postinfectious glomerulonephritis, IgA nephropathy, Goodpasture syndrome, Wegener granulomatosis, Alport syndrome, Idiopathic crescentic glomerulonephritis, Lupus nephritis.
Steps to the Diagnosis All nephritic syndromes involve a process of glomerular inflammation. Poststreptococcal glomerulonephritis Acute glomerulonephritis that occurs as a sequela of recent streptococcal infection (Table 4-6) Most common in children between 1 and 12 years of age History: recent streptococcal infection (1 to 2 weeks prior to symptoms), hematuria, malaise, possible oliguria Physical examination: periorbital edema, hypertension P.122
Table 4-6 Nephritic Syndromes
Ty pe
Pathology
H/P
Poststreptococcal
Sequela of streptoc oc c al
Recent infection (1–2
glomerulonephritis
infection
w eeks prior to onset of symptoms), oliguria, edema, brown urine, hypertension; more common in c hildren (1 to 12 years old)
Tests
Treatment
UA shows hematuria and
Self-limited
proteinuria
Supportive treatment
High antistreptolysin O
(decrease edema and
titer
hypertension)
Bumpy deposits of IgG and
Antihypertensives for
C3 on renal basement
refractory HTN
Prognosis
Excellent in children; 25% mortality in adults if CHF develops
membrane on electron microscopy
IgA nephropathy (i.e.,
Unclear relation to
Intermittent hematuria,
Berger disease)
infection or autoimmune
flank pain, low-grade
process; deposition of IgA
fever; rare HTN and
antibodies in mesangial
edema; symptoms
cells
c oinc ide with the onset of infection
Goodpasture syndrome
Rare progressive
Dy spnea, chest pain,
autoimmune disease of the
hemopty sis, recurrent
lungs and kidneys
respiratory infections,
characterized by
arthralgias, hematuria,
deposition of
chills, nausea, vomiting,
antiglomerular and
fever, tachypnea, rales
anti-alveolar basement membrane antibodies
Inc reased serum IgA
Occasionally self- Limited
Mesangial cell proliferation
ACE-Is and statins are
on electron microscopy
given for persistent
Histology is
proteinuria
indistinguishable from that
Give corticosteroids if
for Henoch-Schonlein
nephrotic syndrome
purpura renal disease
Develops
Anemia
Corticosteroids or
Pulmonary infiltrates on
immunosuppressive
CXR
agents may ease
Restrictive disease on PFTs
symptoms
Serum IgG antiglomerular
Plasmapheresis may be
basement membrane
required to remove
antibodies
autoantibodies
Disease course is highly variable, but the Prognosis is generally Good (especially in children)
Prognosis generally poor without aggressive treatment
Linear pattern of IgG antibody deposition on fluorescence microscopy of glomeruli
Alport syndrome
Hereditary defect in
Hematuria, symptoms of
collagen IV in basement
renal failure,
membrane
highfrequenc y hearing loss
UA shows red cell casts,
ACE-Is may reduce
hematuria, proteinuria, and
proteinuria
pyuria
Renal transplant may be
Renal biopsy shows
required in progressive
glomerular basement
disease
90% of cases will end in renal failure, although some cases may have a benign course
membrane inc onsistenc y on electron microscopy
Idiopathic crescentic
Rapidly progressive renal
Sudden renal failure,
glomerulonephritis
failure due to idiopathic
weakness, nausea,
Positive ANCA
Corticosteroids,
Renal biopsy shows
plasmapheresis, and
Poor prognosis with rapid progression to
causes, other glomerular
vomiting, weight loss,
inflammatory cell deposition
immunosuppressive
diseases, or systemic
dyspnea, hemoptysis,
in Bowman capsule and
agents may be helpful to
infection
myalgias, fever, oliguria
c resc ent formation (i.e.,
slow progression
basement membrane
Renal transplant is
wrinkling) on electron
frequently required
renal failure
microscopy
Lupus nephritis
Sequela of systemic lupus
Possibly asymptomatic,
(mesangial,
erythematosus involving
possible HTN, renal failure,
membranous, focal
proliferation of endothelial
or nephrotic syndrome
proliferative, and
and mesangial cells
diffuse proliferative
Positive ANA and anti-DNA
Corticosteroids or
antibodies
immunosuppressive
Hematuria and possible
agents can delay renal
proteinuria on UA
failure ACE-I and statins help
types)
Prognosis and risk of renal failure correlates with severity of the systemic disease
reduce proteinuria
Wegener granulomatosis
Systemic vasculitis
Dyspnea, chronic sinusitis,
(also see Chapter 2)
characterized by
hemoptysis, myalgias,
granulomatous
fever, hematuria, sensory
inflammation and necrosis
neuropathy, arrhythmias,
of pulmonary and renal
visual deficits
vessels
Positive c -A NCA
Corticosteroids and
Deposition of immune
cyclophosphamide may
complexes in renal vessels
slow disease progression
seen on electron microscopy Pulmonary and renal
Rapidly fatal if untreated; gradual multiorgan deterioration is typical with a 30% 5-year survival
biopsies may detects areas of noncaseating necrosis
ACE-Is, angiotensin converting enzyme inhibitors; ANA, antinuclear antibody; ANCA, antineutrophil cytoplasmic antibody; c-ANCA, cytoplasmic antineutrophil cytoplasmic antibody; CHF, congestive heart failure; CXR, chest x-ray; HTN, hypertension; H/P, history and physical; Ig, immunoglobulin; IV, intravenous; PFTs, pulmonary function tests; UA, urinalysis.
P.123 Tests: UA will show hematuria, proteinuria, and red cell casts Twenty-four-hour urine protein is <3 g High antistreptolysin O titer Renal biopsy will show bumpy deposits of IgG and C3 complement on the renal basement membrane when examined under electron microscopy (Figure 4-3) Treatment: The condition is self-limited and generally requires only supportive care and a low salt diet with water restriction Diuretics, calcium channel blockers, and ACE-Is may be used to treat significant HTN Dialysis is very rarely required Outcomes: Children have an excellent prognosis Adults with the diagnosis may develop CHF and have mortality rates up to 25% Complications include CHF, CKD, and nephrotic syndrome and are more common in adults Clues to the diagnosis: History: hematuria, malaise, recent upper respiratory infection Physical: periorbital edema Tests: UA with hematuria, proteinuria, and red cell casts, positive antistreptolysin O titer
Both nephritic and nephrotic syndromes involve diseases of the glomeruli; they are differentiated by the absence (nephritic) or presence (nephrotic) of proteinuria >3 g per day. Immunoglobulin A nephropathy More thorough discussion in Table 4-6 Why eliminated from differential: symptoms of this diagnosis typically occur at the same time as the related infection, so this diagnosis is less likely than poststreptococcal glomerulonephritis Goodpasture syndrome
More thorough discussion in Chapter 2 and in Table 4-6 (Color Figure 4-2) Why eliminated from differential: the absence of dyspnea in the history and the absence of antibasement membrane antibodies rule out this diagnosis Alport syndrome More thorough discussion in Table 4-6 Why eliminated from differential: the absence of hearing loss makes this diagnosis less likely than the correct diagnosis Nephrotic syndrome More thorough discussion in later case Why eliminated from differential: the 24-hour urine protein level is indicative of a nephritic and not nephrotic condition Acute renal failure More thorough discussion in prior case Why eliminated from differential: some degree of mild ARF is apparent from the mildly increased BUN and creatinine, but the diagnosis of poststreptococcal glomerulonephritis is a better description of the actual etiology of the condition Polycystic kidney disease More thorough discussion in prior case Why eliminated from differential: the normal appearance of the kidneys on the renal US rules out this disease Cold-antibody hemolytic anemia More thorough discussion in Chapter 6 Why eliminated from differential: the normal hemoglobin in the patient makes a hemolytic anemia unlikely Rhabdomyolysis A syndrome following skeletal muscle injury in which the intracellular muscular components (e.g., myoglobin) enter the circulation and clog the renal filtration system History: history of a precipitating factor (e.g., trauma, DKA, heat stroke, heavy drug use), weakness, myalgias, dark urine P.124
Figure 4-3 Electron microscopy of a renal biopsy from a patient with poststreptococcal glomerulonephritis. Bumpy deposits of immunoglobulin G and complement (arrows) are present on the basement membrane, and the capillary lumens (Ls) are compressed. (See color image.)
P.125 Physical examination: muscle tenderness, edema Tests: serum creatine kinase will be significantly increased; UA will indicate the presence of hematuria, but no red cells will be seen Treatment: Use of a causative agent should be stopped IV hydration helps to maintain glomerular filtration; simultaneous use of diuretics may also help to increase glomerular blood flow Abnormal electrolyte levels should be corrected Dialysis may be required in patients with severe renal failure Strenuous activities and substance use should be avoided to prevent further muscle breakdown Outcomes: one third of cases will be complicated by renal failure; the outcome correlates with the rapidity of starting therapy
Why eliminated from differential: the absence of a history of trauma and the presence of red cell casts on the UA make this diagnosis unlikely
Case 4-5: “I'm having problems breathing, and my legs are swollen” A 63-year-old man presents to his PCP with the complaint of progressive edema and dyspnea that have worsened over the past month. He was in his usual state of health until the onset of these symptoms and has never experienced them previously. He states that over this time he has also experienced fatigue, weakness, and a lack of appetite. He denies headache, dizziness, chest pain, abdominal pain, nausea, vomiting or diarrhea, hematemesis or hematochezia, urinary symptoms, hematuria, or paresthesias. He has a history of rheumatoid arthritis that has been controlled with celecoxib and methotrexate. He denies other medical conditions and medication use. He drinks alcohol infrequently and denies other substance use. On examination, he is a tired-appearing man who is mildly dyspneic. He has 4+ pitting edema in his legs and milder edema in his upper body. He has no palpable lymphadenopathy. Auscultation of his lungs detects mild coarse breath sounds but no areas of focal consolidation. Auscultation of his heart and bowels is normal. He has no abdominal masses. His proximal interphalangeal, metacarpophalangeal, and knee joints are mildly swollen and hypertrophic. The following vital signs are measured: T: 98.8°F, HR: 87 bpm, BP: 128/81 mm Hg, RR: 22 breaths/min
Differential Diagnosis Rheumatoid arthritis, CHF, cirrhosis, nephrotic syndrome, pneumonia, acute renal failure, sarcoidosis
Laboratory Data and Other Study Results CBC: WBC: 10.1, Hgb: 13.7, Plt: 240 Chem10: Na: 137 mEq/L, K: 5.3 mEq/L, Cl: 106 mEq/L, CO2: 29 mEq/L, BUN: 43 mg/dL, Cr: 2.1 mg/dL, Glu: 79 mg/dL, Mg: 2.3 mg/dL, Ca: 9.7 mg/dL, Phos: 4.2 mg/dL BNP: 89 pg/mL Liver function tests (LFTs): alkaline phosphatase (AlkPhos): 56 U/L, alanine aminotransferase (ALT): 35 U/L, aspartate aminotransferase (AST) 21 U/L, total bilirubin (TBili): 1.0 mg/dL, direct bilirubin (DBili): 0.4 mg/dL UA: cloudy, pH: 5.5, specific gravity: 1.031, multiple fatty casts, 3+ proteinuria, no glucose/ketones/nitrites/leukocyte esterase/hematuria CXR: mild diffuse infiltrates; no focal consolidations; small pleural effusions at the costodiaphragmatic gutters Based on these results, the following studies are ordered: Albumin: 2.3 g/dL Twenty-four-hour urine protein: 5,036 mg/day P.126 Renal US: normal-sized kidneys; no apparent hydronephrosis or obstruction Renal biopsy (US-guided): histology shows generalized thickening of glomerular capillaries with spiky endothelial deposits
Diagnosis Membranous glomerulonephritis (nephrotic syndrome)
Treatment Administered The patient was admitted to the hospital and placed on a low salt diet Enalapril, furosemide, and prednisone were initiated
Follow-up The patient had a gradual improvement in his edema following the initiation of therapy During his inpatient stay, the patient developed a deep vein thrombosis (DVT) of the left femoral vein and was anticoagulated with heparin and transitioned to warfarin The patient was discharged to home when his edema had decreased enough to significantly improve his symptoms Due to lingering edema and proteinuria, the patient was eventually started on cyclophosphamide as an outpatient Over the course of the subsequent year, the patient experienced progressive renal failure that required the eventual initiation of dialysis
The list of nephrotic glomerular diseases may be remembered by the mnemonic “Most Dogs Find Meat Mesmerizing”: Minimal change disease, Diabetic nephropathy, Focal segmental glomerular sclerosis, Membranous glomerulonephritis, Membranoproliferative glomerulonephritis.
Steps to the Diagnosis The variants of nephrotic syndrome are frequently difficult to differentiate, and a renal biopsy is often indicated to provide a definitive diagnosis. Membranous glomerulonephritis More thorough discussion in Table 4-7 (Color Figure 4-3) Clues to the diagnosis: History: anorexia, fatigue, weakness, progressive edema and dyspnea Physical: anasarca Tests: significant proteinuria, hypoalbuminemia, renal biopsy findings
Minimal change disease is more common in children, while focal segmental glomerular sclerosis is more common in adults. Minimal change disease More thorough discussion in Table 4-7 Why eliminated from differential: the edema in the case is more severe than that expected for this diagnosis, and the renal biopsy appearance is more consistent with membranous glomerulonephritis Focal segmental glomerular sclerosis More thorough discussion in Table 4-7 Why eliminated from differential: the renal biopsy is more consistent with membranous glomerulonephritis Membranoproliferative glomerulonephritis More thorough discussion in Table 4-7 (Color Figure 4-4) Why eliminated from differential: the renal biopsy is more consistent with membranous glomerulonephritis Rheumatoid arthritis More thorough discussion in Chapter 9 Why eliminated from differential: although the patient does have an existing diagnosis of rheumatoid arthritis, severe edema is not a characteristic of this disease, and an alternative explanation must be provided Congestive heart failure More thorough discussion in Chapter 1 Why eliminated from differential: the normal range BNP makes this diagnosis less likely, and the edema seen in severe CHF is not associated with significant proteinuria P.127
Table 4-7 Nephrotic Syndromes
Ty pe
Minimal change disease
Pathology
H/P
Autoimmune injury of
Edema, malaise,
foot processes on
weight gain, possible
basement membrane
HTN or increased
leading to protein
frequency of
leaking
infections
Tests
Treatment
Hyperlipidemia
Corticosteroids
Hypoalbuminemia
Cyclophosphamide may
Proteinuria on UA
be used in refractory
Renal biopsy shows flattening of
cases
basement membrane foot processes
Segmental sclerosis of
Edema, HTN, possible
glomerular sclerosis
the glomeruli that is
rapid renal
either idiopathic or
deterioration
associated with drug use or HIV
Relapse is possible, but most patients have a very good prognosis with eventual cessation of relapses and a low rate of renal failure
on electron microscopy
Focal segmental
Prognosis
Hyperlipidemia
Salt restriction and
Hypoalbuminemia
diuretics help improve
Hematuria and high proteinuria on UA
edema
Renal US may show shrunken kidneys
ACE-Is decrease
Renal biopsy shows solidification of
proteinuria and HTN
Disease course is variable, but most patients require dialysis within 10 years
some glomeruli due to sclerosis on
Corticosteroids
electron microscopy
with/without cyclophosphamide are used to induce disease remission
Membranous
Deposition of immune
Edema, dyspnea;
glomerulonephritis
complexes in the
history of infection or
glomeruli basement
medication use may
membrane
lead to diagnosis
Hyperlipidemia
Low salt diet and
Hypoalbuminemia; proteinuria on UA
diuretics to improve
“Spike and dome” basement
edema
membrane thic kening on electron
ACE-Is to improve
microscopy
proteinuria and HTN Corticosteroids
Risk of end-stage renal disease increases with the degree of proteinuria and renal failure at the time of diagnosis; increased risk of DVT
with/without cyclophosphamide are used to induce disease remission Anticoagulants if DVT occurs
Membranoproliferative
Chronic proliferation of
Edema, HTN, history
glomerulonephritis
mesangial cells with
of systemic infection
interposition into the
or autoimmune
capillary walls due to an
condition, hematuria,
idiopathic, infectious,
oliguria, fatigue,
or autoimmune process
retinal lesions
Hyperlipidemia
Treat underlying
Hypoalbuminemia; possible
infectious or rheumatic
hypocomplementemia
conditions
Proteinuria and hematuria on UA
Corticosteroids may be
Renal biopsy shows IgG deposits on
beneficial in children
the basement membrane on
ASA or dipyridamole is
fluorescence microscopy and
useful in adults
Gradual progression to end-stage renal disease occurs over >10 years
basement membrane thic kening with a double-lay er “train trac k” appearance on electron microscopy
Diabetic nephropathy
Basement membrane
History of DM, HTN,
(e.g., diffuse, nodular)
and mesangial
edema, foamy urine
thickening related to diabetic vascular changes
Hyperlipidemia
Treat underlying DM
Hypoalbuminemia; proteinuria on UA
Dietary protein
Basement membrane thickening on
restriction
electron microscopy seen in both
ACE-I for proteinuria
types
and HTN
Round nodules (Kimmelstiel-W ilson
End-stage renal disease occurs in 50% of patients with DM type I and in 10% of patient with DM type II within 10 years of the diagnosis of nephropathy
nodules) are seen within glomeruli in the nodular type
ACE-Is, angiotensin converting enzyme inhibitors; ASA, aspirin; DM, diabetes mellitus; DVT, deep vain thrombosis; HIV, human immunodeficiency virus; HTN, hypertension; H/P, history and physical; Ig, immunoglobulin; UA, urinalysis; US, ultrasound.
P.128 Cirrhosis More thorough discussion in Chapter 3 Why eliminated from differential: the normal LFTs rule out a primary hepatic pathology Pneumonia More thorough discussion in Chapter 2 Why eliminated from differential: the normal WBC count, lack of a fever, and unimpressive chest x-ray (CXR) rule out this diagnosis Acute renal failure More thorough discussion in prior case Why eliminated from differential: although some of the clinical signs and lab abnormalities are consistent with renal failure (e.g., edema, increased BUN and creatinine), the significant proteinuria requires a more definitive explanation for the patient's condition Sarcoidosis More thorough discussion in Chapter 2 Why eliminated from differential: the absence of hilar adenopathy on the CXR and the normal calcium, alkaline phosphatase, and WBC lab values make this diagnosis less likely
Case 4-6: “I am always so thirsty” A 36-year-old woman presents to a trauma surgeon for a follow-up appointment 2 months after a motor vehicle accident. She was a restrained passenger in a car without airbags that was hit on the side by another car. Her trauma work-up was significant for a minor splenic laceration and small subarachnoid hemorrhage, both of which were able to be treated nonoperatively. She was able to be discharged to home 10 days after her admission to the hospital and denies any current neurologic deficits. She reports some gradually resolving left abdominal and flank pain. Her main complaint, however, is significant thirst that has been present since she returned home. She reports urinating frequently, but feeling the need to drink constantly
because of a feeling of being “dry.” She denies headaches, nausea, vomiting or diarrhea, dysuria, paresthesias, or weakness. Prior to her trauma, she was healthy with no medical problems. She takes no medications. She drinks alcohol socially. On examination, she is a healthy appearing woman in no distress. She has a large bottle of water with her in the office. Her mucus membranes are mildly dry, and she has slightly decreased skin turgor. She has no palpable lymphadenopathy. Auscultation of her heart and lungs is normal. She has minimal abdominal tenderness in her left upper quadrant. She has no hepatosplenomegaly, but a distended bladder is palpable in the suprapubic region. Neurologic examination is normal. The following vital signs are measured: T: 98.5°F, HR: 88 bpm, BP: 110/78 mm Hg, RR: 24 breaths/min
Differential Diagnosis Diabetes mellitus, chronic kidney disease, diabetes insipidus, psychogenic polydipsia
Laboratory Data and Other Study Results Chem7: Na: 157 mEq/L, K: 3.6 mEq/L, Cl: 108 mEq/L, CO2: 25 mEq/L, BUN: 14 mg/dL, Cr: 0.5 mg/dL, Glu: 83 mg/dL UA: light straw colored, pH: 6.0, specific gravity: 1.003, no glucose/ketones/nitrites/leukocyte esterase/hematuria/proteinuria Urine electrolytes: Na: 4 mEq/L, K: 2 mEq/L Urine osmolality: 68 mOsm/kg Plasma osmolality: 298 mOsm/kg Brain MRI: continued resolution of small subarachnoid hemorrhage in the pituitary region when compared to imaging performed 2 months prior P.129 After receipt of these results, the following test is ordered: Water deprivation test: urine osmolality increased from 70 mOsm/kg to 125 mOsm/kg following administration of vasopressin
Diagnosis Central diabetes insipidus secondary to cerebral trauma
Treatment Administered Intranasal desmopressin was prescribed on an outpatient basis The patient was encouraged to drink enough water to avoid dehydration
Follow-up Following the initiation of therapy, the patient's urine and serum osmolalities normalized, and she was able to reduce her water intake
Steps to the Diagnosis Diabetes insipidus (DI) A disorder of antidiuretic hormone (ADH)-directed water reabsorption leading to dehydration and hypernatremia Types: Central: failure of the posterior pituitary to secrete ADH due to idiopathic causes, cerebral trauma, pituitary tumors, hypoxic encephalopathy, or anorexia nervosa Nephrogenic: failure of the kidneys to respond to ADH due to congenital kidney disease, lithium toxicity, hypercalcemia, or hypokalemia Hypernatremia Serum sodium >155 mEq/L Causes include dehydration, cutaneous water loss (e.g., burns, wounds), gastrointestinal (GI) fluid losses (e.g., vomiting, diarrhea), diabetes insipidus, and excess aldosterone secretion (Figure 4-4) Symptoms and signs include oliguria, polydipsia, weakness, mental status changes, and seizures
Figure 4-4 Algorithm for the evaluation of hypernatremia. Na, sodium. (See color image.)
P.130 Fluid correction Gradual hydration with hypotonic saline for an inadequate fluid intake or excess fluid loss at a maximum rate of 12 mEq Na per day Calculation of the water deficit is used to determine the approximate required correction in a patient with fluid loss as a cause for the hypernatremia:
Half of the deficit is given in 24 hours in addition to maintenance fluids, and the remainder is given over the following 24 to 48 hours Sodium levels are closely monitored to avoid overcorrection The calculated water deficit may be artificially high because the normal total body water content will be slightly greater than the patient's body water content in the presence of hypernatremia History: polydipsia, polyuria Physical examination: signs of dehydration may be present, or the examination may be normal Tests: Hypernatremia Low urine osmolality in the presence of a large urine volume Water deprivation test The patient is deprived of water for 2 to 3 hours followed by administration of vasopressin Normal patients will show normal or mildly increased urine osmolality before vasopressin administration with little change afterward Patients with central DI will show low urine osmolality before vasopressin administration and normal osmolality afterward Patients with nephrogenic DI will have low urine osmolality before and after vasopressin administration Treatment: Any underlying causes are treated Daily intranasal desmopressin is given in central DI Salt restriction and high water consumption are required in patients with nephrogenic DI; thiazide diuretics may reduce net fluid loses by inducing distal convoluted tubule water secretion to cause a reflexive increase in proximal tubule water reabsorption
Outcomes: complications include dehydration, seizures, and iatrogenic cerebral edema from excessive rehydration; treatable cases have an excellent prognosis Clues to the diagnosis: History: polydipsia, polyuria, recent head trauma Physical: signs of dehydration Tests: hypernatremia, low urine osmolality, water deprivation test results
Causes of hypernatremia may be remembered by the six Ds: Diuretics, Dehydration, Diabetes insipidus, Docs (iatrogenic), Diarrhea (and vomiting), Disease of kidney (hyperaldosteronism). Diabetes mellitus More thorough discussion in Chapter 5 Why eliminated from differential: the normal serum glucose level and absence of glucosuria or urinary ketones on the UA rule out this diagnosis Chronic kidney disease More thorough discussion in prior case Why eliminated from differential: the normal BUN and creatinine levels rule out this diagnosis Psychogenic polydipsia Excess water consumption related to a psychiatric cause and possibly associated with dysfunction of the cerebral thirst regulation system History: polydipsia, polyuria, confusion Physical examination: edema, mental status changes Tests: hyponatremia P.131 Treatment: treat underlying psychiatric conditions; water restriction Outcomes: success in correcting the electrolyte abnormality is tied to the success in treating the underlying disease Why eliminated from differential: the presence of hypernatremia rules out this diagnosis
Case 4-7: “My father is lethargic and confused” A 72-year-old man is seen by a consultant nephrologist in the emergency department in evaluation of newly onset hyponatremia. The patient's son is present in the examination room. The patient is lethargic and is unable to communicate well, so his son answers most of the physician's questions. The patient's son says that his father has become increasingly lethargic, weak, and confused over the past few weeks. He was diagnosed with small cell lung cancer 1 month ago and has been undergoing chemotherapy for the past 3 weeks. The son thought that his father's symptoms were related to his therapy, so they were not initially overly concerned. He found his father in bed earlier today more sedated than usual and decided to bring him to the hospital. He says that his father had a cough, dyspnea, and weight loss in the months leading up to his diagnosis, but that he has never had episodes of confusion before the past 4 weeks. He is unaware of any changes in bowel habits, complaints of numbness, or chest or abdomen pain. Besides lung cancer, the patient has a history of hypercholesterolemia and gout for which he takes atorvastatin and allopurinol. He is on a chemotherapy regimen of cyclophosphamide, doxorubicin, and vincristine. He was a heavy smoker for many years but no longer smokes or drinks alcohol. On examination, he is a thin man who is lethargic and speaks little. He has no palpable lymphadenopathy. Auscultation of his heart and lungs detects mildly decreased breath sounds on the left side and normal heart sounds. He has no abdominal tenderness or masses. He appears to have normal sensation but is very slow to follow motor commands. The following vital signs are measured: T: 98.4°F, HR: 90 bpm, BP: 135/82 mm Hg, RR: 20 breaths/min
Differential Diagnosis Syndrome of inappropriate ADH secretion, CHF, cerebral vascular accident, ATN, ARF, Addison disease
Laboratory Data and Other Study Results The following results are from tests ordered by the emergency medicine staff: CBC: WBC: 4.6, Hgb: 12.5, Plt: 189 Chem10: Na: 125 mEq/L, K: 4.1 mEq/L, Cl: 106 mEq/L, CO2: 30 mEq/L, BUN: 10 mg/dL, Cr: 0.6 mg/dL, Glu: 77 mg/dL, Mg: 1.9 mg/dL, Ca: 10.5 mg/dL, Phos: 4.2 mg/dL UA: amber colored, pH: 6.7, specific gravity: 1.040, no glucose/ketones/nitrites/leukocyte esterase/hematuria/proteinuria
BNP: 78 pg/mL D-dimer: 167 ng/mL LFTs: AlkPhos: 87 U/L, ALT: 45 U/L, AST: 35 U/L, TBili: 1.1 mg/dL, DBili: 0.3 mg/dL CXR: nodular density in left lower lung field with associated infiltrates; no areas of consolidation; normal right lung fields Head CT: no focal lesions or hemorrhage; normal cortical and ventricle sizes The following tests are ordered by the nephrologist: Serum osmolality: 265 mOsm/kg Urine osmolality: 128 mOsm/kg Urine electrolytes: Na: 35 mEq/L, K: 11 mEq/L, Cr: 9 mg/dL FENa: 1.9 Serum cortisol (afternoon): 12 µg/dL
Diagnosis Syndrome of inappropriate antidiuretic hormone secretion (SIADH), likely as a paraneoplastic syndrome
Treatment Administered The patient was admitted for regulation of his sodium levels Fluid restriction was initiated IV furosemide was administered
Follow-up The patient's serum sodium level was able to normalized over 2 days with a resultant improvement in the patient's mentation The patient was discharged to home on a regimen of fluid restriction and oral furosemide
Steps to the Diagnosis Syndrome of inappropriate antidiuretic hormone secretion Nonphysiologic secretion of ADH that results in hyponatremia Causes include cerebral insults, sarcoidosis, paraneoplastic syndromes, psychiatric drugs, major surgeries, pneumonia, and human immunodeficiency virus (HIV) Hyponatremia Serum sodium <135 mEq/L
To calculate the sodium (Na) that will result from the correction of hyperglycemia, add 1.6 mEq/L Na for every 100 mg/dL glucose above 100 mg/dL. Causes include CHF, SIADH, thiazide diuretics, hyperglycemia, or excess fluid consumption (Figure 4-5) Symptoms and signs include nausea, weakness, confusion, and mental status changes Fluid correction
Figure 2-5 Algorithm for the evaluation of hyponatremia. ACE-1, angiotensin-converting enzyme inhibitor; CHF, congestive heart failure; FENa, fractional excretion of sodium; Na, sodium; SIADH, syndrome of inappropriate antidiuretic hormone secretion.
Hypertonic and isotonic hyponatremia are treated with glucose and lipid regulation, respectively Hypotonic hypervolemic and euvolemic hyponatremia are corrected with fluid restriction and the possible use of furosemide Hypotonic hypovolemic hyponatremia is corrected with IV saline
Pseudohyponatremia is an artifact of hyperlipidemia in which serum sodium falsely appears to be low. Hypertonic saline may be infused to correct cases with sodium <120 mEq/L (correct no faster than 12 mEq per day) History: chronic nausea, weakness, progressive confusion Physical examination: neurologic deficits develop in severe cases Tests: Hyponatremia; decreased serum osmolality (i.e., hypotonicity) Increased urine sodium; urine osmolality will be higher than expected for the coincident hyponatremia (>100 mOsm/kg) Treatment: Fluid restriction Furosemide should be used for diuresis in symptomatic patients Demeclocycline may be used in patients with chronic disease who do not respond to diuresis and fluid restriction Hypertonic saline may be used to correct severe cases Outcomes: complications include progressive cerebral damage and central pontine myelinolysis from overly rapid correction; prognosis correlates with the ability to treat the underlying cause Clues to the diagnosis: History: lethargy, weakness, confusion Physical: noncontributory Tests: hyponatremia, low serum osmolality, relatively high urine osmolality, high urine sodium Congestive heart failure More thorough discussion in Chapter 1
Why eliminated from differential: the absence of diffuse pulmonary edema on the CXR and the normal BNP make this an unlikely cause Acute tubular necrosis More thorough discussion in prior case Why eliminated from differential: the normal BUN and creatinine rule out this diagnosis Acute renal failure More thorough discussion in prior case Why eliminated from differential: the normal BUN and creatinine rule out this diagnosis Cerebral vascular accident More thorough discussion in Chapter 7 Why eliminated from differential: the normal findings of the head CT make this diagnosis unlikely; stroke may cause mental status changes but cannot typically explain the hyponatremia Addison disease More thorough discussion in Chapter 5 Why eliminated from differential: the normal serum cortisol level helps to rule out this diagnosis
Case 4-8: “Her heart isn't beating right” A 68-year-old woman is brought to the emergency department by a friend because she is having palpitations. The patient says that these palpitations started about 2 hours ago and seem to be worsening. She says that she has never had this experience previously. She also notes feeling tired, weak, and nauseous for several days. She denies chest pain, dyspnea, abdominal pain, vomiting or diarrhea, and paresthesias. She admits that she skipped her dialysis appointments two times this past week because she “just didn't feel like going.” She has a history of systemic lupus erythematosus and chronic kidney disease secondary to lupus nephritis. Her medication list is ASA, enalapril, simvastatin, and prednisone. She has been on hemodialysis for 9 months. She smokes three cigarettes per day and drinks four beers per week. She denies illicit substance use. On examination, she is an anxiousappearing woman who is not in significant distress. She has a light malar rash and a few small ulcers inside her mouth. Auscultation of her lungs finds mildly decreased breath sounds in the bases and lower axillary regions. Auscultation of her heart detects multiple ectopic beats and no abnormal heart sounds. Auscultation of her abdomen detects normal bowel sounds. She has no abdominal tenderness or masses. Her knees and metacarpal joints are mildly nodular and have crepitus with range of motion. She has mildly decreased two-point discrimination in her lower extremities. Her coordination is normal, but her motor strength is mildly decreased to 4+/5. The following vital signs are measured: T: 99.7°F, HR: 75 bpm, BP: 132/85 mm Hg, RR: 18 breaths/min
Differential Diagnosis MI, systemic lupus erythematosus (SLE), paroxysmal supraventricular tachycardia (PSVT), ventricular tachycardia (Vtach), atrial flutter (Aflutter), atrial fibrillation (Afib), DM, hyperkalemia, hypokalemia, hypocalcemia
Laboratory Data and Other Study Results CBC: WBC: 8.5, Hgb: 11.9, Plt: 198 Chem10: Na: 136 mEq/L, K: 6.1 mEq/L, Cl: 108 mEq/L, CO2: 24 mEq/L, BUN: 87 mg/dL, Cr: 5.9 mg/dL, Glu: 98 mg/dL, Mg: 1.9 mg/dL, Ca: 9.6 mg/dL, Phos: 4.0 mg/dL UA: straw colored, pH: 5.7, specific gravity: 1.010, 2+ hematuria, 2+ proteinuria, no glucose/ketones/nitrites/leukocyte esterase Electrocardiogram (ECG): sinus rhythm; occasional premature ventricular complex (PVCs); normal P wave and QRS interval morphology; tall peaked T waves CXR: small bilateral pleural effusions in the costodiaphragmatic recesses; clear lung fields Cardiac enzymes: CK: 285 U/L, CK-MB: 4.1 ng/mL, troponin-I: 0.2 ng/mL
Diagnosis Hyperkalemia secondary to renal failure
Treatment Administered Polystyrene sulfonate was administered promptly to help decrease the serum potassium level The patient was admitted for cardiac monitoring
Dialysis was performed on an urgent basis to normalize the patient's electrolyte levels
Follow-up Following normalization of her potassium level, the patient's palpitations resolved, and a repeat ECG demonstrated resolution of the peaked T waves and PVCs The patient was able to be discharged to home with strong encouragement to adhere to her dialysis schedule
Steps to the Diagnosis Hyperkalemia Serum potassium >5.0 mEq/L Causes include metabolic acidosis, aldosterone deficiency, tissue breakdown, insulin deficiency, adrenal deficiency, renal failure, and potassium-sparing diuretics History: nausea, vomiting, weakness, palpitations, paresthesias Physical examination: arrhythmias, motor impairments Tests: increased serum potassium; ECG shows tall, peaked T waves Treatment: Polystyrene sulfonate is used to bind potassium and remove it via the GI tract Calcium gluconate, sodium bicarbonate, or glucose with insulin may be administered to encourage potassium uptake by cells Dialysis is required to decrease levels in severe cases Outcomes: sudden death or MI may result from cardiac arrhythmias Clues to the diagnosis: History: fatigue, weakness, nausea, missed dialysis appointments, history of renal failure Physical: paresthesias Tests: hyperkalemia, ECG appearance
Causes of hyperkalemia can be remembered by the mnemonic CRAMP KIT: Catabolism of tissues (e.g., trauma, chemotherapy, radiation), Renal failure, Aldosterone deficiency, Metabolic acidosis, Pseudohyperkalemia, K-sparing diuretics, Insulin deficiency, Tubular (renal) acidosis type 4.
Pseudohyperkalemia occurs from red blood cell hemolysis following blood collection, so potassium should be measured immediately in drawn blood and increased serum potassium should be confirmed with a repeat blood sample using a large-gauge needle. Hypokalemia Serum potassium <3.5 mEq/L Causes include poor dietary intake, metabolic or respiratory alkalosis, hypothermia, vomiting, diarrhea, hyperaldosteronism, insulin excess (e.g., treatment of diabetic ketoacidosis), potassium-wasting diuretics (e.g., loop, thiazide), or renal tubular acidosis types 1 and 2 (Figure 4-6, Table 4-8) History: fatigue, weakness, palpitations, muscle cramps Physical examination: arrhythmias, hyporeflexia, motor weakness Tests: decreased serum potassium; ECG will show T wave flattening, ST depression, and U waves Treatment: administer supplemental potassium (oral or IV depending on severity) Outcomes: complications include fatal cardiac arrhythmias, ileus, HTN, and insulin resistance; prognosis depends on the underlying condition
Figure 2-6 Algorithm for the evaluation of hypokalemia. GI, gastrointestinal; K, potassium.
Table 4-8 Types of Renal Tubular Acidosis
Distal (Ty pe 1)
Defect
Proximal (Ty pe 2)
Low Renin/Aldosterone (Ty pe 4)
HCO 3- reabsorption
Primary or secondary aldosterone deficiency
Idiopathic, autoimmune diseases, drugs, chronic
Idiopathic, multiple myeloma, Fanconi syndrome, Wilson disease,
Primary renin or aldosterone deficiency, DM,
infection, nephrocalcinosis, cirrhosis, SLE, obstructive
amyloidosis, vitamin D deficiency, autoimmune diseases,
Addison disease, sickle cell disease, interstitial
nephropathy
c arbonic anhy drase inhibitors
disease
Urine pH
>5.3
<5.3
<5.3
Serum
Low K
Low K, HCO 3-
High K, Cl
Radiology
Possible stones
Bone lesions
Treatment
Oral HCO 3-, K, thiazide diuretic
Oral HCO 3-, K; thiazide or loop diuretic
Impaired H+ secretion leading to secondary hyperaldosteronism
Cause
electrolytes
Fludrocortisone, K restriction, furosemide, polystyrene sulfonate
Cl, chloride; DM, diabetes mellitus, H, hydrogen; HCO 3,; K, potassium; SLE, systemic lupus erythematos.
Why eliminated from differential: arrhythmias are common with hypokalemia, but this diagnosis is ruled out by the lab results
Common causes of hypokalemia can be remembered by the mnemonic GRAPHIC IDEA: GI losses (e.g., vomiting, diarrhea), Renal tubular acidosis (types I and II), Aldosterone (high), Periodic paralysis, Hypothermia, Insulin excess, Cushing syndrome, Insufficient intake, Diuretics (e.g., loop, thiazide), Elevated β-adrenergic activity, Alkalosis (e.g., metabolic, respiratory). Hypocalcemia Serum calcium <8.5 mg/dL Caused by hypoparathyroidism, hyperphosphatemia, vitamin D deficiency, loop diuretics, pancreatitis, renal failure, or alcoholism History: abdominal pain, dyspnea, palpitations, muscle cramping, sensory tingling, dysphagia
Physical examination: tetany, Chvostek sign (i.e., tapping the facial nerve elicits a muscle spasm), Trousseau' sign (i.e., inflation of a blood pressure cuff causes carpal spasm), dry skin, papilledema, wheezing, S3 heart sound Tests: decreased serum calcium and ionized calcium; ECG will show QT interval prolongation Treatment: supplemental calcium (oral or IV depending on severity) and vitamin D Outcomes: prognosis is good with correction of the deficiency Why eliminated from differential: arrhythmias secondary to hyperkalemia or hypocalcemia are risks in patients with renal failure, but in this case the serum electrolyte labs and ECG appearance shows hyperkalemia to be the cause
Adjust the serum calcium in cases of comorbid hypoalbuminemia by decreasing the lower acceptable limit of the serum calcium by 0.8 mg/dL for every 1.0 g/dL albumin below 4.0 g/dL. Myocardial infarction More thorough discussion in Chapter 1 Why eliminated from differential: the normal cardiac enzymes and absence of ST interval depression or elevation rule out this diagnosis Systemic lupus erythematosus More thorough discussion in Chapter 9 Why eliminated from differential: SLE can explain many of the symptoms, signs, and lab values seen in this case, but hyperkalemia is a more specific cause of the patient's acute palpitations and is not characteristic of SLE Diabetes mellitus More thorough discussion in Chapter 5 Why eliminated from differential: the normal serum glucose level and absence of glucosuria on the UA rules out this diagnosis Atrial fibrillation, atrial flutter, paroxysmal supraventricular tachycardia, ventricular tachycardia More thorough discussion in Chapter 1 Why eliminated from differential: the appearance of the ECG rules out these other arrhythmias
Case 4-9: “I just ache down deep” A 49-year-old woman presents to an internist with the complaint of generalized aching. She feels a deep ache in her extremities and on the sides of her chest. She says that she has had the pain for the past few months, and that she is tired of waiting for it to resolve. She notes that she has presented to an emergency department two times in the past 2 months because of the pain. She was found to have two metatarsal fractures on the most recent visit and is currently wearing a hard-soled shoe for treatment. No explanation was provided for her generalized aching at either visit. Her job's medical benefits only started 2 weeks ago, so she was unable to find a PCP for the past few months. She is very animated in her descriptions of her experiences. In addition to the pain, she notes occasional nausea, constipation, and vomiting. She feels thirsty often and has urinary frequency. She denies dyspnea, headache, diarrhea, weakness, or paresthesias. She denies any medical history and uses multiple herbal supplements in her diet. She occasionally smokes marijuana and drinks alcohol socially. On examination, she is an agitated woman, but does not appear to be in any distress. She is notable for tongue fasciculations while she is not talking. She has no palpable lymphadenopathy. Auscultation of her heart, lungs, and bowels does not detect any abnormalities. Her abdomen is nontender with no palpable masses. She has normal motor strength and sensation but has hyperreflexia. She has a normal gait and no tremors besides the tongue fasciculations. The following vital signs are measured: T: 98.8°F, HR: 65 bpm, BP: 115/80 mm Hg, RR: 17 breaths/min
Differential Diagnosis Hypercalcemia, hyperthyroidism, diabetes mellitus, sarcoidosis, CKD, lung cancer, hypokalemia, cerebral vascular accident, Parkinson disease
Laboratory Data and Other Study Results CBC: WBC: 6.8, Hgb: 13.8, Plt: 418 Chem10: Na: 140 mEq/L, K: 4.1 mEq/L, Cl: 105 mEq/L, CO2: 23 mEq/L, BUN: 16 mg/dL, Cr: 0.8 mg/dL, Glu: 82 mg/dL, Mg: 2.0 mg/dL, Ca: 12.6 mg/dL, Phos: 1.8 mg/dL UA: straw colored, pH: 6.5, specific gravity: 1.010, 1+ hematuria, no glucose/ketones/nitrites/leukocyte esterase/proteinuria Thyroid panel: thyroid-stimulating hormone (TSH) 2.3 µU/mL, T4: 6.9 µg/dL, free T4 index: 7.1, T3: 1.1 ng/mL, T3: reuptake 0.96 Parathyroid hormone (PTH): 124 pg/mL Head CT: no focal lesions or hemorrhage; normal cerebral and ventricle volume
Chest CT: no discernable masses; lung fields clear; no effusions Abdominal CT: no masses; normal appearance of viscera and bowels; multiple nonobstructing renal calculi in both kidneys Following receipt of these results, the following tests are ordered: Ionized calcium: 2.3 mmol/L Twenty-four-hour urine calcium: 298 mg/day Sestamibi nuclear scan of neck: significantly increased uptake in one parathyroid gland; no abnormal thyroid uptake
Diagnosis Hyperparathyroidism and associated hypercalcemia secondary to parathyroid adenoma
Treatment Administered The patient was referred to otorhinolaryngology for further treatment of the parathyroid adenoma Parathyroid exploration was performed, and a parathyroidectomy was performed for one abnormal gland
Follow-up The patient was observed postoperatively to monitor her calcium levels The patient was briefly hypocalcemic following surgery, but regained a normal calcium level by 2 days after surgery The patient was discharged to home successfully The patient had continued normal serum calcium and PTH levels at the first follow-up appointment and gradual resolution of her symptoms
Steps to the Diagnosis Primary hyperparathyroidism Excess PTH secretion leading to hypercalcemia and osteopenia Most cases are due to a parathyroid adenoma; less frequently cases are due to multiple gland involvement, parathyroid carcinoma, or familial hypocalciuric hypercalcemia Hypercalcemia Serum calcium >10.5 mg/dL Causes include hyperparathyroidism, neoplasms, immobilization, thiazide diuretics, high ingestion of calcium carbonate and milk (i.e., milk-alkali syndrome; more often seen in children), sarcoidosis, or hypervitaminosis A or D Symptoms and signs include deep bone pain, susceptibility to fractures, nausea, vomiting, constipation, weakness, polydipsia, polyuria, and mental status changes Correction of abnormal calcium levels
Causes of hypercalcemia may be remembered by the mnemonic CHIMPANZEES: Calcium supplementation; Hyperparathyroidism; Immobility; Milk-alkali syndrome; Paget disease; Addison disease; Neoplasms; ZollingerEllison syndrome; Excess vitamin A; Excess vitamin D; Sarcoidosis. Calcitonin and bisphosphonates are used to decrease the serum calcium level IV hydration helps decrease the risk of nephrolithiasis Glucocorticoids help to decrease intestinal absorption of calcium History: occasionally asymptomatic, but symptoms of hypercalcemia are frequently present Physical examination: tongue fasciculations, hyperreflexia, mental status changes Tests: Increased serum calcium, ionized calcium, and PTH; decreased phosphate Increased urine calcium Sestamibi nuclear scan or US of the neck are useful to detect abnormally functioning glands
Decreased bone density is apparent on a dual-energy x-ray absorptiometry (DEXA) scan Treatment: IV hydration and bisphosphonates are used to medically lower the serum calcium level and lessen the risk of nephrolithiasis Surgical resection of a single adenoma is typically performed Hyperplasia of all four glands requires total parathyroidectomy with implantation of part of one gland at a distant site Outcomes: complications include nephrolithiasis, cerebral degeneration, fragility fractures, heart failure, and iatrogenic hypocalcemia; prognosis is very good following successful excision of an abnormal gland Clues to the diagnosis: History: bone pain, nausea, vomiting, fractures, polydipsia, polyuria Physical: tongue fasciculations, hyperreflexia, agitation Tests: hypercalcemia, hypercalciuria, hypophosphatemia, increased ionized calcium, increased PTH, nuclear scan findings, nephrolithiasis on CT
Differentiate between familial hypocalciuric hypercalcemia (genetic disorder of calcium-sensing receptors) and other causes of hypercalcemia by noting a family history of hypercalcemia, low urine calcium, and absence of osteopenia, nephrolithiasis, and mental status changes in the former.
Hypercalcemia is characterized by the rhyming mnemonic “bones” (fractures), “stones” (nephrolithiasis), “groans” (GI symptoms), and “psychiatric overtones” (changes in mental status).
Decreased calcium with increased PTH suggests a hyperparathyroidism secondary to malnutrition, malabsorption, renal disease, or calcium-wasting drugs. Hyperthyroidism More thorough discussion in Chapter 5 Why eliminated from differential: the normal thyroid panel rules out this diagnosis Diabetes mellitus More thorough discussion in Chapter 5 Why eliminated from differential: the normal serum glucose level and absence of urinary glucose rule out this diagnosis Sarcoidosis More thorough discussion in Chapter 2 Why eliminated from differential: no hilar adenopathy is seen on the chest CT, and a better explanation can be provided for the hypercalcemia (i.e., sarcoidosis is mainly a diagnosis of exclusion) Chronic kidney disease More thorough discussion in prior case Why eliminated from differential: the normal BUN and creatinine and absence of any electrolyte abnormalities besides calcium and phosphorus rule out this diagnosis Lung cancer More thorough discussion in Chapter 2 Why eliminated from differential: the absence of a lesion on the chest CT makes this diagnosis unlikely Hypokalemia More thorough discussion in prior case Why eliminated from differential: some similar clinical findings may occur in both hypercalcemia and hypokalemia, but the serum electrolyte labs rule out this diagnosis Cerebral vascular accident
More thorough discussion in Chapter 7 Why eliminated from differential: the normal head CT makes this diagnosis unlikely Parkinson disease More thorough discussion in Chapter 7 Why eliminated from differential: although this patient does have tongue fasciculations and hyperreflexia, the absence of any other characteristic movement abnormalities makes this diagnosis unlikely
Case 4-10: “I'm having problems going to the bathroom” A 59-year-old man presents to a urologist with difficulty urinating over the past 6 months. He says that he has problems initiating urination and frequently feels as if he has not emptied his bladder after going to the bathroom. Despite this experience he often gets urinary urgency and never wants to stray too far from a bathroom. He has to get out of bed at least twice per night to urinate. He says these symptoms started about 6 months ago and were initially mild but have gradually become worse. He denies dysuria, hematuria, abdominal pain, hematochezia, or purulent penile discharge. He has a history of HTN and hypercholesterolemia for which he takes losartan, ASA, and ezetimibe. He drinks less than eight alcoholic drinks per week and infrequently smokes cigars. On examination, he is a healthy appearing man in no acute distress. Auscultation of his abdomen detects normal bowel sounds. He has no abdominal tenderness or hepatosplenomegaly and has a palpable bladder. He has no palpable inguinal lymphadenopathy. A genital examination detects no testicular masses or tenderness. Rectal examination detects a nontender, large, and rubbery prostate. Neurologic examination is normal. The following vital signs are measured: T: 98.5°F, HR: 72 bpm, BP: 128/82 mm Hg, RR: 14 breaths/min
Differential Diagnosis Benign prostatic hypertrophy, prostate cancer, UTI, bladder cancer, prostatitis
Laboratory Data and Other Study Results Chem7: Na: 142 mEq/L, K: 4.0 mEq/L, Cl: 102 mEq/L, CO2: 25 mEq/L, BUN: 20 mg/dL, Cr: 1.1 mg/dL, Glu: 86 mg/dL UA: straw colored, pH: 6.8, specific gravity: 1.010, no glucose/ketones/nitrites/leukocyte esterase/hematuria/proteinuria Prostate-specific antigen (PSA): 10 ng/mL Based on these results, the following study is ordered: Transrectal US: diffusely enlarged prostate with no detectable focal lesions Transrectal US-guided prostate biopsy: epithelial and smooth muscle hyperplasia without malignant cells
Diagnosis Benign prostatic hypertrophy
Treatment Administered The patient was prescribed tamsulosin on an outpatient basis The patient was scheduled for a 1-month follow-up to note any changes in his symptoms and to remeasure his PSA
Follow-up The patient had improved voiding symptoms after tamsulosin was initiated The patient's PSA remained in the high-normal or minimally elevated range on future screenings
BPH develops in the central zone of the prostate adjacent to the urethra and does not predispose patients to prostate cancer.
Steps to the Diagnosis Benign prostatic hypertrophy (BPH) Benign enlargement of the prostate due to epithelial, stromal, and muscular hyperplasia Increases in frequency after 40 years of age in men Compression of the urethra causes a partial obstruction of urine flow and results in multiple urinary symptoms
History: urinary hesitancy, urinary frequency, urgency, nocturia, weak or intermittent urine stream, straining during urination, possible urge incontinence Physical examination: digital rectal examination (should be performed annually) detects a uniformly enlarged, rubbery prostate Tests: PSA may be normal or mildly increased Transrectal US is indicated in patients with a high PSA or significant urinary obstruction and will show a large homogenous prostate Prostatic biopsy will show hyperplasia of epithelial, stromal, and smooth muscular cells Treatment: α1-receptor blockers (e.g., terazosin, tamsulosin) and 5-α-reductase inhibitors (e.g., finasteride) improve symptoms by decreasing periurethral prostate size Transurethral resection of the prostate (TURP) is indicated for patients with significant obstructive symptoms who do not respond to medical therapy Open prostatectomy may be required for relief from very large prostates Laser, radiofrequency, and microwave ablation are techniques that use minimally invasive approaches to relieve urethral compression Outcomes: complications include urethral obstruction, hydronephrosis, renal failure, recurrent UTIs, and bladder stone formation Clues to the diagnosis: History: urinary hesitancy, urgency, nocturia Physical: enlarged prostate on digital rectal examination Tests: mildly increased PSA, enlarged prostate on US, epithelial and muscular hyperplasia on biopsy
A digital rectal examination and PSA measurement may be performed annually to screen for prostate cancer in all men older than 50 years old or those with a history of BPH.
Prostate cancer is the most common nondermatologic cancer in men; however, lung cancer is the greatest cause of cancer-related death in males, while prostate cancer is the second highest. Prostate cancer Adenocarcinoma of the prostate that occurs in the peripheral zone of the gland Risk factors: age >50 years, family history, high fat diet, history of prostatitis History: frequently asymptomatic, but symptoms of urinary hesitancy, weak urine stream, feeling of incomplete bladder emptying, weight loss, and back pain may occur in later disease Physical examination: digital rectal examination detects a nodular or irregular prostate Tests: Increased PSA, increased alkaline phosphatase UA may show hematuria Transrectal US will demonstrate an irregular prostate; bone scan, CXR, and CT are useful to detect metastases Biopsy results are graded according to the Gleason scale to determine disease severity and prognosis Biopsies are graded from 1 to 5 based on near-normal to highly atypical glandular appearance The grades of the two most common tissue patterns are added to form the Gleason score A score of 2 to 4 is considered low grade, 5 to 7 moderate grade, and 8 to 10 high grade Treatment: Older men with a Gleason score <6 and barely palpable disease are candidates for observation Early disease without extension may be treated by prostatectomy or radiation therapy Prostatectomy methods include radical retropubic, perineal, laparoscopic, and robotic techniques Radiation therapy may be by external beam radiation or implantation of radioactive seeds in the prostate (i.e., brachytherapy)
Locally extensive disease is typically treated by external beam radiation Antiandrogen therapy is used as an adjunctive therapy in patients with locally extensive disease or nonmetastatic disease with a Gleason score >7 and as a primary therapy in metastatic disease Chemotherapy may serve a role in metastatic disease that is nonresponsive to antiandrogen therapy Outcomes: Complications from the disease include metastasis and urinary obstruction Complications of prostatectomy include incontinence and impotence Complications of radiation therapy include incontinence, impotence, and colitis Patients with low grade disease have a 5% 15-year mortality rate, moderate 30%–70%, and high 60%–87% Why eliminated from differential: the findings of the digital rectal examination and the US make this diagnosis unlikely; the biopsy results rule it out Prostatitis Inflammation of the prostate that may be idiopathic or secondary to a UTI The nonbacterial form is more common than the infectious form History: perineal pain, dysuria, urinary frequency, urgency Physical examination: fever, digital rectal examination detects a boggy or nodular prostate that is frequently tender Tests: UA shows hematuria and white blood cells; nitrites and leukocyte esterase will be positive if the condition is due to an infectious nature Increased serum WBCs if due to infection Transrectal US demonstrates prostatic calcifications and gland enlargement Treatment: Infectious cases are treated with TMP-SMX or fluoroquinolones Nonbacterial cases are treated with analgesics and either α1-receptor blockers or 5-α-reductase inhibitors Suprapubic catheter placement may be performed for significant urethral obstruction Outcomes: prognosis is very good following institution of therapy Why eliminated from differential: the UA does not suggest an inflammatory process, the US is more consistent with BPH, and there is no prostatic tenderness during the digital rectal examination Bladder cancer Transitional cell carcinoma, squamous cell carcinoma, or adenocarcinoma of the bladder occurring within epithelial tissue Transitional cell carcinoma makes up the large majority of cases Risk factors: tobacco use, schistosomiasis, aniline dye exposure, petroleum byproducts exposure, recurrent UTI, and male gender History: painless gross hematuria, and suprapubic pain, urinary frequency, urgency, and dysuria occurring later in the disease course Physical examination: palpable suprapubic mass, lymphadenopathy Tests: UA shows hematuria; urine cytology shows malignant cells Cystoscopy is useful for the visualization of lesions, collection of specimens, and biopsy of lesions Biopsy is diagnostic for a malignant process and can determine the cells of origin CT, MRI, or IVP is important to determine the local extent of the disease Treatment: Transurethral cystoscopic resection may be performed for superficial tumors Partial or total cystectomy is performed for invasive tumors Adjuvant intravesical radiation therapy, chemotherapy, or immunotherapy are commonly performed
Extensive disease requires regional radiation therapy and systemic chemotherapy Outcomes: Complications include significant hematuria, urethral obstruction, urinary retention, incontinence, and bowel obstruction Recurrence following treatment is common (up to 25%) and may involve any location along the urinary tract Five-year survival is >80% in early disease but <20% in metastatic disease Why eliminated from differential: the absence of hematuria makes this diagnosis less likely; the prostatic examination findings make that region much more likely to be the cause of the patient's symptoms Urinary tract infection More thorough discussion in prior case Why eliminated from differential: the results of the UA do not suggest an infectious pathology
Case 4-11: “I'm having horrible groin pain!” A 20-year-old man presents to the emergency department with the acute onset of excruciating genital pain. The patient says he was participating in soccer practice at his university when he began to experience severe pain in his testicles. He quickly left the soccer field because of the pain, became nauseous, and vomited. He says that he can barely walk because of the pain. He was brought to the hospital by one of his coaches because of his distress. The coach says that this event first started 1 hour ago. The patient denies these symptoms ever having happened previously. He is active in collegiate level soccer but denies any recent genital trauma. He denies any abdominal pain, hematuria, urinary symptoms, penile discharge, hematochezia, constipation, or neurologic deficits. He denies any past medical history or medication use. He admits to often drinking more than five alcoholic drinks on three nights per week (typically weekends) in social settings but denies daily alcohol use, tobacco use, or any illicit substance use. He says that he has had three sexual partners in the past year but insists that he uses condoms consistently during sexual encounters. On examination, he is a healthy appearing man in obvious discomfort. He has no abdominal tenderness or masses. Auscultation reveals normal bowel sounds. Genital examination reveals a red, swollen scrotum that is tender to the touch. The left testicle appears to be elevated. Further lifting of the left testicle does not improve his pain. It is difficult to palpate any spermatic cord abnormalities because of the patient's discomfort. No cremasteric reflex is able to be elicited. No penile discharge is notable. A rectal examination notes a normal-sized prostate that is relatively nontender. The following vital signs are measured: T: 100.1°F, HR: 100 bpm, BP: 130/90 mm Hg, RR: 18 breaths/min
Differential Diagnosis Testicular torsion, epididymitis, urethritis, prostatitis, testicular cancer, cryptorchidism, hydrocele, varicocele, nephrolithiasis
Laboratory Data and Other Study Results UA: straw colored, pH: 6.8, specific gravity: 1.010, no glucose/ketones/nitrites/leukocyte esterase/hematuria/proteinuria Penile fluid culture: Gram stain and culture pending Testicular US: apparent twisting of the left testicle; Doppler suggests impaired perfusion to left testicle; no apparent testicular masses; normalappearing epididymis
Diagnosis Testicular torsion
Treatment Administered Manual derotation was attempted in the emergency department but failed The patient was rushed to the operating room by a urologist for an emergent scrotal exploration The left testicle was derotated and demonstrated to be viable, and an orchiopexy was performed to prevent recurrence
Follow-up The patient had a significant improvement in his symptoms following the procedure The patient was able to resume his normal activities following surgical healing and experienced no recurrences The infectious work-up performed on the penile fluid was negative for sexually transmitted diseases The patient was provided education regarding the risks of sexually transmitted diseases and binge drinking
Steps to the Diagnosis Testicular torsion Twisting of the testicle on the spermatic cord leading to the obstruction of vascular flow to the affected testicle History: very painful testes, nausea, vomiting Physical examination: swollen scrotum, testicular tenderness, superior displacement of a testicle, absent cremasteric reflex, fever, possible spermatic cord mass Tests: testicular US may show torsion, and Doppler studies will show insufficient vascular flow to the affected testicle Treatment: Manual derotation may be attempted but is not always successful and does not prevent recurrence Emergent surgical reduction of the testes should be performed within hours of the onset of symptoms The affected testicle is sutured to the scrotal wall (i.e., orchiopexy) following reduction to prevent recurrence Outcomes: delay of reduction beyond 6 hours of torsion significantly reduces testicular viability Clues to the diagnosis: History: painful testicle with sudden onset, nausea, vomiting Physical: testicular tenderness, absent cremaster reflex, superiorly displaced testicle Tests: US findings
If there is strong clinical suspicion for testicular torsion, tests will frequently only delay surgery, so the patient should be rushed to the operating room as soon as possible.
Ninety-five percent of all testicular tumors are of germ cell origin.
Testicular cancer is the most common malignancy in men between the ages of 15 and 35 years. Testicular cancer Cancer of the testicle from either a germ cell or stromal cell origin Germ cell tumors are either seminomatous or nonseminomatous Stromal cell tumors may arise from Leydig, Sertoli, or granulose cells Risk factors: prior history of testicular cancer, cryptorchidism, family history History: painless testicular mass, possible lower abdominal pain; GI or pulmonary symptoms may result from metastases Physical examination: palpable testicular mass that is nontender, possible gynecomastia Tests: Germ cell tumors frequently are associated with increased β-hCG (human chorionic gonadotropin) and α-fetoprotein Increased estrogen may be seen with stromal cell tumors Testicular US is useful to detect testicular masses Biopsy is usually required to determine the degree of malignancy and the cells of origin CXR and CT are useful to detect metastases and the extent of the disease Treatment: Radical orchiectomy either with or without adjuvant chemotherapy and radiation therapy is performed for early stage seminomas Radical orchiectomy with or without a retroperitoneal lymph node dissection and chemotherapy is performed for early stage nonseminomas Radical orchiectomy and chemotherapy is indicated for stromal cell tumors Primary chemotherapy and possible postchemotherapy debulking may be performed for extensive disease Outcomes: prognosis for early disease tends to be very good, but nonseminomas are associated with a higher recurrence rate
Why eliminated from differential: testicular tumors are generally painless and would not be associated with the sudden onset of pain seen in this case; the US did not detect a mass and helps to rule out this diagnosis Epididymitis Inflammation of the epididymis associated with other male urogenital tract pathology Occurs due to prostatitis, sexually transmitted diseases (STDs), or urinary reflux History: gradual onset epididymal pain relieved by supporting the scrotum (i.e., Prehn sign), dysuria, urinary frequency and urgency Physical examination: scrotal tenderness, scrotal induration, preserved cremasteric reflex Tests: Possible increased serum WBCs UA may show WBCs Culture of penile fluid is useful to detect bacterial causes Testicular US Doppler studies will show increased testicular blood flow Treatment: ceftriaxone and either doxycycline or a fluoroquinolone are given empirically pending the urethral swab cultures; NSAIDs and scrotal support are recommended for pain relief Outcomes: complications include abscess formation and infertility Why eliminated from differential: the acute onset of pain in this case is less suggestive of this case, Prehn sign is negative, the cremasteric reflex is absent, no WBCs are seen on the UA, and the US demonstrates decreased testicular blood flow
Chlamydia is the most common cause of epididymitis in adolescents and young adults.
Supporting the scrotum will not relieve testicular pain in cases of torsion. Urethritis Infection of the urethra most commonly due to sexually transmitted Neisseria gonorrhoeae or Chlamydia trachomatis History: burning dysuria, urinary frequency, urinary urgency, urethral itching Physical examination: possible purulent penile discharge Tests: N. gonorrhoeae will appear as Gram-negative diplococci on Gram stain and will grow on Thayer-Martin agar Negative Gram stains in the presence of high clinical suspicion are typically suggestive of C. trachomatis infection C. trachomatis infection may be confirmed with polymerase chain reaction (PCR) amplification and testing Treatment: Single dose ceftriaxone with either doxycycline or a fluoroquinolone are used to treat both infections simultaneously Sexual partners must be treated to avoid reinfection Cases typically need to be reported to a public health office Outcomes: complications are rare and include urethral strictures and abscess formation Why eliminated from differential: because of the high clinical suspicion for testicular torsion, treatment for that diagnosis was undertaken before the culture results were reported for the patient; the negative culture and the lack of suggestive urinary symptoms make this diagnosis unlikely Prostatitis More thorough discussion in prior case Why eliminated from differential: the lack of urinary symptoms, nontender prostate, and absence of WBCs on the UA make this diagnosis unlikely Cryptorchidism Testes that do not lie consistently within the scrotum and may remain in the abdominal cavity
History: testes inconsistently located in scrotum Physical examination: possibly empty scrotum, nontender testicles Tests: US may be used to confirm the existence of testes when they are unable to be palpated Treatment: exogenous hCG administration or surgical reduction of the testes into the scrotum is performed before 5 years of age to reduce the risk of testicular cancer and to allow normal testicular development Outcomes: complications include an increased risk of testicular cancer and infertility Why eliminated from differential: due to the severe acuity of the patient's symptoms, it is unlikely that his elevated testicle is due to this diagnosis; the US results further rule out this diagnosis Hydrocele Collection of fluid adjacent to the spermatic cord within the scrotum History: painless enlarged scrotum Physical examination: transillumination of the scrotum shows appreciable scrotal fluid, testes are palpable posterior to the fluid collection Tests: UA may show proteinuria or WBCs; US will detect a cystic mass adjacent to the spermatic cord Treatment: elective surgical correction Outcomes: prognosis is excellent following correction; there is a risk of inguinal herniation prior to correction Why eliminated from differential: the patient's symptoms (e.g., severe acute pain) make this diagnosis unlikely Varicocele Dilation of the testicular venous plexus Dysfunction of sperm production and regulation may occur although the relation to the varicocele is incompletely understood History: frequently asymptomatic although the patient may report infertility or mild pain Physical examination: palpable nontender scrotal mass that feels like a “bag of worms” Tests: testicular US with Doppler imaging is used to confirm the diagnosis Treatment: varicoceles related to a history of infertility may be treated by surgical excision or interventional radiographic embolization Outcomes: approximately 60% of patients will have successful treatment of the associated infertility; rare complications of surgical treatment include hydrocele formation, recurrence, and vascular injury Why eliminated from differential: the patient's pain in this case is in excess to what would be expected for this diagnosis, and the characteristic scrotal mass for this diagnosis is not detected Nephrolithiasis More thorough discussion in prior case Why eliminated from differential: the pattern of pain in this patient (e.g., scrotal) is more consistent with a testicular pathology than a renal pathology (e.g., flank pain); the US findings confirm this assessment
Authors: Van Kleunen, Jonathan P. Title: Step-Up to USMLE Step 3, 1st Edition Copyright ©2009 Lippincott Williams & Wilkins > Table of Contents > Chapter 5 - Endocrinology
Chapter 5 Endocrinology Basic clinical primer Normal Glucose Metabolism Postprandial period Feeding results in an increase in the blood concentration of glucose Insulin is secreted by pancreatic β-islet cells in response to the increased serum glucose concentration (also induced to a lesser extent by other protein and neural input) Insulin drives glucose anabolism Glucose is converted into glycogen, fatty acids, and pyruvate (Figure 5-1) Glycogen is stored in the liver and in skeletal muscle; fatty acids (i.e., triglycerides) are stored in adipose cells Pyruvate and fatty acids are also incorporated into amino acid production Lipolysis of adipose tissue is inhibited by insulin secretion Fasting period Fasting results in a decrease in the blood glucose concentration Glucagon is secreted by pancreatic α-islet cells in response to the dropping glucose level Glucagon drives catabolism of glycogen and fatty acids
Presurgical Assessment of Diabetic Patients Diabetic patients have greater surgical risks of postoperative infection, complicated wound healing, adverse cardiac events, and postoperative mortality Tight glycemic control should be achieved postoperatively via frequent serum glucose checks and the administration of insulin as needed Insulin needs of the patient frequently increase from the baseline level postoperatively due to an augmented body stress response
Thyroid Function Thyroid hormones (i.e., thyroxine, or T4, and triiodothyronine, or T3) are important for determining the body's basal metabolic rate, promoting bone growth, increasing cardiac output in times of stress, and driving the
maturation of the central nervous system (CNS) during fetal growth The hypothalamus secretes thyrotropin-releasing hormone (TRH) to induce the secretion of thyroid stimulating hormone (TSH) from the anterior pituitary (Figure 5-2) Free T4 and T3 determine the degree of metabolic activity; nonfree thyroid hormones are bound to thyroid-binding globulin (TBG) in the circulation The serum T4 level is responsible for the feedback inhibition of TSH secretion Calcitonin is secreted by the parafollicular cells of the thyroid and plays a minor role in serum calcium regulation
If TBG levels increase (e.g., pregnancy, oral contraceptive use), total T4 increases but free T4 remains normal.
Nephrotic syndrome and androgen use decrease TBG levels, leading to a decreased total T4 but a normal free T4. P.133
Figure 5-1 Glucose metabolism and its dependence on feeding states. (A) During the postprandial stage, insulin induces the anabolism of glucose to form glycogen and fatty acids, which are stored in the liver and skeletal muscle and in adipocytes, respectively. (B) During fasting, glucagon induces the breakdown of glycogen and fatty acids to increase the blood glucose level. (See color image.)
P.134
Figure 5-2 Normal thyroid hormone secretion within the hypothalamic-pituitary axis. I-, iodine; T 3, triiodothyronine; T 4, thyroxine; TRH, thyroid-releasing hormone; TSH, thyroid-stimulating hormone. (See color image.)
Figure 5-3 Regulation of serum calcium levels. Ca, calcium; PTH, parathyroid hormone.
P.135
Parathyroid Function The parathyroid glands secrete parathyroid hormone (PTH), the chief hormone behind maintaining serum calcium concentrations (Figure 5-3) Secretion of PTH increases with low serum calcium levels PTH induces osteoclasts to reabsorb bone and subsequently increase serum calcium levels PTH induces the conversion of 25-(OH) vitamin D to 1,25-(OH)2 vitamin D, induces distal tubule calcium reabsorption, and inhibits phosphate reabsorption in the kidneys 1,25-(OH)2 vitamin D increases the intestinal absorption of calcium and increases proximal tubule reabsorption of phosphate in the kidney to balance the activity of PTH
Hypothalamic-Pituitary Axis The hypothalamus is responsible for the modulation of pituitary gland activity (Figure 5-4)
Figure 5-4 Modulation of pituitary activity by the hypothalamus. ACTH, adrenocorticotropic hormone; ADH, antidiuretic hormone; CRH, corticotrophin-releasing hormone; FSH follicle-stimulating hormone; GH, growth hormone; GHRH, growth hormone-releasing hormone; GnRH, gonadotropin-releasing hormone; H 2O, water; LH, luteinizing hormone; PRH, prolactin-releasing hormone; T 3, triiodothyronine; T 4, thyroxine; TRH, thyrotropin-releasing hormone; TSH, thyroid-stimulating hormone.
P.136 The anterior pituitary is regulated by the release of hormones from the hypothalamus into the hypophyseal portal system and by the feedback mechanisms of pituitary hormones The anterior pituitary is responsible for the secretion of prolactin, ACTH, TSH, GH, FSH, and LH The posterior pituitary is regulated by neural impulses sent from the hypothalamus through the hypothalamohypophyseal tract
The posterior pituitary is responsible for the secretion of ADH and oxytocin
The hormones produced by the pituitary gland may be remembered by the mnemonic GOAT FLAP: GH (growth hormone), Oxytocin (posterior), ADH (posterior; antidiuretic hormone), TSH, FSH (follicle-stimulating hormone), LH (luteinizing hormone), ACTH (adrenocorticotropic hormone), Prolactin.
Adrenal Function The adrenal glands are divided into layers that differ in their enzyme products and function Zona glomerulosa Outer layer of the cortex Activity is stimulated by the renin-angiotensin system Aldosterone is the main product, which serves to maintain body sodium and fluid volume Zona fasciculata Middle layer of the cortex Activity is stimulated by ACTH Cortisol is the main product, which maintains glucose production from proteins, aids in fat metabolism, aids in vascular regulation, influences immune function, and aids in neural regulation Zona reticularis Deep layer of the cortex Activity is stimulated by ACTH Androgens are the main products, which drive the development of secondary sexual characteristics, increase bone and muscle mass, and promote male sexual differentiation and sperm production Medulla Activity is stimulated by preganglionic sympathetic neurons Norepinephrine and epinephrine are the main products, which function as postsynaptic neurotransmitters in the sympathetic autonomic nervous system
The layers and activities of the adrenal cortex may be remembered by the mnemonic “Great Attire And Fast Cars Are Really Sexy Attributes”: Granulosa secretes Aldosterone after Angiotensin II stimulation, Fasciculata secretes Cortisol after ACTH stimulation, Reticularis secretes Sex hormones after ACTH stimulation.
Case 5-1: “My son drinks too much” A 10-year-old boy is brought to his pediatrician by his mother, who is concerned that he drinks large amounts of water frequently. She says that he is constantly thirsty and drinking water and that he needs to go to the bathroom many times every day. She is not sure how long this has been occurring, but it has been noticeable at least over the past month. The boy says that he feels thirsty very frequently and guesses that he needs to urinate at least five times per day. He wakes up in the middle of the night needing to urinate on most nights. He also reports being hungry during most of the day. He occasionally feels very tired and not well and usually eats something during these episodes. He
denies any areas of pain, dizziness, numbness, dysuria, vomiting or diarrhea, or hematochezia. His mother says that he is a generally healthy boy but has had several colds in the past year. He takes children's vitamins but no other medications. There are no particular conditions that are common in the family. On examination, he is a thin boy in no distress. Auscultation of his heart, lungs, and bowels is normal. He has no pain in his flanks or abdomen and no palpable masses. His neurologic examination is normal. His mucous membranes are mildly dry. He appears to have no abnormalities of his penis or genitals. The following vital signs are measured: Temperature (T): 98.8°F, heart rate (HR): 85 beats per minute (bpm), blood pressure (BP): 110/70 mm Hg, respiratory rate (RR): 16 breaths/min P.137
Differential Diagnosis Diabetes insipidus, hyperthyroidism, diabetes mellitus (types 1 and 2), psychogenic polydipsia, acute renal failure
Laboratory Data and Other Study Results Complete blood cell count (CBC): white blood cells (WBC): 7.1, hemoglobin (Hgb): 15.8, platelets (Plt): 239 7-electrolyte chemistry panel (Chem7): sodium (Na): 133 mEq/L, potassium (K): 4.2 mEq/L, chloride (Cl): 101 mEq/L, carbon dioxide (CO2): 25 mEq/L, blood urea nitrogen (BUN): 14 mg/dL, creatinine (Cr): 0.5 mg/dL, glucose (Glu): 265 mg/dL Urinalysis (UA): straw colored, pH: 5.7, specific gravity: 1.010, 3+ glucose, 1+ ketones, no nitrites/leukocyte esterase/hematuria/proteinuria Thyroid panel: TSH 1.2 µU/mL, T4: 5.7 µg/dL, free T4: index 6.3, T3: 1.0 ng/mL, T3 reuptake: 0.87 Based on these results, the following tests are performed on a different day: Chem7: Na: 132 mEq/L, K: 4.1 mEq/L, Cl: 101 mEq/L, CO2: 25 mEq/L, BUN: 15 mg/dL, Cr: 0.5 mg/dL, Glu: 267 mg/dL Hemoglobin A1c (HbA1c): 9.8% Serum osmolality: 310 mOsm/kg
Diagnosis Diabetes mellitus type 1
Treatment Administered The patient was started on a insulin regimen of regular insulin at breakfast and dinner and NPH (neutral protamine Hagedorn) insulin at breakfast and bedtime The patient and his family were trained in home glucose measurements and a self-measurement program was initiated The patient was referred to a nutritionist for dietary counseling The patient was referred to an endocrinologist and ophthalmologist to be evaluated for any renal or retinal deficits
Follow-up The patient and his family were able to comply with the recommended therapeutic regimen The patient's follow-up HbA1c levels were able to be maintained near 7%
The patient's symptoms improved following the successful control of his glucose levels
Rubella, Coxsackie virus, and mumps are viruses that have been associated with onset of β-islet cell destruction leading to DM type 1.
HbA1c is a useful tool for measuring the effectiveness of a glycemic control regimen or patient compliance because it indicates the trends of serum glucose levels over an extended period of time.
Steps to the Diagnosis Diabetes mellitus type 1 (DM type 1) Impairment or loss of pancreatic insulin production due to an autoimmune destruction of β-islet cells Strong association with human leukocyte antigen (HLA) DR3, DR4, and DQ genotypes Most frequently diagnosed before puberty History: polyuria, polydipsia, polyphagia, weight loss, rapid onset of symptoms Physical examination: signs of mild dehydration Tests: Increased serum glucose; decreased insulin (Table 5-1) HbA1c is increased and is a more accurate assessment of glucose levels over the most recent 3 months than individual random serum glucose levels UA will demonstrate glycosuria and possible urinary ketones P.138
Table 5-1 Plasma Glucose Diagnostic Criteria for Diabetes Mellitus 1
Plasma Gluc ose Test
Level (mg/dL)
And…
Random plasma glucose
>200
With symptoms of DM
Fasting plasma glucose
>126
On two separate occasions
Plasma glucose
>200
Two hours after 75 g oral glucose load2
DM, diabetes mellitus.
1
Diagnosis is based on the occurrence of at least one of the following findings.
2
This is a positive oral glucose tolerance test.
Treatment:
An insulin regimen of scheduled injections or a continuous infusion pump is the keystone to glycemic control (Figure 5-5, Table 5-2) A healthy diet is important to controlling glucose levels, and referral to a nutritionist may be appropriate
Figure 5-5 Examples of insulin regimens in diabetes mellitus. (A) Injection of regular and NPH (neutral protamine Hagedorn) insulin (in 2:1 ratio) together at breakfast followed by a second regular insulin dose at dinner and a second NPH dose at bedtime. In some cases the second dose of NPH may be given concomitantly with the second regular insulin dose at dinner. (B) Adjustment in regimen (A) to achieve tighter, shorter time control by adding an additional dose of regular insulin at lunch and only administering NPH at bedtime. In patients with overnight hypoglycemia, insulin detemir may be substituted for NPH. (C) Tight control regimen using very-rapid-acting insulin at meals and a bedtime dose of insulin glargine.
P.139
Table 5-2 Formulations of Injected Insulin
Ty pe of Insulin
Time of Onset of Ac tion
Peak Effec t
Duration of Ac tion
Very-rapid-acting 1 (e.g., lispro, aspart, glulisine)
10 min
1 hr
2–4 hr
Regular
30 min
2–4 hr
5–8 hr
NPH
2 hr
6–10 hr
18–24 hr
Insulin glargine
2 hr
No peak
24 + hr
Insulin detemir
2 hr
No peak
~ 24 hr
1
Appropriate for use in continuous infusion pump.
Self-monitoring of serum glucose levels is important to guide insulin dosing Inclusion of a multidisciplinary team (e.g., diabetes specialists, ophthalmologists) is helpful to maintain adequate glycemic control and to avoid the onset of complications Outcomes: Diabetic ketoacidosis may result from acute glycemic instability and poor control Complications from chronic poor glycemic control include retinopathy, nephropathy, neuropathy, impaired wound healing, vascular insufficiency, and atherosclerosis Although the life expectancy for type 1 diabetics is somewhat less than for unaffected individuals, patients should be able to avoid significant complications with tight glycemic control Clues to the diagnosis: History: polyuria, polydipsia, polyphagia Physical: mild dehydration Tests: increased serum glucose, increased HbA1c, glycosuria and urinary ketones Diabetes mellitus type 2 (DM type 2) Development of tissue resistance to insulin leading to hyperglycemia and an eventual decrease in the β-islet cells' ability to produce insulin Although chronic hyperglycemia has similar deleterious effects for both type 1 and type 2 DM, their pathologies and presentations are different (Table 5-3) Type II DM constitutes almost all cases of DM diagnosed in adults (especially in those >40 years old), but may be detected during childhood at similar ages to DM type 1
Table 5-3 Comparisons of Diabetes Mellitus Types 1 and 2
DM Type 1
DM Type 2
Cause
Likely autoimmune destruction of β-islet cells
Development of insulin resistance in tissues
Inheritance/genetics
HLA-linked
Strong family history
Age of onset
Usually <13 years old
Frequently >40 years old
Onset of symptoms
Rapid
Gradual
Pancreatic effects
β-islet cell depletion
Gradual decrease in β-islet cells
Serum insulin
Low
Increased or normal; low in later disease
Body type
Thin
Obese
Acute complications
DKA
HHNC
Treatment
Insulin
Oral hypoglycemic agents, possibly insulin
DKA, diabetic ketoacidosis; DM, diabetes mellitus; HHNC, hyperosmolar hyperglycemic nonketotic coma; HLA, human leukocyte antigen.
P.140
Table 5-4 Oral Hypoglycemic Drugs Used in Treatment of Diabetes Mellitus Type 2
Drug
Mec hanism
Role
Frequently first-line drug
Adverse Effec ts
GI disturbance, rare lactic acidosis,
Biguanides (e.g.,
Dec reases hepatic
metformin)
gluc oneogenesis, inc reases
possible decreased vitamin B12
insulin ac tivity , reduces
absorption; contraindicated in
hyperlipidemia
patients with hepatic and renal insufficiency
Sulfonylureas (e.g.,
Stimulates insulin release from
Frequently used after
Hy poglyc emia; contraindicated in
tolbutamide, glyburide,
β-islet cells, reduces serum
metformin or as first-line drug
patients with hepatic or renal
glipizide)
glucagon, increases binding of
insufficiency (greater risk of
insulin to tissue receptors
hypoglycemia)
Thiazolidinediones
Decreases hepatic
(“glitazones”)
gluconeogenesis, increases tissue uptake of glucose
Adjunct to other drugs
Weight gain, increase serum LDL, rare liver toxicity in some drugs
α-Glucosidase
Dec reases GI absorption of
Monotherapy in patients with
inhibitors (e.g.,
starch and disaccharides
good dietary control of DM;
Diarrhea, flatulence, GI disturbance
adjunct to other drugs; may
acarbose)
be used in patients with DM type 1
Meglitinides (e.g.,
Stimulates insulin release from
Used as secondary drug with
Hy poglyc emia; significantly more
repaglinide,
β-islet cells
metformin or rarely as initial
expensive than sulfonylureas with
drug
no therapeutic advantage
nateglinide)
DM, diabetes mellitus; GI, gastrointestinal; LDL, low-density lipoprotein.
Risk factors: family history, obesity, lack of exercise History: generally asymptomatic in the early stages and associated with a gradual onset of symptoms including polyuria, polydipsia, polyphagia, and symptoms associated with complications Physical examination: frequent obesity, signs associated with complications Tests: Increased serum glucose and HbA1c (Table 5-1) Serum insulin levels are not a reliable indicator of the condition UA will show glycosuria and urinary ketones Treatment: Initial therapy focuses on nutrition (e.g., reduced calorie intake, carbohydrate control, and consistency), exercise, and weight loss Metformin is typically the first oral agent prescribed in patients who are unable to control their glycemic level with lifestyle modification alone (Table 5-4) A sulfonylurea or thiazolidinedione is added to the regimen if the HbA1c is >7% after 3 months of therapy An insulin regimen is initiated if the HbA1c is consistently >8.5% or if the patient demonstrates consistent low insulin levels Self-monitoring of blood glucose levels is important to guide therapy Outcomes: Hyperosmolar hyperglycemic nonketotic coma (HHNC) may result from acute glycemic instability and poor control Complications include retinopathy, nephropathy, neuropathy, impaired wound healing, vascular insufficiency, and atherosclerosis (Figure 5-6, Table 5-5) Prognosis is directly linked to the ability to control glucose levels and avoid the development of complications Why eliminated from differential: the patient's age, body habitus, and rapid onset of symptoms make type 1 DM the most likely form of the disease P.141
Figure 5-6 Anteroposterior x-ray of the right shoulder in a patient with Charcot arthropathy. There is complete destruction of the right proximal humerus following a prior fracture. Due to peripheral neuropathy, these patients may be unaware of the severe destructive process in their joints.
Diabetes insipidus More thorough discussion in Chapter 4 Why eliminated from differential: hypernatremia would be expected for this diagnosis instead of the hyponatremia seen in this case Hyperthyroidism More thorough discussion in later case Why eliminated from differential: the normal thyroid panel rules out this diagnosis Psychogenic polydipsia More thorough discussion in Chapter 4 Why eliminated from differential: low serum osmolality would be expected for this diagnosis and not the elevated serum osmolality seen in this case Acute renal failure More thorough discussion in Chapter 4 Why eliminated from differential: the normal BUN and creatinine seen in this case rule out this diagnosis
Background retinopathy involves no neovascularization and constitutes the majority of cases; proliferative diabetic retinopathy consists of neovascularization and carries a much higher risk of retinal hemorrhage.
Diabetic patients are at an increased risk of silent myocardial infarction (MI) because of impaired pain sensation.
The first step to treating any complication of chronic DM is to improve glycemic control.
Case 5-2: “My ankle looks funny” A 58-year-old woman presents to her primary care provider (PCP) because she feels that her ankle has a strange appearance. She says that she broke her right ankle in a car collision 1 year ago and required open reduction and internal fixation to repair the fractures. She followed up two times with the orthopedic surgeon after her surgery but neglected to follow-up after she felt comfortable walking again. She continues to walk on her right leg without pain but feels that her ankle is unstable. In addition, she feels that her ankle looks swollen and misshapen. She denies having seen this appearance several months prior to the current presentation. She denies any additional episodes of trauma. She feels that her extremities are strong and denies paresthesias. She denies fevers, wounds over her ankle, or drainage from the ankle. She says that she does not have this problem in any of her other joints. She recalls being told by the orthopedic surgeon to follow-up with a PCP because of elevated blood sugar, but she never followed his recommendation. She has not seen an internist for several years because her last PCP told her to stop eating sweets and fried food and to start P.142 eating healthier. She is only seeing a PCP today because her insurance policy requires a new referral for her to see an orthopedic surgeon. She is unsure of her past medical history and denies taking any medications currently. Her previous doctor wanted her to start “some drugs,” but she forgets what he wanted to prescribe for her. She denies any substance use. On examination, she is an obese woman in no acute distress. Auscultation of her heart, lungs, and bowels is normal. She has no pain on palpation of her chest or abdomen. She has no lymphadenopathy. She is able to perform coordinated movements well. She has a slight limp due to some instability with weight on her right leg. She has minimally decreased motor strength in her lower extremities (5–/5) compared to her upper extremities. Testing of her sensation reveals significant sensory deficits in both lower extremities below the midcalf level in regards to two-point discrimination. She P.143 demonstrates some impaired lower extremity proprioception deficits with her eyes closed. Her right ankle appears to be mildly swollen and erythematous. Raising her right leg causes the erythema to resolve after a minute of elevation. The following vital signs are measured: T: 98.5°F, HR: 79 bpm, BP: 145/90 mm Hg, RR: 16 breaths/min
Table 5-5 Complications of Chronic Diabetes Mellitus
Condition
Retinopathy
Pathology
Diagnosis
Prognosis
Vascular occlusion of retinal
Progressive vision loss,
Anti-HTN therapy, routine
Progressive vision loss
vessels with possible
premature cataracts, retinal
ophthalmologic follow-up, laser
without control of HTN
neovascularization, leading to
changes (e.g., arteriovenous
photocoagulation of
and DM; other
microaneurysms,
nicking, hemorrhages, edema,
neovascularization, reduction
complications include
hemorrhage, infarcts, and
infarcts)
of macular edema with
retinal detachment,
intervitreal corticosteroid
cataracts, and
injections
glaucoma
macular edema
Nephropathy
Treatment
Progressive intercapillary
Chronic DM, HTN, proteinuria,
ACE-I or ARBs are used to
End-stage renal
glomerulosclerosis, mesangial
hypoalbu minemia, inc reased
control HTN, low-protein diet,
disease occurs in 50%
expansion, and basement
BUN and Cr, basement
dialysis in late stages
of patients with DM
membrane degeneration
membrane thickening and
type 1 and in 10% of
leading to renal
Kimmelstiel-Wilson nodules on
patients with DM type
insuffic ienc y and eventual
electron microscopy of renal
2 within 10 years of
nephrotic sy ndrome
biopsies
the diagnosis of nephropathy
Neuropathy
Progressive neural damage
Sensory abnormalities include
Neural pain may be treated
Chronic sensory impair
and conduction abnormalities
stocking-glove paresthesias,
with tricyclic antidepressants,
ment leads to Charc ot
due to metabolic and vascular
neural pain, and dec reased
phenytoin, carbamazepine,
joints and foot ulc ers
insufficiency leading to
vibratory and pain sensation;
gabapentin, or analgesics;
that may eventually
sensory autonomic , and
autonomic abnormalities include
patients should be instructed in
require amputations
motor dysfunction
hypotension, impotence,
regular foot examinations to
incontinence, and delayed
detect skin breakdown
gastric emptying; motor abnormalities include weakness and a loss of coordination
Atherosclerosis
Microvascular insufficiency
Increased incidence of MI,
Control of HTN and
Complications include
leading to CAD, PVD, and
extremity ischemia, and ulcer
hyperlipidemia, daily ASA
MI, PVD necessitating
impaired wound healing
formation
amputations, and poor wound healing; cardiac issues are the greatest cause of death in diabetics
ACE-I, angiotensin converting enzyme inhibitor; ARB, angiotensin receptor blocker; ASA, aspirin; BUN, blood urea nitrogen; CAD, coronary artery disease; Cr, creatinine; DM, diabetes mellitus; HTN, hypertension; MI, myocardial infarction; PVD, peripheral vascular disease
Differential Diagnosis Diabetic neuropathy, Charcot-Marie-Tooth disease, rheumatoid arthritis, osteoarthritis, Lyme disease, osteomyelitis, septic arthritis
Laboratory Data and Other Study Results Right ankle x-ray: significant destruction of the ankle joint with near eradication of the distal tibia and fibula and proximal talus; no focal lesions outside of the remaining joint region CBC: WBC: 6.5, Hgb: 14.5, Plt: 431 10-electrolyte chemistry panel (Chem10): Na: 142 mEq/L, K: 4.4 mEq/L, Cl: 109 mEq/L, CO2: 24 mEq/L, BUN: 19 mg/dL, Cr: 1.2 mg/dL, Glu: 223 mg/dL, magnesium (Mg): 2.3 mg/dL, calcium (Ca): 10.2 mg/dL, phosphorus (Phos): 4.2 mg/dL Erythrocyte sedimentation rate (ESR): 11 mm/hr C-reactive protein (CRP): 0.5 mg/dL Rheumatoid factor (RF): negative Lyme antibody enzyme-linked immunosorbent assay (ELISA): negative Electromyography (EMG): significantly decreased bilateral lower extremity sensory action potentials; low normal conduction velocities; mild decrease in motor action potentials
Diagnosis Diabetic neuropathy (DM type 2) with associated Charcot arthropathy
Treatment Administered
The patient was started on metformin to control her hyperglycemia The patient was provided education on foot examinations, healthy life choices, and diabetic diets The patient was referred to a nutritionist to improve her eating habits and an orthopedic surgeon for immobilization of her ankle and possible fusion
Follow-up The patient was able to achieve control of her blood glucose levels with metformin She required close follow-up for the detection of additional diabetic complications A tibial-talar fusion was performed on her right ankle because of her severe degree of joint destruction
Steps to the Diagnosis Diabetic neuropathy, DM type 2 More thorough discussion in prior case Clues to the diagnosis: History: prior fracture, prior concerns regarding diet and blood sugar Physical: swollen and erythematous joint, impaired sensation, impaired proprioception Tests: increased glucose, x-ray appearance, EMG findings P.144 Charcot-Marie-Tooth disease More thorough discussion in Chapter 7 Why eliminated from differential: the patient's hyperglycemia makes this diagnosis less likely, and the lack of conduction velocity delays on the EMG helps to rule this diagnosis out Rheumatoid arthritis More thorough discussion in Chapter 9 Why eliminated from differential: this degree of joint destruction is not common in rheumatoid arthritis, and the negative rheumatoid factor helps to rule this diagnosis out Osteoarthritis More thorough discussion in Chapter 9 Why eliminated from differential: this degree of joint destruction is rare for this diagnosis Lyme disease More thorough discussion in Chapter 9 Why eliminated from differential: the negative antibody test for Lyme infection and the normal ESR and CRP rule this diagnosis out Osteomyelitis/septic arthritis More thorough discussion in Chapter 9
Why eliminated from differential: the normal WBC count, ESR, and CRP rule out this diagnosis
Case 5-3: “I found my roommate passed out” A 19-year-old man is brought into the emergency department by his college roommate after the latter found him poorly responsive in their dorm room. He said that when he came home from dinner he found the patient lying on the floor passed out. When he tried to arouse him, the patient opened his eyes and tried to speak but otherwise was barely interactive. The roommate says that the patient had been “cramming” for upcoming final exams and had been getting very little sleep over the prior week. He notes that the patient said he had not been feeling well for a few days because of nausea and abdominal pain and had been going to the bathroom often. He has never seen his roommate in this condition prior to tonight. The roommate says that the patient is diabetic and uses insulin, but he does not know of any other medical history. He says that the patient drinks socially, but he does not think that the patient has been drinking while he has been studying. On examination, the patient is a thin male who is minimally responsive to questioning. His mucous membranes appear very dry, and he has decreased skin turgor. His breath smells very fruity. Auscultation of his heart and lungs detects slow deep breathing in all lung fields and tachycardia. His abdomen has no detectable masses and has normal bowel sounds. He withdraws to painful stimuli but does not follow commands. The following vital signs are measured: T: 98.4°F, HR: 110 bpm, BP: 100/65 mm Hg, RR: 10 breaths/min
Differential Diagnosis Diabetic ketoacidosis, hyperosmolar hyperglycemic nonketotic coma, intoxication, hypoglycemia, MI
Laboratory Data and Other Study Results CBC: WBC: 13.7, Hgb: 16.5, Plt: 340 Chem10: Na: 131 mEq/L, K: 4.4 mEq/L, Cl: 93 mEq/L, CO2: 17 mEq/L, BUN: 45 mg/dL, Cr: 0.9 mg/dL, Glu: 589 mg/dL, Mg: 1.8 mg/dL, Ca: 9.7 mg/dL, Phos: 2.1 mg/dL Arterial blood gas (ABG; room air): pH: 7.18, partial pressure of oxygen (pO2): 96 mm Hg, partial pressure of carbon dioxide (pCO2): 20 mm Hg, bicarbonate (Bicarb): 18 mEq/L, oxygen saturation (O2 sat): 99% P.145 Serum osmolality: 317 mOsm/kg UA: dark yellow colored, pH: 5.2, specific gravity: 1.411, 4+ glucose, 4+ ketones, no nitrites/leukocyte esterase/hematuria/proteinuria Urine toxicology screen: negative for alcohol or any illicit substances Blood cultures: Gram stain and culture pending Urine culture: Gram stain and cultures pending Electrocardiogram (ECG): sinus tachycardia; no abnormal wave morphology
Diagnosis Diabetic ketoacidosis
Treatment Administered The patient was promptly given intravenous (IV) hydration and placed on an IV insulin infusion The patient was admitted to the intensive care unit (ICU) for close observation, correction of his calculated fluid deficit, and the titration of his serum glucose levels with IV insulin
He was started on empiric vancomycin and gentamicin while cultures remained pending
Follow-up The patient's mental status progressively improved with the normalization of his serum glucose level and the correction of his fluid deficit All culture results were negative, and all antibiotics were stopped The patient's insulin schedule was adjusted to maintain control of his glucose levels Prior to discharge, the patient was educated regarding risk factors and precautions for diabetic ketoacidosis (DKA) and how to avoid future episodes
Inciting factors for DKA and HHNC may be remembered by the mnemonic PHAT MINDS: Pancreatitis, Hot weather, Alcohol, Trauma, MI, Insufficient water intake, Noncompliance with therapy, Drugs, Stroke.
DKA occurs most frequently in patients with DM type 1 and is rarely seen in DM type 2.
Serum glucose is rarely >1,000 mg/dL in DKA.
Steps to the Diagnosis Diabetic ketoacidosis A complication of DM due to extremely low insulin levels and an excess of glucagon Significant degradation of glycogen into glucose and triglycerides into fatty acids occurs Significant hyperglycemia occurs, and metabolism of the fatty acids produces ketones and ketoacids Most commonly occurs in patients with DM type 2 who are noncompliant with their insulin regimens or who have infections, significant stress, recent MI, or significant alcohol use History: weakness, polyuria, polydipsia, abdominal pain, vomiting Physical examination: signs of dehydration (e.g., dry mucous membranes, decreased skin turgor), fruity breath odor, tachypnea or Kussmaul respirations (i.e., slow and deep breathing), mental status changes Tests: Serum glucose 300 to 800 mg/dL; increased serum ketones Decreased sodium and phosphate; normal or increased potassium and BUN Increased serum osmolality High anion gap metabolic acidosis UA will show glycosuria and urine ketones Treatment: IV rehydration with replacement of the fluid deficit over 24 to 48 hours IV insulin titrated to achieve a consistent normal serum glucose
Any underlying pathology should be treated The patient's insulin regimen should be adjusted to avoid relapses P.146 Outcomes: prognosis is generally excellent with prompt treatment, but comatose patients carry a poor prognosis Clues to the diagnosis: History: history of DM and insulin use, stressful environment, abdominal pain Physical: dehydration, fruity breath, Kussmaul respirations Tests: increased glucose, glycosuria and urine ketones, metabolic acidosis Hyperosmolar hyperglycemic nonketotic coma Significant dehydration in diabetic patients due to hyperglycemia and significant osmotic diuresis Occurs in patients with DM type 2 who are able to maintain a sufficient insulin production to prevent DKA Often associated with lengthy infections, stress, or other illnesses History: polyuria, polydipsia Physical examination: signs of dehydration, mental status changes, tachypnea Tests: Serum glucose is frequently >800 mg/dL Decrease sodium and potassium; increased BUN No metabolic acidosis UA shows glycosuria and possible urine ketones Treatment: IV hydration with gradual correction of the fluid deficit IV insulin is used to control the serum glucose level Patient education is needed to prevent recurrences Outcomes: complications include multisystem organ dysfunction and MI; mortality is up to 20% Why eliminated from differential: the patient's serum glucose level is lower than that expected for HHNC, the patient has significant urine ketones and a metabolic acidosis, and the patient's history is suggestive of an existing diagnosis of DM type 1 Hypoglycemia Abnormally low serum glucose that diffusely impairs normal tissue function May occur due to several processes related to insulin excess or an insufficient glucose supply (Table 5-6) History: weakness, diaphoresis, dizziness, palpitations, headache Physical examination: mental status changes, impaired consciousness Tests:
Decreased serum glucose with or without increased insulin Computed tomography (CT) or magnetic resonance imaging (MRI) may be useful for detecting tumors Low cortisol and either increased or decreased ACTH is indicative of a pituitary or adrenal cause Treatment: Prompt administration of glucose Guidance for proper eating habits and titration of insulin use can help avoid recurrences in some patients Patients with insulin-secreting tumors will frequently require surgical resection Patients with pituitary or adrenal pathology will require cortisol replacement Outcomes: easily treatable conditions have an excellent prognosis, but the prognosis for untreatable diseases is poor; untreated hypoglycemia leads to tissue death and mortality Why eliminated from differential: although hypoglycemia is a complication of insulin use, this diagnosis is ruled out by the patient's elevated glucose level Intoxication More thorough discussion in Chapter 13 Why eliminated from differential: the negative toxicology screen rules out this diagnosis Myocardial infarction More thorough discussion in Chapter 1 Why eliminated from differential: the relatively normal ECG in a young patient rules out this diagnosis P.147
Table 5-6 Causes of Hypoglycemia
Cause
Reactive
Pathology
Diagnosis
Treatment
Decrease in serum glucose after eating
Hypoglycemia and symptoms improve with
(e.g., postsurgical, idiopathic)
carbohydrate meal
Iatrogenic (e.g.,
Exc ess insulin administration or an adverse
Inc reased insulin in the presenc e of
Adjust insulin regimen,
excess insulin)
effect of sulfonylurea or meglitinide use
hypoglyc emia, adjustment of drug regimen
consider a different oral
improves symptoms
hypoglycemic drug
Increased insulin in presence of hypoglycemia,
Surgical resection if able to
may be detected on CT or MRI
locate
Underproduc tion of gluc ose due to
Lab abnormalities and history associated with
Proper nutrition, enzyme
hormone deficiencies, malnutrition, or liver
particular etiology
replacement
Glycogen depletion and gluc oneogenesis
History of alcohol use, serum ethanol >45
Proper nutrition, stopping
inhibition by high concentrations of alcohol
mg/dL
high quantity alcohol use
Insulinoma1
Fasting
β-islet c ell tumor producing excess insulin
Frequent small meals
disease
Alcohol-induced
Pituitary/adrenal
Decreased cortisol production leads to
Low serum cortisol; site of defect determined
insufficiency
insufficient hepatic gluconeogenesis in
by ACTH activity tests; possible other
response to hypoglycemia
comorbid endocrine abnormalities
Cortisol replacement
ACTH, adrenocorticotropic hormone; CT, computed tomography; MRI, magnetic resonance imaging. 1
A similar presentation would be expected for tumors with paraneoplastic production of insulin or insulinlike substance.
Case 5-4: “I can't stand the heat” A 38-year-old woman presents to her PCP because of restlessness and heat intolerance that has developed over the past 3 months. She says that she does not remember a specific start to these symptoms and that they have developed insidiously. The symptoms are present most of the day. She says that whenever she is in a warm room she becomes very uncomfortable and feels the need to cool down as quickly as possible. She describes herself as being frequently on edge but tired at the same time. She works as an insurance adjuster and feels that her performance at work has deteriorated recently because she has a difficult time sitting still and focusing on her work. She finds that she has been irritable toward her coworkers when she has unproductive days. She has had difficulty sleeping at night recently. She notices that she is also losing weight despite eating large meals but attributes this to the fact that she tends to move her bowels soon after eating. She denies any headaches, paresthesias, or any episodes of anxiety or depression previous to 3 months ago. She denies any past medical history and currently takes a daily vitamin and oral contraceptive pill. She drinks socially and denies illicit substance use. On examination, she is a thin woman who appears agitated. Her eyes appear to protrude slightly from her orbits, and her eyelids are retracted. Her skin is warm and slightly sweaty. She has no palpable lymphadenopathy, but a nontender mass is palpable on the front of her throat. Auscultation of her heart and lungs detects tachycardia and mild tachypnea but no abnormal sounds. Her abdomen is nontender with no palpable masses, and she has normal bowel sounds. The anterior surfaces of her shins have very dry skin and are swollen. She has hyperactive reflexes but no weakness or paresthesias. The following vital signs are measured: T: 99.3°F, HR: 105 bpm, BP: 130/86 mm Hg, RR: 22 breaths/min P.148
Differential Diagnosis Graves disease, toxic multinodular goiter, subacute thyroiditis, silent thyroiditis, Hashimoto thyroiditis, thyroid cancer, pheochromocytoma, anxiety disorder, cocaine intoxication, heat exhaustion
Laboratory Data and Other Study Results CBC: WBC: 7.9, Hgb: 15.4, Plt: 298 Chem10: Na: 141 mEq/L, K: 4.1 mEq/L, Cl: 103 mEq/L, CO2: 27 mEq/L, BUN: 12 mg/dL, Cr: 0.6 mg/dL, Glu: 89 mg/dL, Mg: 1.9 mg/dL, Ca: 10.1 mg/dL, Phos: 4.6 mg/dL Thyroid panel: TSH: 0.2 µU/mL, T4: 15.7 µg/dL, free T4 index: 18.3, T3: 4.0 ng/mL, T3 reuptake: 2.35 Thyroid scan: diffusely increased uptake in the region of the thyroid gland UA: straw colored, pH: 6.2, specific gravity: 1.010, no glucose/ketones/nitrites/leukocyte esterase/hematuria /proteinuria Urine toxicology screen: negative for alcohol or any illicit substances Based on these results, the following test is ordered: Thyroid-stimulating immunoglobulins (TSIs): positive
Diagnosis Graves disease
Treatment Administered The patient was started on propylthiouracil (PTU) until iodine therapy could be performed Prednisone and artificial tears were prescribed for the ophthalmologic abnormalities Radioactive iodine therapy was administered to the patient on an outpatient basis PTU was discontinued following the completion of iodine therapy
Follow-up The patient had resolution of her symptoms over the following 3 months The ophthalmologic symptoms resolved, and prednisone was discontinued The patient developed hypothyroidism and required thyroid hormone replacement therapy
Causes of hyperthyroidism and how they are differentiated with a thyroid scan may be remembered by the mnemonic “Lots (of) Thyroid Never Fails In Giving Anxiety”: Low thyroid scan uptake—Thyroiditis; Normal thyroid scan uptake—Factitious hyperthyroidism; Increased thyroid scan uptake—Graves disease, Adenomas.
Steps to the Diagnosis Graves disease Autoimmune condition in which TSIs bind to TSH receptors in the thyroid and stimulate excessive thyroid hormone production History: heat intolerance, anxiety, restlessness, irritability, diaphoresis, palpitations, increased bowel activity, weight loss, fatigue, vision abnormalities, dyspnea Physical examination: exophthalmos, lid lag, pretibial myxedema, tachycardia, tremor, warm skim, hyperreflexia, painless goiter, thyroid bruit, proximal muscle weakness (Color Figure 5-1) Tests: Decreased TSH; increased T4, free T4, T3, and T3 resin uptake Diffusely increased uptake of tracer on thyroid nuclear imaging (i.e., thyroid scan) Presence of TSIs in serum Biopsy of thyroid tissue demonstrates lymphocytic infiltrates and hypertrophy of the follicles with minimal colloid material P.149 Treatment: Thionamides (e.g., PTU, methimazole) may be used to control the concentration of thyroid hormones, but they are usually a bridge to more definitive therapy β-blockers may be used to control the cardiac effects of excess thyroid hormones until definitive therapy
can be administered Radioactive iodine therapy is used to gradually ablate the gland and is frequently the primary definitive therapy Thyroidectomy is performed in patients who do not respond to other therapies Corticosteroids and eye lubricants are prescribed to avoid visual complications until the resolution of eye symptoms Outcomes: Most patients respond well to therapy with a resolution of their hyperthyroidism Patients will typically require thyroid hormone replacement therapy following thyroid ablation or resection Thyroid storm Acute severe hyperthyroidism resulting from a significant release of thyroid hormones May be caused by stressful events (e.g., illness, pregnancy) or may be a rare reaction to changes in therapy Symptoms, signs, and lab findings are similar to typical hyperthyroidism but are more severe Treated with an aggressive combination of β-blockers, corticosteroids, thionamides, IV iodide, and appropriate definitive therapy Mortality is up to 50% Clues to the diagnosis: History: anxiety, restlessness, heat intolerance, insomnia, irritability, weight loss Physical: exophthalmos, lid retraction, diaphoresis, warm skin, tachycardia, painless goiter, tachypnea, pretibial myxedema, hyperreflexia Tests: increased thyroid hormones, decreased TSH, presence of TSIs, thyroid scan appearance Toxic multinodular goiter/toxic adenoma (a.k.a. Plummer disease) Presence of one or more thyroid nodules that are responsible for the production of excess thyroid hormones History: symptoms similar to those for Graves disease Physical examination: one or more palpable thyroid nodules, tachycardia, tremor, warm skim, hyperreflexia, painless goiter, thyroid bruit, proximal muscle weakness P.150 Tests: Thyroid panel will be similar to that for Graves disease Increased uptake of tracer on a thyroid scan at the site of the nodules Ultrasound (US) may be useful to localize nodules Treatment: Radioactive iodine is considered the primary therapy Thionamides and β-blockers may be used as temporary therapy until definitive treatment is performed
Surgical resection is performed for patients with large nodules causing compression of nearby structures Outcomes: prognosis is good with definitive treatment; hypothyroidism occurs at a lower rate than for Graves disease following radioactive iodine therapy Why eliminated from differential: the appearance of the thyroid scan in this case (diffuse uptake) is more consistent with Graves disease than this diagnosis Subacute thyroiditis (a.k.a. de Quervain thyroiditis) Self-limited hyperthyroidism due to a viral cause Diffuse enlargement of the thyroid gland occurs in response to the precipitating infection History: neck pain, similar symptoms as toxic multinodular goiter but typically milder in nature Physical examination: painful goiter, fever, tachycardia, tremor, warm skin, hyperreflexia, thyroid bruit, proximal muscle weakness Tests: Increased T4, free T4, T3, T3 resin uptake; TSH is typically extremely low Thyroid scan will show decreased uptake of tracer ESR and CRP are frequently increased Biopsies will show a significant infiltration of lymphocytes Treatment: The condition is self-limited Supportive care with hydration, nonsteroidal anti-inflamatory drugs (NSAIDs), and β-blockers are used to treat the symptoms Thyroid hormone replacement may be required if hypothyroidism occurs during recovery Outcomes: almost all patients recover fully and have an excellent prognosis; <5% of patients will continue to have some degree of thyroid dysfunction following recovery Why eliminated from differential: the thyroid scan appearance and painless goiter seen in this case rule out this diagnosis Silent thyroiditis Temporary hyperthyroidism that has an autoimmune etiology Incitation of the condition may be related to pregnancy or certain medications (e.g., amiodarone, lithium) History: similar symptoms to toxic multinodular goiter but milder in nature Physical examination: painless goiter, tachycardia, tremor, warm skin, hyperreflexia, thyroid bruit, proximal muscle weakness Tests: Decreased TSH; increased T4, free T4, T3, T3 resin uptake Thyroid scan shows low uptake Biopsies will show an infiltration of lymphocytes
Treatment: self-limited; NSAIDs and β-blockers are used to treat symptoms Outcomes: pregnancy-related cases tend to have relapses following future pregnancies and carry a higher risk of long-term thyroid dysfunction; otherwise, cases typically have an excellent prognosis Why eliminated from differential: the thyroid scan appearance (increased uptake) rules out this diagnosis Hashimoto thyroiditis More thorough discussion in later case Why eliminated from differential: the decreased TSH, increased thyroid hormones, and increased uptake on the thyroid scan rule out this diagnosis Thyroid cancer Malignant nodules of the thyroid that may arise from columnar epithelial, follicular, or parafollicular cells (Table 5-7) The majority of thyroid nodules are not malignant in nature and become more common with increasing age Risk factors: female gender, aged 20 to 60 years, history of neck irradiation, poor iodine uptake on a thyroid scan, solid nodules History: asymptomatic or possible dysphagia and hoarseness Physical examination: solitary nontender anterior neck mass, cervical lymphadenopathy Tests: Thyroid hormones may be increased or decreased and are not a reliable indicator of the diagnosis Malignant nodules are more likely to show decreased tracer uptake (i.e., cold nodules) than increased tracer uptake (i.e., hot nodules) (Figure 5-7) US may be used to localize nodules and guide biopsies Biopsy via fine needle aspiration will demonstrate malignant transformation of cells Treatment: Benign nodules may be observed for any changes, and those associated with hyperthyroidism or hypothyroidism are treated accordingly P.151
Table 5-7 Types of Thyroid Carcinoma
Type
Cells A ffec ted
Papillary
Columnar cells of
Most c ommon form (80%
Begins as slow-growing nodule;
gland
cases); more common in
eventually metastasizes to local
younger patients
cervical lymph nodes
Cuboid cells in
10% of thyroid cancers;
May function like normal thyroid
Worse than papillary
follicles
more common in older
tissue; metastasizes to liver, lung,
cancer; 50% 10-year
patients
bone, and brain
survival
Follicular
Frequenc y
Charac teristic s
Prognosis
Good; few recurrences
Medullary
Parafollicular C
4% of thyroid cancers
cells
Anaplastic
Poorly
1% of thyroid cancers
differentiated
Produc es c alc itonin; may
Worse in older patients;
present with other endocrine
metastases common at
tumors (e.g., MEN2a and MEN2b)
diagnosis
V ery aggressive; local extension
Poor
causes hoarseness, dysphagia
neoplasm
MEN, multiple endocrine neoplasia.
Lobectomy is performed for nonanaplastic tumors that are <1.0 cm; total thyroidectomy is performed for larger tumors Radioactive iodine therapy frequently accompanies surgical resection Radiation therapy is used for tumors with local extension Chemotherapy is used for metastatic tumors Thyroid hormone replacement may be required following resection Outcomes: prognosis depends on the type of tumor encountered but is best in papillary cancers Why eliminated from differential: the diffuse nature of the disease in the case (per examination and the thyroid scan) makes this diagnosis unlikely
Figure 5-7 Thyroid scan in a patient with suspected thyroid carcinoma. Note that in the patient's right lobe (on the left side of the figure) there is a region of decreased uptake, suggesting a “cold nodule.”
Pheochromocytoma More thorough discussion in Chapter 1 Why eliminated from differential: the continuous nature of the patients symptoms and the multiple tests suggesting a thyroid pathology make this diagnosis very unlikely Anxiety disorder More thorough discussion in Chapter 13 Why eliminated from differential: the multiple findings suggesting a thyroid pathology make this diagnosis unlikely; if the patient's symptoms of anxiety were to continue following treatment for Graves disease, a psychiatric work-up would be appropriate Cocaine intoxication More thorough discussion in Chapter 13 Why eliminated from differential: the negative toxicology screen rules out this diagnosis Heat exhaustion Acute dysfunction of the body's thermoregulatory system, leading to the onset of dehydration and electrolyte abnormalities (Table 5-8) It occurs when the body's attempts to maintain a normal body temperature become inadequate to prevent hyperthermia (e.g, extremely hot environment, prolonged strenuous activity) History: fatigue, weakness, headache, nausea, vomiting, irritability, significant diaphoresis Physical examination: increased body temperature (below 106°F), tachycardia Tests: typically noncontributory Treatment: rehydration, removal from a hot environment, and rest are the primary therapies Outcomes: the prognosis is excellent with the proper treatment; progression to heat stroke is associated with worse outcomes Why eliminated from differential: although several of the symptoms for this diagnosis are present in this case, the chronology of the patient's complaints and the presence of findings suggestive of a thyroid pathology make this diagnosis unlikely
Table 5-8 Types of Heat Emergencies
Heat Exhaustion
Cause
Thermoregulatory dysfunction
Heat Stroke
Thermoregulatory dysfunction
Malignant Hyperthermia
Genetic condition in which exposure to inhaled anesthetics causes uncontrolled muscle oxidative metabolism
Symptoms/signs
Fatigue, weakness, headache, nausea,
Similar to heat exhaustion plus mental
vomiting, irritability, diaphoresis,
status changes, hallucinations, and late
tachycardia
onset of anhydrosis
Muscular rigidity, tachycardia, cyanosis
Body
Mildly elevated, <106°F
Frequently above 106°F
>104°F and may increase by 4°F per hour
Noncontributory
Increased LFTs, signs of DIC, increased
A muscle biopsy will contract in vitro
creatine kinase, hematuria, proteinuria
following exposure to caffeine or halothane
temperature
Tests
in affected individuals
Treatment
Rehydration, rest, cooling
ABCs, aggressive cooling measures (e.g.,
Aggressive cooling measures, dantrolene,
evaporative cooling, ice baths, cooled
immediate cessation of the offending agent
fluids)
Complications
Progression to heat stroke
Cardiac ischemia, pulmonary edema,
Similar to heat stroke
rhabdomyolysis, ARF, DIC, cerebral vascular accident
ABCs, airway, breathing, circulation; ARF, acute renal failure; DIC, disseminated intravascular coagulopathy; LFTs, liver function tests.
A complication of thyroid surgery is hoarseness due to damage of the recurrent laryngeal nerve. P.153
Case 5-5: “What is this lump on my neck?” A 38-year-old woman presents to an endocrinologist for the work-up of a painless anterior neck mass. She says that she first noticed the mass a few weeks ago and is unsure how long it has been present. She denies any neck pain, anxiety, restlessness, heat intolerance, palpitations, or weight loss. When asked about these symptoms, she says that she feels quite the opposite. She frequently feels tired, has little energy, cannot seem to lose any weight, generally feels down, and needs to wear several layers of clothing because she is always cold. She previously saw this endocrinologist because of a brief episode of hyperthyroidism that resolved by itself and was ruled to be a case of silent thyroiditis. She has a history of depression, constipation, and menstrual irregularity for which she takes fluoxetine, docusate sodium, and an oral contraceptive pill. She denies any substance use. On examination, she is an overweight woman who appears to be in no distress. She has generally dry skin and thinning of her cranial hair. There is mild swelling of her face. She has no lymphadenopathy. Fundoscopic examination is normal. She has a mass on her anterior neck in the region of the thyroid gland with several indistinct nodules. There is no audible bruit over this mass. Auscultation of her heart and lungs detects clear lung sounds and bradycardia. Her abdomen is nontender with no detectable masses. Her motor function is slightly weakened diffusely (4+/5), but her sensation is normal. Her reflexes are slow and difficult to elicit. The following vital signs are measured: T: 98.3°F, HR: 54 bpm, BP: 118/87 mm Hg, RR: 13 breaths/min
Differential Diagnosis Hypothyroidism, toxic multinodular goiter, subacute thyroiditis, silent thyroiditis, Graves disease, hypopituitarism, acute renal failure, nephrotic syndrome, depression
Laboratory Data and Other Study Results CBC: WBC: 5.3, Hgb: 12.1, Plt: 328 Chem7: Na: 133 mEq/L, K: 3.9 mEq/L, Cl: 102 mEq/L, CO2: 28 mEq/L, BUN: 11 mg/dL, Cr: 0.6 mg/dL, Glu: 96 mg/dL
Thyroid panel: TSH: 8.4 µU/mL, T4: 2.1 µg/dL, free T4 index: 1.9, T3: 1.1 ng/mL, T3 reuptake: 0.95 Thyroid scan: heterogenous decreased uptake of tracer in the thyroid UA: straw colored, pH: 7.1, specific gravity: 1.010, no glucose/ketones/nitrites/leukocyte esterase/hematuria /proteinuria Based on these results, the following tests are performed: US-guided thyroid biopsy: diffuse lymphocytic infiltration, follicular hyperplasia, parenchymal atrophy Antithyroglobulin antibodies: positive Antithyroid peroxidase antibodies: positive
Diagnosis Hashimoto thyroiditis with associated hypothyroidism
Treatment Administered The patient was started on levothyroxine for thyroid hormone replacement therapy
Follow-up The patient had improvement in her physical and mood symptoms following the start of treatment P.154 Subsequent measurements of the patient's thyroid panel demonstrated normalization of her thyroid hormones and TSH The patient was able to be weaned off of fluoxetine and docusate sodium following an improvement of her symptoms
Hashimoto thyroiditis is the most common cause of hypothyroidism in the United States, but iodine deficiency is the most common cause worldwide.
Symptoms of hyperthyroidism may be seen in early Hashimoto thyroiditis.
Steps to the Diagnosis Hashimoto thyroiditis Chronic thyroiditis due to autoimmune destruction of the gland that results in hypothyroidism Thyroid surgery, radioactive iodine ablation, pituitary dysfunction, and medications (e.g., lithium) are other potential causes of hypothyroidism History: fatigue, weakness, cold intolerance, weight gain, constipation, menstrual irregularity, depression, hoarseness, memory loss, hair loss, somnolence Physical examination: bradycardia, hyporeflexia, dry skin, facial and extremity edema, painless goiter, brittle nails Tests: Increased TSH; decreased T4 and free T4; T3 and T3 uptake tend to remain normal
Antithyroglobulin and antithyroid peroxidase antibodies are typically detected Thyroid scan shows decreased tracer uptake Thyroid biopsy will show lymphocytic infiltration, follicular hyperplasia, parenchymal atrophy, and damage to the basement membrane of the follicles Treatment: lifelong thyroid hormone replacement is required Outcomes: complications include osteoporosis, cardiomyopathy, and myxedema coma from significantly low thyroid hormone levels; the prognosis is excellent in patients who are correctly diagnosed and are placed on thyroid replacement therapy Clues to the diagnosis: History: fatigue, weakness, depression, cold intolerance, constipation, menstrual irregularity Physical: painless goiter, dry skin, facial edema, bradycardia, hyporeflexia Tests: increased TSH, decreased T4 and free T4, thyroid scan results, biopsy results, positive antibody screens Graves disease, toxic multinodular goiter More thorough discussion in prior case Why eliminated from differential: the thyroid panel results and thyroid scan appearance are opposite to what would be expected for these conditions, so they are ruled out Subacute and silent thyroiditis More thorough discussion in prior case Why eliminated from differential: these conditions do feature decreased uptake on thyroid scans, but since their thyroid panels would show hyperthyroidism and not hypothyroidism, they are ruled out Hypopituitarism More thorough discussion in later case Why eliminated from differential: hypothyroidism does occur in this condition, but the increased TSH level shows that the pituitary is functioning and rules out this diagnosis Acute renal failure (ARF), nephrotic syndrome More thorough discussion in Chapter 4 Why eliminated from differential: the normal BUN, creatinine, and UA rule out these diagnoses Depression More thorough discussion in Chapter 13 Why eliminated from differential: this condition may still exist in this patient, but given the evidence of hypothyroidism, a psychiatric evaluation should only be considered if the patient's mood symptoms do not improve after the start of thyroid replacement P.155
If decreased TSH and hypothyroidism are seen, suspect a pituitary or hypothalamic etiology.
Case 5-6: “My lips and fingers keep tingling” A 56-year-old man is a surgical inpatient after a successful total laryngectomy and radical neck dissection performed for a diagnosis of laryngeal cancer. The patient tolerated the procedure 5 days ago relatively well and has been a stable inpatient in the regular surgical wing for the past few days. He is unable to speak given his recent procedure but is writing on a pad to communicate with the surgical team. During morning rounds, he notifies the team that he is having tingling in his lips and fingers since last night. He denies any paresthesias or weakness in other locations of his body. He has never had this sensation previously. He denies headaches, syncope, or chest pain. He has had some postsurgical pain that has been well controlled with analgesics. He has mild abdominal pain but has eaten minimally since his surgery. Besides laryngeal cancer, he has a history of HTN and hyperlipidemia for which he takes irbesartan, simvastatin, and ASA. He was a heavy smoker for several years but quit once he was diagnosed with laryngeal cancer. On examination, he is a thin man who is mildly anxious-appearing. He has an incision over his anterior throat that is clean, dry, and intact. There is one surgical drain inserted in his anterior neck that has been dry for the past 24 hours. He has no palpable fluid collections around the region of his incision. Auscultation of his chest and lungs detects minimal pulmonary congestion and an extra heart beat. He has mild weakness of his extremities (4/5), but his sensation is grossly intact despite his description of tingling. Tapping of his face near his mouth elicits spasm of his facial muscles and is painful. The following vital signs are measured: T: 98.7°F, HR: 92 bpm, BP: 120/78 mm Hg, RR: 18 breaths/min
Differential Diagnosis Anxiety, trigeminal neuralgia, cerebral vascular accident, hypoparathyroidism, pseudohypoparathyroidism, acute renal failure Laboratory Data and Other Study Results CBC: WBC: 10.3, Hgb: 13.9, Plt: 396 Chem10: Na: 139 mEq/L, K: 3.7 mEq/L, Cl: 107 mEq/L, CO2: 24 mEq/L, BUN: 17 mg/dL, Cr: 0.9 mg/dL, Glu: 92 mg/dL, Mg: 2.0 mg/dL, Ca: 7.2 mg/dL, Phos: 6.1 mg/dL PTH: <1 pg/mL Albumin: 3.9 g/dL Head CT: no hemorrhage or masses; normal parenchymal and ventricular sizes
Diagnosis Hypoparathyroidism with hypocalcemia (complication of the recent neck dissection)
Treatment Administered Supplemental calcium and vitamin D were prescribed
Follow-up The patient had an improvement in the tingling and abdominal pain The patient did not regain any parathyroid function and required long-term calcium and vitamin D supplementation
Steps to the Diagnosis Hypoparathyroidism Deficiency of PTH secretion by the parathyroid glands leading to hypocalcemia
Surgical resection (intended for parathyroid lesions or unintentional during neck surgery) is the most common cause Autoimmune degeneration, congenital dysfunction, hemochromatosis, and Wilson disease are other less common causes History: tingling in the lips and digits, weakness, abdominal pain, dyspnea Physical examination: dry skin, tetany, Trousseau sign (i.e., inflation of a blood pressure cuff causes carpal spasm), Chvostek sign (i.e., tapping of the facial nerve causes spasm and pain), tachycardia, movement disorders, possible cataracts, possible dental hypoplasia Tests: Decreased PTH and calcium; increased phosphate X-rays may demonstrate sclerotic changes of bone in chronic disease Treatment: calcium and vitamin D supplementation Outcomes: Prognosis is good with adequate calcium supplementation Nephrolithiasis occurs more commonly in these patients Patients with significant PTH deficiency may develop fatal cardiac arrhythmias (i.e., long QT disease) without calcium supplementation Clues to the diagnosis: History: tingling in lips and hands, abdominal pain, recent radical neck dissection Physical: Chvostek sign, motor weakness Tests: decreased PTH and calcium, increased phosphate Pseudohypoparathyroidism Hypocalcemia that results from tissue unresponsiveness to PTH Typically occurs as part of the syndrome of Albright hereditary osteodystrophy History: tingling of lips and digits, muscle cramping, weakness, abdominal pain, dyspnea, palpitations, seizures Physical examination: short stature, mental impairments, tetany, Chvostek sign, Trousseau sign Tests: increased PTH and phosphate; decreased calcium Treatment: calcium and vitamin D supplementation Outcomes: prognosis is good with adequate calcium supplementation; nephrolithiasis and cardiac arrhythmias are the chief complications Why eliminated from differential: the decreased PTH level in this case rules out this diagnosis Anxiety More thorough discussion in Chapter 13 Why eliminated from differential: anxiety does not explain the abnormal PTH and calcium levels in this patient, so this diagnosis is unlikely
Trigeminal neuralgia More thorough discussion in Chapter 7 Why eliminated from differential: Chvostek-like signs may be present in both diagnoses, but trigeminal neuralgia does not explain the abnormal lab findings Acute renal failure More thorough discussion in Chapter 4 Why eliminated from differential: the normal BUN and creatinine levels rule out this diagnosis Cerebral vascular accident More thorough discussion in Chapter 7 Why eliminated from differential: sensory phenomena are possible sequelae of a stroke, but a stroke does not explain the lab abnormalities seen in this case
Case 5-7: “My periods never came back” A 32-year-old woman presents to her family practitioner with chronic amenorrhea. She gave birth to a healthy male 1 year ago by vaginal delivery. Her pregnancy was complicated by a significant uterine hemorrhage from retained placental tissue, and she required a dilation and curettage to remove the retained tissue and control her bleeding. She says that she has felt fatigued since her delivery. She was not able to produce sufficient breast milk, so she has fed her infant formula. Despite not breastfeeding, she never resumed menses following her delivery. She feels that her libido has never recovered since the birth of her son. She notes that she feels cold all of the time and wears multiple layers of clothing. More recently, she has begun to lose weight unintentionally and has had no appetite. She says that she never felt this way prior to the delivery of this child. She denies any recent illnesses, sites of pain, dizziness, or paresthesias. She has been pregnant one time (the pregnancy described above). She denies any past medical history and takes no medications. She denies any substance use but drank socially prior to becoming pregnant. On examination, she is a thin, tired-appearing woman in no acute distress. She is notable for dry skin and mild edema of her face and extremities. She has no lymphadenopathy. Auscultation of her heart and lungs finds no abnormal sounds besides mild bradycardia. Her abdomen is nontender, and no masses are detected. A pelvic examination detects mild vaginal atrophy and sparse pubic hair but no tenderness, masses, bleeding, or purulent fluid. Her neurologic examination is notable for hyporeflexia, mild motor weakness (4+/5), and normal sensation. The following vital signs are measured: T: 98.4°F, HR: 60 bpm, BP: 105/70 mm Hg, RR: 15 breaths/min
Differential Diagnosis Hypothyroidism, hypopituitarism, anorexia nervosa, cirrhosis, systemic lupus erythematosus, chronic kidney disease, Addison disease, polycystic ovarian syndrome, premature ovarian failure, pregnancy
Laboratory Data and Other Study Results CBC: WBC: 6.3, Hgb: 12.4, Plt: 278 Chem10: Na: 141 mEq/L, K: 4.0 mEq/L, Cl: 106 mEq/L, CO2: 25 mEq/L, BUN: 16 mg/dL, Cr: 0.8 mg/dL, Glu: 72 mg/dL, Mg: 2.0 mg/dL, Ca: 9.9 mg/dL, Phos: 4.3 mg/dL Liver function tests (LFTs): alkaline phosphate (AlkPhos): 43 U/L, alanine aminotransferase (ALT) 31 U/L, aspartate aminotransferase (AST) 26 U/L, total bilirubin (TBili): 0.8 mg/dL, Direct bilirubin (DBili): 0.4 mg/dL Thyroid panel: TSH: 0.3 µU/mL, T4: 2.1 µg/dL, free T4 index: 2.9, T3: 0.4 ng/mL, T3 reuptake: 0.49
Antinuclear antibodies (ANA): negative Cortisol (afternoon): 4.0 µg/dL LH: 25 U/L FSH: 22 U/L Estradiol: 5 pg/mL Urine pregnancy test: negative Based on these results, the following studies are ordered: ACTH (morning): 19 pg/mL Medroxyprogesterone stimulation test: no vaginal bleeding after 10 days of administration Metyrapone stimulation test: no increase in ACTH level after administration over 24 hours Brain MRI: decreased signal in sella turcica; T1 image shows apparent reduction in pituitary size
Diagnosis Partial hypopituitarism secondary to Sheehan syndrome (i.e., significant deficits of all anterior pituitary hormones)
Treatment Administered The patient was referred to an endocrinologist to direct further therapy She was started on multihormone replacement therapy consisting of an estrogen-progesterone pill, levothyroxine, and hydrocortisone
Follow-up The patient had significant improvement in her symptoms following the institution of treatment but required lifelong hormone replacement
The order in which symptoms of hormone deficiencies develop in panhypopituitarism may be remembered by the mnemonic “Good Luck Finding Treatment (for) Pituitary Accidents”: GH→LH→FSH→TSH→Prolactin→ACTH.
Steps to the Diagnosis Hypopituitarism Deficiency of multiple pituitary hormones, resulting in a corresponding dysfunction in the target glands May be due to tumor, hemorrhagic infarction (i.e., pituitary apoplexy), surgical resection, trauma, sarcoidosis, tuberculosis, postpartum necrosis (i.e., Sheehan syndrome), or hypothalamic dysfunction Because some pituitary hormones are kept in a reserve supply and because the target organs may maintain some autonomous function, the timing of symptoms varies with the deficient hormone (Table 5-9) History: the symptoms depend on the deficient hormone (Table 5-9) Physical examination: the symptoms depend on the deficient hormone (Table 5-9) Tests:
Blood lab abnormalities are indicative of the deficient hormone GH: decreased GH; no increase in GH after administration of insulin LH and FSH: decreased FSH, LH and estradiol (women) or testosterone (men); in the medroxyprogesterone stimulation test, vaginal bleeding will not occur after 10 mg of medroxyprogesterone is administered for 10 days if gonadotropins are deficient (normal response would be the induction of bleeding) TSH: decreased TSH, T4, free T4, T3, and T3 uptake Prolactin: decreased prolactin in the postpartum period
Table 5-9 Progression of Hormone Deficiency in Hypopituitarism
Order of Loss
1
Hormone(s)
GH
History
Growth failure in children; fatigue, depression in adults
Physic al Examination
Short stature in children; decreased muscle mass in adults
2
LH, FSH
3
TSH
Infertility and decreased libido in both sexes; amenorrhea women;
Decreased pubic hair; vaginal atrophy in women;
impotence in men
testicular atrophy in men
Fatigue, cold intolerance, weakness, weight gain, constipation,
No goiter, hyporeflexia, bradycardia, dry skin,
irregular menstruation, depression
edema
4
Prolactin
No postpartum lactation
None
5
ACTH,
Adrenal insufficiency leading to fatigue, weight loss, decreased
Hypotension, tachycardia; decreased skin
MSH
appetite, and poor response to stress
pigmentation due to low MSH
ACTH, adrenocorticotropic hormone; FSH, follicle-stimulating hormone; GH, growth hormone; LH, luteinizing hormone; MSH, melanocyte-stimulating hormone; TSH, thyroid stimulating hormone.
ACTH: decreased ACTH; cortisol does not increase after insulin administration (normal is an increase to >18 µg/mL); in the metyrapone stimulation test ACTH and 11-deoxycortisol do not increase following administration (normal is an increase in both) Treatment: GH: recombinant growth hormone is given to children to allow normal growth, but adults typically do not require treatment unless significantly symptomatic LH/FSH: estrogen and progesterone are given to women, and testosterone is given to men; patients desiring fertility may be given gonadotropin-releasing hormone TSH: levothyroxine is given as thyroid hormone replacement Prolactin: not treated ACTH: hydrocortisone, dexamethasone, or prednisone is required Outcomes: if adequate hormone replacement is provided, the prognosis is good; development of severe deficiencies of hormones may be fatal
Clues to the diagnosis: History: history of postpartum hemorrhage, fatigue, insufficient lactation, amenorrhea, reduced libido, cold intolerance, weight loss, anorexia Physical: dry skin, edema, bradycardia, vaginal atrophy, loss of pubic hair, hyporeflexia, motor weakness Tests: decreased cortisol, FSH, LH, estradiol, thyroid hormones, TSH, and ACTH, results of the metyrapone and medroxyprogesterone stimulation tests, brain MRI findings Hypothyroidism More thorough discussion in prior case Why eliminated from differential: there is definitely an element of hypothyroidism in this case, but hypothyroidism alone cannot account for the other hormone deficiencies Addison disease More thorough discussion in later case Why eliminated from differential: cortisol deficiency is seen in both cases, but the normal electrolyte levels suggest maintained aldosterone activity, and complete adrenal dysfunction is ruled out Polycystic ovarian syndrome More thorough discussion in Chapter 11 Why eliminated from differential: amenorrhea occurs in both diagnoses, but the decrease in all gonadotropin levels rules out this diagnosis Premature ovarian failure More thorough discussion in Chapter 11 Why eliminated from differential: the LH and FSH levels are significantly lower than would be expected for the early stages of this condition, so it is unlikely the diagnosis Pregnancy More thorough discussion in Chapter 12 Why eliminated from differential: the negative pregnancy test rules out this diagnosis Anorexia nervosa More thorough discussion in Chapter 13 Why eliminated from differential: although this diagnosis may account for amenorrhea and several of the clinical symptoms, it cannot explain the widespread deficiencies of multiple hormones Systemic lupus erythematosus More thorough discussion in Chapter 9 Why eliminated from differential: the negative ANA makes this diagnosis unlikely; systemic lupus erythematosus (SLE) cannot easily explain all of the hormone deficiencies Chronic kidney disease More thorough discussion in Chapter 4
Why eliminated from differential: the normal electrolytes, BUN, and creatinine rule out this diagnosis
Case 5-8: “I'm starting to look like a hunchback” A 28-year-old woman presents to her PCP because she is very concerned with several changes in her appearance over the past 6 months. The patient says that she was in good health until a few months ago but has gradually developed several unusual body characteristics since that time. She does not recall any inciting event for these changes and denies that she had ever noticed them previously. She says that she has gradually developed a hump on her back, and her face feels swollen. She has also gained a significant amount of weight and that she is always thirsty. She notes that she has had a flare of significant acne on her face, back, and torso over the past several months. She is very concerned that she has been growing facial hair. She feels tired and has become rather depressed about her appearance. She notes that she has had irregular periods for the past 4 months. She says that she had a history of irritable bowel syndrome when she was younger that responded well to changes in her eating habits. She denies any current medication use. She formerly exercised several times per week but has been unable to do so for the past few months because of fatigue. She says that she drinks infrequently and denies illicit substance use. On examination, she is an overweight woman in no acute distress. She has a rounded face with notable acne and sparse facial hair. Her scalp hair is thin. She has a fatty humplike formation on her back. She has smaller fatty collections above her clavicles. Auscultation of her heart and lungs detects no abnormal sounds. Her abdomen is soft with several purple striae. She has no palpable masses. A pelvic examination detects no abnormal cervical or ovarian lesions. Neurologic examination is normal. The following vital signs are measured: T: 98.6°F, HR: 88 bpm, BP: 140/87 mm Hg, RR: 18 breaths/min
Differential Diagnosis Polycystic ovarian disease, hypothyroidism, Cushing syndrome, acromegaly, congenital adrenal hyperplasia, multiple endocrine neoplasia type 1
Laboratory Data and Other Study Results CBC: WBC: 12.1, Hgb: 13.1, Plt: 327 Chem10: Na: 144 mEq/L, K: 3.2 mEq/L, Cl: 101 mEq/L, CO2: 26 mEq/L, BUN: 17 mg/dL, Cr: 0.7 mg/dL, Glu: 137 mg/dL, Mg: 2.1 mg/dL, Ca: 10.0 mg/dL, Phos: 3.6 mg/dL LH: 20.7 U/L FSH: 12.3 U/L Thyroid panel: TSH: 1.3 µU/mL, T4: 7.6 µg/dL, free T4 index: 8.1, T3: 0.9 ng/mL, T3 reuptake: 0.89 GH: 3 ng/mL Low-dose dexamethasone suppression test: cortisol: 41.5 µg/dL the morning after low-dose (1 mg) dexamethasone administration (insufficient suppression) Based on these results, the following test is ordered: Twenty-four-hour urine cortisol: 628 µg/day The following tests are then ordered: Cortisol (morning): 48 µg/dL High-dose dexamethasone suppression test: cortisol: 34.5 µg/dL the morning after high-dose (8 mg) dexamethasone administration (insufficient suppression) ACTH (morning): 12 pg/mL
Abdominal CT: normal adnexal, bowel, pancreatic, hepatic, and splenic appearance; 3 cm mass on superior pole of right kidney
Diagnosis Cushing disease secondary to an adrenal tumor
Treatment Administered The patient was referred to a general surgeon to schedule surgical resection of the tumor Unilateral laparoscopic adrenalectomy was performed to fully resect the adrenal tumor The patient was placed on hydrocortisone at the time of surgery with a prescribed taper over 2 weeks
Follow-up The patient tolerated the surgery and hydrocortisone taper well Operative pathology determined the tumor to be an adrenal adenoma Cortisol, ACTH, and glucose were found to normalize over the month following the surgical resection The patient had a slow resolution of her physical symptoms lasting >1 year Further follow-up testing demonstrated maintained normal cortisol and ACTH levels
Excess corticosteroid administration is the most common cause of Cushing syndrome; excess ACTH production by a pituitary adenoma is the second most common cause.
Steps to the Diagnosis Cushing syndrome A syndrome of excess cortisol levels due to excess corticosteroid administration, pituitary adenoma (i.e., Cushing disease), adrenal tumor, or paraneoplastic ACTH production History: weakness, depression, menstrual irregularities, polydipsia, polyuria, increased libido, impotence, weight gain, possible headaches, possible fractures Physical examination: HTN, acne, hirsutism, central obesity, “buffalo hump” (i.e., fatty hunchbacklike hump on back), “moon faces” (i.e., rounded face due to fat deposition), superclavicular fat pads, purple abdominal striae, easy bruising, proximal muscle motor weakness, testicular atrophy (Color Figure 5-2)
Color Figure 5-2 Diagram of the common physical characteristics seen in Cushing syndrome.
Figure 5-8 Diagnostic algorithm for a patient with suspected Cushing syndrome due to cortisol excess. ACTH, adrenocorticotropic hormone.
Tests: Increased WBCs and glucose; decreased potassium Due to the time-dependence of serum cortisol levels, a low-dose dexamethasone suppression test is frequently the first test given to patients with multiple symptoms of cortisol excess (Figure 5-8) A 1-mg dose of dexamethasone is given to the patient at bedtime, and a morning serum cortisol level is measured on the following day The postdose morning cortisol should be <1.8 µg/dL, and elevated levels suggest an endogenous source of either excess ACTH or cortisol secretion A 24-hour urine cortisol may be used to differentiate an exogenous from an endogenous cause of excess cortisol A high-dose dexamethasone suppression test is used to differentiate pituitary from nonpituitary sources of excess ACTH or cortisol An 8-mg dose of dexamethasone is given to the patient at bedtime, and a morning serum cortisol level is measured
on the following day If the follow-up cortisol level decreases to <50% of the baseline serum cortisol, a pituitary lesion is likely If the follow-up cortisol level is >50% of the baseline level, a nonpituitary source is likely An ACTH level can help differentiate ACTH-secreting tumors from primary adrenal tumors Chest and abdominal CT and brain MRI are useful for the detection of a tumor once a causative diagnosis has been suggested by lab analysis Treatment: Titration of the prescribed dose of corticosteroids can help resolve the condition when it is due to excess exogenous corticosteroid exposure Surgical resection or radiation therapy may be performed for pituitary tumors Adrenalectomy is performed for primary adrenal tumors Surgical resection of ectopic sources of ACTH may be performed once the source has been localized Corticosteroids should be administered at the time of any surgical procedure and gradually tapered to avoid the effects of sudden withdrawal Mitotane, ketoconazole, and metyrapone may be used to suppress steroid synthesis in patients with unresectable disease or malignancies with a causative paraneoplastic syndrome Cortisol replacement may be required in patients who are unable to produce sufficient cortisol following therapy Outcomes: patients who are able to undergo a curative procedure typically have a good prognosis; possible complications include osteoporosis, infection susceptibility, DM, avascular necrosis of bone, hypopituitarism, adrenal crisis, and increased mortality from cardiovascular or thromboembolic disease Clues to the diagnosis: History: weight gain, polydipsia, fatigue, depression, menstrual irregularity Physical: buffalo hump, moon faces, acne, hirsutism, hair loss, supraclavicular fat pads, abdominal striae Tests: increased WBCs and glucose, positive low-dose and high-dose dexamethasone suppression tests, increased urine cortisol, increased ACTH, CT findings Hypothyroidism More thorough discussion in prior case Why eliminated from differential: some of the findings in this case may be seen in this diagnosis, but it is ruled out by the normal thyroid panel Polycystic ovarian disease More thorough discussion in Chapter 11 Why eliminated from differential: hirsutism and menstrual irregularity are seen with this diagnosis, but the normal LH and FSH levels and the normal appearance of the ovaries on the CT rule it out Acromegaly Excess secretion of growth hormone by a pituitary adenoma History: arthralgias, headache, neural pain (secondary to nerve entrapments), vision changes, excessive
growth for age in children Physical examination: enlargement of the hands and feet, coarsening of facial features (e.g., enlarged jaw, nose, and skin folds), thickened skin, increased body hair growth, hirsutism, oily skin Tests: Increased glucose, phosphate, calcium, and triglycerides Serum GH levels are increased but are unreliable for making a diagnosis when considered alone Failure of the GH level to decrease with glucose administration X-rays may show increased bone density Brain MRI may be used to detect pituitary tumors Treatment: Surgical resection of the adenoma should be performed if possible Dopamine agonists (e.g., bromocriptine) or octreotide may be used to lessen the effects of GH Radiation therapy may be useful in cases unresponsive to other treatments Outcomes: sixty-five percent to 85% of patients will be cured by surgical resection; complications include hypopituitarism, heart failure, DM, spinal cord compression, and vision loss Why eliminated from differential: the patient's growth hormone is in the normal range, and this diagnosis would not explain the excess cortisol levels (although decreased cortisol levels would not rule out this diagnosis) Congenital adrenal hyperplasia (CAH) Enzymatic defects in the synthesis of cortisol that result in decreased cortisol, a reactive increase in ACTH secretion, adrenal hyperplasia, and androgen excess (Figure 5-9)
Figure 5-9 Steroid hormone synthesis and the causes of congenital adrenal hyperplasia. Note the blocks in the pathway from 17-α-hydroxylase deficiency (triangles), 21-α-hydroxylase deficiency (squares), and 11-β-hydroxylase deficiency (circles). 3β, 3-β-hydroxysteroid dehydrogenase; 17α, 17-α-hydroxylase; 21α, 21-α-hydroxylase; 11β, 11-β-hydroxylase; DHEA, dehydroepiandrosterone; NADPH, nicotinamide adenine dinucleotide phosphate (reduced form).
Table 5-10 Enzyme Deficiencies in Congenital Adrenal Hyperplasia
Enzyme Defic ienc y
21-α-hydroxylase
Defic ient Exc ess Hormones
Androstenedione
Hormones
Cortisol, aldosterone
Clinic al Findings
Dehydration, hypotension Men—failure to thrive, macrogenitosomia, precocious
Tests
Increased androgens, dec reased sodium, inc reased
Treatment
Cortisol, fludrocortisone Reconstructive genital surgery
potassium
puberty Women— virilization, hirsutism, menstrual irregularity, possible ambiguous genitalia
17-α-hydroxylase
Deoxycorticosterone
Cortisol, androgens, estrogen
HTN Men—ambiguous genitalia Women—
Increased sodium, decreased potassium,
Cortisol, estrogenprogesterone replacement in
amenorrhea, poor
dec reased
females
breast
androgens
Most children are raised as
development
females Males may have reconstructive genital surgery
11-β-hydroxylase
Deoxyc ortic osterone,
Cortisol,
deoxycortisol,
aldosterone
androgens
HTN Men—precocious puberty, macrogenitosomia Women— virilization,
Increased
Cortisol,
sodium,
fludrocortisone,
decreased
antihypertensive
potassium,
therapy
inc reased androgens
ambiguous genitalia
HTN, hypertension.
Table 5-11 Types of Multiple Endocrine Neoplasia
Endoc rine Ty pe
1
Involvement
Charac teristic s
Treatment
Parathyroid
Hyperparathyroidism, hypercalcemia, possible Zollinger-
Subtotal parathyroidectomy, surgical
Pancreas (islet cell
Ellison syndrome, various pituitary disorders (e.g.,
resection of pancreatic tumor or octreotide,
tumors)
acromegaly, Cushing syndrome, galactorrhea)
surgical resection of pituitary tumor
Medullary thyroid
Medullary c arc inoma, increased calcitonin,
Total thyroidectomy, surgical resection of
Pheochromocytoma
hyperparathy roidism, hypercalcemia, increased serum
pheochromocytoma, subtotal
Parathyroid
and urine catecholamines
parathyroidectomy
Medullary thyroid
Medullary c arc inoma, increased calcitonin,
Total thyroidectomy, surgical resection of
Pheochromocytoma
hyperparathy roidism, hypercalcemia, Marfanoid body
pheochromocytoma
Mucosal neuroma
habitus, mucosal nodules
Pituitary
2a
2b
History: clinical findings are described in Table 5-10 Physical examination: clinical findings are described in Table 5-10 Tests: the excessive and deficient hormones for each enzyme deficiency may be detected by measuring levels at the appropriate steps of the synthetic pathway; sodium and potassium levels are useful to differentiate between the types of deficits Treatment: some combination of cortisol replacement (e.g., hydrocortisone, dexamethasone) and/or aldosterone replacement (e.g., fludrocortisone) is typically required; surgical and social solutions to genital abnormalities should be considered Outcomes: Complications include renal failure, DM, growth impairment, cataracts, and infertility Failure to provide steroid and mineral corticoid replacement can be fatal if the body is not capable of compensating from stressful events
Prognosis is good if adequate hormone replacement is provided Psychologic issues are a concern given gender uncertainty issues that may affect these patients Why eliminated from differential: although some of the clinical characteristics of these conditions are seen in the case, the lab analysis shows that the patient is suffering from cortisol excess, and these diagnoses are ruled out Multiple endocrine neoplasia (MEN) Autosomal dominant syndromes involving the dysfunction of multiple endocrine glands simultaneously (Table 5-11) Dysfunctional glandular activity may be due to hyperplasia or neoplasm History: symptoms correspond to the involved organs Physical examination: signs correspond to the involved organs Tests: the same diagnostic findings are expected for the collection of endocrine abnormalities as for each individual component Treatment: surgical resection of the causative lesions is typically required Outcomes: for uncertain reasons, the prognosis for these conditions tends to be better than for the isolated endocrine conditions Why eliminated from differential: although pituitary dysfunction is a consideration in this case, the findings are insufficient to support a diagnosis of any of the other components of these diagnoses
If you see a child with extremely advanced growth for the given age (i.e., gigantism), perform a work-up for increased GH.
Compare the current appearance of adult patients to multiple pictures at younger ages to aid in the detection of the gradual enlargement in features seen in acromegaly.
Patients with acromegaly have insulin resistance (similar to DM type 2) and develop diabetes in 10% of cases.
Deficient cortisol synthesis is usually not symptomatic in CAH because hyperplasia is able to maintain cortisol in the low to normal range.
A 21-α-hydroxylase deficiency is the most common form of congenital adrenal hyperplasia in the United States, but all forms are found in greater frequency in patients of Jewish heritage.
Although patients with 11-β-hydroxylase deficiency have a deficit of aldosterone, they still exhibit hypernatremia and hypokalemia because deoxycorticosterone functions as a mineralocorticoid.
MEN type 1 may be remembered by the three Ps: Parathyroid, Pancreas, Pituitary. MEN type 2a may be remembered by the three Cs: Carcinoma of thyroid (medullary),
Catecholamine excess (pheochromocytoma), Calcium excess (hyperparathyroidism). MEN type 2b may be remembered by the three Ms: Medullary thyroid carcinoma, Medulla of adrenal tumor (pheochromocytoma), Mucosal neuromas.
Case 5-9: “My diabetes is cured, but I feel awful” A 38-year-old man presents to his endocrinologist because he has found that his daily insulin requirements have significantly decreased over the past several months. The patient is a type 1 diabetic and had been on a schedule of regular insulin before breakfast, lunch, and dinner and NPH insulin at bedtime for several years. He reports that his insulin requirements have been decreasing over the past 6 months to the point that he sometimes is forced to skip doses of regular insulin because his blood sugar is normal or low. Twice in the past month he has measured his blood glucose and found it to be significantly low. In both cases, he was able to consume candy to increase his glucose. He was diagnosed with a case of hepatitis A 14 months ago that he presumably acquired from a seafood buffet, but this infection resolved completely by itself prior to this past year. He felt fine for several months, but over the past 6 months he has developed worsening fatigue and gastrointestinal (GI) symptoms. He feels tired most of the time regardless of his blood glucose. He has a poor appetite and has lost 15 pounds in the past 6 months. He is occasionally nauseous and has vomited on a few occasions. He occasionally becomes dizzy when he stands but denies any syncope. His arms and legs, including the joints, ache most days. He feels worn down most of the time and finds that he is forgetful of things that he would have remembered in previous years. He denies hematemesis or hematochezia, diarrhea, hematuria, paresthesias, chest pain, or flank pain. He feels that his current symptoms are somewhat reminiscent of his hepatitis infection last year. Besides DM type 1 he has no other medical problems. He has maintained tight control of his serum glucose levels with his insulin regimen and careful home glucose measurements and has not been diagnosed with any diabetic complications. He denies any substance use. He exercises three times per week. On examination, he is a thin man in no acute distress. He has mildly dry mucous membranes. He has no lymphadenopathy. He has a noticeable increase in pigmentation across his knuckles and the extensor surfaces of his knees and elbows. Auscultation of his heart and lungs detects tachycardia but no abnormal heart sounds and clear lung fields. His abdomen is nontender with no palpable masses, and he has normal bowel sounds. He has no flank tenderness. A rectal examination detects no gross blood. Neurologic examination detects mild generalized weakness (4+/5) and intact sensation. The following vital signs are measured: T: 98.4°F, HR: 115 bpm, BP: 100/70 mm Hg, RR: 18 breaths/min
Differential Diagnosis Hepatitis, chronic kidney disease, sarcoidosis, hyperparathyroidism, hyperthyroidism, Addison disease, SLE, human immunodeficiency virus (HIV), pancreatitis, endocarditis, cirrhosis, DM type 1
Laboratory Data and Other Study Results CBC: WBC: 7.1, Hgb: 13.4, Plt: 287 Chem10: Na: 129 mEq/L, K: 5.2 mEq/L, Cl: 103 mEq/L, CO2: 27 mEq/L, BUN: 40 mg/dL, Cr: 1.6 mg/dL, Glu: 72 mg/dL, Mg: 1.8 mg/dL, Ca: 11.1 mg/dL, Phos: 3.1 mg/dL LFTs: AlkPhos: 178 U/L, ALT: 70 U/L, AST: 59 U/L, TBili: 0.9 mg/dL, DBili: 0.4 mg/dL Thyroid panel: TSH: 6.2 µU/mL, T4: 6.1 µg/dL, free T4 index: 5.7, T3: 0.9 ng/mL, T3 reuptake: 0.84 Cortisol (afternoon): 2 µg/dL ANA: negative Amylase: 46 U/L Lipase: 127 U/L Anti-HAV (hepatitis A virus) immunoglobulin (Ig) M: negative
Anti-HAV IgG: positive Anti-HIV ELISA: negative CXR: small heart size; no infiltrations or increased vascular markings; no effusions; no hilar adenopathy Abdominal CT: normal appearance of bowels, pancreas, kidneys, liver, and spleen; difficult to visualize adrenal glands; no free air Based on these results, the following additional tests are ordered: PTH: 12 pg/mL Cosyntropin stimulation test: baseline cortisol 3 µg/dL, baseline aldosterone 6 ng/dL, 30-minute cortisol 7 µg/dL, 30-minute aldosterone 9 ng/dL ACTH (morning): 304 pg/mL
Diagnosis Addison disease
Treatment Administered The patient was admitted to the hospital for stabilization of his cortisol and aldosterone deficiency IV hydrocortisone was started and tapered over several days The patient was converted to oral hydrocortisone and fludrocortisone
Follow-up The patient demonstrated improvement in his symptoms by the time of discharge The patient's serum glucose, electrolytes, renal function, and LFTs gradually improved over the following 4 months The patient experienced less than three episodes of adrenal crisis every year requiring an increase in hydrocortisone dosing
Diabetics who develop Addison disease frequently require decreasing amounts of insulin.
Increased skin pigmentation is not seen in secondary or tertiary corticoadrenal insufficiency.
Steps to the Diagnosis Addison disease Cortisol and aldosterone deficiencies due to autoimmune destruction of the adrenal cortices Associated with thyroid diseases, parathyroid diseases, DM, ovarian or testicular failure, cirrhosis, and hepatitis Secondary corticoadrenal insufficiency has a similar presentation and is due to an insufficient ACTH supply
ACTH secretion may be down-regulated by chronic corticosteroid use Insufficient ACTH secretion may result from a pituitary pathology Tertiary corticoadrenal insufficiency has a similar presentation and is due to insufficient corticotrophinreleasing hormone secretion for the hypothalamus History: weakness, fatigue, anorexia, weight loss, nausea, vomiting (more common in primary disease), possible diarrhea, myalgias, arthralgias, impotence, decreased libido (more common in women), amenorrhea, memory impairment, dizziness, depression, possible psychosis Physical examination: hypotension, increased skin pigmentation (more prevalent on extensor surfaces and due to the feedback influence of melanocyte-stimulating hormone), vitiligo, dehydration, hair loss (in women) Tests: Decreased sodium and increased potassium due to aldosterone deficiency Possible decreased glucose and increased calcium Increased BUN and creatinine Mild increase in alkaline phosphatase, ALT, and AST Mild anemia is common TSH may be increased, but thyroid hormones are typically normal ACTH is increased in Addison disease but decreased in secondary and tertiary corticoadrenal insufficiency Serum cortisol may be low but is not the most accurate test for making a diagnosis A cosyntropin stimulation test is the best study to diagnose Addison disease Baseline cortisol and aldosterone levels are measured Synthetic ACTH (i.e., cosyntropin) is administered Serum cortisol and aldosterone are remeasured at either 30 or 60 minutes after cosyntropin administration Cortisol should increase by at least 7 µg/dL and should be >20 µg/dL at the second measurement Aldosterone should double by the second measurement Patients with Addison disease fail criteria for both cortisol and aldosterone; patients with ACTH deficiency will typically respond appropriately A CXR may show a small heart; CT may demonstrate abnormally appearing adrenal glands Treatment: Any underlying condition should be treated Corticosteroid replacement(e.g., hydrocortisone, dexamethasone, prednisone) is required Mineral-corticoid (e.g., fludrocortisone) replacement is not required with high-dose corticosteroid therapy but is needed with lower doses of corticosteroids and during exacerbations of the condition Outcomes: Untreated chronic disease carries a poor prognosis, but early adequate treatment improves survival Addisonian crisis is a severe exacerbation brought on by stress on the body and is characterized by severe weakness, fever, mental status changes, and vascular collapse; it requires aggressive treatment and carries a
poor prognosis if not recognized early Secondary corticoadrenal insufficiency may result from overaggressive cortisol replacement Clues to the diagnosis: History: decrease in insulin need, fatigue, anorexia, weight loss, nausea, vomiting, dizziness, myalgias, arthralgias, memory impairment Physical: mild dehydration, increased pigmentation, mild motor weakness Tests: decreased sodium, increased potassium, increased BUN and creatinine, decreased glucose, increased LFTs, increased TSH, failed cosyntropin stimulation test, increased ACTH Hyperthyroidism More thorough discussion in prior case Why eliminated from differential: the TSH is elevated in this patient, but the thyroid hormone levels are normal, so this diagnosis is ruled out Hyperparathyroidism More thorough discussion in prior case Why eliminated from differential: this patient does have hypercalcemia, but the low normal PTH rules out this diagnosis Diabetes mellitus type 1 More thorough discussion in prior case Why eliminated from differential: this patient does have a preexisting diagnosis of DM type 1, but it is not typical for insulin needs to decrease over time in these patients, and this diagnosis cannot explain the cortisol abnormalities seen in this case Hepatitis A More thorough discussion in Chapter 3 Why eliminated from differential: this patient does have a history of hepatitis A, but the combination of a negative anti-HAV IgM and a positive anti-HAV IgG suggests that active disease has fully resolved; given the timing of his infection it is possible that his case of hepatitis may have played a role in inciting his adrenal disease Human immunodeficiency virus More thorough discussion in Chapter 6 Why eliminated from differential: the negative ELISA test rules out this diagnosis Endocarditis More thorough discussion in Chapter 1 Why eliminated from differential: the absence of fever, abnormal cardiac signs, or an elevated WBC count rule out this diagnosis Chronic kidney disease More thorough discussion in Chapter 4
Why eliminated from differential: there is a mild elevation of the BUN and creatinine, but these levels would be likely even higher in true CKD; in addition, CKD does not explain the cortisol abnormalities Systemic lupus erythematosus More thorough discussion in Chapter 9 Why eliminated from differential: the negative ANA makes this diagnosis unlikely Sarcoidosis More thorough discussion in Chapter 2 Why eliminated from differential: this diagnosis is ruled out on the basis that the patient does not exhibit palpable lymphadenopathy and the CXR does not show hilar lymphadenopathy, thus, corticoadrenal insufficiency provides a better explanation for the constellation of findings Pancreatitis More thorough discussion in Chapter 3 Why eliminated from differential: the normal amylase and lipase values rule out this diagnosis Cirrhosis More thorough discussion in Chapter 3 Why eliminated from differential: there is mild elevation of alkaline phosphatase, ALT, and AST in this case, but cirrhosis would more likely exhibit significantly higher values for these substances; this diagnosis is made less likely by the normal appearance of the liver on the CT
Case 5-10: “My baby isn't growing” A 2-month-old infant boy is brought to a rural health clinic by the child's parents and several family members. They are concerned because the child has grown very little since birth. Otherwise, they say that the child is a “good baby” because he is quiet and sleeps most of the day. The family lives in a rural, mountainous region and has little access to health care. The child's mother received no prenatal care, and the infant was delivered by a cousin who “delivers all the babies in the family.” The mother says that she was pregnant about 9 months. The parents say that the baby does not eat much despite the mother's attempts to breast feed it. They also have noticed that the infant has a bowel movement every few days. They have been giving the infant mineral oil, but this has only slightly increased the frequency of bowel movements. The parents say that they have three daughters who all appear to be healthy. The parents are unaware of any past medical history in themselves. The mother admits that she drank alcohol occasionally during the pregnancy. On examination, the infant is a small boy who is relatively lethargic. The child is mildly jaundiced and has dry skin. No lesions are apparent on the face or the body. The infant's fontanelles are open, but otherwise, the head anatomy appears normal. Examination of the mouth reveals a large tongue. Auscultation of the heart, lungs, and bowels detects no abnormal sounds. A small umbilical hernia is visible and palpable. The child has a normal anus and normal-appearing genitals. The child moves his extremities little during the examination and has general decreased tone. The anatomy of the extremities appears normal. The following vital signs are measured: T: 98.3°F, HR: 105 bpm, BP: 70/50 mm Hg, RR: 32 breaths/min (Normal for 2-month-old infant: HR: 100–170 bpm, BP: 65–115/42–80, RR: 30–50 breaths/min)
Differential Diagnosis Hypothyroidism, cystic fibrosis, phenylketonuria, Down syndrome, fragile X syndrome, cri-du-chat syndrome, fetal alcohol syndrome
Laboratory Data and Other Study Results Length, weight, and head circumference: 20.0" (less than third percentile), 9.5 lb (less than third percentile), 15.0" (fifth percentile) Chem7: Na: 139 mEq/L, K: 4.0 mEq/L, Cl: 99 mEq/L, CO2: 27 mEq/L, BUN: 17 mg/dL, Cr: 0.8 mg/dL, Glu: 78 mg/dL Thyroid panel: TSH: 8.5 µU/mL, T4: 2.1 µg/dL, free T4 index: 2.4, T3: 0.4 ng/mL, T3 reuptake: 0.43 Phenylalanine: 1.3 mg/dL Sweat test: Cl: 21 mEq/L, Na: 26 mEq/L The infant is sent for the following test in the adjacent hospital: Thyroid scan: small area of uptake indicating an atrophic thyroid
Diagnosis Congenital hypothyroidism
Treatment Administered The infant was immediately started on levothyroxine
Follow-up The parents were provided education on the importance of treatment for their infant's condition and the importance of prenatal care for future children Despite the therapy provided, the infant continued to have relative short stature compared to other children his age and had moderate mental retardation
Steps to the Diagnosis Congenital hypothyroidism (a.k.a. cretinism) Congenital deficiency of thyroid hormones due to a severe in utero iodine deficiency or a hereditary disorder of thyroid synthesis that leads to growth failure and mental retardation History: poor feeding, lethargy, poor growth, constipation, significant time spent sleeping Physical examination: large open fontanelles, large tongue, umbilical hernia, jaundice, hypotonicity, dry skin, hypothermia Tests: increased TSH and decreased thyroid hormones; thyroid scan frequently demonstrates abnormal uptake Treatment: levothyroxine should be started as early as possible to avoid permanent complications Outcomes: children not receiving prompt thyroid replacement therapy will have short stature, developmental delays, and mental retardation Clues to the diagnosis: History: poor growth, lethargy, constipation Physical: small size, open fontanelles, dry skin, large tongue, umbilical hernia, hypotonia Tests: increased TSH, decreased thyroid hormones, abnormal thyroid scan
Phenylketonuria A congenital inability to metabolize phenylalanine The accumulation of dietary phenylalanine causes mental impairment History: once phenylalanine accumulates, patients develop developmental delays, vomiting, poor behavior, self-mutilation, and possible seizures Physical examination: normal appearance at birth, eczema, body odor, light skin complexion Tests: increased serum phenylalanine; brain MRI will show areas of demyelination in individuals who have accumulated the protein Treatment: dietary restriction of phenylalanine with tyrosine supplementation Outcomes: normal cognitive development is expected as long as the patient adheres to the phenylalaninerestricted diet; mental retardation and social impairments result from an accumulation of the protein Why eliminated from differential: the normal phenylalanine level rules out this diagnosis Cystic fibrosis More thorough discussion in Chapter 2 Why eliminated from differential: the normal sweat test rules out this diagnosis Down syndrome (i.e., trisomy 21) Genetic disease resulting from autosomal nondisjunction or a genetic translocation in chromosome 21 during gamete formation (Table 5-12) Autosomal trisomies occur less frequently in chromosomes 18 and 13 Nondisjunction most commonly occurs in maternal germ cells, and the risk of nondisjunction increases with maternal age (especially after 35 years of age) History: developmental delays, mental retardation Physical examination: protruding tongue, flat nose, small ears, vision and hearing loss, broad hands with a simian crease, increased space between the first and second toes Tests: the diagnosis is typically clinical, but karyotype analysis and genetic screening can detect the chromosomal abnormalities Treatment: Treat any comorbid conditions as appropriate Special education and a selective environment capable of handling mentally retarded individuals is required in children and possibly throughout life
Table 5-12 Autosomal Trisomies
Condition
Inc idenc e
H/P
Trisomy 21
~1/700 births
Mental retardation, craniofacial abnormalities (e.g., protruding tongue, flat nose, small ears),
(Down
(inc reases w ith
vision and hearing loss, broad hands with simian crease, cervical spine instability, increased space
syndrome)
maternal age)
between first and second toes; increased risk of duodenal atresia and other GI abnormalities, Alzheimer disease, and c ardiac defec ts
Trisomy 18
Trisomy 13
1/6,000 births
Severe mental retardation, small mouth, limb abnormalities (e.g., malposition, roc ker-bottom
(inc reases w ith
feet, overlapping fingers on grasp), cardiac defects, GI abnormalities; frequently fatal within first
maternal age)
year of life
1/5,000 births
Cleft lip and palate, cardiac defects, CNS defects, severe mental retardation, rounded nose,
(inc reases w ith
poly dac tyly ; frequently fatal within first year of life
maternal age)
CNS, central nervous system; GI, gastrointestinal; H/P, history and physical.
Surgical correction of compromising anatomic defects (e.g., duodenal atresia, cardiac defects) should be performed Genetic counseling for the parents may help guide plans for future children Outcomes: The degree of mental impairment determines the patient's ability to function in society or the need for constant care Patients typically survive into the fourth decade of life Congenital cardiac defects and early Alzheimer disease are the main causes of mortality Why eliminated from differential: the child in this case does not exhibit some of the key physical features seen in this diagnosis; in addition, Down syndrome would not explain the thyroid abnormalities Fragile X syndrome An X-linked chromosomal disorder associated with mental retardation in men Women may be carriers and rarely show any effects of the abnormal gene The end of the X chromosome becomes fragile and does not condense normally due to a high number of terminal CGG codon repeats The number of CGG repeats increases with each generation when inherited from a woman but not from a man History: hyperactivity, mental retardation, possible seizures Physical examination: large face, prominent jaw, large ears, mild hand and foot abnormalities, large testicles Tests: genetic screening detects hundreds of CGG repeats at the end of the X chromosome; prenatal DNA analysis may be performed in women with a family history Treatment: special education and monitoring will be needed by affected males; parents should be provided genetic counseling Outcomes: an individual's ability to function in society and his caretaking requirements will depend on the severity of mental impairment; a normal lifespan is expected Why eliminated from differential: the patient does not exhibit the physical features of this diagnosis, and this
diagnosis would not explain the thyroid abnormalities; in the absence of thyroid hormone deficiency, genetic screening would be considered to determine if this condition was the cause of the child's delays Cri-du-chat syndrome An uncommon genetic syndrome resulting from deletion of the entire p arm of chromosome 5 (Table 5-13) History: high-pitched catlike cry, poor feeding, developmental delays, mental retardation, poor social behavior (e.g., aggression, tantrums, withdrawal) Physical examination: small head, large ears, small mouth, flat nose, strabismus, short digits, single palmar crease low birth weight, failure to thrive, hypotonia
Table 5-13 Common Deletion Syndromes
Syndrome
Cri du chat
Wolf-Hirschhorn
Prader-Willi
Angelman
Deletion
H/P
Entire 5p chromosome
High-pitc hed c atlike c ry , small head, low birth weight, mental retardation; early mortality may
arm
result from failure to thrive
4p16 to end of arm
Mental retardation, multiple c ranial abnormalities, seizures
15q11–15q13
Overeating, obesity , decreased muscular tone in infancy, mental retardation, small hands and feet;
(inherited from father)
obesity-related complications may decrease lifespan
15q11–15q13
Puppetlike movement, happy mood, unprovoked laughter, mental retardation, seizures
(inherited from mother)
Velocardiofacial
22q11
Cleft palate, cardiac defects, mild mental retardation, significant overbite, speec h disorders, association with DiGeorge syndrome; early mortality may result from associated cardiac complications or DiGeorge syndrome
H/P, history and physical.
Tests: genetic screening with high-resolution chromosome banding and fluorescence in situ hybridization are used to detect small deletions; large deletions may be detected on karyotype analysis Treatment: supportive care and monitoring; parents should receive genetic counseling Outcomes: patients may die at an early age due to failure to thrive; mental impairments are typically fairly severe Why eliminated from differential: this diagnosis does not explain the thyroid hormone abnormalities, but genetic analysis would be considered if there was not an obvious cause for the patient's condition Fetal alcohol syndrome More thorough discussion in Chapter 12 Why eliminated from differential: it is possible that the mother's alcohol use may have contributed to the child's developmental delays, but it is unlikely to be related to the thyroid deficiencies
Down syndrome is the most common cause of congenital mental retardation when
both genders are considered.
Fragile X syndrome is the most common cause of inherited mental retardation and the second overall most common cause of all mental retardation in men.
The differences between Prader-Willi syndrome and Angelman syndrome may be remembered by the mnemonics POP and MAMA: Prader-Willi, Overeating, Paternal and Maternal, Angelman, Mood (happy), Animated movements.
Authors: Van Kleunen, Jonathan P. Title: Step-Up to USMLE Step 3, 1st Edition Copyright ©2009 Lippincott Williams & Wilkins > Table of Contents > Chapter 6 - Hematology and Oncology
Chapter 6 Hematology and Oncology Basic clinical primer Red Blood Cell (RBC) Physiology RBCs function to transport oxygen (O2) from alveoli to tissues and to transport carbon dioxide (CO2) from tissues to the lungs using hemoglobin (Hgb) as the binding protein for both gases Binding of O2 by Hgb A follows the Hgb-O2 dissociation curve (Figure 6-1) Changes in the inhaled air composition and pressure, Hgb subtype, body temperature, and activity level will cause the curve to shift Circulating RBCs, myeloid cells, and lymphoid cells originate from the same pluripotent stem cells in bone marrow (Figure 6-2) As RBCs mature in bone marrow, they become enucleated and depend on glycolysis for survival
Clotting Function Platelets Circulate in plasma as the primary controllers of bleeding Cause localized vasoconstriction and form a platelet plug at the site of vascular injury in response to adenosine diphosphate (ADP) that is secreted by the injured cells Platelet response may be quantified by measuring the bleeding time (poorly reproducible) Coagulation factors Responsible for the formation of fibrin clot at the site of vascular injury (Figure 6-3) The intrinsic pathway is induced by exposure of vascular tissue to negatively charged foreign substances and is measured by the partial thromboplastin time (PTT) The extrinsic pathway is induced by tissue factor exposed at the site of injury and is measured by prothrombin time (PT)
Monitor heparin anticoagulation with PTT.
Low molecular weight heparins (LMWH) do not require monitoring by PTT.
Monitor warfarin anticoagulation with a normalized PT (i.e., international normalized ratio [INR]) to track its relative effect on the extrinsic pathway.
Antithrombotic Medications Medications used to reduce the risk of pathologic clot formation (e.g., deep vein thrombosis [DVT], thromboembolic stroke, mural thrombus, pulmonary embolism [PE], postsurgical or traumatic thrombus) May affect platelet function and intrinsic or extrinsic pathway function (Table 6-1)
Do not start warfarin therapy for a thrombus until after starting LMWH or until the PTT is therapeutic on unfractionated heparin because warfarin inhibits proteins C and S to cause a short period of hypercoagulability immediately after therapy is initiated.
Treat a warfarin overdose with vitamin K.
Presurgical Assessment of Coagulation A history of abnormal bleeding or easy bruising should raise the concern for a coagulopathy P.157
Figure 6-1 The hemoglobin-oxygen dissociation curve. O2, oxygen; pO2, partial pressure of oxygen. (See color image.)
Patients taking warfarin should stop this medication 3 to 4 days before surgery with a goal INR of <1.5 Fresh frozen plasma (FFP) and vitamin K may be used for rapid warfarin reversal Patients with a recent thromboembolism should be anticoagulated with heparin or LMWH after stopping warfarin until the time of surgery and then restarted on warfarin postoperatively; heparin or LMWH should be restarted 12 hours postoperatively and continued until an INR >2.0 is achieved In general, warfarin, heparin, and LMWH are associated with a lower risk of postoperative thromboembolism than aspirin or antiplatelet medications but carry a greater risk of postoperative bleeding complications
LMWH should not be restarted for at least 2 hours after the removal of an epidural catheter to avoid formation of an epidural hematoma.
AB+ patients are “universal recipients”; they can receive any donor blood type because they have no antibodies to blood antigens in their plasma but can only donate to other AB+ patients.
O-patients are “universal donors.” RBCs from these patients will not induce antibody reactions in other patients, but they can only receive blood from other O– donors.
Transfusions Use of an infusion of blood products to treat an insufficient supply of that component, improve oxygen-carrying capacity, or reverse coagulopathy (Table 6-2) ABO blood groups Blood is defined by A and B antigens and their respective antibodies Blood with one antigen only will have antibodies for the other antigen (i.e., A or B types) Blood with both antigens will not have either form of antibody (i.e., AB type) Blood with neither antigen will have antibodies to both antigens (i.e., O type) Rh blood groups Blood with either carry the Rh antigen (i.e., Rh+) or will not (i.e., Rh-) Rh- blood carries antibodies to the Rh antigen P.158
Figure 6-2 Differentiation of hematopoietic stem cells in bone marrow. (See color image.)
P.159
Figure 6-3 The coagulation cascade. INR, international normalization ratio; PT, prothrombin time; PTT, partial thromboplastin time. (See color image.)
Transfusions must be matched for the particular ABO and Rh blood types to avoid transfusion reactions (Table 6-3)
Clerical errors are the most common cause of transfusion reactions due to incompatible blood.
A transfusion should be stopped promptly in any patient exhibiting signs of a transfusion reaction.
Organ Transplantation Donors are selected based on ABO blood type compatibility, cross-match compatibility (i.e., presence of antidonor antibodies on recipient T cells), and human leukocyte antigen (HLA) matching Transplant rejection may be hyperacute, acute, or chronic (Table 6-4) All transplant recipients require immunosuppressive agents to reduce the risk of rejection (Table 6-5) Transplant patients have greater risks of infection (secondary to immunosuppression), cancer (e.g., skin, B cell lymphoma, oral squamous cell, cervical, vaginal), and infertility HLA mismatching may cause graft versus host disease, in which immune cells transplanted along with the donor
tissue attack the immunocompromised host and cause hepatic, gastric, pulmonary, and dermatologic inflammation
Oncologic Therapy The ultimate goal of therapy is the eradication of neoplastic cells Secondary goals are to delay disease progression or to serve as palliative therapy Surgery Performed to reduce the mass of solid tumors or to remove well-contained tumors Tissue surrounding a lesion is frequently removed to increase the likelihood of resecting microscopic extensions of the tumor Many procedures are associated with significant morbidity P.160
Table 6-1 Common Anticoagulant Drugs
Drug
ASA
Mec hanism
Role
Adverse Effec ts
Inhibits platelet aggregation by
Decreases thrombus risk in CAD and
Increased risk of hemorrhagic
inhibiting cyclooxygenase activity
post-MI, decreases postoperative
stroke and GI bleeding
to suppress thromboxane A2
thrombus risk
synthesis
Thienopyridines (e.g.,
Block ADP receptors to suppress
Decreases risk of repeat MI or stroke
Increased risk of hemorrhage
clopidogrel, ticlopidine)
fibrinogen binding and platelet
in patients with prior MI, stroke, or
and GI bleeding
adhesion to injury sites
PVD; decreases thrombus risk in postvascular intervention patients
GP IIb/IIIa inhibitors
Inhibit platelet aggregation by
Reduce risk of thrombus in unstable
Increased risk of hemorrhage,
(e.g., abciximab)
binding to platelet GP IIb/IIIa
angina or following coronary vessel
nausea, back pain, and
receptors
intervention
hypotension
Adenosine reuptake
Inhibit activity of adenosine
Used in combination with ASA in
Dizziness, headache, and
inhibitors (e.g.,
deaminase and phosphodiesterase
patients with recent stroke or with
nausea
dipyridamole)
to inhibit platelet aggregation
warfarin following artificial heart valve replacement
Heparin
Binds to antithrombin III to
Postoperative prophylaxis for DVT and
Hemorrhage, hypersensitivity,
increase activity and prevent clot
PE, dialysis, decreases post-MI
and thromboc ytopenia;
formation
thrombus risk, safer than warfarin
narrow therapeutic window
during pregnancy
Low molecular weight
Binds to fac tor X a to prevent clot
Postoperative prophylaxis for DVT and
Hemorrhage, fever, and rare
heparin (e.g.,
formation
PE, safest option during pregnancy
thrombocytopenia
Direct thrombin
Highly selective inhibitors of
Alternative anticoagulation in patients
Hemorrhage and hypotension
inhibitors (e.g.,
thrombin to suppress activity of
with history of HIT
lepirudin, argatroban)
factors V, IX, and XIII and platelet
enoxaparin, dalteparin)
aggregation
Direct factor Xa
Highly selective inhibition of factor
DVT prophylaxis, anticoagulation
Hemorrhage, fever, anemia,
inhibitors (e.g.,
Xa without activity against
following acute DVT or PE
edema, rash, constipation
fondaparinux)
thrombin
Warfarin
Antagonizes vitamin K–dependent
Long-term anticoagulation
Hemorrhage, numerous drug
carboxylation of factors II, VII, IX,
postthrombotic event or in cases of
interac tions, and
and X
increased thrombus risk (postsurgery,
teratogenic ity
Afib, artificial valves)
Afib, atrial fibrillation; ASA, aspirin; CAD, coronary artery disease; DVT, deep vein thrombosis; GI, gastrointestinal; HIT, heparin-induced thrombocytopenia; MI, myocardial infarction; PE, pulmonary embolism; PVD, peripheral vascular disease.
Radiation therapy Performed to necrose tumor cells and to decrease tumor size Adverse effects include impaired surgical wound healing, fibrosis of tissue, skin irritation, esophagitis, gastritis, pneumonitis, neurologic deficits, bone marrow suppression, and radiation-induced malignancies (e.g., thyroid, chronic myelogenous leukemia [CML], sarcomas) Chemotherapy Aims to eradicate smaller populations of neoplastic cells and destroy cells not removed through surgery or radiation (Table 6-6) May sensitize neoplastic cells to radiation therapy (i.e., radiosensitizers) May be the primary treatment modality in certain cancers particularly receptive to pharmacological therapy P.161
Table 6-2 Types of Blood Products Used in Transfusions
Blood Produc t
Whole blood
Definition
Donor blood not separated into components (full volume blood)
Indic ations
Rarely used except for massive transfusions for severe blood loss
Packed RBCs
RBCs separated from other donor blood components (two-thirds
Product of choice for treatment of low Hct due to blood
volume of transfusion unit is RBCs)
loss or anemia
Autologous
Blood donated by a patient prior to elective surgery or other
Elective surgery or chemotherapy
blood
treatments; blood is frozen until needed by patient
FFP
Plasma from which RBCs have been separated
Warfarin overdose, clotting factor deficiency, DIC, or TTP
Cryoprecipitate
Clotting factor and vWF-rich precipitate collected during
Same indications as FFP; preferable to FFP in cases where
thawing of FFP
a large transfusion volume is unwanted
Platelets separated from other plasma components
Thrombocytopenia not due to rapid platelet destruction
Platelets
Clotting factors
Concentrations of a specific clotting factor pooled from multiple
Specific clotting factor deficiencies (e.g., hemophilia)
donors
DIC, disseminated intravascular coagulation; FFP, fresh frozen plasma; Hct, hematocrit; RBCs, red blood cells; TTP, thrombotic thrombocytopenic purpura; vWf, von Willebrand factor.
Table 6-3 Types of Transfusion Reactions
Ty pe
Febrile
Cause
Charac teristic s
Treatment
HLA antibodies in matched blood
Fevers during transfusion
Acetaminophen
ABO incompatibility
Severe destruction of host
Aggressive supportive care
nonhemolytic
Acute hemolytic
RBCs
Delayed
Kidd or Rh antibodies
Mild delayed hemolysis
Supportive care
Anti-IgA IgG antibodies transfused to
Rapid onset of shock and
Epinephrine, volume maintenance, and
patient with IgA deficiency
hypotension
airway management
Donor plasma not filtered from blood
Urticarial rash forms on
Diphenhydramine
hemolytic
Anaphylactic
Urticarial
recipient
HLA, human leukocyte antigen; Ig, immunoglobulin; RBCs, red blood cells.
Table 6-4 Forms of Transplant Rejection
Type
Hyperacute
W hen Seen
Initial 24 hours after
Cause
Antidonor antibodies in recipient
transplantation
Acute
6 days to 1 year after
Untreatable; should be avoided by proper cross-matching
Antidonor T cell proliferation in recipient
transplantation
Chronic
Treatment
Frequently reversible through immunosuppressive agents
More than 1 y ear after
Development of multiple cellular and humoral
Usually untreatable; immunosuppression
transplantation
immune reactions to donor tissue
may serve some role
P.162
Table 6-5 Immunosuppressive Drugs to Prevent Transplant Rejection
Drug
Indic ation
Mec hanism
Adverse Effec ts
Cyclosporine
Rejection prevention
Helper T cell inhibition
Nephrotoxic ity , androgenic effects, HTN
Azathioprine
Rejection prevention
Inhibits T cell proliferation
Leukopenia
Tacrolimus
Rejection prevention and
Inhibitor of T cell function
Nephrotoxic ity , neurotoxicity
Rejection prevention and
Inhibits all leukoc yte
Cushing syndrome, weight gain, AVN of bone
reversal
activity
Muromonab-D3
Rejection reversal and
Inhibitor of T cell function
Induces one-time c ytokine release (fever,
(OKT3)
early rejection
and depletes T cell population
bronchospasm), leukopenia; limited to short-term
reversal
Corticosteroids
therapy
maintenance
Rapamycin
Rejection prevention
Helper T cell inhibition
Thrombocytopenia, hyperlipidemia
Mycophenolic acid
Rejection prevention
Inhibits T cell proliferation
Leukopenia, GI toxicity
Antithymocyte
Rejection reversal and
Depletes T cell population
Limited to short-term therapy , serum sickness
globulin
early rejection
Inhibits antigen processing
Visual disturbances
Chronic graft vs. host
Inhibits T cell function and
Sedation, constipation
disease
migration
maintenance
Hydroxychloroquine
Chronic graft vs. host disease
Thalidomide
AVN, avascular necrosis; HTN, hypertension.
Multiple drugs with different cell cycle–specific targets are frequently combined to increase neoplastic cell death while minimizing the toxicity to normal tissues, to have an effect against a broader range of cells, and to slow the development of resistance Adverse effects include bone marrow suppression, alopecia, gastrointestinal (GI) upset, infertility, neurotoxicity, hepatotoxicity, skin changes, pulmonary fibrosis, cardiomyopathy, and renal toxicity
Table 6-6 Mechanisms and Classes of Chemotherapeutic Drugs
Mec hanism
Drug Class
Examples
Free radical production causing cytotoxic alkylation of DNA
Nitrogen mustard alkylating agents
and RNA
Cyclophosphamide, chlorambucil, ifosfamide, mechlorethamine
Nitrosourea alkylating agents
Carmustine, streptozocin
Alkyl sulfonate alkylating agents
Busulfan
Ethylenimine/methylmelamine alkylating
Thiotepa, hexamethylmelamine
agents
Triazine alkylating agents
Dacarbazine
Inhibition of spindle proteins to stop mitosis or to cause
Vinca alkaloids
Etoposide, vinblastine, vincristine
cytotoxic polymerization
Taxanes
Paclitaxel, docetaxel
Inhibition of DNA and RNA synthesis
Antibiotics
Bleomycin, dactinomycin, daunorubicin, doxorubicin, mitomycin
Interference with enzyme regulation or DNA and RNA
Monoamine oxidase inhibitors
Procarbazine
Antimetabolites
Cytarabine, 5-flurouracil, methotrexate,
activity
Modulation of hormones to cause tumor remission
mercaptopurine
Platinum analogues
Carboplatin, cisplatin
Steroid hormones and antagonists
Prednisone, tamoxifen, estrogens, leuprolide
P.163
Case 6-1: “My father's anemia is keeping him from walking” A 62-year-old man is seen in referral by a hematologist for microcytic anemia. The patient had initially seen his primary care provider (PCP) for symptoms of significant weakness, memory impairment, and impaired coordination, which developed over the past year, and the referral is directed toward finding a relationship between anemia that was detected during his initial work-up and his multiple symptoms. The PCP was unable to explain the patient's symptoms and feels that the elucidation of the cause of his anemia is the key to his work-up. He is accompanied to the office by his daughter. She says that she has noticed a gradual deterioration in her father's mobility and function over the past year. She says that her father was a very active man in the past, but he has gradually become less active. He has also been unsteady on his feet while walking and has fallen from standing five times over the past few months. The patient says that he feels tired all of the time. His daughter notes that once he awakens in the morning, he sits in his favorite chair and watches television all day. It is extremely difficult to get him to do any activities because he complains about not having “enough energy.” He has stopped leaving the house to shop, visit family, or go to church because of his lethargy. She says that he also has had difficulty remembering simple instructions and quickly forgets things that have been told to him. The patient says that he is uncertain when his symptoms started. He notes frequent headaches and abdominal pain. He feels that his balance is impaired by constant aching in his knees and ankles. He is frustrated by his problems with short-term memory and his constant tiredness. He was working as a contractor for the military, and his primary assignment was the restoration of retired warships. He was removing old paint from these ships and cleaning them out, but has been unable to work for the past 6 months because of his impaired balance. He denies nausea, vomiting, diarrhea, hematemesis, or hematochezia. He has a past medical history of hypertension (HTN) and
hyperlipidemia for which he takes enalapril and simvastatin. He drinks five alcoholic beverages per week and denies smoking. On examination, he appears well-nourished in no acute distress. His skin is mildly pale. He has mild papilledema on fundoscopic examination. He has grayish streaks in his gums at the bases of his teeth. He has no lymphadenopathy. Auscultation of his heart and lungs detects normal breath and heart sounds. His abdomen is soft and nontender with normal bowel sounds. A rectal examination detects no gross blood. Neurologic examination detects mild impairments of coordinated movement and normal sensation. He has a mildly unsteady gait when his walking is observed. The following vital signs are measured: Temperature (T): 98.6°F, heart rate (HR): 89 beats per minute (bpm), blood pressure (BP): 132/91 mm Hg, respiratory rate (RR): 17 breaths/min
Differential Diagnosis Acute renal failure, hemolytic anemia, iron deficiency anemia, lead poisoning, sideroblastic anemia, acute intermittent porphyria, thalassemia, anemia of chronic disease, mononucleosis, Lyme disease, vitamin B12 deficiency, cerebral vascular accident, alcoholism
Laboratory Data and Other Study Results The following studies were ordered by the patient's PCP: Complete blood cell count (CBC): white blood cells (WBC): 7.9, Hgb: 10.2, platelets (Plt): 301, mean corpuscular volume (MCV): 69 fL Blood smear: microcytic RBCs with basophilic stippling, no other RBC abnormalities 10-electrolyte chemistry panel (Chem10): sodium (Na): 139 mEq/L, potassium (K): 4.1 mEq/L, chloride (Cl): 109 mEq/L, carbon dioxide (CO2): 27 mEq/L, blood urea nitrogen (BUN): 27 mg/dL, creatinine (Cr): 1.5 mg/dL, glucose (Glu): 102 mg/dL, magnesium (Mg): 2.2 mg/dL, calcium (Ca): 10.9 mg/dL, phosphorous (Phos): 3.3 mg/dL P.164 Liver function tests (LFTs): alkaline phosphatase (AlkPhos): 67 U/L, alanine aminotransferase (ALT): 68 U/L, aspartate aminotransferase (AST): 51 U/L, total bilirubin (TBili): 0.9 mg/dL, direct bilirubin (DBili): 0.5 mg/dL Head computed tomography (CT): no evidence of hemorrhage, ischemia, or volume loss The following additional tests are ordered by the hematologist: Whole blood lead level: 63 µg/dL Free erythrocyte protoporphyrin: 42 µg/dL Reticulocyte count: 4% Nutrition panel: albumin: 4.6 g/dL, iron (Fe) 102 µg/dL, transferrin: 320 mg/dL, ferritin: 65 ng/mL Hemoglobin A2: 2.0% Hemoglobin F: 0% Lyme antibody enzyme-linked immunosorbent assay (ELISA): negative Vitamin B12 level: 526 pg/mL Twenty-four-hour urine porphobilinogen level: 0.9 mg
Diagnosis Chronic lead poisoning
Treatment Administered The patient's employer was notified of the potential risk of lead exposure in the patient's job The patient was started on oral 2,3-dimercaptosuccinic acid (DMSA) with a tapering dose over 3 weeks
Follow-up Follow-up whole blood lead levels detected a decrease to 31 µg/dL after the 3 weeks of chelation therapy The patient's symptoms resolved over the following 4 months, and his lead levels were found to continue to decrease The patient was allowed to return to work after his symptoms improved, and he was reassigned to a job that was less likely to be at risk for lead exposure
Steps to the Diagnosis Lead-poisoning anemia Anemia that results from the inhibition of heme synthesis following lead exposure (e.g., ingestion, inhalation) More common in children, especially those in urban environments Chronic alcoholism and isoniazid (INH) use may cause similar symptoms History: fatigue, weakness, abdominal pain, arthralgias, headaches, impaired short-term memory Physical examination: pallor, gingival lead lines, peripheral motor neuropathy, developmental delays in children Tests: Decreased Hgb and MCV; increased reticulocyte count (Table 6-7) Increased erythrocyte protoporphyrin Increased whole blood lead level Mild increases in BUN and creatinine and liver transaminases Blood smear will show microcytic RBCs with basophilic stippling and possible ringed sideroblasts (Color Figure 6-1) Treatment: elimination of the lead source or removal of the patient from the site of exposure is frequently sufficient to prompt gradual reduction in blood lead levels; patients with a whole blood lead level >80 µg/dL or symptomatic patients with levels P.165 >60 µg/dL may be treated with ethylenediamine tetraacetic acid (EDTA) or DMSA as chelation therapy
Table 6-7 Classification of Anemias by Mean Corpuscular Volume and Common Etiologies
Mic roc ytic (MCV <80 µL)
Iron deficiency
Normoc ytic (MCV 80–100 µL)
Hemolytic
Mac roc ytic (MCV >100 µL)
Folate deficiency
Lead poisoning
Chronic disease
Vitamin B12 deficiency
Chronic disease
Hypovolemia
Liver disease
Sideroblastic
Thalassemias
MCV, mean corpuscular volume.
Outcomes: complications include renal, hepatic, cardiac, and neurologic damage; patients who develop encephalopathy are at greater risk for chronic symptoms Clues to the diagnosis: History: fatigue, short-term memory impairments, abdominal pain, headaches, possible work-related exposure Physical: loss of coordination, gingival lead lines, pallor Tests: increased lead level, microcytic anemia, increased erythrocyte protoporphyrin, increased reticulocyte count, blood smear appearance Acute renal failure More thorough discussion in Chapter 4 Why eliminated from differential: the patient has mild increases in his BUN and creatinine, but these levels would not explain his constellation of symptoms or explain his multiple hematologic lab abnormalities Hemolytic anemia Anemia that results from a combination of an abnormally shortened RBC lifespan (normally 120 days) and insufficient marrow replacement of RBCs Common causes include RBC membrane defects, RBC enzyme defects, hemoglobinopathies, drug effects, or mechanical damage (Table 6-8, Color Figure 6-2) History: weakness, fatigue, dyspnea on exertion, other symptoms depending on the particular etiology (Table 6-8) Physical examination: pallor, tachycardia, tachypnea, palpitations, increased pulse pressure, jaundice, hepatosplenomegaly, brownish discoloration of urine, mental status changes, other signs per particular etiology (Table 6-8) Tests: Decreased Hgb, increased reticulocyte count, normal MCV Increased indirect bilirubin, lactate dehydrogenase (LDH), and serum haptoglobin Coombs test may help differentiate causes In a direct test, Coombs reagent is mixed with the patient RBCs, and agglutination indicates the
presence of immunoglobulin IgG and complement on the RBC membranes (e.g., warm and cold agglutinin diseases) In an indirect test, the patient's RBC are mixed with type O RBCs, which are then mixed with Coombs reagent, and agglutination indicates the presence of anti-RBC antibodies in the serum (e.g., Rh alloimmunization) Blood smear will show schistocytes (i.e., fragments of RBCs), spherocytes, and burr cells (Color Figure 6-3) Treatment: treatments vary with each etiology and include avoidance of the underlying stimulus, corticosteroids, and splenectomy (Table 6-8) Outcomes: prognosis varies with each etiology of anemia, but generally, patients will do better if they are able to be separated from the cause of hemolysis P.166
Table 6-8 Types of Hemolytic Anemias
Ty pe
Pathology
Blood Smear
Coombs Test
Drug-induced
Substances bind to
Burr cells,
Direc t +
Recent penicillin, L-dopa,
RBC membranes
schistocytes
(unless due
quinidine, other drug use
and cause
to oxidative
oxidative
destruction)
Other Diagnostic Aids
Treatment
Stop offending agent
destruction, induce production of antidrug antibodies, form immune c omplexes that fix, complement, or induce anti-Rh antibodies
Immune
Anti-RBC
Spherocytes
W arm-reac ting antibodies
Corticosteroids,
antibodies,
(warm
(IgG) or c oldreac ting
avoid c old
autoimmune
agglutinins),
antibodies (IgM)
disease, possibly
RBC
cold-reacting
drug-induced
agglutination
antibodies), stop
(cold
offending agent;
agglutinins)
splenectomy may be
Direc t +
exposure (with
needed in persistent cases
Mechanical
RBCs broken by
Sc histoc y tes
Negative
force or turbulent
Prosthetic heart valve,
Treat underlying
HTN, coagulation disorder
cause
Hepatosplenomegaly
Splenectomy
flow
Hereditary
Genetic defec t of
sphero-
RBC membranes
cytosis
resulting in
Spheroc y tes
Negative
Negative
spherical RBCs
G6PD
Deficiency of G6PD
RBCs with
Low G6PD (by indirect
Avoid oxidants;
deficiency
(i.e., enzyme
“bites” taken
measurement); dizziness
transfusion may be
required to repair
out of them,
and fatigue begins within
needed in severe
oxidative damage
Heinz bodies
days of ingesting the
cases
to RBCs); ingestion
(i.e., small
oxidant; mild form in African
of oxidant (e.g.,
densities of Hgb
Americans and more severe
fava beans, ASA,
in RBC)
form in people of
sulfa drugs) causes excessive RBC hemolysis
Mediterranean decent
ASA, aspirin; G6PD, glucose 6-phosphate dehydrogenase; Hgb, hemoglobin; HTN, hypertension; Ig, immunoglobulin; RBC, red blood cell.
Why eliminated from differential: the absence of the characteristic blood smear findings and the decreased MCV rule out this collection of disorders Iron-deficiency anemia Anemia that results from an insufficient supply of iron, leading to an insufficient production of heme Common causes include hemorrhage, poor dietary iron intake, poor GI absorption of iron, pregnancy, and menstruation History: fatigue, weakness, dyspnea on exertion, pica (i.e., cravings to eat ice, dirt, etc.) Physical examination: pallor, tachycardia, tachypnea, increased pulse pressure, angular celitis (i.e., irritation of the lips and the corners of the mouth), spooning of fingernails Tests: Decreased Hgb and MCV; decreased or normal reticulocyte count Decreased ferritin and iron; increased transferrin (Table 6-9) Blood smear shows hypochromic microcytic RBCs (Color Figure 6-4) Treatment: the cause of iron loss must be determined to rule out malignancies; long-term iron supplementation is required to replete body stores P.167
Table 6-9 Laboratory Distinction of Microcytic Anemias
Type
Serum Iron
Ferritin
TIBC
Mentzer
(transferrin)
Index 1
Blood Smear
Iron deficiency
↓
↓
↑
>13
Hypochromic, microcytic RBCs
Lead poisoning
Normal or
Normal
Normal
>13
Stippled, microcytic RBCs
Normal or
↓
>13
Hypochromic, normocytic or microcytic RBCs
↑
Chronic
↓
disease
↑
Sideroblastic
↑
↑
↓
>13
Ringed sideroblasts
Thalassemia
Normal or
Normal
Normal
<13
Microcytic RBCs, target cells (α), basophilic
↑
stippling (β)
MCV, mean corpuscular volume; RBC, red blood cells; TIBC, total iron-binding capacity; ↑, increased; ↓, decreased. 1
Mentzer index = MCV ⊹ RBC count
Outcomes: the prognosis is excellent with iron supplementation, but neoplasm as a cause must be ruled out Why eliminated from differential: the increased reticulocyte count and normal nutrition panel rule out this condition Sideroblastic anemia Anemia resulting from a defect in heme synthesis that causes a decrease in Hgb levels in RBCs May be due to genetic factors or induced by alcohol or INH use History: fatigue, weakness, dyspnea on exertion, angina Physical examination: pallor, tachycardia, tachypnea, increased pulse pressure, hepatosplenomegaly Tests: Decreased Hgb and possible decreased MCV Increased ferritin and iron and decreased transferrin Blood smear shows a significant variability in the size of RBCs and ringed sideroblasts (Color Figure 6-5) Treatment: Hereditary cases may respond to supplemental vitamin B6 Supplemental erythropoietin (EPO) may be used in acquired cases Therapeutic phlebotomy or iron chelation with deferoxamine may be required in cases of iron overload (occurs because the obstacle to heme synthesis causes a buildup of iron stores) Outcomes: adverse reactions to vitamin B6 therapy may occur; acute leukemia develops in 10% of patients Why eliminated from differential: the normal nutrition panel and the consistency of microcytic RBCs on the blood smear rule out this diagnosis Acute intermittent porphyria Defect in the metabolism of heme, leading to an accumulation of porphobilinogen and amino-levulinic acid May be caused by a genetic defect or may be drug related History: severe prolonged abdominal pain, nausea, vomiting, constipation, depression, psychotic behavior Physical examination: tachycardia, HTN, areflexia, motor dysfunction P.168 Tests: increased urine porphyrins (particularly porphobilinogen); decreased serum sodium Treatment: hematin and high-dose glucose are used to stop exacerbations; precipitating factors should be avoided Outcomes: exacerbations rarely recur if the underlying causes are avoided Why eliminated from differential: the normal urinary porphobilinogen level rules out this diagnosis Thalassemia Defects in Hgb resulting from an abnormal production of heme α-globin and β-globin subunits The disease state arises from the unbalanced production ratio of subunits and not from the subunits themselves
Normally Hgb is composed of two α-subunits, whose synthesis is controlled by four genes, and two β-subunits, whose synthesis is controlled by two genes (Table 6-10) α-thalassemia may feature defects in one to four of the genes and is more common in people of African or Asian descent β-thalassemia may feature defects in one or both genes and is more common in people of Mediterranean descent
Table 6-10 Variants of α- and β-Thalassemias
Thalassemia Type
α
Number of V ariant
Hydrops fetalis
Abnormal Genes
4
Charac teristic s
No α-globin production Fetal death occurs
Hemoglobin H
3
disease
Minimal α-globin production Chronic hemolytic anemia, pallor, splenomegaly Hemoglobin Barts in serum Microcytic RBCs on blood smear Decreased patient lifespan
α-thalassemia
2
minor
Reduced α-globin production Mild anemia Microcytic RBCs and target cells on blood smear
α-thalassemia
1
minima
Generally asymptomatic Children of carriers are at an increased risk for thalassemia pending the genotype of the other parent
β
β-thalassemia
2
major
No β-globin production Asymptomatic until decline of fetal hemoglobin Growth retardation, developmental delays, bony abnormalities, hepatosplenomegaly, anemia Increase in hemoglobin A2 and F Microcytic RBCs on blood smear Patients die in childhood without transfusions
β-thalassemia minor
1
Reduced β-globin production Mild anemia Patients can lead normal lives Transfusions may be needed during periods of stress
RBC, red blood cell.
P.169 History: patients with only one defective gene are frequently asymptomatic, while patients with more severe forms of the conditions may experience fatigue and activity intolerance Physical examination: pallor, facial bony abnormalities, hepatosplenomegaly, jaundice, growth retardation Tests:
Decreased Hgb and MCV; increased reticulocyte count Increased Hgb variants (e.g., Barts, F, A2) Blood smear will show abnormally shaped microcytic RBCs and target cells in α-thalassemia and RBCs with a variable size and shape and basophilic stippling in β-thalassemia (Color Figure 6-6) Treatment: α-thalassemia with one or two defective genes and β-thalassemia with only one defective gene are frequently treated symptomatically only during periods of stress Folate supplementation may help relive symptoms in all variants Transfusions may be required for severe variants and for milder variants during symptomatic exacerbations; chronic transfusions may require iron chelation Bone marrow transplant may benefit young children with an early diagnosis Outcomes: patients with more severe variants of the disease have a high mortality rate due to cardiac and hepatic failure; chronic iron overload from frequent transfusions may cause cardiac and hepatic complications Why eliminated from differential: the absence of increased RBC variability makes this diagnosis unlikely Anemia of chronic disease Anemia common in patients with neoplasms, diabetes mellitus (DM), autoimmune disorders, or chronic infections Frequently associated with the trapping of iron in macrophages, decreased erythropoietin production, and increased hepcidin levels (i.e., inhibitor of iron absorption and mobilization) History: fatigue, weakness, dyspnea on exertion Physical examination: pallor, tachycardia Tests: Mildly decreased Hgb and MCV Decreased iron and transferrin; normal or increased ferritin Blood smear shows mildly microcytic RBCs Treatment: treat the underlying disorder; supplemental EPO may help increase Hgb Outcomes: prognosis is more dependent on the underlying condition than the anemia itself Why eliminated from differential: the lack of a causative medical diagnosis and the normal nutrition panel make this diagnosis unlikely Mononucleosis More thorough discussion in later case Why eliminated from differential: the normal WBC and the absence of atypical WBCs on the blood smear rule out this diagnosis Lyme disease More thorough discussion in Chapter 9 Why eliminated from differential: the negative antibody assay makes this diagnosis unlikely
Vitamin B12 deficiency Anemia that results from an inadequate supply of vitamin B12 Pernicious anemia is an autoimmune deficit of intrinsic factor, leading to the impaired GI absorption of vitamin B12 Acquired causes include poor dietary intake, ileal resection, GI bacterial overgrowth, or a Diphyllobothrium latum parasitic infection History: fatigue, weakness, dyspnea on exertion, memory loss Physical examination: pallor, tachycardia, tachypnea, increased pulse pressure, symmetric paresthesias, ataxia, possible psychosis
Figure 6-4 Testing protocol for the Schilling test. vit., vitamin. (See color image.)
P.170 Tests: Decreased Hgb; increased MCV Decreased serum vitamin B12 levels The Schilling test is useful to determine the etiology of the condition (Figure 6-4) Blood smear will show macrocytic RBCs and hypersegmented neutrophils (Color Figure 6-7) Treatment: periodic intramuscular injections (for pernicious anemia) or dietary supplementation of vitamin B12
Outcomes: the prognosis is more favorable in acquired forms of the disease; chronic neurologic abnormalities may not improve following therapy Why eliminated from differential: the MCV measured in the case and the appearance of the blood smear rule out this diagnosis Cerebral vascular accident More thorough discussion in Chapter 7 Why eliminated from differential: the normal head CT makes this diagnosis unlikely Alcohol-related liver disease More thorough discussion in Chapter 3 Why eliminated from differential: the minimally elevated transaminases and normal other LFTs make this diagnosis unlikely
The list of common microcytic anemias may be remembered by the mnemonic “Look For Those Small Cells”: Lead-poisoning, Fe (iron) deficiency, Thalassemia, Sideroblastic, Chronic disease.
Iron deficiency anemia is the most common form of anemia.
Iron deficiency anemia in elderly patients should be considered due to colon cancer until ruled out.
Patients with α-thalassemia minima usually have a normal MCV.
Differentiate between the causes of microcytic anemia using the ratio of MCV to RBC count (i.e., Mentzer index): MCV:RBC count >13 suggests iron deficiency; MCV:RBC count <13 suggests thalassemia.
If microcytic anemia is found on blood smear, rule out thalassemia before administering supplemental iron to prevent iron overload.
Inadequate vitamin B12 intake is usually only seen in strict vegetarians (e.g., vegans).
Folate-deficiency anemia is the most common macrocytic anemia and is caused by poor nutrition (e.g., alcoholics) or medications affecting folate metabolism (e.g., methotrexate, phenytoin). Its presentation is similar to vitamin B12 deficiency except it has no neurologic symptoms, a normal vitamin B12 level, and a decreased serum folate level. It is treated with folate supplementation.
In cases of poor nutrition, folate deficiency develops significantly more quickly than vitamin B12 deficiency.
Case 6-2: “My daughter's hands and feet hurt” A 4-year-old girl is brought by her parents to her pediatrician because of pain in her arms and legs. The patient's mother says that she and the patient's father first noticed that the child would periodically favor one of her arms or legs about 1 year ago. These episodes would last a couple of days and then resolve. In the past 6 months, the child has periodically complained that her feet or hands hurt. These episodes seem to occur randomly and self-resolve within a week. The parents estimate that such episodes have occurred three times in the past 6 months. When they took her to a local emergency department during one such episode, a radiographic work-up was negative for any bony P.171 pathology. They are also concerned because the patient refrains from normal activities when she has these symptoms. They deny any trauma to the patient, nausea, vomiting, seizures, or impaired coordination. She is notable for a case of streptococcal pneumonia when she was 3 years old that required an inpatient admission and treatment with intravenous (IV) antibiotics. The parents deny any other past medical history. She was the product of an uncomplicated full-term vaginal birth. The family is of African descent and is notable for HTN, hyperlipidemia, and breast cancer in the patient's grandparents. The parents deny smoking and say that they live in a relatively new house without any risks for lead paint or carbon monoxide. On examination, the girl is thin but well nourished and in no acute distress. Her sclera are mildly icteric. She has no lymphadenopathy. Her lungs are clear to auscultation without any abnormal breathing sounds. Auscultation of her heart detects a soft, blowing systolic murmur. Her abdomen is soft and nontender, and her spleen is moderately enlarged and palpable. Her extremities are nontender on palpation but she says that her hands hurt. Her joints have a normal range of motion. She is able to walk normally. Her neurologic examination is normal. The following vital signs are measured: T: 99.0°F, HR: 86 bpm, BP: 105/80 mm Hg, RR: 18 breaths/min
Differential Diagnosis Thalassemia, mononucleosis, leukemia, sickle cell disease, rickets, cystic fibrosis, endocarditis, osteomyelitis
Laboratory Data and Other Study Results Height and weight: 38" (14th percentile), 30 lb (10th percentile) CBC: WBC: 11.9, Hgb: 8.2, Plt: 203, MCV: 85 fL Chem10: Na: 139 mEq/L, K: 4.3 mEq/L, Cl: 100 mEq/L, CO2: 23 mEq/L, BUN: 10 mg/dL, Cr: 0.4 mg/dL, Glu: 99 mg/dL, Mg: 1.9 mg/dL, Ca: 10.0 mg/dL, Phos: 3.4 mg/dL LFTs: AlkPhos: 80 U/L, ALT: 36 U/L, AST: 21 U/L, TBili: 1.4 mg/dL, DBili: 0.4 mg/dL, indirect bilirubin (IBili): 1.0 mg/dL Erythrocyte sedimentation rate (ESR): 2 mm/hr C-reactive protein (CRP): 0.3 mg/dL Sweat test: Cl: 26 mEq/L, Na: 32 mEq/L Blood smear: few sickled red cells and target cells; no abnormal leukocytes Chest x-ray (CXR): normal lung fields; no infiltrations or lymphadenopathy Echocardiogram: slight mitral and tricuspid regurgitation; no wall defects; normal wall motion, vasculature, and estimated chamber pressures After these results are received, the additional studies are ordered: Reticulocyte count: 10% Serum haptoglobin: 39 mg/dL
Hemoglobin A: 0% Hemoglobin A2: 1.3% Hemoglobin F: 8.4% Hemoglobin S: 90.3% Transcranial Doppler ultrasound (US): normal carotid and cerebral blood flow
Diagnosis Sickle cell disease
Treatment Administered The family was referred to a sickle cell support group to discuss coping with recurrent pain, disease complications, and the expected outcomes The patient was provided the pneumococcal vaccine, and all other vaccinations were updated as indicated by the recommended vaccination calendar P.172 Prophylactic penicillin was initiated and scheduled as a daily dose until 5 years of age Folate supplementation was initiated
Follow-up Annual transcranial Doppler studies detected no changes in cerebral perfusion over the following several years The patient experienced sickle cell crises one to two times per year after the age of 8 years that required narcotic pain control and transfusion therapy Through midadolescence, the patient had developed no significant cerebral, musculoskeletal, or vascular complications
β-globin defect: in sickle cell disease causes production of defective β-chains; in β-thalassemia causes decreased production of normal β-chains.
Heterozygous carriers of the sickle cell defect (i.e., sickle cell trait) are asymptomatic and carry an improved resistance to malaria.
Steps to the Diagnosis Sickle cell disease Autosomal recessive disease in which a defect in the β-globin chain of Hgb leads to the production of Hgb S, a form of Hgb that is poorly soluble when deoxygenated Acidosis, hypoxia, and dehydration cause the polymerization of Hgb S, which leads to the distortion of RBCs into a sickle shape that is susceptible to hemolysis and vascular clumping Risk factors: African or Latin American heritage History:
Patients may be asymptomatic between crises or may note bone pain Stressful events (e.g., infection, illness, trauma) incite sickle cell crises characterized by severe bone pain, chest pain, pain and swelling of the hands and feet, dyspnea, and priapism Physical examination: decreased growth velocity, jaundice or pallor, splenomegaly, fever, possible leg ulcers Tests: Decreased Hgb and increased reticulocyte count; WBCs may be increased Decreased serum haptoglobin No Hgb A, increased Hgb F, and presence of Hgb S (normally not present) on electrophoresis Increased indirect bilirubin Solubility tests (e.g., SICKLEDEX, Streck, Omaha, Nebraska) can detect Hgb abnormalities but cannot differentiate between the carrier trait and homozygous disease states Blood smear shows target cells, sickle cells, and nucleated RBCs; deoxygenation of blood significantly increases the number of sickled cells (Color Figure 6-8) X-rays may show “fish-mouth” vertebrae and avascular necrosis of bony regions with a tenuous blood supply (e.g., femoral head) Transcranial Doppler US is useful to measure the carotid and cerebral blood flow and determine the stroke risk Treatment: Referral of the patient to a sickle cell specialty group is beneficial to coordinate treatment and to counsel the patient on chronic pain issues and disease outcomes Sickle cell crises are treated with hydration, supplemental oxygen, and narcotic pain control Transfusions, exchange transfusions, and hydroxyurea are frequently useful to shorten crises or to decrease the number of crises in patients with frequent recurrences All children should be kept current on their vaccinations (particularly pneumococcal vaccine) to reduce the risk of infection Bone marrow transplantation has demonstrated success as a curative therapy, but due to the appreciable mortality and morbidity rates it is currently reserved for patients with severe disease and multiple complications P.173 Outcomes: Complications include chronic anemia, pulmonary HTN, heart failure due to cardiac stresses, vascular insufficiency, renal failure, and infections An aplastic crisis may follow infection (typically parvovirus B19) and is notable for severe anemia requiring aggressive transfusion therapy Acute chest syndrome is severe chest pain due to pneumonia, embolization, or pulmonary infarction and requires pain control and respiratory support Autosplenectomy, stroke, osteonecrosis of the femoral or humeral head, and multiorgan ischemia may result from vascular occlusion Increased risk of infection by encapsulated organisms (e.g., Streptococcus pneumoniae, Haemophilus
influenzae, Neisseria meningitides, Klebsiella) Despite the multiple potential complications, survival at 18 years of age is 86% Clues to the diagnosis: History: hand and foot pain, childhood pneumonia, African descent Physical: lower than median height and weight for age, scleral icterus, splenomegaly, systolic murmur Tests: normocytic anemia, increased reticulocyte count, decreased serum haptoglobin, blood smear appearance, Hgb electrophoresis results Thalassemia More thorough discussion in prior case Why eliminated from differential: the normal MCV makes this diagnosis unlikely, and the Hgb electrophoresis results and the presence of sickle cells on the blood smear rule it out Mononucleosis Infection by Epstein-Barr virus (EBV) affecting B leukocytes and oropharyngeal epithelium Most common transmission is through intimate contact or exposure to saliva History: significant fatigue, sore throat, malaise Physical examination: lymphadenopathy, fever, splenomegaly, tonsillar exudates Tests: Increased WBCs Positive heterophile antibodies and EBV serology Blood smear shows an increased number of lymphocytes with some degree of atypia Treatment: because the condition is self-limited, only supportive care is typically necessary; patients should refrain from contact sports during splenomegaly to reduce the risk of splenic rupture (at least 1 month) Outcomes: prognosis is excellent; rare complications include splenic rupture and airway obstruction due to tonsillar hypertrophy Why eliminated from differential: the minimally elevated WBC count and the appearance of the blood smear make this diagnosis unlikely Leukemia More thorough discussion in later case Why eliminated from differential: the minimally elevated WBC count and appearance of the blood smear make this diagnosis unlikely Rickets More thorough discussion in Chapter 9 Why eliminated from differential: the history of normal-appearing x-rays, normal alkaline phosphatase, and normal calcium and phosphorus levels rule out this diagnosis Cystic fibrosis
More thorough discussion in Chapter 2 Why eliminated from differential: the normal results of the sweat test rule out this diagnosis Endocarditis More thorough discussion in Chapter 1 Why eliminated from differential: the minimally elevated WBC count and the absence of vegetations on the echocardiogram make this diagnosis unlikely
Complications of sickle cell disease may be remembered by the mnemonic SHARP MALICE: Stroke, Hemolytic anemia, Autosplenectomy, Renal necrosis, Priapism, Myocardial infarction, Acute chest syndrome, Lung infarctions, Infections, Crises (painful or aplastic), Eye disease (retinopathy).
Patients with sickle cell disease are particularly susceptible to Salmonella osteomyelitis (although Staphylococcus aureus is still the most common etiology) and sepsis by encapsulated organisms.
Symptoms of mononucleosis do not appear until 2 to 5 weeks after infection with EBV. P.174
Case 6-3: “My baby is sick all of the time” A 13-month-old infant boy is seen by a pediatric immunologist in referral because of frequent infections that are unexplained by his pediatrician. The child's parents say that he has had three cases of S. pneumoniae since birth that have required hospitalization. The first of these cases occurred at 6 months of age. His parents say that he has also had recurrent otitis media that has been treated with antibiotics. He is notable for a chronic cough that never seems to completely resolve. More recently he has had bouts of diarrhea that last for several days and then gradually resolve over a week. His parents deny vomiting, poor feeding, or significant developmental delays. Besides his recurrent respiratory infections, they deny a history of cutaneous, cerebral, or urinary infections. His mother is concerned because her brother and a male cousin exhibited a similar history when they were infants, and both died from pneumonia before the age of 5 years. His parents say that he has not been diagnosed with any medical conditions up to this time. He has not received medications other than antibiotics and has no history of radiation exposure. He has not yet received his 12-month vaccinations but is otherwise current. He was a product of an uncomplicated full-term vaginal birth. There is no significant family history besides that described above. No one in the household is a smoker. The patient is the couple's first child. On examination, the infant appears well nourished and is in no acute distress. He intermittently coughs during the office visit. No anatomic abnormalities of the face are detectable. No tonsils are visible on inspection of his oropharynx. His eardrums appear mildly inflamed. He has no palpable lymphadenopathy. Auscultation of his lungs detects rhonchi and mild wheezing. There is variation in the volume of his breath sounds across his lung fields. Auscultation of his heart detects no abnormal sounds. His abdomen is soft and nontender with normal bowel sounds. He is actively moving all extremities and has normal reflexes. The following vital signs are measured: T: 99.4°F, HR: 90 bpm, BP: 104/76 mm Hg, RR: 22 breaths/min
Differential Diagnosis Recurrent pneumonia, cystic fibrosis, human immunodeficiency virus (HIV), agammaglobulinemia, leukopenia without immune deficiency, common variable immunodeficiency, IgA deficiency, severe combined immunodeficiency, DiGeorge syndrome, hyper-IgM disease, Wiskott-Aldich syndrome, Fanconi anemia
Laboratory Data and Other Study Results Length, weight, and head circumference: 31" (41th percentile), 24 lb (40th percentile), 18.5″ (41th percentile) CBC: WBC: 7.2, Hgb: 14.1, Plt: 402 Lymphocyte flow cytometry: T cells: normal, B cells: significantly decreased Chem10: Na: 138 mEq/L, K: 4.0 mEq/L, Cl: 103 mEq/L, CO2: 28 mEq/L, BUN: 14 mg/dL, Cr: 0.7 mg/dL, Glu: 81 mg/dL, Mg: 2.1 mg/dL, Ca: 10.1 mg/dL, Phos: 4.4 mg/dL HIV ELISA: negative Sweat test: Cl: 29 mEq/L, Na: 28 mEq/L CXR: mild patchy infiltrates in both lung bases; no areas of consolidation; no lymphadenopathy The following additional tests are then performed: IgA: 0 mg/dL IgG: 79 mg/dL IgM: 0 mg/dL IgE: 0.002 mg/dL
Diagnosis Agammaglobulinemia (X-linked) P.175
Treatment Administered Regular infusions of IV immunoglobulin were initiated Bronchodilators and inhaled corticosteroids were prescribed for an as needed basis The measles, mumps, rubella (MMR) vaccine and varicella zoster vaccine (VZV) were withheld, but all other vaccines were administered on schedule
Follow-up Annual pulmonary function tests (PFTs) were started at the age of 5 years Aggressive antibiotic regimens were prescribed in cases of respiratory infection By consistently adhering to a regular IV immunoglobulin schedule, the patient was doing well into his adolescent years with no significant pulmonary complications at the time of most recent follow-up
The presentation of immune disorders does not occur immediately after birth because newborns retain maternally derived antibodies for approximately 3 months.
Steps to the Diagnosis X-linked agammaglobulinemia A defect in B lymphocyte differentiation leading to low B cell and immunoglobulin levels (Table 6-11)
Occurs as a X-linked disorder affecting only males, with females acting as carriers History: recurrent sinopulmonary bacterial infections that become more common after 6 months of age, diarrhea that cannot be explained by antibiotic use, chronic cough, possible arthralgias, and symptoms of inflammatory bowel disease in adulthood Physical examination: hypoplastic tonsils and lymph nodes, rhonchi, rales, wheezing Tests: Normal WBC count Flow cytometry detects very low levels of B lymphocytes Low IgG levels; undetectable IgA and IgM levels Treatment: IV immunoglobulin (IVIG) is able to maintain the humoral immune defenses and can provide a normal life if given on a regular basis Aggressive and prolonged antibiotic therapy for bacterial infections Bronchodilators and inhaled corticosteroids help improve symptoms during respiratory disease Live vaccines (e.g., MMR, VZV) should be avoided because of the risk of inducing the respective infections Outcomes: patients on IVIG frequently live into their fifth decade Clues to the diagnosis: History: recurrent pulmonary infections, diarrhea Physical: undetectable tonsils, signs of pulmonary infection Tests: decreased B cell numbers, low immunoglobulin levels Recurrent pneumonia More thorough discussion in Chapter 2 Why eliminated from differential: three episodes of pneumonia in 15 months is unusual in a child without an underlying condition; the low B cell and immunoglobulin levels confirm that there is an immune disorder causing these infections Cystic fibrosis More thorough discussion in Chapter 2 Why eliminated from differential: the negative sweat test rules out this diagnosis Human immunodeficiency virus More thorough discussion in later case Why eliminated from differential: the negative HIV ELISA test rules out this diagnosis Leukopenia without immune deficiency Decreased lymphocyte or neutrophil (i.e., agranulocytosis) counts that are not attributable to an underlying immune disorder P.176 P 177
Table 6-11 Types of Congenital Immunodeficiency Disorders
Disease
Desc ription
Diagnosis
Treatment
Chromosomal deletion in 22q11
Tetany and fac ial
Calcium, vitamin D,
resulting in thymic and parathy roid
abnormalities on
thymic transplant, bone
hypoplasia, c ongenital heart
examination
marrow transplant,
disease, tetany, and abnormal facial
Decreased serum calcium
surgical correction of
structure
Evidence of congenital
heart abnormalities
Recurrent viral and fungal infections
heart disease
IVIG or prophylactic
occur because of insufficient T cells
CXR may show absence
antibiotics may be helpful
of thymic shadow
to prevent infections
T CELL DISORDERS
DiGeorge syndrome
Genetic screening can detect chromosomal abnormality
Chronic mucocutaneous candidiasis
Persistent infection of skin, mucous
Poor reaction to
Antifungal agents (e.g.,
membranes, and nails by Candida
cutaneous C. albicans
fluconazole)
albicans due to T cell deficiency
anergy test
Frequent associated adrenal
Possible decreased IgG
pathology
B CELL DISORDERS
X-linked agammaglobulinemia
Abnormal B cell differentiation
No B c ells in peripheral
IVIG, appropriate
resulting in low B cell and antibody
smear
antibiotics, supportive
levels
Low total immunoglobulin
pulmonary care
X-linked disorder with boys
levels
experiencing recurrent sinopulmonary infections after 6 months of age
IgA deficiency
Specific IgA deficiency due to an
Dec reased IgA with
Prophylactic antibiotics
abnormal immunoglobulin production
normal levels of other
IVIG is infrequently used
by B cells
immune globulins
because of an increased risk of anaphylaxis
Patients have increased incidence of respiratory and gastrointestinal infections and food allergies
Hyper-IgM disease
Common variable immunodeficiency
Defect in T cell CD40 ligand resulting
Decreased IgG and IgA,
IVIG, prophylactic
in a poor interaction with B cells that
inc reased IgM
antibiotics
leads to low IgG and excessive IgM
Possible decreased Hgb,
Bone marrow transplant
Infections by enc apsulated bac teria
Hct, platelets, and
(pulmonary and GI) are common
neutrophils
Autosomal disorder of B cell
Decreased but not
IVIG, appropriate
differentiation into plasma cells
absent immunoglobulin
antibiotics
leading to low immunoglobulin levels
levels
Patients experience recurrent
Poor response to
respiratory and GI infections
vaccines
beginning in second decade of life
Decreased CD4:CD8 T
Associated with increased risk of
cell ratio
malignant neoplasms and
Family history shows
autoimmune disorders
both men and w omen affec ted
COMBINED B AND T CELL DISORDERS
Severe combined immunodeficiency syndrome (SCID)
Absence of T cells and frequently B
Significantly decreased
Isolation, antibiotics,
cells resulting in severe immune
WBCs, T cell counts are
bone marrow
c ompromise
always low or absent,
transplant
Patients experience significant
decreased
No live or attenuated
recurrent infections by all types of
immunoglobulins
vac c ines should be
pathogens from an early age
administered
Frequently fatal at an early age
Wiskott-Aldrich syndrome
X-linked disorder of actin
Recurrent infections in
Splenec tomy , antibiotic
polymerization in hematopoietic cells
presence of ec zema and
prophylaxis, IVIG, bone
resulting in significant susceptibility
easy bleeding
marrow transplant
to enc apsulated bacteria and
Decreased platelets,
opportunistic pathogens
decreased IgM with
Associated with ec zema and
normal or high other
thromboc ytopenia
immune globulins Genetic analysis detects abnormal WA SP gene
Ataxia-telangiectasia
Autosomal recessive disorder causing
Telangiec tasia and
IVIG and prophylactic
c erebellar dysfunc tion, c utaneous
ataxia develop after
antibiotics may be
telangiec tasias, increased risk of
third year of life
helpful, but treatment is
cancer, and impaired WBC and IgA
Recurrent pulmonary
usually unable to limit
development
infections begin a few
disease progression
years later Decreased WBCs and IgA
PHA GOCY TIC CELL DISORDERS
Chronic granulomatous disease
Defect in neutrophils with impaired
Cutaneous, pulmonary,
Prophylactic antibiotics,
phagoc ytosis of bacteria, resulting in
and perirectal abscess
γ-interferon,
recurrent bacterial and fungal
formation
corticosteroids, bone
infections
Chronic
marrow transplant
lymphadenopathy Genetic analysis detects causative genetic mutations
Hyper-IgE disease
Defect in neutrophil chemotaxis, T cell
Increased IgE, increased
Prophylactic antibiotics
signaling, and overproduction of IgE
eosinophils
Skin hydration and
that results in c hronic dermatitis,
Defective chemotactic
emollient use
rec urrent skin absc esses, and
response of neutrophils
pulmonary infections
upon stimulation
Patients commonly have coarse facial features and retained primary teeth
Chediak-Higashi syndrome
Autosomal recessive dysfunction of
Large granules seen in
Prophylactic antibiotics,
intracellular protein transport in
granulocytes on
bone marrow transplant
neutrophils and other cell resulting in
peripheral smear
recurrent Staphylococcus aureus, streptococcal, Gram-negative bacteria, and fungal infections Associated with abnormal platelets, albinism, and neurologic dysfunc tion
Leukocyte adhesion deficiency (types 1 and 2)
Inability of neutrophils to leave
Increased serum
Prophylactic antibiotics
circulation due to abnormal leukocyte
neutrophils
Bone marrow transplant
integrins (type 1) or E-selectin (type
Defective chemotactic
needed in type 1 disease
2)
response of neutrophils
Type 2 disease treated
Recurrent bacterial infections of
upon stimulation
with fucose
upper respiratory tract and skin
supplementation
Short stature, abnormal facies, and cognitive impairment seen in type 2 disease
COMPLEMENT DISORDERS
Complement deficiencies
Multiple inherited deficiencies of one
Hemolytic complement
Appropriate antibiotics
or more complement components
tests are abnormal and
Treat autoimmune
resulting in recurrent bacterial
indicate problem in
disorders as needed
infections and predisposition to
pathway
autoimmune disorders (i.e., SLE)
Direct testing of components can detect exact deficiency
CXR, chest x-ray; GI, gastrointestinal; Hct, hematocrit; Hgb, hemoglobin; Ig, immunoglobulin; IVIG, intravenous immune globulin; SLE, systemic lupus erythematosus; TMP-SMX, trimethoprim-sulfamethoxazole; WBC, white blood cell.
P.178 May be due to diseases associated with an increased cortisol level, medications (e.g., chemotherapeutics, antithyroid drugs, trimethoprim-sulfamethoxazole [TMP-SMX]), radiation, viral infection (e.g., hepatitis, EBV), or aplastic anemia History: recurrent infections, weakness, chills, exposure to an appropriate causal factor Physical examination: fever Tests: decreased WBCs (either lymphocytes or neutrophils, depending on which cell line is affected) Treatment: Stop exposure to the offending agent Bone marrow transplant, granulocyte colony-stimulating factor, or corticosteroids may be required to induce repletion of the deficient cell line Antibiotic therapy should be used to treat any suspected infection Outcomes: prognosis is dependent on the ability to eliminate the inciting factor and restore the depleted leukocytes Why eliminated from differential: the absence of exposure to an obvious agent and the normal WBC count make this diagnosis less likely Common variable immunodeficiency More thorough discussion in Table 6-11 Why eliminated from differential: this diagnosis tends to occur later in childhood or adolescence, and the immunoglobulin levels tend to be higher than those seen in agammaglobulinemia, so the latter is a more likely diagnosis IgA deficiency More thorough discussion in Table 6-11 Why eliminated from differential: deficiencies in all of the immunoglobulins and not just IgA in this case rules out this diagnosis
Severe combined immunodeficiency syndrome (SCID) More thorough discussion in Table 6-11 Why eliminated from differential: the maintained presence of T lymphocytes rules out this diagnosis DiGeorge syndrome More thorough discussion in Table 6-11 Why eliminated from differential: the normal facial appearance, normal reflexes, and normal T lymphocyte numbers help to rule out this diagnosis Hyper-IgM disease More thorough discussion in Table 6-11 Why eliminated from differential: the undetectable IgM level in this case rules out this diagnosis Wiskott-Aldrich syndrome More thorough discussion in Table 6-11 Why eliminated from differential: the low levels of all immunoglobulins and not only IgM in this case rules out this diagnosis Fanconi anemia An autosomal recessive disorder associated with bone marrow failure, pancytopenia, and an increased risk of leukemia History: fatigue, dyspnea, frequent infections, Physical examination: short stature, abnormal skin pigmentation, thumb abnormalities, abnormal gonads, microcephaly, possible anatomical defects of any organ system Tests: Decreased Hgb, WBCs, and platelets Increased serum α-fetoprotein Chromosome analysis demonstrates multiple strand breakages Bone marrow biopsy shows hypocellularity Treatment: Antibiotics are required for infections Transfusions, bone marrow transplant, or hematopoietic growth factors may be required to induce hematopoietic cell production Androgens or corticosteroids may improve bone marrow activity P.179 Outcomes: prognosis is poor with death frequently occurring in childhood due to bone marrow failure or leukemia; aggressive use of blood products and marrow stimulation can prolong survival Why eliminated from differential: the normal cell count on the CBC rules out this diagnosis
Case 6-4: “He just started having problems breathing”
A 47-year-old man is brought intubated into the emergency department via ambulance after the acute onset of dyspnea. He is accompanied by his wife who is able to quickly describe the chain of events that led to this presentation. She says that she was having lunch with her husband during their respective lunch hours. While eating a salad, her husband began to complain of tightness in his chest and difficulty breathing and swallowing. He also began to break out in a red rash. The patient's wife says that this is the first time she has ever seen this happen. She denies that he mentioned any difficulty breathing, feeling ill, or chest pain prior to the onset of his symptoms. She thought that he was choking and tried to perform the Heimlich maneuver, which did not improve his symptoms. Emergency medical services was called because of his continued difficulty breathing, and he was immediately placed on oxygen via a face mask. During transport to the hospital, the emergency medical technicians (EMTs) were forced to intubate the patient because of further worsening in his ability to breathe. The EMTs note that this procedure was performed with minimal difficulty, but that the patient was notable for moderate tracheal and glottic swelling. The patient's wife says that he has a history of gastric reflux for which he takes ranitidine. She is unaware of any allergies that he has, but he avoids eating peanuts because his father experienced a bad reaction to them on one occasion. She says he drinks about six alcoholic beverages per week and does not use any other substances. On examination, the patient is sedated and intubated. He has a palpable red rash over his face and upper body. Auscultation of his heart and lungs detects no extra heart sounds and significant wheezing in all lung fields. His abdomen is soft with normal bowel sounds. The patient opens his eyes when addressed. The following vital signs are measured: T: 98.7°F, HR: 96 bpm, BP: 102/74 mm Hg, RR: 18 breaths/min while intubated
Differential Diagnosis Anaphylaxis, aspirated foreign body, myocardial infarction (MI), pulmonary embolism
Laboratory Data and Other Study Results Electrocardiogram (ECG): regular rate and rhythm; no abnormal wave morphology Arterial blood gas (ABG) (40% O2): pH: 7.38, partial pressure of oxygen (pO2): 228 mm Hg, partial pressure of carbon dioxide (pCO2): 43 mm Hg, bicarbonate (Bicarb): 24 mEq/L, oxygen saturation (O2 sat): 97%
Diagnosis Anaphylaxis
Treatment Administered Epinephrine, methylprednisolone, and diphenhydramine were immediately administered intravenously Albuterol was administered via the endotracheal tube P.180
Follow-up Following the administration of treatment, the patient's oxygen saturation improved to 100%, and the urticaria began to resolve The patient was able to be extubated in the emergency department and was able to be weaned from supplemental oxygen A follow-up CXR detected no infiltrates or unventilated areas The patient was admitted overnight for observation and was discharged to home the next day Follow-up allergy testing detected a strong reaction to peanuts; discussion with the restaurant revealed that they used peanut oil in their salad dressing
The patient was provided a portable subcutaneous epinephrine injector and instructed to avoid any food containing peanuts or peanut oil
The types of hypersensitivity reactions may be remembered by the mnemonic ACID: Anaphylactic, Complement-mediated, Immune complex-mediated, Delayed.
Steps to the Diagnosis Anaphylaxis A severe type I hypersensitivity reaction following exposure to an allergen (e.g., penicillin, insect stings, latex, certain foods) Hypersensitivity reactions are allergen-induced immune responses involving either cellular or humoral mechanisms (Table 6-12) History: symptoms occur 5 to 60 minutes after exposure and include skin tingling, pruritis, cough, chest tightness, difficulty swallowing and breathing (secondary to angioedema), and syncope Physical examination: tachycardia, urticaria, hypotension, arrhythmias Tests: Typically a clinical diagnosis in the acute setting Skin allergen testing or radioallergosorbent testing (RAST) can confirm a specific allergic response Increased histamine and tryptase levels are apparent immediately following exposure
Table 6-12 Types of Hypersensitivity Reactions
Type
I
Mediated By
Mec hanism
Examples
Treatment
IgE antibodies
Allergens react with
Allergic rhinitis, asthma,
Epinephrine, antihistamines,
attached to
antibodies to cause mast
anaphy laxis
leukotriene inhibitors, bronchodilators,
mast cells
c ell degranulation and
or corticosteroids used during an acute
histamine release
reaction; desensitization is considered to avoid recurrent reactions
II
III
IgM and IgG
Allergens react with
Drug-induc ed or
Anti-inflammatories or
antibodies
antibodies to initiate
immune hemolytic
immunosuppressive agents
c omplement c asc ade and
anemia, hemolytic disease
cell death
of the newborn
IgM and IgG
Antibodies form immune
Arthus reaction, serum
immune
c omplexes with allergens
sickness,
complexes
which are then deposited in
glomerulonephritis
Anti-inflammatories
tissue and initiate complement cascade
IV
T cells and
T c ells present allergens to
Transplant rejection,
Corticosteroids or other
macrophages
macrophages and secrete
allergic c ontac t
immunosuppressive agents
lymphokines that induce
dermatitis, PPD testing
macrophages to destroy surrounding tissue
Ig, immunoglobulin; PPD, purified protein derivative.
P.181 Treatment: Subcutaneous or IV epinephrine is administered to maintain blood pressure and decrease angioedema by inducing vasoconstriction Intubation must be considered if airway patency is at risk Antihistamines and corticosteroids are important to reduce the acute inflammatory reaction to the allergen Bronchodilators are used to decrease bronchoconstriction Continued vasopressor use may be required for recalcitrant hypotension Patient should be observed for at least 12 hours after the reaction because of the rare incidence of delayed rebound anaphylactic reactions in some patients Avoidance of the particular allergen is the key to avoiding recurrence; subcutaneous epinephrine injectors should be carried at all times to avoid a reaction to future exposures Desensitization therapy may be appropriate following recurrent episodes It can only be performed for IgE-mediated type I hypersensitivity reactions Anaphylactoid reactions (i.e., non-IgE-mediated mast cell degranulation) cannot be treated with desensitization Outcomes: mortality and morbidity correlates with delays in treatment and the inability to maintain a patent airway Clues to the diagnosis: History: reaction while eating, acute difficulty swallowing and breathing Physical: urticaria, wheezing, glottic edema Tests: noncontributory in the acute setting (postevent allergy testing helps to confirm the diagnosis) Aspirated foreign body More thorough discussion in Chapter 2 Why eliminated from differential: the lack of visualization of a foreign body during intubation and the presence of adequate oxygenation following the intubation make this diagnosis less likely Myocardial infarction More thorough discussion in Chapter 1 Why eliminated from differential: the normal-appearing ECG makes this diagnosis less likely Pulmonary embolism More thorough discussion in Chapter 2 Why eliminated from differential: the normal A-a gradient, adequate oxygen saturation following intubation,
and normal ECG all make this diagnosis unlikely
Case 6-5: “My face is horribly bruised” A 57-year-old woman is a scheduled overnight admission following cosmetic facial surgeries. Her surgeries were completed without incident. The morning following her procedures she awoke to find her face significantly ecchymotic and swollen. He bandages are also heavily stained with blood. A hematologist is consulted because there is no obvious explanation of why the patient has sustained such a degree of postoperative bleeding. She has mild pain at her surgical sites that has been well controlled with low-dose postoperative narcotics. She denies headaches, problems with her vision or hearing, or difficulty breathing. She denies any numbness or weakness. She denies any additional symptoms anywhere else on her body. She denies any past medical history or previous surgeries of any kind. She admits that she does bruise easily but never felt concerned by this finding. Upon further questioning, she also admits to a history of occasional heavy periods. She says that she has one child and that following this childbirth 30 years ago she had prolonged uterine bleeding that was determined to be due to uterine atony. She takes no medications. She denies any significant alcohol or tobacco use. On examination, she appears to be a well-nourished woman in no acute distress. Her face is significantly ecchymotic and moderately swollen. There is no active bleeding from her surgical sites. Fundoscopic examination is normal. Examination of her auditory canals is normal. Her oropharynx does not appear edematous. Her neck is supple and not swollen. Auscultation of her heart and lungs is normal. The following vital signs are measured: T: 98.9°F, HR: 82 bpm, BP: 133/85 mm Hg, RR: 18 breaths/min
Differential Diagnosis Thrombocytopenia, von Willebrand disease, vitamin K deficiency, hemophilia, disseminated intravascular coagulopathy
Laboratory Data and Other Study Results The following labs were collected preoperatively: CBC: WBC: 8.1, Hgb: 15.3, Plt: 387 Chem7: Na: 140 mEq/L, K: 4.2 mEq/L, Cl: 108 mEq/L, CO2: 27 mEq/L, BUN: 14 mg/dL, Cr: 0.6 mg/dL, Glu: 90 mg/dL Coags: PT 11.3 sec, INR: 1.0, PTT: 31.9 sec The following labs are initially ordered by the hematologist: CBC: WBC: 8.9, Hgb: 14.3, Plt: 376 Chem7: Na: 141 mEq/L, K: 4.1 mEq/L, Cl: 108 mEq/L, CO2: 28 mEq/L, BUN: 13 mg/dL, Cr: 0.6 mg/dL, Glu: 96 mg/dL Coags: PT: 11.5 sec, INR: 1.0, PTT: 32.6 sec von Willebrand factor antigen: 45% expected value Ristocetin cofactor activity: 98% expected value Coagulation factor VIII: 75% expected value Coagulation factor IX: 100% expected value Fibrinogen: 265 mg/dL
Diagnosis
von Willebrand disease, mild quantitative type
Treatment Administered The patient was given intranasal DDAVP (i.e., desmopressin acetate) Low-dose DDAVP was prescribed for home therapy
Follow-up The patient's ecchymoses resolved over the following 2 weeks The patient was advised to avoid aspirin (ASA) use and to schedule von Willebrand factor (vWF) infusions prior to any future surgery
Steps to the Diagnosis von Willebrand disease An autosomal dominant disease of deficient coagulation factor VIII and vWF, leading to abnormal clotting and platelet function vWF functions to mediate platelet adhesion to areas of vascular injury and stabilizes factor VIII Multiple degrees of the disease exist that range from mild quantitative defects to severe qualitative defects associated with severe symptoms History: easy bruising, mucosal bleeding (e.g., epistaxis), menorrhagia Physical examination: significant bruising, possible visible bleeding sites Tests: PTT may be normal or increased; bleeding time is increased but is infrequently measured because of its inconsistent reproducibility Decreased vWF and factor XIII antigens Decreased ristocetin cofactor activity is seen in qualitative defects Treatment: DDAVP is given during periods of minor bleeding vWF or factor VIII concentrate is recommended prior to surgery to prevent excessive bleeding ASA use should be avoided Outcomes: prognosis depends on disease severity; patients with milder forms typically lead normal lives, but those with severe cases require tight regulation of their coagulation ability Clues to the diagnosis: History: significant bruising and bleeding following surgery, history of easy bruising, menorrhagia, heavy postpartum bleeding Physical: significant ecchymoses Tests: decreased vWF, decreased factor VIII
Thrombocytopenia A decreased number of platelets (i.e., <150, 000), leading to an increased risk of hemorrhage Causes may be idiopathic, autoimmune, or due to medications or infection (Table 6-13) History: asymptomatic if mild, otherwise mucosal bleeding and easy bruising Physical examination: petechiae, purpura, multiple ecchymoses Tests: Decreased platelets (<150,000) Other helpful lab tests vary with each cause (Table 6-13) Blood smear shows low platelet numbers, small platelets, abnormal platelet granules, or neutrophilic granules, depending on the etiology Treatment: treatments vary with each etiology but may involve stopping an offending agent, performing a splenectomy, prescribing corticosteroids, or performing a bone marrow transplant (Table 6-13) Outcomes: prognosis depends on the ability to correct the platelet deficit, with lower mortality and morbidity occurring with easily correctable causes Why eliminated from differential: the normal platelet levels on both CBCs makes this diagnosis unlikely Vitamin K deficiency Inadequate vitamin K supply due to poor nutrition, malabsorption, or the eradication of vitamin K-producing GI flora following prolonged antibiotic use
Table 6-13 Causes of Thrombocytopenia
Cause
Pathology
Diagnosis
Treatment
Impaired production (e.g.,
Absent or reduc ed
Findings consistent with
Stop offending agent, treat
drugs, infection, aplastic
megakary oc ytes due to an
precipitating condition; bone
underlying disorder, bone marrow
anemia, folate/Vitamin B12
offending agent or abnormal
marrow biopsy is helpful for
transplantation
deficiency, alcohol)
megakary oc ytes due to
diagnosis
metabolic deficiency
Abnormal pooling
Splenic platelet sequestration
Splenomegaly (90% platelets may be
May not be required;
sequestered); normal bone marrow
splenectomy if symptomatic
biopsy
Heparin-induced
Development of antiplatelet
Diffuse thrombus formation and
Stop all heparin use; direct
thrombocytopenia
antibodies that cause widespread
sudden decrease in platelet level
thrombin inhibitors for thrombi
platelet destruction in response
(<50% normal); positive serotonin
to heparin therapy
release assay and heparin-induced platelet aggregation assay
Idiopathic
Autoimmune B-cell directed
Other explanations for low
Self-limited in children; adults
thrombocytopenia purpura
production of antiplatelet
platelets ruled-out; platelets
require cortico-steroids, delayed
antibodies
c ommonly <50,000
splenectomy, IV immunoglobulin, plasmapheresis, or recombinant factor VIIa
Thrombotic
Diffuse platelet aggregation due
Neurologic defic its, ac ute renal
Corticosteroids, plasma-pheresis,
thrombocytopenic purpura-
to autoantibodies against an
failure, fever; hemolytic anemia,
FFP
hemolytic uremic syndrome
preventative enzyme; associated
thrombocytopenia without severe
with endothelial injury and
bleeding, and increased
Escherichia coli O157:H7 infection
reticulocyte count
Development of antiphospholipid
During pregnancy; presence of
Anticoagulation with heparin and
antibodies during pregnancy
antiphospholipid or lupus
warfarin, hydroxychloroquine
leading to arterial and venous
anticoagulant antibodies
Antiphospholipid syndrome
thrombosis
HELLP syndrome
Sequela of eclampsia associated
During pregnancy; HTN; increased
Induce delivery if fetus >34
with elevated liver enzymes and
LFTs and decreased Hgb;
weeks gestation; anti-HTN drugs
hemolytic anemia
schistocytes on blood smear
and corti-costeroids to speed fetal lung maturity if preterm
ASA, acetylsalicylic acid; FFP, fresh frozen plasma; Hct, hematocrit; HELLP, hemolysis, elevated liver enzymes, low platelets; HTN, hypertension; IV, intravenous; LDH, lactate dehydrogenase; LFT, liver function test; RBC, red blood cell.
Vitamin K is vital to the synthesis of coagulation factors II, VII, IX, and X History: easy bruising, mucosal bleeding, melena, hematuria Physical examination: multiple ecchymoses, petechiae Tests: increased PT and INR; decreased factor II, VII, IX, and X levels Treatment: FFP may be given acutely to correct factor deficiencies Vitamin K through injection or ingestion can restore levels A diet with adequate leafy, green vegetables is a good source of vitamin K Outcomes: prognosis is excellent with timely vitamin supplementation Why eliminated from differential: the normal PT, INR, and factor IX levels rule out this diagnosis Hemophilia X-linked recessive disorder of coagulation factor VIII (i.e., hemophilia A) or IX (i.e., hemophilia B), causing coagulopathy and several comorbidities History: uncontrolled bleeding following minimal trauma or occurring spontaneously, excessive bleeding following any surgical or dental procedure, recurrent hemarthroses (i.e., intra-articular bleeding), intramuscular hematomas, possible GI or urinary bleeding Physical examination: possible visible active bleeding, joint deformity, jaundice Tests: normal PT, increased PTT, and normal bleeding time; decreased factor VIII or IX antigen Treatment: Factor VIII or IX replacement Aggressive factor replacement should be performed for hemarthroses and intracranial bleeds DDAVP may help increase factor VIII production
Transfusions are frequently required for large blood losses Outcomes: Prognosis is most favorable when factor replacement is carefully used to maintain adequate factor concentrations Recurrent hemarthroses may require eventual joint replacement surgery due to chronic degenerative changes Transmission of infection (e.g., HIV) from numerous blood product transfusions is an appreciable risk, especially for patients who have received such products prior to current screening protocols Intracranial bleeds can carry severe neurologic sequelae Death may occur from uncontrolled bleeding Why eliminated from differential: the normal PTT and factor VIII and IX levels rule out this diagnosis Disseminated intravascular coagulation (DIC) Widespread abnormal coagulation due to sepsis, severe trauma, neoplasm, or obstetric complications Extensive activation of the clotting cascade by endothelial tissue factor release (caused by underlying pathology) causes a coagulopathic state Uncontrolled bleeding occurs because pathologic clotting exhausts the supply of platelets and coagulation factors History: appropriate history of a precipitating condition, uncontrolled bleeding from surgical sites and wounds, hematemesis, dyspnea Physical examination: jaundice, digital cyanosis, hypotension, tachycardia, possible signs of neurologic injury or renal failure, possible shock Tests: Decreased Hgb and platelets Increased PT, INR, and PTT Increased D-dimer and decreased fibrinogen Blood smear shows schistocytes and few platelets Treatment: Treatment of the underlying disorder is imperative Multiple transfusion products may be required to replace deficient RBCs, platelets, and coagulation factors Heparin may be required to prevent widespread thrombosis Outcomes: prognosis is poor with a high mortality rate and considerable morbidities due to widespread hemorrhage and thrombosis Why eliminated from differential: the normal CBC, coagulation labs, and fibrinogen rule out this diagnosis
Signs of thrombotic thrombocytopenic purpurahemolytic uremic syndrome (TTP-HUS) may be remembered by the mnemonic “Nasty Fever Torched His Kidneys”: Neurologic deficits, Fever, Thrombocytopenia, Hemolytic anemia, Kidney failure.
vWF and factor VIII are the only clotting factors not synthesized by the liver and remain at normal levels while other factor levels decrease in liver failure.
Patients using an excessive dose of warfarin may present with a clinical picture similar to that of vitamin K deficiency.
Hemophiliacs tend to not develop significant bleeds unless they have <5% clotting activity.
Case 6-6: “My cousin is wasting away” A 34-year-old man is brought to an internist for an initial appointment by his cousin. The patient's cousin says that the patient has been progressively losing weight over the past year and has been ill several times over this period. This weight loss has been unintentional. The patient notes that he has had upper respiratory infections multiple times in the past year that never seem to completely resolve. He has multiple painful oral ulcerations and has developed cold sores on his lips. He has noticed some white spots in the back of his mouth over the past several months. He feels tired all of the time and is generally weak. He frequently wakes in the middle of the night in a cold sweat and with chills. About 1 week prior to this appointment he developed intermittent abdominal pain and frequent episodes of nonbloody diarrhea. He took an over-the-counter antidiarrhea medicine, but his diarrhea has not completely resolved. He denies emesis, numbness, hemoptysis, myalgias, or dyspnea. He lived most of his life in sub-Saharan Africa and moved to the United States 2 years ago. He denies any past medical history and takes no medications other than over-the-counters treatments for his various symptoms. He denies any history of intravenous drug use or any substance use other than an occasional beer. He says that he had multiple heterosexual partners during his 20s but that he has not had intercourse with anyone since arriving in this country. On examination, he is a thin-appearing man in no acute distress. His eyes appear normal to gross and fundoscopic examination. He has two vesicles on the outer portion of his lips. He has three ulcers on the inside of his cheeks. He has several white spots in the back of his tongue and pharynx. His throat is not inflamed, and no exudates are visible. He has no lymphadenopathy. Auscultation of his heart and lungs finds some mild congestion in all lung fields with minimal rhonchi and normal heart sounds. His abdomen is soft with hyperactive bowel sounds and is diffusely tender. He has no gross blood on rectal examination. His has a generalized decrease in strength, and his sensation is normal. He has normal coordination. The following vital signs are measured: T: 99.2°F, HR: 98 bpm, BP: 108/78 mm Hg, RR: 20 breaths/min
Differential Diagnosis Herpes simplex, HIV, tuberculosis, sarcoidosis, infectious diarrhea, malaria, sepsis, leukemia, lymphoma
Laboratory Data and Other Study Results CBC: WBC: 2.1, Hgb: 13.3, Plt: 148 Chem10: Na: 144 mEq/L, K: 3.7 mEq/L, Cl: 105 mEq/L, CO2: 29 mEq/L, BUN: 21 mg/dL, Cr: 1.0 mg/dL, Glu: 95 mg/dL, Mg: 2.1 mg/dL, Ca: 10.3 mg/dL, Phos: 3.8 mg/dL LFTs: AlkPhos: 70 U/L, ALT: 53 U/L, AST: 42 U/L, TBili: 0.8 mg/dL, DBili: 0.3 mg/dL Blood smear: few leukocytes; normal morphology of all cells; no visible bacteria or parasites Blood culture: negative Fecal leukocytes: negative Stool culture: no organisms visible on Gram stain; many round oocytes visible on acid-fast stain Tzanck smear of oral vesicles: multinucleated giant cells
Purified protein derivative (PPD): no induration HIV ELISA: positive CXR: minimal diffuse infiltration of both lung fields; no consolidation or effusions; no hilar lymphadenopathy The following repeat study is ordered following receipt of the above results: HIV ELISA: positive Finally the following tests are ordered: HIV Western blot: positive HIV viral load: 125,000/mL CD4 count: 187/mL
Diagnosis HIV with associated herpes simplex, oral thrush, and Cryptosporidiosis
Treatment Administered The patient was started on an antiviral regimen of efavirenz, emtricitabine, and tenofovir via a combination formulation Paromomycin was prescribed for the Cryptosporidium diarrhea Acyclovir was prescribed for the herpes simplex infection Nystatin suspension was prescribed for the oral thrush The patient was referred to an HIV support group
Follow-up By 6 months following the initiation of treatment, the patient's viral load had decreased to <50 mL, and his CD4 count had increased to 384 mL The patient's thrush and diarrhea resolved over 1 month following the initiation of treatment, and his herpes simplex infection went into remission Three years after the initiation of antiretroviral therapy, the patient's CD4 count had increased to 503 mL, and his viral load was stable at <50 mL
Steps to the Diagnosis Human immunodeficiency virus A RNA retrovirus (HIV1 and HIV2 are most common strains) that infects CD4 lymphocytes (i.e., helper T cells) and destroys them, eventually leading to acquired immune deficiency syndrome (AIDS) Both strains are transmitted in the same manner, share the same risks for opportunistic infections, and are treated in the same manner Compared to HIV1, HIV2 progresses more slowly, is less infectious in the early disease course, is more infectious in the late disease course, and is less common in the United States The HIV virus uses a reverse transcriptase to incorporate its genetic material into the host genome and to
produce copies of the viral DNA Transmission occurs via bodily fluids (e.g., blood, semen, vaginal secretions, breast milk) Risk factors: homosexual or bisexual males, intravenous drug abuse (IVDA), blood transfusions prior to mid-1980s (e.g., hemophiliacs), multiple sexual partners, heterosexual partners of other high-risk individuals, infants born to infected mothers, accidental exposure to bodily fluids (e.g., needle sticks, fluid splashes) among health care workers (low probability but possible) History: Acutely, myalgias, nausea, vomiting, diarrhea, fatigue, sore throat, and weight loss developing 2 to 4 weeks after the exposure and lasting 2 weeks Following the acute infection stage, the patient enters a latent phase with few or no symptoms Once AIDS develops, patient exhibits AIDS-defining illnesses and opportunistic infections, weight loss, night sweats, and dementia (Table 6-14) Physical examination: Acutely, mucosal ulcers, fever, lymphadenopathy, diffuse light red rash Physical findings in AIDS represent the syndrome-defining illnesses and infections
Table 6-14 Opportunistic Infections, Neoplasms, and Defining Illnesses Seen in Acquired Immune Deficiency Syndrome
Prophylaxis Condition/Infec tion
AIDS dementia
Bacterial pneumonia
Indic ated
Undefined
CD4 >200
(e.g., Streptococcus
Symptoms/Signs of Prophylaxis
Ac tive Infec tion
Treatment of Diagnostic Tests
Ac tive Infec tion
Confusion, mental
Clinical diagnosis,
Optimize
status changes,
elevated β-2
anti-retroviral
generalized
microglobulin in
therapy
neurologic
CSF, cerebral
sy mptoms including
atrophy on CT or
tremor
MRI
Pneumococcal
Rapid onset,
Gram stain, lobar
Cephalosporins,
vaccine
productive cough,
consoli-dation on
β-lactams, or
high fevers
CXR
fluoroquinolones
Dysphagia,
Endoscopy with
Fluconazole,
odynophagia
biopsy, Gram stain
itraconazole, or
on lesion scrapings
nystatin
Headache, confusion,
Positive
Pyrimethamine,
possible foc al
toxoplasma IgG
leucovorin, and
neurologic
antibody , ring
sulfadiazine
sy mptoms
enhancing lesions
—
pneumoniae, Haemophilus influenza, Nocardia)
Candida esophagitis
Recurrent
Fluconazole
infection
Cerebral toxoplasmosis
CD4 <100
TMP-SMX
on CT or MRI
Coccidioidomycosis
Prior infection
Fluconazole
Cough, fever, dyspnea
Bilateral
Amphotericin B,
reticulonodular
fluconazole, or
infiltrates on CXR,
itraconazole
positive antibody screen
Cryptococcal
CD4 <50
Fluconazole
meningitis
Headache, neck
Elevated pressure
Amphotericin B
stiffness, fever,
on lumbar
or fluconazole
mental status changes
puncture, y east seen w ith India ink stain of CSF, positive c ryptoc oc c al antigen in CSF or serum
Cytomegalovirus (CMV)
CD4 <50 and
Ganciclovir
positive
V ision loss,
Viral titer, yellow
Ganciclovir or
esophagitis, diarrhea
infiltrates with
valganciclovir
CMV
hemorrhage on
antibodies
fundoscopic examination
Herpes simplex
Recurrent
Acyclovir or
Shingles, oral or
Tzanck smear, viral
Famciclovir,
infection
famciclovir
genital lesions
culture
valaciclovir, or acyclovir
Histoplasmosis
Human papilloma virus
CD4 <100
Undefined
Itraconazole
—
(HPV)
Abdominal pain, GI
Bilateral infiltrates
Amphotericin B
bleeding, skin lesions,
on CXR, positive
or itraconazole
dyspnea, meningitis
antigen test
Painless papular
Detected by
Podofilox or
lesions on genitals
screening
imiquimod
Papanicolaou smear, biopsy confirms diagnosis
Kaposi sarcoma
Undefined
—
Biopsy of lesions
—
Headache, confusion,
Lesion seen on CT
Chemotherapy,
possible foc al
or MRI, positive
radi ation
neurologic
biopsy
therapy
Blood cultures
Clarithromycin
Purple subc utaneous nodules on face, chest, or extremities
Lymphoma (CNS or
Undefined
—
non-Hodgkin's)
sy mptoms
Mycobacterium avium
CD4 <50
complex
Azithromycin
Fatigue, weight loss,
or
fever, diarrhea,
clarithromycin
abdominal pain,
and ethambutol
lymphadenopathy, hepatosplenomegaly
Prolonged diarrhea,
Stool culture,
Optimization of
(e.g., Isospora,
malaise, weight loss,
parasite evaluation
anti-retroviral
Strongyloides,
abdominal pain
Parasitic diarrhea
Undefined
—
therapy, paromomycin
Cryptosporidium)
Pneumocystis carinii
CD4 <200 or
Gradual onset,
Bilateral infiltrates
TMP-SMX,
pneumonia
oral Candida
nonpro-duc tive
on CXR, increased
possible dapsone
infection
c ough, dyspnea on
LDH, sputum Gram
exertion, fever
stain
Ataxia, motor deficits,
Positive PCR for JC
Optimize
mental status changes
virus DNA
anti-retroviral
Progressive multifocal
TMP-SMX
Undefined
leukoencephalopathy
therapy
(i.e., JC virus)
Tuberculosis
PPD >5 mm,
Isoniazid and
Cough, night sweats,
Ac id-fast bac illi,
Isoniazid,
contact
pyridoxine
weight loss, fever
cavitary defects
rifampin,
with
and hilar
pyrazinamide,
infected
adeno-pathy on
and ethambutol
person, or
CXR, positive PPD
prior
(must be checked
untreated
with anergy test)
disease
Varicella-zoster virus
Contact
Varicella-
Diffuse vesicular
Tzanck smear, viral
Acyclovir,
with
zoster immune
lesions, possible
culture
valaciclovir, or
infected
globulin
pneumonia
famciclovir
person
Wasting syndrome
Undefined
Weight loss >10% of
Clinical diagnosis,
Exercise,
baseline weight,
EMG suggesting
corticosteroids
chronic diarrhea,
peripheral nerve
chronic weakness,
dysfunction
fever
AIDS, acquired immune deficiency syndrome; CNS, central nervous system; CSF, cerebral spinal fluid; CT, computed tomography; CXR, chest x-ray; EMG, electromyograph; GI, gastrointestinal; Ig, immunoglobulin; LDH, lactate dehydrogenase; MRI, magnetic resonance imaging; PCR, polymerase chain reaction; PPD, purified protein derivative of tuberculin; TMP-SMX, trimethoprim-sulfamethoxazole.
Tests: Rapid serologic tests are available as an initial screening tool, but formal serologic analyses are required to confirm a positive result ELISA detects HIV antibodies and is 99% sensitive; if positive, a repeat ELISA should be performed following a positive initial test (Figure 6-5) If two positive ELISAs are recorded, a Western blot (lower sensitivity but high specificity) analysis is performed on the patient's serum to rule out false positives The CD4 T lymphocyte count is used to track the degree of immunodeficiency due to the disease; AIDS is defined as a CD4 count <200 cells per mL The disease viral load is also a useful marker of disease progression and response to therapy; because of the lag time in antibody production following exposure, it should be used as a diagnostic tool in patients with a recent suspected exposure Other lab findings include decreased WBCs during the initial and progressed infection, increased aminotransferases, and mildly decreased Hgb and platelets Treatment: Antiretroviral therapy should be initiated for patients with a CD4 count <350 mL, viral load >20,000 mL, HIV-associated nephropathy, or the symptomatic development of opportunistic disease (Table 6-15) Pregnant women should be treated with zidovudine and lamivudine to reduce the risk of transmission to the fetus; viral transmission rarely occurs when the maternal viral load is <1,000 mL
Figure 6-5 Serologic profile of human immunodeficiency virus infection. (See color image.)
Table 6-15 Antiviral Medications Used in Treatment of Human Immunodeficiency Virus
Drug Class
Mec hanism
Drug Examples
Abacavir
A dverse Effec ts
Possible hypersensitivity, fever, rash, nausea, vomiting,
Nucleoside reverse
Inhibit production of viral genome;
transcriptase
prevent incorporation of viral DNA
fatigue, anorexia, sore throat, cough, dyspnea, rare lactic
inhibitors
into host genome through reverse
acidosis with hepatic steatosis
transcriptase inhibition
Didanosine
Pancreatitis, peripheral neuropathy, nausea, rare lactic acidosis with hepatic steatosis
Lamivudine
Zidovudine
Rare lactic acidosis with hepatic steatosis
Bone marrow suppression, anemia, neutropenia, nausea, vomiting, headache, insomnia, weakness, rare lactic acidosis with hepatic steatosis
Tenofovir
Weakness, headache, diarrhea, nausea, vomiting, renal insufficiency, rare lactic acidosis with hepatic steatosis
Nonnucleoside
Inhibit reverse transcriptase activity
reverse
to prevent viral replication
Emtricitabine
Skin discoloration, rare lactic acidosis with hepatic steatosis
Delavirdine
Rash, headache, increased liver transaminases
Efavirenz
Rash, dizziness, confusion, mental status changes, insomnia,
transcriptase inhibitors agitation, increased liver transaminases
Protease inhibitors
Interferes with viral replication to
Nevirapine
Rash, possible hypersensitivity reaction, possible hepatitis
Indinavir
Nephrolithiasis, nausea, vomiting, headache, weakness,
cause production of nonfunctional
blurred vision, dizziness, rash, metallic taste in mouth,
viruses
alopecia, thrombocytopenia, hemolytic anemia, indirect hyperbilirubinemia, hyperlipidemia, hyperglycemia
Nelfinavir
Diarrhea, hyperglycemia, hyperlipidemia, increased liver transaminases
Ritonavir
Nausea, vomiting, diarrhea, paresthesias, weakness, altered taste, hyperlipidemia, hypertriglyceridemia, hepatitis, hyperglycemia
Atazanavir
Indirect hyperbilirubinemia, hyperglycemia, nephrolithiasis
Lopinavir
Nausea, vomiting, diarrhea, paresthesias, weakness, altered taste, hyperlipidemia, hypertriglyceridemia, hepatitis, hyperglycemia
Fosamprenavir
Rash, nausea, vomiting, diarrhea, headache, hyperlipidemia, hyperglycemia, increased liver transaminases
Newborns to HIV-positive mothers should be given zidovudine for 6 weeks after birth and should be tested for the presence of viral genetic material (anti-HIV antibodies will always be present in these children) in the initial 6 months of life Treatment of acute infection with normal laboratory values may prolong immune function, but no studies have supplied proof of benefit Prophylactic reverse transcriptase inhibitors are started immediately in health care workers with an appreciable risk of transmission due to a needle stick (actual risk is 0.3%) and HIV antibody screening is performed at the time of exposure, 6 weeks, 3 months, and 6 months afterward to determine if transmission occurred; treatment is continued until repeat testing shows that transmission did not occur Highly active antiretroviral treatment (HAART) is the recommended approach to treating patients meeting the criteria for therapy who have not been previously treated The initial drug regimen consists of two nucleoside reverse transcriptase inhibitors and either a protease inhibitor or a nonnucleoside reverse transcriptase inhibitor Low-dose ritonavir can be added to the initial regimen to increase the protease inhibitor activity Combination therapy (i.e., multiple drugs combined in one pill) will decrease the number of pills taken at one time and will help avoid dosing schedule mishaps Indications for changing the antiretroviral regimen include a viral load >400 mL after 24 weeks of treatment, a viral load >50 mL after 48 weeks of treatment, detectable HIV RNA after prior suppression, drug toxicity, and poor compliance Patients with a suboptimal decrease in the viral load should be tested for viral drug resistance and reviewed for drug interactions to help guide changes to the drug regimen Poor compliance may be approached by decreasing the complexity of the regimen (i.e., using combination pills) or enlisting family and friends to assist the patient
Antibiotic prophylaxis for opportunistic infections is indicated at certain CD4 count levels; likewise, these infections are treated as they occur (Table 6-14) Outcomes: HAART drug regimens have decreased mortality rates and have made HIV more of a chronic disease The average decrease in life expectancy in patients with HIV is 35 years, and the average lifespan of a patient after being diagnosed with HIV is 24 years The average survival in a patient with <50 CD4 cells per mL is 12 months Clues to the diagnosis: History: weight loss, night sweats, chills, multiple sexual partners in sub-Saharan Africa, delayed recovery from illness, diarrhea, cold sores Physical: oral thrush, mucosal vesicles Tests: positive HIV ELISA and Western blot tests, evidence of herpes simplex and Cryptosporidium infections, decreased WBC count Herpes simplex More thorough discussion in Chapter 10 Why eliminated from differential: the lab test show that herpes infection is present, but this diagnosis does not explain the full spectrum of findings seen in this patient Tuberculosis More thorough discussion in Chapter 2 Why eliminated from differential: the negative PPD and the absence of characteristic findings on the CXR make this diagnosis unlikely Sarcoidosis More thorough discussion in Chapter 2 Why eliminated from differential: the absence of hypercalcemia and hilar adenopathy on the CXR make this diagnosis unlikely Infectious diarrhea More thorough discussion in Chapter 3 Why eliminated from differential: the lab tests show that infection is present, but this diagnosis does not explain the full spectrum of findings seen in this patient Malaria Parasitic infection by Plasmodium spp. (i.e., P. vivax, P. falciparum, P. ovale, P. malariae) transmitted by the Anopheles mosquito Although rare in the United States, malaria is very common in other countries such as sub-Saharan Africa History: chills, diaphoresis, headaches, myalgias, fatigue, nausea, abdominal pain, vomiting, diarrhea Physical examination: periodic fever at 1- to 3-day intervals, splenomegaly, possible neurologic, renal, and mental impairments
Tests: polymerase chain reaction (PCR) detection of Plasmodium is highly sensitive; blood smear will show Plasmodium infiltration of RBCs (Color Figure 6-9) Treatment: Antimalarials (e.g., chloroquine) are used to destroy Plasmodium Atovaquone-proquanil or mefloquine is used for chloroquine-resistant P. falciparum Travelers to endemic areas for malaria should take prophylactic antimalarials Outcomes: prognosis is typically good once antimalarials are administered; infection with P. falciparum carries a poor prognosis if not treated Why eliminated from differential: the absence of Plasmodium on the blood smear makes this diagnosis less likely Sepsis Bacteremia with an associated excessive systemic inflammatory response, leading to global tissue hypoxia Common community-acquired pathogens include Streptococcus, Staphylococcus, Escherichia coli, Klebsiella, Pseudomonas, and Neisseria meningitides Common nosocomial pathogens include Staphylococcus, Gram-negative bacilli, anaerobes, Pseudomonas, and Candida species History: malaise, chills, nausea, vomiting Physical examination: fever, tachycardia, tachypnea, hypotension, petechiae, cold extremities Tests: Increased (over 12,000 mL) or decreased (below 4,000 mL) serum WBCs Positive urine, blood, or sputum cultures Increased PT and PTT and decreased fibrinogen suggesting DIC CXR may demonstrate pneumonia
Color Figure 6-9 Blood smear from a patient with Plasmodium infection showing two microgametes with large nuclei and loose chromatin (arrow) and several red blood cells with intracellular rings and eosinophilic granules signifying infiltration.
Treatment: Confirm that a patient has adequate oxygenation (secure the airway) Maintain tissue perfusion with vasopressors, IV hydration, inotropes, and blood products Broad-spectrum antibiotic are initially prescribed and switched to organism-specific drugs when culture results are received Catheters or IVs that are a potential source of infection are changed Glycemic control is maintained to improve the tissue response to medication Recombinant human activated protein C may be beneficial in patients with multiorgan failure due to septic shock Outcomes: prognosis is generally guarded and poor in cases of septic shock; mortality in the latter patient group may be 50% Why eliminated from differential: the negative blood cultures make this diagnosis unlikely Leukemia/lymphoma More thorough discussion in later case Why eliminated from differential: the normal appearance of the leukocytes on the blood smear makes this diagnosis less likely, and the serologic testing for HIV confirms that it is the true diagnosis
HIV infection has greatest prevalence in sub-Saharan Africa, where transmission is typically through heterosexual contact.
It may take up to 6 months for HIV antibodies to appear in the serum.
HIV-positive mothers should not breast feed to reduce risk of transmission.
Compliance with antiretroviral therapy is vital to delaying HIV progression, and the significant side effects associated with antiretroviral drugs are the major deterrents to good compliance.
Periodic testing of the patient's viral load and CD4 count are important for measuring responses to treatment and dictating the plan of therapy.
S. aureus is a common cause of sepsis in intravenous drug abusers.
Encapsulated organisms are a more common cause of sepsis in asplenic patients (e.g., sickle cell disease) than in other patients.
Do not start antibiotics until after the first blood culture has been collected to avoid false-negative cultures.
Case 6-7: “All of my blood cells are low” A 65-year-old man is referred to a hematology-oncology practice for evaluation of pancytopenia. The patient was initially evaluated by his primary care physician for a 1-month history of constant fatigue. A routine laboratory evaluation found him to be mildly pancytopenic. Currently the patient reports that he continues to feel fatigued and occasionally feels short of breath. He says that he feels like he has had a persistent upper respiratory infection for the past 2 months with chest congestion and a nonproductive cough. He also notes that he has developed multiple bruises from minor traumas, and his arms and legs ache frequently. He denies having had these symptoms previously and feels that they have gradually worsened over the past month. He has a history of HTN and depression for which he takes losartan and fluoxetine. He denies any surgical history. He denies any substance use and has been in a monogamous relationship for 45 years. On examination, he is mildly pale but is in no acute distress. He is notable for petechiae around his mucosal surfaces. He has no lymphadenopathy. Auscultation of his lungs and heart detects mild rhonchi and a soft systolic murmur. His abdomen is soft and nontender with a palpable spleen. His neurologic examination is normal. The following vital signs are measured: T: 100.5°F, HR: 90 bpm, BP: 105/82 mm Hg, RR: 22 breaths/min
Differential Diagnosis Lymphoma, acute myelogenous leukemia, acute lymphocytic leukemia, chronic lymphocytic leukemia, chronic myelogenous leukemia, hairy cell leukemia, aplastic anemia, polycythemia vera, agranulocytosis, HIV
Laboratory Data and Other Study Results The following studies were performed by the primary care physician: CBC: WBC: 3.1, Hgb: 11.5, Plt: 128 Chem10: Na: 138 mEq/L, K: 3.9 mEq/L, Cl: 109 mEq/L, CO2: 30 mEq/L, BUN: 18 mg/dL, Cr: 1.1 mg/dL, Glu: 89 mg/dL, Mg: 2.1 mg/dL, Ca: 9.9 mg/dL, Phos: 3.5 mg/dL LFTs: AlkPhos: 128 U/L, ALT: 59 U/L, AST: 40 U/L, TBili: 0.9 mg/dL, DBili: 0.4 mg/dL CXR: increased bronchial markings; no significant infiltrates, effusions, or lymphadenopathy The following studies are ordered by the hematologist: HIV ELISA: negative Blood smear: several large myeloblastic leukocytes, some with Auer rods and notched nuclei; normal RBC morphology Bone marrow biopsy: high proportion of blasts staining with myeloperoxidase
Diagnosis Acute myelogenous leukemia
Treatment Administered Following testing to confirm cardiac, renal, and hepatic stability and cytogenic analysis to determine the type of acute myelogenous leukemia (AML), induction chemotherapy with anthracycline and arabinosylcytosine was initiated
The patient was admitted for intravenous antibiotics until his fever ceased
Follow-up The patient entered remission as determined by improved hematopoietic cell counts, improved histological appearance of his blood smears, and repeat bone marrow biopsy Eight months after entering remission the patient developed new significant fatigue and was determined to have a recurrence of his condition Induction therapy was repeated, but the patient did not enter remission Bone marrow transplant was considered, but prior to this procedure being performed, the patient developed a severe case of pneumonia and died following admission to the hospital
Steps to the Diagnosis Acute myelogenous leukemia A hematopoietic malignancy characterized by the overproliferation of myeloid cells in bone marrow All ages are affected, but the prevalence increases with age History: fatigue, easy bruising, dyspnea, frequent infections, arthralgias Physical examination: fever, pallor, hepatosplenomegaly, petechiae, mucosal bleeding, ocular hemorrhages Tests: Decreased Hgb, WBCs, and platelets Blood smear shows large myeloblasts with notched nuclei and Auer rods (Color Figure 6-10) Bone marrow biopsy shows numerous blasts of a myeloid origin that stain with myeloperoxidase Treatment: chemotherapy is guided by a cytogenic analysis to determine the subtype of AML; bone marrow biopsy is considered in young patients and in patients with a recurrence Outcomes: Only 50% of patients will go into remission following chemotherapy Relapse is common, and the prognosis is poor for these patients Death usually occurs as a result of infection or DIC Clues to the diagnosis: History: fatigue, prolonged upper respiratory infection, easy bruising Physical: pallor, splenomegaly, petechiae Tests: pancytopenia, characteristic cells on blood smear, blasts on bone marrow biopsy Acute lymphocytic leukemia (ALL) A malignant proliferation of immature cells of a lymphoid origin Most common in children between the ages of 2 and 5 years Whites are affected more frequently than other races
History: bone pain, frequent infections, fatigue, dyspnea on exertion, easy bruising
Color Figure 6-10 Acute myelogenous leukemia with monocytic differentiation. Note the prominent waxy nucleoli, large size of blasts, and presence of Auer rods.
Color Figure 6-11 Acute lymphocytic leukemia. Note the lymphoblasts with irregular nuclei and prominent nucleoli.
Physical examination: fever, pallor, petechiae, hepatosplenomegaly, lymphadenopathy Tests: Decreased Hgb, WBCs, and platelets Increased uric acid and LDH Copies of the Philadelphia chromosome (i.e., translocation of chromosomes 9 and 22 in the BCR-ABL genes) are found in 15% of adult cases Blood smear and bone marrow biopsy will both show numerous blasts (Color Figure 6-11)
Treatment: induction chemotherapy is performed initially, followed by maintenance chemotherapy; bone marrow transplant is considered in children Outcomes: the 5-year survival rate is 85% in children but lower in adults; the presence of a Philadelphia chromosome carries a poor prognosis Why eliminated from differential: the characteristic appearance of the blood smear and the age of the patient make this diagnosis less likely Chronic myelogenous leukemia A malignant proliferation of mature myeloid cells in middle-aged adults Appears to be related to prior radiation exposure in some cases Tends to follow a stable course for several years before progressing into a blast crisis (i.e., rapid worsening of the disease) that is typically fatal History: asymptomatic for most of the disease course followed by the development of bone pain, fatigue, weight loss, and night sweats Physical examination: fever, splenomegaly Tests: Increased WBCs (>100,000 µL) with a high proportion of neutrophils Decreased leukocyte alanine phosphatase Cytogenic analysis demonstrates the Philadelphia chromosome or a BCR-ABL fusion gene Bone marrow biopsy demonstrates granulocyte hyperplasia Treatment: chemotherapy (frequently with imatinib); bone marrow transplant is considered in children Outcomes: the median survival from the time of diagnosis is <6 years; the development of a blast crisis is frequently fatal Why eliminated from differential: the characteristic appearance of the blood smear and the lack of abundant granulocytes on the bone marrow biopsy make this diagnosis less likely Chronic lymphocytic leukemia (CLL) Malignant proliferation of mature B lymphocytes Typically in patients >65 years old
Color Figure 6-12 Chronic lymphocytic leukemia. Note the small lymphocytes of a comparable size to nearby red blood cells and the presence of smudge cells (fragile lymphocytes disrupted during smear preparation) in upper portion of image.
History: may be asymptomatic or may exhibit fatigue, frequent infections, and night sweats Physical examination: fevers, lymphadenopathy, hepatosplenomegaly Tests: Increased WBCs (>100,000 µL) Blood smear shows numerous small lymphocytes and smudge cells (Color Figure 6-12) Bone marrow biopsy shows lymphocytic infiltration Treatment: Supportive care is administered for the stable portions of the disease Chemotherapy is administered for disease progression, and radiation therapy may be performed for bulky lymphoid masses Splenectomy is frequently performed for splenomegaly Outcomes: malignant B cells may form autoantibodies that lead to severe hemolytic anemia; the disease may take an indolent course or may be aggressive with a high mortality rate within 4 years of the diagnosis Why eliminated from differential: the blood smear and bone marrow biopsy appearances make this diagnosis less likely Hairy cell leukemia A malignant proliferation of B cells that is similar to CLL but carries a better prognosis Sometimes considered an indolent form of lymphoma History: fatigue, frequent infections, abdominal fullness Physical examination: no fever, massive splenomegaly Tests:
Decreased Hgb, WBCs, and platelets Blood smear shows numerous lymphocytes with “hairy” irregular cytoplasmic projections (Figure 6-6) Bone marrow biopsy shows lymphocytic infiltration Treatment: chemotherapy is administered once patients develop symptomatic cytopenia Outcomes: the disease course is indolent and both acute episodes and recurrences tend to respond well to chemotherapy Why eliminated from differential: the blood smear appearance makes this diagnosis unlikely Lymphoma Malignant transformation of lymphocytes in the lymph nodes that may spread to the blood stream and nonlymphatic tissues Histologically classified as Hodgkin's or non-Hodgkin's lymphoma (Table 6-16) History: painless lymphadenopathy, weight loss, night sweats
Figure 6-6 Cells typical of hairy cell leukemia. Note the numerous cytoplasmic projections, giving the cell its name. (See color image.)
Physical examination: fever, hepatosplenomegaly Tests: lymph node biopsy is the definitive diagnostic test (Color Figure 6-13) Treatment: a combination of radiation to localized involved nodes and systemic chemotherapy is used for treatment Outcomes: prognosis tends to be favorable for Hodgkin's lymphoma but much worse for non-Hodgkin's variants Why eliminated from differential: the absence of lymphadenopathy in this patient makes this diagnosis unlikely
Acute leukemias tend to exhibit a proliferation of immature cell forms on the bone marrow biopsy, while chronic leukemias tend to exhibit more mature cells.
Most ALL originates in B cell precursors.
ALL is the most common cancer in children.
Presence of the Philadelphia chromosome [t(9;22)] or BCR-ABL gene is pathognomonic of CML.
CLL and small lymphocytic lymphoma are considered to be the same disease process in different stages of evolution.
The most common explanation for increased RBC production is chronic hypoxia. Polycythemia vera Myeloproliferative disorder of bone marrow stem cells, leading to an increased production of RBCs, WBCs, and platelets More common in older individuals and may be a precursor to the development of leukemia History: fatigue, headache, burning pain in the hands and feet, pruritis that worsens after immersion in warm water, tinnitus, blurred vision, epistaxis, abdominal pain Physical examination: hepatosplenomegaly, large retinal vessels on fundoscopic examination Tests: Increased Hgb and increased or normal WBCs and platelets Decreased EPO
Table 6-16 Characteristics of Hodgkin and Non-Hodgkin's Lymphomas
Charac teristic
Hodgkin Ly mphoma
Non-Hodgkin Lymphoma
Cells of origin
B cells
Lymphocytes (most c ommonly B c ells) or natural killer cells
Classification
Lymphocytic (rare, best prognosis), mixed
Many types, common variants include: follicular small cell (B cells,
(low to higher
cellularity (most c ommon), nodular sclerosis
most common, chromosomes 14 and 18 translocation), small
grade)
(w omen, fibrosis of lymph nodes), lymphocytic
lymphocytic (B cells), diffuse large cell (B cells), peripheral (T
depletion (rare, worst prognosis)
c ells), Burkitt (EBV -related, chromosomes 8 and 14 translocation)
Risk factors,
20–40 years old or >60 years old
EBV, HIV, congenital immunodeficiencies, rheumatic disease, <65 years old
patient population
H/P
Painless lymphadenopathy (nec k), weight loss,
Painless lymphadenopathy (generalized), weight loss, fever, night
pruritus, night sweats, fever,
sweats
hepatosplenomegaly
Labs
Lymph node biopsy shows Reed-Sternberg c ells
Lymph node or bone marrow biopsy shows lymphocyte
(Color Figure 6-10)
proliferation (c leaved c ells seen in follic ular small c ell variant)
Treatment
Radiation, chemotherapy
Palliative radiation, chemotherapy
Prognosis
Good, 80% cure rate unless far-progressed
Poor (months for aggressive types, years for less aggressive variants), worsens with increasing age
EBV, Epstein-Barr virus; HIV, human immunodeficiency virus; H/P, history and physical.
Color Figure 6-13 Hodgkin lymphoma. A histologic section of a lymph node demonstrates pathognomonic binucleated Reed-Sternberg cells that resemble owls' eyes.
Bone marrow biopsy shows hypercellularity Treatment: Serial phlebotomy is performed to decreased hemoconcentration Antihistamines are given for pruritis ASA is given as thrombus prophylaxis Hydroxyurea is given to suppress bone marrow activity Outcomes: complications include stroke, thrombus formation, and leukemia Why eliminated from differential: although polycythemia may precede the onset of leukemia, the pancytopenia seen in this patient rules out this condition as a current diagnosis Aplastic anemia Pancytopenia resulting from bone marrow failure May be caused by several medications (e.g., chloramphenicol, sulfonamides, phenytoin, chemotherapeutics), industrial toxins, viral infection, or idiopathic causes History: fatigue, weakness, persistent or recurrent infections, uncontrolled bleeding, easy bruising,
menorrhagia Physical examination: pallor, petechiae, tachycardia, tachypnea, systolic murmur, increased pulse pressure Tests: decreased Hgb, WBCs, and platelets; bone marrow biopsy shows gross hypocellularity Treatment: Any offending agents should be avoided Blood products are administered to replace insufficient components Immunosuppressive agents may help to suppress marrow destruction Bone marrow transplant may be necessary for survival Outcomes: prognosis worsens with increasing age and severity, with a 5-year survival of 85% in young patients with moderate disease and a 20% 5-year survival in elderly patients with severe disease Why eliminated from differential: the appearance of the bone marrow biopsy rules out this diagnosis Human immunodeficency virus More thorough discussion in prior case Why eliminated from differential: the negative HIV ELISA rules out this diagnosis Agranulocytosis More thorough discussion in prior case Why eliminated from differential: the combination of low RBCs and platelets in addition to decreased WBCs make this diagnosis unlikely
Case 6-8: “My baby is going yellow!” A neonate boy is 24 hours past an uncomplicated vaginal delivery following an approximately 38-week gestation. Apgar scores were 8 and 9 at 1 and 5 minutes, respectively. The patient's mother received no prenatal care and arrived in the emergency department in the early stages of labor. On assessment she reports that she has been pregnant twice previously, but each of these pregnancies ended in an elective abortion. She is unaware of any past medical history and takes no medications. She says that she smoked less than five cigarettes per week during the pregnancy and denies other substance use. The newborn is with the mother in her hospital room when the mother calls the nurse about her baby. She thinks that the newborn is jaundiced looking. She does not recall the baby appearing this way earlier in the day. She has been able to successfully breast feed the child. On examination, the newborn appears to be in no distress. The child has mild scleral icterus and jaundice. Auscultation of the heart and lungs finds normal breathing sounds and a subtle systolic murmur. The child's abdomen is soft with mild hepatosplenomegaly. He is moving all extremities and has normal primitive reflexes. The following vital signs are measured: T: 99.0°F, HR: 165 bpm, BP: 90/40 mm Hg, RR: 40 breaths/min (Normal for neonate: HR: 100–170 bpm, BP: 65–95/30–60, RR: 30–50 breaths/min)
Differential Diagnosis Physiologic jaundice, breast milk jaundice, neonatal sepsis, biliary atresia, Crigler-Najjar syndrome, hemolytic disease of the newborn, glucose-6-phosphate dehydrogenase deficiency, Fanconi anemia
Laboratory Data and Other Study Results CBC: WBC: 7.8, Hgb: 12.7, Plt: 347
Reticulocyte count: 8% Chem7: Na: 140 mEq/L, K: 3.9 mEq/L, Cl: 110 mEq/L, CO2: 24 mEq/L, BUN: 20 mg/dL, Cr: 1.0 mg/dL, Glu: 64 mg/dL LFTs: AlkPhos: 129 U/L, ALT: 43 U/L, AST: 21 U/L, TBili: 6.3 mg/dL, DBili: 0.7 mg/dL, IBili: 5.6 mg/dL Patient blood type: B+ Maternal blood type: BPatient Coombs test: direct negative, indirect positive Maternal Coombs test: direct negative, indirect positive
Diagnosis Hemolytic disease of the newborn
Treatment Administered The neonate was kept in the hospital for an additional 3 days for observation and measurements of his bilirubin; following discharge he was brought back to a clinic on a weekly basis for bilirubin measurements The child's mother was administered Rho(D) immune globulin as soon as the diagnosis was decided The patient's anemia and hyperbilirubinemia remained relatively stable while in the hospital Phototherapy was deemed unnecessary because the patient's total bilirubin levels remained only mildly elevated
Follow-up The child's Hgb and bilirubin levels gradually normalized over several weeks No long-term effects were apparent in the child
Steps to the Diagnosis Hemolytic disease of the newborn Formation of maternal antibodies into fetal blood if there is a difference in ABO or Rh blood types Alloimmunization may occur during a prior pregnancy, abortion, or maternal hemorrhage Pregnancies following the time of alloimmunization may be complicated by fetal hemolysis The severity of hemolysis is mild in 50% of cases and moderate or severe (i.e., fetal hydrops) in 50% of cases History: an appropriate history of exposure to the incompatible blood type is required Physical examination: neonatal pallor, jaundice, and hepatosplenomegaly Tests: Hgb will be decreased according to the severity of the disease; WBCs and platelets may be decreased in severe disease Reticulocytes will be increased Blood type incompatibility between the mother and neonate should be apparent The indirect Coombs test and direct antibody tests will be positive in both the neonate and the mother
Treatment: Most cases do not require treatment or only require phototherapy to improve hyperbilirubinemia Severe hemolysis may require exchange transfusion Intrauterine fetal transfusion may be required if fetal health deteriorates in utero Mothers should be given Rho(D) immune globulin (RhoGAM, Ortho-Clinical Diagnostics, Johnson & Johnson, Rochester, New York) within 72 hours of delivery of the initial Rh+ or incompatible fetus or any time maternal and fetal blood may have mixed Outcomes: if patients are able to be hemodynamically stabilized, there are rarely any long-term effects of neonatal hyperbilirubinemia or anemia Clues to the diagnosis: History: lack of prenatal care, prior abortions Physical: jaundice, hepatosplenomegaly Tests: decreased Hgb, increased reticulocyte count, increased total and indirect bilirubin, positive indirect Coombs test Physiologic jaundice More thorough discussion in Chapter 3 Why eliminated from differential: this diagnosis typically occurs within the first week of life, but the positive Coombs test rules it out as the cause of the child's jaundice Breast milk jaundice More thorough discussion in Chapter 3 Why eliminated from differential: given the extremely short history of breast feeding in this patient, this condition would not be the cause of the neonate's jaundice Neonatal sepsis Sepsis that occurs due to maternal transmission during birth or in delayed fashion due to the neonate's living environment Group B streptococcus (GBS), E. coli, H. influenzae, and Listeria monocytogenes are the most common causes of early sepsis Coagulase-negative staphylococci, S. aureus, E. coli, Klebsiella, Pseudomonas, Enterobacter, Candida, GBS, Serratia, Acinetobacter, and anaerobes may be the cause of delayed sepsis Risk factors: low Apgar scores, maternal fever, maternal urinary tract infection (UTI), poor prenatal care, low socioeconomic status, recurrent abortions, maternal substance abuse, low neonatal birth weight, meconium staining, congenital anomalies History: possible prematurity, presence of risk factors, neonatal pneumonia Physical examination: fever or hypothermia, tachypnea, tachycardia or bradycardia, dyspnea, hypotension, cyanosis, decreased motor activity, jaundice Tests: Increased WBCs, decreased Hgb and platelets
Blood, urine, and cerebral spinal fluid cultures are needed to identify an organism CXR may reveal pneumonia Treatment: IV antibiotics are required to treat the systemic infection IVIG may help improve the immune response Ventriculoperitoneal shunting may be required in infants with meningitis Outcomes: the prognosis is good in neonates that are diagnosed early in the disease course; delays in diagnosis carry a poor prognosis Why eliminated from differential: although this diagnosis is possible in this patient, the normal WBC count makes it less likely; the Coombs test results also makes this an unlikely diagnosis Biliary atresia More thorough discussion in Chapter 3 Why eliminated from differential: this diagnosis is possible but would only be considered if the jaundice was more severe and persisted for multiple weeks Crigler-Najjar syndrome More thorough discussion in Chapter 3 Why eliminated from differential: a higher total and indirect bilirubin would be expected for this diagnosis, and the jaundice would be expected to last several weeks Glucose-6-phosphate dehydrogenase deficiency More thorough discussion in prior case Why eliminated from differential: the positive indirect Coombs test rules out this diagnosis Fanconi anemia More thorough discussion in prior case Why eliminated from differential: the limitation of cytopenia to RBCs on the CBC rules out this diagnosis
Authors: Van Kleunen, Jonathan P. Title: Step-Up to USMLE Step 3, 1st Edition Copyright ©2009 Lippincott Williams & Wilkins > Table of Contents > Chapter 7 - Neurology
Chapter 7 Neurology Basic clinical primer Neuroanatomy Vasculature The circle of Willis forms the backbone of the cerebral vasculature (Figure 7-1, Table 7-1) Neurons Cranial nerves originate in cerebral nerve roots and function primarily in the head and neck (Table 7-2) The spinal cord is organized into distinct tracts of sensory and motor neurons (Figure 7-2, Table 7-3)
The order of cranial nerves may be remembered by the mnemonic “Ooh, Ooh, Ooh, To Touch And Feel Very Green Vegetables, Ah Heaven!”: Olfactory, Optic, Oculomotor, Trochlear, Trigeminal, Abducens, Facial, Vestibulocochlear, Glossopharyngeal, Vagus, Accessory, Hypoglossal.
The function (sensory, motor, or both) of the cranial nerves may be remembered by the mnemonic “Some Say Marry Money, But My Brother Says Big Brains Matter Most”: Sensory, Sensory, Motor, Motor, Both, Motor, Both, Sensory, Both, Both, Motor, Motor (in order of cranial nerves).
Sleep Sleep cycles are divided into several stages that describe the depth of sleep and the corresponding electroencephalogram (EEG) activity (Table 7-4)
Case 7-1: “My neck feels really stiff” A 19-year-old man presents to his college's student health office because of a constant headache and neck pain that has worsened over the past 3 days. The headache is diffuse but is worse posteriorly. He says that a few other people in his dorm have come down with similar symptoms, and he is concerned that they all have a similar illness. He denies having had these symptoms previously. He has been unable to eat for the past day because he feels nauseous. He did not go to class yesterday or today because he did not feel well. He denies any recent head or spine trauma or animal or insect bites. He denies any past medical history and does not take any medications regularly. He has been taking ibuprofen over the past few days and says that it has had little benefit. He drinks heavily on weekends (last use 4 days ago) and denies other substance use. On examination, he is an ill-appearing young adult. He asks that the examination room lights be dimmed because the bright lights worsen his headache. Fundoscopic examination is normal. He has no
palpable lymphadenopathy. He has scattered red petechiae on his trunk. Flexion of his neck does not change his perceived neck pain. Auscultation of his heart and lungs is normal. His abdomen is nontender with no masses, but palpation worsens his nausea. His sensory and motor function is normal, but he is slow to move because of his neck pain. The following vital signs are measured: Temperature (T): 102.3°F, heart rate (HR): 96 beats per minute (bpm), blood pressure (BP): 110/80 mm Hg, respiratory rate (RR): 20 breaths/min P.183
First-order neurons are preganglionic; second-order neurons are postganglionic.
Upper motor neuron conditions are those that originate in the brain or first-order neurons; lower motor neuron conditions are those caused by pathology in second-order neurons.
Rapid eye movements (REM) and dreams occur during REM sleep.
Figure 7-1 (A) Arteries of the brain, including the circle of Willis, and their anatomical relationship to selected cranial nerves. (B) Arterial blood supply to the cortex. CN, cranial nerve. (See color image.)
Differential Diagnosis Bacterial meningitis, viral meningitis, alcohol intoxication/withdrawal, migraine headache, cluster headache, tension headache, mononucleosis, cerebrovascular accident, subarachnoid hemorrhage, brain abscess, viral encephalitis, rabies, cervical spine injury
Most of sleep is spent in stage 2. Benzodiazepines increase stage 2 sleep and decrease stages 3 and 4, and do not reproduce the normal sleep architecture.
Table 7-1 Regions of the Brain Supplied by Vessels in the Circle of Willis
Artery
Region of Brain Supplied
Anterior cerebral artery
Medial and superior surfaces and frontal lobes
Middle cerebral artery
Lateral surfaces and temporal lobes
Posterior cerebral artery
Inferior surfaces and occipital lobes
Basilar artery
Midbrain, brainstem (pons)
Anterior inferior cerebellar artery
Brainstem (pons)
Posterior inferior cerebellar artery
Brainstem (medulla)
P.184
Table 7-2 Cranial Nerves and Their Functions
Nerve
Type
Func tion/Innervation
Olfactory (CN I)
Sensory
Smell
Optic (CN II)
Sensory
Sight
Oculomotor (CN III)
Motor
Medial/superior/inferior rectus muscles, inferior oblique muscle, ciliary muscle, sphincter muscle of eye
Trochlear (CN IV)
Motor
Superior oblique muscle of eye
Trigeminal (CN V)
Both
Sensation of face; muscles of mastication
Abducens (CN VI)
Motor
Lateral rectus muscle of eye
Facial (CN VII)
Both
Taste (anterior two thirds of tongue); muscles of facial expression, stapedius muscle, stylohyoid muscle, digastric muscle (posterior belly); lacrimal, submandibular, sublingual glands
Vestibulocochlear (CN
Sensory
Hearing, balance
Both
Taste (posterior one third of tongue), pharyngeal sensation; stylopharyngeus muscle; parotid gland
VIII)
Glossopharyngeal (CN IX)
Vagus (CN X)
Both
Sensation of trachea, esophagus, viscera; laryngeal, pharyngeal muscles; visceral autonomics
Accessory (CN XI)
Motor
Sternocleidomastoid and trapezius muscles
Hypoglossal (CN XII)
Motor
Tongue
CN, cranial nerve.
Figure 7-2 (Cross-sectional anatomy of the spinal cord defining the main neuronal pathways. Generally for each pathway, fibers closer to the center of the cord supply the upper body, and fibers closer to the periphery supply the lower half of the body.
P.185
Table 7-3 Primary Sensory and Motor Tracts of the Spinal Cord
Pathw ay
Dorsal columns
Loc ation
First-Order Neurons
Sec ond-Order Neurons
Func tion
Posterior
Enter at ipsilateral dorsal horn, ascend in
Decussate at medulla,
Tw o point disc rimination,
spinal cord
fasciculus gracilis and cuneatus, synapse in
ascend as medial lemniscus
sense vibration, sense proprioc eption
nucleus gracilis and cuneatus
Spinothalamic
Anterior
Originate in dorsal root ganglion, synapse in
Decussate in ventral white
Senses pain, senses
tract
spinal cord
dorsolateral tract of Lissauer
commissure, ascend in
temperature
lateral spinothalamic tract
Corticospinal
Lateral
Descend from internal capsule and midbrain,
Exit cord through ventral
V oluntary movement of
tract
spinal cord
decussate in medullary pyramids, descend in
horn
striated muscle
corticospinal tract, synapse in ventral horn through interneurons
Laboratory Data and Other Study Results The patient is immediately sent to the local emergency department where the following tests are performed: Complete blood cell count (CBC): white blood cells (WBC): 21.6, hemoglobin (Hgb): 14.1, platelets (Plt): 412 7-electrolyte chemistry panel (Chem7): sodium (Na): 141 mEq/L, potassium (K): 4.5 mEq/L, chloride (Cl): 102 mEq/L, carbon dioxide (CO2): 27 mEq/L, blood urea nitrogen (BUN): 17 mg/dL, creatinine (Cr): 0.5 mg/dL, glucose (Glu): 89 mg/dL Liver function tests (LFTs): alkaline phosphatase (AlkPhos): 78 U/L, alanine aminotransferase (ALT): 34 U/L, aspartate aminotransferase (AST): 23 U/L, total bilirubin (TBili): 1.1 mg/dL, direct bilirubin (DBili): 0.3 mg/dL Ethyl alcohol (EtOH) level: 0 mg/dL Head and cervical spine computed tomography (CT): no hemorrhage; no cerebral atrophy; no focal lesions; no spinal fracture, dislocation, or malalignment After receipt of these results, a lumbar puncture is performed with the following results: Appearance: cloudy Opening pressure: 230 mm Hg WBCs: 6,538/µL Glucose: 32 mg/dL Protein: 64 mg/dL
Diagnosis Bacterial meningitis
Table 7-4 Normal Stages of Sleep
Stage
Depth of Sleep
EEG
1
Light
Fastθ waves
2
Intermediate
Sleep spindles, k-complexes
3 and 4
Deep
δ waves
REM
Increased brain activity every 90–120 minutes of sleep
Low-voltage, high-frequency waves
EEG, electroencephalogram; REM, rapid eye movement.
P.186
Treatment Administered The patient was admitted to the hospital and immediately placed on dexamethasone and cefotaxime while cerebral spinal fluid (CSF) cultures were pending Cultures grew Neisseria meningitides, and the antibiotic was changed to ampicillin for a total of seven days of treatment
Follow-up Two other students in the same dorm room were diagnosed with N. meningitides and were treated in a similar fashion All close contacts of the infected students were administered prophylactic rifampin The patient was able to recover without any significant neurologic sequelae but continued to have intermittent headaches that gradually improved over 3 months
Steps to the Diagnosis Bacterial meningitis Infection of the meningeal tissue surrounding the brain and spinal cord by a bacterial pathogen The most common agents of infection vary by age group (Table 7-5) Infection may be caused by close contact with infected individuals, hematogenous spread, local extension, or exposure of CSF to bacteria (e.g., neurosurgical procedures, skull fracture) Risk factors: ear infection, sinusitis, immunocompromise, neurosurgery, maternal group β streptococcus infection during birth History: headache, neck pain, photophobia, malaise, nausea, vomiting, confusion Physical examination: fever, Brudzinski sign (i.e., neck flexion in a supine patient prompts reflexive hip flexion), Kernig sign (i.e., painful knee extension occurs with hip flexion in a supine patient), petechiae (especially N. meningitides infection), change in mental status, impaired consciousness, seizures Tests: Increased serum WBCs Possible positive blood cultures Lumbar puncture is the best diagnostic test and may be used to differentiate the causes of meningitis (Table 7-6) CSF appearance, WBC count, protein level, glucose level, and culture are important analyses for CSF collected from a lumbar puncture Imaging is rarely helpful for making the diagnosis Treatment:
Empiric antibiotic therapy is started initially and usually consists of a third-generation cephalosporin Intravenous (IV) corticosteroids have been shown to improve outcomes in some cases but are not helpful for all types of infection
Table 7-5 Common Causes of Meningitis by Age Group
Age
Most Common Agent
Other Common Agents
Newborn
Groupβ streptococci
Escherichia coli, Listeria, Haemophilus influenzae
1 month–2 years old
Streptococcus pneumoniae, Neisseria meningitidis
Group β streptococci, Listeria, H. influenzae
2–18 years old
N. meningitidis
S. pneumoniae, Listeria
18–60 years old
S. pneumoniae
N. meningitidis, Listeria
60+ years old
S. pneumoniae
Listeria, Gram-negative rods
P.187
Table 7-6 Cerebral Spinal Fluid Findings for Different Causes of Meningitis
Status
W BCs
Pressure
Gluc ose
Protein
Healthy patient
<5
50–180 mm H2O
40–70 mg/dL
20–45 mg/dL
Bacterial infection
↑↑ (PMNs)
↑↑
↓
↑
Viral infection
↑ (lymphocytes)
↑
Normal
Normal
Fungal infection or tuberculosis
↑ (lymphocytes)
↑↑
↓
↑
H2O, water; PMNs, polymorphonuclear cells; WBCs, white blood cells; ↑, mild increase; ↑↑, significant increase.
Specific antibiotic therapy is based on the CSF cultures Prophylactic rifampin may be given to close contacts of an infected patient Outcomes: One third of patients will develop neurologic sequelae, including cranial nerve palsies, cerebral infarcts, and brain abscesses Prognosis is best in healthy patients treated promptly with antibiotics
Immunocompromised patients and the very young and old have worse outcomes Clues to the diagnosis: History: headache, neck pain, close contacts with similar symptoms, photophobia Physical: fever, petechiae Tests: increased serum WBCs, lumbar puncture results
While considered “classic” signs of meningitis, Kernig and Brudzinski signs are not reliable signs for diagnosis and are particularly unreliable in children.
Neurologic examination must be performed before lumbar puncture. If there are signs of increased intracranial pressure (e.g., papilledema, focal neurologic deficits, pupil asymmetry), do not perform a lumbar puncture because of the increased risk of uncal herniation.
Treat fungal meningitis with amphotericin B, and treat tuberculosis meningitis with the combination of isoniazid, ethambutol, pyrazinamide, and rifampin.
Symptoms in viral meningitis are typically milder than those for bacterial infection. Viral meningitis (a.k.a. aseptic meningitis) Meningitis due to infection by an enterovirus, echovirus, herpes simplex virus, lymphocytic choriomeningitis virus, or mumps virus History: headache, neck pain, nausea, vomiting, photophobia, malaise Physical examination: fever, viral rash Tests: lumbar puncture is useful for differentiating this cause of meningitis from a bacterial cause (Table 7-6); viral culture provides a more definitive diagnosis Treatment: empiric antibiotics are typically started while the results of a lumbar puncture are pending but may be stopped once a viral cause is suspected; supportive care only is required once the diagnosis has been established Outcomes: the prognosis is better than for bacterial meningitis, and most patients fully recover Why eliminated from differential: the results of the lumbar puncture rule out this diagnosis Migraine headache More thorough discussion in later case Why eliminated from differential: the length of symptoms, diffuse nature of the headache, lack of an aura, and poor response to nonsteroidal anti-inflammatory drugs (NSAIDs) make this diagnosis unlikely Cluster headache More thorough discussion in later case Why eliminated from differential: the length of symptoms, diffuse nature of the headache, and lack of Horner syndrome make this diagnosis unlikely
Tension headache More thorough discussion in later case Why eliminated from differential: this diagnosis would be considered until the further testing ruled it out (i.e., lumbar puncture, CBC), but the length of symptoms would make it less likely Alcohol intoxication/withdrawal More thorough discussion in Chapter 13 Why eliminated from differential: given the patient's drinking history, this diagnosis should be considered, but the lab values rule it out as a diagnosis P.188 Cerebrovascular accident/subarachnoid hemorrhage More thorough discussion in later case Why eliminated from differential: the normal-appearing head CT rules out these diagnoses Mononucleosis More thorough discussion in Chapter 6 Why eliminated from differential: this diagnosis should be seriously considered, but is less likely given the lack of lymphadenopathy; the lumbar puncture results further rule it out Viral encephalitis Inflammation of brain parenchyma due to viral infection Varicella zoster virus, herpes simplex virus, mumps virus, poliovirus, rhabdovirus, Coxsackie virus, arbovirus, flavivirus, and measles virus are all common pathogens The condition may actually reflect an immunologic response to systemic viral infection History: malaise, headache, vomiting, neck pain Physical examination: decreased consciousness, focal neurologic deficits (e.g., hemiparesis, pathologic reflexes, palsies), fever, change in mental status, possible skin lesions (for herpes virus), possible parotid swelling (for mumps), possible flaccid paralysis with a maculopapular rash (for West Nile virus) Tests: Lumbar puncture shows increased WBCs and a normal glucose CSF cultures frequently are negative and are not reliable Serologic viral testing is useful to identify a pathogen Brain biopsy may provide a definitive diagnosis but is generally impractical CT and magnetic resonance imaging (MRI) may show cerebral inflammation with an associated effusion Treatment: supportive care (e.g., maintenance of normal intracranial pressure, seizure prophylaxis); antivirals may demonstrate some usefulness to improving outcomes Outcomes: neurologic sequelae happen in up to 40% of patients and include seizures and motor dysfunction; mortality varies significantly between causative viruses Why eliminated from differential: the lack of neurologic deficits and the results of the lumbar puncture rule
out this diagnosis Brain abscess A collection of purulent material in the brain parenchyma that results from extension of a local bacterial infection, head wound, or hematogenous spread of bacteria History: headache, neck pain, malaise, vomiting Physical examination: change in mental status, focal neurologic deficits, papilledema, seizures Tests: MRI or CT is useful to detect the lesion; CT-guided biopsy may be used to culture the abscess fluid and determine the bacterial identity Treatment: surgical drainage is required; empiric antibiotics and corticosteroids are administered until the culture results are finalized Outcomes: the prognosis is poor with any delays to diagnosis or therapeutic intervention Why eliminated from differential: the negative head CT and normal fundoscopic examination help to rule out this diagnosis Rabies Rhabdovirus infection transmitted to humans via the bite of an infected animal Infection causes a severe encephalitis with neuronal degeneration and cerebral inflammation History: malaise, headache, restlessness, painful laryngospasms, fear of water ingestion (secondary to laryngospasm) Physical examination: significant CNS excitability, foaming at the mouth, alternating mania and stupor Tests: A suspected animal should be caught and tested or observed for signs of rabies Animals suspected of infection should be euthanized, and their brains should be tested for the presence of the virus and histologic Negri bodies (i.e., round eosinophilic inclusions within the neurons) P.189 Viral testing of symptomatic patients (including CSF, skin, and serum) provides a confirmatory diagnosis Treatment: animal bites should be thoroughly cleaned and bandaged; rabies immunoglobulin and vaccine should be administered promptly to any patient bitten by an infected or suspicious animal Outcomes: once symptoms develop, mortality is nearly 100% Why eliminated from differential: the absence of a history of being bitten by any animals makes this diagnosis unlikely Cervical spine injury More thorough discussion in later case and in Chapter 9 Why eliminated from differential: the normal-appearing cervical spine CT makes this diagnosis unlikely
Common arboviruses include the St. Louis and California strains. Common flaviviruses include the West Nile and Japanese strains.
In young children encephalitis may be due to Reye syndrome (i.e., reaction in
children with a viral infection who are given aspirin [ASA]).
Case 7-2: “My head is killing me” A 19-year-old man presents to his college's student health office with recurrent severe headaches. He says that his pain is located in the front of his head near his forehead and eye and on the right side only. He has no pain in his neck. The headaches have been intermittent in nature over the past 5 days and have occurred about three times each day. Each headache lasts approximately 1 hour before resolving. During these headaches the patient has nasal congestion on the right side but has no upper respiratory symptoms in between headaches. He denies any visual abnormalities prior to the headaches. He also denies any numbness or weakness in any of his extremities. He says that he had a similar experience 2 years ago. At that time he had several headaches over a 1-week period that self-resolved, and he has had no recurrences until the present time. A head CT performed during the previous episode was normal. He has a history of childhood asthma that has improved with age, and he no longer requires any medications other than antihistamines during peak seasonal allergy periods. He drinks socially and estimates that he consumes eight alcoholic beverages per week. He says that he used marijuana two times in his previous year at school but has not used this substance in over a year. On examination, he is a well-developed, tall young man who appears mildly uncomfortable. He has a mild lid sag in his right eye. His right eye conjunctiva is mildly injected, and his pupils are asymmetric (the right is smaller). Fundoscopic examination of both eyes is normal. There are no lesions visible on his face or body. He has no lymphadenopathy. Flexion of his neck while he is supine does not elicit any pain. Auscultation of his heart and lungs is normal. His abdomen is nontender with no masses, and he has regular bowel sounds. All of his extremities have normal and symmetric motor and sensory function, and his gait is normal. The following vital signs are measured: T: 98.5°F, HR: 88 bpm, BP: 133/87 mm Hg, RR: 17 breaths/min
Differential Diagnosis Migraine headache, cluster headache, tension headache, sinusitis, cerebrovascular accident, viral meningitis, bacterial meningitis, subarachnoid hemorrhage, shingles, trigeminal neuralgia
Laboratory Data and Other Study Results CBC: WBC: 6.6, Hgb: 15.2, Plt: 285 Head MRI: no masses within the brain or face; no areas of hemorrhage or ischemia
Diagnosis Cluster headaches
Treatment Administered The patient was placed on oral sumatriptan and low-dose verapamil Sumatriptan was stopped after 3 days with no additional headaches P.190
Follow-up The patient continued to take low-dose verapamil for prophylaxis The patient experienced a single similar headache 9 months later after a night of high alcohol consumption; after restarting sumatriptan, he experienced no additional headaches during that episode The patient was able to remain headache free by remaining on verapamil and refraining from heavy drinking
Any sudden onset of severe headache or focal neurologic deficits should be further examined using CT without contrast or MRI to rule out hemorrhage.
Steps to the Diagnosis Cluster headache A primary headache disorder affecting young men that is poorly understood and typified by recurrent (i.e., “clustered”) severe headaches (Table 7-7) Clusters may last for a week up to a year Alcohol and vasodilators have been suggested as precipitating factors History: severe unilateral headaches in the periorbital region that last 30 minutes to 3 hours, nasal congestion, lacrimation Physical examination: possible Horner syndrome (i.e., ptosis, miosis, anhydrosis), conjunctival injection, diaphoresis Tests: typically this is a clinical diagnosis, but head and neck imaging may be useful to rule out other diagnoses Treatment: ergots, 100% oxygen, or sumatriptan are used to break acute headaches; tricyclic antidepressants, calcium channel blockers, β-blockers, and ergots may be used as prophylaxis
Table 7-7 Primary Headache Disorders
V ariable
Migraine
Cluster
Tension
Patients
10–30 years old, female more commonly than male
Young men
Female and male
Pathology
Incompletely understood but related to
Poorly understood but likely an
Poorly understood but
generalized dysfunctional cerebral activity
extra-cerebral cause
related to muscular and psychogenic causes
Alcohol, vasodilators
Stress, fatigue, depression
Severe, unilateral, periorbital,
Bilateral, head or scalp
recurrent (i.e., “in clusters” over
tightness, occipital or neck
time)
pain
Nausea, vomiting, preceding aura (i.e., visual
Horner sy ndrome (i.e., ptosis,
Anxiety, insomnia,
abnormalities), photophobia, temporary
myosis, anhidrosis), lacrimation,
photophobia
neurologic deficits
nasal congestion
4–72 hours
30 minutes–3 hours per headache;
Precipitating
Stress, oral contraceptives, menstruation,
factors
exertion, foods containing tyramine or nitrates (e.g., chocolate, cheese, processed meats)
Pain
Unilateral, throbbing
characteristics
Other symptoms
Duration
30 minutes–1 week
clusters last 1 week–1 year
Acute treatment
NSAIDs, ergots, sumatriptan, possible antiemetics
100% O 2, ergots, sumatriptan
NSAIDs, ergots, sumatriptan, relaxation exercises
Prophylactic
Tricyclic antidepressants, β-blockers, calcium
Tricyclic antidepressants,
Treating underlying
treatment
channel blockers, ergots
β-blockers, calcium channel
depression and stress helps
blockers, ergots
reduce frequency
Typically recurrent but responsive to abortive
Typically recurrent but becomes a
Correction of the
therapies
near-constant chronic disorder in
underlying factors reduces
15% of cases
frequency
Outcomes
O 2, oxygen; NSAIDs, nonsteroidal anti-inflammatory drugs.
P.191 Outcomes: the disease course will be episodic in most patients, with a small subset developing near continuous clusters Clues to the diagnosis: History: multiple headaches in a brief period of time, nature of headaches (e.g., unilateral, periorbital, severe), nasal congestion Physical: Horner syndrome Tests: noncontributory Migraine headache More thorough discussion in Table 7-7 Why eliminated from differential: the absence of auras and recurrent nature of the headaches makes this diagnosis less likely Tension headache More thorough discussion in Table 7-7 Why eliminated from differential: the nature of the headaches (e.g., location, frequency) makes this diagnosis unlikely
Tension headaches are the most common type of headache in adults. Sinusitis More thorough discussion in Chapter 2 Why eliminated from differential: the absence of continuous upper respiratory symptoms makes this diagnosis unlikely Cerebrovascular accident/subarachnoid hemorrhage More thorough discussion in later case Why eliminated from differential: the negative head MRI rules out these diagnoses Bacterial/viral meningitis More thorough discussion in prior case Why eliminated from differential: the absence of fever and neck pain and the normal serum WBC count rule
out these diagnoses Shingles More thorough discussion in Chapter 10 Why eliminated from differential: the absence of skin lesions rules out this diagnosis Trigeminal neuralgia Significant head and facial pain due to compression or irritation of the trigeminal nerve root History: sudden severe pain in the trigeminal nerve maxillary and mandibular distributions that lasts from a few seconds to a couple of minutes and recurs throughout the day Physical examination: stimulation of “trigger points” in the affected nerve distribution may induce pain Tests: MRI may identify lesions compressing the trigeminal nerve Treatment: carbamazepine, baclofen, phenytoin, gabapentin, valproate, clonazepam, or other anticonvulsants are useful to eliminating the pain; surgical decompression may help to alleviate symptoms when deemed appropriate Outcomes: typically becomes a chronic episodic disease unless nerve decompression is achieved Why eliminated from differential: the location and length of the headaches make this diagnosis less likely
Case 7-3: “My father started talking funny, and he can't walk correctly” A 78-year-old man is brought to an emergency department by his daughter after the acute onset 6 hours ago of right-sided weakness and slurred speech. The patient's daughter says that she was visiting her father for dinner. During the meal he developed progressive deterioration in the clarity of his voice and began to slur his words. The daughter later convinced him to go to the hospital, and when he tried to stand, he found that he had weakness in his right arm. He also noted slight weakness in his right leg. He denies any numbness in any of his extremities, headache, neck pain, or difficulty seeing objects. The patient and his daughter both deny that he has had any prior symptoms of this nature. The patient says P.192 that he has a history of hypertension (HTN), hypercholesterolemia, and stable angina. His medications are ASA, lisinopril, atenolol, simvastatin, and hydrochlorothiazide (HCTZ). He had an uncomplicated hip replacement at the age of 67 years. He drinks a glass of wine each day. He has a 80-pack-year smoking history but quit smoking 18 years ago. On examination, he is a well-nourished man in no acute distress. His speech is somewhat slurred during conversation, and he frequently has difficulty finding words. He has a subtle facial droop on the right side. Fundoscopic examination demonstrates arteriovenous nicking in both eyes. He has difficulty seeing objects to the left side of his face. He has no lymphadenopathy. Auscultation of his lungs and heart detects clear breathing sounds and an S 3 extra heart sound. His abdomen is nontender with no masses and normal bowel sounds. Neurologic examination detects 3/5 weakness in his right arm and 4/5 weakness in his right leg. He has a mildly impaired gait due to a slight drag in his right leg. His sensory examination is normal. His coordination is mildly impaired in his right arm. The following vital signs are measured: T: 98.7°F, HR: 95 bpm, BP: 155/90 mm Hg, RR: 20 breaths/min
Differential Diagnosis Cerebrovascular accident, transient ischemic attack, migraine headache, seizure, multiple sclerosis, parenchymal hemorrhage, subdural hematoma, subarachnoid hemorrhage
Laboratory Data and Other Study Results
CBC: WBC: 8.1, Hgb: 13.9, Plt: 421 10-electrolyte chemistry panel (Chem10): Na: 139 mEq/L, K: 3.9 mEq/L, Cl: 108 mEq/L, CO2: 29 mEq/L, BUN: 17 mg/dL, Cr: 0.7 mg/dL, Glu: 90 mg/dL, magnesium (Mg): 2.1 mg/dL, calcium (Ca): 10.3 mg/dL, phosphorus (Phos): 4.1 mg/dL Coagulation panel (Coags): protime (PT): 11.1 sec, international normalized ratio (INR) 1.0, partial thromboplastin time (PTT): 28.5 sec Cardiac enzymes: creatine kinase (CK): 213 U/L, creatine kinase myocardial component (CK-MB): 1.0 ng/mL, troponin-I: 0.1 ng/mL Electrocardiogram (ECG): normal sinus rhythm; no abnormal wave morphology Head CT: no hemorrhage; abnormal signal seen in the superior region of the left hemisphere The patient is admitted to the hospital, and his right-sided weakness increases slightly over the next 12 hours. Besides the treatments listed below, the following additional studies are performed: Head MRI angiogram: near complete occlusion of a branch off of the left middle cerebral artery (MCA), causing impaired perfusion of part of the superior left hemisphere; no hemorrhage; no white matter lesions Echocardiogram: mild-to-moderate systolic dysfunction; mild aortic stenosis; no intra-cardiac thrombus; mild left ventricular wall thickening; no wall anatomic abnormalities Carotid duplex ultrasound (US): 75% stenosis of the left common carotid artery; 10% stenosis of the right common carotid artery
Diagnosis Cerebrovascular accident, likely thrombotic or embolic in nature and related to carotid artery atherosclerosis
Treatment Administered The patient was deemed inappropriate for fibrinolytic therapy because of the time passed since the onset of his symptoms ASA was continued, and simvastatin was increased to the maximum dose Percutaneous transluminal angioplasty and stenting was performed on the left carotid artery Clopidogrel was started postoperatively P.193
Follow-up The patient's weakness and speech defects stabilized within the first 24 hours after admission but did not resolve The patient was kept on ASA and clopidogrel as long-term antiplatelet therapy The patient was referred to speech therapy and physical therapy The patient's speech improved over time, and he regained most of his voice clarity The patient's right leg weakness improved to near normal His right arm weakness improved partially, and he was forced to use his left arm as his primary functioning arm
Atherosclerosis of the carotid, basilar, or vertebral arteries is the most common cause of thrombotic ischemic stroke.
The middle cerebral artery is the most common artery involved in embolic ischemic stroke. Most emboli originate in the heart, aorta, carotid arteries, or intra-cranial arteries.
Steps to the Diagnosis Cerebrovascular accident (CVA) (a.k.a. stroke) An acute focal neurologic deficit lasting more than 24 hours due to cerebral ischemia Ischemia may be a product of impaired perfusion (i.e., ischemic stroke) or hemorrhage (i.e., hemorrhagic stroke) Ischemic strokes may be thrombotic (i.e., obstruction via occlusive thrombus formation in a supplying artery) or embolic (i.e., embolization of a distant thrombus occludes a supplying artery) Risk factors: increased age, family history, obesity, diabetes mellitus (DM), HTN, tobacco, atrial fibrillation (Afib) History: Acute development of focal neurologic deficits that last more than 24 hours The characteristic neurologic deficits correspond to the arteries involved and their distribution (Figure 7-3, Table 7-8)
Figure 7-3 Homunculus representing the sensory (A) and motor (B) cortex and its corresponding arterial supply.
P.194
Table 7-8 Common Stroke Locations and Corresponding Signs and Symptoms
Loc ation of Stroke
Signs and Symptoms
ACA
Contralateral lower extremity and trunk weakness
MCA
Contralateral face and upper extremity weakness and decreased sensation, bilateral visual abnormalities, aphasia (if dominant hemisphere), neglect and inability to perform learned actions (if nondominant hemisphere)
PCA
Contralateral visual abnormalities, blindness (if bilateral PCA involvement)
Lacunar
Focal motor or sensory deficits, loss of coordination, difficulty speaking
arteries
Basilar
Cranial nerve abnormalities, contralateral full body weakness and decreased sen-sation, vertigo, loss of coordination, difficulty
artery
speaking, visual abnormalities, coma
ACA, anterior cerebral artery; MCA, middle cerebral artery; PCA, posterior cerebral artery.
Aphasias are communication disorders that may develop due to ischemia of distinct brain centers (Table 7-9) Physical examination: Thorough serial neurovascular examinations are important to determining the region of involvement and stroke evolution Consistent physical findings indicate a stabilized stroke Progressive findings indicate an evolving stroke in which the full extent of the ischemic involvement has not been defined Tests: Head CT without contrast is frequently the first radiographic tool used to differentiate ischemic from hemorrhagic strokes and to determine the extent of injured cerebrum MRI is more sensitive than CT in defining an ischemic area, and MRI angiogram (CT angiogram to a slightly lesser extent) is useful for defining the site of vascular insult ECG is useful to detect any arrhythmias that may contribute to the formation of a mural thrombus that might embolize to the brain; cardiac enzymes are helpful for detecting an underlying cardiac pathology Echocardiogram and carotid artery duplex US are useful for detecting sites of thrombus and vascular stenosis in the heart and great vessels Treatment: Acute ischemic stroke Thrombolytic therapy may be administered for ischemic stroke if performed within 3 hours of onset and in the absence of any contraindications (e.g., evidence of hemorrhage on CT, recent surgery, anticoagulant use, recent hemorrhage, blood pressure >185/110 mm Hg) (Figure 7-4)
Table 7-9 Common Classifications of Aphasias
Type
Area Injured
Charac teristic s
Broca (i.e.,
Inferior frontal gyrus, dorsolateral
Few words, difficulty producing words (i.e., nonfluent), good
expressive)
frontal cortex, anterior parietal cortex
c omprehension, face and arm hemiparesis, loss of oral coordination
Wernicke (i.e.,
Posterior superior temporal gyrus,
Word substitutions, meaningless words, meaningless phrases
receptive)
inferior parietal lobe
(i.e.,poor c omprehension, “word salad”)
Conduction
Supramarginal gyrus, angular gyrus
Fluent speec h, word substitutions, frequent attempts to c orrec t w ords, word-finding pauses
Global
Large infarcts of left cerebral
Difficulty producing words, nonfluent speec h, poor
hemisphere
c omprehension, limb ataxia
P.195
Figure 7-4 Treatment algorithm for a suspected acute stroke. CT, computed tomography; ECG, electrocardiogram; IV, intravenous; LP, lumbar puncture; O2, oxygen; t-PA, tissue plasminogen activator.
Antiplatelet therapy (e.g., ASA, clopidogrel) should be started within 48 hours of the onset of symptoms to reduce the risk of additional vascular events Heparin and low molecular weight heparin have not been proven to be of benefit but should be considered in patients with an embolic stroke from a cardiac source Control of excessive HTN and the use of lipid-lowering medications are beneficial to preventing additional acute strokes Acute hemorrhagic stroke
Anticoagulation should be reversed Tight control of blood pressure and intracranial pressure (e.g., mannitol, hyperventilation, anesthesia) are important to controlling evolving symptoms Surgical decompression should be considered for bleeds causing a mass effect and impairing consciousness Antiplatelet medications may be restarted 2 weeks after the stroke if the patient's deficits remain stable Long-term care Patients will require multiple therapies to help treat neurologic deficits Physical therapy is useful to optimizing functional disabilities Speech therapy is useful for improving speaking clarity and may assist with treating aphasias Treatment of the underlying pathologies (e.g., blood pressure control, lipid reduction, arrhythmia control) for stroke is vital to the prevention of future strokes Long-term anticoagulation (e.g., warfarin) is indicated for patients with a cardiac mural thrombus P.196 Carotid endarterectomy or angioplasty with stenting is performed for carotid narrowing >60% in asymptomatic men, >50% in symptomatic men, and >70% in symptomatic women Outcomes: Recovery is dependent on the location and the extent of the infarct, patient's age, and medical comorbidities Younger patients and those with limited infarcts carry the best prognosis Residual deficits are very common Patients with multiple risk factors are at high risk for future strokes in addition to cardiac and vascular ischemic events Clues to the diagnosis: History: acute focal neurologic deficits lasting several hours, history of multiple risk factors Physical: acute neurologic deficits localized to a region of one cerebral hemisphere Tests: head CT and MRI abnormalities, vessel occlusion seen on carotid US
Risk factors for stroke may be remembered by the mnemonic HEADACHES: HTN, Elderly, Afib, DM, Atherosclerosis, Cardiac defect (patent foramen ovale), Hyperlipidemia, Excess weight (obesity), Smoking.
Causes of stroke in young patients may be remembered by the seven Cs: Cocaine, Cancer, Cardiogenic emboli, Coagulation (excessive), CNS infection (septic emboli), Congenital vascular lesion, Consanguinity (genetic disease).
Do not treat HTN immediately following stroke unless it is extreme (>220/120) or if patient has coronary artery disease (CAD) in order to maintain cerebral perfusion.
Most TIAs last <2 hours and are recurrent. Transient ischemic attack (TIA) Acute focal neurologic deficits that last 24 hours and are due to a temporarily impaired vascular supply to the brain (e.g., emboli, aortic stenosis, vascular spasm) Risk factors: HTN, DM, CAD, tobacco use, hyperlipidemia, hypercoagulable states History: sudden appearance of weakness, paresthesias, brief unilateral blindness (i.e., amaurosis fugax), other vision abnormalities, or vertigo Physical examination: signs consistent with the site of vascular insult and possible impaired coordination, carotid artery bruits, or heart murmurs Tests: CT or MRI (with or without angiography) may be useful for determining the region of cerebral ischemia; carotid US or echocardiogram may be useful for detecting thrombus formation Treatment: Antiplatelet and antilipid medications should be prescribed to any patient with atherosclerosis β-blockers, valvuloplasty, or valve replacement should be considered for patients with significant aortic stenosis Carotid endarterectomy or angioplasty should be considered using the same guidelines as for stroke patients Long-term anticoagulation is required for patients with arrhythmias Outcomes: TIAs tend to be recurrent Patients have an increased risk of stroke, with 25% of patients having a stroke within 5 years of the initial TIA Severe carotid artery disease significantly increases stroke risk Long-term outcomes correlate with successful control of the underlying pathologies Why eliminated from differential: although stroke and TIA may present identically, the persistent nature of this patient's deficits defines this process as a stroke Migraine headache More thorough discussion in prior case Why eliminated from differential: although focal neurologic deficits may be seen in some variants of migraines, the lack of a headache and persistent nature of the deficits rule out this diagnosis Seizure More thorough discussion in later case Why eliminated from differential: the absence of a witnessed seizure and persistent nature of the neurologic deficits make this diagnosis unlikely Multiple sclerosis More thorough discussion in later case
Why eliminated from differential: the focal and acute onset of the neurologic deficits and the absence of any white matter lesions on the neuroimaging studies make this diagnosis less likely P.197 Parenchymal hemorrhage Bleeding within the brain parenchyma due to excessive HTN (possibly related to stimulant use), arteriovenous malformation, or cerebral aneurysm History: headache, nausea, vomiting, confusion, possible seizures Physical examination: change in mental status, focal neurologic deficits Tests: head CT without contrast is useful for detecting the region of hemorrhage; MRI or CT angiography is useful for localizing the site of bleeding Treatment: Supportive care is used to keep the patient hemodynamically stable Intracranial pressure is managed to prevent dangerous increases Anticonvulsants are administered for seizure prophylaxis Surgical decompression may be required for large hemorrhages to reduce the risk of herniation Surgical repair or embolization of bleeding aneurysms or arteriovenous malformations (AVMs) should be performed Outcomes: Prognosis is dependent on the size of hemorrhage The early development of significant symptoms carries a worse prognosis Complications include permanent neurologic deficits, seizures, spastic paralysis, and death Why eliminated from differential: the absence of a hemorrhage on the CT and MRI rules out this diagnosis Subdural hematoma/subarachnoid hemorrhage More thorough discussion in later case Why eliminated from differential: the absence of an intracranial bleed on the CT and MRI rules out these diagnoses
Case 7-4: “I was in a car crash” A 27-year-old woman is brought to an emergency department by ambulance after being involved in a single-car collision. The patient says that her car slipped off a wet road and hit a telephone pole on the front left side of the car. She says that she was wearing her seat belt but that her car does not have any airbags. She does think that she lost consciousness. She says that she was able to get out of the car somehow and was able to walk around. She says that she has some mild neck and upper back soreness but denies any pain elsewhere in her body. She also denies any numbness or weakness in her extremities, nausea, vomiting, or loss of bowel control. The emergency medical technicians (EMTs) accompanying her say that it appeared that she may have hit her head on the driver's side window. She was found to have a 4 cm laceration on her scalp that was covered with a bandage. No significant amount of blood was visible at the scene, although she had bled an appreciable amount onto her shirt. She was fully alert and oriented at the scene of the event and was placed in a cervical collar and on a backboard for spinal precautions. The patient says that she has no past medical history and only takes birth control medication. She drinks socially but cannot remember if she drank any alcohol this evening. On examination, she is a healthy appearing woman in no acute
distress. Her airway is confirmed to be intact by her speech, auscultation of her heart and lungs detects clear bilateral lung sounds and mild tachycardia. Her abdomen is nontender with no masses and has normal bowel sounds. She has a laceration on the left scalp that is bandaged and only bleeds slowly when the bandage is removed. When she is log-rolled she has some minor paraspinal tenderness in her thoracic and cervical spine but no deformities or palpable step-offs. A rectal examination shows normal tone and no gross blood. She has no gross deformities of any extremities and is fully intact on motor and sensory examinations. On her secondary survey, she is oriented to her name but not the date or location. She also perseverates on several of the questions surrounding the details of her collision and begins to complain of a severe headache. A repeat neurologic examination is normal. The following vital signs are measured: T: 98.9°F, HR: 110 bpm, BP: 127/87 mm Hg, RR: 18 breaths/min P.198
Differential Diagnosis Cerebrovascular accident, subarachnoid hemorrhage, subdural hematoma, epidural hematoma, intoxication, seizure, spinal cord injury
Laboratory Data and Other Study Results CBC: WBC: 5.9, Hgb: 14.2, Plt: 433 Chem10: Na: 140 mEq/L, K: 3.8 mEq/L, Cl: 104 mEq/L, CO2: 26 mEq/L, BUN: 15 mg/dL, Cr: 0.7 mg/dL, Glu: 91 mg/dL, Mg: 2.0 mg/dL, Ca: 10.1 mg/dL, Phos: 4.0 mg/dL Coags: PT: 11.3 sec, INR: 1.0, PTT: 27.8 sec EtOH level: 0 mg/dL Urine toxicology screen: negative Chest x-ray (CXR): clear lung fields; no rib fractures; no pneumothorax Head CT: nondisplaced cranial fracture on the left side of the skull; convex fluid collection under the cranium on the left side, causing a mass effect and midline shifting of the cerebrum; no interparenchymal or intraventricular hemorrhage Full spine CT: no fractures or dislocations; no impingement on the spinal cord Abdomen/pelvis CT: no hemorrhage; no masses; no free abdominal fluid or air; no visceral abnormalities
Diagnosis Epidural hematoma
Treatment Administered The patient was taken emergently to the operating room with neurosurgery for drainage of the hematoma and placement of an intracranial pressure (ICP) monitor The patient was admitted to the neurosurgical intensive care unit (ICU) following the procedure Initially following surgery, the patient was placed on anticonvulsants and kept sedated
Follow-up The ICP monitor was removed after the patient demonstrated consistent stabilization of her intracranial pressure
Sedation was discontinued, and the patient was allowed to fully awaken The tertiary examination detected no neurologic deficits, and the patient was found to be fully awake and oriented The patient was able to be discharged to home in stable condition after a 1-week inpatient admission
An epidural hematoma may appear to cross the brain midline on CT; subdural hematomas do not.
Steps to the Diagnosis Epidural hematoma Collection of blood between the dura mater and the skull due to arterial hemorrhage The most common cause of arterial bleeding is an injury to the middle meningeal artery following blunt head trauma History: initial “lucid interval” after trauma in which the patient demonstrates no neurologic signs of injury followed later by severe headache, confusion, nausea, and possible seizures Physical examination: altered mental status, possible hemiparesis, hemiplegia, and pupil abnormalities (i.e., “blown pupil”) Tests: CT without contrast shows a convex hyperdensity compressing the brain at the site of injury and a possible adjacent skull fracture (Figure 7-5) Treatment: emergent drainage of the hematoma either via burr hole or craniotomy; stabilization of the blood pressure and ICP is required to reduce the risk of herniation P.199
Figure 7-5 Head computed tomography without contrast, demonstrating an epidural hematoma with a convex hyperdensity due to blood accumulation above the dura (arrowheads).
Outcomes: complications include seizures and neurologic deficits; the prognosis worsens with increasing age, additional intracranial injuries, decreasing Glasgow coma scale, and delays to intervention Clues to the diagnosis: History: head trauma, initial normal mental function with delayed deterioration and amnesia Physical: mental status changes Tests: findings of head CT Subdural hematoma Collection of blood between the dura and arachnoid meningeal layers due to rupture of the bridging veins Frequently follows head trauma History: slowly progressive headache (days to weeks), blunt head trauma Physical examination: change in mental status, contralateral hemiparesis, increased deep tendon reflexes,
papilledema, gait abnormalities, visual field defects Tests: head CT shows a concave hyperdensity compressing the brain but not crossing the midline (Figure 7-6) Treatment: supportive care is appropriate for small hematomas with minimal neurologic effects; surgical decompression is required for large hematomas or those with significant neurologic deficits Outcomes: prognosis worsens with increasing age, decreasing Glasgow coma scale scores, and greater pupil asymmetry Why eliminated from differential: the appearance of the bleed on the head CT is characteristic for an epidural hematoma
If you see mental status changes in an elderly patient with a history of falls, perform a work-up for a subdural hematoma.
Do not perform a lumbar puncture in patients with a subdural hematoma because of the increased risk of herniation. Subarachnoid hemorrhage Bleeding between the pia and arachnoid meningeal layers due to rupture of an arterial aneurysm (i.e., berry aneurysm) or AVM or due to head trauma P.200
Figure 7-6 Head computed tomography without contrast, demonstrating a subdural hematoma with a concave hyperdensity due to blood accumulation between the dura and arachnoid layers (arrows).
History: sudden severe headache, neck pain, nausea, vomiting, possible seizure Physical examination: fever, mental status changes Tests: Lumbar puncture will show RBCs, xanthochromia (i.e., yellowish discoloration of the CSF), and increased
opening pressure Head CT will show blood in the subarachnoid space (Figure 7-7) MRI angiography or an angiogram can localize the site of bleeding Treatment: Control ICP by elevating the head of the bed and administering mannitol Control blood pressure to prevent HTN Reverse any anticoagulation Administer anticonvulsants for seizure prophylaxis Perform interventional radiographic embolization of aneurysms or AVMs Outcomes: Mortality ranges from 30% for asymptomatic cases to 90% for cases causing a coma Some degree of cognitive impairment following hemorrhage is common Long-term neurologic deficits are seen in 25% of cases Why eliminated from differential: the appearance of the bleed on the head CT is characteristic for an epidural hematoma Cerebrovascular accident More thorough discussion in prior case Why eliminated from differential: the relatively young age of the patient, lack of medical comorbidities, and relation to recent head trauma make this diagnosis less likely Intoxication More thorough discussion in Chapter 13 Why eliminated from differential: the negative toxicology screening rules out this diagnosis Seizure More thorough discussion in later case Why eliminated from differential: the lack of a history of prior seizures and the recent head trauma make this diagnosis less likely Spinal cord injury More thorough discussion in later case Why eliminated from differential: the negative spinal CT and the patient's lack of neurologic deficits make this diagnosis unlikely P.201
Berry aneurysms are found more commonly in patients with adult polycystic kidney disease.
Patients may describe the headache in a subarachnoid hemorrhage as the “worst
headache of my life.”
Patients with imminent rupture of a berry aneurysm may have multiple, though less severe, sentinel headaches in the preceding weeks.
If a head CT is negative for a subarachnoid hemorrhage but clinical suspicion is high, perform a lumbar puncture to better rule it out.
A declining RBC count over successive collection tubes may occur in a traumatic lumbar puncture and can help differentiate it from xanthochromia.
Figure 7-7 Subarachnoid hemorrhage seen on computed tomography without contrast; blood is evident in the subarachnoid space (arrows).
Case 7-5: “My daughter keeps having fits” The 27-year-old patient from the previous case presents to a neurologist in follow-up for her epidural hematoma. She was seen 2 weeks after her discharge from the hospital to discuss the development of any abnormal neurologic symptoms, and at that time she reported no new issues. This is her second follow-up appointment and is occurring at 2 months after her discharge. The patient is accompanied by her mother, who reports the occurrence of multiple episodes of strange behavior. The patient's mother reports that on a half dozen occasions in the past month, her
daughter will begin flapping her arms for approximately a minute. During these episodes she does not communicate with anyone around her. Following these episodes she is confused for <30 minutes and has no recollection of the arm flapping. The patient says that when these episodes have occurred, she will smell a strong sweet odor and will feel that air is blowing on her face. She is unable to remember what follows these experiences over the subsequent 20 to 30 minutes. All of these episodes have occurred at home. She has been staying with her parents since the car collision. She has not driven her car in the past 3 weeks because she is concerned that she will have an episode while driving, and she is currently on leave from work. She has occasional headaches and bouts of nausea that have been decreasing in frequency. She denies any new numbness or weakness. She has had no other medical issues since her car collision and takes no new medications. She denies any substance use. On examination, she is well appearing and in no acute distress. Her face appears symmetrical. Fundoscopic examination is normal. She has no lymphadenopathy. Auscultation of her heart, lungs, and abdomen is normal. P.202 Her sensation is normal and symmetrical across her body. Her motor function is likewise normal including her coordination and gait. The following vital signs are measured: T: 98.6°F, HR: 78 bpm, BP: 120/82 mm Hg, RR: 17 breaths/min
Differential Diagnosis Postconcussive syndrome, stroke, subarachnoid hemorrhage, subdural hematoma, epidural hemorrhage, seizure disorder, bacterial meningitis, viral meningitis
Laboratory Data and Other Study Results CBC: WBC: 5.9, Hgb: 13.9, Plt: 431 Head MRI: no hemorrhage; increased signal in the left temporal lobe on T2-weighted images; normal cerebral volume Following these tests, these additional studies are performed: Lumbar puncture: clear appearance, opening pressure 87 mm Hg, WBC: 3/µL, glucose: 52 mg/dL, protein: 34 mg/dL EEG: no abnormal activity
Diagnosis Seizure disorder, most likely complex partial seizures due to traumatic temporal lobe injury
Treatment Administered The patient was placed on phenytoin to prevent additional seizures The patient was instructed to refrain from driving until stable control of her seizures was achieved
Follow-up The patient experienced no additional seizures after starting phenytoin Attempted weaning of the patient from phenytoin was attempted after 6 months of therapy The patient experienced a similar complex partial seizure during medication weaning and was returned to her prescribed phenytoin dose; she had no new seizures after resumption of her anticonvulsant therapy
Steps to the Diagnosis
Seizure disorder (a.k.a. epilepsy) Recurrent sudden alterations in cortical neurologic activity due to the excessive synchronized discharge of cortical neurons in either a focal or generalized distribution of the brain Causes vary with age (Table 7-10) History: The characteristics of a seizure vary with its subtype (Table 7-11) Simple seizures have no loss of consciousness Complex or generalized seizures are associated with an altered state of consciousness Partial seizures may be associated with hallucinations or déjà vu Physical examination: Partial seizures involve a focal distribution of abnormal neurologic activity Generalized seizures are characterized by a widespread abnormal neurologic activity Tonic-clonic seizures feature a period of tonic muscle contraction followed by clonic contractionrelaxation movements P.203
Table 7-10 Common Causes of Seizures by Age Group
Age Group
Infant
Causes
Hypoxic injury Metabolic defects Genetic/congenital abnormality Infection
Children
Idiopathic Infection Fever
Adult
Trauma Idiopathic Metabolic defects Drugs/withdrawal Trauma Neoplasm Infection Cerebrovascular disease Psychogenic
Elderly
Stroke/cerebrovascular disease Metabolic defects Drugs/withdrawal Infection Trauma Neoplasm
Table 7-11 Types of Seizures
Type
Involvement
Simple partial
H/P
EEG
Focal
Foc al sensory (e.g., paresthesias, hallucinations) or motor (e.g.,
Distinct focal
cortical
repetitive or purposeless movement) activity, no loss of
conductive
region of
c onsc iousness
abnormality in the causative region of
brain
the brain
Complex partial
Focal region
Halluc inations (e.g., auditory, visual, olfactory), automatisms (i.e.,
Focal abnormalities in
of temporal
repeated coordinated movement), déjà vu, impaired consciousness,
temporal lobe
lobe
postictal confusion
Generalized
Bilateral
Sustained contraction of extremities and back (tonic ); repetitive
Generalized
convulsive (e.g.,
cerebral
muscle contraction and relaxation (c lonic ); brief contraction period
electrical
tonic, clonic,
cortex
followed by repetitive contraction-relaxation (tonic -c lonic ); brief
abnormalities
tonicclonic,
repetitive contractions (myoc lonic ); loss of tone (atonic ); loss of
myoclonic, atonic)
consciousness, incontinence, significant postictal confusion, possible focal neurologic deficits lasting several days after seizure (i.e., Todd paralysis) are common
Absence
Bilateral
Brief (few seconds) episodes of impaired c onsc iousness, normal
Generalized three-
cerebral
muscle tone, possible eye blinking, no postictal confusion; more
cycle-per-second
cortex
common in c hildren
spike and w ave pattern abnormalities
EEG, electroencephalogram; H/P, history and physical.
P.204 Tests: EEG is used to detect abnormal neurologic activity but is frequently normal between seizures Patients may be admitted to an epilepsy observation unit to undergo constant EEG and video monitoring to try and record neurologic activity during a seizure MRI is useful to detect intracranial lesions (e.g., tumors, abscesses, hemorrhage) and areas of brain contusion (increased signal on T2-weighted images) Treatment: Anticonvulsant medications are the standardized first-line therapy (Table 7-12) Only one medication is initially prescribed (e.g., phenytoin, carbamazepine, valproate), but a second-line drug may be added for incomplete control under monotherapy Surgery may be considered for resectable foci of seizures but is complicated by the loss of nearby cerebral tissue Vagal nerve stimulator implantation is an option for patients with refractory seizures not willing to undergo surgical resection
Table 7-12 Anticonvulsant Medications Used in Epilepsy Treatment
Drug
Current Indic ations
Adverse Effec ts
Mec hanism: Inhibition of voltage-dependent sodium channels
Carbamazepine
Phenytoin
Lamotrigine
Monotherapy for partial or generalized
Nausea, vomiting, hyponatremia, Stevens-Johnson syndrome,
convulsive seizures
drowsiness, vertigo, blurred vision, leukopenia
Monotherapy for partial or generalized
Gingival hyperplasia, androgeny, lymphadenopathy, Stevens-
convulsive seizures,status epileptic us
Johnson syndrome, confusion, blurred vision
Partial seizures, second-line drug for tonic-clonic
Rash, nausea, Stevens-Johnson syndrome, dizziness, sedation
seizures
Oxcarbazepine
Zonisamide
Monotherapy for partial or generalized
Hyponatremia, rash, nausea, sedation, dizziness, blurred
convulsive seizures
vision
Second-line drug for partial and generalized
Somnolence, confusion, fatigue, dizziness
seizures
Mec hanism: Inhibition of neuronal calcium channels
Ethosuximide
Absenc e seizures
Nausea, vomiting, drowsiness, inattentiveness
Mec hanism: Enhanced GABA activity
Phenobarbital,
Nonresponsive status epilepticus
pentobarbital
Drowsiness, general cognitive depression, vertigo, nausea, vomiting, rebound seizures
Benzodiazepines
Status epileptic us
Drowsiness, tolerance, rebound seizures
Tiagabine
Second-line drug for partial seizures
Dizziness, fatigue, nausea, inattentiveness, abdominal pain
Mec hanism: Inhibition of sodium channels and enhanced GABA activity
Valproate
Monotherapy or second drug for partial and
Hepatotoxicity, nausea, vomiting, drowsiness, tremor,
generalized seizures
weight gain, alopecia
Mec hanism: Inhibition of NMDA-glutamate receptors and enhanced GABA activity
Topiramate
Second-line drug for partial and generalized
Weight loss, cognitive impairment, heat intolerance,
seizures
dizziness, nausea, paresthesias, fatigue
Monotherapy or second-line drug for partial
Sedation
Mec hanism: Unknown
Gabapentin
seizures
Levetiracetam
Monotherapy for partial seizures, second-line
Fatigue, somnolence, dizziness
drug for partial or generalized seizures
GABA, γ-aminobutyric acid; NMDA, N-methyl-D-aspartate.
P.205 Outcomes: Partial seizures have a higher recurrence rate than generalized seizures Mortality rates are twice as high in patients with partial seizures compared to the general population and considerably higher in patients with recurrent generalized seizures Overall prognosis correlates with the success in controlling recurrences through medical or surgical means Status epilepticus Repetitive, uncontrolled seizures without any period of normal consciousness Maintenance of a stable airway, breathing, and circulation is important to survival Intravenous benzodiazepines and aggressive anticonvulsant therapy are required to break the seizure activity Acute mortality is >20% Clues to the diagnosis: History: recurrent abnormal neurologic activity, postictal confusion, preictal auras Physical: automatisms (i.e., arm flapping) Tests: MRI demonstrating a temporal lobe contusion at the site of the former epidural hematoma Postconcussive syndrome A poorly understood constellation of symptoms that follow head trauma and lasts for several months after the initial injury History: symptoms last for a few months and include headache, dizziness, vertigo, tinnitus, nausea, diplopia, anxiety, fatigue, irritability, sensitivity to noise and light, insomnia, or memory impairments Physical examination: examination is frequently normal but may feature cognitive deficits on testing Tests: there are no diagnostic tests but CT or MRI are useful to rule out other pathologies Treatment: pain management and psychotherapy are beneficial during recovery Outcomes: most patients recover within 3 months; up to 15% of patients will have chronic symptoms (more common in patients reporting dizziness) Why eliminated from differential: this patient likely has some degree of postconcussive syndrome given her history of head trauma and her report of headaches and nausea; however, this diagnosis would not explain her unusual automatisms Stroke More thorough discussion in prior case
Why eliminated from differential: this diagnosis is unlikely because the patient has normal neurologic behavior in between her brief episodes Subarachnoid hemorrhage/epidural hematoma/subdural hematoma More thorough discussion in prior case Why eliminated from differential: these diagnoses are ruled out by the appearance of the brain MRI and by the normal lumbar puncture analysis Bacterial/viral meningitis More thorough discussion in prior case Why eliminated from differential: these diagnoses are ruled out by the normal serum WBC count and the normal lumbar puncture analysis
Generalized seizures involve the entire cortex. Partial seizures involve focal neurologic deficits and may progress to secondary generalization (as distinguished from primary generalized seizures).
Anticonvulsant withdrawal is considered for patients after an extended seizure-free period of time but is frequently complicated by a recurrence of seizure activity.
Case 7-6: “My husband's shaking has gotten worse” A 70-year-old man presents to a neurologist for the work-up of hand tremors. He is accompanied by his wife. He says that he is not sure why he is being seen and has to be reminded on several occasions by his wife that he is there to discuss his tremor. The patient's wife says that she first noticed the tremor in her husband's hands 6 months ago, but it has gradually worsened since that time. The tremor seems to be present both at rest and during activity. The tremor nearly resolves while he is sleeping. She is also concerned because he has become unstable on his feet and has fallen on a couple of occasions. He does not participate in the activities he previously enjoyed and seems to forget things easily. She says that she P.206 never saw any of these symptoms until the past year. The patient is able to contribute to the history, but relies on his wife to explain his symptoms. He says that he has a history of HTN and takes enalapril and a baby ASA daily. He formerly was a social drinker but has not consumed alcohol for over a year. On examination, he is a well-nourished man in no acute distress. He shows little emotion during the examination and history. Fundoscopic examination is normal. His face appears symmetric. He has no lymphadenopathy. Auscultation of his heart and lungs detects clear breath sounds and normal heart sounds. His abdomen is nontender with no masses. He has a resting tremor in both hands that consists of a regular beat of wrist and finger flexion. He has increased tone throughout his body, and it is difficult to passively range his extremities. His sensation is grossly intact. When his gait is observed, he has a difficult time initiating his first steps, walks in a shuffling pattern, and takes several extra steps when he tries to come to a stop. His deep tendon reflexes are moderately increased. The following vital signs are measured: T: 98.4°F, HR: 67 bpm, BP: 130/90 mm Hg, RR: 16 breaths/min
Differential Diagnosis Essential tremor, Parkinson disease, normal pressure hydrocephalus, Alzheimer disease, stroke, subdural hematoma, Huntington disease
Laboratory Data and Other Study Results
CBC: WBC: 8.8, Hgb: 13.9, Plt: 231 Chem10: Na: 143 mEq/L, K: 3.7 mEq/L, Cl: 100 mEq/L, CO2: 29 mEq/L, BUN: 19 mg/dL, Cr: 0.9 mg/dL, Glu: 85 mg/dL, Mg: 2.1 mg/dL, Ca: 9.9 mg/dL, Phos: 4.0 mg/dL Head MRI: no intracranial hemorrhage; normal ventricles; normal cortical volume; no focal lesions
Diagnosis Parkinson disease
Treatment Administered The patient was started on levodopa to improve his symptoms
Follow-up The patient experienced an improvement in his symptoms after starting levodopa Five years after initiating medical therapy, the patient began to experience further worsening of his neurologic function He was considered an appropriate candidate for implantation of a deep brain stimulator and underwent the procedure The patient experienced a further improvement in his symptoms after this surgery
Steps to the Diagnosis Parkinson disease An idiopathic disease of dopamine depletion, loss of dopaminergic striated neurons in the substantia nigra, and Lewy body formation (i.e., eosinophilic cytoplasmic inclusions) in substantia nigra neurons, leading to an abnormally increased inhibition of the thalamic-cortical neural pathways A similar syndrome may be seen following repeated blunt injuries to the head (e.g., boxers), MPTP (1-methyl-4-phenyl-1,2,3,6-tetrahydropyridine) intoxication (i.e., a side product of illicit opioid production), or exposure to certain industrial toxins History: memory loss, sleep disturbances, depression, constipation P.207 Physical examination: resting tremor (i.e., “pill-rolling” in the hands), bradykinesia (i.e., decreased voluntary movement) with difficulty initiating movement, masklike face, shuffling gait, involuntary gait acceleration following initiation, “cogwheel” rigidity (i.e., increased tone of agonist and antagonist muscles), postural instability Tests: no lab tests or neuroimaging is reliable for the diagnosis; positron emission tomography (PET) scan is useful for detecting decreased dopamine uptake in the substantia nigra Treatment: Dopaminergic agonists (e.g., levodopa, carbidopa, bromocriptine, amantadine), monoamine oxidase B inhibitors (MAOI; e.g., selegiline), anticholinergic agents (e.g., benztropine), and amantadine play a role in treatment (Table 7-13) Initially a single dopaminergic agonist is prescribed; a second drug may be added if symptoms begin to worsen while on monotherapy
Deep brain stimulation is an approved option for disease that is refractory to medications or progressive in nature Outcomes: as the disease progresses, patients become susceptible to other medical comorbidities (e.g., pneumonia, dementia, falls) that increase their overall mortality rate Clues to the diagnosis: History: memory loss, depression Physical: pill-rolling tremor, gait disturbance, masklike face, cogwheel rigidity Tests: noncontributory
Common signs of Parkinson's disease may be remembered by the mnemonic SMART: Shuffling gait, Masklike face, Akinesia, Rigidity (“cogwheel”), Tremor (resting). Essential tremor Development of a slowly progressive resting or activity-related tremor that is most common in the upper extremities Involves abnormal neural activity in the thalamic-cortical-brainstem pathway History: tremor worsens with stress and emotion, tremor resolves during sleep Physical examination: fixed frequency tremor in either the head or upper extremities that occurs during activity Tests: lab tests and imaging are nondiagnostic and typically are used to rule out other diagnoses Treatment: primidone, propanolol, and anticonvulsants are used to reduce the degree of tremor; thalamotomy or implantation of a deep brain stimulator is considered for severe cases Outcomes: the tremor is progressive and requires occasional adjustments in therapy
Table 7-13 Medications Used in Treatment of Parkinson Disease
Drug
Levodopa
Mec hanism
Dopamine precursor
Indic ations
Initial therapy
Adverse Effec ts
Nausea, vomiting, anorexia, tachycardia, hallucinations, mood changes, dyskinesia with chronic use
Carbidopa
Dopamine decarboxylase
Combined with levodopa to
Reduces adverse effects of levodopa by
inhibitor that reduces levodopa
augment effects
allowing smaller dosage
Increases response to
Hallucinations, confusion, nausea,
levodopa in patients with
hypotension, cardiotoxicity
metabolism
Bromocriptine
Dopamine receptor agonist
declining response
Selegiline
Monoamine oxidase type B
Early disease; may help delay
inhibitor
the need to start levodopa
Nausea, headache, confusion, insomnia
Amantadine
Antimuscarinic agents
Increases synthesis, release, or
More effective against rigidity
reuptake of dopamine
and bradykinesia
Block cholinergic transmission
Adjuvant therapy
(e.g., benztropine)
Agitation, hallucinations
Mood changes, dry mouth, visual abnormalities, confusion, hallucinations, urinary retention
P.208 Why eliminated from differential: the multiple additional symptoms seen in this case besides upper extremity tremor make this diagnosis unlikely Normal pressure hydrocephalus The collection of excessive CSF in the cerebral ventricles and spinal thecal sac May be a sequela of subarachnoid hemorrhage or chronic meningitis History: cognitive impairments, incontinence Physical examination: gait abnormalities Tests: MRI will show enlarged cerebral ventricles, white matter lesions, and aqueduct atrophy Treatment: ventriculoperitoneal shunting is performed to decompress the accumulation of CSF but improves symptoms in a minority of cases Outcomes: the disease carries an unfavorable prognosis in which a minority of cases improve following shunting, and complications involving shunt occlusion are common Why eliminated from differential: the normal appearance of the MRI rules out this diagnosis Alzheimer disease Slowly progressive dementia due to the development of neurofibrillary tangles, neuritic plaques, amyloid deposition, and neuronal atrophy of cortical neurons Risk factors: increased age, family history, Down syndrome, female gender History: progressive short-term memory loss, depression, confusion, inability to complete complex movements or tasks, depression, possible personality changes or delusions Physical examination: typically noncontributory, but spasticity may be seen Tests: CT or MRI demonstrates cortical atrophy Treatment: Cholinesterase inhibitors (e.g., donepezil, rivastigmine, galantamine) and vitamin E may slow progression Memantine may improve symptoms in moderate cases Antidepressants are useful for treating comorbid depression Occupational therapy is useful to prolong independence Outcomes: Dementia is progressive and associated with a worsening ability to perform daily activities and a greater susceptibility to other medical comorbidities
Mortality rates are based on the age at the time of diagnosis Patients diagnosed at 65 may live 10 years Patients diagnosed at 90 survive <5 years Why eliminated from differential: the multiple physical findings detected in addition to the memory impairment make this diagnosis less likely Stroke More thorough discussion in prior case Why eliminated from differential: the normal-appearing MRI rules out this diagnosis Subdural hematoma More thorough discussion in prior case Why eliminated from differential: the normal-appearing MRI rules out this diagnosis Huntington disease An autosomal dominant disease defined by the presence of multiple CAG repeats on chromosome 4 A higher number of CAG repeats correlates with an earlier development of the disease History: symptoms develop in middle age and include dementia, irritability, antisocial behavior, and possible seizures Physical examination: chorea of the extremities (i.e., rapid irregular involuntary movements) Tests: genetic analysis can detect the chromosomal abnormality; CT or MRI will demonstrate atrophy of the caudate nucleus and putamen Treatment: dopaminergic antagonists may improve choreatic movements; genetic screening may be useful for family members to help plan for future cases P.209 Outcomes: the prognosis is poor with mortality occurring <20 years from the time of diagnosis Why eliminated from differential: the movement abnormalities seen in this case are not choreatic in nature, and the patient is older than would be expected for the development of the characteristic symptoms
The signs of normal pressure hydrocephalus may be remembered by the three Ws: Wacky (cognitive impairment), Wet (incontinence), Wobbly (gait abnormalities).
Alzheimer disease is the most common cause of dementia in adults.
In elderly patients with greater than mild memory loss or dementia, Alzheimer disease is the most common explanation.
Distinguish dementia due to Alzheimer disease from that due to multiple cortical infarcts with MRI. Multiple small lesions or infarcts will be apparent on MRI when there is a vascular cause.
Huntington disease has 100% genetic penetrance but does not become symptomatic until middle age.
Case 7-7: “I can't feel my hands or feet” A 35-year-old man presents to an emergency department with numbness and severe pain in both hands and feet for 3 days. He says that he has had a strange feeling in his feet for about a week and his hands for a slightly shorter period of time. Three days ago he began to notice that he was having a difficult time feeling things with his hands. He also noticed that he cut his left foot but does not remember stepping on any sharp objects. Around the same time he says that he began to have weakness in his hands and that his feet seemed to drag on the floor during walking. He has been unable to walk today because of weakness and numbness in his feet. Yesterday he developed a sharp pain in all of his extremities that goes from the hands to the proximal forearm in both arms and from the feet to the knees in both legs. This pain has intensified since yesterday and is now fairly significant. He has been taking ibuprofen for the pain without much effect. He denies that this has happened previously. He says that his vision is starting to become blurry. He denies any headache, neck pain, syncope, nausea, vomiting, or diarrhea. He denies any medical problems, although he had a bad case of diarrhea a month ago for which he was prescribed an antibiotic. He had an open reduction and internal fixation of his right femur 2 years ago for a femur fracture sustained in a motorcycle crash. He drinks socially and smokes a half pack of cigarettes per day. On examination, he is an anxious-appearing man who is in obvious discomfort. He has a slight facial droop on the right side. Fundoscopic examination is normal. His visual fields are normal, but he has difficulty making out fine print because of blurry vision. He has no lymphadenopathy. Auscultation of his lungs detects clear breathing sounds. Auscultation of his heart detects a regular tachycardia. His abdomen is nontender with no masses and few bowel sounds. He has a small laceration on the bottom of his left foot that has minimal erythema and no purulent drainage. He has no rashes or skin lesions. He is unable to detect any light touch sensation in any fingertips up to the level of midforearm in both upper extremities. Normal sensation is not detected until the mid-upper arm region. He has no detectable light touch sensation in either foot up to the midcalf level; his sensation is decreased to some extent up to the midthigh level. He has 2/5 motor function in finger and wrist extension and flexion. His elbow flexion and extension is 4/5 bilaterally. His foot dorsiflexion and plantar flexion is 1/5 bilaterally, his knee flexion and extension is 3/5 bilaterally, and his hip flexion and extension is 5/5. He has impaired proprioception in both hands and feet. He notes significant pain in all extremities, but does not have any worsening symptoms with palpation or range of motion. His pulses are palpable in all extremities. His brachioradialis, biceps, and patellar reflexes are absent. He is unable to walk when gait testing is attempted and is unable to maintain a standing position. The following vital signs are measured: T: 98.4°F, HR: 112 bpm, BP: 105/74 mm Hg, RR: 20 breaths/min
Differential Diagnosis Multiple sclerosis, diabetic neuropathy, stroke, spinal cord compression, brain abscess, amyotrophic lateral sclerosis, myasthenia gravis, syringomyelia, poliomyelitis, Guillain-Barré syndrome, Charcot-Marie-Tooth disease, complex regional pain syndrome, brain neoplasm
Laboratory Data and Other Study Results CBC: WBC: 9.5, Hgb: 14.7, Plt: 224 Chem10: Na: 138 mEq/L, K: 4.0 mEq/L, Cl: 105 mEq/L, CO2: 27 mEq/L, BUN: 15 mg/dL, Cr: 0.6 mg/dL, Glu: 94 mg/dL, Mg: 2.0 mg/dL, Ca: 10.4 mg/dL, Phos: 3.8 mg/dL Head MRI: no intracranial lesions or hemorrhages; normal cortical and ventricular volume Full spine MRI: no disc herniations or sites of cord compression; no spinal cord lesions Following admission to the hospital for pain control and a greater neurologic work-up, the following additional tests are ordered:
Polio-specific antibody: negative Acetylcholine receptor antibodies: negative Tensilon test: no change in motor examination with administration of edrophonium Lumbar puncture: clear appearance, opening pressure 90 mm Hg, WBC: 3/µL, glucose: 57 mg/dL, protein: 65 mg/dL, no oligoclonal bands Electromyography (EMG) (performed on left arm and leg): poor motor recruitment, significantly slowed conduction, prolonged conduction latency
Diagnosis Guillain-Barré syndrome
Treatment Administered The patient was provided supportive care, including aggressive pain control with narcotics and gabapentin Intravenous immunoglobulin (IVIG) was administered to the patient The patient developed respiratory muscle weakness requiring ventilatory support The patient was able to be weaned from the ventilator after 1 week
Follow-up The patient's weakness and numbness slowly improved over a 1-month period until he reported having discernable sensation in his hands and feet and improved motor control The patient was transferred to a rehabilitative hospital to continue his physical recovery with physical and occupational therapy The patient's pain symptoms gradually improved, and he was able to be weaned from analgesics and gabapentin The patient was able to return home after a 2-month rehabilitation course
Steps to the Diagnosis Guillain-Barré syndrome An autoimmune demyelinating disorder of peripheral nerves associated with recent infection, surgery, or vaccination (rare) Camphylobacter, cytomegalovirus, and Epstein-Barr virus are the most common infectious associations History: rapidly progressive bilateral weakness beginning in a “stocking-glove” distribution and proceeding proximally, paresthesias, neuropathic pain, possible history of recent infection, possible dyspnea Physical examination: impaired proprioception, decreased light touch sensation, gait impairment, decreased strength on motor testing, heart rate and blood pressure lability Tests: Increased CSF protein on lumbar puncture MRI may show increased signal at the nerve roots (low specificity) EMG demonstrates signs of demyelination, including slowed nerve conduction, increased latency, blocked
conduction, and poor motor recruitment Treatment: Cases self-resolve (approximately 1 month) but require supportive care in the interim IVIG helps to speed recovery Adequate analgesia is required for neuropathic pain Ventilatory support may be required for patients with respiratory weakness Considerable physical therapy is frequently required during the recovery Outcomes: Mortality is <10% in the acute setting Respiratory compromise is the chief cause of death Up to 85% of patients will recover without any long-term sequelae, but others may have prolonged degrees of motor weakness Clues to the diagnosis: History: progressive weakness and numbness in all extremities in a stocking-glove pattern, neuropathic pain, recent gastrointestinal (GI) infection Physical: impaired motor and sensory function, facial droop, decreased reflexes, gait impairment Tests: increased CSF protein, EMG finding consistent with demyelination Diabetic neuropathy More thorough discussion in Chapter 5 Why eliminated from differential: although many of the symptoms and signs in this case are consistent with this diagnosis, the absence of a history of DM and the normal glucose level make it unlikely Multiple sclerosis A progressive demyelinating disease of the brain and spinal cord related to a possible autoimmune cause Most patients are women between 20 and 40 years old History: The initial symptoms consist of multiple unrelated neurologic complaints including vertigo, visual disturbances, paresthesias, weakness, or urinary retention Symptoms progress slowly but may be worse during stressful events Late symptoms include worsening vision, dysarthria, and significant limits to mobility Physical examination: poor coordination, postural instability, impaired sensation, spasticity, increased deep tendon reflexes, positive Babinski sign (i.e., great toe dorsiflexion with plantar stimulation) Tests: Lumbar puncture demonstrates CSF with increased protein, mildly increased WBCs, oligoclonal bands, and increased immunoglobulin (Ig) G MRI will show multiple white matter lesions in the brain or spinal cord
The McDonald criteria consider clinical and radiographic findings and are used to define the diagnosis (Table 7-14) Treatment: Corticosteroids, methotrexate, and avoidance of stress help to reduce the length of exacerbations β-interferon and glatiramer acetate decrease the frequency of exacerbations Supportive care is required for worsening neurologic dysfunction Outcomes: the course of the disease is progressive with one third of patients becoming severely disabled by 25 years after the diagnosis Why eliminated from differential: the normal-appearing MRIs and the results of the CSF analysis rule out this diagnosis
Remember that Multiple Sclerosis affects the Myelin Sheath and is more common in MS. (women) than in MR. (men).
Highly suspect multiple sclerosis in a young woman with a confusing constellation of neurologic symptoms. Perform an MRI to look for white matter lesions and a lumbar puncture to look for oligoclonal bands. Stroke More thorough discussion in prior case Why eliminated from differential: the normal appearance of the head MRI rules out this diagnosis Spinal cord compression Excessive pressure on the spinal cord or cord transection due to traumatic injury of the spinal columns, development of an abscess or tumor in the epidural or periepidural space, degeneration of the spinal columns or intervertebral discs, or herniation of a ruptured intervertebral disc History: history of associated etiology, patterns of paresthesia and paralysis depending on the level of injury, radicular pain, bowel incontinence, urinary retention Physical examination: ipsilateral deficits of light touch sensation and proprioception below the injury, contralateral deficits of pain and temperature sensation below the injury, ipsilateral motor deficits below the level of the lesion, decreased reflexes at the level of the injury, increased reflexes below the level of the injury
Table 7-14 McDonald Criteria for the Diagnosis of Multiple Sclerosis
At least two episodes of neurologic dysfunction consistent with a demyelinating disease and objective clinical evidence of at least two sites of neurologic lesions
OR
At least two episodes of neurologic dysfunction that are consistent with a demyelinating disease, objective clinical evidence of at least one lesion, and MRI evidence of white matter lesions
OR
One episode of neurologic dysfunction that is consistent with a demyelinating disease, objective clinical evidence of at least two sites of lesions, and either MRI evidence of white matter disease or the later occurrence of an additional episode of neurologic dysfunction
OR
One episode of neurologic dysfunction that is consistent with a demyelinating disease, objective clinical evidence of one lesion, MRI evidence of multiple lesions, and either new lesions on follow-up MRI or the occurrence of new episodes of neurologic dysfunction
OR
Progressive neurologic symptoms over course of 1 year, multiple white matter lesions on MRI, and the presence of oligoclonal bands on CSF analysis
CSF, cerebral spinal fluid; MRI, magnetic resonance imaging.
Tests: CT is useful for detecting bony injuries or abnormalities, but MRI is the most sensitive study for detecting actual cord compression Treatment: IV methylprednisolone is given at a dose of 5.4 mg/kg/hr for 23 hours if the patient presents within 8 hours of an injury; surgical decompression and stabilization (e.g., laminectomy, intervertebral body fusion, disc excision) will need to be performed for cases of compression that do not respond to corticosteroids alone Outcomes: the degree of permanent neurologic injury depends on the degree of cord compromise, the time until treatment, and the ability to successfully restore sufficient space within the spinal column for the cord Why eliminated from differential: the normal MRI rules out this diagnosis Brain abscess/neoplasm More thorough discussion in prior/later cases Why eliminated from differential: the normal MRI rules out these diagnoses Amyotrophic lateral sclerosis (ALS) Progressive loss of both upper motor neurons and lower motor neurons in the brain and spinal cord due to degeneration of anterior horn cells History: asymmetric progressive weakness in the face and extremities, no paresthesias, possible personality changes or impaired judgment Physical examination: tongue and limb fasiculations, positive Babinski sign, increased deep tendon reflexes (DTRs) Tests: EMG demonstrates poor motor recruitment and significant denervation but retained conduction velocity Treatment: riluzole may slow progression of the disease; supportive care is required as neurologic function deteriorates
Outcomes: half of patients die within 3 years of the diagnosis, most commonly related to respiratory complications (e.g., pneumonia, respiratory failure) Why eliminated from differential: the presence of impaired sensation in this case and the results of the EMG (impaired nerve conduction) rule out this diagnosis
Signs of upper motor neuron disease (i.e., brain or spinal cord) include spasticity, increased DTRs, and a positive Babinski sign. Signs of lower motor neuron disease (i.e., peripheral nerves) include flaccid paralysis, decreased DTRs, fasciculations, and a negative Babinski sign.
A clinical diagnosis of ALS requires lower motor neuron signs in at least two extremities and upper motor neuron signs in one region. Myasthenia gravis Autoimmune disorder in which antibodies for acetylcholine receptors bind to receptors at the neuromuscular junction and block normal neuromuscular transmission to cause easy fatigability Frequently associated with thymoma and thyrotoxicosis Most common in young adult women History: periodic weakness and muscle fatigue that worsens over the course of the day, diplopia, dysphagia, possible dyspnea Physical examination: ptosis, facial weakness, possible limb weakness Tests: Presence of acetylcholine receptor antibodies in the serum Positive tensilon test (i.e., administration of edrophonium causes a temporary improvement in symptoms) EMG will show a progressive decrease in motor recruitment over repetitive electric stimulations Treatment: Anticholinesterase agents (e.g., neostigmine, pyridostigmine) improve the function of acetylcholine receptors Thymectomy, plasmapheresis, immunosuppressives, and IVIG may help decrease the autoimmune response Supportive care should be utilized for any difficulties with food consumption or breathing Outcomes: patients frequently respond well to treatment, and mortality is very low; mortality in untreated cases is >25% Why eliminated from differential: the absence of acetylcholine receptor antibodies and the negative tensilon test rule out this diagnosis
Edrophonium is a short-acting anticholinesterase agent, making it ideal for myasthenia gravis testing but ineffective for therapy.
If myasthenia gravis is diagnosed in a patient, always perform a chest CT to look for a thymoma.
Lambert-Eaton syndrome is a paraneoplastic disorder (small cell lung cancer) with a similar presentation to myasthenia gravis. It occurs because of antibodies present in presynaptic calcium channels and is treated with immunosuppressive agents and plasmapheresis. Syringomyelia Posttraumatic cystic degeneration of the spinal cord due to an unknown mechanism A syrinx cavity (i.e., centralized channel within spinal cord) expands and compresses adjacent neural tissue History: weakness and decreased sensation in patterns dependent on the location of the lesion Physical examination: loss of pain and temperature sensation, flaccid paralysis, decreased deep tendon reflexes, and fasciculations at the level of the lesion; loss of proprioception and vibration sense, spasticity, and increased deep tendon reflexes below the level of the lesion Tests: MRI will show the syrinx cavity Treatment: surgical decompression of the cord with possible shunting is required Outcomes: recovery of neurologic deficits worsens as the original size of the syrinx cavity increases; Charcotlike arthropathy may develop in the upper extremities Why eliminated from differential: the spine MRI rules out this diagnosis Poliomyelitis Poliovirus infection of the brain and anterior horn spinal cord neurons, causing motor weakness Acute infections have been nearly eradicated due to childhood immunization protocols History: initially asymptomatic or characterized by headache, neck pain, vomiting, and sore throat Physical examination: fever, muscle weakness, or paralysis in the distribution of the infected neurons, normal sensation Tests: positive presence of polio-specific antibodies in the serum; CSF analysis is consistent with viral meningitis Treatment: supportive care, including possible respiratory support Outcomes: mortality is very low with adequate respiratory support; patients will develop a postpolio syndrome later in life characterized by progressive weakening in the muscle groups that were originally affected by the disease (due to decreased innervation of muscle groups and the impaired ability to regenerate neurons) Why eliminated from differential: the negative serum test and the presence of paresthesias in this case rule out this diagnosis Charcot-Marie-Tooth disease (CMT) A disorder of impaired neural conduction due to axonal degeneration or myelin hypertrophy History: foot weakness, impaired balance, cold feet, mild paresthesias later in life Physical examination: cavus and equinus feet, decreased deep tendon reflexes, lower extremity weakness typically much greater than in the upper extremities Tests: CSF protein is inconsistently elevated; EMG demonstrates slowed nerve conduction velocities Treatment: physical therapy, bracing, and surgical correction of foot deformities help to maintain mobility
Outcomes: there is no increase in mortality associated with the disease, but leg weakness may limit patient mobility Why eliminated from differential: the severity of the symptoms in both the arms and legs, significance of the sensory symptoms, and rapidity of onset make this diagnosis less likely Complex regional pain syndrome (CRPS) (a.k.a. reflex sympathetic dystrophy, RSD) Chronic pain syndrome that occurs after trauma due to abnormal neurologic reorganization at the site of injury Pain fibers may become “wired” into sympathetic neuronal pathways to cause an abnormal activation of pain signals during regular activity History: limb hypersensitivity following injury, increased sweating at the site of injury Physical examination: cyanosis, edema, muscle atrophy, abnormal muscular tone near the injury site, increased localized hair growth Tests: bone scan or MRI may detect decreased perfusion at the site of injury; x-rays may show osteopenia Treatment: Selective nerve blocks may be useful for pain control Physical therapy, transcutaneous electrical nerve stimulation (TENS), and topical anesthetics are all potential therapies during recovery Corticosteroids and anticonvulsants may be useful in refractory cases Referral to a pain management center is frequently required Outcomes: better outcomes are seen with earlier aggressive therapy and in younger patients Why eliminated from differential: the absence of an injury history and the presence of multiple findings besides severe neuropathic pain rule out this diagnosis
Case 7-8: “I can't seem to see out of my right eye” A 28-year-old man presents to a neurologist for the work-up of unilateral blindness and the new onset of difficulty walking. The patient states that he was in his usual state of health until 2 weeks ago when he experienced a sudden inability to walk. At that time, his legs twitched for a few minutes, and he was unable to control them. Afterward, he was able to walk again but felt unsteady for about an hour. The patient did not seek medical attention at that time. Five days ago, the patient experienced a similar episode. Within the past week he has also noticed that he is having difficulty seeing out of his right eye. He became aware of this abnormality when he was almost hit by a car while crossing a street because he did not see it in the right side of his field of vision. He is unsure how long this visual defect has existed. He notes a mild headache behind his right eye that has been fairly persistent for the past several weeks. He also says that he has been feeling rundown and occasionally nauseous for the past month. He denies any loss of consciousness, head trauma, visual auras, grand mal seizures, recent illnesses, or fevers. He denies impaired coordination besides the two episodes described above. He denies any past medical history and takes no medications. He has a beer after work each day and denies any other substance use. On examination, he is a thin-appearing man in no acute distress. He has significant light-red freckling of his face and red hair. He has slight asymmetry of his pupils with his right pupil being 2 mm larger in diameter. Fundoscopic examination detects mild vascular congestion of the right retina but no hemorrhages. Visual field testing shows that he has a significant impairment to the vision in his right eye. He has no lymphadenopathy. Auscultation of his heart and lungs detects clear breath sounds and normal heart sounds. His abdomen is nontender with no masses and normal bowel sounds. Testing of his coordination and proprioception is normal in all extremities. He has full strength and sensation in his arms and 4/5 strength and normal sensation in his legs. His gait is normal. The following vital signs are measured:
T: 98.6°F, HR: 78 bpm, BP: 126/82 mm Hg, RR: 18 breaths/min
Differential Diagnosis Cerebrovascular accident, glaucoma, bacterial/viral meningitis, brain abscess, chronic alcohol use, tension headache, migraine headache, cluster headache, subarachnoid hemorrhage, subdural hematoma, Lyme disease, seizure disorder, multiple sclerosis, brain neoplasm, neurofibromatosis
Laboratory Data and Other Study Results CBC: WBC: 8.7, Hgb: 15.2, Plt: 219 LFTs: AlkPhos: 54 U/L, ALT: 27 U/L, AST: 20 U/L, TBili: 0.7 mg/dL, DBili: 0.3 mg/dL Head MRI: enhancing ring lesion involving a significant portion of the right frontal lobe closely posterior to the right eye; brain edema surrounding the lesion is notable on T2-weighted images; no hemorrhage Spine MRI: no lesions, hemorrhages, or disc herniations Lyme antibody enzyme-linked immunosorbent assay (ELISA): negative The additional following studies are performed: Chest CT: clear lung fields; no lesions, lymphadenopathy, atelectasis, or effusions Abdominal/pelvic CT: normal-appearing viscera; no lesions or hemorrhages Bone scan: no sites of increased signal
Diagnosis Primary brain neoplasm
Treatment Administered Intramarginal debulking was performed to reduce the mass effect of tumor External beam radiation and chemotherapy were initiated postoperatively
Follow-up Histology from the surgical debulking demonstrated poorly differentiated astrocytes, microvascularization, and cell necrosis consistent with glioblastoma multiforme The patient had an initial decrease in the tumor size as demonstrated by a repeat MRI The onset of new seizures 8 months later and the appearance of new tumor growth on a repeat MRI indicated progression of the disease The patient's neurologic function progressively declined after the onset of seizures, and he died 14 months after his diagnosis
Steps to the Diagnosis Primary brain neoplasm Brain tumors that arise within the brain parenchyma or meninges (Table 7-15)
Not due to a distant source More common in young and middle-aged adults, but the relative frequency has increased in the older population Tumors in adults typically occur above the level of the tentorium (i.e., fold of meninges that separates the cerebellum from the cortex) (i.e., supratentorial) Tumors in children typically occur below the tentorium (i.e., infratentorial) Initial symptomatology results from the mass effect of the tumor and may include secondary hydrocephalus, increased intracranial pressure, and venous obstruction History: headache, vomiting, lethargy, weight loss, possible seizures Physical examination: focal neurologic abnormalities, change in mental status, pupil asymmetry Tests: MRI will detect a ring-enhancing lesion with associated cerebral edema (Figure 7-8) PET scan is useful for determining the extent of the tumor Biopsy (either CT-guided or during surgical debulking) provides a histologic diagnosis
Table 7-15 Primary Central Nervous System Neoplasms
Tumor
Meningioma
Astrocytoma
Loc ation
Dura mater
Cortex
H/P
Treatment
Prognosis
Headache, seizures,
Surgical resection,
focal neurologic deficits
radiation
Good
Headache, gradual
Surgical resection,
Median survival
increase in ICP, facial
radiation
7.5 years
sensory and motor deficits
Headache, rapid increase
Surgical resection,
Poor; median
multiforme (i.e.,
in ICP, facial sensory and
radiation,
survival 8 months
high-grade
motor deficits
chemotherapy
Headache, gradual
Surgical resection,
75% 5-year
increase in ICP, facial
radiation
survival if low
Glioblastoma
Cortex
astrocytoma)
Oligodendroglial tumor
Cortex
grade
sensory and motor deficits
CNS lymphoma
Meninges or cortex
Headache, seizures,
Corticosteroids,
Variable; poor in
(immunocompromised
focal neurologic deficits
radiation,
HIV-positive
chemotherapy
patients
Hydrocephalus,
Surgical resection,
Poor
increased ICP
radiation,
patients)
Medulloblastoma
Fourth cerebral ventricle
chemotherapy
CNS, central nervous system; HIV, human immunodeficiency virus; H/P, history and physical; ICP, intracranial pressure.
Figure 7-8 T1-weighted brain magnetic resonance images demonstrating a ring-enhancing lesion in the left temporal lobe consistent with a glioblastoma multiforme.
Treatment: Surgical resection may be attempted but is typically unable to remove the tumor fully Radiation therapy and chemotherapy are performed independent of or in addition to surgery Corticosteroids are useful for decreasing cerebral edema Anticonvulsants may be required for seizure prophylaxis Outcomes: prognosis is poor, and the median survival is 8 months Clues to the diagnosis: History: headache, acute neurologic gait abnormality (likely simple seizures), unilateral blindness, lethargy Physical: mild leg weakness Tests: brain MRI findings of a cerebral lesion Metastatic brain neoplasm Brain tumors that have metastasized from a distant source Lung, breast, kidney, gastrointestinal, and melanoma are the most common types of primary tumors History: headache, vomiting, lethargy, weight loss, possible seizures
Physical examination: focal neurologic abnormalities, change in mental status, pupil asymmetry Tests: Head MRI or CT is able to detect the intracranial mass Body CT, bone scan, CXR, and PET scan should be considered to determine if an extracranial source for the brain mass exists (should be performed before a diagnosis of a primary brain tumor is made) Biopsy can provide more conclusive evidence as to the origin of the neoplasm Treatment: Surgical resection may be performed for single small lesions Treatment of the underlying primary tumor is important to preventing additional metastases Radiation therapy and chemotherapy are useful for decreasing the number and the size of lesions
Table 7-16 Diagnostic Criteria for Neurofibromatosis Type I
At least two of the following findings are required for diagnosis:
At least five café-au-lait macules >15 mm in diameter
At least one neurofibroma or plexiform neurofibroma
Axillary or inguinal freckling
Presence of an optic glioma (i.e., tumor of the optic nerve)
At least one iris hamartoma (i.e., Lisch nodule)
Bone lesions (e.g., cortical thinning, sphenoid dysplasia)
A first-degree relative with a history of neurofibromatosis type I
Outcomes: prognosis is poor with a median survival <1 year Why eliminated from differential: the absence of extracranial lesions on the other imaging studies decreases the likelihood that the tumor is metastatic
Glioblastoma multiforme is the most common primary brain tumor in adults. Medulloblastoma is the most common brain tumor in children.
Metastatic brain tumors are more common than primary tumors.
Most metastases to the brain are supratentorial.
Neurofibromatosis Autosomal dominant disorders characterized by the growth of neurologic tumors Neurofibromatosis type I is most common Characterized by multiple neurofibromas (i.e., tumors with a mix of Schwann cells, fibroblasts, and mast cells) and dermatologic signs (Table 7-16) Due to an abnormal NF1 gene on chromosome 17 Neurofibromatosis type 2 is linked to chromosome 22 and is characterized by the development of bilateral acoustic neuromas History: limitations in movement or gait (due to skeletal dysplasia), visual abnormalities (due to compression of the optic nerve), cognitive impairments, possible seizures Physical examination: freckling, café-au-lait spots, Lisch nodules (i.e., iris hamartomas), palpable neurofibromas, short stature, scoliosis (Color Figure 7-1) Tests: MRI will be able to detect the presence of neurofibromas throughout the body; genetic testing is confirmatory of the condition Treatment: physical therapy should be directed at maintaining function; surgical resection of particularly bothersome masses may be performed Outcomes: potential complications include an increased risk of cerebral malignancy, developmental delays, mental retardation, peripheral neuropathy, pheochromocytoma, vision loss, bony pseudoarthroses and nonunion of fractures, and seizures; life expectancy is slightly decreased compared to unaffected individuals
Color Figure 7-1 Patient with multiple neurofibromas on the face and chest consistent with neurofibromatosis type I.
Why eliminated from differential: facial freckling is not one of the criteria for this diagnosis, and the presence of
the other criteria are not satisfied
Diagnostic criteria for neurofibromatosis type 1 may be remembered by the mnemonic COFFINS: Café-au-lait spots, Optic glioma, Freckling, Familial history, Iris hamartomas, Neurofibromas, Skeletal lesions. Cerebrovascular accident More thorough discussion in prior case Why eliminated from differential: the absence of hemorrhage or cerebral ischemia on the MRI rules out this diagnosis Glaucoma More thorough discussion in Chapter 8 Why eliminated from differential: although this diagnosis could explain the vision loss, it would not explain the lower extremity neurologic symptoms or the presence of the intracranial lesion on MRI Bacterial/viral meningitis, brain abscess More thorough discussion in prior case Why eliminated from differential: the absence of a fever and the normal WBC count make these diagnoses unlikely Chronic alcohol use More thorough discussion in Chapter 13 Why eliminated from differential: despite the patient's admitted history of daily alcohol consumption, the normal LFTs make this diagnosis unlikely; in addition, this diagnosis would not explain the presence of the intracranial mass Migraine/cluster/tension headache More thorough discussion in prior case Why eliminated from differential: the MRI findings make these diagnoses unlikely Subarachnoid hemorrhage/subdural hematoma More thorough discussion in prior case Why eliminated from differential: the absence of hemorrhage on the MRI rules out these diagnoses Lyme disease More thorough discussion in Chapter 9 Why eliminated from differential: the negative Lyme ELISA and the MRI findings rule out these diagnoses Seizure disorder More thorough discussion in prior case Why eliminated from differential: the presence of the brain lesion offers an explanation for the bizarre neurologic symptoms in the patient's legs; if no lesion was detected, an EEG may be useful to determine the origin of the lower extremity symptoms
Multiple sclerosis More thorough discussion in prior case Why eliminated from differential: the absence of cerebral or spinal white matter lesions makes this diagnosis unlikely
Case 7-9: “This patient sustained a severe head injury” A 37-year-old woman is in an ICU after being involved in a high-speed motor vehicle collision. The patient was an unrestrained passenger and was ejected from the car when it hit a telephone pole at a moderately high rate of speed. EMTs found the patient to be unresponsive at the scene but detected spontaneous breathing and palpable peripheral pulses. A thorough trauma evaluation was performed by the trauma team upon the patient's arrival at the hospital's emergency department. This work-up detected a subarachnoid hemorrhage, multiple rib fractures on the patient's right side, a small splenic hematoma, bilateral closed radius and ulna fractures, and a closed left femur fracture. The patient was found to be hemodynamically stable upon arrival but soon developed worsening respiratory effort and was intubated to allow mechanical ventilation. The orthopedic services splinted her arms and placed her left leg in skeletal traction as temporary measures until surgical reduction could be performed. The patient was admitted to the surgical ICU. Despite the absence of sedation, the patient remained unresponsive to stimuli. A repeat head CT the following morning demonstrated a slight increase in the size of the subarachnoid hemorrhage and a diffuse cerebral contusion. A neurologist was called in consultation to evaluate the patient for her level of neurologic function. A discussion with the patient's family revealed that the patient had no history of medical illnesses and only took oral contraceptive pills. She drank alcohol occasionally but was not known to use any other substances. On examination, the patient is unresponsive to any stimuli including pain. Placement of ice water in the patient's ear, movement of the ventilatory tube, and light touch of the corneas elicit no reaction. The patient has ecchymoses at the base of her skull. Her pupils are both relatively dilated with the left being 3 mm greater in diameter than the right. Rotation of the head produces no eye movement, and the pupils are not reactive to light. Auscultation of the heart and lungs detects mildly congested breath sounds and normal heart sounds. Abdominal examination finds a palpable spleen. The patient has no spontaneous movements and does not withdraw from painful stimuli. The following vital signs are measured: T: 98.3°F, HR: 72 bpm, BP: 130/86 mm Hg, RR: 16 breaths/min
Differential Diagnosis Brain death, persistent vegetative state, locked-in syndrome, psychogenic coma, status epilepticus, intoxication, hypothermia
Laboratory Data and Other Study Results CBC: WBC: 12.1, Hgb: 10.2, Plt: 279 Chem10: Na: 139 mEq/L, K: 4.1 mEq/L, Cl: 99 mEq/L, CO2: 27 mEq/L, BUN: 12 mg/dL, Cr: 0.6 mg/dL, Glu: 76 mg/dL, Mg: 1.9 mg/dL, Ca: 9.9 mg/dL, Phos: 4.2 mg/dL Urine toxicology screen: negative for any substances Head CT: subarachnoid hemorrhage with hematoma collection; diffuse brain swelling with near obliteration of the ventricles; no focal lesions Apnea test: no respiratory effort seen within 8 minutes of ventilation stoppage; serial blood gases show partial pressure of carbon dioxide (PaCO2) to increase from 39 mm Hg to 62 mm Hg After these studies are performed, the following test is ordered: EEG: absence of electrical activity during a 30-minute measurement period
Diagnosis Brain death
Treatment Administered The withdrawal of ventilatory support was approved by the patient's family
Follow-up Per the patient's family and the patient's documented wishes, organ donation was allowed to proceed Following organ harvesting, ventilatory support was withdrawn, and the patient was allowed to expire
Steps to the Diagnosis Brain death Complete cessatigts on of electrical brain activity without the possibility of recovery May occur following significant head trauma, cerebral ischemia, or prolonged excessive intracranial pressure History: appropriate occurrence of a causative event, absence of intoxication or hypothermia Physical examination: unresponsiveness, absent brainstem reflexes, no improvement with serial examinations, impaired respiratory drive, normal body temperature Tests: CBC, metabolic panels, LFTs, and toxicology labs help to rule out other diagnoses (Figure 7-9) MRI or CT demonstrates underlying cerebral damage Apnea test
Figure 7-9 Diagnostic approach to the patient in a coma. CN, cranial nerve.
Mechanical ventilation is halted following a baseline arterial blood gas (ABG) The patient is observed for 8 minutes to look for signs of spontaneous respiration A repeat ABG after 8 minutes with a PCO2 >60 mm Hg confirms apnea and an impaired respiratory drive EEG demonstrates no electrical activity Treatment: incompatible with recovery; withdraw of care is typically performed Outcomes: recovery is impossible Clues to the diagnosis: History: severe head trauma Physical: unresponsiveness, absence of brainstem reflexes Tests: appearance of intracranial trauma on CT, EEG without activity, positive apnea test Persistent vegetative state A form of coma lasting greater than a month in which the patient has normal sleep cycles, an inability to perceive and interact with the environment, and preserved autonomic function
History: appropriate cerebral insult (e.g., head trauma, ischemic injury) Physical examination: unresponsiveness, no improvement with serial examinations, brainstem reflexes may be present Tests: Neuroimaging is useful for elucidating the degree of cerebral injury The apnea test may or may not demonstrate retained normal respiration EEG will demonstrate some degree of cerebral activity Treatment: patients may be maintained with supportive care while awaiting potential recovery; withdraw of care may be performed if recovery is considered an unlikely possibility Outcomes: coma lasting >3 months rarely results in recovery Why eliminated from differential: the absence of EEG activity in this case rules out this diagnosis Locked-in syndrome Injury of the brainstem, rendering the patient severely quadriplegic with retained consciousness The patient is still be able to communicate through eye blinks or other upper cranial nerve motor function History: appropriate history of brainstem injury, maintained awareness and communication (may be subtle and easy to overlook), possible ventilator dependence Physical examination: absence of movement or sensation, variability in the presence of brainstem reflexes Tests: neuroimaging is useful to detect the site of insult; EEG shows maintained higher cortical activity Treatment: supportive care including ventilatory support Outcomes: prognosis is poor with rare recovery Why eliminated from differential: the lack of patient interaction and the absence of EEG activity in this case rule out this diagnosis Status epilepticus More thorough discussion in prior case Why eliminated from differential: the absence of EEG activity rules out this diagnosis Intoxication More thorough discussion in prior case Why eliminated from differential: the absence of EEG activity rules out this diagnosis (the apnea test is not able to fully rule out intoxication) Hypothermia A body temperature below 95°F (35°C) due to prolonged cold exposure Risk factors: intoxication, advanced age History: Lethargy, weakness, and severe shivering are early symptoms Confusion and a feeling of false warmth occur with worsening of the condition
Further cold exposure and significant decreases in body temperature result in coma Physical examination: Tachypnea, tachycardia, and loss of coordination are seen with body temperatures 89.6°F to 95°F Mydriasis, hyporeflexia, and bradycardia are seen with body temperatures 82.4°F to 89.6°F Hypotension and a coma mimicking death are seen with body temperatures below 82.4°F Tests: as hypothermia progresses, hypoglycemia, hyperkalemia, increased BUN and creatinine, and increased Hct; ECG may show J waves (i.e., an extra positive deflection immediately after the QRS), ventricular tachycardia (Vtach), or ventricular fibrillation (Vfib) Treatment: the patient must be warmed externally (e.g., warm bed, bath, blankets) or internally (e.g., warm IV fluids or ingested fluids); arrhythmias, respiratory compromise, and any other sequelae must be appropriately treated Outcomes: complications of prolonged cold exposure and hypothermia include arrhythmias, frost bite, pneumonia, pulmonary edema, pancreatitis, and disseminated intravascular coagulation (DIC); while the prognosis for mild cases is good, mortality is >50% for severe cases Why eliminated from differential: although a patient with severe hypothermia may present in the same manner and may have the same physical findings as the patient in this case, this patient is at a normal body temperature
Case 7-10: “My child isn't learning to walk” A 20-month-old boy is brought to his pediatrician by his mother because he has never walked. His mother says he has never made the attempt to walk and rarely stands. He is able to be helped to a standing position but quickly falls without support. He able to pick up objects with a pincer grasp and appears to have normal arm coordination. He interacts well with his parents and is able to play with others when he is sitting on the ground. His mother says that he speaks several words and has recently started combining words into two-word statements. He still requires diaper use, and his parents have not had any success in early toilet training. His mother says that he was the product of a full-term vaginal delivery without any apparent complications. Their family has moved several times in the past 3 years because of work, so she changed obstetricians multiple times before her delivery and the patient has had multiple pediatricians since his birth. She says that because of these changes, she missed several prenatal appointments and her son has only been to the pediatrician four times since his birth. She denies any seizures or history of trauma in the patient. She denies any diseases or substance use during her pregnancy. On examination, the patient is a well-nourished child appearing the stated age. His eyes appear symmetric and equally reactive, and he tracks movement well. He has no rashes or skin lesions but has a small tuft of hair above his gluteal crease. He has no lymphadenopathy. Auscultation of his heart, lungs, and abdomen is normal. He has no abdominal tenderness or masses. He has strong motor function in his arms and has normal upper extremity coordination. He has a paucity of voluntary movement in his lower extremities. The tone in his legs is increased. He reflexively withdraws to pain in both legs but has decreased sensation to light touch. His deep tendon reflexes are mildly increased in the lower extremities. The following vital signs are measured: T: 98.6°F, HR: 90 bpm, BP: 110/70 mm Hg, RR: 18 breaths/min
Differential Diagnosis Cerebral palsy, spina bifida, hydrocephalus, Duchenne muscular dystrophy, spinal trauma, child abuse
Laboratory Data and Other Study Results Length, weight, and head circumference: 32″ (18th percentile), 27 lb (53th percentile), 19″ (52th percentile)
CBC: WBC: 5.1, Hgb: 14.2, Plt: 352 Skeletal survey: no signs of healing fractures or acute injury; missing posterior elements in the lumbar spine Chem7: Na: 139 mEq/L, K: 4.2 mEq/L, Cl: 100 mEq/L, CO2: 26 mEq/L, BUN: 11 mg/dL, Cr: 0.6 mg/dL, Glu: 86 mg/dL Head MRI: no intracranial lesions or hemorrhage; normal ventricle and parenchymal volume Spine MRI: incomplete posterior bony elements over the spinal cord at L1 level; tapering of spinal cord at L1 level; no tethering of the cord; small saccular bulging of the dura and neurologic contents at L1
Diagnosis Spina bifida with small myelomeningocele
Treatment Administered The child was referred to a pediatric neurosurgeon to discuss surgical repair of the myelomeningocele The child was referred to a pediatric orthopedic surgeon to discuss possible bracing and long-term spine follow-up The child was referred to a rehabilitative specialist to discuss physical therapy, bowel and bladder care, adaptive training, and long-term rehabilitative needs
Follow-up The parents were able to keep the child on a bowel and bladder regimen to prevent incontinence The child was able to be trained to walk by the age of 3 years with the assistance of a rolling pediatric walker The child maintained normal intelligence and was able to attend school but required the use of access ramps and an elevator
Steps to the Diagnosis Spina bifida A failure of neural tube closure during fetal development, resulting in multiple neurologic defects following birth The severity of neurologic deficits is dependent on the size and location of the spinal cord defect Spina bifida occulta: most benign variety in which a failure of closure of the posterior elements of the bony spine occurs without spinal cord involvement; it is present in 10% of the population and is not associated with any neurologic deficits Meningocele: herniation of the meninges through a dorsal vertebral defect Myelomeningocele: herniation of the meninges and spinal cord elements through the dorsal vertebral defect, resulting in bowel and bladder incontinence, impaired sensation, flaccid paralysis, and possible hydrocephalus Arnold-Chiari deformity: malformation of the cerebellum and brainstem in which the cerebellum is displaced into the fourth ventricle, the lower brainstem is hypoplastic, and hydrocephalus occurs Anencephaly: severe failure in closure of the upper neural tube in which the absence of the forebrain, meninges, and portions of skull occur; typically incompatible with life beyond a few days Risk factors: poor prenatal folate intake, anticonvulsant use during pregnancy, gestational DM
History: symptoms and their severity depend on the quality of the defect and may include delayed motor or cognitive development Physical examination: possible tuft of hair over the defect or a saccular lesion, impaired lower extremity motor and sensory function, bowel and bladder incontinence Tests: The maternal α-fetoprotein and acetylcholinesterase measured in the prenatal quadruple screen are increased Prenatal US may detect the spinal abnormality MRI in the affected child is useful for detecting the spinal abnormality and any hydrocephalus Treatment: Neurosurgical closure of almost all defects in the meninges and cord elements is required Ventricular shunting of hydrocephalus is required Establishment of a bowel and bladder regimen is needed to treat incontinence Long-term physical therapy and adaptive training is required to optimize mobility and social function In utero repair of the defect is being studied as a means of preventing or lessening the multiple neurologic sequelae Outcomes: Survival and the avoidance of complications correlate with the size of the defect and an early diagnosis Patients with smaller defects and those who receive early treatment live well into adulthood and are able to cope better with any physical limitations Complications include impaired mobility, bladder and bowel infections, fractures from falls, cognitive deficits due to hydrocephalus, and lower limb contractures Clues to the diagnosis: History: failure to walk, normal upper extremity function Physical: lumbar hair tuft, lower extremity motor and sensory impairments, retained reflexes Tests: appearance of the spine x-rays and MRI
All women trying to conceive should receive folate supplementation to reduce the risk of spina bifida.
Spastic cerebral palsy is due to damage of pyramidal tracts. Dyskinetic cerebral palsy is due to extrapyramidal pathology. Cerebral palsy Disorders of motor and possibly cognitive function, resulting from central nervous system damage sustained in utero or during infantile development Most cases result from perinatal complications May be classified as spastic (i.e., spastic paresis of multiple limbs) or dyskinetic (i.e., hyperkinetic
movement abnormalities) Risk factors: prematurity, intrauterine growth restriction, birth trauma, neonatal seizures, cerebral hemorrhage, perinatal asphyxia, multiple gestations, intrauterine infection History: hyperactivity, possible mental retardation, possible seizures Physical examination: Increased tone, increased deep tendon reflexes, weakness, and gait abnormalities are seen in the spastic type Choreoathetoid, dystonic, or ataxic movements and dysarthria are seen in the dyskinetic type Tests: MRI may be useful for detecting a causative lesion, but the diagnosis is typically clinical Treatment: Pharmacologic therapy for spasticity (e.g., botulinum toxin, dantrolene, baclofen, benzodiazepines), bracing of extremities or the back, physical therapy, or surgical contracture releases and reconstructions may all help to improve function Speech therapy may be useful to improve communication Special education is needed for patients with cognitive deficits Social and psychological support of the parents will help ease the burden of care and coordinate the multiple services required for patient care Outcomes: improvements in rehabilitation and coordinated care have increased the life expectancy of these patients well into adulthood, but lifelong care is frequently required on some level to optimize function Why eliminated from differential: although this diagnosis should be seriously considered, the presence of the spinal abnormality provides a better explanation for the child's presentation Childhood hydrocephalus The development of hydrocephalus in children due to either the obstruction of CSF circulation in the ventricles (i.e., noncommunicating) or the dysfunction of subarachnoid cisterns or arachnoid villi (i.e., communicating) History: lethargy, vomiting, poor appetite, irritability, headache, diplopia, delays in walking Physical examination: bulging fontanelles, dilated scalp veins, papilledema, delayed skull suture fusion Tests: US, CT, or MRI will show ventricular dilation Treatment: acetazolamide or furosemide is used to decrease the CSF volume and temporarily lessen symptoms; surgical shunting is frequently required to decompress the ventricles and the associated pressure on the brain Outcomes: complications include cognitive impairments, seizure disorder, and an increased risk of neurologic infections due to shunting; adequate treatment can allow children to lead relatively normal lives, but untreated cases are fatal within 3 years in half of the cases Why eliminated from differential: the brain MRI shows a normal ventricular volume and rules out this diagnosis Duchenne muscular dystrophy An X-linked disorder resulting from a deficiency of dystrophin in muscle Most common lethal muscular dystrophy The onset typically occurs between two to six years of age
Becker muscular dystrophy is a less severe deficiency of dystrophin that presents in similar fshion except that symptoms are less severe History – progressive clumsiness, easy fatigability, difficulty standing and walking, waddling gait, positive Gower's maneuver (i.e., child must push on thighs to stand up) Physical examination – weakness in proximal muscle groups eventually extending to distal muscles, pseudohypertrophy of calf muscles (due to fatty infiltration) Tests – Increased creatine kinase Muscle biopsy will show muscle fiber degeneration, fibrosis, and basophilic staining of fibers Immunostaining for dystrophin shows absence of the protein (reduced but present for Becker muscular dystrophy) EMG will show polyphasic potentials and an increased fiber recruitment Treatment – Physical therapy helps to maintain function Corticosteroids may help to slow progression, and ACE-Is help to decrease cardiac workload Pulmonary support will eventually be required in these patients Outcomes – patients develop progressive cardiopulmonary compromise, scoliosis, and flexion contractures; death typically occurs by before the third decade of life due to respiratory complications Why eliminated from differential – this diagosis is unlikely given that the patient never had normal activity in his lower extremities and that his upper extremities appear unaffected Spinal trauma More thorough discussion in prior case Why eliminated from differential: the absence of signs of a traumatic injury on the skeletal survey or MRI makes this diagnosis unlikely Child abuse More thorough discussion in Appendix D Why eliminated from differential: this diagnosis must always be considered in an unusual presentation, but the skeletal survey and the absence of any cutaneous signs of abuse lessen the concern for this diagnosis; the poor pediatric follow-up on the part of the parents is somewhat negligent, however, and the parents should be counseled on the importance of close follow-up for their child
Case 7-11: “I think my baby had a seizure” A 6-month-old infant girl is brought to an emergency department by her parents after the occurrence of a seizure. A pediatric neurologist is called in consultation to examine the child. The parents say that about 30 minutes prior to their arrival their daughter suddenly became tense and then started twitching for a few minutes. Afterward the infant appeared to be okay and was breathing normally. She started crying soon after this event but was eventually able to be consoled by the mother. The parents say that this has never happened to their child before. They were changing the child's diaper when the event occurred, and they say that there was nothing that occurred beforehand that suggested that something was about to occur. Their child has had a cold for the past few days and has been coughing during this time. They gave her acetaminophen yesterday for a fever of 101.6°F but have not given her any medicine
yet today. They note that the child has been lethargic for the past few days. There has been no change in bowel habits during this time. She was the product of a full-term uncomplicated vaginal delivery, and the mother received the recommended prenatal care. She has met all of her developmental milestones for her age and is up to date on her vaccinations. She has not been diagnosed with any conditions by her pediatrician and takes no medicines besides the children's acetaminophen as described above. On examination, she is a mildly ill-appearing infant in no current distress. She coughs occasionally and has rhinorrhea. Her eyes appear symmetric and track well. She has no lymphadenopathy. Flexion of the legs and neck produces no painful response. Auscultation of her lungs detects mild congestion but audible breath sounds in all lung fields. Auscultation of her heart detects a subtle systolic murmur but no other abnormalities. Her abdomen is soft with no masses and she has normal bowel sounds. She is actively moving all four extremities and withdraws to painful stimuli. The following vital signs are measured: T: 102.1°F, HR: 110 bpm, BP: 105/65 mm Hg, RR: 35 breaths/min (Normal for 6-month infant: HR: 100–170 bpm, BP: 65–115/42–80, RR: 30–50 breaths/min)
Differential Diagnosis Seizure disorder, febrile seizure, viral/bacterial meningitis, encephalitis
Laboratory Data and Other Study Results CBC: WBC: 10.6, Hgb: 14.1, Plt: 287 Lumbar puncture: clear appearance, opening pressure 90 mm Hg, WBC: 2/µL, Glu: 59 mg/dL, protein: 32 mg/dL
Diagnosis Febrile seizure
Treatment Administered The child was administered acetaminophen to reduce her fever and given IV fluids
Follow-up The infant's temperature was 99.0°F when measured 60 minutes after the administration of acetaminophen The family was discharged to home with the instructions to maintain an adequate fluid intake during the illness, to use acetaminophen to control any additional fevers, and to follow-up with the pediatric neurologist to discuss if any additional seizures occurred in the interim period of time The child had experienced no additional seizures by the age of 2 years at the time of the most recent pediatric follow-up
Steps to the Diagnosis Febrile seizures Childhood seizures that occur between the ages of 6 months to 6 years that are associated with the occurrence of a high fever They occur in the absence of neurologic infection, cerebral lesions, metabolic abnormalities, or a history of prior afebrile seizures History: concurrent illness, tonic-clonic seizure lasting <15 minutes
Physical examination: fever >102°F (38.9°C) Tests: Frequently a clinical diagnosis, and an extensive work-up is not required Lumbar puncture should be considered in any child suspicious for meningitis or in children <1 year old because of the difficulty diagnosing meningitis in this age group EEG is not typically performed except for a child with an atypical seizure (consider a laboratory work-up and MRI) Treatment: respiratory function should be confirmed to be stable through examination; acetaminophen should be administered as an antipyretic Outcomes: Thirty-five percent of children with a first febrile seizure have a recurrence The risk of epilepsy in a child with a history of a febrile seizure is mildly greater than the general population Atypical seizures have a greater recurrence rate and a greater association with epilepsy Clues to the diagnosis: History: recent upper respiratory infection, single witnessed seizure, recent fever Physical: evidence of ongoing infection, fever Tests: negative CSF analysis
Febrile seizures are the most common seizures in children.
Consider childhood seizures occurring at a fever temperature <102°F or for longer than 15 minutes to be atypical and requiring a more thorough work-up. Seizure disorder More thorough discussion in prior case Why eliminated from differential: the clinical scenario is more consistent with a febrile seizure, but recurrent seizures warrant a thorough work-up Viral/bacterial meningitis More thorough discussion in prior case Why eliminated from differential: the normal serum WBC count and normal CSF analysis rule out this diagnosis Encephalitis More thorough discussion in prior case Why eliminated from differential: the normal serum WBC count and normal CSF analysis make this diagnosis unlikely
Case 7-12: “My daughter seems to have difficulty hearing things” A 3-year-old girl is brought to her pediatrician by her father with a complaint of ear pain and possible difficulty hearing. The father says that he is concerned that his daughter may have poor hearing from her left ear. He says that she does not respond sometimes when people are speaking to her on the left side. She responds to loud sounds or voices in a quiet room but appears to not hear as well when there is background noise. She also is delayed in meeting the developmental milestones for language and only speaks in three-word sentences. The father says that she appeared to be doing well until about a year ago and had made little improvements since. He also notes that she occasionally rubs her ears or pulls at them. Whenever he or her mother have taken her temperature, she has been afebrile, so they have assumed that she was not sick. He denies any lethargy, bleeding from the ears, vomiting, or diarrhea. He denies any sore throats or fevers in the past year. He says that she was an uncomplicated full-term delivery and has been in good health until the current findings 1 year ago. She is up to date on her vaccinations and only takes a children's vitamin each day. On examination, she is a well-nourished child appearing the stated age. Her eyes are symmetrical and track well. She has no facial abnormalities. She has mild posterior auricular swelling on the left side. Otoscopic examination finds a minimally inflamed right tympanic membrane with a poor light reflex and an inflamed, retracted, hypomobile left tympanic membrane with a poor light reflex. There is no purulent drainage from her ears. Auscultation of her lungs, heart, and abdomen are normal. She actively moves all four extremities and responds to light touch. The following vital signs are measured: T: 98.9°F, HR: 96 bpm, BP: 105/75 mm Hg, RR: 18 breaths/min
Differential Diagnosis Otitis media, otitis externa, viral/bacterial meningitis, congenital hearing loss
Laboratory Data and Other Study Results CBC: WBC: 10.1, Hgb: 15.1, Plt: 198 Tympanometry: high normal impedance in the right external auditory canal; significantly increased impedance in the left external auditory canal Acoustic reflectometry: sound volume greater than the breakpoint in the left ear, suggesting a tympanic effusion
Diagnosis Otitis media with effusion
Treatment Administered The child was placed on a course of oral amoxicillin The patient was referred to an otorhinolaryngologist for placement of myringotomy tubes
Follow-up The patient's parents reported decreased ear pulling after the course of antibiotics A recurrent episode of apparent ear pain prompted a course of amoxicillin/clavulanate Follow placement of the myringotomy tubes, the child's language slowly improved; by the time she was 5 years old, she had caught up in her language milestones
Steps to the Diagnosis Otitis media
Infection of the middle ear caused by Streptococcus pneumoniae, Haemophilus influenzae, Moraxella catarrhalis, Streptococcus pyogenes, or a virus Children are predisposed to the infection because their auditory canal is shorter and more horizontal than that in an adult History: ear pain, possible otorrhea, possible decreased hearing Physical examination: possible fever (two thirds of cases), otoscopic examination shows bulging tympanic membranes, impaired eardrum motility, and a poor light reflex Tests: tympanometry will show a higher impedance across the ear canal due to increased fluid and pressure medial to the tympanic membrane; acoustic reflectometry will demonstrate a greater volume in sounds reflected off the tympanic membrane when there is an underlying effusion Treatment: Amoxicillin is typically considered the first-line antibiotic for acute cases and more chronic cases with an associated effusion Amoxicillin/clavulanate or broader spectrum cephalosporins should be considered for recurrent cases Placement of myringotomy tubes is considered for patients with recurrent cases, cases that do not respond well to antibiotic therapy alone, or cases with associated hearing loss Outcomes: The overall prognosis is excellent Hearing loss frequently improves to some degree following successful treatment A very low number of cases may be complicated by mastoiditis, meningitis, brain abscess, or labyrinthitis Clues to the diagnosis: History: new onset hearing loss, ear pulling Physical: appearance of tympanic membranes on otoscopic examination Tests: results of tympanometry and acoustic reflectometry Otitis externa Infection of the outer ear most commonly due to Staphylococcus aureus, Pseudomonas, or Staphylococcus epidermis More common in swimmers History: painful, swollen ear Physical examination: erythematous and swollen ear canal, tenderness of the pinna, otorrhea Tests: culture of otorrhea may be useful in resistant cases; a temporal bone CT should be considered in patients with worsening symptoms who are a concern for a deeper spread of infection Treatment: Topical polymyxin or neomycin with or without hydrocortisone is applied to the ear canal Oral antibiotics should be given to anyone with symptoms nonresponsive to topical treatments alone Topical drying agents used following water exposure may help prevent recurrent infection
Outcomes: the prognosis is excellent, and cases typically resolve after the initiation of therapy Why eliminated from differential: the appearance of the child's ears is more consistent with otitis media Viral/bacterial meningitis More thorough discussion in prior case Why eliminated from differential: the chronicity of symptoms, normal serum WBC count, and absence of symptoms beyond those concerning the ears make this diagnosis unlikely Congenital hearing loss Inborn hearing deficits frequently associated with a syndromic illness involving abnormal facial and cranial development History: developmental delays in language and communication, presence of an affected relative Physical examination: possible abnormal auditory canal anatomy, possible craniofacial abnormalities Tests: MRI may help detect anatomic auditory abnormalities; audio-evoked brainstem response testing measures EEG changes during the receipt of a sound to detect if that sound is processed Treatment: Children should be provided the appropriate educational and therapeutic resources to optimize cognitive and social development Hearing aids are useful in children who only require amplification of sound to hear normally Cochlear implants may allow children with severe hearing loss to “hear” through stimulation of the auditory nerve Outcomes: these disorders are often difficult to treat, and appropriate therapy and adaptive training are frequently the most important contributions to helping these patient lead more normal lives Why eliminated from differential: the absence of a hearing deficit earlier in life and the absence of any apparent craniofacial abnormalities make this diagnosis less likely
Case 7-13: “I feel so dizzy, and I can't hear right” A 34-year-old woman presents in referral to a neurologist for episodic dizziness over the past week and progressive hearing loss. The patient says that her left ear felt “clogged” a week ago, and she heard a ringing in that ear. Shortly thereafter, she has had difficulty hearing from that ear. She denies any crackling or popping in the ear and denies pain in the ear. She also complains of three episodes of significant dizziness over the past week. Each episode has lasted more than 2 hours. She has felt very nauseous during these episodes and has vomited each time. She has been forced to lie down each time until the dizziness resolves. She denies any headache, neck pain, or fevers. She says that her sister had similar symptoms on a few occasions during middle age, but she was killed in a car collision at the age of 38 years. The patient says that she has no medical problems and takes an oral contraceptive pill in addition to a daily multivitamin. She drinks alcohol and smokes cigarettes socially. On examination, she is a well-appearing woman in no acute distress. Her pupils are symmetric and equally reactive to light. Her fundoscopic examination is normal. Her otoscopic examination demonstrates no tympanic erythema, and her tympanic membranes are mobile. She has no lymphadenopathy. Auscultation of her heart and lungs is normal. She has normal strength and sensation in all four extremities. Her gait is normal. Rapidly moving the patient into a supine position while turning her head does not induce any dizziness. The following vital signs are measured: T: 98.5°F, HR: 73 bpm, BP: 120/75 mm Hg, RR: 16 breaths/min
Differential Diagnosis
Benign paroxysmal positional vertigo, Ménière disease, acoustic neuroma, migraine headache, hypothyroidism, cerebrovascular accident, multiple sclerosis
Laboratory Data and Other Study Results Chem7: Na: 143 mEq/L, K: 3.8 mEq/L, Cl: 101 mEq/L, CO2: 25 mEq/L, BUN: 17 mg/dL, Cr: 0.6 mg/dL, Glu: 86 mg/dL Thyroid panel: TSH: 2.6 µU/mL, T4: 7.2 µg/dL, free T4 index: 7.0, T3: 1.1 ng/mL, T3 reuptake: 0.84 Head MRI: no intracranial lesions or hemorrhages; normal parenchymal and ventricular size Audiometry: impaired hearing for low-register sounds in the left ear
Diagnosis Ménière disease
Treatment Administered The patient was started on acetazolamide for the prophylaxis of dizziness and given a prescription for meclizine to use if her dizziness episodes recurred The patient was told to decrease the amount of salt in her diet
Follow-up The patient remained nearly symptom free and only had one recurrence over the following 5 years
Steps to the Diagnosis Ménière disease (a.k.a. endolymphatic hydrops) Vertigo caused by the distension of the endolymphatic compartment of the ear The cause is poorly understood but a genetic component seems to play a role in the pathology History: acute vertigo lasting several hours, nausea, vomiting, feeling of ear fullness, tinnitus Physical examination: impaired hearing Tests: audiometry demonstrates low-frequency hearing loss Treatment: Anticholinergics, antiemetics, and antihistamines are useful in treating acute exacerbations Acetazolamide and other diuretics can serve a prophylactic role Salt restriction decreases the frequency of exacerbations Surgical decompression of the inner ear may be required in refractory cases Outcomes: the prognosis is inconsistent with some patients responding well to treatments and others suffering significant permanent hearing loss Clues to the diagnosis: History: dizziness, ear fullness, nausea, vomiting, tinnitus, hearing loss
Physical: noncontributory Tests: low-frequency hearing loss seen on audiometry Benign paroxysmal positional vertigo (BPPV) Vertigo caused by a dislodged otolith in the inner ear that interferes with semicircular canal stabilization History: brief episodic vertigo brought on by certain head movements, nausea, vomiting Physical examination: nystagmus, symptoms are induced by the Dix-Hallpike maneuver (i.e., moving the patient from a sitting to supine position suddenly while turning the patient's head to the side) Tests: typically a clinical diagnosis; other tests may be used to rule out other diagnoses Treatment: repositioning maneuvers designed to dislodge the otolith from the semicircular canal can alleviate symptoms Outcomes: prognosis is good with a low recurrence rate Why eliminated from differential: the existence of hearing loss in this case makes this diagnosis unlikely Acoustic neuroma (a.k.a. acoustic schwannoma) Benign Schwann cell tumor of the auditory-vestibular nerve, leading to hearing loss History: hearing loss, dizziness, tinnitus Physical examination: possible unilateral facial palsy Tests: MRI may detect the tumor on the eighth cranial nerve; audiometry shows sensorineural hearing loss Treatment: surgical excision Outcomes: facial weakness resolves following excision; hearing tends to improve to some degree after excision Why eliminated from differential: the absence of a lesion on MRI and the distinct audiometry results make this diagnosis unlikely
Conductive hearing loss: Pathology occurs along conductive pathway from the outer ear to the inner ear Audiometry shows preserved air conduction but consistently low hearing threshold (negative Rinne test) Sensorineural hearing loss: Pathology in neural pathways from the ear to the brain Audiometry shows both impaired bone and air conduction (asymmetric Weber test, positive Rinne test) Migraine headache More thorough discussion in prior case Why eliminated from differential: the absence of headache or visual auras rule out this diagnosis Hypothyroidism More thorough discussion in Chapter 5
Why eliminated from differential: the normal thyroid panel rules out this diagnosis Cerebrovascular accident More thorough discussion in prior case Why eliminated from differential: the normal head MRI rules out this diagnosis Multiple sclerosis More thorough discussion in prior chapter Why eliminated from differential: the absence of white matter lesions on the MRI and the absence of any neurologic findings separate from the left ear make this diagnosis unlikely
Authors: Van Kleunen, Jonathan P. Title: Step-Up to USMLE Step 3, 1st Edition Copyright ©2009 Lippincott Williams & Wilkins > Table of Contents > Chapter 8 - Ophthalmology
Chapter 8 Ophthalmology Basic clinical primer Normal Ophthalmic Anatomy Vasculature The retinal artery and vein supply the retina Vascular pathology is associated with visual defects Neurologic supply The optic nerve (CN II) is responsible for vision The trochlear nerve (CN IV) controls the superior oblique muscle (downward medial gaze, inward eye motion) The abducens nerve (CN IV) controls the lateral rectus muscle The oculomotor nerve (CN III) controls all of the other eye muscles The medial longitudinal fasciculus (MLF) maintains conjugate gaze in one eye when the other eye abducts
Common Visual Abnormalities Pupil defects Typically occur due to an injury or defect along the neural signaling pathway for vision (Table 8-1) Visual field or focal defects Typically occur because of eye shape or neural injury (Figure 8-1, Table 8-2) May be correctable through lenses, visual training, or surgery
Case 8-1: “I woke up, and my eye was all red” A 19-year-old woman presents to her college's student health office after she woke up this morning with a red, painful left eye. She says that she had some mild itching in her left eye when she went to bed the previous night. She also reports that she has never had these symptoms previously. Her eye feels mildly painful, itchy, and teary. She denies any visual difficulties. She denies any problems in her right eye, fevers, headache, neck pain, sore throat, or ear pain. She denies any past medical history and takes no medications. She drinks alcohol socially on the weekends and works three afternoons per week in a children's day care center. On examination, she appears to be in no acute distress. The sclera and conjunctiva of her left eye are inflamed. No purulent discharge is notable, but
the left eye is watery. Her right eye has slight inflammation of the conjunctiva. Her pupils are symmetrical, react equally to light, and track normally. Fundoscopic examination detects normal vascular markings in each eye. A slit lamp examination with fluorescein staining detects no corneal defects. Otoscopic examination detects mobile, pearly tympanic membranes. She has a mildly palpable preauricular lymph node on the left side. Her neck P.211 has no lymphadenopathy and is supple. Examination of her oropharynx is normal. The following vital signs are measured: Temperature (T): 98.8°F, heart rate (HR): 70 beats per minute (bpm), blood pressure (BP): 115/80 mm Hg, respiratory rate (RR): 17 breaths/min
Table 8-1 Common Pupil and Gaze Abnormalities
Abnormality
Argyll-Robertson
Presentation
Cause
Normal accommodation to near objects, nonreactive to light
Syphilis, SLE, DM
Light in the affected pupil causes minimal bilateral constriction, light in the normal pupil
Afferent nerve defect
pupil
Marcus Gunn pupil
causes normal bilateral constriction
Horner syndrome
Ptosis, miosis, anhidrosis
Sympathetic trunk lesion (e.g., Pancoast tumor)
Adie pupil
Minimally reactive dilated pupil
Abnormal innervation of the iris
MLF syndrome
With lateral gaze there is absent contralateral eye adduction
Intracranial lesion, MS
DM, diabetes mellitus; MLF, medial longitudinal fasciculus; MS, multiple sclerosis; SLE, systemic lupus erythematosus.
Differential Diagnosis Conjunctivitis, uveitis, closed-angle glaucoma, corneal abrasion
Laboratory Data and Other Study Results None performed
Diagnosis Viral conjunctivitis
Figure 8-1 Visual field defects resulting from neuronal injury. (See color image.)
P.212
Table 8-2 Common Vision Abnormalities
Disorder
Myopia
Cause
H/P
Refracting power of the eye is too great,
Blurred vision, vision quality worsens
causing the image focal point to be anterior to
as objects move farther away
Treatment
Corrective lenses, laser correction
the retina
Hyperopia
Refracting power of the eye is insufficient,
Blurred vision, vision quality worsens
causing the image focal point to be posterior
as objects move closer
Corrective lenses, laser correction
to the retina
Astigmatism
Asymmetrical cornea surface, causing an
Blurred vision
Corrective lenses
Deviation of an eye that is unable to be
Gaze for each eye is in a different
Vision training, surgery is
overcome by normal motor control
direction, double vision, progressive
frequently required to achieve
blindness
bilateral alignment
inconsistent refraction of light
Strabismus
Amblyopia (i.e.,
Developmental defect in the neural pathways
Poor visual acuity, spatial
Vision training, levodopa,
“lazy eye”)
of the eye
differentiation in the affected eye
carbidopa
H/P, history and physical.
Treatment Administered The patient was provided artificial tears and a 3-day course of ciprofloxacin drops
Follow-up The patient's symptoms resolved over the following 3 days
Steps to the Diagnosis Conjunctivitis Inflammation of the eye mucosa that occurs as a result of viral infection, bacterial infection, or an allergic reaction Typically highly contagious and spread by close contact or through towels and linens Adenovirus is the most common cause Staphylococcus aureus and Streptococcus pneumoniae are common bacterial causes Neisseria gonorrhoeae and Chlamydia trachomatis may be transmitted by sexual contact or to a newborn by an infected mother History: mild eye pain, pruritis, excessive tearing Physical examination: inflamed conjunctiva, possible preauricular lymphadenopathy, possible purulent discharge (bacterial infections) Tests: tests are typically not required, but Gram stain of the discharge is helpful to define a species in bacterial cases Treatment: Typically self-limited, so supportive care (e.g., artificial tears, cold compresses) is the mainstay of therapy Broad-spectrum antibiotic drops decrease the duration of bacterial infections and serve a prophylactic role in viral infections Antihistamines are useful for allergic reactions Prevention through hand washing and proper laundry services is important Outcomes: prognosis is good, but N. gonorrhoeae infections require aggressive therapy to prevent secondary meningitis Clues to the diagnosis: History: itchy, painful, inflamed eye Physical: conjunctival and scleral inflammation, preauricular lymphadenopathy Tests: noncontributory
P.213 Uveitis Inflammation of the iris, choroids, and ciliary bodies due to infectious (e.g., viral, syphilis), autoimmune (e.g., ankylosing spondylitis, juvenile rheumatoid arthritis), or inflammatory (e.g., ulcerative colitis, Crohn disease) conditions May occur as an anterior or posterior eye process History: Anterior uveitis: eye pain, photophobia Posterior uveitis: blurry vision Physical examination: Anterior uveitis: slit lamp examination shows eye inflammation and corneal keratin deposits Posterior uveitis: slit lamp examination shows eye inflammation and retinal lesions Tests: typically not required Treatment: Infectious cases should be treated with antibiotic drops Noninfectious cases will benefit from topical or systemic corticosteroid use Treatment of an underlying condition will help limit or prevent recurrences Outcomes: the prognosis is good with treatment; an acute increase in the intraocular pressure is a rare complication and may cause permanent visual deficits Why eliminated from differential: the normal slit lamp examination makes this diagnosis unlikely Corneal abrasion Traumatic injury of the corneal epithelial surface The injury may occur thorough mechanical means or due to chemical exposure History: appropriate exposure to an irritant, eye pain, photophobia, sensation of a foreign body in the eye, excessive tearing Physical examination: eye inflammation, slit lamp examination with fluorescein staining shows corneal defects Tests: typically not required Treatment: topical antibiotics are administered prophylactically; debridement may be performed for larger abrasions to reduce the risk of ulceration Outcomes: the prognosis is good with most injuries healing in <2 days; inadequately treated abrasions that ulcerate may result in permanent vision deficits Why eliminated from differential: the normal slit lamp examination rules out this diagnosis Closed-angle-glaucoma More thorough discussion in later case
Why eliminated from differential: the normal symmetry and reactivity of the pupils and mild degree of eye pain make this diagnosis unlikely
Case 8-2: “My eyesight is getting really bad” An 84-year-old woman presents to an ophthalmologist with the complaint of progressive vision loss over several years. The patient says that she is unsure of when she first started to have difficulty seeing objects, but she knows that she cannot see things as well as she could 5 years ago. She was last seen by an ophthalmologist at that time for a vision examination and eyeglass prescription. She says that all objects look fuzzy. She especially has problems seeing in rooms with bright lights because of the glow surrounding the lights. She has not driven in the past year because she is afraid of hitting something. She feels that her vision is equally bad throughout her visual fields. She denies eye pain, pruritis, eye discharge, or any eye trauma. She notes a history of hypertension (HTN), hypercholesterolemia, and breast cancer treated with a lumpectomy 10 years ago. She takes losartan, hydrochlorothiazide (HCTZ), and simvastatin as medications. She denies any substance use. On examination, she appears to be in no distress. Her eyes both appear cloudy, with the right being worse P.214 than the left. Her pupils are equally reactive and track well. Fundoscopic examination is difficult but detects one example of arteriovenous nicking in each eye and no evidence of retinal hemorrhage or fragmentation. Visual acuity testing detects significant deficits in each eye, with the right worse than the left. Slit lamp examination detects no corneal lesions but does demonstrate opacity of both lenses. No lymphadenopathy is detected. The following vital signs are measured: T: 98.5°F, HR: 80 bpm, BP: 120/82 mm Hg, RR: 18 breaths/min
Differential Diagnosis Cataracts, glaucoma, macular degeneration, retinal detachment, retinal vessel occlusion
Laboratory Data and Other Study Results Ocular tonometry: normal intraocular pressure in both eyes
Diagnosis Cataracts
Treatment Administered The patient was scheduled for lens replacement surgery of the right eye
Follow-up Following surgery, the patient reported a significant improvement in her right eye vision The patient elected to schedule lens replacement for her left eye following recovery from the first surgery
Steps to the Diagnosis Cataracts Clouding of the lens of the eye, leading to progressive vision loss May occur as an insidious process, following eye trauma, or as a congenital abnormality
Risk factors: trauma (e.g., caustic chemicals), diabetes mellitus (DM), corticosteroid use, advanced age, low education, alcohol use, tobacco use History: progressive hazy and blurred vision, glare around lights, diplopia Physical examination: lens opacity, decreased red reflex (Color Figure 8-1) Tests: none required Treatment: lens extraction and replacement is the definitive treatment for the condition Outcomes: The prognosis is good with lens replacement Patients are at a risk for accidents due to impaired vision prior to treatment Complications from lens replacement surgery may cause permanent visual deficits Clues to the diagnosis: History: progressive vision loss, hazy vision, glare from lights Physical: lens opacity, impaired visual acuity Tests: noncontributory Open-angle glaucoma A gradual increase in intraocular pressure, leading to progressive vision loss Risk factors: increased age, DM, myopia, family history, African heritage P.215 History: gradual loss of visual fields (peripheral to central), halos seen around lights, headache, poor light adaptation Physical examination: Fundoscopic examination shows cupping of the optic disc Slit lamp examination shows corneal thickening with keratin deposits and iris atrophy Visual field testing shows vision loss in the peripheral extent of the visual fields Tests: tonometry demonstrates increased intraocular pressure consistently for several tests performed at intervals over multiple weeks Treatment: Topical β-blockers (e.g., timolol) and α-adrenergic agonists decrease the aqueous humor production to help reduce intraocular pressure Prostaglandin analogues, α-adrenergic agonists, and cholinergic agonists (e.g., pilocarpine) increase aqueous humor removal Laser surgery improves aqueous humor drainage in refractory cases Regular ophthalmologic examinations are the key to prevention in at-risk groups Outcomes: prognosis correlates with the ability to normalize intraocular pressure; poor pressure control results in progressive vision loss and blindness
Why eliminated from differential: the normal tonometry measurements rule out this diagnosis Close-angle glaucoma An acute increase in intraocular pressure due to narrowing of the anterior chamber angle that causes obstructed drainage of aqueous humor from the eye Risk factors: increased age, hyperopia, dilated pupils, Asian heritage History: severe eye pain, blurry vision, halos seen around lights, nausea, vomiting Physical examination: inflamed and hard eye, dilated and nonreactive pupil Tests: tonometry demonstrates an increased intraocular pressure Treatment: Acetazolamide and β-blockers acutely decrease the intraocular pressure Pilocarpine is administered after pressure reduction to reduce the degree of obstruction to aqueous humor drainage Laser iridotomy should be performed to prevent recurrences and may be performed prophylactically on the unaffected eye Outcomes: permanent vision loss will result without prompt treatment Why eliminated from differential: the mild degree of eye pain, normal reactivity of the eye, and normal intraocular pressure rule out this diagnosis Macular degeneration Atrophic (slow) or exudative (rapid) degeneration of the retina leading to retinal fibrosis and permanent vision loss Risk factors: tobacco use, family history, white race, increased age, prolonged sun exposure, HTN, female gender History: painless gradual vision loss (central to peripheral) for all distances Physical examination: fundoscopic examination shows either increased retinal pigmentation (atrophic type) or retinal hemorrhage (exudative type) in the macula and possible retinal detachment Tests: fluorescein angiography may demonstrate neovascularization of the retina and membrane formation Treatment: Intravitreal ranibizumab may help treat exudative lesions near the fovea Laser photocoagulation of discrete lesions may delay progression Outcomes: prognosis is poor for the exudative type with a gradual loss of vision; the atrophic type is associated with a slower progression of vision loss Why eliminated from differential: the findings of the fundoscopic examination make this diagnosis unlikely Retinal detachment Separation of the retina from the adjacent epithelium leading to an acute loss of vision Risk factors: trauma, cataract surgery, myopia, family history P.216
History: painless acute loss of vision, numerous “floaters” or the feeling of a “window shade pulled over the eye” Physical examination: fundoscopic examination shows retinal fragmentation or a gray retina floating in the vitreous humor Tests: none required Treatment: laser photocoagulation or cryotherapy is performed to halt tear progression; intraocular repair is performed to reattach the retina Outcomes: the degree of retinal involvement and the time until repair correlate with the degree of vision loss; vision may not be fully restored following retinal reattachment Why eliminated from differential: the findings of the fundoscopic examination and the gradual onset of the vision loss make this diagnosis unlikely Retinal vessel occlusion Occlusion of a retinal artery or vein resulting in the sudden loss of vision Most commonly occurs as a sequela to atherosclerosis, DM, HTN, or thromboembolic disease (Color Figures 8-2, 8-3, and 8-4) History: Retinal artery occlusion: sudden, painless loss of vision Retinal vein occlusion: slow, painless loss of vision Physical examination: Retinal artery occlusion: fundoscopic examination shows a cherry-red spot in the fovea and poor arterial filling Retinal vein occlusion: fundoscopic examination shows cotton wool spots, retinal edema, retinal hemorrhages, and dilated veins Tests: none required Treatment: Thrombolysis of arterial occlusions should be performed within 8 hours of onset Acetazolamide and oxygen administration are used to decrease retinal venous congestion and increase arterial perfusion Laser photocoagulation may be useful for venous occlusion Outcomes: the degree of vision loss correlates with the time until treatment occurs Why eliminated from differential: the findings of the fundoscopic examination make these diagnoses less likely, but persistent vision deficits following lens replacement for the obvious cataracts should warrant a more extensive work-up given the patient's past medical history
Open-angle glaucoma is more common than closed-angle glaucoma.
Any patient who requires frequent changes of lens prescriptions
should be suspected for having glaucoma, and tonometry should be performed to rule out the condition.
Closed-angle glaucoma is typically unilateral.
Never induce additional pupil dilation during an examination of the patient with suspected closed-angle glaucoma because it will acutely worsen the condition.
Case 8-3: “My son has a white eye” A 3-year-old boy is brought to an ophthalmologist by his mother for evaluation of his right eye. The patient's mother says that when she took a photo of her son 2 weeks ago she noticed that one of his eyes had a white spot in the center of the pupil. When she looked at his eyes afterward she did not notice any abnormality but found that the white spot was reproduced when a light was shined in his eyes. She says that her son has not been complaining of pain in his right eye and has not been rubbing it. He has not been behaving strangely lately. He has met all of his developmental milestones and has no problem recognizing colors and objects. He is not accident prone and does not fall more than is expected for his age. He has not had any fevers, recent colds, or trauma to his right eye. He was the product of a full-term vaginal delivery that was uncomplicated. He has no medical problems and is up to date on his vaccinations. There is no family history of eye diseases. On examination, he is a well-appearing child in no distress. When light is shined in his eyes, the left eye has a normal red reflex, but the right eye has a central white appearance. His pupils react normally, and he is able to track objects well. His conjunctiva are pink, and his sclera are not injected. Fundoscopic slit lamp examination detects a small whitish growth involving the posterior wall of the retina. There are no corneal or lens defects. P.217 There are no arteriovenous malformations or hemorrhages. There is no lymphadenopathy. The following vital signs are measured: T: 98.5°F, HR: 85 bpm, BP: 114/73 mm Hg, RR: 18 breaths/min
Differential Diagnosis Retinoblastoma, uveitis, congenital cataract
Laboratory Data and Other Study Results Head computed tomography (CT): small mass of the posterior right eye globe without extension into the posterior orbit; normal cerebral and ventricular size; no hemorrhage Retinoblastoma (RB1) gene testing: mutation in the patient's RB1 gene; no mutation found in either parent
Diagnosis Retinoblastoma
Treatment Administered Laser photocoagulation was performed on the lesion
Follow-up Examination confirmed complete destruction of the tumor
As the child grew older, it was confirmed that he had a visual acuity deficit in the right eye that was treated with glasses and a small central visual field defect that created a blind spot The child was able to learn normally in school given his maintained left eye vision The family was counseled that the patient's genetic abnormality was a new defect and that future children were at no increased risk for the disease
Steps to the Diagnosis Retinoblastoma Malignant tumor of the retina found most commonly in children Most common intraocular tumor in children Cases may be due to a new mutation in the RB1 gene or a hereditary gene defect History: patients are frequently initially asymptomatic but may have vision loss or eye inflammation Physical examination: poor red reflex in the affected eye, possible white spot in the eye (i.e., leukokoria), fundoscopic examination may detect a white mass in the retina (Color Figure 8-5) Tests: genetic testing is useful for detecting mutations in the RB1 gene; CT, ultrasound (US), or magnetic resonance imaging (MRI) is needed to determine the full extent of the lesion and to detect any calcification of the tumor Treatment: Enucleation is performed for large tumors with no vision potential External beam radiation may be used for bilateral tumors or those near the optic nerve but has been associated with stopping facial bone growth or causing other forms of cancer Cryotherapy and laser photocoagulation may be used for smaller tumors in the posterior retina Chemotherapy is used in metastatic disease and for large tumors P.218 Outcomes: overall survival is 85%, but this rate decreases with extension of the tumor posterior to the globe Clues to the diagnosis: History: leukokoria in photo Physical: leukokoria, white mass on fundoscopic examination Tests: head CT demonstrating the lesion Uveitis More thorough discussion in prior case Why eliminated from differential: the absence of eye inflammation makes this diagnosis unlikely Congenital cataract More thorough discussion in prior case Why eliminated from differential: the slit lamp examination rules out this diagnosis
Authors: Van Kleunen, Jonathan P. Title: Step-Up to USMLE Step 3, 1st Edition Copyright ©2009 Lippincott Williams & Wilkins > Table of Contents > Chapter 9 - Orthopedics and Rheumatology
Chapter 9 Orthopedics and Rheumatology Case 9-1: “My hands feel numb” A 49-year-old woman presents to her primary care provider (PCP) with a 1-year history of progressive numbness and tingling in her hands. She says that she first noticed tingling in her hands during her work as a secretary about a year ago. Her symptoms have gradually worsened, and she now reports frequent numbness in her hands and weakness when gripping items. She says that her symptoms are typically mild at rest but become worse with activity. She says that she is right-handed and that her symptoms are slightly worse in this hand. She denies any numbness proximal to her hands. She denies any trauma to her upper extremities or neck and has no neck or back pain. She denies headaches and dizziness. She has a past medical history for type 2 diabetes mellitus (DM) and takes metformin for glucose regulation. She denies any allergies. She has an occasional alcoholic beverage but denies other substance use. On examination, she appears to be in no distress. Her face and body appear symmetrical. She has no lymphadenopathy. Examination of her upper extremities is notable for slight thenar muscle wasting in the right hand. She has 5/5 strength with all active ranges of motion in the upper extremities. She has subjectively decreased sensation to light touch in the palmar surface of both index fingers and the medial side of her palms, but her sensation is otherwise normal. She has strong pulses in both arms. Tapping her volar wrists elicits a sharp pain in both hands. Tapping on the medial side of her elbow does not elicit this response. Provocational maneuvers of her neck do not elicit such symptoms. When her wrists are held in a flexed position for over a minute, she reports numbness in both hands. She has no pain on palpation of her back and spine. No rashes or lesions are visible on her body. The following vital signs are measured: Temperature (T): 98.6°F, heart rate (HR): 72 beats per minute (bpm), blood pressure (BP): 128/78 mm Hg, respiratory rate (RR): 14 breaths/min
Differential Diagnosis Carpal tunnel syndrome, cervical disc herniation, cervical spinal stenosis, brachial plexopathy, diabetic neuropathy, multiple sclerosis
Laboratory Data and Other Study Results 7-electrolyte chemistry panel (Chem7): sodium (Na): 138 mEq/L, potassium (K): 4.2 mEq/L, chloride (Cl): 106 mEq/L, carbon dioxide (CO2): 27 mEq/L, blood urea nitrogen (BUN): 14 mg/dL, creatinine (Cr): 0.7 mg/dL, glucose (Glu): 75 mg/dL Electromyelogram (EMG)/nerve conduction studies (NCS): prolonged conduction latency, decreased amplitude of action potentials, and fibrillation potentials distal to the wrist only in both upper extremities P.220
Diagnosis Carpal tunnel syndrome
Treatment Administered
The patient was referred to an occupational therapist to be fit with wrist splints and to be instructed in activity modification to avoid inducing recurrent symptoms
Follow-up After 1 month, the patient reported no improvement with nonoperative treatments, so she was referred to an orthopedic surgeon to discuss surgical releases of the transverse carpal ligaments Following carpal tunnel releases the patient experienced a notable improvement in her symptoms, and her function improved
Steps to the Diagnosis Carpal tunnel syndrome A syndrome of median nerve dysfunction, resulting from compression of the nerve at the wrist by the transverse carpal ligament (Figure 9-1) More common in 30- to 55-year-olds Risk factors: pregnancy, rheumatoid arthritis (RA), DM, acromegaly, hypothyroidism, obesity, overuse activity (e.g., typing, writing, piano playing), female gender History: wrist pain that worsens with grip and hand flexion, numbness in thumb and index and long fingers Physical examination: decreased grip strength, decreased radial-sided palmar two-point discrimination, positive Tinel sign (i.e., tapping over median nerve elicits tingling and shocking pain), positive Phalen sign (i.e., placing the dorsal surfaces of the hands together and flexing the wrists induces symptoms), thenar muscle atrophy Tests: EMG and NCS will show increased conduction latency, fibrillation potentials, and a decreased conduction amplitude below the wrist only Treatment: Wrists splints, activity modification, and nonsteroidal anti-inflamatory drugs (NSAIDs) are the common first-line treatments Corticosteroid injections through the transverse carpal ligament may help symptoms resolve in mild cases
Figure 9-1 Tendinous and neurovascular structures superficial and deep to the transverse carpal ligament in the wrist. (See color image.)
P.221 Surgical release of the transverse carpal ligament is required in cases that do not respond to nonoperative management Outcomes: ninety percent of patients will experience an improvement in their symptoms following surgical release; complete restoration of strength and sensation may not occur in long-standing cases Clues to the diagnosis: History: numbness and weakness in hands, work as a secretary, history of DM, absence of symptoms above the wrist Physical: thenar muscle atrophy, positive Tinel and Phalen signs Tests: EMG/NCS findings Spinal stenosis A generalized narrowing of the spinal foramina due to arthritic changes Results in the symptomatic compression of exiting nerve roots More common in middle-aged and older adults History: radiating pain that corresponds with the distribution of the compressed nerve root (i.e., radiculopathy), pain may worsen with standing and walking Physical examination: chronic nerve compression may result in motor and sensory deficits, abnormal gait and loss of lumbar lordosis are seen when occurring in the lumbar spine Tests: x-ray or spinal magnetic resonance imaging (MRI) is used to confirm the diagnosis and to determine the degree of nerve impingement Treatment:
NSAIDs and physical therapy are first-line treatments Epidural injections of corticosteroids or anesthetic agents may be used in cases not responding to conservative treatment Surgical decompression (e.g., laminectomy, foraminectomy) may be performed in patients not responding to nonoperative treatments Outcomes: myelopathy may occur in patients with long-standing nerve compression; progression of nerve compression occurs in almost all patients with lumbar stenosis and one third of patients with cervical stenosis Why eliminated from differential: the limitation to below the wrist for the patient's symptoms and the EMG/NCS abnormalities makes this diagnosis unlikely Cervical disc herniation Degeneration of the vertebral discs, leading to herniation of the nucleus pulposus from the disc and subsequent nerve impingement The normal disc is composed of a ring of dense fibrous tissue (i.e., annulus fibrosis) enclosing a gelatinous core (i.e., nucleus pulposus) Disc herniation is most common in the lumbosacral region and second most common in the cervical spine History: pain extending along the path of the compressed nerve, motor and sensory deficits with chronic compression (Table 9-1) Physical examination: Valsalva maneuver causes a worsening of symptoms, patients with cervical herniations have a decreased neck range of motion and a positive Spurling test (i.e., compression, lateral bending, and extension of the neck produces radicular symptoms), patients with lumbosacral herniations have a reproduction of their symptoms with a straight leg raise and have an abnormal gait Tests: MRI is used to confirm the presence of a herniation (Figure 9-2) Treatment: Some cases are self-limited and may be treated with analgesics, NSAIDs, and activity modification Epidural injections of anti-inflammatory agents may be used in cases not resolving in a timely manner Surgical discectomy and possible decompression may be performed in cases not responding well to nonsurgical treatment or in cases with an acute onset of significant neurologic deficits Outcomes: many cases can be treated nonoperatively, but recurrence is more common in lumbosacral herniations; myelopathy can result from chronic nerve compression P.222
Table 9-1 Neurologic Deficits Seen with Compression of Specific Cervical and Lumbosacral Nerve Roots
Nerve Root
Reflex
Motor Defic it
Sensory Defic it
C5
Biceps
Deltoid, biceps
Anterior/lateral shoulder
C6
Brachioradialis
Biceps, wrist extensors
Lateral forearm
C7
Triceps
Triceps, wrist flexors, finger extensors
Posterior forearm
C8
None
Finger flexors
4th and 5th fingers, medial forearm
T1
None
Finger interossei
Axilla
L4
Patellar
Tibialis anterior (foot dorsiflexion)
Medial leg
L5
None
Extensor hallucis longus (1st-toe dorsiflexion)
Lateral lower leg, 1st web space
S1
Achilles
Peroneus longus and brevis (foot eversion), gastrocnemius (foot plantar
Lateral foot
flexion)
Why eliminated from differential: symptoms related to nerve root compression would be expected to be manifested along the length of the nerve and not just below the wrist Brachial plexopathy Traumatic injury of the brachial plexus, resulting in a peripheral neuropathy involving the distinct injured nerve roots (Figure 9-3)
Figure 9-2 Magnetic resonance imaging of lumbar spine, demonstrating herniation of the L5–S1 disc (arrows) and the resulting compression of the spinal cord. (See color image.)
P.223
Figure 9-3 Diagram of the major neural branches of the brachial plexus. (See color image.)
History: pain, weakness, and paresthesias corresponding to the injured nerve roots (Table 9-2) Physical examination: motor and sensory deficits correspond to the injured nerve roots, contractures may develop from chronic denervation of muscular units Tests: EMG and NCS are useful to isolate the nerves affected and to determine the degree of injury; MRI may be able to detect sites of nerve rupture or compression Treatment: physical therapy is important to maximizing the degree of function attainable for a given nerve injury; tendon transfers, nerve grafting, or nerve transfers may be performed in hopes of improving the function of the affected limb Outcomes: Recovery corresponds to the degree of nerve injury; traction injuries (i.e., neuropraxia) have the best chance of recovery, while nerve disruption (i.e., neurotmesis) is associated with a partial recovery of
function at best P.224
Table 9-2 Common Brachial Plexopathies
Condition
Erb-Duchenne
Site of Injury
Superior trunk
palsy
Claw hand
Ulnar nerve
Cause of Injury
Clinic al Features
Hyperadduction of the arm causing widening
W aiter's tip (i.e., arm extended and
of the humeral-glenoid gap (e.g., birth,
adducted with pronated forearm), lateral
shoulder dy stoc ia)
shoulder paresthesia
Epiphyseal separation of medial epicondyle of
Weak finger adduction, poor 4th/5th-
humerus
finger flexion, clawed 4th/5th fingers from lumbric al w eakness
Wrist drop
Deltoid
Posterior
Midhumerus fracture causes nerve
Inability to extend the wrist and fingers,
cord/radial nerve
impingement or tear
loss of sensation from the dorsal hand
Axillary nerve
Anterior shoulder disloc ation causes
Impaired shoulder abduction and
axillary nerve impingement or stretching
elevation, lateral shoulder paresthesia
Posterior/medial
Hyperabduction of the arm places excess
Claw hand, poor wrist and hand function,
cords
tension on the lower cords and nearby
association with Horner syndrome
paralysis
Klumpke palsy
sympathetic chain
Functional recovery is best in young children Nerve regeneration in nerves with some degree of integrity proceeds at about 1 mm per day Why eliminated from differential: symptoms would be expected to occur along the length of the nerve and not just below the wrists Diabetic neuropathy More thorough discussion in Chapter 5 Why eliminated from differential: hyperglycemia would be expected in a patient with DM, and symptoms would not be expected to be limited to only below the wrists Multiple sclerosis More thorough discussion in Chapter 7 Why eliminated from differential: although the absence of any other neurologic complaints makes this diagnosis unlikely, it should be considered if the patient does not respond to any prescribed treatments
The organization of the brachial plexus may be remembered by the mnemonic “Real Texans Drink Cold Beer”: Roots, Trunks, Divisions, Cords, Branches (proximal to distal).
Case 9-2: “I really hurt my ankle” A 35-year-old woman presents to an emergency department after falling off a 6-foot ladder while painting a bedroom
wall. The patient says that she landed on her left leg and immediately experienced severe ankle pain. She has been unable to bear weight on her ankle since her fall. She denies any numbness in her foot but is reluctant to move her ankle or toes because of the significant amount of pain. She denies having ever had a similar injury to her ankle in the past. She has no significant past medical history and only takes oral contraceptive pills. She drinks alcohol socially and denies other substance use. On examination, she appears to be in a significant amount of pain from her injury. Inspection of her ankle finds a large of amount of swelling and an abnormal position of the ankle. There are no lacerations around the site of injury. She has palpable dorsalis pedis and posterior tibial pulses and brisk capillary refill. She is able to detect light touch throughout her foot. With encouragement, she is able to flex and extend her toes, but she will not move her ankle due to pain. She has no pain with palpation or range of motion of her knee and hip, but palpation of her lateral lower leg and her medial malleolus is painful. The following vital signs are measured: T: 98.9°F, HR: 110 bpm, BP: 130/90 mm Hg, RR: 14 breaths/min P.225
Differential Diagnosis Fracture, dislocation, sprain
Laboratory Data and Other Study Results Ankle x-ray: displaced fractures of the distal tibia and fibula; no joint dislocation
Diagnosis Ankle fracture
Treatment Administered Closed reduction was performed, the patient was placed in a splint to immobilize her ankle, her leg was elevated, ice was applied to her ankle, and she was admitted to the hospital Open reduction and internal fixation of the patient's ankle was performed the following day
Follow-up The patient had an uncomplicated postoperative course and demonstrated good bony healing in her follow-up x-rays The patient was able to return to full weight-bearing by 2 months after surgery
Steps to the Diagnosis Fracture Injury to the bone resulting in a discontinuity of the bony architecture (Table 9-3) Most commonly results from trauma, but may also be due to pathologic instability History: pain at the site of injury, inability to bear weight or resist a force Physical examination: swelling at the site of injury, ecchymoses, pain with palpation or range of motion, crepitus at the site of injury, deformation of the extremity at the site of injury Tests: x-rays are the standard test for detecting fractures; incomplete or nondisplaced fractures may be diagnosed with bone scan or MRI when x-rays are inconclusive
Treatment: Nondisplaced fractures, fractures not extending into a joint, and fractures in non–weight-bearing locations may be treated with immobilization, protected activity, and analgesics Displaced fractures and those extending into a joint require surgical fixation and reduction Open fractures (i.e., fractures with a full-thickness soft tissue injury and exposure of bone to air) require urgent irrigation and debridement prior to surgical fixation Outcomes: The majority of fractures will heal with adequate stabilization Nonunion or malunion of a fracture may result from inadequate stabilization or vascular insufficiency at the fracture site Open fractures are associated with a significantly increased risk of infection Clues to the diagnosis: History: traumatic injury, ankle pain, inability to bear weight Physical: ankle deformity, pain on palpation and movement Tests: x-ray findings
The Ottawa Ankle Rules should be applied to any blunt foot or ankle trauma. If there is ankle pain and bone tenderness in either malleoli, an ankle x-ray should be performed. If there is foot pain and tenderness at either the base of the fifth metatarsal or the navicular bone, get a foot x-ray.
Most shoulder dislocations are anterior in nature and associated with trauma.
Posterior shoulder dislocations are associated with seizures and electrical shock injuries. Joint dislocation Incongruency of the articular surfaces of a joint due to trauma, deficient stabilizing structures, or ligamentous instability P.226
Table 9-3 Common Fractures, Their Mechanism of Injury, and Appropriate Treatment
Type
Clavicle
Bones Involved
Clavicle
H/P
Fall onto the shoulder
Treatment
Immobilization in a sling; surgery is indicated for displaced fractures
Clinic al Pearls
Most c ommonly frac tured bone
Humerus
Humerus
Trauma (e.g., motor
Closed reduction and
Radial nerve injuries may occur
vehicle accident, blunt
immobilization; possible surgery
with midshaft fractures due to
trauma, etc.); more
the proximity of the nerve to the
common in elderly
bone
patients
Colles
Distal radius +/-
Fall on an outstretc hed
Closed reduction and
Most common wrist fracture;
distal ulna
hand; “dinner fork”
immobilization; surgery is
particularly common in
deformity of the distal
frequently required
osteoporotic bone
Fall onto a radially
Immobilization for nondisplaced
Possible avascular necrosis of the
deviated outstretched
fractures; surgery is frequently
bone; not seen on x-ray for 1–2
hand; “snuffbox”
required
w eeks after injury ; most
forearm
Scaphoid
Scaphoid
common carpal fracture
tenderness
Boxer
5th-metacarpal
Punching a hard object or
Closed reduction and
Beware the “fight bite,” an open
neck
surface with a strong
immobilization; surgery is
wound from impaction on a tooth
force applied to 5th
occasionally required
that will need surgical exploration and debridement to prevent
metacarpal
infection
Rib
Nonfloating ribs
Trauma; pain worse
Pain control
during deep breathing
Multiple rib fractures may result in a “flail chest” with a significant impairment of breathing due to pain
Vertebral
Components of the
Trauma, osteoporosis
Limited and stable fractures
Unstable fractures may be
spine
(c ompression frac tures)
only require pain control;
associated with spinal cord injury
fractures causing instability of the spinal column require surgical fixation
Pelvic
Pelvis
Significant trauma
Pain control; surgery is required
Risk of vascular injury increases
if in a weight-bearing portion of
with the severity of the fracture
the pelvis or for unstable fractures
Femur
Femur
Trauma
Surgical fixation
The thigh may be a site of significant blood loss that is not readily apparent on examination
Tibial
Tibia
Trauma
Immobilization for nondisplaced
Increased risk for c ompartment
fractures; surgery for displaced
syndrome
fractures
Ankle
Medial, lateral,
Trauma; excessive
Immobilization or surgery
May lead to posttraumatic ankle
and/or posterior
twisting of the ankle
depending on the degree of
arthritis; the Ottawa Ankle Rules
displacement and joint
are useful to making the
involvement
diagnosis
malleoli
H/P, history and physical.
History: a history or trauma or prior joint laxity is associated with dislocations, joint pain, impaired ability to move a joint Physical examination: gross deformity of a joint, impaired range of motion, pain with attempted range of motion, possible motor or sensory deficits with an associated nerve injury
Tests: x-ray or CT is useful to detect joint incongruency Treatment: Prompt closed reduction under sedation or nerve block Irreducible joints require a prompt open reduction Recurrent instability frequently requires surgical intervention to prevent future dislocations P.227 Outcomes: failure to reduce a joint in a timely fashion increases the risk of a permanent neurologic injury; the risk of future dislocations increases significantly following an initial traumatic dislocation Why eliminated from differential: the appearance of the ankle x-ray rules out this diagnosis Sprains Partial or complete tears of the ligaments and surrounding soft tissues in a joint History: traumatic injury to a joint, joint pain, feeling of joint instability Physical examination: increased joint laxity, pain with motion, joint swelling Tests: MRI is the best means of detecting ligament injury and associated inflammation and should be performed for joints in which the physical exam is equivocal; x-ray may show joint subluxation in cases of complete ligament rupture Treatment: activity modification, ice, compressive bandaging, and elevation are used to treat incomplete tears; surgical reconstruction may be required for completely torn ligaments and joint instability Outcomes: partial tears will heal with adequate rest; complete tears with associated joint instability may lead to early arthritis Why eliminated from differential: the finding of a boney injury on the x-ray makes this diagnosis less likely; MRI is rarely indicated to diagnose acute ankle sprains
Dashboard injuries are a frequent cause of hip dislocations in motor vehicle crashes.
Suspect an axillary nerve injury if a patient has difficulty abducting the arm (deltoid malfunction) following a shoulder dislocation.
CT is generally more useful in the diagnosis of bone pathology, while MRI is more useful for soft tissue injuries.
The initial treatment for sprains may be remembered by the mnemonic RICE: Rest, Ice, Compression, Elevation.
Case 9-3: “My knee has been bothering me for a long time” A 72-year-old man presents to his PCP with a complaint of chronic right knee pain that has gradually worsened over the past few years. The patient denies any significant trauma to his knee. He says that he feels minimal pain in the morning, but his knee pain worsens as the day passes. He particularly has pain with prolonged standing and walking. He says that his knee feels stiff, but he does not have any episodes of the joint locking or giving way. He says that the knee makes grinding sounds when he walks. He denies any fevers or erythema, but has swelling of the knee on days on which he does a considerable amount of walking. He has been taking ibuprofen for his pain, and he feels that this
helps reduce his symptoms to some extent. He denies similar symptoms in other joints or any neurologic symptoms. He has a past medical history significant for hypertension (HTN), hypercholesterolemia, and benign prostatic hypertrophy and takes the medications enalapril, hydrochlorothiazide (HCTZ), simvastatin, and dutasteride. He smokes an occasional cigar and drinks three alcohol beverages per week. On examination, he is a well-appearing man in no distress. His right knee does not appear swollen or erythematous, and there is no effusion. He has dull pain along the tibial-femoral joint line. He has mild deep pain with range of motion that worsens at the extremes of flexion and extension. There is palpable crepitus during movement. Neurologic examination finds normal motor and sensory function. The following vital signs are measured: T: 98.5°F, HR: 68 bpm, BP: 125/84 mm Hg, RR: 16 breaths/min
Differential Diagnosis Osteoarthritis, rheumatoid arthritis, septic arthritis, meniscal tear, Lyme disease, gout, pseudogout
Laboratory Data and Other Study Results Knee x-ray: considerable loss of the tibial-femoral joint space with subchondral cyst and osteophyte formation
Diagnosis Osteoarthritis P.228
Treatment Administered The patient was given an intra-articular injection of methylprednisolone and started on regular oral celecoxib
Follow-up The patient experienced an improvement in his symptoms for approximately 2 months; after that time he noted that his knee pain began to worsen The patient was referred to an orthopedic surgeon, who recommended total joint replacement The patient underwent a total knee replacement and had eradication of his knee pain following his recovery from surgery
Steps to the Diagnosis Osteoarthritis (OA) Chronic, minimally inflammatory degeneration of joints through gradual deterioration of the articular cartilage Can occur in any joint but most commonly occurs in the hips, knees, spine, hands, wrists, and shoulders Degeneration of the spinal facet joints can lead to spinal stenosis and nerve compression symptoms Risk factors: advanced age, family history, obesity, previous joint trauma, repetitive joint stress (e.g., heavy labor occupations) History: insidious onset of joint stiffness and pain that worsens with activity and weight-bearing and improves with rest, pain tends to be worse at the end of the day, pain worsens following mild trauma to the joint Physical examination: decreased and painful range of motion at the affected joint, mild joint swelling, joint
crepitus, bony protuberances in the distal interphalangeal (DIP) (i.e., Heberden nodes) and proximal interphalangeal (PIP) (i.e., Bouchard nodes) joints with hand involvement Tests: X-rays will show joint space narrowing, subchondral cyst formation, sclerotic subchondral bone, and osteophyte formation (i.e., small periarticular outgrowths of bone in response to joint degeneration) (Figure 9-4) Aspiration of the joint detects <2,000 white blood cells (WBC)/mm3 Erythrocyte sedimentation rate (ESR), C-reactive protein (CRP), and serum WBCs are normal Treatment: Activity modification, physical therapy, and NSAIDs are used as the initial treatments Patients with persistent symptoms or acute exacerbations of joint pain will have an improvement in their symptoms with an intra-articular corticosteroid injection Some patients may have an improvement in their general symptoms with intra-articular injections of a viscosupplement Total joint replacement is the definitive means of treating significant joint degeneration and has a high success rate of relieving joint pain Outcomes: Most patients will have worsening of their joint pain with time as joint degeneration progresses Intra-articular injections provide temporary relief, but repeat injections are frequently due to the recurrence of pain Joint replacement is associated with approximately a 95% satisfaction rate regarding pain relief and restoration of function Clues to the diagnosis: History: chronic, worsening knee pain that increases over the course of the day, joint stiffness, improvement of pain with NSAID use P.229
Figure 9-4 Osteoarthritis in a right hip joint; synovial cysts (black arrows) and osteophytes (white arrow) are evident. (See color image.)
Physical: joint crepitus, joint pain with range of motion Tests: appearance of x-rays
Causes of joint pain may be remembered by the mnemonic HOT INFARCTS: Hemarthrosis, Osteoarthritis, Trauma (fracture), Infection (septic joint, osteomyelitis), Neuropathy, Fibromyalgia, Avascular necrosis, Rheumatic disease (RA, systemic lupus erythematosus [SLE], ankylosing spondylitis), Crystal disease (gout, calcium pyrophosphate dehydrate deposition [CPPD] disease), Tumor, Soft tissue (tendon, ligament, meniscus).
A diagnosis of osteoarthritis is typically based on the history, physical examination, and x-rays, and joint aspiration and laboratory testing is rarely indicated in the diagnosis.
Intra-articular corticosteroid injections should not be performed regularly at <3-month intervals because of the increased risk for infection and soft tissue rupture. Rheumatoid arthritis A chronic inflammatory arthropathy with infiltration of the synovial joint tissues by inflammatory cells and progressive erosion of both cartilage and bone Synovial hypertrophy with the formation of granulation tissue on the articular cartilage (i.e., pannus
formation) occurs due to chronic joint inflammation Most commonly seen in middle-aged women and those with a human leukocyte antigen (HLA) DR4 serotype PIP and metacarpophalangeal (MCP) joints in the hands are frequently the first affected A symmetric polyarthropathy involving the other major joints occurs after the initial joint development History: malaise, weight loss, insidious onset of morning joint pain and stiffness that improves slightly over the day Physical examination: decreased range of motion, joint warmth, joint swelling and hypertrophy, fever, ulnar deviation of the fingers, swan neck deformities (i.e., DIP held in a flexed position with a hyperextended PIP), boutonniere deformities (i.e., flexed PIP with a hyperextended DIP), subcutaneous nodules (Color Figure 9-1) Tests: Rheumatoid factor (RF) is positive in 75% of patients but is not specific for the disease Antinuclear antibody (ANA) is positive in 40% of patients; ESR and anti–citrulline-containing protein immunoglobulin IgM antibodies are increased Joint aspiration typically shows 5,000 to 50,000 WBC/mm3 X-rays demonstrate soft tissue swelling, joint space narrowing, marginal bony erosions, and possible joint subluxation; MRI may be useful for detecting similar findings (Figure 9-5) Treatment: NSAIDs and physical therapy are the initial treatments to reduce pain and optimize function Patients still mildly symptomatic following NSAID use may be started on sulfasalazine or hydroxychloroquine and analgesics as needed P.230
Figure 9-5 Hip x-ray in a patient with rheumatoid arthritis. Marginal erosions in the bone (arrow) are due to pannus formation (arrowheads). (See color image.)
Moderate disease may be treated with methotrexate; anti-TNF (tumor necrosis factor) drugs (e.g., infliximab) or corticosteroids may also be utilized Severe disease is treated with anti-TNF drugs and corticosteroids Joint replacement is frequently required in patients with severe joint degeneration; soft tissue repairs are frequently performed for joint subluxations (particularly the hand) Outcomes: in the absence of antirheumatic medications the disease is progressive and occasionally debilitating Why eliminated from differential: the appearance of the x-rays is more consistent with OA, and the absence of any other joint involvement makes this diagnosis less likely Meniscal tear Injury to the cartilaginous meniscus of the knee, causing altered knee mechanics Results from repetitive meniscal microtrauma and degeneration or an acute forceful twisting of a planted knee Frequently associated with tears of the anterior cruciate ligament (ACL) of the knee History: deep knee pain, locking of the knee Physical examination: joint line tenderness, clicking of the knee during range of motion
Tests: MRI is the modality of choice for detecting tears Treatment: arthroscopic debridement or repair is performed to eliminate impingement of the torn meniscus; NSAIDs and physical therapy may be utilized in patients refusing surgery to help ease their symptoms Outcomes: symptoms are typically eliminated following surgery; patients who have meniscal tears are at a higher risk for developing OA regardless of the treatment Why eliminated from differential meniscal tears are fairly common in this age group, but the appearance of the x-ray suggests that joint degeneration is a much better explanation for the patient's symptoms Septic arthritis More thorough discussion in later case Why eliminated from differential: the lack of erythema, effusion, or micromotion pain makes this diagnosis unlikely P.231 Lyme disease More thorough discussion in later case Why eliminated from differential: the x-ray is highly suggestive of OA, and the absence of any dermatologic, neurologic, or cardiac symptoms make this diagnosis unlikely Gout/pseudogout More thorough discussion in later case Why eliminated from differential: the lack of erythema, an effusion, or signs of chondrocalcinosis on the x-ray makes this diagnosis unlikely
OA: typically a symmetrical and may only affect one joint; the DIP joints are frequently involved in the hands. RA: affects joints on both sides of the body in a symmetrical distribution; the DIP joints are spared in the hands.
Meniscal tears are common in patients after the age of 60 years and are frequently asymptomatic in these patients.
A medially directed blow to the lateral side of the knee (i.e., a valgus stress) may cause the unhappy triad: medial meniscus tear, medial collateral ligament (MCL) tear, and ACL tear.
Case 9-4: “This knee is killing me!” A 44-year-old man presents to an emergency department with the complaint of severe left knee pain for the past day. He says that he has felt sick and fatigued for the past week, but that his knee only began to hurt last night. He denies any trauma to his knee over this time. He says that he woke up in the middle of the night with moderate knee pain and swelling that has steadily increased in severity. He says that he is unable to bear weight on his left leg because of the pain and that any movement causes severe pain. He denies that this has ever occurred previously. He also describes having chills at home prior to coming to the hospital. He denies any locking of the knee, lesions on the knee, or drainage from the knee. He has no symptoms in any other joint. He notes a history of multiple chlamydia infections
that have been treated with antibiotics. He has had multiple human immunodeficiency virus (HIV) screenings (most recently 1 month ago) that have all been negative. He admits to drinking alcohol and smoking marijuana on a daily basis and started to inject cocaine intravenously about 3 months ago. On examination, he is an uncomfortableappearing man in no respiratory distress. He is mildly diaphoretic. He has palpable inguinal lymph nodes on the left side. Auscultation of his lungs and heart detects clear breathing sounds and a regular heart beat without any abnormal sounds. His abdomen is nontender without any masses. His left knee is deeply erythematous and swollen. There is a palpable effusion in the knee, and the patella is easily ballottable. There do not appear to be any skin lesions over the knee, and there are no sites of drainage. Any range of motion causes significant discomfort in the patient. Palpation of his calf detects mild swelling, but the calf is generally soft. Neurovascular examination of the left leg detects normal motor and sensory function and a bounding dorsalis pedis pulse. The following vital signs are measured: T: 102.1°F, HR: 110 bpm, BP: 140/95 mm Hg, RR: 14 breaths/min
Differential Diagnosis Septic arthritis, Lyme disease, osteomyelitis, fracture, gout, pseudogout, osteoarthritis, rheumatoid arthritis, compartment syndrome
Laboratory Data and Other Study Results Complete blood cell count (CBC): WBC: 21.2, hemoglobin (Hgb): 12.2, platelets (Plt): 312 Chem7: Na: 146 mEq/L, K: 3.4 mEq/L, Cl: 104 mEq/L, CO2: 32 mEq/L, BUN: 23 mg/dL, Cr: 1.2 mg/dL, Glu: 71 mg/dL ESR: 73 mm/hr CRP: 9.3 mg/dL Joint aspiration: 87,259 WBC/mm3 (95% polymorphonuclear cells [PMNs]), 265 red blood cells [RBC]/mm3; no crystals Knee x-ray: significant effusion; no fracture or dislocation; no signs of joint degeneration Blood cultures: Gram stain showing Gram-positive cocci in clusters P.232
Diagnosis Septic arthritis
Treatment Administered The patient was taken urgently to the operating room by orthopedic surgery for an incision and drainage of the left knee The patient was placed on vancomycin awaiting the culture results for the aspiration fluid and the intraoperative cultures
Follow-up Following the initial surgical procedure, the patient noted improvement in his pain The patient was taken back to the operating room 2 days after his initial surgery for a repeat washout of the joint and examination of the tissues Both blood cultures and synovial fluid cultures from the knee grew methicillin-resistant Staphylococcus aureus
Vancomycin was continued, and an infectious disease consultant recommended a 6-week course of treatment Following two consecutive negative blood cultures, the patient was transferred to a skilled nursing facility to continue the remainder of his antibiotic treatment
Steps to the Diagnosis Septic arthritis Infection of a joint by bacteria or fungi, resulting in a significant host immune reaction Damage to the joint occurs as a result of both bacterial toxins and the host inflammatory response Occurs through the hematogenous spread of infection (e.g., bacteremia), extension of a local infection (e.g., cellulitis), or direct inoculation (e.g., open fracture) S. aureus is the most common bacteria involved, and Neisseria gonorrhoeae is also common in sexually active patients Patients with DM, vascular disease, and cancer may also be susceptible to Gram-negative rod infections History: sudden onset of joint pain, refusal to bear weight or move the joint (particularly children) Physical examination: significant joint swelling and erythema, warm skin overlying the joint, micromotion pain (i.e., significant pain with only a few degrees of movement), possible overlying skin lesions or draining sinus tracts Tests: Increased WBC, ESR, and CRP Aspiration of synovial fluid demonstrates >50,000 WBC/mm3 with a high percentage of PMNs (greater than 75%) (Table 9-4) Treatment: Incision and drainage (I + D) is required for any infection other than N. gonorrhoeae Repeat washouts may be required to ensure removal of any diseased tissue Long-term IV antibiotics (approximately 6 weeks) are prescribed once the offending organism has been identified Outcomes: one third of cases will be complicated by chronic infection, early joint degeneration, or the need for surgical reconstruction Clues to the diagnosis: History: sudden and severe knee pain, history of IV drug use, and recurrent sexually transmitted diseases, inability to bear weight Physical: swollen, erythematous knee, significant effusion, micromotion pain Tests: increased ESR and CRP, increased WBCs, synovial fluid analysis results
N. gonorrhoeae is the most common cause of septic arthritis in young adults.
Culture results are frequently falsely negative in N. gonorrhoeae infections.
Because the inflammatory response to N. gonorrhoeae is not as severe as that for other bacteria, I + D is not required for treatment; treat these cases with intravenous (IV) ceftriaxone and doxycycline. P.233
Table 9-4 Analysis of Synovial Fluid Aspiration
Noninflammatory A rthropathy (e.g., OA, Trauma)
Inflammatory Arthropathy (e.g., RA, Gout)
Septic Arthritis
Appearance
Straw colored, viscous
Yellow, thin
Purulent
WBCs
<2,000
2,000–50,000
>50,000
<25%
>50%
>75%
Fluid glucose
Similar to blood
Decreased
Decreased
Histology
Increased RBCs with trauma
Needle-shaped, negatively birefringent crystals in gout;
Positive Gram stain and culture
rhomboid, positively birefringent crystals in pseudogout
for the infectious pathogen
(WBC/mm3)
PMNs (% of WBCs)
OA, osteoarthritis; PMNs, polymorphonuclear cells; RA, rheumatoid arthritis; RBCs, red blood cells; WBCs, white blood cells.
Osteomyelitis Bone infection due to pathogen exposure (e.g., hematogenous spread, local extension, direct inoculation) S. aureus and Pseudomonas are the most common causes, but Salmonella is also common in sickle cell and asplenic patients History: deep bone pain, difficulty bearing weight due to pain, chills Physical examination: bone tenderness, fever, possible overlying cellulitis or a draining sinus tract Tests: Increased WBCs, ESR, and CRP Bone culture provides a definitive identification of the infecting agents MRI is the most sensitive means of detecting infection (Figure 9-6) Bone scan will frequently show changes after the initial 72 hours of established infection; sensitivity is increased when performed as a labeled-WBC scan (using gallium or indium isotopes) X-rays are usually not helpful for identifying infection in the initial 2 weeks of the disease Treatment: Long-term antibiotics are the primary form of treatment (initially empirically, then pathogen-specific after
culture) I + D is required for any abscess within the bone (i.e., sequestrum) or in the surrounding tissue Debridement of affected bone in extensive cases may cause significant bone loss and disability and is avoided if possible Outcomes: chronic infection is a problem in these patients, and the best chance of eradication correlates with a quick diagnosis and institution of treatment Why eliminated from differential: the synovial aspirate confirms septic arthritis in this patient, but the long-term follow-up should consist of imaging (x-ray or MRI) to look for any development of infection beyond the joint itself Lyme disease Systemic infection caused by Borrelia burgdorferi, a spirochete transmitted by the bite of the Ixodes tick Progression of the disease is divided into early localized, early disseminated (i.e., spread of disease to the cardiac and neurologic systems), and late disseminated (i.e., establishment of chronic cardiac, neurologic, and joint infection) stages P.234
Figure 9-6 X-ray and magnetic resonance imaging (MRI) of the proximal tibia in a patient with osteomyelitis. (A) X-ray of the proximal tibia demonstrating a poorly defined lesion (arrows) involving much of the proximal tibia. (B) A T1-weighted MRI image of the same
region demonstrates extensive involvement of the marrow from the epiphysis to the proximal diaphysis.
History: Early localized: chills, fatigue, arthralgias, headache Early disseminated: chronic fatigue, headache, development of paresthesias Late disseminated: chronic arthralgias Physical examination: Early localized: erythema chronicum migrans (i.e., bull's eye rash), fever (Color Figure 9-2) Early disseminated: arrhythmias from myocarditis, possible heart block, Bell's palsy Late disseminated: signs of subacute encephalopathy (e.g., insomnia, mood disturbances, memory loss, ataxia), polyneuropathy Tests: enzyme-linked immunosorbent assay (ELISA) and Western blot tests for antibodies; joint aspiration is typically not helpful Treatment: doxycycline, amoxicillin, or cefuroxime (oral form is appropriate for early disease, but IV forms are needed for disseminated disease) Outcomes: The long-term prognosis is variable, but Bell's palsy and carditis gradually resolve Arthralgias and cognitive deficits appear to remain a chronic problem in disseminated cases Recognizing the disease and treating it before it becomes disseminated appears to be the key to preventing long-term sequelae P.235 Why eliminated from differential: the sudden onset of severe symptoms in this patient and the synovial fluid results are more consistent with acute joint sepsis Gout A peripheral monoarthritis due to the deposition of sodium urate crystals in joints The first metatarsophalangeal joint is the most commonly affected (i.e., podagra), but other joints may be involved Risk factors: renal disease, male gender, urate underexcretion, diuretic use, cyclosporine use, cancer, hemoglobinopathies, excessive alcohol consumption History: sudden severe pain in one joint, chills, malaise Physical examination: fever, warm and red joint, joint swelling Tests: Joint aspiration shows needle-shaped, negatively birefringent crystals and several WBCs (Color Figure 9-3) Serum uric acid may be normal or increased and is not a reliable test for diagnosis X-ray may show punched-out bone lesions in chronic cases
Treatment: NSAIDs (especially indomethacin), colchicine, or corticosteroids may be used to treat acute flares Decreasing alcohol and diuretic use and avoiding foods high in purines (e.g., red meats, fish) help to prevent exacerbations Probenecid (inhibits renal uric acid resorption) and allopurinol (inhibits uric acid formation) are used in chronic cases to prevent flare-ups Outcomes: long-standing, poorly controlled disease leads to chronic tophaceous gout with the formation of nodular tophi (i.e., large deposits of crystals in soft tissues) that cause permanent deformity Why eliminated from differential: although the symptoms leading to presentation are consistent with this diagnosis, the results of the synovial fluid analysis rule it out Pseudogout (a.k.a. calcium pyrophosphate dehydrate deposition disease, CPPD) The deposition of calcium pyrophosphate dehydrate crystals in joints leading to an inflammatory reaction Frequently associated with the presence of other endocrine conditions (e.g., DM, hyperparathyroidism) The knee and wrist are most commonly affected History: similar to gout but with less severe pain Physical examination: joint swelling, erythema, and warmth Tests: synovial fluid analysis detects rhomboid, positively birefringent crystals (Color Figure 9-4); x-ray may show chondrocalcinosis (i.e., calcification of articular cartilage) Treatment: NSAIDs or colchicine are used to treat attacks Outcomes: prognosis is fair with few extra-articular effects of the disease, but recurrent flares are possible Why eliminated from differential: the results of the synovial fluid analysis rule out this diagnosis Fracture More thorough discussion in prior case Why eliminated from differential: the appearance of the x-ray rules out this diagnosis Osteoarthritis More thorough discussion in prior case Why eliminated from differential: the appearance of the x-ray and the synovial fluid analysis rule out this diagnosis Rheumatoid arthritis More thorough discussion in prior case Why eliminated from differential: the appearance of the x-ray and the synovial fluid analysis rule out this diagnosis P.236 Compartment syndrome Swelling and an increase of pressure within a fascial compartment, leading to eventual compression of neurovascular structures and ischemia of tissues within the compartment and distal to the involved region;
delayed reperfusion may worsen tissue ischemia Most frequently occurs as a sequela of extremity trauma (especially crush injuries) Occurs most commonly in the lower leg and forearm, but the hand, foot, and thigh are also potential sites of involvement History: pain that does not respond to increasing doses of analgesia, numbness, weakness Physical examination: pallor of the affected extremity, cool-feeling extremity (i.e., poikilothermia), paresthesia, motor weakness, eventual pulselessness, increased pain with passive stretching of the compartment Tests: intracompartmental pressures (measured by inserting a needle attached to a manometer into the compartment of interest) will be elevated Treatment: emergent fasciotomies are indicated for compartments with pressures >30 mm Hg or within 30 mm Hg of the diastolic blood pressure; delayed closure of the wounds is performed at a later date Outcomes: Tissue perfusion and neurologic function may be preserved with emergent fasciotomies Patients undergoing fasciotomies are at an increased risk of infection due to the open wounds Delays in treatment or missed compartment syndromes carry a devastating sequela of significant tissue necrosis and loss of function Why eliminated from differential: the symptoms in this patient are more attributable to the knee than the calf, and examination of the calf carries a low clinical suspicion for this diagnosis
Although Salmonella should be considered in patients with sickle cell disease and Pseudomonas should be considered in intravenous drug abusers, S. aureus is still the most common cause of osteomyelitis in these patients.
Allopurinol should not be administered in acute attacks of gout because it is only useful as prophylaxis in chronic cases.
Young athletes may get an exertional compartment syndrome during athletic activity with a mild elevation of compartment pressures that resolves following activity cessation and that carries a minimal risk of significant tissue ischemia.
A painful leg that has a pulse never rules out compartment syndrome.
The signs of compartment syndrome may be remembered by the six Ps: Pain, Paresthesias, Paralysis, Pallor, Poikilothermia, and Pulselessness.
Case 9-5: “I can't control my bowels” A 76-year-old man is brought to an emergency department by his wife after the acute onset of multiple neurologic symptoms. The patient said that he woke up in the middle of the night with low back pain. After he awoke he lost control of his bowels while in bed. He says that he now also feels numb around his groin. He denies ever having had these symptoms previously. He says that his legs feel weak and that he is having difficulty ambulating. He also has pain radiating down both legs. He denies numbness or weakness in his upper extremities. He also denies headaches or vision
abnormalities. He states that a pulmonary nodule was detected 1 week ago on a chest x-ray being performed as part of a preoperative evaluation for a knee arthroscopy that he had scheduled to treat a torn meniscus. He was scheduled to have this lesion evaluated in the near future. He denies any shortness of breath or chest pain. He reports a past medical history of HTN, hypercholesterolemia, benign prostatic hypertrophy (BPH), and mild hyperglycemia. He takes the medications lisinopril, HCTZ, atorvastatin, aspirin, and terazosin. His hyperglycemia has been controlled with dietary modification. He reports smoking a pack of cigarettes per day for the past 60 years and drinks about six beers per week. On examination, he appears to be anxious but in no acute distress. Fundoscopic examination appears to be normal. He has no lymphadenopathy. Auscultation of his lungs detects clear breath sounds. Auscultation of his heart detects a regular rhythm with no extra heart sounds. His abdomen is nontender, but he has an easily palpable bladder. A rectal examination demonstrates poor rectal tone and incontinence of stool. He has decreased sensation in the perineal region. On neurologic examination he has normal function in his cranial nerves and upper extremities. He has generalized 3/5 strength in his lower extremities. On filament testing his sensory discrimination is worse in the legs than in his arms. He has negative Babinski signs bilaterally. Examination of his back detects no obvious step-offs or focal regions of pain. The following vital signs are measured: T: 98.4°F, HR: 90 bpm, BP: 127/85 mm Hg, RR: 16 breaths/min P.237
Differential Diagnosis Disc herniation, spinal stenosis, cauda equina syndrome, cerebral vascular accident, DM neuropathy, Guillain-Barre syndrome, multiple sclerosis
Laboratory Data and Other Study Results CBC: WBC: 11.3, Hgb: 12.4, Plt: 173 10-electrolyte chemistry panel (Chem10): Na: 141 mEq/L, K: 4.1 mEq/L, Cl: 105 mEq/L, CO2: 30mEq/L, BUN: 13 mg/dL, Cr: 0.6 mg/dL, Glu: 98 mg/dL, magnesium (Mg): 2.0 mg/dL, calcium (Ca): 10.9 mg/dL, phosphorus (Phos): 3.3 Head CT: no sign of intracranial lesion; no fractures; normal ventricles and cerebral volume Chest CT: 3 × 4 cm lesion in the right lung adjacent to a segmental bronchi Spine CT: no fractures; indiscrete mass involving the L5 vertebral body Spine MRI: 3 × 3 cm mass in the L5 vertebra, with expansion into the spinal canal
Diagnosis Cauda equina syndrome
Treatment Administered A urethral catheter was placed into the patient's bladder to drain the volume of contained urine The patient was taken emergently to the operating room for decompression of his lumbar spine IV corticosteroids were started and administered for 48 hours to decrease spinal inflammation
Follow-up The patient recovered control of his bowels and sensation in his perineum A biopsy of the lung and spinal lesions confirmed the diagnosis of a pulmonary malignancy Radiation therapy was started to decrease the size of the vertebral lesion
The patient's neurologic examination improved, and he was discharged to continue his tumor work-up with oncology as an outpatient
Steps to the Diagnosis Cauda equina syndrome Compression of the cauda equina (i.e., an extension of the dural-arachnoid sac beyond the end of the spinal cord and the terminal nerve roots contained within it) following trauma or growth of a neoplasm History: urinary retention, incontinence of stool, possible lower extremity weakness, low back pain Physical examination: decreased rectal tone, perineal anesthesia (i.e., saddle anesthesia), possible decreased lower extremity sensation and strength Tests: MRI or CT myelogram is used to locate the site of compression Treatment: Emergency surgical decompression is performed to relieve pressure on the terminal nerve roots IV corticosteroids are frequently administered to decrease inflammation around the spinal cord Radiation is used in cases of spinal neoplasm to decrease the size of the tumor Outcomes: recovery of function is most likely with prompt treatment; delays in surgical decompression result in permanent neurologic deficits P.238 Clues to the diagnosis: History: low back pain, bowel incontinence, radicular pain, recently detected pulmonary nodule Physical: decreased rectal tone, palpable bladder, sensory and motor deficits in the legs Tests: appearance of the MRI Disc herniation More thorough discussion in prior case Why eliminated from differential: the absence of a herniation on the MRI rules out this diagnosis Spinal stenosis More thorough discussion in prior case Why eliminated from differential: the sudden onset of symptoms makes this diagnosis unlikely, and the MRI rules it out Cerebral vascular accident More thorough discussion in Chapter 7 Why eliminated from differential: the normal head CT rules out this diagnosis Guillain-Barre syndrome More thorough discussion in Chapter 7 Why eliminated from differential: the pattern of neurologic findings makes this diagnosis unlikely
DM neuropathy More thorough discussion in Chapter 5 Why eliminated from differential: the normal serum glucose and sudden onset of symptoms rule out this diagnosis Multiple sclerosis More thorough discussion in Chapter 7 Why eliminated from differential: the sudden onset of symptoms and lack of white matter lesions on the MRI make this diagnosis unlikely
Treat cauda equina syndrome with immediate surgical decompression because it can quickly result in permanent neurologic injury.
Case 9-6: “There's a hole in my bone” A 48-year-old woman is referred to an orthopedic surgeon for evaluation of an abnormal x-ray finding in her left femur. The patient reports that she has been having an aching pain in her left groin for the past 3 weeks and went to her PCP to be evaluated. She was sent for x-rays of the left hip and pelvis, which detected an area of lucency in the proximal left femur. She denies any history of trauma and did not have groin pain prior to three weeks ago. She says that she gets the pain primarily during ambulation, but notices some minimal soreness when crossing her legs. She says that the pain was very mild initially but has increased in the past week. She denies any numbness, weakness, or other regions of pain. She says that she has never had an x-ray of her left hip previously. She says that she has lost 30 pounds in the past year but has been dieting significantly during this time. She says that she has had yearly mammograms performed and has never had an abnormal lesion detected. She denies any past medical history except for being overweight. She does not take any medications. She smokes two cigarettes every day and has done so for the past 20 years. She drinks alcohol socially. On examination, she is a well-appearing woman in no acute distress. Range of motion of her left hip is minimally painful, and there are no limits to her motion. No masses around the hip are palpable. She has a mildly antalgic gait because of her left groin pain. Neurologic examination detects normal strength and sensation in both lower extremities. The following vital signs are measured: T: 98.7°F, HR: 82 bpm, BP: 122/81 mm Hg, RR: 18 breaths/min
Differential Diagnosis Benign musculoskeletal lesion, osteosarcoma, bone metastasis, osteoporosis, Paget disease of bone, multiple myeloma, Ewing sarcoma, osteomyelitis
Laboratory Data and Other Study Results CBC: WBC: 9.9, Hgb: 16.5, Plt: 199 Chem7: Na: 142 mEq/L, K: 3.9 mEq/L, Cl: 101 mEq/L, CO2: 27 mEq/L, BUN: 25 mg/dL, Cr: 1.1 mg/dL, Glu: 89 mg/dL ESR: 26 mm/hr CRP: 0.7 mg/dL Left hip x-ray: presence of an area of radiolucency in the proximal femur at the base of the femoral neck; no periosteal reaction Left hip MRI: presence of a 2 × 3 × 2 cm solid mass in the proximal femur between the trochanters; the mass is relatively dark on the T1 images and enhances on the T2 images
After these tests have been completed, the additional following studies are performed: Chest CT: normal lung appearance; no masses; no effusions; normal pericardial and cardiac appearance; normal thoracic spine appearance Abdominal/pelvic CT: 5 × 6 cm cystic mass on the left kidney with an associated 2 × 2 cm solid mass; normal right kidney appearance; normal bilateral adrenal glands; no hepatic or gastrointestinal (GI) abnormalities noted Bone scan: significantly increased signal at the left kidney and left proximal femur Femoral lesion biopsy: consistent with poorly differentiated clear cell renal carcinoma
Diagnosis Bone metastasis from renal cell carcinoma
Treatment Administered Prophylactic intramedullary nailing was performed on the left femur The patient was referred to an oncologist to discuss chemotherapy and local radiation of the left femur and to a urologist to discuss excision of the left kidney
Follow-up After stabilization of the patient's femur and her postoperative recovery, the patient noted an improvement in her groin pain Nephrectomy of the left kidney was performed by a urologist Radiation of the left proximal femur and systemic chemotherapy were performed At 6 months following her diagnosis, the patient was being cautiously observed, and treatment was continuing
Steps to the Diagnosis Bone metastases Metastasis of a nonskeletal primary tumor to bone, creating a secondary lesion Most common malignancy found in bone Breast, lung, prostate, renal cell, thyroid, and lymphoma cancers are the most common sources History: presence of a primary form of cancer, deep bone pain, fracture following minor trauma Physical examination: possible palpable mass or pain with stress placed on the bone Tests: Biopsy is important to identifying the source of the secondary lesion X-rays can locate the lesion in the bone Bone scan is useful for determining the extent of the disease in the body MRI is useful to define the local extent of a given lesion Treatment: Prophylactic fixation is recommended for tumors in areas of high stress that are at risk for fracturing;
pathologic fractures are treated with adequate stabilization and fixation Radiation therapy decreases the size of metastases Systemic chemotherapy that is appropriate for the primary tumor is frequently used to prevent additional metastases Bisphosphonates are useful as a means of slowing bone loss Outcomes: prognosis is dependent on the primary tumor; in general metastatic disease carries a poor prognosis with dismal survivorship Clues to the diagnosis: History: bone pain Physical: pain with manipulation of the affected bone Tests: results of the x-rays, MRI, CT, bone scan, and biopsy
Because most bone tumors are metastases and not primary tumors, any patient with a new bone tumor should undergo a full work-up to look for a tumor source.
Tumors that metastasize to bone may be remembered by the mnemonic “Really Large Lesions Pulverize Troubled Bones”: Renal cell, Lung, Lymphoma, Prostate, Thyroid, Breast. Osteosarcoma The most common primary malignant bone tumor Occurs most often in the second and third decades of life and in males Most commonly occurs in the distal femur, proximal tibia, or proximal humerus Risk factors: Paget disease of bone, p53 genetic proto-oncogene, familial retinoblastoma, radiation exposure, bone infarcts History: deep bone pain Physical examination: possible palpable bony mass Tests: Increased alkaline phosphatase, ESR, and lactate dehydrogenase (LDH) Biopsy is required for a definitive diagnosis X-ray shows a bony lesion with a sunburst pattern and Codman triangle (i.e., periosteal new bone formation at the diaphyseal end of the lesion) (Figure 9-7) MRI or positron emission tomography (PET) scan is useful for determining the extent of a lesion Chest CT is routinely performed to look for lung metastases Treatment: radical surgical excision is required; adjuvant chemotherapy is administered in most cases Outcomes: the 5-year survival is 90% in low-grade cases and 50% for higher grade lesions
Figure 9-7 Osteosarcoma in left proximal femur. Note the dense sunburst pattern of the lesion (solid black arrows) and presence of Codman's triangle (open arrow).
Why eliminated from differential the presence of a renal cell tumor in the patient makes the chance of a coincident osteosarcoma low; biopsy formally rules out this diagnosis Ewing sarcoma A highly malignant tumor of neural crest cells occurring in the diaphysis of long bones Most common in children between 5 and 15 years of age History: bone pain, fatigue, weight loss, fractures with minor trauma Physical examination: swelling, fever, palpable bone mass Tests: Increased WBCs, decreased Hgb, and increased ESR Bone biopsy is needed to make a diagnosis X-ray may detect a large destructive lesion with significant periosteal reaction; MRI is useful to determine
the extent of a lesion Treatment: radical resection, radiation, adjuvant chemotherapy Outcomes: a 60% 5-year survival when both radiation and chemotherapy are used in nonmetastatic disease; 20% 5-year survival in the presence of metastases Why eliminated from differential: the patient's age and the location of the bone lesion make this diagnosis unlikely Benign musculoskeletal tumor Nonmalignant tumor of bone or cartilage (e.g., osteochondroma, osteoblastoma, enchondroma, fibrous dysplasia) that may cause symptoms through mechanical effects History: bone pain, surrounding soft tissue pain Physical examination: palpable mass, pain with range of motion Tests: x-ray or MRI is frequently useful for detecting lesions Treatment: generally speaking, masses are only removed if causing irritation or impaired function Outcomes: most lesions do not cause any problems besides their mechanical effects Why eliminated from differential: the biopsy is able to confirm the true identity of the lesion Paget disease of bone A disorder of overactive osteoblasts and osteoclasts, leading to excessive bone turnover and a disorganized bony structure History: may be asymptomatic or have deep bone pain and fractures following minor trauma Physical examination: tibial bowing, kyphosis, increases in cranial diameter, deafness (due to changes in auditory ossicles) Tests: Increased alkaline phosphatase and urine hydroxyproline Normal calcium and phosphorus X-rays may demonstrate osteolytic lesions and expanded hyperdense bone Bone scan will detect “hot spots” in the areas of acute disease Treatment: bisphosphonates and calcitonin Outcomes: adverse outcomes are related to impairments of function (e.g., deafness, kyphosis); there is an increased risk of developing osteosarcoma with cases tending to have a poor prognosis Why eliminated from differential: the patient lacks several of the key clinical findings of this condition, so it is unlikely the diagnosis in this case
If a patient complains that “my hats no longer fit,” consider a work-up for Paget disease or osteopetrosis. Multiple myeloma A malignant proliferation of plasma cells, resulting in multiple osteolytic defects in bone
Abnormal monoclonal protein (i.e., M protein) is produced from IgG and IgA heavy chains and κ and λ light chains (i.e., Bence Jones proteins) Patients may be notable for a prior finding of monoclonal gammopathy of undetermined significance (MGUS) History: back pain, radicular pain, weakness, fatigue, weight loss, constipation, pathologic fractures, frequent infections Physical examination: pallor, bone tenderness Tests: Decreased hemoglobin, decreased hematocrit, decreased WBCs, increased BUN and creatinine (secondary to renal insufficiency) Serum protein electrophoresis (SPEP) and urine protein electrophoresis (UPEP) detect high M protein and Bence Jones proteins Bone marrow biopsy will show an increased number of plasma cells X-ray will show multiple punched-out lesions in bone Treatment: Radiation therapy to sites of extensive involvement and chemotherapy Bone marrow transplantation may be needed following eradication of malignant marrow Pathologic fractures require surgical reduction and fixation Infections are treated aggressively Outcomes: complications include renal failure, recurrent infections, hypercalcemia, and spinal cord compression; the median survival following diagnosis is 3 years Why eliminated from differential: the presence of only a single osteolytic lesion makes this diagnosis less likely Osteomyelitis More thorough discussion in prior case Why eliminated from differential: the normal ESR, CRP, and WBCs rule out this diagnosis Osteoporosis Substantial osteopenia (i.e., decreased bone density) with normal mineralization in the general bone stock Can result from decreased bone formation or increased bone resorption Risk factors: inadequate dietary calcium during young adulthood, smoking, excessive alcohol consumption, sedentary lifestyle, decreased estrogen (e.g., postmenopausal), long-term steroid use, hyperparathyroidism, hyperthyroidism History: usually asymptomatic until fractures (e.g., Colles, femoral neck, and vertebral) and neurovascular impingement occur Physical examination: prior to the development of fractures there are few notable findings on examination Tests: x-ray will demonstrate visibly decreased bone stock (e.g., thin cortices, decreased contrast); dual energy x-ray absorptiometry (DEXA) will demonstrate a bone marrow density >2.5 standard deviations less
than a healthy young adult Treatment: Exercise, calcium, and vitamin D are important for maintaining bone stock Bisphosphonates decrease osteoclast activity (less bone resorption), increase bone density, and decrease fracture risk Selective estrogen receptor modulators (e.g., raloxifene) help increase bone density with less adverse effects than classic hormone replacement therapy Outcomes: the main complications are related to fractures and the related impairments in mobility Why eliminated from differential: the normal-appearing bone surrounding the osteolytic lesion makes this diagnosis unlikely
The peak bone mass in the body occurs at 20 to 25 years of age.
Osteoporosis is less likely to occur in obese people because the increased load placed on bones helps to prevent osteopenia.
Case 9-7: “I ache and I feel lousy” A 36-year-old woman presents to an internist as a new patient with pain in multiple joints. She says that she has multiple things that bother her, but chronic pain in her knees, ankles, and hands is what finally led her to see a physician. She says that these problems have slowly developed over the past 10 years. She says that the pain is inconsistent and that there are some days when she feels fine and other days when she can barely stand. She denies any trauma. She says that she has minimal swelling and no erythema at her joints and has not noticed any significant deformities. The pain seems to respond to ibuprofen use. She also notes that she cannot go out in the sun because she breaks out in a blotchy red rash. This rash resolves over a day or so of onset. She feels generally worn down and has days where it is difficult to go to work. She feels that she gets sick often and has frequent “chest colds” and mouth sores. She denies any chills or fevers, pain in her back, dry skin, chest pain, shortness of breath, vomiting or diarrhea, syncope, or seizures. She denies any weakness or tingling in her extremities. She says that she has not experienced any significant weight loss in recent history. She works as an accountant and denies any exposure to toxic materials. She denies any past medical history but says that she has not seen a doctor in several years. She socially smokes and drinks alcoholic beverages. On examination, she is a mildly overweight female in no apparent distress. She has a red, nonpapular rash over her face. Fundoscopic examination is normal. She does not exhibit any lymphadenopathy. Auscultation of her lungs finds some faintly coarse breathing sounds. Auscultation of her heart does not find any abnormalities. Her abdomen is nontender with no masses. Examination of her extremities finds a patchy light red rash without scaling on her arms and legs. Palpation of her joints does not elicit pain. Range of motion causes only slight pain in her knees. She has no swollen or erythematous joints. She has full strength in all extremities but notes that the sensation in her left foot is not as good as the right. She has easily palpable pulses. She walks with a normal gait. The following vital signs are measured: T: 99.0°F, HR: 78 bpm, BP: 130/78 mm Hg, RR: 18 breaths/min
Differential Diagnosis Systemic lupus erythematosus, rheumatoid arthritis, sarcoidosis, Lyme disease, fibromyalgia, polymyositis, dermatomyositis, polymyalgia rheumatica, scleroderma, mixed connective tissue disease, psoriasis and psoriatic arthritis, Sjögren syndrome, ankylosing spondylitis
Laboratory Data and Other Study Results
CBC: WBC: 6.0, Hgb: 10.9, Plt: 157 ESR: 84 mm/hr CRP: 2.1 mg/dL Lyme antibody ELISA: negative Bilateral knee x-rays: no fracture or dislocation; normal bone alignment; no sign of joint effusion Bilateral ankle x-rays: no fracture or dislocation; normal bone alignment; no sign of joint effusion Bilateral hand x-rays: no fracture or dislocation; normal bone alignment Chest x-ray (CXR): mild congestion with increased bronchial markings; no atelectasis, infiltrates, or effusion After these results are received, the following additional studies are ordered: ANA: positive RF: negative Chem10: Na: 145 mEq/L, K: 4.1 mEq/L, Cl: 108 mEq/L, CO2: 30 mEq/L, BUN: 21 mg/dL, Cr: 1.3 mg/dL, Glu: 87 mg/dL, Mg: 2.1 mg/dL, Ca: 9.3 mg/dL, Phos: 3.2 mg/dL Liver function tests (LFTs): alkaline phosphatase (AlkPhos): 101 U/L, alanine aminotransferase (ALT): 45 U/L, aspartate aminotransferase (AST): 33 U/L, total bilirubin (TBili): 0.6 mg/dL, direct bilirubin (DBili): 0.3 mg/dL Urinalysis (UA): cloudy, pH: 7.4, specific gravity: 1.020, no nitrates/leukocyte esterase/glucose/ketones /hematuria/proteinuria After referral to a rheumatologist, the following studies are also performed: Anti-dsDNA antibodies: positive Anti-Sm antibodies: positive Antihistone antibodies: negative Anti-RNP antibodies: negative Anti-scl-70 antibodies: negative Anticentromere antibodies: negative Anti-SSA (Ro) antibodies: negative Anti-SSB (La) antibodies: negative
Diagnosis Systemic lupus erythematosus
Treatment Administered Through the consulting rheumatologist, the patient was started on hydroxychloroquine and told to take naproxen as needed for her joint pain The patient was instructed to avoid prolonged sun exposure and stress
Follow-up The patient experienced an improvement in her dermatologic and musculoskeletal complaints after beginning
hydroxychloroquine The patient was followed periodically during each year to monitor her renal function and the development of any new complaints After 5 years of this therapy, the patient's creatinine and BUN increased, and she was placed on prednisone to help treat the renal effects of the disease
The symptoms and signs of SLE may be found in any combination and may wax and wane during the course of the disease.
The common defining signs and symptoms of SLE may be remembered by the mnemonic MD SOAP CHAIR: Malar rash, Discoid rash, Serositis (e.g., pleuritis, pneumonitis, pleural effusion), Oral ulcers, Arthralgias, Photosensitivity, Central nervous system signs (e.g., psychosis, seizures, cerebrovascular accident [CVA], neuropathy), Heart and hematologic signs (e.g., pericarditis, myocarditis, arrhythmia, Libman-Sacks endocarditis, anemia, leucopenia, thrombocytopenia, Raynaud phenomenon), ANA positive, Immunologic findings (e.g., anti-dsDNA and anti-Sm antibodies), Renal findings (e.g., nephritis, proteinuria, increased BUN and creatinine).
In Libman-Sacks endocarditis autoimmune attack of cardiac tissue causes the formation of vegetations on the valves resulting in abnormal valve function.
Steps to the Diagnosis Systemic lupus erythematosus (SLE) A multisystem autoimmune disorder involving autoantibodies that affect multiple body systems Antibody-mediated cellular attack occurs with the deposition of antigen-antibody complexes in the affected tissues Hydralazine, procainamide, isoniazid, methyldopa, quinidine, and chlorpromazine can cause a similar constellation of symptoms that resolve when the drug is discontinued Risk factors: female gender, young adulthood, African, Asian, or Latino heritage History: malaise, weight loss, abdominal pain, vomiting, impaired vision, chest pain, arthralgias, photosensitivity (with an associated rash), weakness, paresthesias, patchy alopecia, and a cold sensation in hands or feet are all possible symptoms Physical examination: malar (i.e., butterfly rash across the cheeks and nose bridge) or patchy (i.e., discoid) rash, decreased or coarse breath sounds, oral ulcers, painful joint range of motion, psychosis, extra heart sounds, arrhythmias (Color Figure 9-5) Tests: Decreased hemoglobin, WBCs, and platelets are all possible Increased BUN and creatinine reflect renal involvement; liver transaminases may be mildly elevated in the acute onset of the disease ESR and CRP will be increased Multiple immunologic lab findings are notable (Table 9-5)
Color Figure 9-5 Patient exhibiting the classic malar rash of systemic lupus erythematosus.
ANA is positive in 95% of patients Anti-dsDNA antibodies are found in 60% of patients but are also common in other autoimmune conditions Anti-Sm antibodies are a highly specific finding but lack sensitivity for the diagnosis of SLE Complement C3 and C4 levels are frequently low Antihistone antibodies are frequently seen with drug-induced SLE-like symptoms Patients frequently have a false-positive test for syphilis Treatment: The avoidance of prolonged sun exposure and stressful environments help to prevent exacerbations
Table 9-5 Immunologic Markers Found in Rheumatic Diseases
Disease
SLE
Immunologic Markers
ANA (95% of patients) Anti-dsDNA antibodies (60% of patients) Anti-Sm antibodies False-positive RPR (syphilis test)
Drug-induced lupus
Antihistone antibodies
RA
RF (75% of patients) ANA (<50% of patients) HLA-DR4 common
Polymyositis/dermatomyositis
ANA Anti-Jo-1 antibodies
Ankylosing spondylitis
HLA-B27 (90% of patients)
Psoriatic arthritis
Possible HLA-B27
Scleroderma
Anti-scl-70 ANA
CREST syndrome
Anti-centromere antibodies
MCTD
Anti-RNP ANA
Sjögren syndrome
Anti-SSA (Ro) ANA Anti-SSB (La) ANA
ANA, antinuclear antibodies; CREST, calcinosis, Raynaud's, esophageal dysmotility, sclerodactyly, and telangiectasia; HLA, human leukocyte antigen; MCTD, mixed connective tissue disease; RA, rheumatoid arthritis; RF, rheumatoid factor; RPR, rapid plasma reagin; SLE, systemic lupus erythematosus.
NSAIDs are useful in the treatment of arthralgias Hydroxychloroquine is useful for treating the dermatologic and (mild) renal symptoms Corticosteroids are indicated to treat exacerbations of symptoms and more severe forms of the disease Immunosuppressants (e.g., cyclophosphamide, methotrexate) are used in cases resistant to corticosteroids Anticoagulants may be indicated in a patient who develops coagulopathy Outcomes: Lupus anticoagulant and anticardiolipin antibodies increase the risk of miscarriage and fetal death Coagulopathy may develop predisposing patients to thrombus formation, CVA, and cardiac events The disease follows a highly variable course, with some patients remaining benign and others progressing rapidly Patient death occurs from progressive impairment of the pulmonary, cardiac, cerebral, and renal systems The development of renal or cerebral abnormalities carries a worse overall prognosis than cases without these system involvements Clues to the diagnosis: History: polyarthralgias, mouth ulcers, rashes Physical: malar rash, mild paresthesia, mild joint motion pain Tests: anemia, increased ESR and CRP, positive ANA, positive anti-dsDNA and anti-Sm antibodies Rheumatoid arthritis More thorough discussion in prior case Why eliminated from differential: the presence of extra-articular symptoms and a negative rheumatoid factor
make this diagnosis less likely Sarcoidosis More thorough discussion in Chapter 2 Why eliminated from differential: the multiple immunologic lab abnormalities and dermatologic manifestations in this case make this diagnosis unlikely Lyme disease More thorough discussion in prior case Why eliminated from differential: the negative Lyme antibody ELISA and the multiple immunologic lab findings rule out this diagnosis Polymyositis/dermatomyositis Progressive systemic diseases with skeletal muscle inflammation One third of patients with polymyositis will also have dermatomyositis (i.e., skin manifestations) Pulmonary involvement may occur as a result of respiratory muscle involvement or interstitial disease Risk factors: female gender, African heritage, advanced age History: Polymyositis symptoms include myalgias and proximal muscle weakness Patients with pulmonary involvement will have dyspnea Physical examination: Polymyositis signs include muscle atrophy and motor weakness in the more proximal extremity muscles (legs affected before arms) Dermatomyositis signs include a red heliotropic rash on the face, arms, chest, and back, violet discoloration of the eyelids, or scaly patches on the hands Tests: Pulse oximetry may show poor oxygen saturation Increased creatinine, aldolase, creatine kinase, ALT, AST, and LDH are all possible ANA is frequently positive Anti-Jo-1 antibodies are frequently positive with lung involvement Muscle biopsy shows inflammatory infiltration and muscle degeneration Inflammatory cells are seen within muscle fascicles in patients with polymyositis Inflammatory cells are seen surrounding fascicles in patients with dermatomyositis EMG demonstrates spontaneous fibrillations with muscles Treatment: high-dose corticosteroids, methotrexate, or azathioprine are administered for 4 to 6 weeks followed by a tapered dosing; intravenous immunoglobulin (IVIG) or multiple immunosuppressants may be utilized in resistant cases Outcomes: most cases respond well to immunosuppressant medications; mortality is related to the
progression of interstitial lung disease or the development of lymphoma, lung cancer, or bladder cancer Why eliminated from differential: the absence of muscular symptoms (arthralgias is the primary musculoskeletal finding) make this diagnosis less likely Fibromyalgia A disease of chronic pain in musculotendinous tissues in the absence of inflammation Unknown etiology but frequently associated with depression, anxiety, and irritable bowel disease Abnormalities in the hormones of the hypothalamic-pituitary axis have been considered but not proven to be related to the condition History: myalgias, weakness, fatigue, possible depression, insomnia, dizziness, headaches, moodiness Physical examination: “trigger points” (i.e., specific locations on the body that when stimulated reproduce painful symptoms), no signs of inflammation at the sites of pain Tests: antipolymer antibodies are present in 50% of patients, but no other helpful tests are available Treatment: physical therapy and patient education are the cornerstones of treatment; tricyclic antidepressants (TCAs) and selective serotonin reuptake inhibitors (SSRIs) may be helpful for improving symptoms Outcomes: although the disease does not carry any mortality risks, it may be a chronic condition that is difficult to treat successfully Why eliminated from differential: the multiple lab abnormalities rule out this diagnosis Polymyalgia rheumatica (PMR) A rheumatic disease with multiple sites of muscle pain that is frequently associated with temporal arteritis (more thorough discussion in chapter 1) Most common in elderly women History: pain and stiffness in the shoulders and pelvis, difficulty raising the arms and with getting out of bed due to pain, malaise, weight loss Physical examination: fever, minimal joint swelling, intact strength despite painful motion Tests: Decreased hemoglobin Markedly increased ESR Negative RF MRI will show increased signal at the tendon sheaths and synovium PET scan will show increased uptake in large vessels Treatment: low-dose corticosteroids with a tapered withdrawal Outcomes: The condition is typically self-limited over several months to a few years Corticosteroids speed recovery Recurrences will occur in 25% of patients
Why eliminated from differential: the presence of multiple nonmuscular findings and the immunologic lab findings make this diagnosis unlikely
Once polymyalgia rheumatica has been diagnosed, the patient should automatically be worked-up for temporal arteritis.
Weakness helps to differentiate polymyositis (present) from PMR (absent).
Patients with PMR will frequently experience significant symptomatic improvement after just 1 day of corticosteroid administration.
CREST syndrome is a variant of the disease with Calcinosis of tissues, Raynaud phenomenon, Esophageal dysmotility, Sclerodactyly, and Telangiectasia. Symptoms are predominant in the hands and feet, anticentromere antibodies are seen in the work-up, and it has a better prognosis than classic scleroderma. Scleroderma Chronic multisystem sclerosis with an accumulation of dense connective tissue, skin thickening, and visceral involvement History: arthralgias, myalgias, hand swelling, difficulty swallowing (due to esophageal dysmotility), dyspnea Physical examination: Raynaud phenomenon (i.e., blue and cold hands and feet due to arteriolar spasm), skin thickening, possible arrhythmia Tests: ANA is positive in 95% of cases; anti-scl-70 and anticentromere antibodies may be present Treatment: Supportive care is administered for the skin complaints Calcium channel blockers and the avoidance of caffeine, nicotine, and decongestants are used in the treatment of Raynaud syndrome Methotrexate or corticosteroids may improve skin thickening and any pulmonary symptoms Angiotensin-converting enzyme inhibitors (ACE-Is) are administered if malignant renal hypertension develops Outcomes: Complications include finger and toe necrosis, pulmonary hypertension, wound infections and impaired healing, renal failure, and myocarditis Patients with mild cases have a 10-year survival of up to 70% Severe cases have a 10-year survival of 20% Why eliminated from differential: the absence of connective tissue thickening and negative anti-scl-70 antibodies help to rule out this diagnosis Sjögren syndrome More thorough discussion in later case Why eliminated from differential: the absence of dry mucosal surfaces and the negative anti-Ro and anti-La
antibodies rule out this diagnosis Mixed connective tissue disease An autoimmune disorder combining features of SLE, scleroderma, and polymyositis Cases may evolve into a single syndrome and may eventually change the diagnosis History: Raynaud phenomenon, arthralgias, swollen hands, proximal muscle weakness and myalgias, difficulty swallowing, dyspnea Physical examination: rash, fever, skin thickening, arrhythmias Tests: positive anti-RNP antibodies; ANA and RF are frequently positive Treatment: NSAIDs, corticosteroids, and ACE-Is Outcomes: cases typically evolve into one of the other rheumatologic diseases and have a better prognosis than those conditions Why eliminated from differential: the negative anti-RNP antibodies and the collection of findings that are more indicative of isolated SLE rule out this diagnosis Psoriatic arthritis Arthritis that develops in approximately 1% of patients with psoriasis Affects the DIP joints and the spine most commonly History: joint pain and stiffness that is worse in the morning and improves with activity, possible eye pain (from uveitis) Physical examination: joint line tenderness, painful joint range of motion, pitting of nails Tests: Negative RF and ANA; HLA-B27 may be positive X-rays show highly destructive lesions of the DIP and PIP joints (“pencil-in-cup” deformities); MRI may detect marrow edema Treatment: NSAIDs, methotrexate, sulfasalazine, or anti-TNF drugs may be utilized depending on the disease severity Outcomes: the functional detriments are similar to those seen with RA, and the long-term course of disability is similar to that for RA Why eliminated from differential: the absence of erosions on the x-rays and characteristic plaques of psoriasis and the presence of a positive ANA rule out this diagnosis Ankylosing spondylitis More thorough discussion in later case Why eliminated from differential: the absence of back pain and stiffness and the positive ANA rule out this diagnosis
Raynaud phenomenon may prohibit accurate measuring of pulse oximetry via a fingertip probe.
The pattern and timing of joint pain and stiffness are similar for both rheumatoid and psoriatic arthritis, but the pain for psoriatic arthritis is typically less severe.
Arthritic conditions that are negative for rheumatoid factor may be remembered by the mnemonic PEAR: Psoriatic arthritis, Enteropathic arthritis (inflammatory bowel disease), Ankylosing spondylitis, Reiter disease.
Case 9-8: “Why do I feel so dry all of the time?” A 39-year-old woman presents to her rheumatologist with the complaint of dry eyes and mouth. She has a known history of RA for which she is taking celecoxib and methotrexate. She has minimal disability and is able to work as a high school guidance counselor without any problems. She states that she has developed dry, painful eyes over the past month. She also says that she has difficulty articulating when speaking unless she frequently drinks water during conversations. She feels that this problem is due to a dry feeling in her mouth. When she looked in the mirror 1 week ago, she realized that her face was swollen in the preauricular region on both sides. This swelling is painless. She denies any drainage from the swollen areas or any strange taste in her mouth. She has had some difficulty swallowing solids unless she drinks frequently while eating. She denies any chest pain or shortness of breath. She denies any painful sensations in her face. She has occasional pain in both of her hands that responds well to taking celecoxib. She denies any new pain symptoms, weakness, or numbness. She has no past medical history besides her RA and takes no medications besides those listed earlier. She denies tobacco or alcohol use. On examination, she is a thin-appearing woman in no acute distress. She is found to weigh a similar amount to her previous visits. Her sclera and conjunctiva are injected, and she does not tear when asked to keep her eyes open during the examination. Fundoscopic examination detects a normal-appearing retina and vascularity. She does not have any palpable lymphadenopathy. Her parotid glands are enlarged bilaterally and nontender. She also has mild painless enlargement of her submandibular glands. She has no facial pain with palpation. Examination of her oropharynx detects minimal saliva formation but no plaques, sites of inflammation, or exudates. Auscultation of her lungs and heart finds clear breath sounds, a regular heart beat, and no abnormal sounds. Her hands and extremities in general are consistent with prior examinations. Neurologic examination detects no new weakness or paresthesias. The following vital signs are measured: T: 98.7°F, HR: 82 bpm, BP: 115/75 mm Hg, RR: 20 breaths/min
Differential Diagnosis Sjögren syndrome, salivary gland obstruction, sarcoidosis, scleroderma, bulimia
Laboratory Data and Other Study Results CBC: WBC: 8.1, Hgb: 11.5, Plt: 286 Chem7: Na: 137 mEq/L, K: 3.2 mEq/L, Cl: 110 mEq/L, CO2: 26 mEq/L, BUN: 11 mg/dL, Cr: 0.6 mg/dL, Glu: 81 mg/dL LFTs: AlkPhos: 86 U/L, ALT: 31 U/L, AST: 25 U/L, TBili: 0.6 mg/dL, DBili: 0.4 mg/dL Anti-Ro antibodies: positive Anti-LA antibodies: positive Anti-scl-70 antibodies: negative CXR: no atelectasis, infiltrates, effusions, or mediastinal masses Parotid gland biopsy: lymphocytic infiltration; no abnormal glandular cells
Diagnosis
Sjögren syndrome
Treatment Administered The patient was started on artificial tears and instructed to drink water regularly Corticosteroids were initiated with a tapering dose The patient was referred to an ophthalmologist to be evaluated for any eye abnormality
Follow-up The patient had an improvement in her symptoms once beginning steroid therapy No additional recommendations were made by the ophthalmologist
Steps to the Diagnosis Sjögren syndrome An autoimmune disorder with a lymphocytic infiltration of exocrine glands May be seen in association with RA, SLE, or primary biliary cirrhosis History: dry eyes, dry mouth, possible arthralgias Physical examination: enlarged parotid glands, purpura on legs, possible motor or sensory abnormalities in the extremities Tests: positive anti-Ro and anti-La antibodies Treatment: artificial tears, encouraging frequent water intake; corticosteroids may be given for significant cases Outcomes: the prognosis is typically good and correlates with the underlying autoimmune disorder Clues to the diagnosis: History: dry eyes and mouth, history of RA Physical: conjunctival and sclera injection, parotid and submandibular gland enlargement Tests: positive anti-Ro and anti-La antibodies Salivary gland obstruction Ductal obstruction of the sublingual, submandibular, or parotid glands due to sialolithiasis (i.e., ductal stone), sarcoidosis, infection, or tumor History: enlarged and painful gland; pain worsening with eating Physical examination: enlarged glands with tenderness to palpation (parotid glands may be painless) Tests: sialography may be performed to locate the site of obstruction; biopsy may be performed to rule out neoplasm Treatment: Warm compresses, massage, or cough drops are useful to help remove ductal stones Antibiotics and hydration are given for infection
Surgical decompression may be required in refractive cases Outcomes: the prognosis is good with adequate treatment of the underlying cause Why eliminated from differential: the biopsy results are more consistent with Sjögren syndrome than a primary salivary gland tumor or infection Sarcoidosis More thorough discussion in Chapter 2 Why eliminated from differential: the absence of hilar adenopathy and the results of the immunologic labs make this diagnosis unlikely Scleroderma More thorough discussion in prior case Why eliminated from differential: the absence of skin thickening, Raynaud phenomenon, and anti-scl-70 antibodies makes this diagnosis less likely Bulimia More thorough discussion in Chapter 13 Why eliminated from differential: the consistent patient weight, absence of dental erosions, and immunologic lab findings help to rule out this diagnosis
Sicca syndrome is Sjögren syndrome without a secondary autoimmune association.
Case 9-9: “I'm too young for chronic back pain” A 20-year-old man who is a college student presents to an orthopedic spine specialist with the complaint of chronic back pain that has lasted the past few years. He denies any history of back trauma and says that the pain began insidiously approximately 3 years ago. He does not recall what he was doing when he first noticed the pain. He says that he has pain in his mid- and lower back upon waking and feels stiff. The pain and stiffness improve after he is out of bed for a few hours. He says that he has difficulty remaining comfortable in long classes because he feels that the pain worsens with prolonged sitting. He says that prior to a few years ago he never had these symptoms. He played tennis in high school but has not competed in sports in college. He runs three times per week and does not feel that his back feels worse after this activity. In truth, he feels that his back feels better when he is running consistently several times per week. He denies any numbness, weakness, incontinence, headaches, syncope, or difficulty breathing. He does not have any pain in his extremities. He denies any past medical history and only takes ibuprofen for his back pain. He denies tobacco use but drinks heavily on weekends when he goes to parties. On examination, he is a well-appearing man in no acute distress. His face appears symmetrical, and his eyes are not inflamed. He has no lymphadenopathy. Auscultation of his lungs and heart detect clear breath sounds, a regular heart beat, and no abnormal sounds. His abdomen is nontender with no masses. Palpation of his back reveals mild tenderness only at his sacroiliac joints. There are no step-offs palpated in his spine. He has difficulty bending forward and laterally because of stiffness in his lower back. No asymmetry or lateral curvature of his spine is noted on examination. A neurologic examination detects normal motor and sensory function. The following vital signs are measured: T: 98.6°F, HR: 67 bpm, BP: 123/81 mm Hg, RR: 18 breaths/min
Differential Diagnosis Ankylosing spondylitis, scoliosis, disc herniation, spinal stenosis, musculotendinous strain
Laboratory Data and Other Study Results ESR: 55 mm/hr Thoracolumbar x-rays: straight spine with a decreased lumbar lordosis; multiple lumbar spine fusions with bone bridging the disc spaces; sclerosis of the sacroiliac joints; no fracture or lateral curvature Spine MRI shows mildly increased signal at the sacroiliac and intervertebral joints of the lumbar spine; no disc herniations or spinal cord edema
Diagnosis Ankylosing spondylitis
Treatment Administered The patient was referred to physical therapy to be instructed in exercises to maintain back flexibility and strengthening The patient was referred to an ophthalmologist to rule out uveitis Piroxicam was prescribed on an as-needed basis for periods of increased back pain
Follow-up The patient had some improvement in his back pain with therapy; he founds that his symptoms remained mild with regular exercise and stretching Anterior uveitis was ruled out by an ophthalmologist Over several years, the patient experienced occasional flare-ups of his back pain and stiffness, requiring short courses of oral corticosteroids to improve his symptoms
Steps to the Diagnosis Ankylosing spondylitis A chronic inflammatory disease of the spine and pelvis that results in the eventual fusion of adjacent vertebrae Some patients may have peripheral arthritis, anterior uveitis, or cardiac arrhythmias Risk factors: male gender, white race, young adulthood History: hip and low back pain that is worse in the morning and following periods of inactivity, pain improves over the course of the day and with exercise, possible eye pain and blurry vision, possible dyspnea, possible jaw pain Physical examination: limited range of motion in the spine and chest wall, possible eye redness and increased tearing, possible arrhythmia Tests: Increased ESR Positive HLA-B27 (90% of patients); negative RF and ANA X-ray shows a “bamboo spine” with squaring of the vertebrae, multiple intervertebral fusions, and sclerosis of the sacroiliac joints (Figure 9-8)
MRI will show increased signal at the intervertebral and sacroiliac joints due to inflammation Treatment: Physical therapy and NSAIDs are the first-line treatments in patients with mild or early disease and may help delay the progression of stiffness Sulfasalazine, methotrexate, corticosteroids, or anti-TNF drugs may ease symptoms in more severe disease Joint replacement in the extremities or decompression, osteotomies, or fusion in the spine may be required in progressive cases
Figure 9-8 Lateral x-ray of the cervical spine in a patient with ankylosing spondylitis. Note the “bamboo spine” appearance with squaring of the vertebrae and multiple intervertebral fusions leading to a significantly decreased range of motion.
Outcomes: complications include vertebral fractures, spinal instability, and heart block; progression of back symptoms may lead to functional disability Clues to the diagnosis: History: back pain that is worse in the morning and after sitting that improves with activity Physical: sacroiliac tenderness, decreased spine range of motion Tests: appearance of the spine x-rays Scoliosis
A resting lateral curvature of the spine with an associated rotational deformity The curve is at a risk of progressing during periods of rapid growth Larger curves are more likely to progress Principally a cosmetic issue, butlarge curves may lead to chronic back pain, functional limitations, and pulmonary compromise History: back pain, perceived spinal deformity Physical examination: palpable curvature of the spine, asymmetry of the back with forward bending Tests: x-rays are important to detecting the location and extent of curves Treatment: Adolescent curves >25 degrees are observed Curves between 25 and 40 degrees are treated with bracing Surgical correction is indicated for adolescent curves >40 degrees, curves in young children, and rapidly progressing curves Outcomes: Pulmonary dysfunction is rare and only occurs in severe curves Neurologic injury is the most-feared complication of corrective surgery Patients with corrected curves tend to be more active and hold a better self-image Why eliminated from differential: the appearance of the x-ray rules out this diagnosis Disc herniation More thorough discussion in prior case Why eliminated from differential: the absence of radicular symptoms and the appearance of the MRI rule out this diagnosis Spinal stenosis More thorough discussion in prior case Why eliminated from differential: the appearance of the x-ray and MRI rule out this diagnosis Musculotendinous strain Partial tear of a muscle or related tendon due to overuse or acute excessive loading History: pain at the site of injury, history of excessive or chronic strain placed on a specific muscle group Physical examination: tenderness to palpation at the site of injury, pain with resisted motion, mild swelling, weakness Tests: studies are typically not required for the diagnosis unless significant disability is noted; MRI will demonstrate inflammation and partial tears or complete rupture of a given muscle or tendon Treatment: Rest, ice, elevation, and compressive dressings are the standard treatment for incomplete injuries
NSAIDs are useful for decreasing pain and inflammation Physical therapy is useful to decrease pain and to improve function Surgical repair is indicated for complete muscle or tendon ruptures Outcomes: the majority of partial injuries recover uneventfully; complete tears of musculotendinous units are associated with functional disability unless a timely stable repair is performed Why eliminated from differential: the evidence of bony abnormalities on the x-ray make this diagnosis unlikely
Case 9-10: “My son is having trouble walking” A 5-year-old boy is brought to a pediatrician by his mother because she has noticed that he has been limping for several days. She says that she is unaware of any trauma sustained by the child in the past week. Her other children deny having seen the patient sustain any injuries. During this time the child has been hesitant to walk much and has asked to be carried by his parents. He has started to complain that his right hip hurts. When the patient's mother tried to move his hip, he appeared uncomfortable but not in excessive pain. She denies any fevers in the patient. None of her other children have been sick recently or have complained of similar symptoms. The patient has not complained of pain in any joint other than his right hip. The mother denies any past medical history in her son. He was the product of an uncomplicated vaginal delivery at 40 weeks of gestation and had a vertex presentation. He takes children's vitamins but no other medications. He is up to date on all of his vaccinations and well-child visits. There is no history of inflammatory arthropathies in the family. On examination, the child is a healthy-appearing boy in no acute distress. Examination of his eyes detects a normal light reflex and no irritation. Fundoscopic examination is normal. Examination of his ears detects normal tympanic membranes. He has no lymphadenopathy. Auscultation of his lungs and heart is normal. His abdomen is nontender with no masses and normal bowel sounds. He is mildly uncomfortable with range of motion of his right hip. His range of motion is somewhat less in the right hip compared to the left. No abnormal sounds are elicited with motion of the right hip. Range of motion is normal and painless in all other joints. When asked to walk, the child is notable for a Trendelenburg limp on the right side. Neurologic examination detects normal motor and sensory function throughout. The following vital signs are measured: T: 98.7°F, HR: 90 bpm, BP: 110/70 mm Hg, RR: 18 breaths/min
Differential Diagnosis Septic arthritis, developmental dysplasia of the hip, juvenile rheumatoid arthritis, Legg-Calve-Perthes disease, slipped capital femoral epiphysis, fracture
Laboratory Data and Other Study Results CBC: WBC: 7.6, Hgb: 14.8, Plt: 238 ESR: 10 mm/hr CRP: 0.3 mg/dL ANA: negative Pelvis x-ray: asymmetry of the hips with the left hip appearing normal; the right hip is notable for a small and incomplete-appearing femoral head and a widened joint space; the right acetabulum appears similar to the left side; no fractures or dislocations
Diagnosis Legg-Calve-Perthes disease
Treatment Administered The patient was referred to a pediatric orthopedic specialist for definitive care The child was referred to physical therapy to improve his range of motion The patient was placed in an abduction brace following an improvement in his motion to prevent full weight bearing
Follow-up Serial hip x-rays were performed at monthly intervals to look for an improvement in the right femoral bone quality After 3 months of bracing, improvement was noted in the size and shape of the femoral head The patient was gradually weaned from bracing but was withheld from athletic activities until the femoral head regained its normal appearance in another 2 years
Steps to the Diagnosis Legg-Calve-Perthes disease Avascular necrosis of the epiphysis of the femoral head related to its fragile blood supply Most commonly occurs in children between 3 and 8 years of age History: insidious onset of hip pain, painful or painless limp In a Trendelenburg limp hip abductor weakness causes the contralateral hip to sag during single-leg stance on the affected hip Physical examination: decreased hip range of motion Tests: x-ray of the hips will demonstrate hip asymmetry, and the affected hip will have some degree of femoral head collapse followed by gradual regrowth Treatment: the femoral head must be contained within the acetabulum by bracing or surgical reconstruction; acetabular osteotomies are performed in cases where permanent hip dysplasia develops Outcomes: The prognosis is dependent on the ability to keep the femoral head contained within the acetabulum and the age of the patient Patients over 6 years of age tend to have worse outcomes on average compared to younger patients Patients with an incomplete recovery of the femoral head are at a high risk for early osteoarthritis, progressive avascular necrosis of the femoral head with structural collapse, and a need for joint arthroplasty at an early age Clues to the diagnosis: History: painful limp, insidious onset of pain Physical: decreased hip range of motion Tests: appearance of the pelvis x-ray Septic arthritis
More thorough discussion in prior case Why eliminated from differential: the normal WBC, ESR, and CRP make this diagnosis unlikely, but an aspiration of the hip joint should be considered if any of these labs are elevated Developmental dysplasia of the hip (DDH) Perinatal displacement of the femoral head from the acetabulum, disrupting the normal development of the hip joint Occurs as a result of poor development of the acetabulum in utero Risk factors: female gender, first-born child, breech presentation, oligohydramnios History: delayed age of walking, abnormal gait in newly ambulatory children Physical examination: positive Barlow (i.e., posteriorly directed force on an adducted hip produces a dislocation) and Ortolani (i.e., reduction of a dislocated hip with abduction) maneuvers, positive Galeazzi sign (i.e., knees sit at unequal heights when the hips and knees are flexed together), positive Trendelenburg gait, asymmetric skin folds of the thighs Tests: ultrasound of the hips is the most commonly used means of determining hip congruity; x-rays tend to not be helpful until at least after 4 months of age because of the limited calcification of the femoral head Treatment: The Pavlik harness is used in children <6 months of age to keep the hips in a reduced position Closed or open reduction and spica casting are performed in children between 6 months and 2 years of age Open reduction is performed after 2 years of age Correction may not be performed once the acetabulum has formed (approximately 8 years old) because it is frequently dysplastic by that time and unable to retain the femoral head Outcomes: inadequate treatment results in permanent hip dysplasia and a high likelihood of early joint degeneration and a need for early arthroplasty Why eliminated from differential: the appearance of the x-ray in this case is more consistent with Perthes disease than DDH Juvenile rheumatoid arthritis (JRA) A nonmigratory arthropathy that occurs in children, affecting one or more joints that lasts longer than 3 months Classified as pauciarticular, polyarticular, or systemic, depending on the constellation of symptoms (Table 9-6) History: arthralgias, morning stiffness Physical examination: fever, other signs depending on the subtype Tests: WBCs, ESR, CRP, ANA, and RF are useful labs for making the diagnosis; x-rays may demonstrate osteopenia and subchondral sclerosis Treatment: NSAIDs are the frequent first-line drug; methotrexate, sulfasalazine, or corticosteroids may be used in resistant cases Outcomes: prognosis is dependent on the variant of the disease encountered, and outcomes may range from full recovery to chronic arthritis
Why eliminated from differential: the negative ANA and the x-ray appearance in this case make this diagnosis unlikely Slipped capital femoral epiphysis (SCFE) Separation of the femoral head epiphysis through the growth plate, resulting in slippage of the epiphysis
Table 9-6 Variants of Juvenile Rheumatoid Arthritis
Pauc iartic ular
Joints involved
<4 joints; major joints except
Poly artic ular
Systemic
5+ joints; hips less common
Any number
2–3 years old
2–5 and 10–14 years old
Any age <17 years old
Insidious swelling and decreased
Symmetric joint involvement; spine
Significant acute pain; pain severity
range of motion
involvement; hand deformities
may follow fevers; neck stiffness is
hips
Age at presentation
Joint symptoms
common; occasional jaw involvement
Extraosseous
30% of cases have uveitis or
Growth retardation, fever, rare
Spiking fevers, maculopapular rash,
symptoms
iridocyclitis
iridocyclitis
organomegaly, lymphadenopathy, pericarditis, growth retardation
Labs
Treatment
Prognosis
Complications
Mildly increased ESR; mildly decreased Hgb;
Increased WBC; decreased Hgb;
weakly positive ANA in younger ages;
increased ESR; negative ANA; rarely
positive RF in older ages
positive RF
NSAIDs; methotrexate for
NSAIDs, methotrexate, sulfasalazine, or
NSAIDs, methotrexate,
chronic cases
etanercept
corticosteroids, or cytotoxic drugs
Most cases resolve in <6 months;
60% patients enter remission within 15
Highly variable course; 50% patients
uncommon chronic arthritis
years; higher rate of severe chronic
will achieve eventual remission,
arthritis than pauciarticular type; worse
significant minority have chronic
prognosis with older onset
disease
Chronic arthritis; leg length discrepancy
Leg length discrepancy; chronic
Weakly positive ANA
Blindness from iridocyclitis; leg length discrepancy; rare
arthritis; jaw arthritis; amyloidosis
chronic disease with progressive arthritis
ANA, antinuclear antibodies; ESR, erythrocyte sedimentation rate; Hgb, hemoglobin; NSAIDs, nonsteroidal anti-inflamatory drugs; RF, rheumatoid factor; WBC, white blood cells.
Risk factors: adolescent, obesity, African heritage, hypothyroidism History: thigh and knee pain, reluctance to bear weight Physical examination: limp, limited abduction and internal rotation of the hip, obligatory external rotation of the hip with flexion Tests: x-ray will show a posterior and medial displacement of the femoral head epiphysis Treatment:
Surgical pinning of the affected hip is performed The patient is kept non–weight bearing after surgery if unable to bear weight prior to surgery Patients with hypothyroidism frequently undergo prophylactic pinning of the unaffected side Outcomes: complications include avascular necrosis of the femoral head and the development of early arthritis if surgery is not performed in a timely fashion Why eliminated from differential: the appearance of the x-ray in this case is not consistent with this diagnosis Fracture More thorough discussion in prior case Why eliminated from differential: the appearance of the x-ray rules out this diagnosis
Avascular necrosis of the femoral head is a common complication in children with sickle cell disease.
Case 9-11: “Our daughter's leg is very bowed” A 10-year-old girl is brought to a pediatric orthopedic specialist by her parents because she has developed a progressive lateral curvature in her left leg over the past 2 years. Her parents say that she has become very knock-kneed on the left side only. Her right leg appears to be normal. The patient has begun to have difficulty walking and running because of her leg deformity. She has become very conscious about her appearance because of this problem. The patient reports pain on the lateral side of her left knee. She says that she is also sore in the remainder of both legs and in her lower back. Prior to 8 years of age, the patient's legs were straight. At that age, the patient fell off a horse and broke her left femur. She was treated with temporary intramedullary nails for this injury. Her parents say that this fracture healed and the nails were removed. The patient has not followed-up with an orthopedic surgeon since 6 months after the surgery. Her parents say that their daughter is accident-prone and has had four other fractures throughout her childhood. With the exception of her femur fracture, these have been treated with casting. They are unaware of any other extremity deformities. They say that their daughter has no other medical problems. She is a very picky eater, so they give her a daily children's vitamin. She was the product of an uncomplicated vaginal delivery at 39 weeks gestation. She has two younger siblings who are in good health. On examination, she is a thin-appearing girl in no distress. Her eyes appear normal with white sclera. Examination of her mouth finds normal dentition. She has no lymphadenopathy. Her arms appear symmetrical and have normal movement. Her right leg is straight and has painless range of motion. Her left leg is bowed inward at the knee and her lower leg projects away from her body by 15 degrees. Because of this angle, her left leg appears mildly shorter than the right side. She has mild pain at the lateral joint line of the knee. Neurologic examination finds normal motor and sensory function. The following vital signs are measured: T: 98.4°F, HR: 70 bpm, BP: 110/70 mm Hg, RR: 17 breaths/min
Differential Diagnosis Rickets, growth plate arrest, osteogenesis imperfecta, juvenile rheumatoid arthritis
Laboratory Data and Other Study Results Chem10: Na: 142 mEq/L, K: 4.0 mEq/L, Cl: 101 mEq/L, CO2: 24mEq/L, BUN: 10 mg/dL, Cr: 0.6 mg/dL, Glu: 88 mg/dL, Mg: 2.2 mg/dL, Ca: 9.1 mg/dL, Phos: 4.0 25-hydroxyvitamin D3: 40 ng/mL 1, 25-dihydroxycholecalciferol: 32 pg/mL
LFTs: AlkPhos: 54 U/L, ALT: 41 U/L, AST: 32 U/L, TBili: 0.8 mg/dL, DBili: 0.4 mg/dL ANA: negative RF: negative Left knee x-ray: no fracture or dislocation; fusion of the lateral distal femoral physis; 15-degree valgus angulation of the distal femur and knee The following additional study is ordered: Knee CT: obliteration of the lateral 40% of the distal femoral physis with bridging bone and the remainder of the physis intact
Diagnosis Growth plate arrest
Treatment Administered Surgical resection of the bone, bridging the lateral physis and a distal femoral osteotomy, was performed to better align the distal femur
Follow-up The patient's left leg was straighter following the surgical correction By 14 years of age the patient had a subtle recurrent valgus deformity of the left knee and a 1-cm leg length discrepancy Because the patient was asymptomatic for her leg alignment, no treatment besides observation was undertaken
Steps to the Diagnosis Growth plate arrest Cessation of normal bony growth at part or all of the physis due to prior physeal injury or a congenital deformity The majority of physeal arrests occur as a result of a prior Salter-Harris growth plate fracture (Table 9-7) History: progressive angulation of a bone with growth over time Physical examination: angulation of a bone at the physis, altered gait or extremity mechanics due to the deformity Tests: x-ray will show angled growth of a bone and a closed physis at the region of prior injury; CT is useful to quantify the degree of physis replaced by a bony bridge Treatment: Bone bridges involving <50% of the physis are treated with physeal bridge resection and interposition of fatty tissue to prevent reformation of the bridge Bone bridges involving >50% of the physis are treated either with completion of the physeal arrest and contralateral epiphysiodesis or an ipsilateral limb lengthening procedure Corrective osteotomy of the abnormal bone is frequently required to improve alignment Outcomes: the degree of deformity is dependent on the age of the child at the time of injury and the
remaining time of bony growth prior to physeal fusion; careful operative planning must be undertaken to choose the correct combination of procedures and their timing to minimize any permanent deformity or leg length discrepancy Clues to the diagnosis: History: prior femoral fracture, normal leg alignment prior to injury Physical: unilateral bone angulation Tests: appearance of the distal femoral physis on x-ray and CT Rickets Impaired calcification of bone in children due to defects in vitamin D and phosphate metabolism Nutritional rickets results from an inadequate intake of calcium, vitamin D, or phosphate
Table 9-7 Salter-Harris Classification of Physeal Fractures
Type
Desc ription
Prognosis
I
Physeal separation without extension into adjacent bone
Good with adequate reduction, quick healing
II
Partial physeal separation with proximal extension into
Good; rare growth disturbance
metaphysis
III
IV
V
Partial physeal separation with distal extension into
Poor unless accurate reduction; fixation usually required to maintain
epiphysis
stability
Fracture extends through metaphysis, physis, and
Perfect reduction must be achieved; guarded prognosis even with good
epiphysis
reduction
Crush injury of physis
High likelihood of partial growth arrest
Familial rickets results from an abnormal metabolism of vitamin D or an impaired absorption of phosphate in the kidneys Regardless of the cause, hypertrophy of the epiphyseal cartilage and impaired calcification leads to progressive deformity of bones at the physis History: bone pain, delayed walking age, fractures from minimal trauma Physical examination: bowed legs (i.e., genu varum), kyphoscoliosis, proximal limb weakness, short stature, skull softness Tests: Increased alkaline phosphatase and decreased phosphorus in all types Decreased calcium, decreased 25-droxyvitamin D3 and 1, 25-dihydroxycholecalciferol, and increased parathyroid hormone in all types, except rickets due to impaired phosphate absorption Increased 25-droxyvitamin D3 and 1, 25-dihydroxycholecalciferol in rickets due to impaired phosphate
absorption X-rays will demonstrate widening of the physis, bowing of long bones, translucent lines in bones, flattening of the skull, and enlarged costal cartilages Treatment: calcium, vitamin D, and phosphate supplementation are administered to help normalize physeal development Outcomes: the likelihood of permanent bone deformities correlates with the time until treatment is initiated Why eliminated from differential: the unilateral nature of the deformity in this patient and the normal lab studies rule out this diagnosis Osteogenesis imperfecta A defect in the production of collagen I, resulting in abnormal bone development characterized by significant bone fragility and impaired healing History: frequent fractures from minimal trauma, possible deafness Physical examination: blue sclera (most cases), poor dentition, joint hypermobility Tests: Collagen electrophoresis may be used to identify the abnormal collagen production X-rays show evidence of multiple fractures, excessive callus production, and abnormal healing; malunions and bone shortening may be apparent Treatment: Protected activity and closely supervised physical therapy are recommended to prevent fractures and maximize function Surgery should be reserved for significant bony injury or malunion Bisphosphonates are helpful to prevent excessive bone reabsorption Outcomes: Prognosis depends on the severity of the disease Mild cases have a descent prognosis if injury is avoided Severe cases are highly associated with premature death or significant disability Why eliminated from differential: the normal sclera and dentition make this diagnosis less likely Juvenile rheumatoid arthritis More thorough discussion in prior case Why eliminated from differential: the negative ANA and RF and the appearance of the x-ray in this cases make this diagnosis less likely
Rickets in adults is called osteomalacia.
Osteogenesis imperfecta may be detected during a work-up for suspected child abuse.
Authors: Van Kleunen, Jonathan P. Title: Step-Up to USMLE Step 3, 1st Edition Copyright ©2009 Lippincott Williams & Wilkins > Table of Contents > Chapter 10 - Dermatology
Chapter 10 Dermatology Basic clinical primer Wounds and healing Wound types Clean Surgical incisions through disinfected skin No gastrointestinal (GI) or respiratory contamination Generally a 1%–3% infection risk for first-time procedures Clean-contaminated Surgical wounds with GI or respiratory tract exposure A 2%–8% infection risk Contaminated Gross contact of a wound with GI or pulmonary contents or environmental exposure Includes traumatic wounds A 6%–15% infection risk Dirty Established infection in tissues prior to incision The risk of continued infection following debridement ranges from 7%–40% Wound healing Surgical wounds are typically covered for 48 hours following closure with dressing changes as needed following the removal of the initial bandage Open wounds require debridement and specialized dressings (e.g., vacuum-assisted, Dakin solution, wet-to-dry dressings) designed to promote healing and reduce the risk of infection Wound healing may be inhibited by malnutrition, corticosteroid use, smoking, hepatic or renal failure, or diabetes mellitus (DM) Types of wound healing
Primary intention Surgical closure of the wound (i.e., deep tissues and skin) performed to optimize the healing environment Low risk of infection Typically performed in clean and clean-contaminated wounds or in contaminated wounds following substantial cleaning Secondary intention Wound is left open and allowed to heal through epithelialization Allowed to occur in infected wounds or with significant superficial skin loss preventing full wound closure Incorporates various dressing techniques to promote successful healing Higher risk of infection Delayed primary closure Wounds are left open for multiple days prior to a repeat cleansing and closure P.240
Table 10-1 Common Types of Skin Grafts and Tissue Flaps Used in Wound Repair
Ty pe
Desc ription
Split-thickness
Skin graft composed of epidermis and
graft
partial dermis
Common Donor Sites
Abdomen, thighs, buttocks
Indic ations
Skin replacement in wounds; useful to cover an extensive surface area
Full-thickness
Skin graft composed of epidermis and
Above ears (for face),
graft
full dermis
forearm, groin
Composite graft
Skin grafts that also contain other
Fingertip, ear, etc.
tissues (e.g., cartilage, nail bed, fat)
Defects on face and hands
Site-specific anatomical reconstruction
Fasciocutaneous
Skin and subcutaneous tissue with an
Forehead, groin,
Large defects with a good
flap
attached vascular supply
deltopectoral region, thighs
vascular supply requiring padding
Muscle flap
Transferred muscle that either
Tensor fascia lata, gluteal
Areas with inadequate
includes skin (i.e., myocutaneous
muscles, sartorius, rectus
vascularized tissue, exposed deep
flap) or that requires additional skin
abdominus, latissimus dorsi
tissues, or severe radiation
graft
injury
Performed in contaminated wounds in which closure is physically possible and in which serial debridement is able to provide a clean wound Tissue transport Transfer of the cutaneous and/or deeper soft tissues (i.e., subcutaneous tissue, fascia, muscle) to a wound that cannot be physically closed where adequate protection of deeper neurovascular, visceral,
or musculoskeletal structures cannot be provided (Table 10-1) Skin grafts are most commonly autografts (i.e., from healthy tissue on the same patient), but may occasionally be allografts (i.e., donor tissue), or rarely xenografts (i.e., donor tissue from another species) Flaps may be rotational or transpositional (i.e., left partially attached to the donor site) or may be free flaps (i.e., tissue completely removed from the donor site, transferred to the wounds, and revascularized at the wound)
Case 10-1: “My skin has been red and blistered since the picnic” A 23-year-old woman presents to her primary care provider (PCP) because of a 1-day history of erythema and pain in her face and hands. She says that she was at an all-day picnic 2 days ago and has had the above symptoms since that time. The picnic was held outdoors in a field. The patient says that it was a cloudy day and that she did not wear sunscreen. By the evening after the picnic she noticed pain in her face and hands. She found that both her face and the back of her hands were erythematous and that she had some small blisters on her forehead and nose. She denies any pruritus or numbness. She reports that she has had sunburns in the past but that they were not as severe as the current presentation. She admits that she was playing in the grass with her dog and both her face and hands were in contact with vegetation in the field. She has not noticed any insects on herself or her dog and denies finding any insect bites or sores on her body. She was wearing a long-sleeved shirt, jeans, and closed shoes, and her face and hands were the only part of her body exposed. She denies any past medical history and uses only oral contraceptives as a regular medication. She denies any known allergies. She drinks socially and denies other substance use. On examination, she is a well-appearing woman in no distress. Her face and the dorsum of both hands up to the wrist are quite erythematous. A few tiny blisters are notable on her forehead and on the bridge of her nose. Her skin is warm to the touch and blanches easily with pressure, but it is apparent that touching the inflamed regions is somewhat painful to the patient. No discrete lesions, patterns of inflammation, or wounds are noticeable. Examination of her arms above her wrists, her torso, and her legs finds normal-appearing skin without the characteristics of her hands and face. No flaking of the skin or dry patches are apparent. Examination of her eyes detects pink conjunctiva and white P.241 sclera. Her lips are inflamed, but her mouth appears normal without any swelling. She has no lymphadenopathy. The following vital signs are measured: Temperature (T): 98.7○F, heart rate (HR): 84 beats per minute (bpm), blood pressure (BP): 122/81 mm Hg, respiratory rate (RR): 18 breaths/min
Differential Diagnosis Sunburn, contact dermatitis, cellulitis, atopic dermatitis, porphyria cutanea tarda, systemic lupus erythematosus
Laboratory Data and Other Study Results None performed
Diagnosis Mild second- and first-degree thermal burns due to sun exposure
Treatment Administered The patient was advised to take moderate-dose (600 mg) ibuprofen as needed, to maintain her fluid intake, and to use a skin moisturizer with a mild topical anesthetic as needed
Follow-up
The patient's symptoms resolved over several days The patient was educated about the risks of sunburn and was provided with literature describing ways to prevent future burns
Steps to the Diagnosis Burns Injury to the epithelium and dermis of the skin due to exposure to significant heat, radiation, caustic substances, or electrical shock Classified by the depth of involvement First degree: epidermis only Second degree: partial thickness dermal involvement Third degree: full epidermis and dermis and some fatty tissue involvement Estimation of the surface extent of burns may be classified by the “Rule of 9s” (Figure 10-1) History: First degree: pain at the site of involvement Second degree: moderate to severe pain at the site of involvement Third degree: the site of true third-degree injury is typically painless Physical examination: First degree: erythema at the site of exposure Second degree: erythema and blistering at the site of exposure Third degree: charred, leathery, or gray skin at the site of involvement Electrical burns: similar appearance to third-degree burns, possible cardiac arrhythmias and neurologic abnormalities (e.g., visual abnormalities, seizures, sensory or motor dysfunction) Tests: burns are typically a clinical diagnosis, so additional testing is typically not required; serum carboxyhemoglobin should be measured in patients suspected of smoke inhalation to determine the risk of impaired tissue oxygenation Treatment: Any burning agents or chemicals should be removed from the skin (including affected clothing) to prevent further injury Caustic substances should be neutralized or diluted P.242
Figure 10-1 “Rule of 9s” for calculating the surface extent of burns. (A) The surface anatomy of the adult is divided into sections of 9% body surface area (genitals are considered 1% surface area). Note that the distribution considers both the front and back of the head and arms as single contributions. (B) Because of its greater relative size, the contribution of the head is increased in the child. Note that the front and back for the head, arms, and legs are considered as single contributions. (See color image.)
First-degree and mild second-degree burns may be treated with cooling, cleansing, bandaging, topical antimicrobials, and topical agents intended to reduce rapid epidermal loss (e.g., moisturizers, emollients) Vigilant use of sunscreen is important to preventing sunburn (usually a first-degree or mild second-degree radiation burn) Second-degree burns involving more than 10% body surface area (BSA) and third-degree burns involving more than 2% BSA require inpatient treatment for intravenous (IV) hydration, wound care, and possible surgical escharectomy Second- and third-degree burns affecting the face, hands, genitalia, or major skin flexion creases must be considered for inpatient treatment because serious scarring or functional loss is a potential complication Second- or third-degree burns involving more than 25% BSA or with significant face involvement require airway management (possible intubation), IV fluids, and careful body temperature regulation Patients with significant smoke inhalation should receive high-flow oxygen and close monitoring for respiratory distress Patients with electrical burns should be monitored until urine output is consistently normal, cardiac and neurologic examinations remain normal, and adequate perfusion is confirmed in all extremities; fasciotomies or escharotomies should be performed for any extremity demonstrating impaired perfusion from an impending compartment syndrome or constrictive eschar Cardiac and neurologic complications from shocks should be managed appropriately to reduce mortality
Outcomes: Prognosis is related to the depth of the burn and the BSA involved, with outcomes being worse as each increases Potential complications of acute burns include significant fluid loss, infection, neurovascular impairment, and significant scarring that compromises function Repetitive burns (e.g., sunburn) increase the overall risk of skin cancers P.243 Clues to the diagnosis: History: prolonged outdoor exposure, lack of sunscreen use, involvement only on exposed areas Physical: erythema, blistering Tests: nonapplicable
Sunburn results from exposure to ultraviolet radiation from the sun and may occur on both sunny and mildly cloudy days.
Burns secondary to electrical shock are sometimes called fourth-degree burns because they may involve muscles, bones, and other internal structures.
Determine the postburn IV fluid resuscitation need with the Parkland formula: lactated Ringer's solution is given in a total volume of [(4 mL) × (kg body weight) × (% body surface area burned)]. Half of the volume is given during the initial 8 hours, and the remaining half is given over the following 16 hours. Contact dermatitis An allergic reaction in the skin due to cutaneous contact (e.g., plants, animal dander) with a given allergen A somewhat similar but more diffuse reaction is seen with ingestion of the allergen An allergic reaction occurs through one of two general mechanisms Type I: due to mast cell degranulation; light diffuse rash (i.e., urticaria) appears shortly after exposure and lasts several hours Type II: due to lymphocyte activity; measleslike (i.e., morbilliform) rash appears several days after a second exposure to the allergen (most contact dermatitis cases) History: pruritus, history of contact or ingestion of an allergen, history of a previous similar reaction Physical examination: erythematous rash in a distinct pattern (e.g., lines, shapes) in contact dermatitis, rash in a characteristic location or poorly defined area following ingestion of an allergen (Color Figure 10-1) Tests: application testing (i.e., a small amount of the allergen is applied to the skin to elicit a reaction) is useful to identify culprit allergens Treatment: Stop use of the offending agent or remove contact with the allergen Mild cases may be treated with topical corticosteroids and antihistamines
Oral corticosteroids may be required in severe case Outcomes: an inability to identify the causative allergen may lead to repeat exposures; secondary bacterial infection is an uncommon complication Why eliminated from differential: the diffuse nature of the erythema without a pattern makes this diagnosis less likely
Common causes of allergic contact dermatitis include plants (e.g., poison ivy, poison oak, etc.), nickel, soaps, and latex.
Use the pattern of a rash to distinguish an external cause (defined shape) from an internal cause (nondefined distribution) of the rash. Cellulitis Acute skin infection most commonly due to Staphylococcus aureus or group A streptococcus Methicillin-resistant S. aureus (MRSA) has evolved as a cause of cellulitis that is difficult to treat because of antibiotic resistance Risk factors: IV drug use, DM, immunocompromise, skin penetration, previous cellulitis, vascular or lymphatic dysfunction History: pain at the site of involvement, myalgias, chills Physical examination: erythema, swollen skin, lymphadenopathy (Color Figure 10-2) Tests: increased serum white blood cells (WBC), C-reactive protein (CRP), and erythrocyte sedimentation rate (ESR); occasionally magnetic resonance imaging (MRI) or ultrasound (US) may be used to rule out deeper infection Treatment: Oral cephalosporins or penicillinase-resistant β-lactam for 10 to 14 days in mild cases IV antibiotics are required for more severe cases or for bacteremia Diabetic patients should receive broad coverage because of the increased risk of multiorganism infections Patients with MRSA will require treatment with vancomycin, TMP-SMX, doxycycline, or linezolid Outcomes: Complications include the extension of the infection into joint spaces, fascia, muscle, or other deeper tissues Abscess formation may result in severe infections Recurrent cases may result in lymphedema Prognosis is usually very good when the appropriate antibiotic therapy is utilized Why eliminated from differential: although this diagnosis should be considered, the limitation of the findings to the exposed areas makes sunburn more likely; a lack of improving findings within days should lead to a full cellulitis work-up P.244 Atopic dermatitis (a.k.a. eczema)
A chronic inflammatory skin rash characterized by dry skin patches with papules Both infantile (resolves in the initial few years of life) and adult (recurrent history) forms exist Risk factors: asthma, allergic rhinitis, family history History: pruritus, recurrent lesions in adults Physical examination: erythematous patches of dry skin with possible blisters on the flexor surfaces, dorsal hands and feet, chest, back, or face (face and scalp more common in infants) (Color Figure 10-3) Tests: typically not needed to make the diagnosis Treatment: Avoidance of precipitating factors Use of moisturizing creams, emollients, or topical corticosteroids or tacrolimus may help to improve the lesions Severe cases may be treated with oral corticosteroids and antihistamines Outcomes: prognosis is generally good, although a small portion of patients will develop asthma or allergic rhinitis Why eliminated from differential: the absence of dry skin in this patient makes this diagnosis unlikely Porphyria cutanea tarda A disease resulting from a deficiency of hepatic uroporphyrinogen decarboxylase, an enzyme involved in heme metabolism Exposure to a hepatotoxic substance (e.g., alcohol, tobacco) or history of a hepatic disease (e.g., viral hepatitis, HIV, hemochromatosis, Wilson's disease) results in the excessive production of sun-sensitive porphyrins Risk factors: alcohol use, hepatitis C, iron overload, estrogen use, tobacco use History: blistering on sun-exposed areas Physical examination: blistering and erosions on chronic sun-exposed areas, hypopigmented scars, hyperpigmented patches of skin, dark urine Tests: decreased serum uroporphyrinogen decarboxylase; increased porphyrins in the serum, urine, and feces Treatment: Avoidance of excessive sunlight exposure helps to prevent the lesions Chloroquine is used to bind porphyrins and improve their excretion Periodic phlebotomy is performed to decrease the body's excessive iron stores and reduce the excess porphyrins Outcomes: chronic excessive iron stores from porphyrin accumulation increases the risk of cirrhosis and hepatocellular carcinoma Why eliminated from differential: since blistering does not appear to be a consistent occurrence in this patient with sun exposure, this diagnosis is unlikely Systemic lupus erythematosus
More thorough discussion in Chapter 9 Why eliminated from differential: the absence of any other significant symptoms makes this diagnosis unlikely
Although MRSA was initially limited to nosocomial transmission, the rates of community-acquired MRSA have increased significantly in recent years.
Skin and wound cultures are rarely useful in cellulitis because they frequently contain other normal skin flora or are falsely negative.
Case 10-2: “My child has sores around his mouth” A 5-year-old boy is brought to his pediatrician by his mother with a 2-day history of blistering around the mouth. The patient's mother says that she noticed a single tiny blister near his lips 2 days ago, but now there are multiple small blisters around his mouth and nostrils. Several of the blisters have broken and have become encrusted. She has not noticed any other lesions elsewhere on his face or on his body. She says that he has never had these lesions previously. She says that her son has been complaining about his mouth hurting and itching and has been rubbing his lips. He has not complained of any congestion, ear or throat pain, or cough. She has not noticed any shortness of breath or abnormal activities P.245 besides the lip rubbing. The child has not consumed any new foods recently or used any new personal hygiene products. She adds that he is in kindergarten, and when she talked to another student's mother the day prior to presentation, the other parent reported that her daughter had a similar finding. The patient is an only child. He has no past medical history. The family history is significant for adult-onset hypertension, hypercholesterolemia, breast cancer, and cervical cancer. The patient takes children's vitamins and has no known allergies. He is up to date on his vaccinations. On examination, the patient is a well-appearing boy in no distress. He has multiple yellow-crusted lesions around his lips and nostrils. A few small (1 to 2 mm) pustules are also near his lips. There are no scars, telangiectasias, or dry skin patches. No lesions are found on his forehead, around his eyes, across his nose, on his cheeks, or on his neck. Examination of his eyes detects white sclera, pink conjunctiva, and no exudates. Examination of his ears detects translucent tympanic membranes with normal motion to air. Examination of his oropharynx detects no inflammation or exudates. He has mild cervical lymphadenopathy bilaterally. Auscultation of his heart and lungs finds clear breath sounds, normal heart sounds, and no extra sounds. Examination of his torso and extremities does not detect any abnormal lesions. The following vital signs are measured: T: 98.7°F, HR: 90 bpm, BP: 110/70 mm Hg, RR: 16 breaths/min
Differential Diagnosis Acne vulgaris, impetigo, herpes simplex, varicella, atopic dermatitis, cutaneous fungal infection, scabies, molluscum contagiosum, pemphigus vulgaris, seborrheic dermatitis, Stevens-Johnson syndrome
Laboratory Data and Other Study Results Complete blood cell count (CBC): white blood cells (WBC): 12.2, hemoglobin (Hgb): 14.1, platelets (Plt): 234 Urinalysis (UA): straw colored, pH: 6.9, specific gravity: 1.010, no nitrates/leukocyte esterase/glucose/ketones /hematuria/proteinuria Gram stain and culture of pustule contents: Gram-positive cocci in clusters, culture pending Tzanck smear of lesions: no multinucleated giant cells Potassium hydroxide (KOH) preparation of lesions: no hyphae or pseudohyphae Hematoxylin and eosin microscopy: small collections of purulent and necrotic material in the stratum corneum of
the epidermis; many neutrophils; occasional collections of cocci in clusters
Diagnosis Impetigo
Treatment Administered The patient was prescribed dicloxacillin for 10 days The patient's mother was instructed to wash her child's face with a chlorhexidine antimicrobial solution daily until the lesions appeared to be nearly resolved The child was kept home from school during his treatment
Follow-up Culture results indicated methicillin-sensitive S. aureus and multiple potential contaminant bacteria but no evidence of MRSA The child's lesions appeared much improved at a 1-week follow-up appointment The child's lesions resolved completely in <2 weeks, and he was allowed to return to school P.246
Steps to the Diagnosis Impetigo A highly contagious skin infection caused by S. aureus or group A streptococci Most common in infants and school-aged children History: facial pruritus and pain, no symptoms of otitis media or pharyngitis Physical examination: erythematous vesicles or pustules around mucosal surfaces that eventually break and become yellow crusted, lymphadenopathy (Color Figure 10-4) Tests: Gram stain and culture of exudates beneath the yellow crusting or within the pustules may be helpful to identify an organism (although contamination is possible) but is more useful to rule out MRSA as a causative organism Treatment: Topical antibiotics may be used for limited cases with only a few pustules Oral antibiotics are indicated for more extensive lesions Lesions should be washed with a antimicrobial soap or solution until they have resolved Outcomes: streptococcal glomerulonephritis and scarring are rare complications; lesions rarely resolve without treatment, but the prognosis is good once treatment is initiated Clues to the diagnosis: History: evolution of lesions from a few pustules to yellow-crusted lesions, evidence of other children at school with similar findings, facial itching Physical: yellow-crusted lesions and small pustules around the mouth and nose, lymphadenopathy
Tests: evidence of Gram-positive bacteria on microscopy
Children with impetigo should be held out of school until the lesions have resolved to prevent transmission to other students. Acne vulgaris Inflammation of hair follicles and sebaceous glands associated with Propionibacterium acne infection Risk factors: adolescence, androgens (use or elevated levels), obstruction of skin pores (e.g., exfoliated skin, cosmetic products, personal care products) History: possible pain at the site of lesions Physical examination: erythematous pustules or cystic lesions on the face, neck, chest, or back (Color Figure 10-5) Tests: typically a clinical diagnosis; outbreaks in adult patients may warrant a work-up for androgen excess Treatment: Topical retinoids decrease sebaceous gland activity and are the recommended first-line treatment Oral or topical antibiotics may inhibit bacterial growth Benzoyl peroxide helps prevent follicular obstruction by decreasing skin oiliness (second-line therapy) Oral contraceptives may be useful in women with excess androgen production to help in hormone regulation Oral isotretinoin may be prescribed for severe cases but requires close monitoring of liver enzymes and mandatory pharmacologic contraception (high teratogenic risks) Outcomes: prognosis is generally excellent, and most cases resolve by the end of adolescence; scarring may result from severe cases of cystic acne Why eliminated from differential: the age of the patient and the limit of the lesions to only the mouth and nose make this diagnosis unlikely
There is no proven association between acne vulgaris and certain types of food.
Acne usually decreases in severity as adolescence ends. Corticosteroid use and androgen production disorders are common causes of outbreaks in adulthood.
Women should have at least two negative urine pregnancy tests before being prescribed oral isotretinoin. Herpes simplex A recurrent viral infection of mucocutaneous surfaces caused by herpes simplex virus 1 or 2 (HSV-1, HSV-2) Transmitted through contact with oral or genital fluids HSV-1 causes primarily oral disease; HSV-2 causes primarily genital disease Following primary infection, viral genetic material remains dormant in the sensory ganglia but becomes
reactivated during periods of stress P.247 History: painful vesicles around the mouth or genitals lasting several days, possible dysphagia and odynophagia with esophageal involvement Physical examination: small vesicles around mucosal surfaces, possible vision deficits if lesions are periocular (Color Figure 10-6) Tests: Tzanck smear of the lesions will show multinucleated giant cells; viral culture may be used to confirm the diagnosis Treatment: The condition is essentially incurable, so therapy is directed toward minimizing the severity and frequency of flare-ups Acyclovir, famciclovir, or valacyclovir shorten the duration and frequency of recurrences Sexual partners should use barrier contraception consistently and should avoid sexual contact during exacerbations to avoid transmission Outcomes: herpes meningitis, ocular infection, and bacterial infection of sores are uncommon complications; maternal transmission to a neonate during delivery can result in severe central nervous system infection in the child Why eliminated from differential: the negative Tzanck smear and the yellow-crusted appearance of the lesions rule out this diagnosis Varicella Infection by varicella zoster virus (a.k.a. herpes zoster) that may present as a primary disease (i.e., chickenpox) or recurrent presentation (i.e., shingles) (Table 10-2) History: an eruption of vesicles that differ between the primary and recurrent forms Physical examination: numerous small vesicles in a wide distribution (i.e., chickenpox) or limited to a single dermatome (i.e., shingles) (Color Figure 10-7) Tests: typically a clinical diagnosis; Tzanck smear may show giant cells Treatment: supportive care; acyclovir may be used to shorten the course of the disease or prevent worsening in immunocompromised patients
Table 10-2 Characteristics of Primary (Chickenpox) and Recurrent (Shingles) Varicella
V aric ella Condition
Patients
Chic kenpox (primary)
More common in c hildren
affected
Shingles (rec urrent)
Patient with a prior history of varicella zoster infection
Timing of
Symptoms 2 or more weeks after infection occurs; symptoms of
Myalgias, fever, and malaise preceding lesions by
presentation
headache, malaise, myalgias, and fever precede the development of
approximately 3 days
lesions by <3 days
Type of lesion
Distribution of
Small red macules that evolve into papules and then vesicles that
Small red macules that evolve into papules and then
eventually become crusted
vesicles that eventually become crusted
W ide distribution
Limited to a single or few distinct dermatomes;
lesions
involvement of multiple dermatomes indicates disseminated disease
Course of
Lesions may develop for up to 1 week and resolve a few days after
Lesions exist for a week and may be painful; infective
disease
appearing; infective until lesions crust over
until lesions crust over
Treatment
Complications
Antipruritics aid symptoms; acyclovir used in severe cases or in
Analgesics and possible corticosteroids; acyclovir is
immunocompromised patients; vac c ination has reduced disease
used in immunocompromised patients and with
incidence significantly
trigeminal nerve distribution
More severe course in older and pregnant patients (increased risk of
Postinfectious neuralgia (i.e., long-lasting pain at site
varicella pneumonia); may have severe consequences if passed from
of eruption), trigeminal neuropathy
infec ted mother to unborn fetus
P.248 Outcomes: the prognosis is good in most primary cases; refer to Table 10-2 for more potential complications Why eliminated from differential: the limited distribution of the lesions is not consistent with primary disease, and shingles in a vaccinated child is highly unlikely Atopic dermatitis More thorough discussion in prior case Why eliminated from differential: although the appearance of the lesions for atopic dermatitis and impetigo may be similar, the presence of lymphadenopathy is more indicative of an infectious process in this case Cutaneous fungal infection More thorough discussion in Table 10-3 (Color Figure 10-8) Why eliminated from differential: the negative KOH preparation makes this diagnosis less likely Scabies Cutaneous infestation by the Sarcoptes scabiei mite Risk factors: crowded living conditions, poor hygiene History: severe pruritus at the site of involvement, pruritus worsens after soaking in hot water Physical examination: visible mite burrows, papular rash around burrows (Color Figure 10-9) Tests: microscopic examination of skin scraping may detect mites and eggs Treatment: Permethrin cream or oral ivermectin is used to kill the existing mites Diphenhydramine is useful to control itching All clothing, towels, and linens used by the patient must be washed in hot water Outcomes: infection of close contacts is common; prognosis is excellent with the proper treatment and
cleansing Why eliminated from differential: the absence of mite burrows on inspection and mites on microscopy rule out this diagnosis Molluscum contagiosum A viral skin infection most frequently seen in children and in patients with immunodeficiencies History: painless papules Physical examination: shiny papules with a central umbilication Tests: Giemsa and Wright stains will help demonstrate large inclusion bodies on histology of the lesions Treatment: frequently self-limited; chemical, laser, or cryotherapy may be considered for removal
Table 10-3 Common Cutaneous Fungal Infections
Condition
Tinea versicolor
Fungus
Malassezia furfur
Lesions
Diagnosis
Treatment
Small scaly macules most
KOH prep shows short
Topical antifungal agent
frequently on chest and
hyphae, Wood's lamp
for several weeks or oral
back
examination shows
ketoconazole for 1–5
extent of disease
days
KOH prep shows hyphae
Topical antifungal agent
Tinea not due to
Microsporum,
Pruritic, erythematous,
M. furfur Described by
Trichophyton,
scaly plaques with
for multiple weeks Oral
location: corporis (body),
Epidermophyton
central clearing
antifungal agent for resistant cases
cruris (groin), pedis (feet), unguium (nail beds), capitis (scalp)
Intertrigo
Candida albicans
Pruritic, painful,
KOH prep shows
Topical antifungal agent
erythematous plaques
pseudohyphae
Topical corticosteroid
with pustules most commonly in skin creases
KOH, potassium hydroxide.
P.249 Outcomes: prognosis is excellent Why eliminated from differential: yellow crusting and pustule formation are not typically seen in this diagnosis Pemphigus vulgaris More thorough discussion in later case Why eliminated from differential: the age of the patient and limited extent of the lesions are not typical for this diagnosis Seborrheic dermatitis Chronic hyperproliferation of the epidermis most commonly on the scalp or face Most common in infants and adolescents
History: pruritus at the site of involvement Physical examination: erythematous plaques with greasy, yellow scales Tests: the diagnosis is typically clinical Treatment: shampoo containing selenium, tar, or ketoconazole may be used on the scalp; topical corticosteroids or antifungals may be used on other regions of the body Outcomes: recurrences are common Why eliminated from differential: although the lesions for this diagnosis and impetigo may appear somewhat similar, the location of the lesions in this case would be unusual for this diagnosis Stevens-Johnson syndrome More thorough discussion in later case Why eliminated from differential: the limited extent of the lesions and the absence of systemic symptoms make this diagnosis unlikely
The primary demonstration of herpes simplex infection is typically more severe than subsequent flares.
Check the varicella immunity status (i.e., prior vaccination or disease history) in all pregnant women; varicella immune globulin should be given to all nonimmune pregnant women who contract the disease.
Immunocompromised patients are at an increased risk for developing encephalopathy or retinitis as complications from varicella infection.
Lesions of molluscum contagiosum are found on the face, torso, and extremities in children and in the perineal region in adults.
“Cradle cap” is seborrheic dermatitis of the scalp in infants.
Case 10-3: “My arm is turning black” A 57-year-old man presents to an emergency department with the complaint that his left arm is swollen and turning a darker color. The patient says that he has chronic sores on his arms and in his left axilla. He says that he scratches his left armpit often because it itches and occasionally has scratched open the lesions located there. He says that while he was fishing on the river bank 2 days ago he slipped and fell into the water. He has not bathed since this event. He feels that his axilla looked worse than normal when he awoke today, and his left arm has become progressively swollen over the course of the morning. He says that the upper arm has become erythematous over the past few hours and that it feels “funny.” He has not noticed any drainage from his axilla. At the same time he has begun to feel fatigued and generally not well. He has had chills for part of today. It is painful to move his left shoulder because of the swelling. He states that his symptoms have never been this bad previously. He describes a past medical history of diabetes mellitus and hypertension (HTN). He has not taken any medications for several months but says that he was prescribed a few drugs for his “heart and blood sugar.” He denies any allergies. He is homeless and generally lives outdoors. He drinks alcohol on multiple days per week but denies tobacco or other substance use. On examination, he is an unkempt man in mild distress. Both of his arms are notable for a few 2-mm sores that are in various states of healing. His left arm is quite swollen from the axilla to the elbow. It is warm to the touch, and palpation elicits a subtle popping sensation under the skin near the axilla. He has several 1-cm palpable masses in the axilla. When his shoulder is
abducted to better visualize the undersurface, purulent material begins to drain from a small wound in his armpit. His skin is a dark red proximally and fades to a light red by the elbow, and multiple small blisters are notable proximally. His right arm appears normal other than the aforementioned small sores. Auscultation of his lungs and heart find clear lung and heart sounds and no abnormal sounds. He has full strength in both arms, although movement of his left shoulder is painful. Sensory testing reveals a subjective decrease in upper arm light touch sensitivity compared to the right. The following vital signs are measured: T: 102.5°F, HR: 105 bpm, BP: 108/75 mm Hg, RR: 20 breaths/min P.250
Differential Diagnosis Necrotizing fasciitis, gangrene, cellulitis, skin abscess, hidradenitis suppurativa
Laboratory Data and Other Study Results CBC: WBC: 27.1, Hgb: 12.4, Plt: 319 7-electrolyte chemical panel (Chem7): sodium (Na): 145 mEq/L, potassium (K): 4.1 mEq/L, chloride (Cl): 100 mEq/L, carbon dioxide (CO2): 24 mEq/L, blood urea nitrogen (BUN): 15 mg/dL, creatinine (Cr): 0.9 mg/dL, glucose (Glu): 302 mg/dL Coagulation panel (Coags): protime (PT): 12.1, international normalized ratio (INR): 1.0, partial thromboplastin time (PTT): 31.1 ESR: 24 mm/h CRP: 12.4 mg/dL Shoulder x-ray: no fracture or dislocation; significant soft tissue swelling in the axilla and visible upper arm; subcutaneous air in the axilla tracking down to the mid-portion of the upper arm
Diagnosis Necrotizing fasciitis
Treatment Administered The patient was taken emergently to the operating room where an extensive irrigation and debridement of the left upper arm was performed (significant purulent material was found along the fascial planes of the medial upper arm) Operative Gram stain and culture of diseased tissue was performed The patient's wound was packed following debridement and bandaged without primary closure IV vancomycin was started until culture results were reported The patient was placed on an insulin regimen to better control his blood glucose levels
Follow-up The patient was returned to the operating room every 2 days for additional irrigation and debridement procedures Partial closure was achieved after the third irrigation and debridement, and the remainder of the wound was covered with a skin graft Cultures indicated an infection by MRSA, and vancomycin was continued
Successful control of the patient's glucose was achieved, and he was transitioned to oral medication The patient was discharged to a skilled nursing facility to continue his wound management and IV antibiotics
Steps to the Diagnosis Necrotizing fasciitis A quick-spreading infection of the fascial planes of an extremity, leading to extensive soft tissue destruction and systemic infection S. aureus and group A streptococci are the most common causative organisms History: significant pain at the site of involvement that evolves into numbness; history of a recent surgery or wound Physical examination: significant spreading erythema, warm and swollen skin, decreased sensation near the affected area, fever, subcutaneous crepitus, vesicles or bullae around the affected region, purulent drainage P.251 Tests: Increased WBC, ESR, and CRP Intraoperative cultures are important for determining the causative organism Subcutaneous collections of air are seen on x-ray or computed tomography (CT) Treatment: Emergent surgical debridement is required; amputation is a frequent definitive procedure Multiple repeat surgeries are frequently required to eliminate all necrotic material; skin grafts or tissue flaps may be needed to achieve adequate wound coverage IV antibiotics are used to treat any extended or systemic infection Outcomes: complications include renal failure, sepsis and septic shock, and scarring; acute mortality is 70% Clues to the diagnosis: History: rapidly evolving arm swelling and erythema, chronic sores with an acute exposure to river water Physical: significant erythema, arm swelling, subcutaneous crepitus, purulent drainage Tests: increased WBC, ESR, and CRP, intraoperative cultures Gangrene Tissue necrosis due to a poor vascular supply or severe infection May be classified as either wet or dry, depending on the appearance Clostridium is the bacteria most commonly implicated in cases due to infection History: prior skin infection or wound, severe skin pain Physical examination: fever, hypotension, subcutaneous crepitus, rotten-smelling skin Tests: Ankle-brachial indices will demonstrate asymmetry between the limbs
WBC, ESR, and CRP are elevated in infectious cases Culture is useful to identify a pathogen in cases due to infection Subcutaneous collections of air are seen on x-ray or CT in cases due to infection Angiography or magnetic resonance angiography (MRA) is used to demonstrate vascular insufficiency Treatment: debridement and antibiotic therapy are required; amputations are frequently indicated for nonviable extremities Outcomes: Prompt treatment carries the best chance for limb salvage The need for distal amputations in vascular disease is usually a harbinger for the eventual need of additional, more proximal amputations Cases due to infections can result in systemic involvement Why eliminated from differential: although this diagnosis and necrotizing fasciitis may appear very similar, the rapid onset of symptoms and limb compromise in this case is more consistent with the process of necrotizing fasciitis
Dry gangrene is gradual necrosis of the skin from vascular insufficiency and features hard and dry skin. Wet gangrene is necrosis due to acute vascular obstruction or infection and features blistering and swelling of the involved area. Cellulitis More thorough discussion in prior case Why eliminated from differential: the extensive involvement of the upper arm, obvious collections of purulent material, and rapid evolution are suggestive of a process much more severe than cellulitis Skin abscess A subcutaneous collection of pus most commonly due to staphylococcal bacteria May occur as a collection of multiple infected hair follicles (i.e., carbuncle) History: painful region of focal cutaneous swelling Physical examination: erythema, fluctuant subcutaneous mass, tenderness to palpation Tests: possibly increased WBC, ESR, and CRP; cultures of abscess contents can determine the pathogen but carry high false-positive rates if not performed under sterile conditions Treatment: irrigation and debridement with healing by secondary intention; antibiotic therapy (oral or IV depending on the extent of the abscess) P.252 Outcomes: the prognosis is good if treated in a timely fashion; large abscesses may erode into adjacent areas (e.g., joint spaces, fascial planes), leading to worsening infection that requires additional treatment Why eliminated from differential: the rapidly evolving nature of this case is suggestive of a more advanced infection Hidradenitis suppurativa A condition of chronic follicular occlusion and apocrine gland inflammation, resulting in recurrent abscesses in
the axilla, groin, and perineum History: pruritus at the site of involvement, recurrent abscesses in the above-mentioned regions Physical examination: multiple fluctuant subcutaneous masses, tenderness to palpation, evidence of significant scarring from earlier abscesses, skin tethering Tests: culture of abscess material may be useful for identifying a pathogen; CT is useful to determine the extent of the disease prior to surgery Treatment: Antibiotics should be given for any abscess formation Corticosteroid injections into nonpurulent collections may help them to resolve before becoming infected Irrigation and debridement of abscesses, excision of sinus tracts, and extensive resection are all surgical options, depending on the extent of the disease Outcomes: fistula formation, chronic infection, and significant soft tissue fibrosis are potential complications; recurrences are extremely common without some type of wide resection of the involved area Why eliminated from differential: this diagnosis is a chronic condition, while the presentation in the case is an acute, rapidly worsening process
Abscesses in the lower back and perineal regions have an increased risk of being due to anaerobic bacteria.
Case 10-4: “My skin is falling off!” A 34-year-old woman is a recently admitted inpatient (three days ago) on a medicine floor following the new onset of seizures. A dermatology consultant is called following the acute onset of a widespread macular rash and desquamation of the hands, feet, and face. The patient reports feeling a generalized achiness and itching of her mouth, hands, and feet. She says that these symptoms began the prior night and have worsened overnight. She became extremely concerned when she looked in the mirror this morning and saw her skin peeling off around her eyes, nose, and mouth. She says that in addition to the pruritus her mouth and eyes have become painful. She feels that her vision is slightly blurry this morning. She feels nauseous and has been having diarrhea since last night. She denies any headache or dizziness. She says that she has never had this experience previously. She has a history of a closed head injury sustained 3 months ago in a motor vehicle crash, reflux disease, and migraine headaches. She takes an oral contraceptive, ranitidine, and occasional sumatriptan. She was started on phenytoin during the current admission. She denies any previous surgery or seizures. She drinks socially and denies other substance use. On examination, the patient is an anxious-appearing woman in no distress. Examination of her face shows flat, dark-red macules around her eyes, mouth, and nose. There is central blistering in these macules that is oozing a serous fluid. No bleeding is evident. No fluctuant masses are detected. The skin around these lesions is peeling heavily, and the underlying dermis is red. Her sclera and conjunctiva are injected, and she is tearing. Funduscopic examination detects normal retina vasculature and no intraocular abnormalities. Examination of the nasopharynx and oropharynx finds irritation at the external borders but no erythema or swelling internally. A few macules and blisters are detected on the patient's chest, arms, and legs. The epidermis is peeling from her palms and the plantar surface of her feet. Auscultation of her lungs and heart detects mildly coarse breath sounds, a regular heart rate, and no extra heart sounds. Her abdomen is nontender with no masses and hyperactive bowel sounds. Neurologic examination detects normal motor and sensory function. The following vital signs are measured: T: 99.5°F, HR: 108 bpm, BP: 115/80 mm Hg, RR: 20 breaths/min
Differential Diagnosis Erythema multiforme, Stevens-Johnson syndrome, toxic epidermal necrosis, pemphigus vulgaris, bullous
pemphigoid, atopic dermatitis, contact dermatitis
Laboratory Data and Other Study Results CBC: WBC: 3.9, Hgb: 12.6, Plt: 174 Chem7: Na: 141 mEq/L, K: 3.8 mEq/L, Cl: 101 mEq/L, CO2: 27 mEq/L, BUN: 15 mg/dL, Cr: 0.7 mg/dL, Glu: 87 mg/dL Liver function tests (LFTs): alkaline phosphatase (AlkPhos): 103 U/L, alanine aminotransferase (ALT): 73 U/L, aspartate aminotransferase (AST): 57 U/L, total bilirubin (TBili): 1.0 mg/dL, direst bilirubin (DBili): 0.3 mg/dL ESR: 34 mm/hr CRP: 3.1 mg/dL Skin biopsy: widespread epidermal necrosis; dermis lymphocytic infiltration
Diagnosis Stevens-Johnson syndrome
Treatment Administered Phenytoin was immediately stopped The patient was transferred to a burn unit and placed on contact isolation Warmed IV fluids were administered to maintain an adequate urine output Careful cleansing with a silver nitrate solution was performed daily to reduce bacterial contact
Follow-up The patient required several weeks of hospitalization to allow adequate healing of the cutaneous tissues The patient complained of chronic dry eyes even after her skin had healed The patient developed hypopigmentation of the affected regions and frequently wore cosmetic makeup to conceal these blemishes
Steps to the Diagnosis Stevens-Johnson syndrome A severe hypersensitivity reaction to certain medications, infection, or vaccinations Mucous membrane involvement predominates Sulfonamides and anticonvulsants are the most common culprit medications History: malaise, myalgias, pruritic macules, nausea, vomiting, diarrhea, mouth and eye pain, changes in vision Physical examination: red macular rash (mucosal surfaces are most commonly involved), central blistering in the macules, desquamation, conjunctiva irritation Tests: Lab tests are not required for the diagnosis
Decreased WBCs, hemoglobin, and platelets Increased ALT, AST, ESR, and CRP Skin histology will demonstrate epidermal-dermal separation (mild cases) or epidermal necrosis (severe cases), macrophage and lymphocytic infiltration, and a relatively intact dermis Treatment: Immediate cessation of all new medications Patients should be treated in a burn unit and placed in isolation to minimize the risk of infection Careful skin cleansing using chlorhexidine or a silver nitrate solution should be performed to reduce the risk of infection Adequate IV hydration must be maintained to prevent dehydration Adequate pain control Respiratory or nutritional support as needed for cases involving the lungs or GI tract Outcomes: Prognosis is generally good with a prompt diagnosis Potential complications include long-term ocular sequelae (e.g., dry eyes, worsening vision, blurry vision), hypopigmentation, renal failure, respiratory or cutaneous bacterial infections, and esophageal strictures Mortality is 5% per case Clues to the diagnosis: History: new phenytoin use, pruritus, eye and mouth pain, nausea, diarrhea, blurry vision Physical: red macular rash, rash around mucosal surfaces, desquamation, conjunctivitis, and scleritis Tests: relative pancytopenia, elevated ALT and AST, skin biopsy results Erythema multiforme A cutaneous hypersensitivity reaction that is less severe than Stevens-Johnson syndrome Manifestations are typically only cutaneous in nature History: malaise, myalgias, pruritus Physical examination: macular rash with occasional blister formation on the face, palms, and soles, target lesions (i.e., erythematous center surrounded by a pale inner ring and an erythematous outer ring) Tests: increased eosinophils; skin biopsy will show lymphocytic and macrophage infiltration and limited epidermal necrosis Treatment: Stop any offending agents Typically a self-limited condition once the inciting substance is identified Adequate analgesia Outcomes: prognosis is good, and most patients recover without any significant sequelae; failure to recognize the diagnosis may result in a progression to Stevens-Johnson syndrome
Why eliminated from differential: the widespread extent of this presentation and the presence of GI symptoms suggest a condition more severe than this diagnosis
HSV and Mycoplasma pneumoniae are common infectious causes of erythema multiforme. Toxic epidermal necrosis (TEN) The most severe form of a cutaneous hypersensitivity reaction characterized by significant skin sloughing and full-thickness epidermal necrosis Frequently considered a more extensive variant of Stevens-Johnson syndrome (i.e., >30% of the BSA) History: malaise, myalgias, pruritic macules, nausea, vomiting, diarrhea, mouth and eye pain, changes in vision Physical examination: widespread red macular rash (mucosal surfaces are most commonly involved), central blistering in the macules, extensive desquamation, conjunctiva irritation (Color Figure 10-10) Tests: Decreased WBCs, hemoglobin, and platelets Increased ALT, AST, ESR, and CRP Skin histology will demonstrate extensive epidermal necrosis, macrophage and lymphocytic infiltration, and a relatively intact dermis Treatment: Stop the offending agent immediately Surgical debridementof necrotic tissue is frequently required The remainder of treatment is the same as for Stevens-Johnson syndrome
Color Figure 10-10 Toxic epidermal necrolysis. This severe dermatologic condition begins as a generalized erythematous rash that progresses into widespread desquamation and erosion formation.
Color Figure 10-11 Pemphigus vulgaris. Fragile bullae develop which rupture easily leading to widespread erosions and desquamation.
Outcomes: although the prognosis correlates with the extent of BSA involved, mortality is generally >10% Why eliminated from differential: although this diagnosis and Stevens-Johnson syndrome are essentially
variants of the same process, the limitation of the presentation in this case to mainly the face, hands, and feet makes Stevens-Johnson syndrome a better classification Pemphigus vulgaris An autoimmune disorder of autoantibodies to epidermal adhesion molecules, resulting in widespread blistering Separation between epidermal layers occurs to produce blistering Occurs in middle-aged and elderly patients History: painful blistering on the chest, face, mouth, and perineum Physical examination: blister are fragile and many appear ruptured during examination; cutaneous erosions may be visible (Color Figure 10-11) Tests: skin biopsy will show separation of the epidermal layers with an intact basement membrane; immunofluorescence will demonstrate the deposition of antiepidermal antibodies within the epidermis Treatment: corticosteroids or immunosuppressive agents are used to control the disease Outcomes: the prognosis is poor without treatment but has been brought to a 10% mortality rate with the appropriate therapy; potential complications include cutaneous infections and sepsis Why eliminated from differential: the acute nature of this case and the recent new use of phenytoin make this diagnosis less likely; should the lesions persist, a more extensive histologic work-up would be warranted Bullous pemphigoid An autoimmune disorder characterized by autoantibodies to the epidermal basement membrane Separation between the epidermis and dermis occurs to cause blistering Most patients are elderly History: pruritic blistering Physical examination: widespread firm blistering (especially flexor surfaces and in the perineal region), occasional erosions from rupture blisters (Color Figure 10-12) Tests: skin biopsy will show separation between the epidermis and dermis; immunofluorescence will demonstrate the deposition of antibasement membrane antibodies beneath the epidermis Treatment: oral or topical corticosteroids, tetracycline, and immunosuppressants are used to control the disease Outcomes: the prognosis is better than for pemphigus vulgaris; recurrences are infrequent with long-term therapy once remission is achieved and blisters have healed
Color Figure 10-12 Bullous pemphigoid. Multiple large bullae form on an erythematous base leading to severe erosions.
Why eliminated from differential: the acute nature of this case and the recent new use of phenytoin make this diagnosis less likely; should the lesions persist, a more extensive histologic work-up would be warranted Atopic dermatitis More thorough discussion in prior case Why eliminated from differential: the extensive nature of the lesions seen in this case makes this diagnosis unlikely Contact dermatitis More thorough discussion in prior case Why eliminated from differential: the extent and severity of the lesions seen in this case make this diagnosis unlikely
Case 10-5: “My arms and legs are scaly” A 22-year-old man presents to a dermatologist with scaly, erythematous lesions on his elbows and shins. The patient says that he has gradually developed these lesions over the past 2 years. They have grown more rapidly in the past 8 months. He has applied several moisturizers and emollient creams to the lesions, but they have not improved the lesions. He found that an over-the-counter antifungal cream did seem to help somewhat, so he is concerned that he has a bad fungal infection on his arms and legs. He says that the lesions are slightly itchy, and when he scratches them, they bleed easily. The lesions are not painful and do not limit his ability to work as an accountant. He always wears long pants and rarely wears short-sleeved shirts because he is concerned about his appearance. He states that he has never had these lesions anywhere else on his body and that he did not have any of them at all before 2 years ago. He denies any GI symptoms or any weakness or numbness in his extremities. He does not have any joint pain. He denies any past medical history and does not take any medications. He smokes a pack of cigarettes per week and drinks socially. On examination, he is a well-appearing man in no distress. He has erythematous plaques with white and silver scales on the back of his elbows and distal upper arms. Similar lesions are present on his knees and shins. No lesions are apparent on his face, torso, or flexor surfaces. No swelling beneath the lesions is apparent. No pustules are apparent within the lesions, and there is no drainage from them. When a piece of scaled lesion is removed with a pair of forceps, the tissue underneath bleeds slowly. Some mild pitting of his fingernails is detected. Neurovascular examination detects normal motor and sensory function and strong distal pulses. The following vital signs are measured:
T: 98.6°F, HR: 72 bpm, BP: 125/82 mm Hg, RR: 16 breaths/min
Differential Diagnosis Psoriasis, pityriasis rosea, seborrheic dermatitis, erythema nodosum, systemic lupus erythematosus, cutaneous fungal infection
Laboratory Data and Other Study Results ESR: 5 mm/hr CRP: 0.2 mg/dL KOH preparation of lesions: no hyphae or pseudohyphae Skin biopsy: epidermal thickening and hyperproliferation of keratinocytes; hypervascularity of the dermis with an infiltration of lymphocytes and neutrophils
Diagnosis Psoriasis
Treatment Administered The patient was prescribed topical vitamin D and betamethasone creams to apply to the lesions The patient was encouraged to limit his alcohol intake
Follow-up After consistent use of the creams, the patient experienced a slow improvement in his lesions The patient was unable to achieve complete resolution of his lesions, but the improvement was significant enough that he elected to continue using the topical therapies and not add any further treatments
Steps to the Diagnosis Psoriasis An inflammatory skin disorder characterized by keratinocyte hyperproliferation and epidermal thickening History: pruritus of lesions Physical examination: red plaques with silvery scales on extensor surfaces, lesions bleed easily with scale removal (i.e., Auspitz sign), possible small pustules, pitting of the nails (Color Figure 10-13) Tests: typically a clinical diagnosis; skin biopsy will show a hyperproliferation of epidermal keratinocytes, thickening of the stratum corneum in the epidermis, nucleated cells and neutrophils in the stratum corneum, hypervascularity of the dermis, and an infiltration of lymphocytes and neutrophils Treatment: Topical corticosteroids, vitamin D, tar, retinoids, or antifungal agents are the first-line therapies Phototherapy combined with a topical agent is used in cases not responsive to topical therapy alone Methotrexate, cyclosporine, or anti–tumor necrosis factor drugs may be used in severe cases Outcomes: the response of cases to treatment is variable, but the condition is not associated with any
mortality Clues to the diagnosis: History: slow-developing lesions, pruritus, bleeding when scratching lesions Physical: red plaques with scaling, Auspitz sign, lesions on extensor surfaces only Tests: skin biopsy results
Twenty percent of patients with psoriasis also have psoriatic arthritis. Pityriasis rosea A mild inflammatory skin disorder in children and young adults characterized by papular lesions on the trunk and extremities Probably related to a viral cause History: pruritic lesions, appearance of a “herald patch” (i.e., single round lesion) preceding a widespread eruption of similar lesions Physical examination: papular rash with white scales on the torso and extremities distributed in a “Christmas tree” pattern (Color Figure 10-14) Tests: skin biopsy will show epidermal hyperplasia and perivascular inflammation Treatment: typically a self-limited condition; topical corticosteroids or erythromycin or phototherapy may help decrease the duration of exacerbations Outcomes: the prognosis is excellent although pigmentation changes may occur in some patients Why eliminated from differential: the distribution pattern of the rash and the biopsy appearance make this diagnosis unlikely
Color Figure 10-13 Red plaques with silver scales on the extensor forearm surface of a patient with psoriasis; similar lesions may also be seen on the extensor surfaces of the knee.
Color Figure 10-14 Pityriasis rosea. These scaled papules fan out across the chest or back to give the overall appearance of a Christmas tree pattern.
Seborrheic dermatitis More thorough discussion in prior case Why eliminated from differential: the location of the lesions and appearance of the scales make this diagnosis unlikely Erythema nodosum Inflammation of subcutaneous fat septa, resulting in painful erythematous nodules Classically seen on the anterior shins Occurs due to a delayed immunologic reaction to an infection, collagen-vascular disease, inflammatory bowel disease, or medication History: malaise, arthralgias Physical examination: tender pretibial erythematous nodules, fever, possible lymphadenopathy Tests:
Conditions associated with erythema nodosum may be remembered by the mnemonic SPUD BITS: Streptococcal infection, Pregnancy, Unknown (idiopathic), Drugs, Bechet
disease (autoimmune vasculitis), Inflammatory bowel disease, TB, Sarcoidosis. Increased ESR Possible positive antistreptolysin O titer (when due to infection) Fatty inflammation is seen on a skin biopsy Treatment: typically self-limited; nonsteroidal anti-inflammatory drugs (NSAIDs), potassium iodide, or corticosteroids may be used to decrease pain Outcomes: the prognosis is excellent, but the underlying cause should be identified Why eliminated from differential: the absence of nodules and the normal ESR rule out this diagnosis Systemic lupus erythematosus More thorough discussion in Chapter 9 Why eliminated from differential: the absence of any other significant symptoms makes this diagnosis unlikely Cutaneous fungal infection More thorough discussion in prior case Why eliminated from differential: the negative KOH preparation makes this diagnosis unlikely
Case 10-6: “The mole on my back is growing” A 36-year-old man presents to a dermatologist for assessment of a lesion on his back. The patient says that his girlfriend felt that the lesion looked like it had grown in size compared to a photograph from a few years ago. The patient says that he has several moles on his body and that he was aware of this mole having been present for quite some time. Because it is in a location that is difficult for him to see, he was unaware of any changes in appearance. He works as a professional lifeguard and has had considerable sun exposure through work. He notes that he has had more than ten blistering sunburns in the past 20 years. He denies any back pain or pruritus. He denies any bleeding or drainage coming from the area of this lesion. He denies recent fatigue or weight loss. He notes a history of a squamous cell carcinoma on his right arm that was excised 4 years ago. He has no other medical problems and takes no medications. He infrequently uses marijuana but denies any other substance use. On examination, he is a well-appearing man in no distress. He is notable for a large number of small circular nevi throughout his body. Examination of the lesion in question reveals a dark asymmetrical lesion with irregular borders but measuring 15 mm at its longest diameter. There is no drainage from the lesion or any apparent skin opening. There is no palpable lymphadenopathy. No other skin lesions share these characteristics. The following vital signs are measured: T: 98.7°F, HR: 80 bpm, BP: 1150/80 mm Hg, RR: 14 breaths/min
Differential Diagnosis Melanoma, melanocytic nevus, squamous cell carcinoma, basal cell carcinoma, actinic keratosis
Laboratory Data and Other Study Results Excisional skin biopsy: high number of large, pleomorphic melanocytes; penetration of the lesion beneath the level of the dermis; 6-mm lesion thickness Following receipt of these results, the following tests were performed: Brain MRI: no focal lesions; normal size of ventricles; normal cerebral volume Bone scan: increased signal in the paravertebral region of the thoracic spine
Chest/abdomen/pelvis MRI: no bony, pulmonary, or visceral lesions; likely lymphadenopathy of the thoracic paravertebral modes
Diagnosis Malignant melanoma
Treatment Administered A lymph node biopsy was performed on the sentinel paravertebral node and indicated the presence of malignant cells; regional lymph node dissection was performed at that time A revised excision of the lesion tumor bed was performed with a 2-cm margin created around the site of the lesion The patient was placed on adjuvant α-interferon
Follow-up The patient was able to tolerate his immunotherapy but required a skin graft to help heal his tumor excision site Three years after his initial diagnosis the patient developed a recurrence of malignant melanoma near the site of his original lesion; a metastatic work-up detected the presence of brain metastases Despite a repeat excision and the use of an experimental melanoma vaccine, the patient passed away 3 months after the discovery of the recurrent lesion
Steps to the Diagnosis Melanoma A malignant melanocyte tumor with rapid spread Types: Superficial spreading: most common type; grows laterally before invasive growth occurs Nodular: primarily grows vertically and become invasive rapidly Acral lentiginous: involves the palms, soles, and nail beds Lentigo maligna: long-lasting in situ stage prior to vertical growth Risk factors: sun exposure, fair complexion, family history, numerous moles History: painless pigmented lesion with a recent change in appearance Physical examination: an asymmetric pigmented lesion with irregular borders, variable coloring, and a diameter frequently >6 mm (Color Figure 10-15) Tests: Excisional biopsy is the primary means of diagnosis and will show large atypical melanocytes with possible invasion into the dermis A metastatic work-up may include MRI of the brain, MRI of any regional lymph nodes, bone scintigraphy, and positron emission tomography (PET) scan Sentinel node biopsy may be performed for any concerning imaging findings or for an invasive tumor Treatment:
Surgical excision of the lesion (0.5 cm margin if in situ, 1 cm margin if less than 2 mm thick, 2 cm margin if more than 2 mm thick) Lymph node dissection is performed for disease with sentinel node involvement Adjuvant α-interferon or melanoma vaccines may be used for metastatic disease Outcomes: the 5-year survival rates are >90% for in situ and small lesions, 45%–80% for lesions deeper than 2 mm, 25%–65% for tumors with some degree of nodal involvement, and <20% for metastatic cases Clues to the diagnosis: History: significant sun exposure, known lesion with a changing appearance Physical: large, asymmetric lesion with irregular borders Tests: biopsy results
Periodic skin checks should be performed in anyone with a history of significant sun exposure and a positive family history for melanoma.
Nevi should be followed to look for the ABCDEs of melanoma—Asymmetry, Border (irregular), Color (variable), Diameter (>6 mm), and Enlargement.
The most important prognostic factor for melanoma is thickness of lesion (>0.76 mm associated with increased risk of metastasis). Melanocytic nevi Common benign lesions composed of a proliferation of melanocytes History: painless dark skin lesions Physical examination: small (some congenital nevi can be very large) and dark skin lesions with regular borders and even coloring (Color Figure 10-16)
Color Figure 10-15 Melanoma, superficial spreading type; note the ABCDs of the lesion—asymmetry, irregular border, inconsistent color, and large diameter (>20 mm).
Color Figure 10-16 Melanocytic nevus; unlike melanoma, this lesion is near symmetrical, has better border regularity, is a more consistent color, and is a smaller diameter.
Tests: further testing is unwarranted in the absence of a concern for malignancy; skin biopsy will show contained collections of normal-appearing melanocytes along the dermoepidermal border Treatment: Usually none is needed Patients who are concerned about the cosmetic appearance of some lesions may have them electively excised Patients with a high number of nevi should be cautioned about the importance of using sunscreen because of the increased risk of melanoma Any concerning nevi (i.e., atypical-appearing) should be observed for changes over time Outcomes: the prognosis is excellent will little morbidity; patients with a high number of nevi have an
increased risk of developing melanoma Why eliminated from differential: the appearance of the lesion in this case is consistent with a malignant lesion, and these suspicions are confirmed by the biopsy Squamous cell carcinoma A skin cancer involving the squamous cells of the epithelium Risk factors: sun exposure (particularly UVB radiation), prior actinic keratosis, arsenic exposure, fair complexion, radiation exposure History: painless skin lesion in a sun-exposed area Physical examination: erythematous papule with scales or keratinized thickening, possible ulceration, or bleeding with progressive lesions (Color Figure 10-17)
Color Figure 10-17 Squamous cell carcinoma with erythematous base and ulceration.
Color Figure 10-18 Basal cell carcinoma; note the pearly appearance of a papule with central ulceration.
Tests: biopsy will show anaplastic epidermal cells extending down to the dermis Treatment: Surgical excision is typically curative Mohs excision (i.e., serial shallow excisions with intermittent histological analyses that are performed to minimize the cosmetic damage) may be performed for facial lesions Radiation may play a role in decreasing the size of large lesions Outcomes: the 3-year survival ranges from 70% for large invasive or recurrent lesions to near 100% for small lesions Why eliminated from differential: the biopsy results confirm that the tumor in this case is of a melanocytic origin
Use of a good sunscreen (SPF 15 or greater) is important in the prevention of skin cancer associated with sun exposure. Basal cell carcinoma A skin cancer arising in the basal epidermal cells Risk factors: sun exposure History: painless papule in a sun-exposed area Physical examination: pearly papule with fine telangiectasias (Color Figure 10-18) Tests: skin biopsy shows basophilic-staining basal epidermal cells arranged in palisades Treatment: surgical excision, Mohs excision, radiation therapy, or cryotherapy may be performed to remove the tumor Outcomes: excision is curative, but recurrences may recur with partial excisions Why eliminated from differential: the biopsy results confirm that the tumor in this case is of a melanocytic origin
Basal cell carcinoma is the most common type of skin cancer.
Shave biopsy should never be used to study a suspicious lesion because it does not provide enough tissue for clear diagnosis and cannot be used to measure lesion depth. Actinic keratosis A precancerous skin lesion that is at risk for progressing to squamous cell carcinoma Risk factors: sun exposure
Color Figure 10-19 Actinic keratosis; these lesions are superficial papules covered by dry scales and are a result of sun exposure.
History: painless scaly papule in a sun-exposed area Physical examination: erythematous papule with rough, yellow-brown scales (Color Figure 10-19) Tests: skin biopsy will show dysplastic keratinocytes without invasive potential Treatment: topical 5-flurouracil or imiquimod or cryotherapy are used to remove the lesions Outcomes: the prognosis is good, but there is a 0.1% per year risk of progression to squamous cell carcinoma Why eliminated from differential: the biopsy in this case is indicative of a malignant melanocytic tumor
Sixty percent of squamous cell carcinoma arise from actinic keratoses.
Authors: Van Kleunen, Jonathan P. Title: Step-Up to USMLE Step 3, 1st Edition Copyright ©2009 Lippincott Williams & Wilkins > Table of Contents > Chapter 11 - Gynecology
Chapter 11 Gynecology Basic clinical primer Menstrual Physiology Gynecologic development Follicle-stimulating hormone (FSH), luteinizing hormone (LH), and estrogens are the key hormones that drive reproductive development (Figure 11-1, Table 11-1) Androgens also play a role in the development of secondary sexual characteristics The Tanner stages are used to describe the development of the genitalia, breasts, and pubic hair growth in growing children (Table 11-2) Normal menstrual cycle The primary hormones involved in the menstrual cycle are LH, FSH, estrogen, progesterone, and human chorionic gonadotropin (hCG) (Figure 11-2, Table 11-3) Follicular phase Begins at the first day of menstruation (i.e., menses) FSH stimulates the growth of the ovarian follicle Granulosa cells of the ovarian follicle secrete estradiol Estradiol induces endometrial proliferation and further increases FSH and LH secretion due to positive feedback of the pituitary Luteal phase A sudden rise in LH (i.e., LH surge) induces ovulation The residual follicle (i.e., corpus luteum) secretes estradiol and progesterone to maintain the endometrium and to induce the development of secretory ducts High estradiol levels inhibit FSH and LH secretion If the ovum is not fertilized, the corpus luteum degrades, progesterone and estradiol levels decrease, and the endometrial lining degrades (i.e., menses) Fertilization If the ovum is fertilized, it will implant in the endometrium The endometrial tissue secretes hCG to maintain the corpus luteum
The corpus luteum continues to secrete progesterone until a sufficient production is achieved by the developing placenta (approximately 8 to 12 weeks)
The mean age of menarche is 13 years old in the United States and tends to occur earlier in African Americans than in whites.
Contraception Behaviors, medicines, instruments, or procedures designed to prevent pregnancy from occurring (Table 11-4) Method activity Methods may alter menstrual physiology (e.g., preventing ovulation), prevent fertilization (e.g., barrier contraception), or prevent implantation and development of the fertilized ovum (e.g., some intrauterine devices)
The rate of pregnancy in 1 year without any contraception is 85%. P.254
Figure 11-1 Changes in hormone and oogonia (egg) levels with gestation and age. DHEA, dehydroepiandrosterone; FSH, follicle-stimulating hormone; hCG, human chorionic gonadotropin; LH, luteinizing hormone. (See color image.)
Method selection The physician and patient must consider the likelihood of compliance; patients who are more at risk for noncompliance should use a method requiring less responsibility (e.g, progestin implants or injections) The patient must be able to tolerate certain side effects (e.g., possible nausea or headaches with OCPs) Some medical conditions may be contraindications for certain methods (e.g., DVT and OCP use) Some sexual behaviors or the risk of transmission of sexually transmitted diseases may affect selection (e.g., barrier methods to reduce STD risk)
Barrier methods (not spermicide alone) are the only contraception shown to reduce the spread of sexually transmitted diseases.
Table 11-1 Gynecologic Development by Age
Age
Fetal–4 yr old
Hormone Levels
Charac teristic s
High intrauterine FSH and LH that peak at 20 weeks gestation and
All oocytes are formed and partially matured by 20 weeks
decrease until birth
Tanner stage 1 characteristics
FSH and LH increase again from birth until 6 months of age and then gradually decrease to low levels by 4 years old
4–8 yr old
Low FSH, LH, and androgen levels due to GnRH suppression
Tanner stage 1 characteristics Any sexual development is considered precocious
8–11 yr old
LH, FSH, and androgen levels begin to increase
Initial pubertal changes including early breast development and pubic and axillary hair growth
11–17 yr old
Further increase of LH, FSH, and androgens to mature levels
Puberty
Hormones secreted in a pulsatile fashion (higher at night) due to a
Progression through Tanner stages
sleep-associated increase in GnRH secretion
Development of secondary sexual characteristics and growth spurt Menarche in females (beginning of menstrual cycles) and further oocyte maturation
17–50 yr old (females)
≥ 50 yr old (females)
LH and FSH levels vary during the menstrual cycle
Menstrual cycles
Gradual increase in FSH and LH as ovarian insensitivity increases
Mature sexual characteristics
LH and FSH levels increase with the onset of ovarian failure
Perimenopause: menstrual cycles become inconsistent (oligomenorrhea) Menopause: menstrual cycles cease (amenorrhea)
FSH, follicle stimulating hormone; GnRH, gonadotropin-releasing hormone; LH, luteinizing hormone.
P.255
Table 11-2 Tanner Stages for Sexual Characteristic Development
Female
Male
Tanner Stage
Penile/Testic ular Breast Development
Pubic Hair Development
Development
Pubic Hair Development
1
Prepubertal; raised papilla (nipple) only
Prepubertal; no hair growth
Prepubertal; small genitals
Prepubertal; no hair growth
2
Breast budding, areolar enlargement
Slight growth of fine labial
Testicular and scrotal
Slight growth of fine genital
hair
enlargement with skin
and axillary hair
coarsening
3
Further breast and areolar enlargement
Further growth of hair
Penile enlargement and
Further growth of hair
further testicular growth
4
5
Further breast enlargement—areola and
Hair becomes coarser and
Further penile glans
Hair becomes coarser and
papilla form a secondary growth above
spreads over much of the
enlargement and darkening
spreads over much of the
the level of the breast
pubic region
of scrotal skin
pubic region
Mature breast—areola recedes to the
Coarse hair extends from
Adult genitalia
Coarse hair extends from
level of the breast while the papilla
the pubic region to the
the pubic region to the
remains extended
medial thighs
medial thighs
Case 11-1: “My daughter is growing up too fast” A 6-year-old girl is brought to her pediatrician by her mother who is concerned about the patient's early breast development. It has been almost 1 year since the patient's previous pediatric check-up, and the patient was in good health at that time. The patient's mother says that she first noticed breast budding in her daughter 8 months ago while clothes shopping. She assumed it was related to recent rapid growth and did nothing at the time. She P.256 says that since that time her daughter's breasts have continued to develop and enlarge. There has been no point at which her breasts have decreased in size. The mother denies the presence of any nipple discharge or breast erythema. The mother also says that she began to notice the growth of pubic hair in her daughter after a bath 2 months ago. This hair growth has not receded since the discovery. The patient's mother denies any vaginal bleeding in her daughter. The patient denies any breast or genital pain or headaches. The mother says that she chose to bring her daughter to the pediatrician at this time because the breast and pubic hair growth were continuing to evolve and not reverse. She says that her daughter is a happy child and is doing well in her first grade class. She is active and likes to draw pictures. Her mother denies any physical limitations. She denies any prior medical problems, and her daughter only takes children's vitamins. The patient was the product of a full-term uncomplicated pregnancy and vaginal delivery. The mother denies any medical conditions during her pregnancy and is only aware of hypertension (HTN), hypercholesterolemia, and ischemic heart disease in the family history. Neither parent smokes or uses any illicit substances. On examination, the patient is a happy appearing child in no distress. She is able to participate well in the examination. She appears tall for her age. She has minimal P.257 P.258 acne on her face but not on her trunk. Her ears appear clear. Examination of her eyes finds them equally reactive with no asymmetry. Her oropharynx appears normal. She has no lymphadenopathy. Auscultation of her lungs and heart detects clear and equal breath sounds, a normal heart rate, and no extra heart sounds. Her abdomen is nontender with no masses. Her extremities are symmetric, and her motor and sensory function is normal. She has Tanner stage 3 breast development and Tanner stage 2 pubic hair development. She does not appear to have any signs of vaginal bleeding or clitoromegaly. The following vital signs are measured: Temperature (T): 98.5○F, heart rate (HR): 85 beats per minute (bpm), blood pressure (BP): 112/78 mm Hg, respiratory rate (RR): 18breaths/min
Table 11-3 Roles of Hormones Involved in the Menstrual Cycle
Hormone
Effec ts
Luteinizing hormone (LH)
Midcycle surge induces ovulation Regulates cholesterol conversion to pregnenolone in ovarian theca cells as the initial step in estrogen synthesis
Follicle-stimulating hormone (FSH)
Stimulates the development of the ovarian follic le Regulates ovarian granulosa cell activity to control estrogen synthesis
Estrogens (estradiol, estriol)
Stimulates endometrial proliferation Aids in follicle growth Induces the LH surge High levels inhibit FSH secretion A dec rease in levels leads to menstruation Principal role in sexual development
Progesterone
Stimulates endometrial gland development Inhibits uterine contraction Increases the thickness of cervical mucus Inc reases basal body temperature Inhibits LH and FSH secretion and maintains pregnancy A dec rease in levels leads to menstruation
Human chorionic gonadotropin (hCG)
Acts like LH after the implantation of the fertilized egg Maintains c orpus luteum viability and progesterone secretion
Figure 11-2 Hormone levels during the menstrual cycle with appropriate ovarian, endometrial, and basal body temperature responses. FSH, follicle-stimulating hormone; LH, luteinizing hormone. (See color image.)
Table 11-4 Methods of Contraception
Method
Desc ription
Effec tiveness1
Side Effec ts
Ideal
Typic al
(%)
(%)
HORMONAL METHODS
Oral contraceptive pills (combined formulation)
Estrogen-progestin combination pills that
99
92
Possible nausea, headache, weight
inhibit follicle development and ovulation,
gain
change endometrial quality, and increase
Increased risk of DVT
cervical mucus viscosity to prevent
Contraindicated for heavy smokers,
fertilization and implantation
or women with a history of DVT, estrogen-related cancer, liver disease, or hypertriglyceridemia
Oral contraceptive pills (progestin formulation)
Medroxyprogesterone acetate (e.g., Depo-Provera)
Progestin implant
Progestin-only pills that change endometrial
98
92
Increased breakthrough bleeding
quality and increase cervical mucus viscosity to
Less nausea and vomiting than
prevent fertilization and implantation
combination pills
May be option for women with contraindication
Must be taken at same time every
for estrogen
day to maximize efficacy
Progestin analogue injected by health care
99
97
provider every 3 months that inhibits ovulation
Nausea, headache, weight gain Irregular bleeding
and endometrial development
Subcutaneous implant that slowly releases
1002
1002
99
92
Irregular bleeding, breast pain
progestin over approximately 3 years (similar activity to progestin-only pill)
Transdermal contraceptive patch
Transdermal delivery of estradiol and progestin
Risk of patch detachment
analogue to act in similar manner to OCPs
Nausea, headache, weight gain
Patch must be changed weekly
Irregular bleeding, breast pain Less effective in heavier women because of diffusion into adipose tissue Increased risk of DVT
Intravaginal ring
Ring inserted intravaginally that releases
99
92
Withdrawal bleeding, device-related
ethinyl estradiol over 3 weeks to prevent
discomfort, headache
ovulation (less estrogen than OCP)
Increased risk of DVT
Replaced each month
Emergency contraception
Regimen of estradiol and progestin or progestin
—3
—3
Nausea, headache more severe than
alone taken within 72 hours of unprotected
that seen with OCPs
intercourse or intercourse with failed
Menstrual bleeding expected within 1
contraception method (e.g., poor withdrawal,
week of administration
broken condom) to prevent ovulation, inhibit fertilization, or interrupt new pregnancy
BARRIER METHODS
Condom
Barrier (most frequently latex) placed over
98
85
Risk of condom breakage
penis and left in place until withdrawal following
Latex significantly more effective
ejaculation
than other materials (with possible
Frequently used with spermicide
exception of polyurethane)
Polyurethane condoms are produced for those
Risk of latex allergy
with latex allergy
Diaphragm/cervical cap
Barrier inserted into vagina before intercourse
94
84
Inconvenient
to cover cervix
Frequent poor compliance
Used with spermicide and left in place for
Increased risk of UTI
several hours after intercourse
Contraceptive sponge
Polyurethane sponge implanted with spermicide
914
804
that releases spermicide over 24 hours after
Possible increased risk of toxic shock syndrome
insertion to inhibit fertilization
Spermicide alone
Insertion of spermicidal jelly or cream into
82
71
vagina immediately before intercourse
Correct usage and quantity is difficult to achieve consistently
SEX UAL PRACTICE METHODS
Abstinence
Rhythm method
Not engaging in intercourse
Recording the occurrence of menses, daily
100
100
95
83
96
73
None
May be useful in diagnosing infertility
basal body temperature, and cervical mucus viscosity to determine the timing of the cycle, the occurrence of ovulation, and the period of fertility
Withdrawal method
Withdrawal of penis from vagina immediately before ejaculation
Decreased pleasure Difficult to conduct in an effective manner
Lactation
Unprotected intercourse during the active
98
95
postpartum lactation period
Only able to be performed if actively breast feeding, <6 months postpartum, and amenorrheic (pregnancy rate equals no contraception rate otherwise)
INTRAUTERINE DEV ICES
Copper intrauterine device
Object inserted into the uterus by a physician
99
99
Small risk of spontaneous abortion
with a slow release of copper to prevent
and uterine perforation
fertilization and interfere with sperm
Menorrhagia
transportation Left in place approximately 10 years May be placed soon after intercourse as emergency contraception (90% decrease in pregnancy rate)
Progestin-releasing IUD
Object inserted into the uterus by a physician
99
99
with a slow release of progestin to prevent
Small risk of spontaneous abortion and uterine perforation
fertilization, interfere with sperm transportation, and inhibit ovulation Left in place approximately 5 years
SURGICA L METHODS
Sterilization
Cutting of the vas deferens in men (vasectomy) or tubal ligation in women to prevent
~100
~100
May be difficult to reverse Increased risk of ectopic pregnancy
fertilization
in cases of failure or after voluntary ligation reversal
DVT, deep venous thrombosis; HTN, hypertension; IUD, intrauterine device; OCPs, oral contraceptive pills; UTI, urinary tract infection. 1
As defined by pregnancy rate with 1 year of use (6 months for lactation).
2
Only one efficacy study performed; no pregnancies occurred in study.
3
Rate of pregnancy decreased 75%–90% with timely use.
4
Effectiveness decreases to 80% and 60%, respectively, in women with prior vaginal birth history.
P.259
Differential Diagnosis Central precocious puberty, ovarian tumor, congenital adrenal hyperplasia, McCune-Albright syndrome, hypothyroidism
Laboratory Data and Other Study Results Height and weight: 54"(>97th percentile), 60 lb (94th percentile) Skeletal survey x-ray: ossification of multiple cartilage growth centers prior to what is expected for the patient's age; no closure of growth plates; no fractures of any age; no skeletal lesions FSH: 8.1 U/L LH: 7 U/L Gonadotropin-releasing hormone (GnRH) stimulation test: FSH: 32 U/L, LH: 73 U/L Estradiol: 23 pg/mL Cortisol (afternoon): 7 µg/dL Dehydroepiandrosterone (DHEA): 2.7 ng/mL Thyroid-stimulating hormone (TSH): 1.2 µU/mL After the review of these test results, the additional following studies are ordered: Abdominal ultrasound (US): no renal, adrenal, or ovarian mass Brain magnetic resonance imaging (MRI): normal ventricular size; normal cerebral volume for age; no areas of ischemia or cerebral masses
Diagnosis Idiopathic central precocious puberty
Treatment Administered The patient was referred to a pediatric endocrinologist to direct further care The patient was placed on histrelin at monthly intervals to decrease her serum FSH and LH levels
Follow-up By 2 months after starting histrelin therapy, the patient's FSH and LH had returned to prepubertal levels
The patient was maintained on monthly histrelin therapy until she was 11 years old, at which time therapy was stopped, and she was allowed to progress through normal puberty
Steps to the Diagnosis Central precocious puberty Premature development of the sex-appropriate pubertal process in boys or girls (Table 11-5) Due to the premature activation of the hypothalamic-pituitary-gonadal axis The majority of cases are due to idiopathic causes History: premature development of sexual characteristics in prepubertal-aged children Physical examination: premature breast development, genital enlargement, pubic hair growth, possible mild acne Tests: FSH and LH will be elevated from age-appropriate levels; administration of GnRH (i.e., GnRH stimulation test) will cause an additional increase in FSH and LH levels P.260 Low FSH and LH or a poor response to GnRH are more consistent with a direct overproduction of androgens or estrogens (e.g., ovarian neoplasm, congenital adrenal hyperplasia) An abdominal US may be used to rule out any adrenal or ovarian tumors Brain MRI should be performed to determine if a cerebral mass is the cause of the activated hypothalamicpituitary-gonadal axis Treatment: GnRH agonists (e.g., leuprolide acetate) or LH-releasing hormone agonists (e.g., histrelin) are used to decrease FSH and LH levels and prevent further sexual maturation FSH and LH inhibition is stopped at the age of normal pubertal onset to allow sexual maturation to continue Children near the age of the appropriate onset of puberty are frequently allowed to progress through puberty at the slightly early age Outcomes: Children undergoing puberty inhibition develop normal sexual characteristics once inhibition is stopped Short stature may result if bone maturity has advanced significantly by the time treatment has been initiated Parents and physicians should be aware of the social and emotional adjustment issues in school-aged children that begin premature sexual maturation Clues to the diagnosis: History: young age, observation of both breast and pubic hair growth, period of rapid growth Physical: Tanner stage 3 breast and Tanner stage 2 pubic hair growth in a 6-year-old child, minimal acne Tests: increased FSH and LH, positive GnRH stimulation test
Precocious puberty may be defined by breast or genital growth and pubic hair
growth before the age of 8 years in girls and 9 years in boys.
Adrenal hyperplasia is the most common cause of precocious puberty in boys.
A central nervous system (CNS) lesion or trauma is the cause of central (isosexual) precocious puberty in approximately 10% of cases.
Table 11-5 Classification of Precocious Puberty
Heterosexual
Virilization of girls or femininization of boys
Isosexual
Gender-appropriate premature sexual development
In girls most commonly due to CAH, exogenous androgen exposure, or an androgen-secreting neoplasm Complete
Inc omplete
Normal pubertal changes occur at a
Isolated premature breast development
premature age for all sexual
(i.e., thelarche) or pubic hair growth (i.e.,
characteristics
pubarche)
CAH, congenital adrenal hyperplasia.
Always measure the TSH in patients with precocious puberty to rule out chronic hypothyroidism (TSH is increased). Ovarian tumor More thorough discussion in later case Why eliminated from differential: the negative abdominal US, absence of excessive elevation in the estradiol level, and elevated LH and FSH levels in the presence of a normal estradiol level combine to rule out this diagnosis Congenital adrenal hyperplasia More thorough discussion in Chapter 5 Why eliminated from differential: the normal cortisol and DHEA levels and the negative abdominal US make this diagnosis unlikely McCune-Albright syndrome A syndrome of precocious puberty, multiple sites of fibrous skeletal dysplasia, and multiple café-au-lait lesions of the skin Precocious puberty occurs as a result of excessive gonadal activity in the presence of normal hypothalamicpituitary function Secondary Cushing syndrome may result from an associated overproduction of cortisol P.261
History: premature breast development, premature menarche in the absence of pubic hair growth Physical examination: greater height than expected for age, premature labial and clitoral maturation, inconsistent development of pubic hair, multiple café-au-lait spots, possible gross deformity of extremities due to fibrous skeletal dysplasia Tests: Increased estradiol and testosterone; prepubertal FSH and LH levels Possible increased alanine aminotransferase (ALT), aspartate aminotransferase (AST), and bilirubin Cortisol is frequently elevated and does not decrease following the administration of either low-dose or high-dose dexamethasone Abdominal US or computed tomography (CT) will detect ovarian cysts and possible adrenal enlargement X-rays will detect multiple skeletal lesions consistent with fibrous dysplasia Treatment: Aromatase inhibitors (e.g., testolactone, fadrozole) or tamoxifen help to prevent the conversion of testosterone into estrogens Lesions of fibrous dysplasia are observed unless fracture occurs (necessitating fixation); bisphosphonates may help to heal lesions more quickly Adrenalectomy is considered in patients with Cushing syndrome due to adrenal enlargement Outcomes: complications include short stature due to premature growth plate closure, pathologic fractures, failure to thrive, acromegaly, and sudden death; the prognosis depends on the timeliness of the initiation of treatment Why eliminated from differential: the absence of café-au-lait spots and skeletal lesions and an abnormal cortisol level rule out this diagnosis Hypothyroidism More thorough discussion in Chapter 5 Why eliminated from differential: the normal TSH level rules out this diagnosis
Case 11-2: “I still haven't gotten my period” An 18-year-old female presents to a gynecologist because she has never undergone menses. She feels that she has never fully grown up. Her parents told her that she “must just be a late-bloomer.” She reports that she has never had a single menstrual period or ever developed breasts. She has grown pubic hair. She has had vaginal intercourse as an adolescent without any difficulties. She denies any difficulty eating, abnormal eating behaviors, or any heat or cold intolerance. She says that she looked similar to the other girls in her primary school grades as a younger child. Once she reached high school, she did not grow as much as the other girls and was the shortest girl in her grade. She says that she was a healthy child and is unaware of any medical history. She does not take any medications. Her family history is significant for breast and pancreatic cancer, HTN, coronary artery disease (CAD), hypercholesterolemia, and stroke. She smokes a quarter-pack of cigarettes each day and drinks alcohol on some weekends. On examination, she is a short, well-nourished female in no distress. She has a mild ptosis of both eyes. Gross and fundoscopic examination of her eyes detects normal vasculature and color. Her ear canals and oropharynx appear normal. She has no lymphadenopathy. Her neck is short and broad. Auscultation of her lungs and heart detects clear breath sounds, an extra heart sound between a normal S 1 and S2, and a faint systolic murmur. Her abdomen is soft and nontender with normal bowel sounds and no masses. Her extremities are symmetric although both of her elbows bow slightly inward.
She has Tanner stage 1 breast development and Tanner stage 5 pubic hair development. A manual and speculum vaginal examination detects a perforate narrow vaginal canal and a visible but small cervix without any discharge. She has no vaginal or cervical tenderness. A neurologic examination detects normal motor and sensory function. The following vital signs are measured: T: 98.4○F, HR: 90 bpm, BP: 132/83 mm Hg, RR: 18 breaths/min P.262
Differential Diagnosis Genital tract abnormality, anorexia nervosa, hypothyroidism, sex chromosome abnormality (e.g., Turner syndrome), polycystic ovarian syndrome, pregnancy, congenital adrenal hyperplasia, prolactinoma
Laboratory Data and Other Study Results Height and weight: 4﹐ 11", 128 lbs, BMI 25.9 Serum pregnancy test: negative TSH: 2.6 µU/mL FSH: 75 U/L LH: 81 U/L Estradiol: 2 pg/mL Cortisol (afternoon): 9 µg/dL DHEA: 3.5 ng/mL Prolactin: 3 ng/mL Progestin challenge: no vaginal bleeding in the week following progestin administration Following receipt of these test results, the following tests are ordered: Abdominal US: no renal, adrenal, or ovarian masses Karyotype: 45XO Y chromosome genetic probe: negative Echocardiogram: bicuspid aortic valve with mild stenosis and increased flow velocity; minimal concentric left ventricle hypertrophy; normal left atrial and right-heart chamber size and motion
Most pregnancies with a 45XO karyotype end in spontaneous abortion.
Diagnosis Turner syndrome
Treatment Administered The patient was started on estrogen and progesterone replacement therapy The patient was referred to a cardiologist to monitor her bicuspid aortic valve and the development of any related cardiac sequelae
Causes of amenorrhea may be remembered by the mnemonic NO CATCHUP: Nutrition (e.g., anorexia, malnutrition), Ovarian disease (e.g., failure, cysts), Cushing syndrome, Anatomic abnormalities, Thyroid disease, Chromosome abnormalities, Hypothalamic-pituitary dysfunction, Uterine disease, Pregnancy.
Follow-up The patient was able to achieve pharmacologic-driven menstrual cycles and developed breasts after the initiation of hormone replacement therapy
Causes of short stature may be remembered by the mnemonic ABCDEFGHIJKL: Abuse, Bad cancers, Chromosomal (Turner), Delayed growth (constitutional), Endocrine (growth hormone deficiency, hypothyroidism), Familial short stature, Gastrointestinal disease (irritable bowel syndrome, celiac), Heart (congenital disease), Immune disorders, Joint and bone dysplasias, Kidney failure, Lung disease (asthma, cystic fibrosis).
Steps to the Diagnosis Turner syndrome A sex chromosome abnormality in which patients carry only one full X chromosome in their genome (Table 11-6) The patient's karyotype is 45XO It is an uncommon cause of primary amenorrhea, short stature, and infertility and is typically detected during the work-up for one of these findings (Figure 11-3) History: primary amenorrhea, failure to have a pubertal growth spurt, infertility Physical examination: little or no breast development, normal pubic hair growth, short stature, possible nail deformities, ptosis, strabismus, low-set ears, broad neck, numerous nevi, cubitus valgus (i.e., lateral angulation of the forearms at the elbow), abnormal heart sounds, or lymphedema P.263
Table 11-6 The Most-Common Sex Chromosome Disorders
Condition
Karyoty pe
H/P
Turner
45XO or
Female with short stature, infertility , and primary amenorrhea; increased incidence of renal and cardiac
syndrome
mosaicism
defects, craniofacial abnormalities, and lymphedema
Klinefelter
47XXY
Male with testic ular atrophy , tall and thin body, gynecomastia, infertility, mild mental retardation, and
syndrome
psychosocial adjustment abnormalities
XYY
47XYY
Male with tall body, significant acne, and mild mental retardation
XXX
47XXX
Female with increased incidence of mental retardation and menstrual abnormalities
H/P, history and physical.
Tests: Karyotyping is the definitive test for diagnosing the condition LH and FSH will be elevated to menopausal levels; estradiol will be at prepubertal levels An echocardiogram and abdominal US should be performed to detect renal and cardiac abnormalities
Figure 11-3 Approach to the patient with amenorrhea. GnRH, gonadotropin-releasing hormone; FSH, follicle-stimulating hormone; β-hCG, human chorionic gonadotropin; LH, luteinizing hormone. Neg, negative; PCOS, polycystic ovary syndrome; Pos, positive.
P.264 Treatment: Estrogen and progesterone replacement therapy is indicated to achieve menses and breast growth Donor ovum implantation and fertilization is required for pregnancy Good cardiac and renal follow-up is required to detect any evolving complications for those systems Outcomes: The prognosis is generally good with adequate hormone replacement therapy until the age of menopause Permanent short stature is typical unless the diagnosis is made at a young age Infertility is almost universal Cardiac anomalies (e.g., coarctation of the aorta, aortic valve abnormalities), renal abnormalities (e.g., collecting duct anomalies), hearing loss, osteoporosis, hypothyroidism, and lymphedema are possible
comorbid conditions and complications Clues to the diagnosis: History: primary amenorrhea, absence of growth spurt Physical: short stature, no breast development, ptosis, broad neck, extra heart sounds, normal pubic hair growth Tests: 45XO karyotype, elevated LH and FSH, low estradiol
Patients with Turner syndrome taking hormone replacement therapy are not at a significantly increased risk of breast cancer or cardiovascular disease because of their pre-existing hormone deficiencies. Genital tract abnormality Abnormal development of the genital tract, resulting in the inability to have normal menses Variations include vaginal septa, imperforate hymen, uterine agenesis, or intrauterine adhesions Androgen insensitivity is a rare condition in 46XY individuals who fail to develop a male genital tract due to nonfunctional hormone signaling pathways History: amenorrhea, periodic pelvic pain Physical examination: possible visualization or detection of the anatomic abnormality during manual and speculum pelvic examination Tests: pelvic US is the best means of detecting an abnormality Treatment: surgical treatment is required to recreate the normal uterine-vaginal tract in cases amenable to reconstruction; hormone replacement therapy is administered in cases with inadequate hormone production or in androgen insensitivity syndrome Outcomes: the prognosis is contingent on the ease of treating the condition; cases with vaginal septa or an imperforate hymen typically regain the greatest degree of normal genital tract function Why eliminated from differential: the history of successful attempts at vaginal intercourse, the normal pelvic examination, and the normal abdominal US make this diagnosis unlikely Anorexia nervosa More thorough discussion in Chapter 13 Why eliminated from differential: the patient's weight (actually mildly overweight for her height) rules out this diagnosis Hypothyroidism More thorough discussion in Chapter 5 Why eliminated from differential: the normal TSH rules out this diagnosis Polycystic ovary syndrome (PCOS) A disease of the hypothalamic-pituitary axis with androgen and LH overproduction that leads to amenorrhea, infertility, and virilization Excess LH secretion by the pituitary causes an overproduction of androgens by the ovaries
The excessive androgens are converted into estrogens that further induce androgen production and lead to virilization The high estrogen levels cause an inhibition of FSH secretion and result in infertility and amenorrhea Some cases are associated with hyperinsulinemia, which causes further androgen overproduction P.265 History: amenorrhea or oligomenorrhea, breakthrough bleeding, infertility Physical examination: obesity, hirsutism (e.g., excess facial, chest, and abdominal hair), virilization (e.g., pattern baldness, clitoral enlargement, voice deepening), palpable bilateral ovarian enlargement Tests: Increased LH; LH/FSH ratio >3 Increased DHEA and androstenedione Positive progestin challenge (i.e., administration of progestin causes uterine bleeding within 5 days of administration) Abdominal US will show enlarged ovaries with multiple cysts Treatment: Clomiphene may be used to induce follicle maturation to allow pregnancy Estrogen-progesterone oral contraceptives may be used to regulate the menstrual cycles and decrease the androgen unbalance to lessen hirsutism and virilization Metformin and rosiglitazone may be used to control glucose intolerance and assist in inducing ovulation Outcomes: glucose intolerance with subsequent diabetes mellitus (DM) and an increased risk of endometrial cancer from chronic hormone stimulation are the main complications Why eliminated from differential: the elevation of both FSH and LH, the normal DHEA level, and the normal US rule out this diagnosis
PCOS is the most common cause of androgen excess in women.
Ovarian cysts are not the cause of disease in PCOS but are a result of androgen hypersecretion. Pregnancy More thorough discussion in Chapter 12 Why eliminated from differential: the negative pregnancy test rules out this diagnosis Congenital adrenal hyperplasia More thorough discussion in Chapter 5 Why eliminated from differential: the normal cortisol and DHEA levels make this diagnosis unlikely
A β-hCG pregnancy test is always the first step in the work-up of any type of amenorrhea.
Prolactinoma A pituitary tumor that produces excess prolactin and is related to the occurrence of amenorrhea Effects of the tumor are a result of both the overproduction of prolactin and the mass effect of the tumor History: amenorrhea or oligomenorrhea, headache, vaginal dryness, dyspareunia (i.e., painful intercourse) Physical examination: galactorrhea (i.e., ability to express milk from the breast) in the absence of pregnancy, possible vision abnormalities Tests: increased serum prolactin level; brain MRI may be performed to detect the pituitary tumor Treatment: bromocriptine is used to decrease prolactin secretion; surgical resection is indicated for large tumors and in women desiring pregnancy Outcomes: Microscopic tumors have an excellent prognosis, but larger tumors have more complications Large tumors can cause permanent vision deficits due to impingement upon the optic nerve Surgical resection is associated with possible hypopituitarism and a small risk of death Why eliminated from differential: the normal prolactin level and absence of breast tissue makes this diagnosis unlikely
Although a prolactinoma is the most common pituitary tumor, any hypothalamic or pituitary tumors can disturb the normal hypothalamic-pituitary-gonadal signaling pathway and cause abnormal menstrual function.
Ischemic injury of the hypothalamus or pituitary (e.g., Sheehan syndrome) will also present as amenorrhea.
Case 11-3: “My periods are disappearing” A 49-year-old woman presents to her gynecologist and reports that she has had few menses in the past year. She has not had menses in consecutive months within the past P.266 year, and she has not had a menstrual period at all in the past 6 months. Two of her menstrual periods in the past year have been heavier than normal. She denies any breakthrough bleeding between menstrual periods. She denies any abdominal pain and has had mild pelvic cramping during menses that has been similar to that experienced in the past during her menstrual periods. Sexual intercourse has been painful at times because she has felt “dry,” and she has lost interest in sex recently. She says that she has felt worn down and has been irritable at work. She feels very warm at times and occasionally breaks out into a sweat when she feels too warm. She denies pain during urination, urinary frequency, or vaginal discharge. She reports having a good appetite and denies any recent significant weight loss. She has a past medical history significant for three full-term vaginal child births, HTN, and controlled atrial flutter. She takes atenolol and lisinopril. She drinks five alcohol drinks per week and denies any other substance use. On examination, she is a well-appearing woman in no distress. Her face appears symmetric, and her eyes appear grossly normal. She has no lymphadenopathy or neck masses. Auscultation of her lungs and heart detects normal breath sounds and heart sounds. Her abdomen is soft and nontender. A neurovascular examination is normal. A breast examination detects mild diffuse bilateral breast tenderness. A pelvic examination finds a dry and minimally irritated vaginal wall, no cervical or adnexal tenderness, and no cervical discharge. No masses on her uterus or ovaries are palpated. The following vital signs are measured: T: 98.5○F, HR: 80 bpm, BP: 122/82 mm Hg, RR: 16 breaths/min
Differential Diagnosis Menopause, pregnancy, premature ovarian failure, polycystic ovarian syndrome, hypothyroidism, anorexia nervosa
Laboratory Data and Other Study Results Height and weight: 5﹐ 8", 150 lbs, BMI 22.8 Serum pregnancy test: negative TSH: 3.1 µU/mL FSH: 120 U/L LH: 100 U/L Estradiol: 29 pg/mL DHEA: 3.1 ng/mL Abdominal US: no masses of the kidneys, adrenals, or ovaries; normal uterine appearance
Diagnosis Perimenopause (i.e., early menopause)
Treatment Administered The patient was started on calcium, vitamin D, and alendronate to prevent osteopenia Lubricating agents were recommended for use during intercourse
Follow-up The patient continued to not have menstrual periods and was amenorrheic for 18 months at the time of last follow-up The patient reported to have some improvement in her mood and libido over time The patient reported the elimination of pain during intercourse when lubricating agents were utilized P.267
Steps to the Diagnosis Menopause The permanent end of menstruation due to the cessation of regular ovarian function Occurs typically around 50 years of age Requires 12 consecutive months without a menstrual period; the period of time leading up to this point is the perimenopausal period During the evolution of menopause, the ovarian response to FSH and LH decreases, FSH and LH levels increase, and the serum estrogen level gradually decreases History: hot flashes (i.e., periods of feeling excessively warm due to thermoregulatory dysfunction), diaphoresis, breast pain, menstrual irregularity with eventual amenorrhea, fatigue, anxiety, irritability, depression, dyspareunia (i.e., pain during sexual intercourse due to vaginal wall atrophy and decreased
lubrication), urinary frequency or dysuria Physical examination: breast tenderness, vaginal wall atrophy Tests: FSH and LH are increased; estrogen is decreased Treatment: Calcium and vitamin D supplementation and bisphosphonates are indicated to prevent osteopenia Selective estrogen receptor modulators (e.g., raloxifene, tamoxifen) may help to reduce osteoporosis and cardiovascular risks Regular cardiologic follow-up is important to detect evolving ischemic disease Lubricating agents or topical vaginal estrogen may be used to treat dyspareunia Outcomes: postmenopausal women are at a higher risk for CAD, osteoporosis, and cognitive deterioration Clues to the diagnosis: History: hot flashes, secondary amenorrhea, irritability, dyspareunia Physical: breast tenderness, vaginal dryness Tests: increased FSH and LH, decreased estradiol
One year of amenorrhea is required for a diagnosis of menopause.
Hormone replacement therapy, considered the standard of care for many years, has fallen into disfavor because of its minimal clinical benefits and an increased risk for breast cancer and deep vein thrombosis.
Topical estrogen use is contraindicated in any patient with a history of breast cancer. Pregnancy More thorough discussion in Chapter 12 Why eliminated from differential: the negative serum pregnancy test rules out this diagnosis
The increased risk of osteoporosis in menopausal women is due to decreased estrogen production by the ovaries. Premature ovarian failure Onset of menopausal changes prior to the age of 40 years Although several etiologies exist, they typically involve either premature ovarian follicle loss or follicle dysfunction Unlike menopause, patients retain infrequent menstrual activity during the course of the process Risk factors: tobacco use, pelvic radiation, chemotherapy, autoimmune disease, prior abdominal or pelvic surgery History: similar to menopause except that patients may occasionally have menses
Physical examination: similar to menopause Tests: Increased FSH and LH; decreased estradiol Gonadotropin and estrogen levels during the periods of residual ovarian activity may be normal and warrant repeat periodic testing Karyotyping and ovarian antibody screening may be helpful for determining an underlying cause Treatment: Estrogen and progesterone replacement therapy is indicated until menopausal age is reached to reduce symptomatology Donor ovum may be required to achieve pregnancy Treatment of an underlying disorder may help prevent further ovarian deterioration Outcomes: the prognosis for fertility is poor with few patients being able to achieve an unaided pregnancy; because the complications are the same as for menopause but P.268 occur at a younger age, attention to maintaining cardiac and bone health is important to avoid related sequelae Why eliminated from differential: the age of the patient rules out this diagnosis Polycystic ovarian syndrome More thorough discussion in prior case Why eliminated from differential: the normal DHEA level and the similarity in FSH and LH levels rule out this diagnosis Hypothyroidism More thorough discussion in Chapter 5 Why eliminated from differential: the normal TSH makes this diagnosis unlikely Anorexia nervosa More thorough discussion in Chapter 13 Why eliminated from differential: the normal weight for height in this patient makes this diagnosis unlikely
Case 11-4: “I've been spotting a little” A 57-year-old woman presents to her gynecologist with a complaint of occasional vaginal bleeding. She says that she has experienced occasional bleeding about two to three times per week for the past 3 months. She has been wearing absorbent pads in her underwear consistently because she is unable to predict when this bleeding will occur. The amount of blood is small she says; but she estimates that she finds approximately one tablespoon of blood on her absorbent pads a couple of times each week. She has some mild pelvic cramping around the time of these bleeding episodes. She reports that she thought that she went through menopause 5 years ago and has had no menstrual periods since she was 52 years old. She says that she is married and only has intercourse with her husband. She denies any pain during urination or urinary frequency. She says that her urine is typically light yellow in color. She has a past medical history of morbid obesity, noninsulin-dependent DM, hypercholesterolemia, and HTN. She had two children through full-term vaginal births. She takes metformin, simvastatin, ezetimibe, hydrochlorothiazide (HCTZ), and losartan. She denies any substance use. On examination, she is a morbidly obese woman in no distress. She has no lymphadenopathy.
Her abdomen is soft and nontender, but it is difficult to palpate any viscera due to her body habitus. A rectal examination detects no gross blood. A pelvic examination detects a small amount of blood within the vagina and at the os of the cervix. The cervix is nontender to palpation but is relatively fixed in place. It is difficult to palpate her ovaries or uterus, but the posterior uterine wall near the cervix feels firm. The following vital signs are measured: T: 98.7○F, HR: 85 bpm, BP: 140/84 mm Hg, RR: 20 breaths/min
Differential Diagnosis Uterine fibroids, endometrial cancer, cervical cancer, urinary tract infection, pelvic inflammatory disease, bladder cancer, pregnancy with threatened abortion, menopausal atrophic vaginitis
Laboratory Data and Other Study Results Complete blood cell count (CBC): white blood cells (WBC): 6.9, hemoglobin (Hgb): 12.2, platelets (Plt): 294 7-electrolyte chemistry panel (Chem7): sodium (Na): 145 mEq/L, potassium (K): 4.2 mEq/L, chloride (Cl): 102 mEq/L, carbon dioxide (CO2): 28 mEq/L, blood urea nitrogen (BUN): 24 mg/dL, creatinine (Cr): 1.2 mg/dL, glucose (Glu): 132 mg/dL Serum pregnancy test: negative UA: straw colored, pH: 6.9, specific gravity: 1.015, no glucose/ketones/nitrites/leukocyte esterase/hematuria /proteinuria P.269 Papanicolaou (Pap) smear: atypical squamous cells of undetermined significance; numerous red blood cells (RBCs) Transvaginal US: no cervical masses; significant thickening of the posterior uterine wall; no distinct masses within the uterine cavity Following these studies, a dilation and curettage of the endometrium is scheduled and performed, and the following results are noted: Endometrial cytology: hyperplastic glandular tissue with significant neovascularization The following imaging studies are then performed: Chest CT: normal lung volumes with no atelectasis, infiltrates, or effusions; no pulmonary masses; normal appearance of the heart and pericardium Abdomen/pelvis CT: enlargement of the uterus with significant thickening of the posterior and right lateral walls; close approximation of the right ovary to the uterus; normal size and shape of both ovaries; normal appearance of all remaining viscera; no discrete abdominal or pelvic fluid collections
Diagnosis Endometrial cancer
Treatment Administered Hysterectomy and bilateral salpingo-oophorectomy was performed with regional lymph node biopsies; extension of a uterine mass to the right-sided broad ligament was noted at the time of surgery A chemotherapeutic regimen using cisplatin was initiated following surgery
Follow-up
The patient recovered well from surgery without any complications Lymph node cytology did not detect any presence of metastases Surgical pathology found negative margins surrounding the surgical specimen At 5 years following surgery and chemotherapy, the patient was doing well and considered to be disease free
Steps to the Diagnosis Endometrial carcinoma Adenocarcinoma of the endometrial lining of the uterus Most frequently associated with exposure to high estrogen levels Squamous cell carcinomas of the endometrium occur but are far less common than adenocarcinoma Risk factors: postmenopausal women, unopposed exogenous estrogen use, obesity, PCOS, nulliparity, DM, HTN, family history, high fat diet, hereditary nonpolyposis colorectal cancer (HNPCC) History: heavy menses, midcycle bleeding, or postmenopausal uterine bleeding, possible abdominal pain Physical examination: possible visualization of blood at the cervical os, possible pelvic mass or decreased mobility of the uterus or ovaries Tests: Transvaginal US may be able to detect uterine masses or uterine wall thickening and is frequently the first test performed because it is noninvasive Endometrial biopsy (performed in the clinic or as part of a dilation and curettage procedure) will show hyperplastic glandular tissue with vascular invasion; while office-based biopsies may be performed for most postmenopausal bleeding, abnormal US findings necessitate a dilation and curettage P.270 Chest x-ray (CXR) or chest CT may be used to look for the presence of metastases; abdominal/pelvic CT is not imperative to the diagnosis but may be useful in surgical planning CA125 tumor marker may be increased and can be used for monitoring the response to therapy Treatment: Total abdominal hysterectomy and bilateral salpingo-oophorectomy (TAH-BSO) with lymph node sampling are the primary treatments Radiation therapy is frequently performed for any tumors with extension to the pelvic contents Chemotherapy may be used in addition to or in place of radiation therapy for tumors extending beyond the uterus Surgical debulking should be performed for extensive tumors unable to be fully resected Hormone therapy (e.g., progesterone, tamoxifen) may be beneficial to treating extensive tumors not responding to other treatments Outcomes: Tumors typically metastasize to the peritoneum, aortic and pelvic lymph nodes, lungs, and vagina The 5-year survival rate is 87% for tumors limited to the endometrial lining, 76% for tumors with extension to the cervix, 63% for tumors extending to the vagina, ovaries, or regional lymph nodes, and 37% for tumors
extending to the gastrointestinal (GI) viscera or with distant metastases Endometrial cancer not related to excess endogenous or exogenous estrogen exposure carries a worse prognosis than estrogen-related tumors Clues to the diagnosis: History: postmenopausal bleeding, mild abdominal pain Physical: bleeding from the cervical os, fixed cervix, firm uterine wall Tests: endometrial thickness on transvaginal US, endometrial biopsy results
Menses with <21-day intervals, >35-day intervals, or lasting >7 days, or blood loss >80 mL are considered abnormal.
Common causes of abnormal female genital tract bleeding may be remembered by the mnemonic PANAMA CUTIE: Pregnancy, Anovulation, Neoplasm (benign or malignant), Anatomic abnormality, Medications, Atrophy (uterine), Coagulation disorders, Urinary tract disorders (infection, prolapse), Trauma, Infection, Endocrine disorders (PCOS, thyroid disease). Uterine fibroids (a.k.a. uterine leiomyoma) Benign uterine masses composed of smooth muscle Masses vary in size with the menstrual cycle and generally shrink following menopause Risk factors: nulliparity, alcohol consumption, African American heritage, diet high in meat consumption, family history History: patient may be asymptomatic or may have abdominal pain, menorrhagia (i.e., heavy periods), or abnormal uterine bleeding Physical examination: a uterine mass may be palpable Tests: Transvaginal US or hysteroscopy may be used to detect and visualize uterine masses MRI is useful to determine the size and extent of uterine masses Biopsy is typically required to rule out a malignant neoplasm Treatment: Asymptomatic fibroids may be followed with serial US to look for any abnormal growth GnRH agonists reduce uterine bleeding and the size of the fibroid but should be considered a temporary therapy Hysterectomy is performed for women with symptomatic fibroids Myomectomy may be considered for the resection of symptomatic fibroids in women wishing to maintain fertility Uterine artery embolization following a pelvic MRI to rule out other soft tissue pathology may be performed to selectively infarct small fibroids in women wishing to avoid surgery (carries a high likelihood of infertility)
Outcomes: although most fibroids are asymptomatic, those that do cause symptoms typically require treatment; infertility is the most common complication Why eliminated from differential: the results of the endometrial biopsy rule out this diagnosis
Uterine fibroids do not continue to grow after menopause because of their estrogen sensitivity and the coincident decreased postmenopausal estrogen levels.
While menopausal atrophic vaginitis and uterine fibroids are the most common causes of vaginal bleeding in postmenopausal women (80% cases), endometrial cancer must be ruled out for any postmenopausal woman presenting with this complaint (i.e., perform an endometrial biopsy). P.271
Table 11-7 Bethesda Classification of Cervical Squamous Cell Dysplasia and Appropriate Therapy
Grade
Charac teristic s
Treatment
Atypical squamous cells of
Cellular abnormalities not explained by reactive
HPV screening; repeat Pap smear in 6 and 12 months; repeat HPV
undetermined significance (ASCUS)
changes; not suggestive of intraepithelial lesions
testing in 12 months
Atypical squamous cells, cannot
Cellular abnormalities not explained by reactive
HPV screening; endocervical biopsy; repeat Pap smear in 6 and
exclude HSIL (ASCH)
changes; HSIL cannot be excluded
12 months; repeat HPV testing in 12 months
Low-grade squamous intraepithelial
Mild cellular dysplasia
Repeat Pap smear in 6 and 12 months; repeat HPV testing in 12
lesion (LSIL) (a.k.a. CIN 1)
months; excision by LEEP or conization or laser ablation may be performed
High-grade squamous intraepithelial
Moderate or severe cellular dysplasia including
Excision by LEEP or conization or laser ablation; repeat cervical
lesion (HSIL) (a.k.a. CIN 2 or 3)
carcinoma in situ
cytology every 6 months
Squamous cell carcinoma
Highly atypical cells with stromal invasion
Varies with degree of invasion and extent of involvement
ASCUS, atypical squamous cells of undetermined significance; ASCH, atypical squamous cells, cannot exclude HSIL; CIN, cervical intraepithelial neoplasia; HPV, human papilloma virus; HSIL, high-grade squamous intraepithelial lesion; LEEP, loop electrocautery excision procedure; LSIL, low-grade squamous intraepithelial lesion; Pap smear, Papanicolaou smear.
Cervical cancer Cancer of the cervix that results from the malignant transformation of cervical dysplasia The majority of cases are squamous cell carcinomas (80%), but adenocarcinoma (15%) and adenosquamous carcinoma (5%) are possible Cervical dysplasia Precancerous squamous cell lesions of the cervix that may progress to invasive cervical cancer in up to 22% of cases depending on the cellular grade The cellular grade is classified by the Bethesda system (Table 11-7)
Risk factors: young age at first vaginal intercourse, human papilloma virus (HPV) types 16, 18, 31, and 33, multiple sexual partners, high-risk sexual partners, history of prior sexually transmitted diseases (STDs) History: although typically asymptomatic in the early stages, vaginal bleeding (spontaneous or following intercourse), pelvic pain, or cervical discharge may occur in more progressed disease Physical examination: possible palpable or visible cervical mass, possible decreased cervical mobility Tests: Pap smear is the primary screening tool for detecting cervical dysplasia or cancer Biopsy of concerning lesions (punch or cone) is required to make a formal histologic diagnosis CT, MRI, or transvaginal US may be useful for determining the extent of neoplasms Treatment: Cervical dysplasia is treated with observation or excision depending on the histologic grade (Table 11-7) Malignant neoplasms with <5 mm of invasion may be treated with conization if the patient desires to maintain fertility and are treated with hysterectomy otherwise Chemotherapy is administered for any cancer with close surgical margins A combination of hysterectomy, lymph node excision, radiation therapy, and chemotherapy is used for any invasive lesions that do not extend to abdominal viscera or the distal vagina Neoplasms with extension to pelvic contents or the distal vagina or those with metastases are treated with radiation therapy and chemotherapy P.272 Outcomes: the 5-year survival is >90% for microscopic lesions confined to the cervical epithelium, 65%–85% for visible lesions limited to the uterus, 50% for lesions extending beyond the uterus, and <30% for metastatic lesions Why eliminated from differential: the endometrial biopsy results are more indicative of endometrial cancer; the hysterectomy performed in this case eliminates the need for further work-up of the cervical dysplasia Pelvic inflammatory disease More thorough discussion in later case Why eliminated from differential: the absence of cervical motion tenderness makes this diagnosis unlikely Urinary tract infection More thorough discussion in Chapter 4 Why eliminated from differential: the normal UA makes this diagnosis unlikely Bladder cancer More thorough discussion in Chapter 4 Why eliminated from differential: the absence of hematuria on the UA makes this diagnosis unlikely Pregnancy/threatened abortion More thorough discussion in Chapter 12 Why eliminated from differential: the negative serum pregnancy test rules out these diagnoses
Atrophic vaginitis (due to menopause) More thorough discussion in prior case (refer to section on menopause/perimenopause) Why eliminated from differential: although vaginal irritation commonly occurs in menopause, this diagnosis would not account for the uterine bleeding or biopsy abnormalities
All women should receive annual Pap smears beginning approximately 3 years after the onset of vaginal intercourse or no later than 21 years of age. In those who have never engaged in intercourse, Pap smears may be deferred because of the low risk of cervical cancer related to HPV.
Case 11-5: “My belly hurts so bad!” A 19-year-old woman presents to an emergency department with significant abdominal pain that has been increasing over the course of the day. She says that she had mild lower abdominal pain when she went to bed last night and had difficulty sleeping because the pain gradually increased during the night. She was unable to go to work this morning because the pain had increased in severity. She says the pain is now significant and that she is having a difficult time finding a comfortable position on the stretcher. She used a sister's oxycodone this morning, and this provided some pain relief. The pain has returned since the medication wore off. She has been nauseous for most of the day and has had nonbilious vomiting twice since arriving in the emergency department. She has not eaten anything since breakfast. She denies any pain with urination or bowel movements. She denies seeing any blood in her urine, stool, or emesis. She denies chest pain, pleuritic pain, or headaches. She says her last menstrual period was 5 days ago and was lighter than normal. She has not noticed any vaginal bleeding since that time. She denies ever having had this type of pain previously. She denies having eaten any unusual foods recently. She is sexually active with two partners and uses a contraceptive patch for birth control. She denies any past medical history and does not take any medications besides birth control. She smokes a half-pack of cigarettes per day and denies any other substance use. On examination, she is a well-nourished woman in mild-moderate distress due to her abdominal pain. Auscultation of her lungs and heart detects normal breath sounds, regular tachycardia, and no extra heart sounds. She voluntarily guards her lower central abdomen, but her epigastrium is soft. No masses are palpable, and she has no rebound tenderness. She has few audible bowel sounds. A rectal examination shows no gross blood. A pelvic examination is performed and detects some purulent fluid in the posterior recess of her vaginal canal. No vaginal or cervical lesions or uterine bleeding are appreciated. She has exquisite cervical tenderness. The bimanual examination detects no ovarian masses, but she is very uncomfortable during adnexal palpation. The following vital signs are measured: T: 102.4°F, HR: 110 bpm, BP: 130/85 mm Hg, RR: 18 breaths/min
Differential Diagnosis Ectopic pregnancy, appendicitis, vaginitis, cervicitis, pelvic inflammatory disease, premenstrual syndrome, endometriosis, ovarian torsion, ovarian cyst, gastroenteritis, urinary tract infection, diverticulitis
Laboratory Data and Other Study Results CBC: WBC: 13.8, Hgb: 13.9, Plt: 356 Urine pregnancy test: negative UA (via catheterization): straw colored, pH: 7.1, specific gravity: 1.020, no glucose/ketones/nitrites/leukocyte esterase/hematuria/proteinuria Erythrocyte sedimentation rate (ESR): 27 mm/hr C-reactive protein (CRP): 4.3 mg/dL Vaginal fluid microscopy (saline): no mobile organisms or clue cells
Vaginal fluid microscopy (potassium hydroxide [KOH]): no hyphae or pseudohyphae Vaginal fluid Gram stain and culture: numerous WBCs and moderate Gram-negative diplococci on Gram stain; culture pending Neisseria gonorrhoeae and Chlamydia trachomatis immunoassays: pending Stool Hemoccult (Beckman Coulter, Fullerton, California) test: negative Transvaginal US: no ovarian masses, cysts, or abscesses; mildly enlarged uterus; normal Doppler flow in both ovaries Abdominal/pelvic CT: mild diffuse enlargement of the uterus; no intra-abdominal or pelvic masses; no abscesses or free fluid collections; normal appearing viscera including the cecum and colon
Diagnosis Pelvic inflammatory disease
Treatment Administered The patient was admitted to hospital for IV antibiotics, pain control, and IV hydration The patient was started on IV clindamycin and gentamicin
Follow-up The patient was more comfortable after initiating pain medicines Immunoassays were positive for N. gonorrhoeae The patient's symptoms further improved after a few days of antibiotic therapy; the patient was then switched to oral clindamycin for a total of 14 days of antibiotics The patient was discharged once she was afebrile and her pain was controlled The patient was told that it was vital for her sexual partners to be treated to prevent her from becoming reinfected Contraception options and safe sexual practices were discussed with the patient prior to discharge
Steps to the Diagnosis Pelvic inflammatory disease (PID) Progressive N. gonorrhoeae or C. trachomatis infection of the genital tract, resulting in extension to the uterus, fallopian tubes, ovaries, or abdominal cavity Escherichia coli, Bacteroides, and streptococcus are less common causes Risk factors: multiple sexual partners, unprotected vaginal intercourse, prior PID, douching, young age at first intercourse History: lower abdominal pain starting a few days after menses, nausea, vomiting Physical examination: cervical motion tenderness, adnexal tenderness, fever, abdominal pain, purulent cervical discharge, possible abdominal guarding Tests:
Increased ESR and CRP; possible increased serum WBCs Gram stain, culture, and immunoassays of vaginal fluid are important to identifying the pathogen Culdocentesis (i.e., aspiration of intraperitoneal fluid from the cul de sac posterior to uterus) will yield purulent fluid, but is rarely performed because of its high false negative rate Pregnancy must be ruled out with a pregnancy test Transvaginal US may detect an enlarged and inflamed uterus, fallopian tube or ovarian abscess, or abdominal free fluid Laparoscopy may be used to directly visualize the inflamed abdominal viscera Treatment: Patients with severe pain, nausea and vomiting disrupting oral antibiotic use, immunodeficiency, failure to improve with outpatient therapy, pelvic abscesses, or coincident pregnancy should be admitted for inpatient care Empiric antibiotics should be given until a pathogen is identified, and then agent-specific antibiotics should be used Sexual partners must be treated to avoid reinfection Surgical drainage of pelvis abscesses must be performed Outcomes: complications include chronic pelvic pain, infertility due to uterine and fallopian tube scarring, and a significantly increased risk of future ectopic pregnancy Clues to the diagnosis: History: severe abdominal pain, nausea, vomiting, recent menses, multiple sexual partners Physical: cervical motion tenderness, adnexal tenderness, pus in the vaginal canal Tests: increased ESR and CRP, Gram stain appearance of vaginal fluid, uterine enlargement on US and CT
Patients with PID may exhibit the “chandelier sign”—palpating the cervix during the pelvic examination may be so painful that the patient almost jumps off of the examination table (and hits the ceiling).
Barrier contraception use can reduce the risk of PID.
Suspect a tubo-ovarian abscess in a patient with PID who also has signs of sepsis or peritonitis. Inpatient treatment is required with IV hydration, IV antibiotics, and surgical drainage. Cervicitis Infection of the columnar cervical epithelium most commonly by N. gonorrhoeae or C. trachomatis Infection of the urinary tract, rectum, or oropharynx may also occur via sexual contact History: may be asymptomatic or may have dyspareunia, postcoital bleeding, purulent vaginal or urethral discharge Physical examination: purulent fluid in the vaginal canal or cervical os may be visualized, possible cervical
motion tenderness Tests: Enzyme immunoassays are available for detecting both pathogens DNA probes and DNA amplification testing via polymerase chain reaction (PCR) are highly sensitive tests for detecting pathogens in a vaginal fluid sample A Gram stain of cervical scrapings will show Gram-negative diplococci with N. gonorrhoeae infection, but bacteria is frequently not seen with C. trachomatis infection Culture of vaginal fluid or cervical scrapings on Thayer-Martin agar will yield N. gonorrhoeae Treatment: Ceftriaxone or a macrolide is given for N. gonorrhoeae, and doxycycline or azithromycin is administered for C. trachomatis In pregnant patients, ceftriaxone and azithromycin are the drugs of choice because of the teratogenic risks of the other viable antibiotics Antibiotics for both pathogens are typically administered because dual infection is common Sexual partners must be treated because of the high risk of reinfection Outcomes: untreated infection can progress to PID or septic arthritis Why eliminated from differential: although this case must have been cervicitis at some point during its evolution, the severity of symptoms and the development of gastrointestinal symptoms make PID a more accurate diagnosis
Chlamydia infection is the most common STD because it may be asymptomatic (especially in men) and frequently goes unrecognized.
C. trachomatis infection is asymptomatic in >50% of cases, and the symptoms are typically milder than those with N. gonorrhoeae infection.
Clinical cervicitis with negative Gram stain and cultures is highly suggestive of Chlamydia infection. Vaginitis Vaginal infection caused by an overgrowth of protozoans (e.g., Trichomonas), fungus (e.g., Candida albicans), or normal vaginal bacteria (e.g., Gardnerella vaginalis) Risk factors: unprotected sex, multiple partners, young age at first vaginal intercourse, douching, intrauterine device (IUD) use, tobacco use, DM, human immunodeficiency virus (HIV) infection History: vaginal pain or itching, vaginal discharge Physical examination: a pelvic examination will visualize vaginal inflammation (Figure 11-4, Table 11-8) Tests: examination of vaginal fluid under a microscope (i.e., wet mount) in both saline and KOH and pH testing of a vaginal fluid specimen are both useful to identify the causative pathogen Treatment: metronidazole (for G. vaginalis or Trichomonas), clindamycin (for G. vaginalis), and fluconazole (for C. albicans) are the most commonly administered antibiotics
Outcomes: the prognosis is excellent with treatment, but failure to treat sexual partners in cases of Trichomonas infections may lead to reinfection Why eliminated from differential: the negative wet mounts and the severity of symptoms in this case rule out this diagnosis
Lactobacilli are a common normal bacteria whose presence on a wet mount does not suggest infection.
Treatment of sexual partners is unnecessary for G. vaginalis or C. albicans but is required with Trichomonas infection. Ectopic pregnancy More thorough discussion in Chapter 12 Why eliminated from differential: the negative pregnancy test rules out this diagnosis Appendicitis More thorough discussion in Chapter 3 Why eliminated from differential: the negative CT and the presence of cervical motion tenderness make this diagnosis less likely, but it should still be considered in the presence of worsening abdominal pain that does not improve with antibiotics for PID
Figure 11-4 Vaginal wet mount (saline prep) showing clue cell with multiple bacteria attached to the border. A normal epithelial cell is adjacent to the clue cell (1,000X magnification).
Premenstrual syndrome (PMS)/premenstrual dystrophic disorder (PMDD) Syndromes seen in women with normal functioning ovaries that precede menses and are characterized by multiple pain, mood, and autonomic symptoms that interfere with the ability to function normally
Abnormal mood is more severe in cases of PMDD, and this differentiates it from PMS Pelvic pain and moodiness are common preceding menses but a diagnosis of PMS or PMDD constitutes a significant interference with daily function Risk factors: family history History: weight gain, headache, pelvic pain, abdominal bloating, change in bowel habits, food cravings, moodiness (e.g., depression, irritability), and fatigue are all possible complaints; these symptoms precede menses and occur at the same time point in each menstrual cycle Physical examination: breast tenderness, edema, acne, abdominal tenderness Tests: this is generally a clinical diagnosis, and lab tests are not required
Table 11-8 Common Infectious Causes of Vaginitis
Charac teristic s
Physical
Gardnerella V aginalis
Mild vaginal inflammation
examination
Tric homonas
Vaginal and cervical
Candida albic ans
Significant vaginal inflammation
inflammation, c ervic al petec hiae
Discharge
Thin, white, fishy odor
Malodorous, frothy, greenish
Thick, white, “c ottage c heese-like”
Wet mount
Clue c ells (i.e., epithelial cells with
Motile tric homonads
Normal
(saline)
multiple attached bacteria)
Wet mount
Fishy odor (i.e., positive whiff test)
Possible fishy odor
Pseudohyphae
Vaginal pH
>4.5
>4.5
3.5–4.5
Treatment
Metronidazole
Metronidazole (also treat
Topical clotrimazole, miconazole, or
partner)
nystatin, or oral fluconazole (single dose)
(KOH)
KOH, potassium hydroxide.
Treatment: Exercise, relaxation therapy, vitamin B6, nonsteroidal anti-inflamatory drugs (NSAIDs), oral contraceptives, progestins, benzodiazepines, and selective serotonin reuptake inhibitors (SSRIs) are all viable options in improving symptoms SSRIs and benzodiazepines take on a more prominent role in PMDD Hysterectomy and oophorectomy are frequently curative but should be reserved for the most severe cases Outcomes: The prognosis is guarded because no one regimen of treatment works for everyone, and a combination of different therapies must be designed for each patient The use of psychiatric medications tends to be very helpful for controlling the mood symptoms of the
diagnoses Symptoms typically wane after menopause Why eliminated from differential: the timing of the symptoms, presence of cervical motion tenderness, and lab tests suggesting infection rule out this diagnosis
If a patient suspected of having PMS or PMDD has mood symptoms throughout her menstrual cycle, initiate a psychiatric work-up for a mood disorder. Menstrual-related symptoms should only occur in the second half of the cycle. Endometriosis The presence of ectopic endometrial tissue outside the uterine cavity Retrograde menstruation, vascular spread of endometrial tissue from the uterus to the pelvic cavity, or the iatrogenic spread of endometrial tissue (e.g., cesarian section) are the most plausible explanations for the evolution of the condition Tissue may be located on the ovaries, broad ligament, or abdominal wall and viscera The ectopic tissue behaves according to the menstrual cycle, which causes the characteristic symptoms Risk factors: family history, infertility, nulliparity, low body mass index (BMI) History: pelvic pain, dyspareunia, dysmenorrhea (i.e., painful menses), dyschezia (i.e., painful bowel movements), possible infertility Physical examination: uterine or ovarian tenderness, possible palpable uterine or ovarian adhesions Tests: Diagnostic laparoscopy is the principal means of identifying the ectopic tissue (appears as “powder burn” lesions and cysts) (Color Figure 11-1) Biopsy of the lesions (via laparoscopy) will show the presence of endometrial tissue CA125 is frequently increased but is more useful for tracking the response to treatment than for making a diagnosis Treatment: Oral contraceptives, progestins, danazol, and GnRH agonists may supply symptomatic relief Laparoscopic ablation of the lesions may be curative, but it is difficult to locate and remove all of the symptomatic lesions
Color Figure 11-1 Pathology specimen of an ovary demonstrating the characteristic powder-burn lesions of endometriosis.
Hysterectomy, lysis of adhesions, or salpingo-oophorectomy may be indicated in severe cases Outcomes: approximately 50% of patients will continue to have pain despite treatment, and 33% of patients will have full resolution regardless of treatment; infertility is the most common and frustrating complication Why eliminated from differential: the increased inflammatory markers are more suggestive of an infectious process, but this diagnosis should still be considered if there is no improvement following antibiotic administration
Primary dysmenorrhea (i.e., without pelvic pathology) symptoms occur in the beginning of menstruation and resolve over several days; secondary dysmenorrhea (i.e., associated with pelvic pathology) symptoms begin midcycle before the onset of menstruation and increase in severity until the conclusion of menstruation.
Endometriosis is the most common cause of female infertility and may be responsible for up to 50% of cases. Ovarian torsion Rotation of an ovary around the oviduct, leading to ovarian ischemia History: acute severe unilateral abdominal pain, nausea and vomiting Physical examination: ovarian tenderness Tests: US with Doppler blood flow imaging is the best means of detecting the abnormality Treatment: laparoscopic derotation must be performed promptly to prevent infarction; oophorectomy is indicated if ovarian necrosis occurs Outcomes: although the prognosis is excellent and the likelihood of recurrence is small with timely treatment, delays in treatment are associated with worse outcomes Why eliminated from differential: the normal appearance of the transvaginal US makes this diagnosis unlikely Ovarian cyst
More thorough discussion in later case Why eliminated from differential: the normal appearance of the transvaginal US makes this diagnosis unlikely Gastroenteritis More thorough discussion in Chapter 3 Why eliminated from differential: the presence of clinical signs of significant cervical inflammation make this diagnosis less likely Urinary tract infection More thorough discussion in Chapter 4 Why eliminated from differential: the normal UA makes this diagnosis unlikely Diverticulitis More thorough discussion in Chapter 3 Why eliminated from differential: the negative Hemoccult test and normal abdominal CT make this diagnosis unlikely
Case 11-6: “I've got a weird bump down there” A 27-year-old woman presents to her gynecologist because she has noticed the development of a single red bump in her vulvar region. She says that she is unsure how long this bump had been present, but she noticed it for the first time 3 weeks ago while shaving. Last week she saw that the bump had started to disappear but was being replaced by a small “hole in the skin,” so she decided to schedule an emergency appointment. She denies any pain or pruritus at the site of the lesion. She has never had a similar lesion in the past. She has not noticed any drainage of pus from the lesion or vaginal discharge. She has examined herself multiple times since discovering this lesion but has been unable to find any other lesions around her vagina. She denies any nausea, vomiting, diarrhea, abdominal pain, arthralgias, or fatigue. Her last menstrual period was 3 weeks ago and was considered to be normal. She has had multiple sexual partners in the past year. She is notable for having had two prior Chlamydia infections that were treated with doxycycline. She has never been pregnant and receives medroxyprogesterone acetate injections every 3 months for birth control. She drinks alcohol 3 to 4 days per week but denies any other substance use. On examination, she is a well-appearing woman in no acute distress. Her abdomen is soft and nontender and has normal bowel sounds. She has a few small but palpable inguinal lymph nodes. There is a 1-cm wide shallow ulceration with raised edges on her right labia without any active drainage. There are no other lesions present in the groin or remainder of the body. The region around the ulcer is nontender. A pelvic examination is performed and finds no purulent fluid in the vagina or coming from the cervix. There are no apparent vaginal or cervical lesions. There is no cervical or adnexal tenderness and no palpable masses on the bimanual examination. No lesions, masses, or tenderness is apparent on a rectal examination. A neurovascular examination is normal. The following vital signs are measured: T: 98.7°F, HR: 65 bpm, BP: 120/75 mm Hg, RR: 16 breaths/min
Differential Diagnosis Syphilis, human papilloma virus, herpes simplex, chancroid, lymphogranuloma venereum, granuloma inguinale, varicella, molluscum contagiosum, HIV
Laboratory Data and Other Study Results CBC: WBC: 9.7, Hgb: 14.4, Plt: 251 Venereal Disease Research Laboratory test (VDRL): negative
Rapid plasma reagin (RPR): positive N. gonorrhoeae and C. trachomatis immunoassays: negative Tzanck smear: no giant cells seen HIV ELISA: negative A biopsy of the ulcer is performed during the initial assessment, and the following tests are performed: Gram stain and culture: several macrophages and lymphocytes but no bacteria visualized; culture pending Giemsa stain: no bacteria or inclusion bodies Hematoxylin and eosin stain: several macrophages and lymphocytes; no visualized bacteria The following test is performed after receipt of these results (2 weeks later): Fluorescent treponemal antibody absorption (FTA-ABS): positive for treponemal antibodies Repeat VDRL: positive, high-titer
Diagnosis Syphilis
Treatment Administered The patient was given a single intramuscular dose of penicillin G The case was reported to the state health agency Safe sex and STD-avoidance practices were discussed with the patient The patient was instructed to tell all recent sexual contacts to be seen by a physician for syphilis screening and treatment
Follow-up The lesion healed over the course of 6 weeks Follow-up VDRL tests demonstrated a decreased titer
Steps to the Diagnosis Syphilis A sexually transmitted disease caused by the spirochete Treponema pallidum Vertical transmission from a mother to child is also possible The disease is classified into stages based on its extent of involvement and the degree of symptomatology Primary: limited to the genitals Secondary: onset of systemic infection and the corresponding findings Latent: multiyear asymptomatic period following the second stage Tertiary: onset of neurologic and deep tissue involvement
History: Primary: appearance of a single firm papule that evolves into a painless ulcer (i.e., primary chancre) at the area of transmission Secondary: headaches, malaise, and a rash on the palms and soles that occur following the resolution of the primary chancre, self-resolve, and briefly relapse over the subsequent few years Latent: asymptomatic for multiple years Tertiary: deep pain, mood deterioration Physical examination: Primary: solitary ulceration with raised edges on the genitals, possible lymphadenopathy Secondary: fever, maculopapular rash on the palms and soles, lymphadenopathy, plaques in intertriginous areas (i.e., condyloma lata) Tertiary: cutaneous ulcerations (i.e., gummas), jaundice, fever, worsening two-point discrimination and proprioception (i.e., tabes dorsalis), Argyll-Robertson pupils (i.e., bilateral small pupils that further constrict with concentration on an object), decreased deep tendon reflexes Tests: The RPR and VDRL are the initial screening tests and are 80% sensitive; the VDRL titer may remain normal for the initial few weeks following infection and should decrease at least four-fold following appropriate treatment FTA-ABS or the microhemagglutination assay for antibodies to treponemes (MHA-TP) is used as a confirmatory test following a positive RPR or VDRL Dark-field microscopy is required to visual the spirochetes Examination of the cerebral spinal fluid (CSF) is required in any patient suspected of having tertiary syphilis to confirm the diagnosis Treatment: Penicillin G remains the mainstay of treatment, but doxycycline and tetracycline are other antibiotic options Penicillin G is routinely used in cases of tertiary disease Cases should be reported to a state health agency Outcomes: The majority of complication occur with long-term disease having entered the tertiary stage Gummas develop in the skin, bones, and liver, causing extensive erosive lesions Aortic regurgitation, aortitis, and aortic aneurysm formation develop from cardiac involvement Neurosyphilis is characterized by tabes dorsalis, the development of mood instability, and cognitive decline Initiation of treatment will frequently cause a brief exacerbation of symptoms and signs (i.e., JarischHerxheimer reaction) if initiated in the secondary or later stage The prognosis is excellent with treatment up to the tertiary stage Although the vast majority of patients with tertiary syphilis will be cured of the disease and will have
symptomatic improvement following treatment, visceral gummatous destruction and neurosyphilis are permanent sequelae Disease progression is faster in patients with comorbid HIV, and longer treatment is required Clues to the diagnosis: History: papule that evolved into an ulcer, multiple sexual partners, prior STDs Physical: appearance of the primary chancre, lymphadenopathy Tests: positive RPR and repeat VDRL, positive FTA-ABS
The primary chancre of syphilis appears 1 to 13 weeks after transmission of the disease and heals within 9 weeks of appearing.
Thirty-three percent of patients with untreated syphilis will progress to the tertiary stage within 30 years following infection.
RPR and VDRL will eventually become negative following syphilis treatment, but FTA-ABS will remain positive for life.
Treponema pallidum cannot be cultured. Human papillomavirus (HPV) A collection of several types (>100) of papillomavirus that may be causative for genital warts (types 6 and 11) and cervical cancer (types 16 and 18) History: typically painless warts or small papules Physical examination: multiple small pink papules or isolated large cauliflowerlike warts (depending on the causative viral type) at the site of contact (externally or internally) Tests: HPV DNA typing is performed on lesion specimens and can identify the causative type of virus; it should be performed for any patient with an abnormal Pap smear Colposcopy allows close examination of cervical lesions during a pelvic examination; the application of acetic acid to lesions will turn them white and make them easier to identify (Color Figure 11-2) Histologic examination of biopsies will show squamous cells with shrunken nuclei and perinuclear clearing Treatment: HPV vaccine is effective for prevention of types 6, 11, 16, and 18 of the virus Topical podophyllin, trichloroacetic acid, and 5-flurouracil may be used to remove lesions Injectable α-interferon, laser therapy, and cryotherapy are additional options for lesion removal
Color Figure 11-2 Colposcopy view of the cervix with multiple lesions consistent with human papilloma virus infection. Application of acetic acid to the cervix during the examination has made these lesions more apparent.
Outcomes: scarring may result from the removal of large warts; cervical cancer is the most concerning sequelae and requires close observation in patients infected with the associated virus types Why eliminated from differential: the degeneration of a HPV lesion into an ulcer would be unusual, and the positive RPR and VDRL are confirmative of the true diagnosis; given the patient's sexual history, screening for HPV infection and regular Pap smears would be highly recommended
The HPV vaccine acts as a prophylaxis for 90% of genital warts and 70% of cervical cancers. Herpes simplex More thorough discussion in Chapter 10 Why eliminated from differential: the negative Tzanck smear and painless nature of the lesion make this diagnosis unlikely Varicella More thorough discussion in Chapter 10 Why eliminated from differential: the limited extent of the lesion in the case (only one lesion) and the negative Tzanck smear make this diagnosis unlikely HIV
More thorough discussion in Hematology and Oncology chapter Why eliminated from differential – the negative ELISA rules out this diagnosis Chancroid A highly contagious disease of genital ulceration caused by the Gram-negative bacillus Haemophilus ducreyi Most cases occur in tropical or subtropical regions or in immunocompromised patients History: painful ulcer that appears within 2 weeks of exposure Physical examination: ulcer with a grayish base and a foul odor at the site of contact, significant inguinal lymphadenopathy (i.e., bubo formation) Tests: Gram stain of a lesion specimen will show Gram-negative rods; culture is the most definitive test but difficult to perform Treatment: ceftriaxone, erythromycin, and azithromycin are potential antibiotics Outcomes: buboes that rupture may cause severe scarring and sinus tract formation; the prognosis is excellent for recovery following treatment Why eliminated from differential: the painless nature of the ulcer in this case and the negative Gram stain make this diagnosis unlikely
The characterization of genital ulcers may be remembered by the mnemonic “Some Girls Love Licorice, but Fellows Hate Candy”: Syphilis, Granuloma inguinale, and Lymphogranuloma venereum = painLess. PainFul = Herpes simplex and Chancroid. Lymphogranuloma venereum A sexually transmitted disease caused by the L-1, L-2, and L-3 serotypes of C. trachomatis The causative serotypes are different from the types of C. trachomatis responsible for cervicitis History: malaise, headaches, and a short-lived painless papule or ulcer occur within 2 weeks of transmission Physical examination: fever, presence of a small papule that evolves into an ulcer, formation of significant buboes after a month of infection Tests: immunoassays for C. trachomatis will be positive for the appropriate serotypes Treatment: tetracycline, erythromycin, or doxycycline Outcomes: buboes may rupture and cause scarring, sinus tract formation, abscess formation, and lower extremity elephantiasis (i.e., severe lymphedema) Why eliminated from differential: the negative immunoassays for C. trachomatis and the positive RPR and VDRL rule out this diagnosis Granuloma inguinale Sexually transmitted disease with genital ulcerations caused by Klebsiella granulomatis infection History: genital papule that becomes a painless ulcer several weeks after transmission Physical examination: genital ulcer with a beefy red base and irregular borders, mild lymphadenopathy Tests: Giemsa staining of lesion specimens will show Donovan bodies (i.e., red encapsulated intracellular Gram-negative rods)
Treatment: doxycycline or trimethoprim-sulfamethoxazole (TMP-SMX) Outcomes: complications include recurrence of infection, scarring from the ulcers, elephantiasis, and an increased risk of basal cell and squamous cell carcinoma Why eliminated from differential: the negative Giemsa stain histology makes this diagnosis unlikely Molluscum contagiosum More thorough discussion in Chapter 10 Why eliminated from differential: the negative Giemsa stain histology makes this diagnosis unlikely
Case 11-7: “I think there is a lump on my ovary” A 32-year-old woman presents to her gynecologist because she is concerned that she may have a mass on her right ovary. She reports having a mild achy pain in her lower abdomen and feeling bloated for 1 week. When she was rubbing her abdomen in the area of discomfort a few days ago, she thinks that she may have felt a deep mass. She says that she is still able to feel the possible mass, but it is too deep to appreciate where it is located. When she tries to palpate it, she has pain with deep pressure. She also has some lower abdominal pain when she bears down during bowel movements. She denies having had this experience in the past. Her last menses was 2 weeks ago and was considered to be normal. She has not had any vaginal bleeding since the end of her menses or any vaginal discharge. She denies any nausea, vomiting, diarrhea, blood in her urine or stool, dysuria, or urinary frequency. She says that the pain is not in her vagina and that she has not had pain during intercourse. She denies any past medical history and takes oral contraceptive pills and a calcium supplement. She drinks socially on weekends and denies other substance use. She says that her mother had a history of ovarian cancer, so she is concerned about her current symptoms. On examination, she is a thin woman in no distress. Auscultation of her heart and lungs is normal. Her abdomen is soft, and she has some mild tenderness on deep palpation of her lower right-central abdomen. She has no rebound pain or guarding. There is a subtle fullness in her abdomen in the region of greatest discomfort. A pelvic examination detects no cervical or vaginal discharge or irritation. She has no cervical motion tenderness. On the bimanual examination, she has an enlarged, moderately tender right ovary and normal left ovary. A rectal examination detects no tenderness or gross blood. The following vital signs are measured: T: 98.6°F, HR: 68 bpm, BP: 115/78 mm Hg, RR: 16 breaths/min
Differential Diagnosis Ectopic pregnancy, appendicitis, uterine fibroids, cervicitis, endometriosis, ovarian torsion, ovarian cyst, ovarian cancer, urinary tract infection, bladder cancer, colon cancer, diverticulitis
Laboratory Data and Other Study Results Urine pregnancy test: negative UA: straw colored, pH: 7.0, specific gravity: 1.010, no glucose/ketones/nitrites/leukocyte esterase/hematuria /proteinuria Hemoccult test: negative Transvaginal US: 5 cm fluid-filled cyst with smooth borders and a homogenous appearance on the right ovary; normal left ovary; normal uterine wall thickness; normal Doppler flow in both ovaries Abdominal/pelvic CT: 5 cm cystic mass on the right ovary; normal-appearing uterus and left ovary; normal abdominal viscera Following these tests, the additional following test is ordered: CA125: 7 U/mL
Diagnosis Benign ovarian cyst, most likely follicular type
Treatment Administered The patient was instructed to take ibuprofen as needed The patient was scheduled for a repeat transvaginal US in 8 weeks
Follow-up The cyst had resolved completely when the US was repeated
Steps to the Diagnosis Benign ovarian cyst Benign cystic mass of an ovary There are several types of benign ovarian cystic and solid masses exist (Table 11-9) History: although frequently asymptomatic, benign masses may be associated with lower abdominal pain, nausea, vomiting, or abdominal fullness Physical examination: palpable ovarian mass on the bimanual examination, abdominal tenderness, possible guarding and rebound tenderness in cases of rupture Tests: CA125 is rarely increased in cases of benign masses but is more frequently increased in cases of malignancy Transvaginal US is the principal diagnostic tool and is used to determine the type of mass (e.g., cystic, solid) and the quality of its structure (e.g., irregular, septated, smooth)
Table 11-9 Common Types of Benign Ovarian Masses
Tumor
Follicular cyst
Corpus luteum
Origin
Charac teristic s
Ovarian follicle
Granulosa cells; cystic (~3
Abdominal pain and
Observation; ovarian
cm diameter); occurs in
fullness; palpable tender
cystectomy is performed if
first 2 weeks of cycle and
mass on bimanual exam;
mass does not regress or with
may regress over
peritoneal signs if torsion
an increased suspicion of
menstrual period
or rupture occur
cancer
Thec a cells; cystic or
Abdominal pain; palpable
Observation; ovarian
hemorrhagic corpus
tender mass on bimanual
cystectomy is performed if
luteum; usually larger and
examination; greater
mass does not regress or with
firmer than follicular cyst;
risk of torsion or rupture
an increased suspicion of
more common in later
with significant bleeding
cancer; rupture with
weeks of cycle
than follicular cyst
significant hemorrhage
Corpus luteum
cyst
H/P
Treatment
requires surgical hemostasis and cystectomy
Mucinous or
Epithelial
May resemble endometrial
Frequently asymptomatic
Unilateral salpingo-
serous
tissue
or tubal histology; cystic
until significant growth
oophorectomy; TAH-BSO if
cystadenoma
Endometrioma
Endometrium
postmenopausal
with serous or mucinous
has occurred; palpable
contents; may form
mass on bimanual
calcifications (i.e.,
examination or during
psammoma bodies); may
abdominal examination if
bec ome extremely large
large
Spread of endometriosis to
Frequently
OCPs, GnRH agonists,
involve ovary; similar
asymptomatic; abdominal
progestins, or danazol may
behavior to other sites of
pain and tenderness and
improve symptoms;
endometriosis
dyspareunia; infertility
cystectomy or oophorectomy is frequently required due to a high recurrence rate
Composed of multiple
Frequently
Cystectomy with attempted
teratoma (i.e.
dermal tissues including
asymptomatic; oily
preservation of the ovary if
desmoid cyst)
hair, teeth, and sebaceous
contents released during
benign; 1%–2% undergo
glands
rupture may cause
malignant transformation and
peritonitis
require salpingo-
Benign cystic
Germ cells
oophorectomy
Stromal cell
Granulosa,
Secrete hormones
Prec oc ious puberty
Unilateral salpingo
tumor
theca, or
appropriate to cells of
(granulosa theca cell
oophorectomy; TAH-BSO if
Sertoli Leydig
origin; has malignant
tumors) or virilization
postmenopausal
cells
potential
(Sertoli-Leydig cell tumors)
GmRH, gonadotropin-releasing hormone; OCPs, oral contraceptive pills; TAH-BSO, total abdominal hysterectomy with bilateral salpingo-oophorectomy.
MRI is useful in difficult-to-categorize masses to determine their site of origin and structure Biopsy is the definitive means of making a diagnosis but is usually reserved for persistent masses or those with an abnormal appearance (e.g., irregular borders, both cystic and solid elements) Treatment: Observation is acceptable for early cysts in the presence of a normal CA125 level Cystectomy or salpingo-oophorectomy may be considered in persistent cysts, those associated with an elevated CA125, or those in postmenopausal women The treatment of other benign ovarian masses varies with their origin (Table 11-9) Outcomes: The prognosis for benign cysts is excellent The prognosis for noncystic benign ovarian masses is also good with low risks of malignant transformation Potential complications include ovarian torsion or cystic rupture Clues to the diagnosis: History: lower abdominal pain, possible palpable mass Physical: tender ovarian mass on pelvic examination Tests: transvaginal US and CT results, normal CA125 level
US findings of a cystic mass with smooth edges and few septa are more consistent with benign ovarian tumors. Findings of irregularity, nodularity, multiple septa, and
pelvic extension are more suggestive of malignancy. Ectopic pregnancy More thorough discussion in Chapter 12 Why eliminated from differential: the negative pregnancy test rules out this diagnosis Uterine fibroids More thorough discussion in prior case Why eliminated from differential: the normal appearance of the uterus on US makes this diagnosis unlikely Cervicitis More thorough discussion in prior case Why eliminated from differential: the absence of dyspareunia, cervical motion tenderness, and cervical discharge make this diagnosis unlikely; immunoassays for C. trachomatis and N. gonorrhoeae should be considered for persistent discomfort or the evolution of lower genital tract symptoms Endometriosis More thorough discussion in prior case Why eliminated from differential: although this diagnosis is possible, the occurrence of the abdominal discomfort following menses and not during menses decreases its likelihood; the transvaginal US is supportive of the true diagnosis, but a diagnostic laparoscopy could be considered for persistent pain and mass Ovarian torsion More thorough discussion in prior case Why eliminated from differential: the normal Doppler studies make this diagnosis less likely Ovarian cancer Malignant neoplasm of the ovary from most likely either an epithelial (90% of cases) or germ cell origin Most diagnoses occur only after significant growth has occurred Risk factors: family history, infertility, nulliparity, presence of BRCA1 or BRCA2 gene mutations History: usually asymptomatic until late in the course of the disease when abdominal pain and bloating, fatigue, weight loss, change in bowel habits, and menstrual irregularity develop Physical examination: later in the disease course a palpable ovarian mass, ascites, or a pleural effusion may detected Tests: CA125 is increased in 80% of cases, but is normal in the early stages in 50% of those cases Germ cell tumors may be associated with increased α-fetoprotein, hCG, or LDH US, MRI, or CT is used to detect the mass and determine the extent of its involvement Although biopsy can be helpful in confirming a diagnosis, surgical resection is indicated for most ovarian masses with a concerning appearance regardless of biopsy results Treatment:
Surgical resection is the primary treatment and involves salpingo-oophorectomy or TAH-BSO, peritoneal sampling, pelvic and para-aortic lymph node biopsies, and omentectomy Extensive tumors may require serial debulking surgeries Chemotherapy (most commonly platinum based) is given following surgery Outcomes: the 5-year survival for disease limited to the ovaries (with or without ascites) is 73%, 45% for disease with extension to the uterus or fallopian tubes, 21% for disease with peritoneal or hepatic extension and lymph node involvement, and 5% for distant metastases Why eliminated from differential: the cystic and benign appearance of the mass on the transvaginal US and the normal CA125 make this diagnosis less likely; persistence of the mass or the development of any other concerning characteristics should prompt surgical excision
The CA125 level should be followed after starting treatment for ovarian cancer to monitor response of the tumor to therapy. Urinary tract infection More thorough discussion in Chapter 4 Why eliminated from differential: the normal UA and appearance of the transvaginal US make this diagnosis unlikely Bladder cancer More thorough discussion in Chapter 4 Why eliminated from differential: the normal UA and appearance of the transvaginal US make this diagnosis unlikely Colon cancer/diverticulitis More thorough discussion in Chapter 3 Why eliminated from differential: the negative Hemoccult test and normal appearance of the colon on the abdominal CT make these diagnoses unlikely
Case 11-8: “I found a lump in my breast” A 47-year-old woman presents to her gynecologist after having felt a lump in her left breast. The patient says that she was showering earlier in the week and noticed a poorly defined lump deep in her breast tissue near her areola. She says that the lump is nontender and that she had never noticed it previously. She denies any discharge from her left nipple or any erythema or lesions overlying the area of the lump. She says that she fully examined both of her breasts after having located this mass and could find no other abnormalities. She admits that she is not very consistent with performing self-examinations and has not performed one for several months. She last had a mammogram 2 years ago. She denies any fevers. She says that her last menses was 3 weeks ago and that she continues to have monthly menstrual periods. She has a past medical history significant for hypothyroidism and takes levothyroxine, a calcium supplement, and vitamin D. She drinks a glass or two of wine each day and denies any other substance use. She says that her mother had ovarian cancer at the age of 53 years that was successfully treated with a TAH-BSO and cisplatin chemotherapy. On examination, she is a well-appearing woman who appears mildly anxious. She has no palpable lymphadenopathy. Auscultation of her lungs and heart is normal. Her abdomen is nontender without any masses. Examination of the left breast detects a deep mass adjacent to the lateral edge of her areola. The mass feels firm and immobile and is approximately 1 cm in diameter. No discharge is able to be expressed from the breast, and the nipple appears normal. The skin overlying the mass is smooth and nonerythematous, and no lesions are noticeable. The mass
is nontender to palpation. The following vital signs are measured: T: 98.5°F, HR: 90 bpm, BP: 130/82 mm Hg, RR: 18 breaths/min
Differential Diagnosis Breast cancer, fibroadenoma, fibrocystic hyperplasia, breast abscess, intraductal papilloma
Laboratory Data and Other Study Results CBC: WBC: 7.1, Hgb: 14.8, Plt: 411 10-electrolyte chemistry panel (Chem10): Na 143 mEq/L, K 3.8 mEq/L, Cl 102 mEq/L, CO2 28 mEq/L, BUN 15 mg/dL, Cr 0.8 mg/dL, Glu 87 mg/dL, magnesium (Mg): 2.0 mg/dL, calcium (Ca): 9.9 mg/dL, phosphorus (Phos): 3.7 mg/dL Mammogram: 12 mm diameter left breast lesion with a few speckled calcifications and poorly defined borders; normal right breast appearance After completion of these tests, the following test is performed: US-guided core biopsy, left breast lesion: dedifferentiated cells in the ductal tissue without invasion into the stroma; scattered microcalcifications; medium grade malignancy HER2/neu protein receptor analysis: negative Estrogen and progesterone receptor analysis: positive
Diagnosis Breast cancer, likely ductal carcinoma in situ
Treatment Administered The patient underwent lumpectomy of the left breast with a satellite lymph node biopsy Radiation therapy was initiated following surgery
Follow-up Surgical pathology demonstrated full excision of the malignant tumor with at least 11 mm margins surrounding the neoplasm; biopsy of the satellite lymph node was normal
Table 11-10 Features of Breast Cancer Variants
V ariant
Charac teristic s
H/P
Ductal carcinoma
Malignant cells in ducts w ithout stromal invasion; possible calcifications;
Usually asymptomatic, possible nipple discharge or
in situ
unifocal; higher risk of subsequent invasive cancer than LCIS
palpable lump
Lobular
Malignant cells in lobules w ithout stromal invasion; no calcifications; may be
Incidental finding, asymptomatic
carcinoma in situ
multifocal; lower risk of invasion than DCIS but inc reased risk of c ontralateral malignanc y
Infiltrating
Malignant cells in ducts with stromal invasion and microcalcifications; fibrotic
Firm palpable mass, skin dimpling, nipple
ductal carcinoma
response in surrounding breast tissue; most c ommon form of invasive
retraction, peau d'orange, or nipple discharge
breast cancer (80% of cases)
Infiltrating
Malignant cells in breast lobules with insidious infiltration and less fibrous
Firm palpable mass, skin dimpling, nipple
lobular carcinoma
response; more frequently bilateral or multifoc al than ductal carcinoma;
retraction, peau d'orange, or nipple discharge;
slower metastasis; greater association with hormone replac ement therapy
may be more subtle than ductal carcinoma
Medullary
Well-circumscribed mass; rapid grow th; better prognosis than ductal
Soft well-circumscribed mass
carcinoma
carcinoma
Mucinous
Well-circumscribed mass; slow grow th; more common in older women; better
carcinoma
prognosis than ductal carcinoma
Tubular
Slow-growing malignancy of well-formed tubular structures invading the
carcinoma
stroma; patients typically in late 40s; excellent prognosis
Gelatinous well-circumscribed mass
Rarely detected prior to mammography
Inflammatory
Subtype of ductal carcinoma characterized by rapid progression and
Breast pain, tender breast, erythema, warmth,
carcinoma
angioinvasive behavior; poor prognosis
peau d'orange, lymphadenopathy
DCIS, ductal carcinoma in situ; H/P, history and physical; LCIS, lobular carcinoma in situ.
The patient completed a protocol of radiation therapy without any complications With yearly monitoring mammograms, the patient remained disease free at 5 years following surgery
Steps to the Diagnosis Breast cancer Malignant neoplasm of breast tissue arising from either ductal or lobular elements (Table 11-10) Ductal neoplasms constitute 80% of cases and tend to be more aggressive Lobular neoplasms constitute 20% of cases, are less aggressive, and tend to be more difficult to detect Risk factors: family history, BRCA1 or BRCA2 gene mutations, ovarian cancer, endometrial cancer, prior breast cancer, excessive estrogen exposure, early age of menarche, late age of menopause, nulliparity, late age of first pregnancy (>35 years old), advanced age, obesity, alcohol use, diethylstilbestrol exposure, industrial chemical or pesticide exposure, radiation exposure History: frequently asymptomatic, although a painless breast lump or nipple discharge may be detected by the patient Physical examination: palpable solid and immobile breast lump, possible nipple retraction or skin dimpling, possible peau d'orange (i.e., breast skin looks similar to an orange peel due to lymphedema and skin thickening), possible axillary lymphadenopathy Tests: Mammography is the principal means of screening for breast cancer and is indicated to assess any palpable lump; lesion asymmetry and speckled calcifications are more consistent with a malignant lesion (Figures 11-5 and 11-6)
Figure 11-5 Approach to the patient with suspected breast cancer. FNA, fine needle aspiration.
Figure 11-6 Comparison of normal and abnormal mammography. (A) A normal mediolateral mammogram showing normal glandular tissue. (B) An abnormal mammogram with a large, dense lesion that was later determined to be malignant.
US may be useful for differentiating solid from cystic lesions and guiding biopsies MRI may be useful to determine the soft tissue extent of the disease CT and bone scintigraphy are used to determine if bone metastases have occurred with invasive tumors Biopsy is indicated for any palpable mass or concerning mammography finding Fine needle aspiration (FNA) may be performed with US-guided localization but is technician dependent and carries an appreciable false-negative rate Core biopsy provides a more definitive histologic diagnosis and is able to determine if a tumor is invasive Open biopsy may be performed with US direction for poorly defined lesions Testing for estrogen and progesterone receptors and HER2/neu protein receptors helps guide treatment for invasive neoplasms Sentinel lymph node biopsy and axillary node sampling performed during surgical resection are important to determining the extent of the disease Treatment: Treatment protocols depend on the type and extent of the tumor
Ductal carcinoma in situ (DCIS) Breast conserving surgery (i.e., lumpectomy) is performed commonly with radiation therapy Sentinel lymph node biopsy and axillary node sampling is frequently performed at the time of surgery Mastectomy may be performed in patients with multiple risk factors Lobular carcinoma in situ (LCIS) May be closely observed because of the lower risk of invasion Tamoxifen and raloxifene may sometimes be administered as chemoprevention Elective bilateral mastectomy is sometimes chosen by women who do not desire lifelong observation Invasive carcinomas
One percent of all cases of breast cancer occur in males.
Risk factors for breast cancer may be remembered by the mnemonic A PLACE WRONGED: Age (older patients), Pesticides, Late menopause, Alcohol, Cancer (prior breast, ovarian, endometrial, or colon), Early menarche, Women, Radiation exposure, Obesity, Nulliparity, Genetics (family history BRCA gene mutations), Estrogen (excess exposure), Diethylstilbestrol exposure.
The majority of breast cancers are detected through an abnormal screening mammogram, but 20% of malignancies are not detected on mammogram (typically in the upper-outer quadrant of the breast).
FNA of a solid breast mass carries a 20% chance of a false negative, so any negative FNA of a solid breast mass requires a more definitive biopsy. Lumpectomy with adjuvant radiation therapy is an option in early tumors with minimal invasion Mastectomy is indicated for multifocal lesions, tumors >5 cm, and in patients with multiple risk factors A positive sentinel node biopsy indicates the need for an axillary lymph node dissection Chemotherapy, hormone therapy (in hormone receptor-positive tumors), or trastuzumab (in HER2/neu receptorpositive tumors) is administered for all cancers with positive lymph nodes, tumors >1 cm, or tumors with high-grade histology Advanced cases Chemotherapy and hormone therapy are utilized for any cancer with extension beyond the breast Debulking surgery with adjuvant radiation therapy is performed to decrease tumor size Chemotherapy is used to treat distant metastases Solitary metastases may be amenable to surgical resection Surgical, radiation, and chemotherapy are all utilized for significantly high-grade tumors such as inflammatory cancers Outcomes:
Tumor size, histologic grade, presence of hormone receptors, presence of BRCA gene mutations, age of patient, and presence of lymph node or distant metastases are all factors that contribute to determining a prognosis The 5-year survival rate for small tumors without positive lymph nodes is >90%, tumors 2 to 5 cm in diameter have a 80%–90% survival, depending on the presence of few or no positive nodes, and tumors >5 cm range from >80% with no positive nodes to ?50% with more than four positive nodes Bone, chest wall, brain, and liver are the most common sites of metastases Lymphedema is a frequent sequela of lymph node dissection and may be complicated by cosmetic disfigurement, impaired wound healing, decreased range of motion, and increased risk of infection Clues to the diagnosis: History: painless breast lump detected incidentally Physical: firm, immobile palpable breast mass Tests: results of the mammogram and biopsy
Patients with BRCA1 or BRCA2 gene mutations should be followed very closely to look for breast or ovarian cancer, and they may want to consider prophylactic mastectomies and oophorectomies if they have multiple other risk factors. Fibroadenoma A benign breast tumor that is caused by a proliferative process in a single duct Most common benign breast tumor More common in women <30 years old History: painless breast mass that may vary in size during the menstrual cycle Physical examination: solitary, solid, and mobile breast mass with well-defined edges Tests: mammography, US, or MRI can determine the location of the mass and will show benign qualities (e.g., well defined, no speckled calcifications); biopsy is confirmative of the diagnosis Treatment: surgical excision or US-guided cryotherapy Outcomes: the prognosis is good, although recurrence is common Why eliminated from differential: in this case the appearance of the lesion on mammography is concerning for malignancy; this concern is confirmed with the biopsy results
Suspicious lesions on mammogram are those with hyperdense regions or calcifications. Fibrocystic hyperplasia An increased number of benign cysts and increases fibrous tissue growth that is found in women of childbearing age The size of the tissue varies during the menstrual cycle History: breast pain preceding menses that improves afterwards, multiple small breast masses that vary in size
during the menstrual cycle Physical examination: multiple bilateral small tender breast masses that are mobile Tests: mammography demonstrates a benign appearance of the involved tissue; biopsy performed on atypical lesions will show epithelial hyperplasia Treatment: caffeine and dietary fat reduction, oral contraceptives, progesterone, or tamoxifen may improve symptoms in confirmed benign lesions Outcomes: the prognosis is excellent with no expected complications Why eliminated from differential: the appearance of the lesion on mammography and the biopsy results rules out this diagnosis
Monthly breast self-examinations after each menstrual period are the best way to distinguish developing lesions from monthly variations in breast tissue makeup, but have not been shown to decrease mortality.
Perform yearly mammography after the age of 40 years to screen for the development of concerning breast lesions. Breast abscess A local infection of breast tissue due to S. aureus, streptococcus, or anaerobic bacteria Risk factors: breast feeding, smoking History: painful breast mass Physical examination: fever, warm and erythematous breast mass, breast tenderness, purulent drainage from the mass or the nipple Tests: Increased WBCs, ESR, and CRP US localizes a cystic breast mass Aspiration of the abscess is used for culture and confirms the presence of purulent material Treatment: Incision and drainage of the abscess is performed unless it is extremely small Agent-appropriate oral or IV antibiotics Breast feeding may be continued during treatment Recurrent abscesses may require further debridement and excision of any fistula tracts Outcomes: although patients recover well with the appropriate treatment, recurrence is common Why eliminated from differential: the appearance of the lesion in this case is not consistent with an infectious process; the biopsy further rules out this diagnosis Intraductal papilloma Benign lesions of ductal tissue that may carry a malignant potential
History: breast pain, bloody or nonbloody nipple discharge Physical examination: palpable tender breast mass behind the areola, nipple discharge may be expressed with stimulation Tests: excisional biopsy is performed to diagnose and treat the lesion; ductal lavage may be performed using a microcatheter to look for abnormal intraductal cells on histology Treatment: surgical excision Outcomes: the prognosis is excellent with excision, cases with a greater degree of cell atypia may be at higher risk for ductal carcinoma Why eliminated from differential: the absence of nipple discharge and the biopsy results rule out this diagnosis
Authors: Van Kleunen, Jonathan P. Title: Step-Up to USMLE Step 3, 1st Edition Copyright ©2009 Lippincott Williams & Wilkins > Table of Contents > Chapter 12 - Obstetrics
Chapter 12 Obstetrics Basic clinical primer Maternal Physiology in Pregnancy Fetal development Following fertilization of the ovum, the newly created zygote begins mitotic cell division Implantation of the zygote in the endometrium occurs approximately 6 days following fertilization, and the placenta begins to develop as the supporting conduit for maternal-fetal circulation Fetal maturity occurs through a developmental process taking approximately 38 weeks (Figure 12-1) Changes in maternal physiology Physiologic changes occur in every maternal organ system in response to the need to maintain fetal viability (Table 12-1)
Prenatal Care Nutrition Maternal nutrition demands change with the beginning of pregnancy in order to fully support both the mother and the developing fetus (Table 12-2) Some nutrients are important to reduce the risk of birth defects (e.g., folate, iron) The ideal maternal weight gain during pregnancy is dependent on the mother's preexisting body mass index (BMI) (Table 12-3) Consumption of fish (due to methylmercury contamination) and caffeine (due to an increased risk of spontaneous abortion) should be limited during pregnancy
Daily caloric intake during pregnancy should be approximately 2,500 kcal.
Exercise is encouraged during pregnancy to improve maternal feelings of well-being, improve symptoms due to positional effects of the fetus, and promote healthy blood sugar levels.
Work and travel may be continued throughout pregnancy as long as fatigue and excessive stress are avoided; airline travel is permitted up to 36 weeks of gestation.
Sexual intercourse may be continued during pregnancy unless the mother is considered a high risk for spontaneous abortion, premature labor, or placenta previa. Prenatal obstetrics care Thorough prenatal care should be considered a vital component of pregnancy because its goals are to prevent or manage any conditions that may be harmful to the mother or fetus The initial prenatal visit includes a detailed history, physical, and risk assessment Maternal weight, urinalysis (to detect urinary tract infections [UTI] and gestational diabetes mellitus [DM]), blood pressure, fundal height (to estimate fetal growth), and fetal heart sounds (to confirm fetal viability) are evaluated at each visit Patients should be educated about weight gain, nutrition, drug and substance abstinence, animal handling or avoidance, seat belt use, symptoms and signs for risks to the mother or fetus, scheduling of care and tests, childbirth and breast feeding classes, and confidentiality issues Labs and ultrasound (US) are performed at certain time points during gestation to screen for infection and fetal abnormalities (Table 12-4) Screening tests that are not routinely performed at the first prenatal visit but should be considered in at-risk populations include a purified protein derivative (PPD) (for P.274 P.275 P.276 tuberculosis [TB]), red blood cell (RBC) indices and hemoglobin electrophoresis (for anemias), hexosaminidase A (for Tay-Sachs disease), phenylalanine levels (for phenylketonuria), hepatitis C serology, toxoplasmosis screening, and cystic fibrosis genetic screening
Figure 12-1 Time line of fetal development during gestation. CNS, central nervous system; hCG, human chorionic gonadotropin.
Table 12-1 Normal Changes in Maternal Physiology during Pregnancy
Anatomy /Sy stem
Cardiovascular
Changes
Cardiac output increases 40% with associated increases in SV (10%–30%) and HR (12–18 bpm) A systolic murmur may be heard due to the increased cardiac output Myocardial O 2 demand increases Systolic and diastolic blood pressures decrease slightly The uterus displaces the heart slightly superiorly
Respiratory
The uterus displaces the diaphragm superiorly and causes a decrease of residual volume, functional residual capacity, and expiratory reserve volume Total body O 2 consumption increases 20% Tidal volume increases 40% with an associated increase in minute ventilation due to stimulation by progesterone PCO 2 decreases to approximately 30 mm Hg; dyspnea is a frequent complaint despite the increased minute ventilation and normal breathing rate
Renal
Renal plasma flow and the glomerular filtration rate increase 40% Decrease in BUN and creatinine Increased renal loss of bicarbonate to compensate for respiratory alkalosis Blood and interstitial fluid volumes increase
Endocrine
Nondiabetic hyperinsulinemia with an associated mild glucose intolerance Production of human placental lactogen contributes to the glucose intolerance by interfering with insulin activity Fasting triglycerides increase Cortisol increases
Hematologic
Increased RBC production Hematocrit decreases due to increased blood volume Hypercoagulable state
Gastrointestinal
Increased salivation Decreased gastric motility
BUN, blood urea nitrogen; HR, heart rate; O 2, oxygen; PCO 2, partial pressure of carbon dioxide; RBC, red blood cell; SV, stroke volume.
Table 12-2 Increased Nutritional Demands during Pregnancy
Substanc e
Calcium
Inc reased Need
1,000–1,300 mg/day (50% increase)
Reason for Need
Effec ts of Insuffic ienc y
Lactation reserves, increased
Impaired maternal bone mineralization, HTN,
utilization by the fetus
premature birth, low birth weight
Fluids
Adequate hydration is important
Increased total maternal-fetal
Relative dehydration
fluid volume
Folate
Iron
0.8–1 mg/day (should be started 4 weeks
Normal fetal neural tube
before attempted conception)
development
30 mg/day (100% increase)
RBC production
Neural tube defects
Maternal anemia, premature birth, low birth weight, maternal cardiac complications
Protein
60 g/day (30% increase)
Additional needs of maternal,
Impaired fetal and placental growth
fetal, and placental tissue
HTN, hypertension; RBC, red blood cell.
Table 12-3 Ideal Weight Gain during Pregnancy
Prepregnant BMI
Ideal W eight Gain (lb)
<19.8
28–40
19.8–26
25–35 1
>26
15–25
BMI, body mass index; lb, pounds. 1
Weight gain should be approximately 2 lb in the first trimester and 0.75 to 1 lb/week in the remainder of the pregnancy.
Table 12-4 Common Screening Labs Performed during Pregnancy
Length of Gestation
Initial visit
Labs or Study Performed
CBC Blood antibody and Rh typing Pap smear Gonorrhea/chlamydia screening Urinalysis RPR or VDRL Rubella antibody titer Hepatitis B surface antigen HIV screening (with maternal consent)
6–11 weeks
US to determine gestational age
16–18 weeks
Quadruple screen (maternal serum α-fetoprotein, hCG, unconjugated estradiol, maternal serum inhibin A) to look for trisomies 21 and 18 and neural tube defects
18–20 weeks
24–28 weeks
32–37 weeks
US for detecting gross fetal and placental abnormalities
1-hour glucose challenge to screen for gestational DM
Cervical culture for Neisseria gonorrhoeae and Chlamydia trachomatis (selected populations) Group B streptococcus screening
CBC, complete blood count; DM, diabetes mellitus; hCG, human chorionic gonadotropin; Pap, Papanicolaou; PPD, purified protein derivative of tuberculin; Rh, rhesus factor; RPR, rapid plasma reagin; US, ultrasound; VDRL, Venereal Disease Research Laboratories.
Table 12-5 Prenatal Assessment for Congenital Diseases in High-Risk Pregnancies
Test
Desc ription
Indic ations
Maternal serum α-fetoprotein, estradiol, hCG, and maternal serum inhibin
Performed in all pregnant women between
A levels are measured to assess the risk for neural tube defects and
16–18 weeks of gestation; frequently the
trisomies 18 and 21
initial marker for fetal complications
US measurement of nuchal translucency and serum measurement of
Women who present in the first trimester who
PAPP-A in the first trimester and a quadruple screen in the second
desire noninvasive testing with the lowest
trimester; lowest false-positive rate for noninvasive tests
false-positive risk
Transabdominal needle aspiration of amniotic fluid from the amniotic sac
Abnormal quadruple screen, women >35 years
after 16 weeks of gestation to measure amniotic-fetoprotein and
old, risk of Rh sensitization; carries an excess
determine the fetal karyotype; detects neural tube defects and
1% risk of spontaneous abortion over the
chromosome disorders with greater sensitivity than triple screen alone
normal risks for abortion
Chorionic villi
Transabdominal or transcervical aspiration of chorionic villus tissue
Early detection of chromosomal abnormalities
sampling
between 912 weeks of gestation to detect chromosomal abnormalities
in higher-risk patients (e.g., advanced age,
Quadruple screen
Full integrated test
Amniocentesis
history of children with genetic defects)
Percutaneous
Blood collection from the umbilical vein after 18 weeks of gestation to
Late detection of genetic disorders,
umbilical blood
identify chromosomal defects, fetal infection, Rh sensitization
pregnancies with high risk for Rh sensitization
sampling
hCG, human chorionic gonadotropin; PAPP-A, pregnancy-associated plasma protein A; Rh, rhesus factor.
Women considered to be at a higher risk for congenital fetal defects (e.g., older than 35 years, prior spontaneous abortion, teratogen exposure, DM, prior fetal demise) may be screened for fetal complications using more complex or invasive testing (Table 12-5) Leopold maneuvers (i.e., external abdominal examination) are performed in the third trimester to determine the fetal presentation
Maternal serum α-fetoprotein levels: Only valid if performed during the correct gestational window (16 to 18 weeks' gestation) High levels are associated with an increased risk of neural tube defects or multiple gestation Low levels are associated with increased risk of trisomies 21 and 18.
In trisomy 18 all quadruple screen markers are low except inhibin A. In trisomy 21 maternal serum α-fetoprotein and unconjugated estradiol are low, while human chorionic gonadotropin (hCG) and inhibin A are high.
Low maternal pregnancy-associated plasma protein A (PAPP-A) levels are associated with an increased risk of trisomies 21 and 18.
Labor and Delivery Assessment of fetal well-being Tests of fetal activity, heart rate, and responses to stress are used to confirm fetal well-being and to detect fetal distress Nonstress test Performed during the prenatal assessment and in early labor The mother reclines in the left lateral decubitus position and the fetal heart rate is monitored externally; uterine contractions are also monitored externally The mother reports each fetal movement, and the effects of fetal movement on the fetal heart rate are noted A normal (i.e., “reactive”) test is considered to be two or more 15-beat-per-minute accelerations of the fetal heart rate that each last 15 seconds during a 20-minute monitoring period A vibroacoustic stimulator can be applied to the mother's abdomen to encourage fetal activity and shorten the time of the test A nonreactive test should prompt the performance of a biophysical profile Biophysical profile A nonstress test is repeated with the addition of an US assessment A transabdominal US is performed to measure the amniotic fluid index (i.e., total linear measurement in centimeters of the largest amniotic fluid pocket P.277 detected in each of four quadrants of the amniotic sac), fetal breathing rate, fetal movement, and fetal tone (i.e., active extension of the fetal spine or limb with a return to flexion)
Table 12-6 Scoring of the Biophysical Profile
Measurement
Sc ore 1
Criteria
Nonstress test
Reactive
0 or 2
Amniotic fluid index
5–23 cm
0 or 2
Fetal breathing rate
One or more episodes of rhythmic breathing lasting at least 20 seconds in a 30-minute time period
0 or 2
Fetal movement
Two or more discrete spontaneous movements in a 30-minute time period
0 or 2
Fetal tone
One or more episodes of spine and limb extension with a return to flexion
0 or 2
1
Satisfaction of the criteria equals a score of 2; failure to satisfy the criteria equals a score of 0 (no scores of 1).
A scoring system is applied to the nonstress test and the US measurements (Table 12-6) A reassuring profile is a score of 8 or 10 and suggests a minimal risk of fetal asphyxia; a lower score suggests fetal distress Contraction stress test Performed late in pregnancy or during labor to assess uteroplacental dysfunction The fetal heart rate is recorded with an external fetal monitor or scalp electrode Beat-to-beat variability of approximately 5 beats per minute, long-term heart rate variability, and occasional heart rate accelerations (i.e., two or more accelerations of 15 beats per minute lasting at least 15 seconds within a 20-minute period) are reassuring signs Decelerations of the fetal heart rate and their relationship to uterine contractions may be indicative of normal activity during labor or possible fetal distress (Figure 12-2, Table 12-7) Fetal scalp blood sampling Performed when a consistently abnormal fetal heart rate tracing is measured or when a significant amount of meconium is detected in the amniotic fluid A normal fetal blood pH is reassuring; a decreased pH, hypoxemia, or increased lactate level indicates fetal distress Fetal scalp monitoring A monitor is attached to the fetal scalp to track pulse oximetry and to perform continuous fetal heart rate monitoring and an electrocardiogram (ECG)
Table 12-7 Types of Decelerations Seen on Fetal Heart Rate Tracings
Dec eleration
Appearanc e
Cause
Treatment
Early
Decelerations begin and end w ith
Head compression
uterine contractions
Late
Variable
None required; not a sign of fetal distress
Begin after the initiation of
Uteroplac ental insuffic ienc y,
Test fetal blood from a scalp sample
uterine contraction and end after
maternal venous compression,
to diagnose hypoxia or acidosis;
a contraction has finished
maternal hypotension, or abruptio
recurrent late decelerations or
placentae; may suggest fetal
fetal hypoxia direct a prompt
hypoxia
delivery
Umbilical cord compression
Change the mother's position
Inc onsistent onset, duration, and diagnose hypoxia or acidosis; recurrent late decelerations or fetal hypoxia direct a prompt delivery
P.278
Figure 12-2 Examples of fetal heart and uterine tone tracings for early, late, and variable decelerations of the fetal heart rate (FHR) following uterine contraction (UC).
Monitoring should be started in the first stage of labor to determine an accurate baseline ECG It should only be performed in pregnancies beyond 36 weeks of gestation with a vertex presentation to avoid tangling of the wires around the fetus
Although the full integrated test is the most sensitive screening test for trisomies with the lowest false-positive rate, it is not routinely performed because abnormal first trimester results frequently prompt the decision to abandon the second trimester tests and perform a more invasive test to find a definitive answer.
Normal fetal heart rate is 120 to 160 beats per minute. Stages of labor Labor (i.e., contractions and cervical effacement) typically begins at 38 to 42 weeks of gestation and involves four stages of progression (Table 12-8) Nulliparous and multiparous women proceed through labor at different rates Induction of labor Oxytocin is administered to initiate uterine contractions or to speed the progress of labor Indications include pre-eclampsia, DM, stalled stage of labor, chorioamnionitis, prolonged pregnancy (i.e., >40 to 42 weeks), intrauterine growth restriction (IUGR), premature rupture of membranes (PROM), and some congenital defects P.279
Table 12-8 Stages of Labor
Stage
Beginning/End
1
2
3
Ac tivity
Latent phase—start
Latent phase—cervical
of uterine
effacement and gradual
contractions until 4
dilation
cm cervical dilation
Active phase—regular
Active phase—4 cm
uterine contractions, quick
cervical dilation
progression of cervical
until near 10 cm
dilation (1.5 cm/hr if
cervical dilation
nulliparous, 1 cm/hr if
with consistent
multiparous) and
progression
effacement
Deceleration phase—
Deceleration phase—slow
transition from
down of dilation and
active phase to
effacement shortly before
second stage of
engagement of fetal head in
labor
pelvis
Management
Nulliparous
Monitor fetal heart
6–20 hr
rate and uterine
(latent,
contractions, assess
active)
Multiparous
2–10 hr
progression of cervical changes periodically during active phase
Full (10 cm) cervical
Fetal descent through the birth
Monitor fetal heart
30 min–3
dilation until delivery
canal driven by uterine
rate and movement
hr
contractions
through birth canal
Delivery of neonate until
Placenta separates from uterine
Uterine massage,
placental delivery
wall up to 30 minutes after
examination of
delivery of neonate and emerges
placenta to confirm no
through birth canal, uterus
intrauterine remnants
5–30 min
0–30 min
0–30 min
1 hr
1 hr
contracts to expel placenta and prevent hemorrhage
4
Initial postpartum hour
Hemodynamic stabilization of
Monitor maternal pulse
mother
and blood pressure, look for signs of hemorrhage
Contraindications to induction include prior uterine surgery, fetal lung immaturity, malpresentation, acute fetal distress, active maternal herpes infection, placenta or vasa previa The likelihood of vaginal delivery following induction is predicted by measuring fetal station and cervical dilation, effacement, consistency, and position (i.e., Bishop score) (Table 12-9) A higher Bishop score is associated with a greater chance of vaginal delivery, while a lower score is associated with a higher likelihood of cesarian delivery The rate of cesarian delivery is 30% if the Bishop score following induction is <3 and 15% if >3
Table 12-9 Bishop Scoring System
Sc ore
0
1
2
3
Dilation (cm)
0
1–2
3–4
5–6
Effacement (%)
0–30
40–50
60–70
>70
Station (cm)1
-3
-2
-1, 0
Cervical Consistency
Firm
Medium
Soft
Cervical Position
Posterior
Middle
Anterior
1
+1, +2
Distance of presenting body part above (-) or below (+) ischial spines.
(Taken from Bishop, Obstet Gynecol, 1964.)
During the last few weeks of gestation a woman may experience multiple false (i.e., Braxton-Hicks) contractions not associated with true labor. P.280 Cesarian section Surgical delivery of the fetus through an incision in the uterine wall Vertical type A vertical incision is made in the anterior portion of the uterus (i.e., classical) or lower uterine segment (i.e., low vertical) Utilized when the fetus lies in a transverse presentation, if adhesions or fibroids prevent access to the lower uterus, if hysterectomy is scheduled to follow delivery, if cervical cancer is present, or in a postmortem delivery to remove a living fetus from a mother who has just died Low transverse type A transverse incision is made in the lower anterior portion of the uterus It carries a decreased risk of uterine rupture, bleeding, bowel adhesions, and infection and is preferred to and performed more commonly than the vertical type Indications include eclampsia, prior uterine surgery, prior classic cesarean section, maternal cardiac disease, birth canal obstruction, maternal death, cervical cancer, active maternal herpes simplex infection, acute fetal distress, malpresentation, umbilical cord prolapse, macrosomia, failure to progress in labor, placenta previa, abruptio placentae, and cephalopelvic disproportion In subsequent pregnancies, vaginal delivery may be attempted only if a transverse cesarean section was performed If a vertical incision has been utilized previously, repeat cesarean delivery must be performed due to an increased risk of uterine rupture during attempted vaginal delivery Potential complications include maternal hemorrhage, infection and sepsis, thromboembolism, and injury to surrounding structures
Risk of maternal mortality is similar for elective cesarean section and vaginal delivery, but emergency cesarian section carries a higher risk of mortality.
Normal Puerperium and Postpartum Activity
Care of the newborn Following delivery, the neonate's mouth and nose are suctioned to aid in breathing and to prevent aspiration The neonate is dried and wrapped in a blanket to prevent heat loss The umbilical cord is clamped and cut, and a cord blood sample is used to measure blood type and blood gases If spontaneous respiration does not begin within 30 seconds of birth, resuscitation must be initiated Tracheal injection of a synthetic or exogenous surfactant may be administered in cases of lung immaturity Apgar scores are performed at 1 and 5 minutes after birth (Table 12-10) Scores of at least 7 at 1 minute and 9 at 5 minutes are reassuring Maternal changes after delivery The uterus decreases in size, and the cervix becomes firm over 3 weeks; the vaginal wall also gradually becomes more firm
Table 12-10 Apgar Scoring System for Determining Neonatal Well-Being
Sc ore
Sign
0
1
2
Heart rate
None
<100 bpm
>100 bpm
Respirations
None
Poor, weak cry
Good, strong cry
Muscle tone
Poor
Some movement
Active movement
Response to stimulation
None
Grimace
Strong cry
Color
Blue, pale
Pink torso, blue extremities
Pink
bpm, beats per minute. (Taken from Apgar, Curr Res A nesth A nalg, 1953.)
P.281 Uterine discharge (i.e., lochia) is red during the initial days after birth but becomes paler and white by the 10th postpartum day Total peripheral resistance increases rapidly due to the elimination of uteroplacental circulation, diuresis of the excess fluid volume causes a significant weight loss in the first postpartum week, and cardiac output gradually returns to normal Postpartum depression is common for a few days after delivery, but most cases resolve without complications Menses return 6 to 8 weeks following delivery in nonnursing mothers, but may not occur for several months in nursing mothers
Breast milk is considered the ideal infant nutrition because it contains important immunoglobulin (Ig) A antibodies for the newborn, is in sufficient supply, is cost-free, and enhances mother-infant bonding.
Early breast milk (i.e., colostrum) is rich in proteins, fat, minerals, and contains IgA; after 1 week postpartum breast milk contains primarily proteins, fat, water, and lactose.
Case 12-1: “Does pregnancy give you bad headaches?” A 26-year-old woman, who is G1P0000 (i.e., one pregnancy, no prior deliveries, premature births, spontaneous abortions, or current living children), presents to her obstetrician at 34 weeks of gestation for a scheduled prenatal visit. She says that she has developed fairly significant headaches within the past week. She gets these headaches on a daily basis. They are dull and poorly defined, and she describes the severity as a 4/10. She claims that she has slight tunnel vision when the headaches are at their worst. The headaches last for a few hours and then self-resolve. She has also experienced epigastric abdominal pain and nausea and has vomited four times this past week. Her emesis has not been bilious or bloody. She denies any numbness or weakness in her extremities. She denies having had any seizures at any point in her life including this past week. She denies any vaginal bleeding. She denies any significant past medical history. She is taking prenatal vitamins but no other medication. She denies any substance use. On examination, she is a well-appearing woman in no distress. Her face and eyes appear symmetric, not swollen, and not inflamed. Auscultation of her heart and lungs is normal. Her abdomen is gravid and nontender. Pelvic examination detects a closed cervical os with no discharge and no cervical tenderness. A neurovascular examination is normal. She has some mild swelling of her hands. She has no rashes. Her fundal height is 32 cm, and a regular fetal heart rate is easily detected. The following vital signs are measured: Temperature (T): 98.7°F, heart rate (HR): 90 beats per minute (bpm), blood pressure (BP): 165/90 mm Hg, respiratory rate (RR): 18 breaths/min
Differential Diagnosis Eclampsia, pre-eclampsia, hyperemesis gravidarum, abruptio placentae, migraine headache, tension headache, cerebral vascular accident
Laboratory Data and Other Study Results Height and weight: 5﹐ 6" 159 lb (initial weight 140 lb, BMI 22.6) Complete blood cell count (CBC): white blood cells (WBC): 6.4, hemoglobin (Hgb): 12.5, platelets (Plt): 189 7-electrolyte chemistry panel (Chem7): sodium (Na): 142 mEq/L, potassium (K): 4.1 mEq/L, chloride (Cl): 104 mEq/L, carbon dioxide (CO2): 28 mEq/L, blood urea nitrogen (BUN): 17 mg/dL, creatinine (Cr): 1.0 mg/dL, glucose (Glu): 105 mg/dL Liver function tests (LFTs): alkaline phosphatase (AlkPhos): 87 U/L, alanine aminotransferase (ALT): 71 U/L, aspartate aminotransferase (AST): 50 U/L, total bilirubin (TBili): 0.9 mg/dL, direct bilirubin (DBili): 0.2 mg/dL Urinalysis (UA): straw colored, pH: 7.1, specific gravity: 1.025, 2+ protein, no glucose/ketones/nitrites/leukocyte esterase/hematuria Biophysical profile: score of 8 (fetal tone criteria not satisfied); intact placenta seen on US
Diagnosis Pre-eclampsia P 282
Treatment Administered The patient was admitted to the labor and delivery floor Hydralazine and magnesium sulfate were administered to control the patient's blood pressure and to serve as seizure prophylaxis The patient was permitted restricted activity but was maintained on constant cardiac and external fetal monitoring
Follow-up The patient's blood pressure was reduced to 125/80, and no seizures occurred Delivery was induced once the patient reached 37 weeks of gestation, and an uncomplicated vaginal delivery ensued Medications were continued and weaned over 3 weeks after delivery to ensure that seizures did not occur and that the patient's blood pressure returned to the normal levels
In the GP notation used to describe pregnant women, the number following “G” notes the historical number of gestations in the patient and the four numbers following “P” refer to the number of deliveries (any type), premature births, spontaneous abortions, and living children.
Steps to the Diagnosis Pre-eclampsia Pregnancy-induced hypertension (HTN), proteinuria, and edema that develops after 20 weeks of gestation Occurs in 5% of pregnancies and is due to an unknown cause HELLP syndrome is a subtype of pre-eclampsia characterized by hemolysis, elevated liver enzymes, and low platelets (note the mnemonic!) Risk factors: HTN, nulliparity, prior history of pre-eclampsia, younger than 15 years or older than 35 years, multiple gestations, vascular disease, chronic renal disease, DM, obesity, African heritage History: although it is asymptomatic in mild cases, patients may experience rapid weight gain, headaches, epigastric abdominal pain, and visual disturbances as the severity increases Physical examination: hand and face edema, hyperreflexia, blood pressure >140/90 Tests: UA will demonstrate 2+ proteinuria or >4 g protein in a 24-hour collection Decreased platelets, normal or mildly increased creatinine, decreased glomerular filtration rate, increased ALT and AST A fetal nonstress test should be performed to confirm fetal well-being Treatment: If at 37 weeks of gestation or longer, induce delivery If the patient is at <37 weeks of gestation and has mild symptoms, prescribe restricted activity and
re-examinations, proteinuria assessments, and fetal nonstress tests twice per week If the patient is at <37 weeks of gestation and has significant symptoms, admit the patient to the hospital for inpatient fetal and maternal health monitoring, maintain the maternal blood pressure at <155/105 with hydralazine or labetalol, administer intravenous (IV) magnesium sulfate as seizure prophylaxis, and induce delivery as soon as the fetus is considered viable Antihypertensives and magnesium sulfate should be continued postpartum until the patient remains consistently normotensive Patients with preexistent HTN should be treated with labetalol, hydralazine, or a long-acting calcium channel blocker to keep their blood pressure <140/95 Outcomes: Potential complications include eclampsia with seizures, cerebral vascular accident, IUGR, pulmonary edema, maternal organ failure, oligohydramnios, and preterm delivery HELLP syndrome is associated with abruptio placentae, renal insufficiency, encephalopathy, and disseminated intravascular coagulation (DIC); it is associated with a 1% maternal mortality P.283 The recurrence rate is 5% in patients with mild symptoms and up to 70% in patients with severe symptoms Clues to the diagnosis: History: headaches, epigastric pain Physical: HTN, hand edema Tests: proteinuria, increased ALT and AST
Findings suggestive of pre-eclampsia may be remembered by the mnemonic HELP HER: HTN, Edema (hands and face), Liver enzymes increased, Proteinuria, Headache, Eye disturbances, Renal impairment (decreased glomerular filtration rate).
The only definitive cure for pre-eclampsia is delivery.
Do not discontinue antihypertensives postpartum in patients with pre-eclampsia until the patient remains normotensive because seizures may occur up to a month following delivery.
ACE-Is are contraindicated in the treatment of HTN during pregnancy because of the risk of teratogenic effects. Eclampsia The progression of pre-eclampsia, leading to maternal seizures that may be severe and fatal for both the mother and fetus if not treated promptly History: similar to pre-eclampsia with the addition of seizure activity Physical examination: similar to pre-eclampsia Tests: similar to pre-eclampsia
Treatment: Delivery is the definitive treatment, and the mother should be induced following the conclusion of any seizures and when she is medically stable Magnesium sulfate with phenytoin (short-term use only) and diazepam as second-line agents should be used to control seizures Supplemental oxygen should be supplied to the patient, and blood pressure should be carefully controlled Magnesium sulfate and antihypertensives must be continued for at least 48 hours postpartum because 25% of seizures occur within 24 hours following delivery Outcomes: The maternal and fetal mortality rates are 1% and 12%, respectively There is up to a 70% risk of pre-eclampsia and 2% risk of eclampsia in future pregnancies More than half of mothers who have eclampsia will have temporary neurologic deficits following seizures Why eliminated from differential: the absence of seizures in this patient rules out this diagnosis
Patients with preexisting epilepsy who become pregnant should be treated with a single anticonvulsant, valproic acid and carbamazepine should be avoided because of the increased teratogenic risks, and supplemental vitamin K and folate should be administered to reduce the risk of teratogeny. Hyperemesis gravidarum Severe persistent nausea and vomiting in pregnancy that leads to dehydration, weight loss, and electrolyte abnormalities The cause is poorly understood but likely involves a combination of psychological, hormonal, and gastrointestinal factors History: persistent nausea and vomiting, dizziness, fatigue Physical examination: signs of dehydration (e.g., dry mucous membranes, decreased skin turgor, altered mental status) may be present Tests: UA will show the presence of ketones Sodium, potassium, BUN, and creatinine abnormalities are common; ALT and AST are increased Increased T4 and decreased thyroid-stimulating hormone (TSH) are common Treatment: Eating small frequent meals and avoiding foods that are spicy or high in fat may decrease nausea Adequate hydration must be maintained to avoid dehydration Antiemetics (e.g., promethazine), antihistamines, and vitamin B6 have been shown to improve symptoms in severe cases IV correction of dangerous electrolyte abnormalities should be performed in an inpatient setting Outcomes: complications include seizures, encephalopathy, renal failure, pancreatitis, and hepatic
dysfunction; symptoms rarely last beyond the first trimester Why eliminated from differential: the gestational age of the patient and the presence of significant HTN make this diagnosis unlikely
Although the majority of pregnant women have nausea and vomiting in the first trimester, it does not lead to the volume depletion and electrolyte abnormalities seen in hyperemesis gravidarum. Abruptio placentae More thorough discussion in later case Why eliminated from differential: the absence of vaginal bleeding and the appearance of the placenta on the US rule out this diagnosis P.284 Migraine/tension headache More thorough discussion in Chapter 7 Why eliminated from differential: the presence of significant HTN and hand edema make these diagnosis less likely Cerebral vascular accident More thorough discussion in Chapter 7 Why eliminated from differential: the normal neurologic examination reduces the concern for this diagnosis, and computed tomography (CT) should be avoided, if possible, during pregnancy; the development of more significant neurologic findings or worsening headaches would necessitate a more significant work-up
Case 12-2: “I'm here for my normal prenatal appointment” A 34-year-old woman, who is G4P2012 (i.e., four pregnancies, two deliveries, one spontaneous abortion, two living children), presents to her obstetrician at 26 weeks of gestation for a scheduled prenatal visit. All of her prenatal examinations and tests have been normal in her previous visits. The patient says that she has been doing relatively well. She says that she occasionally becomes fatigued. She denies any headaches, visual disturbances, numbness, or weakness. She says that the occasional nausea and vomiting that she had earlier in the pregnancy has completely resolved. She says that she feels minimally out of breath after prolonged walking but usually has no problems breathing. She reports urinating multiple times per day without any significant urgency. She has mild swelling in her feet and ankles after standing for prolonged periods of time. She walks two miles four times per week. She has continued to work as a producer for a local television news program without any difficulty other than occasional fatigue. She is taking prenatal vitamins and supplemental folate, calcium, and iron. She has no significant past medical history. She denies any substance use. On examination, she is a well-appearing woman in no distress. Her face is symmetrical and nonedematous. She has no lymphadenopathy. Auscultation of her heart and lungs detects normal breath sounds and a soft systolic murmur. Her abdomen is gravid and nontender with normal bowel sounds. A pelvic examination finds a closed cervical os, no cervical discharge, and no cervical or adnexal tenderness. Her fundal height is 26 cm, and fetal heart sounds are detectable with a Doppler. She has minimal edema in her feet. Her neurovascular examination is normal. The following vital signs are measured: T: 98.7°F, HR: 87 bpm, BP: 105/70 mm Hg, RR: 17 breaths/min
Laboratory Data and Other Study Results Height and weight: 5﹐ 7" 143 lb (initial weight 132 lb, BMI 20.7)
UA: light yellow colored, pH: 7.4, specific gravity: 1.000, trace glucose, no ketones/nitrites/leukocyte esterase/hematuria/proteinuria 1-hour glucose tolerance test: 172 mg/dL Following receipt of these results, the following test is ordered: 3-hour glucose tolerance test: fasting glucose: 127 mg/dL, 1-hour glucose: 173 mg/dL, 2-hour glucose: 159 mg/dL, 3-hour glucose: 147 mg/dL
Differential Diagnosis Gestational diabetes mellitus, preexisting diabetes mellitus
Diagnosis Gestational diabetes mellitus
Treatment Administered The patient was placed on a 2,400 kcal/day diet and encouraged to continue her exercise regimen P.285
Follow-up Repeat glucose testing 1 week following initiation of the diet detected a fasting glucose of 113 mg/dL and a 1-hour postprandial glucose of 153 mg/dL A low-dose insulin regimen with daily self-monitoring of glucose levels was initiated Following the start of insulin therapy, the patient was able to maintain a fasting glucose of <90 mg/dL The patient's pregnancy proceeded with no further events, and she was able to have an uncomplicated vaginal delivery of a healthy child; her glucose levels remained normal off her insulin regimen following her recovery
Steps to the Diagnosis Gestational diabetes mellitus New onset glucose intolerance that begins during pregnancy Risk factors: family history of DM, older than 25 years, obesity, prior polyhydramnios, recurrent abortions, prior stillbirth, prior macrosomia, HTN, African or Pacific Islander heritage, corticosteroid use, polycystic ovary syndrome (PCOS) History: usually asymptomatic Physical examination: there are typically no significant physical findings for new onset DM Tests: fasting glucose >126 mg/dL or an abnormal 1-hour glucose tolerance test performed between 24 and 28 weeks of gestation; any abnormal 1-hour glucose tolerance test should indicate the need for a 3-hour glucose tolerance test (Figure 12-3) Treatment: The initial treatments are directed at control through diet and exercise Women with a prepregnant BMI of <22 should be placed on a 40 kcal/kg/day diet
Women with a prepregnant BMI of 22 to 27 should be placed on a 30 kcal/kg/day diet Women with a pre-pregnant BMI of 27 to 29 should be placed on a 24 kcal/kg/day diet Women with a pre-pregnant BMI of >29 should be placed on a 12 to 15 kcal/kg/day diet Patients who continue to have abnormal fasting and 1-hour postprandial glucose levels 1 week after initiating the decreased caloric diet should be started on insulin therapy Self-monitoring of glucose levels is prescribed to maintain tight control and to keep fasting levels below 90 mg/dL and 1-hour postprandial levels below 120 mg/dL Periodic fetal nonstress tests and US should be performed to assess fetal well-being Outcomes: Several complications of pregnancy are associated with gestational diabetes including polyhydramnios, pre-eclampsia, renal insufficiency, diabetic ketoacidosis (DKA), hyperosmolar hyperglycemic nonketotic coma (HHNC), retinopathy, fetal hypoglycemia, macrosomia, IUGR, neural tube defects, fetal cardiac defects, and intrauterine fetal demise Macrosomia is associated with increased rates of birth trauma and cesarian delivery Mothers who develop gestational DM are at an increased risk of developing nongestational DM later in life; 70% of women requiring insulin during pregnancy will develop DM during their lifetimes Children from a pregnancy complicated by gestational DM are at a higher risk of being obese during childhood or adulthood Clues to the diagnosis: History: noncontributory (gestational DM is typically asymptomatic, and the patient's minor complaints are typical for pregnancy) Physical: noncontributory Tests: presence of glucose in the UA, results of both the 1-hour and 3-hour glucose tolerance tests P.286
Figure 12-3 Screening for gestational diabetes mellitus performed at 24 to 28 weeks of gestation using the glucose tolerance tests.
Diabetes mellitus (preexisting) More thorough discussion in Chapter 5 Why eliminated from differential: the absence of glucose intolerance in the patient's past medical history and the normal prior prenatal testing suggest that preexisting DM was not present in this patient
Gestational DM occurs in 1%–6% of all pregnancies.
Continue glucose assessments after delivery because maternal glucose needs will change suddenly in patients with gestational DM and because it is uncertain if the mother will remain diabetic following delivery.
Gestational DM occurs most frequently in the second or third trimester. If the mother presents with signs of DM earlier in pregnancy, suspect nongestational (type 1 or 2) DM.
Case 12-3: “I'm pregnant and bleeding” A 31-year-old woman who is G3P1011 presents to an emergency department at 35 weeks of gestation with the complaint of vaginal bleeding. She says that she noticed the bleeding earlier in the day and denies any bleeding prior to this time. She says that the bleeding has been light and she has only worn one absorbent pad today to absorb the blood. She reports mild-to-moderate abdominal pain and pelvic cramping. The pain radiates into her lower back. She denies any nonbloody vaginal discharge or seeing any expelled clots or tissue. She says that she tripped over a cat and fell down two stairs last evening but did not feel any abdominal pain or notice any bleeding until this morning. She denies any nausea, vomiting, diarrhea, dizziness, or shortness of breath. Although she feels crampy, she also says that she feels like she is having uterine contractions. She has not noticed much movement by the fetus since this morning. She says that she had a spontaneous abortion at 12 weeks of gestation during her last pregnancy for an unknown reason. Her first pregnancy was uncomplicated, and she delivered vaginally at 40 weeks of gestation. She has a history of P.287 chronic low back pain. She currently takes prenatal vitamins and no other medications. She admits that she smokes one to two cigarettes per day. She denies any alcohol or illicit drug use. On examination, she is an anxious-appearing pregnant woman. Auscultation of her heart and lungs detects normal breath sounds and a soft systolic murmur. Her abdomen is gravid and mildly tender in the lower portion. Lower abdominal palpation detects a firm uterus. She has normal bowel sounds. A pelvic speculum examination detects a small amount of blood at her cervical os, which is nearly closed. A bimanual examination is not performed. Her fundal height is 33 cm, and fetal heart sounds are detected with a Doppler. The following vital signs are measured: T: 98.8°F, HR: 98 bpm, BP: 124/72 mm Hg, RR: 17 breaths/min
Insulin is used preferentially over the oral hypoglycemics during pregnancy because of the better safety profile.
Differential Diagnosis Abruptio placentae, placenta previa, spontaneous abortion, ectopic pregnancy, preterm labor, pre-eclampsia, appendicitis, hydatidiform mole, choriocarcinoma
Laboratory Data and Other Study Results CBC: WBC: 8.9, Hgb: 12.3, Plt: 231 Chem7: Na: 141 mEq/L, K: 3.9 mEq/L, Cl: 100 mEq/L, CO2: 26 mEq/L, BUN: 16 mg/dL, Cr: 0.8 mg/dL, Glu: 91 mg/dL Coagulation panel (Coags): protime (PT) 12.1 sec, international normalized ratio (INR): 1.0, partial thromboplastin time (PTT): 27.2 sec Serum β-hCG: 74,318 U/L Biophysical profile: score of 6 (fetal tone criteria and fetal movement not satisfied); placental implantation on the upper posterior wall of the uterus seen on US; small hyperechoic signal adjacent to the placenta consistent with hemorrhage seen on US; single fetus visualized with no abnormal appearance of the ovaries or fallopian tubes
Diagnosis Abruptio placentae
Treatment Administered The patient was admitted to the labor and delivery floor and placed on modified bedrest Constant maternal and fetal monitoring was initiated IV fluids were started to maintain maternal intravenous volume Magnesium sulfate was administered to prevent further contractions
Follow-up Under monitoring, the fetal heart rate remained stable, as did maternal hemodynamic monitoring A repeat biophysical profile 12 hours later was found to have a score of 8 (fetal tone criteria not met) The lecithin/sphingomyelin ratio of the amniotic fluid was measured and found to be consistent with fetal lung maturity; the decision was made to proceed with delivery Magnesium sulfate was stopped, and the patient rapidly entered labor Both the mother and the fetus remained stable during labor, and an uncomplicated vaginal delivery was performed
Steps to the Diagnosis Abruptio placentae Premature separation of the placenta from the uterine wall, leading to maternal hemorrhage P.288 Risk factors: HTN, prior abruptio placentae, trauma, tobacco use, cocaine use, PROM, multiple gestations, multiparity History: painful vaginal bleeding in the third trimester, abdominal pain radiating to the back, uterine contractions, decreased fetal activity Physical examination: pelvic and abdominal tenderness, increased uterine tone, possible maternal HTN, and increased fundal height with significant hemorrhage Tests: Decreased hemoglobin and fibrinogen and increased PT, INR, and PTT are seen with significant hemorrhage Abdominal US inconsistently shows separation of the placenta from the uterus; it may also show a maternal hemorrhage A biophysical profile is important to assess fetal well-being Treatment: Mild cases with stable maternal hemodynamics and fetal assessments may be treated with modified bedrest (i.e., bedrest except for bathroom privileges and necessary mobilization) and close maternal and fetal monitoring Delivery typically proceeds rapidly due to uterine irritation Delivery may be delayed using tocolytic therapy (e.g., magnesium sulfate) in cases in which greater fetal maturity is desired Cesarian delivery is indicated in cases of maternal hemodynamic instability or fetal distress
Outcomes: Potential complications include DIC, severe maternal hemorrhage, maternal or fetal death, and the need for a hysterectomy to control uterine bleeding The fetal mortality rate is 12% The risk of abruptio placentae in future pregnancies is 12% Clues to the diagnosis: History: painful uterine bleeding in the third trimester, uterine contractions, recent trauma, tobacco use Physical: abdominal tenderness, uterine bleeding, uterine firmness Tests: abdominal US results
Placenta previa and abruptio placentae are the most common causes of vaginal bleeding after 20 weeks gestation. Bleeding in placenta previa is painless, and bleeding in abruptio placentae is painful.
Causes of abdominal pain during pregnancy may be remembered by the mnemonic CRUEL CRAMP: Constipation, Round ligament stretching, UTI, Ectopic pregnancy, Labor (preterm or term), Cholestasis, Rupture (ectopic or uterine), Abruptio placentae, Miscarriage, Pre-eclampsia.
Delivery should be delayed until at least 34 weeks of gestation if the mother and fetus remain stable during abruptio placentae. Placenta previa Implantation of the placenta near the cervical os Frequently associated with uterine bleeding during pregnancy Variations (Figure 12-4) Low implantation: the placenta implants in the lower uterus but does not infringe upon the cervical os until full dilation occurs during labor Partial placenta previa: the placenta partially covers the cervical os Complete placenta previa: the placenta completely covers the cervical os Risk factors: multiparity, maternal age >30 years, prior placenta previa, multiple gestations, uterine fibroids, prior abortion, tobacco use History: painless vaginal bleeding in the third trimester Physical examination: speculum examination detects bleeding from the cervical os Tests: US is performed to determine the location of the placenta (transvaginal is most accurate) Treatment: Mild, periodic bleeding should be treated with modified bedrest Active bleeding requires an inpatient admission for maternal and fetal monitoring
Delivery by cesarian section should be performed once fetal lung maturity has occurred Tocolytics (e.g., magnesium sulfate, ritodrine, terbutaline, indomethacin, nifedipine) may be used to delay delivery and to reduce the maternal bleeding risk in cases of a preterm fetus with immature lungs and mild maternal bleeding Vaginal delivery may be attempted in cases of a low-lying placenta Outcomes: Potential complications include severe maternal hemorrhage, IUGR, malpresentation, PROM, and vasa previa (i.e., fetal vessels overlying the cervical os) P.289
Figure 12-4 Uterine profiles and cross-sections demonstrating normal placental implantation and examples of low-lying placental implantation, partial placenta previa, and complete placenta previa.
Preterm delivery occurs in 50% of patients Fetal mortality occurs in 3% of cases; maternal mortality occurs in <0.5% of cases Why eliminated from differential: the painful nature of the bleeding and the location of the placenta indicated by the US rule out this diagnosis
The 30th week of gestation is the most common time for placenta previa to be diagnosed.
Do not perform a manual vaginal examination in a pregnant patient with vaginal bleeding until placenta previa can be ruled out with an US; manual examination
increases the risk of inducing greater hemorrhage.
Spontaneous abortions occur in up to 25% of pregnancies.
Causes of recurrent spontaneous abortion may be remembered by the mnemonic CUPID'S SIGHT: Coagulopathy, Uterine issues (fibroids, cervical incompetence), PCOS, Immunologic causes, DM, Substance use (tobacco, alcohol), Stress, Infection, Genetic issues (chromosome abnormalities), Hyperprolactinemia, Thyroid disorder. Spontaneous abortion (a.k.a. miscarriage) Nonelective termination of a pregnancy prior to 20 weeks of gestation (Table 12-11) First trimester spontaneous abortions are usually due to fetal chromosomal abnormalities Second trimester spontaneous abortions are usually due to infection, cervical incompetence, uterine abnormalities, hypercoagulable states, poor maternal health, or drug use
Table 12-11 Types of Spontaneous Abortions
Abortion Ty pe
Threatened
Inevitable
Uterine
In the initial 20
Initial 20 weeks plus
bleeding
weeks of
pain
Inc omplete
Complete
Missed
SIGN/SY MPTOMS
Initial 20 weeks
Initial 20 weeks
Present or with pain
gestation
Cervical os
Closed
Open
Open
Open
Closed
Uterine
None
None
Some
All
None
US detects
Possible detection
Based on history of
Based on history of
US detects
viable fetus
of fetus by US
expelled products of
expelled products of
nonviable
conception
conception
intrauterine fetus
Misoprostol or D&C
None
Misoprostol or
contents expelled
DIAGNOSIS
TREATMENT
Bedrest,
Misoprostol or D&C
limited activity
to remove uterine
D&C
contents
D&C, dilation and curettage; US, ultrasound.
P.290 Risk factors: increased maternal age, multiparity, uterine abnormalities, tobacco use, alcohol use, nonsteroidal anti-inflammatory drugs (NSAIDs), cocaine, caffeine, low folate levels, and congenital infections History: abdominal pain, crampy vaginal bleeding
Physical examination: visualization of blood at the cervical os, possible open cervical os, possible abdominal tenderness Tests: β-hCG levels may be used to determine fetal viability and estimate the gestational age; US is used to detect a fetus and to assess viability (via a biophysical profile) Treatment: dependent on the type of spontaneous abortion (Table 12-11) Outcomes: fetal demise is inevitable in all but threatened abortions; unless other risk factors exist, patients should not have any further increased risk for spontaneous abortion in future pregnancies Why eliminated from differential: the time of gestation is beyond what is considered consistent with a spontaneous abortion
The most common causes of vaginal bleeding in early pregnancy are ectopic pregnancy, threatened or inevitable spontaneous abortion, physiologic bleeding (related to implantation), and uterine-cervical pathology. Ectopic pregnancy Implantation of a fertilized ovum outside of the uterus The ampulla of the fallopian tube is the most common location of implantation (95% of cases), but the ovary, cervix, and the abdominal cavity are also potential sites Risk factors: pelvic inflammatory disease (PID), history of sexually transmitted diseases (STDs), prior gynecologic surgery, prior ectopic pregnancy, multiple sexual partners, tobacco use History: Abdominal pain, nausea, amenorrhea or known pregnancy, mild vaginal bleeding Abdominal pain becomes severe if rupture occurs Physical examination: Abdominal tenderness, visualized vaginal bleeding, possible palpable pelvis mass Abdominal guarding, rebound tenderness, hypotension, and tachycardia develop if rupture occurs Tests: Elevated β-hCG in the presence of an US that is unable to locate an intrauterine pregnancy β-hCG for an intrauterine pregnancy should double every 48 hours, so a β-hCG level that is low for the time of gestation should raise the suspicion for an ectopic pregnancy Transabdominal and transvaginal US should be able to visualize a pregnancy once the β-hCG reaches 6,500 mIU/mL and 1,500 mIU/mL, respectively Treatment: an unruptured ectopic pregnancy of <6 weeks duration is treated with methotrexate to induce an abortion; longer term or ruptured ectopic pregnancies require surgical excision with an attempt to preserve the fallopian tube Outcomes: complications include inevitable fetal death, severe maternal hemorrhage, increased risk of future ectopic pregnancies, infertility, Rh (rhesus factor) sensitization, and maternal death; the risk of future ectopic pregnancy is up to 25% Why eliminated from differential: based on the US findings and despite the small possibility of multiple
gestations with one gestation being ectopic, it is much more likely that the patient has a single pregnancy complicated by abruption
Any woman of childbearing age who presents with abdominal pain must be given a β-hCG pregnancy test. Preterm labor Onset of labor before 37 weeks of gestation Risk factors: multiple gestation, infection of genital tract, PROM, placenta previa, abruptio placentae, previous preterm labor, polyhydramnios, cervical incompetence, poor nutrition, stressful environment, tobacco use, substance abuse, lower socioeconomic status History: low back pain, abdominal cramping, more than eight uterine contractions per hour Physical examination: cervical dilation and effacement prior to 37 weeks of gestation Tests: US may be helpful to assess the cervical length, amniotic fluid volume, fetal well-being, and to verify gestational age P.291 Treatment: Patients should be supplied with adequate hydration and restricted in their activity Tocolytics can be used to inhibit contractions and delay delivery if the patient is at <34 weeks of gestation Corticosteroids may be given to the mother to hasten fetal lung maturity if the pregnancy is at <34 weeks of gestation Antibiotics should be given for group B streptococcus prophylaxis or if there is evidence of an active infection Expectant management may proceed once the pregnancy has reached 34 weeks Outcomes: the rates of fetal demise, fetal sepsis, respiratory distress of the newborn, and necrotizing enterocolitis increase with the degree of prematurity before 34 weeks Why eliminated from differential: although the patient is at only at 35 weeks of gestation, the presence of vaginal bleeding and the US appearance signify that preterm labor is not a complete explanation for the current presentation and that there is an underlying etiology
A cervical length >35 mm is associated with a very low risk of preterm birth; a cervical length <25 mm has an increased risk of preterm birth. Pre-eclampsia More thorough discussion in prior case Why eliminated from differential: the normal maternal blood pressure rules out this diagnosis Appendicitis More thorough discussion in Chapter 3 Why eliminated from differential: this diagnosis is unlikely because it would not account for the vaginal bleeding
Hydatidiform mole A benign neoplasm of trophoblastic cells (i.e., placental cells) that carries a risk of malignant transformation Types: Complete: 46, XX genotype; completely derived from the father (i.e., an empty egg is penetrated by sperm) Incomplete: 69,XXY genotype; fertilization of an egg by two sperm simultaneously Risk factors: low socioeconomic status, age >40 years during pregnancy, prior molar pregnancy, Asian heritage, tobacco use History: heavy or irregular painless vaginal bleeding during the first or second trimester of a suspected pregnancy, severe nausea and vomiting, dizziness, anxiety Physical examination: large fundal height for gestational age, expulsion of “grape-like” vesicles from the vagina, no detectable fetal movement or heart tones Tests: β-hCG is higher than expected for the gestational age; US detects a “snow storm” pattern in the uterus without the presence of a gestational sac (Figure 12-5)
Figure 12-5 Ultrasound of the uterus demonstrating the “snow storm” appearance consistent with a hydatidiform mole.
P.292 Treatment: Dilation and curettage is performed to remove the neoplasm Pregnancy should be avoided for 6 months to a year to avoid recurrences β-hCG levels should be followed for a year to confirm a decreasing level Outcomes: a malignant trophoblastic neoplasm develops in 20% of cases but carries an excellent prognosis Why eliminated from differential: the appearance of the US rules out this diagnosis
Choriocarcinoma A malignant trophoblastic neoplasm that arises from hydatidiform moles (in 50% of cases) or following a spontaneous abortion, ectopic pregnancy, or normal pregnancy History: vaginal bleeding, hemoptysis, dyspnea, headache, dizziness, rectal bleeding Physical examination: enlarged uterus, visualized bleeding from the cervical os Tests: Increased β-hCG (usually >100,000 U/L) US will detect a uterine mass with a mix of hemorrhagic and necrotic areas and possible parametrial invasion CT is useful to detect metastases Treatment: Chemotherapy is administered routinely Pregnancy should be avoided for a year to avoid recurrences β-hCG levels should be followed for a year to confirm a decreasing level Hysterectomy is performed in women older than 40 years if the disease is limited to the uterus Outcomes: potential sites of metastases include the lungs, brain, liver, kidneys, and gastrointestinal tract; the prognosis is good in the absence of brain or liver metastases Why eliminated from differential: although the US in this case and that for this diagnosis may be similar appearing, the β-hCG is not consistent with choriocarcinoma
High β-hCG is seen in both hydatidiform mole and multiple gestation; differentiate the conditions with an US.
Highly suspect a molar pregnancy if pre-eclampsia occurs in the first half of pregnancy, and perform an US to confirm the diagnosis.
Case 12-4: “My baby's not right” A 6-day-old neonate boy is a patient in the neonatal intensive care unit (NICU) after developing respiratory distress. His mother is a 16-year-old girl who received no prenatal care. She says that she was trying to hide the fact that she was pregnant from other people. She says that she has had multiple sexual partners and is unsure who is the father of the child. She denies any previous pregnancies. She says that she was treated for a Neisseria gonorrhoeae infection 1 year ago but denies any other past medical history. She denies any medication or substance use. She says that she has a family history of HTN and DM. Dating based on the mother's last menstrual period estimates that the pregnancy was at 37 weeks of gestation at the time of delivery. The child was delivered via an uncomplicated vaginal delivery. Apgar scores were 5 at 1 minute and 9 at 5 minutes. No meconium was visible at the time of delivery. Initially, the neonate appeared to be doing well. The child's mother says that she was unsuccessful in breast feeding him and that any attempts made him cry. Attempts to formula feed the child were also unsuccessful in all but a couple of attempts. He had two nonbloody bowel movements after eating. Because of the child's poor feeding, he was kept as an inpatient when his mother was discharged on the second day following the delivery. On the fourth day after his birth the child experienced a grand mal seizure and exhibited respiratory distress. He was urgently intubated and transferred to the NICU. This morning he was weaned from the ventilator and extubated but has remained lethargic, despite receiving IV fluids and parenteral nutrition. No additional seizures have been witnessed. On examination, the child is an
ill-appearing neonate with little spontaneous activity. He is mildly jaundiced. His right eye conjunctiva is inflamed and there is a cluster of small blisters at the corner of the eye. A few similar vesicles are seen on his scalp. His fontanelles are bulging slightly. He has no palpable lymphadenopathy. Auscultation of his lungs detects mild rhonchi. Auscultation of his heart detects no abnormal heart sounds. P.293 His abdomen is soft with few bowel sounds. He has little spontaneous movement but withdraws and grimaces to pinching. The following vital signs are measured: T: 98.5°F, HR: 153 bpm, BP: 65/45 mm Hg, RR: 31 breaths/min (Normal for neonate: HR: 100–170 bpm, BP: 65–95/30–60, RR: 30–50 breaths/min)
Differential Diagnosis Hemolytic disease of the newborn, congenital infection, maternal substance use during pregnancy, necrotizing enterocolitis, congenital heart disease, meconium aspiration syndrome
Laboratory Data and Other Study Results CBC: WBC: 12.5, Hgb: 19.5, Plt: 97 Neonatal blood type: A+ Maternal blood type: A+ Chem7: Na: 138 mEq/L, K: 4.3 mEq/L, Cl: 100 mEq/L, CO2: 28 mEq/L, BUN: 12 mg/dL, Cr: 0.6 mg/dL, Glu: 79 mg/dL LFTs: AlkPhos: 178 U/L, ALT: 121 U/L, AST: 156 U/L, TBili: 10 mg/dL, DBili: 0.2 mg/dL Chest x-ray (CXR): mild diffuse infiltrates; normal lung size; no focal lesions or effusions; no fractures Abdominal x-ray (AXR): sparse bowel air; no air-fluid levels; no apparent organomegaly Head CT: increased signal in both temporal lobes with associated edema; mild to moderate ventricular compression; no focal masses Echocardiogram: normal blood flow and chamber size; normal valve, vessel, and chamber anatomy Lumbar puncture: cloudy appearance, opening pressure 89 mm Hg, WBC: 37/µL, Glu: 45 mg/dL, protein: 60 mg/dL Neonatal cytomegalovirus (CMV) IgM: negative Neonatal varicella IgM: negative Maternal human immunodeficiency virus (HIV) enzyme-linked immunosorbent assay (ELISA): negative Maternal herpes simplex virus (HSV)-1 and -2 antibodies: positive HSV-2 antibodies Maternal rapid plasma reagin (RPR): negative Maternal urine toxicology screen: negative Maternal group B streptococcus antigen: negative Maternal N. gonorrhoeae and Chlamydia trachomatis immunoassays: negative Maternal rubella IgM antibodies: negative Maternal rubeola IgM antibodies: negative The following confirmatory tests are ordered:
Tzanck smear of neonatal vesicles: multinucleated giant cells Neonatal cerebral spinal fluid (CSF) HSV-2 polymerase chain reaction (PCR): positive
Diagnosis Neonatal herpes simplex-2 infection with encephalopathy
Treatment Administered Acyclovir was started in the neonate Parenteral feeding and supportive measures were continued
Follow-up Despite the administration of acyclovir, the neonate experienced an additional seizure on the eighth day of life but did not experience respiratory distress P.294
Table 12-12 Congenital Infections and Their Effects on the Fetus and Neonates
Maternal Infec tion
Cytomegalo-virus
Possible Fetal/Neonatal Effec ts
IUGR, chorioretinitis, CNS abnormalities, mental retardation, vision abnormalities, deafness, hydrocephalus, seizures, hepatosplenomegaly
Gonorrhea/chlamydia
Increased risk of spontaneous abortion; neonatal sepsis, c onjunc tivitis
Group B
Respiratory distress, pneumonia, meningitis,
streptococcus
sepsis
Diagnosis
Treatment
Possible mononucleosislike
No treatment if infection
illness
Ganciclovir may decrease
IgM antibody screening or
effects in neonates
PCR of viral DNA within
Good hygiene reduces risk
first few weeks of life
of transmission
Cervical culture and
Erythromycin given to
immunoassays
mother or neonate
Antigen screening after 34
IV β-lactams or clindamycin
weeks gestation
during labor or in infected neonates
Hepatitis B
Increased risk of prematurity and IUGR; increased risk of neonatal death if acute disease develops
Prenatal surface antigen
Maternal vac c ination;
screening
vaccination of neonate and administration of immune globulin shortly after birth
Herpes simplex
Increased risk of prematurity, IUGR, and spontaneous abortion; high risk of neonatal death or CNS abnormalities if disease transmission occurs
Clinical diagnosis
Delivery by c esarean
confirmed with viral
sec tion to avoid disease
culture or immunoassays
transmission if active lesions present or if primary outbreak Acyclovir may be beneficial in neonates
HIV
Viral transmission in utero (5% risk), rapid progression of disease to AIDS
Early prenatal maternal
AZT significantly reduces
blood screening (consent
vertical transmission risk
required)
Continue prescribed antiviral regimen but avoid
efavirenz, didanosine, stavudine, and nevirapine
Rubella
Increased risk of spontaneous abortion, skin lesions (“blueberry muffin baby”), c ongenital rubella syndrome (i.e., IUGR,
Early prenatal IgG
Mother should be immunized
screening
before attempting to become pregnant
deafness, cardiovascular abnormalities, vision
No treatment if infection
abnormalities, CNS abnormalities, hepatitis) if
develops during pregnancy
disease transmission occurs
No proven benefit from rubella immune globulin
Rubeola (measles)
Increased risk of prematurity, IUGR, and spontaneous abortion; high risk (20% if term birth, 55% if preterm) of neonatal death if disease transmission occurs
Clinical diagnosis in mother
Mother should be immunized
confirmed by IgM or IgG
before attempting to
antibodies after rash
become pregnant
develops
Immune serum globulin given to mother if infection develops during pregnancy
Syphilis
Neonatal anemia, deafness, hepatosplenomegaly, pneumonia, hepatitis,
Early prenatal RPR or VDRL
penicillin
osteody-strophy; 25% neonatal mortality
Toxoplasmosis
Hydrocephalus, intracranial calcifications, chorioretinitis, microcephaly, spontaneous abortion, seizures
Maternal or neonatal
Possible mononucleosislike
Pyrimethamine, sulfadiazine,
illness
and folinic acid
Amniotic fluid PCR for
Mother should avoid
Toxoplasma gondii or
gardening, raw meat, cat
serum antibody screening
litter boxes, and
may be helpful for the
unpasteurized milk
diagnosis
Varicella zoster
Prematurity, enc ephalitis, pneumonia, IUGR, CNS abnormalities, limb abnormalities, blindness; high risk of neonatal death if birth occurs during active infection
IgG titer screening in
Varicella immune globulin
women with no know n
given to nonimmune mother
history of disease
within 96 hours of exposure
IgM and IgG antibody
and to neonate if born
titers can confirm
during active infection
diagnosis in neonates
Vaccine is contraindicated during pregnancy (live attenuated virus carries risk of fetal infection)
AIDS, acquired immune deficiency syndrome; AZT, zidovudine; CNS, central nervous system; Ig, immunoglobulin; IUGR, intrauterine growth restriction; IV, intravenous; PCR, polymerase chain reaction; RPR, rapid plasma reagin; VDRL, Venereal Disease Research Laboratories.
P.295 The neonate entered status epilepticus the following day and subsequent cardiopulmonary collapse; despite resuscitative efforts, he was unable to be revived
Steps to the Diagnosis Congenital infections Maternal infections during pregnancy that can have significant deleterious effects on fetal development or viability (Table 12-12) History: the mother may exhibit findings consistent with each disease, neonatal symptoms and findings are also disease dependent Physical examination: disease dependent (Table 12-12)
Tests: prenatal screening is vital to the early detection of these infections; diagnostic tests following delivery are disease dependent (Table 12-12) Treatment: frequently maternal therapies are available to prevent fetal infection or to reduce the severity of the disease effects; treatment of infected neonates is disease dependent (Table 12-12) Outcomes: generally these infections carry significant long-term sequelae that may result in life-long disabilities and a decreased lifespan (Table 12-12) Clues to the diagnosis: History: lack of prenatal care, previous maternal STD infection, poor feeding, child lethargy, neonatal seizures, respiratory distress Physical: lethargy, jaundice, bulging fontanelles, vesicles on scalp and near eye, conjunctivitis Tests: CSF analysis consistent with infection, positive HSV serology, head CT appearance
Congenital infections are frequently referred to as the TORCHS infections: Toxoplasmosis, Other (varicella-zoster, group B streptococcus, Chlamydia, Neisseria), Rubella/Rubeola, Cytomegalovirus, Herpes simplex/Hepatitis B/HIV, Syphilis. Maternal substance use during pregnancy Several medicinal and illicit substances carry teratogenic risks during pregnancy (Tables 12-13 and 12-14) History: the mother should be able to provide a list of any medications that she is taking Physical examination: dependent on the abnormalities caused by the substance
Table 12-13 Recreational Drug Use and Associated Risks to Mother and Fetus during Pregnancy
Drug
Cocaine
Ethanol
Maternal Risks
Fetal Risks
Arrhythmia, myocardial infarction, subarachnoid
IUGR, prematurity, facial abnormalities, delayed intellectual development,
hemorrhage, seizures, stroke, abruptio placentae
stroke
Minimal
Fetal alc ohol sy ndrome (i.e., mental retardation, IUGR, sensory and motor neuropathy, facial abnormalities), spontaneous abortion, intrauterine fetal demise
Hallucinogens
Personal endangerment (poor decision-making)
Possible developmental delays
Marijuana
Minimal
IUGR, prematurity
Opioids
Infec tion (from needles), narcotic withdrawal,
Prematurity, IUGR, meconium aspiration, neonatal infections, narc otic
premature rupture of membranes
w ithdraw al (may be fatal)
Lack of appetite and malnutrition, arrhythmia,
IUGR, congenital heart defects, cleft palate
Stimulants
withdrawal depression, HTN
Tobacco
Abruptio plac entae, plac enta previa,
Spontaneous abortion, prematurity, IUGR, intrauterine fetal demise,
premature rupture of membranes
impaired intellectual development, higher risk of neonatal respiratory infections
HTN, hypertension; IUGR, intrauterine growth restriction.
P.296
Table 12-14 Common Medications that Carry Teratogenic Risks
Medic ation
Teratogenic Risks
ACE-I
Renal abnormalities, decreased skull ossification
Aminoglycosides
CN VIII damage, skeletal abnormalities, renal defects
Carbamazepine
Facial abnormalities, IUGR, mental retardation, cardiovascular abnormalities, neural tube defects
Chemotherapeutics (all drug
Intrauterine fetal demise (approximately 30% of pregnancies), severe IUGR, multiple anatomical abnormalities (e.g.,
classes)
palate, bones, limbs, genitals, etc.), mental retardation, spontaneous abortion, secondary neoplasms
Diazepam
Cleft palate, renal defects, secondary neoplasms
Diethylstilbestrol
Vaginal and cervical cancer later in life (e.g., adenocarcinoma)
Fluoroquinolones
Cartilage abnormalities
Heparin
Prematurity, intrauterine fetal demise; considered much safer than warfarin
Iodide
Goiter, hypothyroidism, mental retardation
OCPs
Spontaneous abortion, ectopic pregnancy
Phenobarbital
Neonatal withdrawal
Phenytoin
Facial abnormalities, IUGR, mental retardation, cardiovascular abnormalities
Retinoids
CNS abnormalities, cardiovascular abnormalities, facial abnormalities, spontaneous abortion
Sulfonamides
Kernicterus (i.e., bile infiltration of brain)
Tetracycline
Skeletal abnormalities, limb abnormalities, teeth discoloration
Thalidomide
Limb abnormalities
Valproic acid
Neural tube defects (approximately 1% of pregnancies), facial abnormalities, cardiovascular abnormalities, skeletal abnormalities
Warfarin
Spontaneous abortion, IUGR, CNS abnormalities, facial abnormalities, mental retardation, Dandy-Walker malformation
ACE-I, angiotensin converting enzyme inhibitors; CN, cranial nerve; CNS, central nervous system; IUGR, intrauterine growth restriction; OCPs, oral contraceptive pills.
Tests: toxicology screening may be useful to determine traces of any substances in the mother's system (legal or illicit) but must be obtained with the mother's consent Treatment: Honest disclosure must be encouraged for all mothers-to-be in order to make any necessary adjustments to current medication regimens or to provide counseling and education for nonprescribed substance use In some cases the cessation of a medication may be more harmful than the teratogenic risks, and proper counseling about these risks should be provided Outcomes: the risks of teratogen use to the fetus are directly associated with the ability to avoid the use of such substances during pregnancy; teratogenic birth defects often carry significant lifelong sequelae Why eliminated from differential: the negative urine toxicology screen makes this diagnosis less likely, but this diagnosis must still be considered in any mother who voluntarily forgoes prenatal care
Teratogens will either kill the fetus or will have no effect within the initial 2 weeks of gestation. They may cause abnormal organ formation between 2 to 12 weeks. Hemolytic disease of the newborn More thorough discussion in Chapter 6 Why eliminated from differential: the lack of disparity between the maternal and neonatal blood types rules out this diagnosis P.297 Necrotizing enterocolitis More thorough discussion in Chapter 3 Why eliminated from differential: the absence of bloody vomiting or diarrhea, abdominal distention, and excessive bowel gas on the AXR make this diagnosis less likely Congenital heart diseases More thorough discussion in Chapter 1 Why eliminated from differential: the absence of extra heart sounds and the normal echocardiogram rule out these diagnoses Meconium aspiration syndrome
More thorough discussion in Chapter 2 Why eliminated from differential: the absence of meconium during the delivery, neonatal cyanosis, and lung hyperinflation on the CXR make this diagnosis unlikely
Case 12-5: “I don't think that my belly is getting any bigger” A 26-year-old woman who is G2P0010 reports for a routine prenatal visit at 34 weeks of gestation. She is slightly concerned because she has not gained any weight for the past 4 weeks and she does not feel that her abdomen has increased in size recently. She says that she feels fetal movements several times per day. She has attended all of her prenatal appointments, and no gestational problems have been previously detected. An US at 20 weeks of gestation detected the fetus to have appropriate growth and development. She was very happy to have heard this news because she has a history of systemic lupus erythematosus (SLE), which was related to a spontaneous abortion in a prior pregnancy. She denies feeling ill recently or having sustained any trauma. She says that she has been careful to consume an adequate caloric intake and gets regular exercise. She denies any vaginal bleeding or discharge of any kind. She has not had any uterine contractions. Besides SLE, she has no other past medical history. She had been taking naproxen and hydroxychloroquine prior to her pregnancy, but she stopped these medications after becoming pregnant. She has noticed greater fatigue, arthralgias, and skin photosensitivity since stopping these medications. She now only takes prenatal vitamins. She denies any substance use. On examination, she is a thin, well-appearing pregnant woman in no distress. Her face is symmetrical and has a faint malar rash. She has no lymphadenopathy. Auscultation of her heart and lungs detects normal breath sounds and a mild systolic murmur. Her abdomen is gravid and nontender with normal bowel sounds. A pelvic examination finds a closed cervical os, no cervical discharge or bleeding, and no cervical or adnexal tenderness. Her fundal height is 29 cm, and fetal heart sounds are detectable with a Doppler. She has mild edema in her feet and ankles. Her neurovascular examination is normal. She has a patchy discoid rash on her arms and trunk. The following vital signs are measured: T: 98.8°F, HR: 90 bpm, BP: 110/80 mm Hg, RR: 20 breaths/min
Differential Diagnosis Intrauterine growth restriction, premature rupture of membranes, abruptio placentae, placental dysfunction, idiopathic oligohydramnios, fetal renal anomaly
Laboratory Data and Other Study Results Height and weight: 5﹐ 8" 142 lb (initial weight 130 lb, BMI 19.8) Chem7: Na: 141 mEq/L, K: 3.7 mEq/L, Cl: 103 mEq/L, CO2: 25 mEq/L, BUN: 21 mg/dL, Cr: 1.1 mg/dL, Glu: 100 mg/dL UA: light yellow colored, pH: 7.0, specific gravity 1.010, no glucose/ketones/nitrites/leukocyte esterase/hematuria/proteinuria Nonstress test: reactive Abdominal US: single intrauterine gestation; no fetal visceral anomalies; no placental separation or intrauterine hemorrhage; low fetal abdominal circumference in proportion to head circumference and femur length; predicted fetal weight eighth percentile for gestational age; amniotic fluid index is 6 cm; Doppler detects normal placental blood flow Microscopic examination of vaginal fluid: few squamous cells; no WBCs or bacteria; no “ferning” of fluid
Diagnosis Intrauterine growth restriction due to maternal SLE
Treatment Administered The patient was put on modified bedrest at home and encouraged to drink fluids Weekly US were scheduled to monitor fetal growth
Follow-up The abdominal US at 36 weeks of gestation suggested little fetal growth, so the patient was admitted to the hospital for an induced delivery A uncomplicated vaginal delivery proceeded following oxytocin induction; the neonate was underweight and admitted to the NICU for close observation The neonate grew slowly in the postpartum period and was able to be discharged to home in stable condition after a 3-week stay in the NICU and regular neonatal nursery
Steps to the Diagnosis Intrauterine growth restriction Fetal growth that lags behind the gestational age Symmetric Makes up 20% of IUGR cases There is a generalized decrease in fetal size as evidenced by a lower-than-normal head circumference, abdominal circumference, and femoral length Occurs in early pregnancy and is most commonly due to congenital infection, chromosomal abnormalities, or maternal substance use Asymmetric Makes up 80% of IUGR cases There is a disproportionate decrease in fetal size demonstrated by a normal head circumference and femoral length and low abdominal circumference Occurs late in the pregnancy and is due to multiple gestations, maternal comorbid conditions (particularly autoimmune disorders), or placental insufficiency (e.g., vascular anomalies) History: asymptomatic or perceived small abdominal growth Physical examination: fundal height is at least 3 cm smaller than what is expected for the gestational age Tests: US is the primary tool used to diagnose decreased fetal growth Head/abdomen circumference and femoral length/abdominal circumference ratios are normal in symmetric IUGR and are increased in asymmetric IUGR Estimation of the fetal weight will be lower than the tenth percentile Oligohydramnios is common Doppler studies are performed to detect abnormalities in the fetal, maternal, or placental blood flow
Treatment: The pregnancy may be treated expectantly as long as fetal growth progresses, fetal well-being is satisfactory, and maternal health is stable; deviation from these requirements should prompt an induced or cesarian delivery It is important to perform periodic US to track fetal growth and well-being The administration of maternal corticosteroids should be considered for any delivery preceding fetal lung maturity Outcomes: the incidence of respiratory distress of the newborn, hemorrhage, necrotizing enterocolitis, and prolonged NICU stays are all increased in IUGR neonates when compared to normal-growth births; neonatal mortality increases significantly in deliveries occurring before 32 weeks of gestation Clues to the diagnosis: History: perceived small abdominal size and low weight gain, history of SLE Physical: fundal height that is small-for-gestational age Tests: findings of the abdominal US Premature rupture of membranes Spontaneous rupture of the amniotic sac and the spillage of amniotic fluid prior to the onset of labor Small ruptures can lead to a chronic slow leak of amniotic fluid and subsequent oligohydramnios
Figure 12-6 Microscopic examination of vaginal fluid demonstrating ferning, which indicates the presence of amniotic fluid.
Risk factors: genital tract infection, cervical incompetence, maternal malnutrition, PROM in prior pregnancies History: amniotic fluid seen emitting from the vagina Physical examination: amniotic fluid is seen within the vaginal canal on pelvic examination Tests: Microscopic examination of vaginal fluid will show “ferning” if amniotic fluid is present (Figure 12-6)
Nitrazine paper will turn blue in the presence of amniotic fluid when vaginal fluid is applied to it US should be used to confirm the presence of oligohydramnios, to assess the residual amniotic volume, and to determine the fetal presentation Amniotic fluid should be cultured to rule out infection Treatment: If PROM occurs before 32 weeks of gestation, give corticosteroids to the mother to hasten fetal lung maturation and antibiotics for group B streptococcus prophylaxis; induce labor once fetal lung maturity is maximized If PROM occurs after 34 weeks of gestation, administer prophylactic antibiotics and induce labor If PROM occurs between 32 to 34 weeks of gestation, perform an amniotic fluid lecithin/sphingomyelin ratio analysis (if possible) to determine fetal lung maturity and guide treatment If amniotic fluid is infected (i.e., chorioamnionitis), delay delivery until antibiotic therapy is initiated Outcomes: the prognosis is good when PROM occurs after 34 weeks of gestation; the rates of fetal demise, neurologic abnormalities, and respiratory distress of the newborn increase when PROM occurs earlier in the pregnancy Why eliminated from differential: the absence of vaginal amniotic fluid leakage by report of the patient and the absence of ferning on vaginal fluid microscopy make this diagnosis unlikely Abruptio placentae/placental dysfunction More thorough discussion in prior case Why eliminated from differential: the absence of witnessed vaginal bleeding and the normal appearance of the placenta on US make these diagnoses unlikely Idiopathic oligohydramnios A deficiency of amniotic fluid in the gestational sac that cannot be explained by a comorbid maternal or fetal condition (Table 12-15) History: asymptomatic or perceived small abdominal size Physical examination: small fundal height for the gestational age
Table 12-15 Abnormalities of Amniotic Fluid Volume
Volume
Oligohydramnios
Polyhydramnios
Decreased
Increased
Second trimester—fetal renal abnormalities, pre-eclampsia,
Insufficient fetal swallowing of fluid (e.g., esophageal atresia),
maternal vascular insufficiency (e.g., collagen-vascular
increased fetal urine output (e.g., maternal DM), multiple
disease, DM, SLE), placental thrombosis, amniocentesis
gestations, fetal anemia, chromosomal abnormalities
abnormality
Amniotic fluid volume
Causes
Third trimesterPROM, pre-eclampsia, abruptio placentae, idiopathic causes
Effect on fundal
Decreased
Increased
<7 cm; no individual pockets of 3 cm or more
>24 cm; at least one pocket of 8 cm or more
Maternal hydration, bedrest, and amnioinfusion; expectant
Amnioreduction and indomethacin if <32 weeks of gestation;
management if fetus is stable; delivery if fetal distress
amnioreduction alone if >32 weeks of gestation
height
Amniotic fluid index
Treatment
occurs
Potential
Spontaneous abortion, intrauterine fetal demise, anatomic
Preterm labor, PROM, fetal malpresentation, maternal
Complications
compression abnormalities
respiratory compromise
DM, diabetes mellitus; PROM, premature rupture of membranes; SLE, systemic lupus erythematosus.
Tests: US is used to determine the amniotic fluid volume and to perform a fetal assessment; to meet the criteria for oligohydramnios, the amniotic fluid index must be <7 cm with no individual pockets 3 cm or larger in size Treatment: Expectant management may be considered if the fetus passes the tests of well-being Maternal hydration, modified bedrest, and amnioinfusion may help to improve the amniotic fluid volume Labor should be induced or cesarian delivery should be performed if the fetus begins to exhibit signs of distress Outcomes: potential complications include spontaneous abortion, intrauterine fetal demise, and anatomic abnormalities (e.g., limb, facial) from fetal compression Why eliminated from differential: the mother's history of SLE is a much more likely explanation for the IUGR and oligohydramnios in this case Fetal renal anomaly Abnormal development of the fetal kidneys, resulting in abnormal urologic function The majority of all abnormalities are obstructive in nature History: asymptomatic or a maternal perception of small abdominal size Physical examination: small fundal height for the gestational age Tests: US will detect oligohydramnios; fetal renal abnormalities can be detected by US in 90% of cases prior to birth Treatment: although no intrauterine treatment for the renal pathology exists, management should focus on the treatment of oligohydramnios Outcomes: potential complication include those of oligohydramnios, poor lung maturation, congenital heart disease, and fetal demise; the long-term prognosis of children with a congenital renal anomaly depends on the ability to reconstruct a normal urologic drainage tract following birth Why eliminated from differential: the absence of fetal visceral abnormalities on the US makes this diagnosis less likely
Case 12-6: “I can't seem to stop bleeding” A 29-year-old woman, G2P2002, is an inpatient on the labor and delivery floor 1 day after a vaginal delivery at 40 weeks' gestation. Her delivery was notable for a first stage of labor, lasting 15 hours. Her pregnancy had been uncomplicated. She calls a nurse for assistance because she has just awoken from a nap to find that she is lying in a pool of blood. The obstetrician on call is asked to evaluate the patient. She says that she had noticed some persistent slow vaginal bleeding when she would go to the bathroom since the delivery but had never seen this much bleeding previously. She denies seeing any soft tissue expelled from her vagina since the delivery. Before taking a nap she had used a total of four absorptive pads since the delivery to absorb this vaginal bleeding. She denies any trauma since the delivery. She feels a little lightheaded but denies any syncope. She says that her heart feels like it is racing. She denies any shortness of breath. She denies any previous history of easy bruising or bleeding or any other medical problems. She has taken acetaminophen a few times since the delivery for some pelvic pain but denies taking any other medications. She denies any substance use since the delivery. On examination, she is an anxious-appearing woman. Auscultation of her lungs and heart detect clear breath sounds, tachycardia, and no extra heart sounds. Her abdomen is soft, and she has slight lower abdominal tenderness. A rectal examination detects no tenderness or gross blood. A pelvic examination detects no lacerations of the vulva or vaginal canal. There is a slow drainage of blood seen coming from the cervical os. The bimanual examination detects mild cervical tenderness and no adnexal tenderness. Her uterus feels soft. Examination of her skin detects no petechiae or mucosal bleeding. The following vital signs are measured: T: 98.6°F, HR: 110 bpm, BP: 105/65 mm Hg, RR: 20 breaths/min
Differential Diagnosis Uterine atony, retained placental tissue, birth canal trauma, disseminated intravascular coagulopathy, von Willebrand disease, hemophilia, thrombocytopenia disorders
Laboratory Data and Other Study Results CBC: WBC: 6.9, Hgb: 11.9, Plt: 300 Coags: PT: 11.2 sec, INR: 1.0, PTT: 25.8 sec Fibrinogen: 302 mg/dL Abdominal US: no sites of focal uterine hemorrhage; no violation of the uterine wall visible; no obvious retained placental tissue
Diagnosis Uterine atony
Treatment Administered IV hydration was started to help improve the patient's intravascular volume A bolus of oxytocin was administered to induce uterine contraction
Follow-up Following the oxytocin administration, the uterus was found to tighten on repeat palpation The patient's vital signs remained stable, so she was kept under close observation The vaginal bleeding resolved after the initiation of treatment
Steps to the Diagnosis Uterine atony Failure of the uterine musculature to contract following delivery, resulting in the impaired ability to stop any generalized postdelivery uterine bleeding Risk factors: prolonged labor, macrosomia, polyhydramnios, multiple gestations, tocolytic use, chorioamnionitis, low placental implantation History: postpartum bleeding, possible palpitations, dizziness, or syncope Physical examination: boggy-feeling uterus, tachycardia, bleeding from the cervix on pelvic examination Tests: a CBC should be performed to rule out significant anemia; US is rarely helpful Treatment: IV hydration is important to maintain intravascular volume Blood transfusion should be considered for significant anemia Oxytocin is administered as a bolus to induce uterine contraction Outcomes: the prognosis is very good with oxytocin therapy; complications result from profound blood loss and are primarily concerned with the effects of significant hypotension (e.g., tissue ischemic, anoxic brain injury, cardiac ischemia, renal insufficiency) Clues to the diagnosis: History: significant postpartum blood loss, prolonged labor Physical: boggy uterus Tests: noncontributory
Abnormal postpartum bleeding is defined as blood loss >500 mL/day following a vaginal delivery or >1,000mL/day following cesarian delivery. Retained placental tissue Failure of part of the placenta to separate from the uterine wall during delivery, resulting in persistent postpartum bleeding Risk factors: preterm delivery, accessory placental lobes, placenta accreta (i.e., excessive invasion of the placenta into the uterine wall during implantation) History: postpartum bleeding, palpitations, dizziness, possible syncope Physical examination: bleeding from the cervix, uterine distention Tests: US will detect retained placental tissue; a CBC is performed to detect significant anemia Treatment: Volume resuscitation with IV fluids and blood products should be performed as necessary Oxytocin may be administered to induce uterine contraction and help control bleeding Uterine massage may be performed to help induce the retained tissue to dislodge Dilation and curettage should be performed to remove any placental tissue that is still retained following
nonoperative treatments Hysterectomy or arterial ligation may be considered for substantial bleeding that is difficult to control Outcomes: the potential complications are those associated with hypovolemia and hypotensive ischemia Why eliminated from differential: the absence of retained placental tissue on the US makes this diagnosis unlikely
Retained placental tissue causes the most substantial volume of postpartum bleeding. Birth canal trauma Injury to the birth canal during the delivery, resulting in significant bleeding from lacerations A similar presentation results from an episiotomy (i.e., an incision made in the vaginal opening to ease passage of the neonate) that is poorly repaired History: difficult labor, utilization of an episiotomy, postpartum bleeding, symptoms of hypotension Physical examination: visualization of birth canal trauma and bleeding during the pelvic examination Tests: a CBC rules out significant anemia Treatment: surgical repair of the lacerations is required to slow the bleeding Outcomes: scar formation in the birth canal may lead to postdelivery dyspareunia and difficulties in future births; other potential complications are those associated with hypovolemia and hypotensive ischemia Why eliminated from differential: the pelvic examination is unable to detect any birth canal trauma Disseminated intravascular coagulopathy/von Willebrand disease/hemophilia/thrombocytopenia More thorough discussion in Chapter 6 Why eliminated from differential: the normal range CBC, coagulation labs, and fibrinogen make these diagnoses unlikely
Case 12-7: “My cousin is going to have a baby” A 29-year-old woman, who is G3P2002, is brought to an obstetrician by her cousin to evaluate her pregnancy. The patient formerly lived in Southeast Asia, and she had received her prenatal care there. Her family was displaced by a fire, and she has spent the past 4 months living with relatives after immigrating to the United States. She has not received any prenatal care since her 18th week of gestation. She is now 36 weeks pregnant based on the time of her last menstrual period. She says that she had multiple blood tests performed during her prenatal work-up, but that she never had an US performed. Despite her uncertain living situation for the past several months, she says that she has been doing well. She says that she feels fatigued most days by the end of the day. She has mild low back pain but denies any abdominal or pelvic pain. She denies any shortness of breath. She denies any vaginal or rectal bleeding or discharge. She complains of swelling in her legs that has developed over the past 2 months. She says that she is generally healthy and has not had any serious illnesses. She received the diphtheria and tetanus toxoids and acellular pertussis, Haemophilus influenzae type b, mumps, measles, rubella, hepatitis A and B, BCG, and inactivated polio vaccines in her youth. She has never had any STDs or sexual trauma. She takes no medications currently. She denies any exposure to industrial chemicals and had worked as an administrative assistant prior to becoming pregnant. She says that she smoked tobacco in the past but quit 2 years ago. She had two previous pregnancies that went on to full-term uncomplicated vaginal deliveries. Her children are both healthy and have never been diagnosed with any medical problems. On examination, she is a well-appearing woman in no acute distress. Her face is symmetric and not
edematous. She has no lymphadenopathy. Auscultation of her lungs and heart detects clear breath sounds and normal heart sounds. Her abdomen is gravid and nontender. Her neurovascular examination is normal. A pelvic examination detects a closed cervical os with no discharge and no purulent fluid in the vaginal canal. Her cervix and ovaries are nontender. Her fundal height is 35 cm. Doppler sonography detects fetal heart sounds in the superior portion of the gravid abdomen. The following vital signs are measured: T: 98.4°F, HR: 90 bpm, BP: 110/80 mm Hg, RR: 19 breaths/min
Differential Diagnosis Malpresentation, multiple gestations
Laboratory Data and Other Study Results The following records from the patient's prior prenatal care are brought by her to the obstetrics appointment: Initial height and weight: 5﹐ 3", 117 lb, BMI 20.7 CBC: WBC: 10.0, Hgb: 14.2, Plt: 403 10-electrolyte chemistry panel (Chem10): Na: 139 mEq/L, K: 4.0 mEq/L, Cl: 101 mEq/L, CO2: 25 mEq/L, BUN: 14 mg/dL, Cr: 0.6 mg/dL, Glu: 88 mg/dL, magnesium (Mg): 1.9 mg/dL, calcium (Ca): 9.4 mg/dL, phosphorus (Phos): 3.7 mg/dL Coags: PT: 11.6 sec, INR: 1.0, PTT: 31.2 sec Maternal blood type: B+ UA: straw colored, pH: 6.8, specific gravity: 1.015, no glucose/ketones/nitrites/leukocyte esterase/hematuria /proteinuria RPR: negative Maternal rubella IgM antibodies: negative Maternal rubeola IgM antibodies: negative Papanicolaou (Pap) smear: multiple normal squamous cells without atypia The following additional tests are ordered on this obstetrics visit: Height and weight: 5﹐ 3", 142 lbs UA: straw colored, pH: 6.9, specific gravity: 1.015, no glucose/ketones/nitrites/leukocyte esterase/hematuria /proteinuria Chem7: Na: 140 mEq/L, K: 4.0 mEq/L, Cl: 103 mEq/L, CO2: 24 mEq/L, BUN: 14 mg/dL, Cr: 0.7 mg/dL, Glu: 90 mg/dL Maternal N. gonorrhoeae and C. trachomatis immunoassays: negative Hepatitis B surface antigen (HBsAg): negative HIV ELISA: negative Anti-HCV: negative Maternal group B streptococcus antigen: positive Biophysical profile: score of 10; single intrauterine pregnancy in frank breech presentation with no visceral anomalies seen on US
Diagnosis Frank breech malpresentation (also positive maternal group B streptococcus infection)
Treatment Administered External cephalic version was performed to attempt to correct the fetal presentation but was unsuccessful
Follow-up A repeat US performed 1 week later found the fetus to still be in the frank breech presentation An elective cesarian delivery was scheduled for the 39th week of gestation IV penicillin G was administered at the time of the cesarian delivery for fetal prophylaxis against group B streptococcus infection The cesarian delivery proceeded successfully, and both the mother and baby were stable following the procedure
Steps to the Diagnosis Malpresentation The normal fetal presentation is a vertex presentation with the fetal head inferior, the chin tucked, and the occiput directed toward the birth canal Other possible presentations include the brow, face, and breech presentations Breech presentation is the most common form of malpresentation (Figure 12-7) Frank breech makes up the majority of cases and occurs with the hips flexed and knees extended
Ninety-five percent of pregnancies will be in the vertex presentation at the time of delivery.
Figure 12-7 Examples of frank (A), complete (B), and footling (C) variations of breech presentation.
In complete breech both the hips and knees are flexed In footling breech one or both hips are extended Brow presentation is rare and occurs with the head inferior and the neck partially extended Face presentation is rare and occurs with the head inferior and the neck hyperextended Risk factors: preterm labor, multiple gestations, polyhydramnios, uterine anatomic anomaly, placenta previa History: typically asymptomatic Physical examination: external examination of the fetal presentation (i.e., Leopold maneuvers) will detect the fetal head superiorly for breech presentation Tests: US will clearly define the fetal presentation Treatment: The majority of cases will resolve prior to the onset of labor External cephalic version may be attempted at 37 weeks of gestation to attempt to reposition the fetus Vaginal delivery may be attempted for frank breech and face presentations Cesarian delivery is required for the other forms of malpresentation and is utilized in most frank breech presentations Outcomes: complications include cord prolapse, head entrapment within the birth canal, failure of labor to progress, fetal hypoxia, abruptio placentae, and both fetal and maternal birth trauma; because of the success and relative low risk of cesarian delivery, this option is frequently selected for any concerning malpresentation Clues to the diagnosis: History: noncontributory Physical: superior displacement of the fetal heart tones Tests: US appearance of the fetal presentation
Although 25% of pregnancies will have the fetus in a breech presentation prior to 28 weeks of gestation, the majority of cases will self-correct before they are full term.
External cephalic version is successful in only 40% of attempts. Multiple gestations Any pregnancy in which more than one fetus develops at the same time Types Monozygotic: division of the zygotes results in the development of two identical fetuses that may or may not share the same amniotic sac (Figure 12-8) Dizygotic: fertilization of multiple ova released during the same ovulation that results in the development of
dissimilar (i.e., fraternal) fetuses with separate amniotic sacs Risk factors: family history, older maternal age, infertility therapy (e.g., clomiphene, transfer of multiple ova) History: perceived greater-than-normal abdominal size Physical examination: increased fundal height for gestational age, multiple fetal heart tones detected by Doppler sonography Tests: US will detect the multiple developing fetuses Treatment: Close maternal observation is required from 24 weeks of gestation onward Activity restrictions
Figure 12-8 The various placenta and amniotic arrangements of twin pregnancies. The percentages correspond to the frequency in all twin gestations.
Figure 12-9 Various fetal presentations in twin gestations. (A) Vertex-vertex. (B) Vertex-breech. (C) Breech-breech. (D) Vertex-transverse.
Frequent reassessments of fetal well-being, and weekly nonstress tests are indicated after 36 weeks of gestation Preterm labor should be prevented with tocolytics Vertex-vertex presentations may be delivered vaginally, but other presentations (e.g., vertex-transverse, vertex-breech) will require cesarian delivery (Figure 12-9) Outcomes: there is an increased incidence of pre-eclampsia, preterm labor, fetal malpresentation, placenta previa, abruptio placentae, PROM, IUGR, birth trauma, cerebral palsy, and respiratory distress syndrome with multiple gestations; the rate of fetal demise is four times higher than single gestations Why eliminated from differential: the detection of a single gestation on the US rules out this diagnosis
Conjoined twins only occur in cases of monozygotic twinning.
If fetal heart sounds are found in an unusual location of the abdomen, continue to look for additional sets of heart sounds to help differentiate malpresentation of a single gestation from multiple gestations.
The average delivery age for twins is 36 weeks of gestation.
If the umbilical cords for multiple fetuses are fused, twin-twin transfusion syndrome may result, in which one twin is inadequately perfused and there is an increased risk of fetal ischemic complications.
Authors: Van Kleunen, Jonathan P. Title: Step-Up to USMLE Step 3, 1st Edition Copyright ©2009 Lippincott Williams & Wilkins > Table of Contents > Chapter 13 - Psychiatry
Chapter 13 Psychiatry Basic clinical primer Initial Psychiatric Evaluation Patient history The patient's chief complaint (i.e., what he or she perceives as wrong) may not be the only issue that needs to be addressed A review of systems and discussion of any past medical history should occur to help determine if any medical conditions are contributing to a psychiatric disturbance A family history is useful to determining if any psychiatric behaviors are inherited The social history should address substance use, employment history, legal issues, relationships, education, sexual history, and any precipitating social factors for behaviors; the effects of these issues on daily function should be discussed It may be helpful to involve family members or friends in the evaluation to help obtain a more complete history (with the patient's permission) Physical examination The physical examination should be used to detect any contributory medical conditions (frequently performed by another provider prior to the psychiatric evaluation) Diagnostic tests Blood or urine analysis is useful for identifying any medical conditions or substance use A head computed tomography (CT) or magnetic resonance imaging (MRI) may be used to detect brain lesions Classification Psychiatric disorders are organized by the multiaxis classification system (Table 13-1)
Mental Status Examination Appearance, behavior, and cooperation Patient appearance and behavior help define the ability to function and the patient's mental state (e.g., depression, mania, psychosis) Patient gait, coordination, posture, and facial expressions are considered to be components of appearance
The patient's ability to cooperate with the evaluation helps to suggest the impact of his or her mental state on the ability to function with others Speech The tone, rate, rhythm, articulation, and volume of the patient's speech should be noted Excessive or exaggerated qualities may suggest mania or aggressive behavior Impoverished qualities may suggest a depressed mood, social withdrawal, or impaired consciousness
The components of the mental status examination may be remembered by the mnemonic ABC STAMP LICKER: Appearance, Behavior, Cooperation, Speech, Thought processes and content, Affect, Mood, Perceptions, Level of consciousness, Insight and judgment, Cognition, Knowledge base, Endings, Reliability.
Ability to function generally refers to a patient's ability to live independently, perform normal activities of daily life, and function as a contributing member of society. P.299
Table 13-1 Multiaxis Classification of Psychiatric Disorders
Axis
Category
Examples
I
Clinical psychiatric disorders
Mood disorders, anxiety disorders, substance abuse, delirium
II
Personality and development disorders
Borderline personality disorder, mental retardation
III
Medical conditions
Encephalopathy, neoplasm, HIV
IV
Psychosocial stresses
Support structures, social environment, occupational factors
V
Global assessment of functioning
(100-point scale that describes how well the patient has functioned in society)
HIV, human immunodeficiency virus. (From Diagnostic and Statistical Manual of Mental Disorders. 4th ed. Washington DC: American Psychiatric Association, 1994, with permission.)
Thought process and content Production of thought abnormalities include impoverished thoughts (i.e., decreased interaction with others or difficulty reasoning), blocked thoughts (i.e., sudden stops in the thought process), or “flight of ideas” (i.e., rapid transitions between incomplete thoughts) Form of thought abnormalities include circumstantiality (i.e., unnecessary detail en route to a conclusion), tangentiality (i.e., deviations from the original topic to another), loose associations (i.e., illogical jumps between different subjects), “word salad” (i.e., use of individual words without logical sense or sentence
form), or neologisms (i.e., invented words) Abnormal thought content may include phobias (i.e., abnormal fears), obsessions (i.e., persistent and pervading thoughts), compulsions (i.e., urges to perform a particular task), and delusions (i.e., false beliefs without a realistic basis) Mood and affect Mood is the patient's subjective emotional state Affect is the way a patient expresses his or her state of mood Perception Involves the processing of sensory information Hallucinations are the abnormal processing of sensory information or the self-creation of a sensory experience Level of consciousness Level of alertness and any variability during an evaluation Insight and judgment Insight is the patient's awareness of any problems and his or her effect on daily life A patient may not realize the effect of a psychiatric problem on his or her ability to function or may not be aware that treatment can improve function A patient's insight into his or her condition will progress through stages of precontemplation (i.e., denial), contemplation (i.e., thought of addressing the problem), preparation (i.e., forming a plan to deal with the problem), action (i.e., implementing the plan), and maintenance (i.e., making sure that changes are maintained) Judgment is a patient's response to a given situation Cognition Orientation is the awareness of time, place, and identity Memory is the ability to retrieve mentally stored information Concentration is the ability to maintain attention to a task Knowledge base Any logical or chronological gaps in common knowledge should be noted The patient's intelligence (i.e., the ability to learn, process, and retain new information) must be taken into account P.300 Endings Suicidal or homicidal ideations must be taken seriously, and a proper risk assessment must be performed in these cases Reliability
Any information gathered during an evaluation must be reassessed to determine if it is reliable The ability of the patient to participate in the evaluation should be considered
Case 13-1: “My wife is very moody” A 34-year-old woman presents to a psychiatrist for an initial evaluation for moodiness. She is accompanied by her husband. She says that her moods have fluctuated significantly since she was in her mid-20s, and she currently feels that she is unable to cope with the daily demands on her life. Her husband was concerned for her well-being and finally convinced her to see a psychiatrist. She says that she has been “in a funk” for the past month and that she feels tired and “down” all of the time. There have been some days in the past month that she has stayed in bed almost the entire day. She is upset that she has stopped keeping the house clean and cooking in the past few weeks. She says that she is more of a burden than a help to her husband and two teenaged children, and that she cannot see this situation ever getting better. She denies having visions or hearing voices or having thoughts of harming herself. She says that she is fatigued but denies any significant weight gain. She denies headaches, chest pain, or abdominal pain. Her husband confirms most of her story. He says that she has been very lethargic in the past month and that she spends most of her time in bed. He has absorbed most of the duties of running the household during this time. He must convince her to eat because she says that she never has an appetite. He denies that she has ever discussed harming herself or others. He says that his wife goes through these episodes of feeling down once or twice per year and that they usually last between a few weeks to a month. They then seem to resolve over the course of a few days, and she is able to function normally as a stay-at-home mother. When she is functionally normal, it seems as that the down episodes had never happened and that she does not seem to recall the extent of her previous behavior. On three occasions in the past 10 years she has emerged from one of her down periods and become extremely active. She tends to undertake multiple jobs at these times that are not always completed because she switches between tasks. He remembers coming home from work on one occasion to find that she had repainted every bedroom in the house in one day. On these few occasions she is unable to sleep and performs housework through the night because she has so much excess energy. He says that the last time one of these energetic episodes occurred was a year ago and that it lasted 1 week before she became “more normal.” The patient denies having any medical problems or taking medications. She says that she occasionally drinks alcohol but denies other substance use. Her husband says that she tends to drink more when she is in a down period. On further mental status examination, the patient appears slightly disheveled. Her hair appears unwashed and uncombed. She is wearing a clean sweat suit. She answers questions appropriately but otherwise volunteers little information beyond what is pertinent to the current question. She is able to complete her sentences and thoughts. She has no abnormal patterns of speech. Her affect appears to be depressed based on her speech and lack of activity. She remains alert throughout the interview. She is oriented to her name, the current place, and the date. When given the name of three objects, she is able to repeat the names of the objects but can only remember two of them 5 minutes later. A brief physical examination is normal and does not detect any abnormal cardiopulmonary findings, masses, sites of tenderness, or neurologic abnormalities. The following vital signs are measured: Temperature (T): 98.5○F, heart rate (HR): 78 beats per minute (bpm), blood pressure (BP): 122/81 mm Hg, respiratory rate (RR): 17 breaths/min
Differential Diagnosis Bipolar disorder, major depressive disorder, dysthymia, cyclothymia, alcohol abuse, schizoaffective disorder, hypothyroidism, Cushing syndrome P.301
Laboratory Data and Other Study Results Complete blood cell count (CBC): white blood cells (WBC): 7.7, hemoglobin (Hgb): 14.1, platelets (Plt): 248 7-electrolyte chemistry panel (Chem7): sodium (Na): 139 mEq/L, potassium (K): 3.8 mEq/L, chloride (Cl): 102
mEq/L, carbon dioxide (CO2): 27 mEq/L, blood urea nitrogen (BUN): 17 mg/dL, creatinine (Cr): 0.8 mg/dL, glucose (Glu): 79 mg/dL Thyroid-stimulating hormone (TSH): 3.1 µU/mL Urinalysis (UA): straw colored, pH: 6.7, specific gravity: 1.010, no glucose/ketones/nitrites/leukocyte esterase/hematuria/proteinuria Urine toxicology screen: negative
Diagnosis Bipolar disorder, type I
Treatment Administered The patient was started on lithium and paroxetine The patient was referred to psychotherapy to help identify stressors and precipitating events for her depression
Follow-up By report of the patient's husband, the patient's mood improved 2 weeks later The patient and her husband reported a more consistent mood over the next 2 years with no recurrent “high” episodes and only three episodes of minor depression with no major depressive episodes
Steps to the Diagnosis Bipolar disorder (a.k.a. manic depression) Cyclic depression and mania (or hypomania) that impairs a patient's ability to function during episodes of significant depression and elation The patient is able to function normally between the episodes Types: Type I (i.e., manic): periodic depression with the history of at least one manic episode Type II (i.e., hypomanic): periodic depression with the occurrence of at least one hypomanic episode The diagnosis requires a history of at least one manic or hypomanic episode and recurrent major depressive episodes History: Depression is consistent with that seen in major depressive disorder Mania Elation or irritability lasting at least 1 week Three or more of the following findings: grandiosity, pressured speech, decreased need for sleep, flight of ideas, easy distractibility, increased goal-oriented activity, or increased risky pleasurable activity
Episodes cause a significant impairment in the ability to function Hypomania Elation or irritability lasting at least 3 days At least three of the following findings: grandiosity, pressured speech, decreased need for sleep, flight of ideas, easy distractibility, psychomotor agitation, engaging in risky pleasurable activity The ability to perform daily functions is maintained and not impaired significantly P.302 Physical examination: Depression Decreased activity, disheveled or unkempt appearance, poor eye contact, depressed affect Mania and hypomania Excessive activity, agitation, outlandish appearance Tests: typically noncontributory but may help to rule out other diagnoses Treatment: Any patient considered to be psychotic or at risk to him- or herself or others should be admitted for inpatient treatment until his or her mood can be stabilized Mood stabilizers (e.g., lithium, anticonvulsants atypical antipsychotics) are used to control and prevent manic and hypomanic episodes Lithium has long been the first-line treatment for maintenance therapy but is associated with multiple adverse effects, including hypothyroidism, polyuria, tremors, weight gain, renal insufficiency, teratogenesis, and decreased cognitive function Some of the atypical antipsychotics are now also considered initial medications because of a similar degree of effectiveness and a better side effect profile Rapid cycling may respond better to carbamazepine and valproate than lithium Antidepressants are used to treat depression Outcomes: Full control of episodes is difficult to achieve, and 50% of patients will have additional episodes of mania or major depression In patients who are successfully treated, the minority will never have a recurrent episode, and many will have recurrent, but less frequent, episodes Substance abuse, prolonged depression, psychotic features, frequent manic episodes, and suicidal or homicidal ideation are all associated with a worse prognosis Suicide or homicide is considered the worst potential complication Clues to the diagnosis: History: recurrent major depressive elements (e.g., depressed mood, fatigue, poor appetite, difficulty performing normal activities), occurrence of manic behaviors (e.g., increased goal-oriented activity,
decreased sleep need, easy distractibility) Physical: unkempt appearance of patient, paucity of speech, depressed affect, forgetfulness Tests: noncontributory
On average, most bipolar patients experience four cycles of episodes in a 10-year period, but some may experience rapid cycles.
The characteristics of manic episodes may be remembered with the mnemonic DIGFAST: Distractibility, Insomnia, Grandiosity (feelings of), Flight of ideas, Activity (increase in goal-oriented), Speech (pressured), Taking risks.
The characteristics of major depressive disorder may be remembered with the mnemonic SIG E CAPS: Sleep disturbances (insomnia), Interest loss, Guilt, Energy reduction (fatigue), Concentration impairment, Appetite changes, Psychomotor disturbances, Suicidal ideation.
Always assess the depressed patient for suicidal risk.
A prior history of mania must be ruled out by a thorough history in a patient suspected of having major depressive disorder before antidepressants are prescribed. Antidepressants given to a patient with bipolar disorder who is not taking mood stabilizers can induce a manic episode. Major depressive disorder Significant depression that impairs the patient's ability to function, lasts at least 2 weeks, and is not attributable to substance use, medical comorbidities, or bereavement The patient is able to function normally between episodes Psychotic features are rare but may occur The causative pathology is not fully understood but may be related to serotonin, norepinephrine, or dopamine activity The diagnosis requires the presence of five depressive historical findings including either depressed mood or anhedonia lasting >2 weeks History: depressed mood, anhedonia (i.e., a loss of interest in previously pleasurable activity), changes in sleep patterns (e.g., insomnia, hypersomnia), feelings of worthlessness, fatigue, inability to concentrate, changes in appetite, psychomotor retardation (i.e., decreased motor activity), suicidal ideation Physical examination: frequently noncontributory, although the patient's appearance may be disheveled Tests: noncontributory Treatment: Antidepressants are the mainstay of pharmacologic therapy (Table 13-2) Psychotherapy consists of cognitive or behavioral counseling and instruction designed to provide insight
into the depressive condition and to modify exacerbating behaviors and environments P.303
Table 13-2 Antidepressant Medications
Drug/Class
Mec hanism
Indic ations
Adverse Effec ts
SSRIs (e.g., fluoxetine,
Block presynaptic serotonin
First-line treatment for
Require 3–4 w eeks of administration
sertraline, paroxetine,
reuptake to increase synaptic
depression
before they take effect; sexual
citalopram,
free-serotonin concentration
dysfunction, decreased platelet
escitalopram)
and postsynaptic serotonin
aggregation, possible increased risk
receptor occupancy
of suicidal ideation in adolesc ents
SNRIs (e.g.,
Inhibit reuptake of both
First-line treatment for
venlafaxine,
serotonin and norepinephrine
depression with comorbid
sedation, constipation, HTN; side
duloxetine)
in similar fashion to TCAs
neurologic pain; second-line
effects more benign than TCAs
Nausea, dizziness, insomnia,
treatment for patients failing SSRIs
TCAs (e.g., imipramine,
Block norepinephrine and
Second-line treatment for
Easy to overdose and may be fatal at
amitriptyline,
serotonin reuptake to
depression; may be useful in
only five times the therapeutic
desipramine,
potentiate postsynaptic
patients with comorbid
dose (due to cardiac QT interval
nortriptyline)
receptor activity
neurologic pain
prolongation that causes arrhythmias), sedation, weight gain, sexual dysfunction, antic holinergic symptoms
MAOIs (e.g.,
Block monoamine oxidase
Second-line treatment for
Dry mouth, indigestion, fatigue,
phenelzine,
activity to inhibit deamination
depression; may be
headache, dizziness; consumption of
isocarboxazid,
of serotonin, norepinephrine,
particularly useful in
foods containing tyramine (e.g.,
tranylcypromine,
and dopamine and increase
treatment of depression
cheese, aged meats, beer) may cause
selegiline)
levels of these substances
with neurologic symptoms
hy pertensive c risis
or in refractory cases
Bupropion
Poorly understood but may be
Depression with fatigue and
related to inhibition of
difficulty concentrating or
dopamine reuptake and
comorbid ADHD
Headache, insomnia, weight loss
augmentation of norepinephrine activity
Trazodone
Mirtazapine
Poorly understood but related
Depression with significant
Hypotension, nausea, sedation,
to serotonin activity
insomnia
priapism; seizure risk at high doses
Blocks α2 receptors and
Depression with insomnia
Dry mouth, weight gain, sedation
serotonin receptors to increase adrenergic neurotransmission
St. John's wort
Decreases reuptake of
Used as first-line agent in
GI distress, dizziness, sedation; drug
(Hypericum
serotonin and to a lesser
Europe but only considered
interactions are common
perforatum)
extent norepinephrine and
an alternative therapy in
dopamine
the United States
ADHD, attention-deficit hyperactivity disorder; HTN, hypertension; GI, gastrointestinal; MAOIs, monoamine oxidase inhibitors; SNRIs, selective serotonin/norepinephrine-reuptake inhibitors; SSRIs, selective serotonin-reuptake inhibitors; TCAs, tricyclic antidepressants.
Electroconvulsive therapy (ECT) may be used to treat refractory or severe cases to decrease the frequency of episodes
Outcomes: up to 80% of patients will have an improvement in their symptoms with treatment; suicide is considered the most concerning complication Why eliminated from differential: the description by the patient's husband of multiple manic episodes in the patient makes a bipolar mood disorder more likely than a depressive disorder alone
Risks for a successful suicide attempt include age >45 years, violent behavior, drug use, prior suicidal attempts, existence of a suicide plan, male gender, recent significant loss, depression, unemployment, or being single, widowed, or divorced. Dysthymia Feelings of a depressed mood that occur on more days than not for more than 2 years The patient must have no prior history of any major depressive episodes A diagnosis requires a depressed mood and at least two historical findings for a majority of days for at least 2 years P.304 History: depressed mood, feelings of hopelessness, changes in sleep patterns, changes in appetite, fatigue, inability to concentrate, or low self-esteem Physical examination: typically noncontributory Tests: noncontributory Treatment: psychotherapy is frequently the initial approach to treatment; antidepressants may be added for persistent symptoms Outcomes: these cases are much milder than major depressive disorder but may be more chronic in nature; dysthymia in children or younger adults may be a precursor to other mood disorders Why eliminated from differential: the significant impact of the patient's symptoms upon her daily life and the occurrence of multiple manic episodes rule out this diagnosis
Seventy-six percent of children with dysthymic disorder develop major depressive disorder, and 13% develop bipolar disorder. Cyclothymia Rapid cycling of hypomanic and mild depressive episodes for >2 years with no period of normal mood lasting >2 months Patients retain the ability to function normally despite their mood History: alternating symptoms of dysthymia (e.g., depressed mood, feelings of hopelessness, etc.) and hypomania (e.g., irritability, grandiosity, etc.) Physical examination: typically noncontributory Tests: noncontributory Treatment: a combination of antidepressants, mood stabilizers, and psychotherapy is utilized to optimize mood control Outcomes: the prognosis is similar to that for bipolar disorder, with recurrences being common despite
therapy Why eliminated from differential: the significant impact of the patient's symptoms upon her ability to function and the absence of rapid cycles rule out this diagnosis Alcohol abuse More thorough discussion in later case Why eliminated from differential: although the husband's report of the patient's alcohol use is of concern and substance abuse is very common in patients with mood disorders, the patient's alcohol use does not sound as if it is a constant significant problem, and the significant fluctuations and inconsistency of the patient's mood are unlikely to be due to alcohol alone Schizoaffective disorder More thorough discussion in later case Why eliminated from differential: the absence of psychotic symptoms makes this diagnosis unlikely Hypothyroidism More thorough discussion in Chapter 5 Why eliminated from differential: the normal TSH rules out this diagnosis Cushing syndrome More thorough discussion in Chapter 5 Why eliminated from differential: the absence of any significant physical findings decreases the concern for this diagnosis
Case 13-2: “I hate talking to people” A 36-year-old woman presents to a psychiatrist because she is frustrated with her persistent discomfort in talking to people. She says that she has always been a shy person and that this has affected many aspects of her life. As part of this shyness, she has the frequent fear that she will do something to embarrass herself around others. She says that when she was a teenager she never felt comfortable in social situations because she was afraid of how people perceived her. Whenever she was invited to a dance or out on a date, she would become dizzy and would have palpitations and shortness of breath. When she went to college, she chose to remain in her dorm room most of the time not spent in class for the same reason. P.305 She never went out with people at night or on the weekends because of not feeling comfortable around others. She says that she rationalized that her studies should take precedence over any social activity. Following college, she began to work as an actuary for an insurance company. She has performed well at her work, in part, she believes, because she rarely has to interact with people outside of the office. Recently, she was passed over for a promotion because she became extremely nervous before her interview and performed poorly. On that occasion, she began to feel short of breath and lightheaded during the conversation with her boss. She is extremely frustrated over this event because she says that she was the most qualified person interviewing for the job. She says that she has never been in a committed relationship because of her concern over meeting new people. She would like to meet someone, but when she has been set up for dates by other people, she has consistently had the feelings of dizziness and shortness of breath on the night of the date and has canceled them. She has had a few close friendships over the years but typically has few general acquaintances. She says that she enjoys talking to people via email or on Internet chat rooms and does not have symptoms in these situations. Likewise, she says that she never has had a problem working with classmates in college or interacting with people at work within the work setting. She says that
when she is at home or alone at her desk at work she feels fine and does not experience the feeling of uneasiness. She denies feeling sad, but is very frustrated over the perceived limitation of her behavior. She denies any hallucinations or suicidal ideation. She says that she realizes that this behavior is not normal but feels incapable of changing. She denies any past medical history and only takes a multivitamin. She is unaware of a similar history in any family members. She occasionally drinks wine at home but uses no other substances. On the mental status examination, she is well groomed and dressed professionally. She is anxious-appearing during the entire interview but is able to answer questions appropriately and volunteers her history. Her speech is rapid and slightly pressured, and she chews her fingernails occasionally when not talking. She is able to fully describe her concerns and problems without deviating from the subject. There are no abnormalities in her phrasing of statements. She remains alert and focused throughout the interview. She is oriented to name, place, and time. She is able to repeat the names of three listed objects and recalls all three objects later in the conversation. The following vital signs are measured: T: 98.6○F, HR: 105 bpm, BP: 135/86 mm Hg, RR: 18 breaths/min
Differential Diagnosis Panic disorder, social phobia, specific phobia, generalized anxiety disorder, schizoid personality disorder, avoidant personality disorder
Laboratory Data and Other Study Results None performed
Diagnosis Social phobia
Treatment Administered The patient was placed on paroxetine The patient was referred to behavioral psychotherapy to focus on a gradual progressive exposure to social situations and to evaluate her perceptions of others in such situations
Follow-up By 2 months after the onset of therapy, the patient reported feeling more comfortable in social situations and was no longer having any physical symptoms during interactions with others P.306 By six months, the patient continued to report feeling generally “shy,” but was able to interact with acquaintances on a regular basis and was able to begin dating other people
Steps to the Diagnosis Social phobia Excessive fear of social situations and anxiety that results when the patient encounters such situations The onset is typically in childhood The anxiety cannot be associated with any medical conditions or substance use History: anxiety that consistently occurs in certain social settings (e.g., performances, conversations), severe anxiety that may lead to panic attacks (i.e., multiple physical symptoms that accompany the anxiety),
persistent fear of being embarrassed, avoidance of social situations because of anxiety, difficulty functioning normally or in forming relationships because of the anxiety, realization that the feelings of anxiety are abnormal Physical examination: the patient frequently appears anxious during the interview Tests: typically noncontributory Treatment: Selective serotonin reuptake inhibitors (SSRIs) are frequently effective at reducing anxiety and improving social interactions Benzodiazepines and monoamine oxidase inhibitors (MAOIs) are alternative treatments but carry more side effects than SSRIs; benzodiazepines may be particularly helpful for reducing acute anxiety (Table 13-3) β-blockers may be useful in mild cases to decrease palpitations, tachycardia, and diaphoresis Psychotherapy is useful for instructing the patient not to focus on others' perceptions and to desensitize him- or herself to social interactions Outcomes: mild cases have a good prognosis, but more disabling cases may not improve to the same extent Clues to the diagnosis: History: anxiety in social situations with occasional panic attacks, fear of embarrassment, difficulty forming new relationships, realization that behavior is abnormal Physical: anxious affect Tests: none performed Panic disorder The experience of recurrent, spontaneous panic attacks with the associated fear of recurrence Typically begins in adolescence Anxiety cannot be explained by a comorbid medical condition or substance use Severe panic disorder may lead to agoraphobia (i.e., a severe fear of public places)
Table 13-3 Anxiolytic Medications
Drug
Mec hanism
Indic ations
Adverse Effec ts
Benzodiazepines (e.g.,
Increase GABA
Alprazolam has a rapid onset and
Sedation, confusion; stopping
alprazolam, clonazepam,
inhibition of
short half-life and is particularly
alprazolam usage is associated with
diazepam, lorazepam)
neuronal firing
useful to break panic attacks;
withdrawal symptoms of restlessness,
clonazepam and diazepam are more
confusion, and insomnia (especially
useful for prolonged therapy
with frequent use)
Unclear, but
Anxiety disorders in which abuse or
Headaches, dizziness, nausea
related to
sedation is a concern
Buspirone
serotonin and dopamine receptors
GABA, gamma aminobutyric acid.
P.307 A diagnosis requires both a history of recurrent panic attacks and a pervasive fear that the attacks will recur History: recurrent panic attacks that occur without warning, last up to 30 minutes, and consist of extreme anxiety, feelings of impending danger, chest pain, dyspnea, palpitations, nausea, dizziness, feelings of losing control, and chills or hot flashes Physical examination: tachycardia and diaphoresis during the attacks Tests: typically noncontributory but useful to rule out cardiac conditions Treatment: SSRIs are considered the first-line therapy to reduce the frequency of panic attacks Benzodiazepines or tricyclic antidepressants (TCAs) are considered second-line therapy Psychotherapy may be useful for reducing the fear of recurrence between attacks and reducing the frequency of attacks Outcomes: the prognosis is excellent, with most patients having a significant reduction in the number of attacks after starting treatment Why eliminated from differential: despite the occurrence of multiple panic attacks, the close relationship of the patient's symptoms to social settings makes social phobia a better diagnosis
Patients with panic disorder have a higher incidence of mitral valve prolapse. Specific phobia Fear of a particular object, activity, or situation that causes a patient to avoid the feared subject Typically begins in childhood and cannot be associated with any medical comorbidity or substance use History: occurrence of a panic attack when the feared subject is encountered, avoidance of contact with the feared subject, realization that behavior is irrational, possible syncope during panic attacks Physical examination: typically noncontributory except in the presence of the feared subject (then consistent with a panic attack) Tests: noncontributory Treatment: psychotherapy is the cornerstone of treatment, with a focus on repeated graduated exposure, relaxation techniques, insight modification, and possible hypnosis Outcomes: the prognosis is good, with the majority of patients being able to function normally; more specific phobias (e.g., snakes) tend to have a better response to treatment than more general ones (e.g., public places) Why eliminated from differential: social phobia is essentially a form of specific phobia with a generalized fear of social interactions
Generalized anxiety disorder (GAD) Excessive, persistent anxiety that impairs a patient's ability to function Occurs on the majority of days for at least 6 months Typically begins in early adulthood and cannot be explained by medical comorbidities or substance use Diagnosis requires excessive anxiety, impaired ability to function, and at least three anxiety-related symptoms for at least 6 months History: restlessness or feeling on edge, inability to concentrate, insomnia, irritability, tenseness of muscles, sleep disturbances Physical examination: possible tachycardia, tachypnea, tremor, or diaphoresis Tests: typically noncontributory Treatment: SSRIs, serotonin–norepinephrine reuptake inhibitors (SNRIs), and buspirone are now considered the first line of therapy Benzodiazepines are useful for the treatment of acute exacerbations Psychotherapy is useful to improve insight into the overlying anxiety and to teach relaxation techniques Outcomes: the prognosis depends on the level of impairment in daily function Why eliminated from differential: the patient does not have persistent anxiety, and her symptoms occur in a particular situation (i.e., social interactions), so this diagnosis is unlikely P.308 Schizoid personality disorder More thorough discussion in later case Why eliminated from differential: the demonstrated ability to form some relationships and the presence of normal (albeit, excessive) emotion make this diagnosis unlikely Avoidant personality disorder More thorough discussion in later case Why eliminated from differential: although social phobia and avoidant personality disorder may be very similar in some regards, social phobia is a better diagnosis in this case because this patient is able to interact socially via some formats (i.e., email, Internet)
All anxiety disorders have a greater frequency in women.
It is very difficult to commit suicide using an overdose of benzodiazepines because their lethal dose is >1,000 times the therapeutic dose. Flumazenil is a benzodiazepine antagonist that can reverse the effects of an overdose.
Case 13-3: “I keep thinking about my partner” A 26-year-old man presents to a psychiatrist on the request of his employer to be evaluated for repetitive intrusive
thoughts. The patient is a police officer in a large city but has been unable to work for the past few months. He was in a good state of health until 3 months ago. At that time he and his partner were involved in a narcotics bust that ended violently in a shootout between the police and several drug dealers. Although the patient was unhurt in this incident, his partner, who was standing next to him at the time, was shot in the face and killed. The patient had difficulty performing his job after this incident. He says that he has a dream multiple nights per week in which the scene of his partner's death replays. He denies having any hallucinations. Whenever he hears gun shots or any other sudden noise, he becomes very anxious and feels like he is short of breath. He has been unable to hold a gun since the incident because it reminds him of his partner. He says that he constantly feels guilty that he did not save his partner's life. The patient says that he has had difficulty sleeping and frequently becomes angry at people for minor issues. He no longer exercises or goes out with friends because he feels that he lacks the energy to do so. Because he was unable to perform his job as a police officer, he was put on medical leave. His department is now trying to determine if he will ever be able to return to work. The patient says that he has a history of reflux disease for which he takes ranitidine. He smokes a half pack of cigarettes per day and has done so for 10 years. He drinks about ten beers per week. On the mental status examination, he appears well groomed. He is cooperative with questioning and answers all questions appropriately. The tone of his speech is low with decreased volume. He is able to complete his sentences completely without any abnormal structure. His affect is flat, and he displays little emotion. He is able to remain alert during the extent of the interview. He is oriented to his name, the current place, and the time. He is able to repeat the names of three objects spoken to him and can recall one of the objects several minutes later. The following vital signs are measured: T: 98.6○F, HR: 85 bpm, BP: 130/80 mm Hg, RR: 15 breaths/min
Differential Diagnosis Major depressive disorder, dysthymia, posttraumatic stress disorder, adjustment disorder, bereavement, obsessive-compulsive disorder, schizophrenia, substance abuse
Laboratory Data and Other Study Results None performed
Diagnosis Posttraumatic stress disorder P.309
Treatment Administered The patient was prescribed sertraline Psychotherapy was initiated, including group therapy with police officers and veterans who had witnessed traumatic events The patient was provided counseling regarding moderation of his alcohol intake
Follow-up Because of the inability to perform his job, the patient was placed on long-term disability Over time, the patient reported resolution of his nightmares and inappropriate responses to loud noises; by 18 months after the incident he felt that he would be able to return to work The patient returned to the police force 20 months after the incident, working initially at a desk job and later on patrol
Steps to the Diagnosis Posttraumatic stress disorder (PTSD) A syndrome of anxiety symptoms that occurs following an exposure to a significantly stressful event Symptoms begin within 3 months of the event Diagnosis requires the patient to have been exposed to a traumatic event, to have symptoms of reliving the event, to avoid situations associated with the event, and to have symptoms of increased arousal Symptoms lasting more than a 1 month are considered acute, and those lasting longer than 3 months are considered chronic History: vivid dreams or recurrent intrusive thoughts about the event, avoidance of activity associated with the event, anhedonia, feelings of detachment, survivor guilt, social withdrawal, increased arousal (e.g., irritability, insomnia, difficulty concentrating) Physical examination: possible disheveled appearance, exaggerated startle reaction Tests: typically noncontributory Treatment: SSRIs and MAOIs are the most commonly used medications to reduce feelings of anxiety and obtrusive thoughts; psychotherapy, particularly in a group setting or activity-directed therapy, is useful to teach coping skills Outcomes: two thirds of patients will recover fully from the condition, although the average time for recovery is 3 years in treated patients and 5 years in untreated patients Clues to the diagnosis: History: history of traumatic event, recurrent dreams of the event, avoidance of gun use due to an association with the event, insomnia, irritability, anhedonia Physical: flat affect, forgetfulness Tests: none ordered Major depressive disorder More thorough discussion in prior case Why eliminated from differential: despite having several symptoms seen in major depressive disorder, the relation of the patients symptoms to a distinct traumatic event makes PTSD a better diagnosis Dysthymia More thorough discussion in prior case Why eliminated from differential: the patient's symptoms do not satisfy the time criteria for this diagnosis Adjustment disorder Behavioral and mood changes that occur within 3 months of a stressful event (e.g., death of a loved one, assault, divorce) and causes significant impairment in the ability to function Symptoms begin within 3 months of the event and end within 6 months of the conclusion of the event P.310
The symptoms are generalized and are not specific for situations associated with the stressful event History: distress in excess of what is expected for a given stressful event, difficulty concentrating, self-isolation, changes in sleep patterns, change in appetite Physical examination: noncontributory Tests: noncontributory Treatment: psychotherapy is the first-line treatment and is aimed at helping the patient cope with the stressful event; antidepressants may be considered in patients who do not respond to psychotherapy alone Outcomes: suicide rates and substance abuse are increased in patients with adjustment disorder Why eliminated from differential: the specificity of the patient's symptoms for the stressful event and elicitation of his symptoms by exposure to situations reminiscent of the event make PTSD a better explanation for his symptoms Bereavement A natural period of mourning after the death of a loved one The patient's ability to function is not impaired History: sadness, repetitive thoughts about the recently deceased, difficulty concentrating, feelings of guilt, difficulty accepting the loss with an eventual acceptance, changes in sleep or appetite Physical examination: noncontributory Tests: noncontributory Treatment: most cases will self-resolve; psychotherapy should be considered in longer-lasting cases Outcomes: the prognosis is excellent, with almost all patients recovering; persistent symptoms are suggestive of adjustment disorder or another condition Why eliminated from differential: the significant impact of the patient's symptoms on his life and the severe pervasive nature of his symptoms rule out his diagnosis Obsessive-compulsive disorder (OCD) Significant recurrent obsessions (i.e., recurrent intrusive thoughts that are difficult to suppress and are recognized as being abnormal) and compulsions (i.e., repetitive behaviors performed in response to an obsession that are directed at reducing distress) that affect daily life and the ability to function Typically begins in adolescence History: recurrent obsessions and compulsions that affect the ability to function normally, obsessions and compulsions that consume a significant amount of time in daily life, awareness that behaviors are abnormal, feelings of an inability to control behaviors, worsening of abnormal behaviors in stressful situations Physical examination: although there are no generalized physical findings, an examination may detect findings consistent with a particular compulsion (e.g., dry skin from constant hand washing, wounds from scratching or picking of skin) Tests: typically noncontributory Treatment: SSRIs or SNRIs are considered the first-line pharmacologic therapy
Antipsychotics may be used in patients not responding to other medications Behavioral psychotherapy is aimed at the recognition of obsessive thoughts and the conscious avoidance of the reflex compulsive behaviors Outcomes: although 70% of patients will have a significant improvement in their symptoms, OCD tends to be a chronic condition with periodic exacerbations Why eliminated from differential: although the patient is fixated upon the death of his partner, the lack of a compulsive behavior in response to these thoughts rules out this diagnosis
Patients with OCD may be at a higher risk for tic disorders. Schizophrenia More thorough discussion in later case Why eliminated from differential: although the patient has recurrent dreams and a flattened affect, he is not having any hallucinations or abnormalities in the processing of thoughts; furthermore, the distinct onset of his symptoms with the stressful event makes this diagnosis unlikely P.311 Substance abuse More thorough discussion in later case Why eliminated from differential: although the patient is consuming more than one alcoholic beverage per day, his alcohol and tobacco use are not the inciting force behind his intrusive thoughts and anxiety
Case 13-4: “We found this guy harassing people” A 48-year-old man is brought to a psychiatric emergency evaluation center by police after he was found yelling at people on the street. The police officers say that he was wandering on the sidewalk and approaching people demanding that they return his jacket. Several people contacted the police after feeling threatened. The officers say that they have seen the patient in the area for the past year and are aware of some erratic behavior, but are unaware of him threatening anyone previously. They say that he is homeless and lives underneath a bridge by the river. He has been seen wandering the area periodically asking for money but does not have a history of violence. The patient is resistant to speaking with others and requires much coaxing to answer questions. When the patient is asked by the psychiatrist on call why he was upset and accosting people, the man says that his jacket lets him “travel in time” and that the “bad people stole it.” He then begins to talk about buying “apples for [his] elephant” and later that “the president is [his] father.” When further questioned about his jacket, he is unable to describe the jacket or when he last had a jacket. He intermittently turns to his right side and speaks as if he was conversing with someone next to him, but no one is seated there. When asked if he hears voices, he does not respond but stares straight ahead. When asked if he is angry, sad, or happy, a similar response is offered. He is also unable to provide any information about any surviving family members, previous places of residence, or past medical history. He denies any substance use upon questioning. On the mental status examination, he is very disheveled and unkempt. He is dirty and smells bad. He is not very cooperative with questioning and requires encouragement to answer questions. He has little facial expression. The tone of his voice is low, and his words are not well articulated. He makes loose associations between different subjects as evidenced by the transition from talking about his jacket to talking about apples and the president. He displays little emotion during the interview, but becomes mildly agitated when talking about his jacket. Despite his lack of cooperation, he remains awake during the assessment. He knows his name but is not able to correctly name the day or place. When given the names of three objects, he is only able to repeat one named object and cannot recite any of them several minutes later. A physical examination detects no asymmetry of his face of lymphadenopathy. Auscultation of his heart, lungs, and abdomen is normal. He has no
abdominal tenderness or masses. He has no pain with range of motion of his neck or any of his extremities. A neurovascular examination is normal in regards to all sensory and motor function. He has multiple bites and wounds on his skin in various stages of healing. The following vital signs are measured: T: 98.4○F, HR: 72 bpm, BP: 138/86 mm Hg, RR: 18 breaths/min
Differential Diagnosis Schizophrenia, brief psychotic disorder, substance abuse, delusional disorder, schizoaffective disorder, schizophreniform disorder, bipolar disorder, dementia, stroke, brain tumor or abscess, encephalitis, vitamin B12 deficiency, hyperthyroidism, hypothyroidism, acute intermittent porphyria, syphilis, human immunodeficiency virus
Laboratory Data and Other Study Results CBC: WBC: 10.5, Hgb: 15.1, Plt: 372, mean corpuscular volume (MCV): 90 fL 10-electrolyte chemistry panel (Chem10): Na: 144 mEq/L, K: 4.2 mEq/L, Cl: 99 mEq/L, CO2: 25 mEq/L, BUN: 20 mg/dL, Cr: 1.1 mg/dL, Glu: 99 mg/dL, magnesium (Mg): 1.6 mg/dL, calcium (Ca): 10.1 mg/dL, phosphorus (Phos): 4.2 mg/dL P.312 Liver function tests (LFTs): alkaline phosphatase (AlkPhos): 76 U/L, alanine aminotransferase (ALT): 41 U/L, aspartate aminotransferase (AST): 40 U/L, total bilirubin (TBili): 0.8 mg/dL, direct bilirubin (DBili): 0.5 mg/dL ESR: 9 mm/hr Folate: 5.1 mg/mL Vitamin B12: 384 pg/mL Thyroid panel: TSH: 1.98 µU/mL, T4: 7.6 µg/dL, free T4 index: 7.0, T3: 1.0 ng/mL, T3 reuptake: 0.79 Rapid plasma reagin (RPR): negative Human immunodeficiency virus enzyme-linked immunosorbent assay (HIV ELISA): negative Urine toxicology screen: positive for alcohol and tobacco Head CT: no focal masses, lesions, or sites of ischemia; normal cerebral and ventricular volume; no sites of hemorrhage
Diagnosis Schizophrenia
Treatment Administered The patient was convinced to consent to an inpatient psychiatric admission Risperidone was prescribed for the patient
Follow-up Following 2 weeks of risperidone use, the patient was more organized and was better able to perform personal hygiene He remained relatively unemotional, but did not become agitated during conversations and ceased discussing his
former delusions The patient was able to disclose the names of local family members and was discharged to live at a relative's house 4 weeks after his admission During a follow-up appointment, the patient stated that he did not believe that he needed to continue taking his medicine; the patient was encouraged to continue his use of the risperidone and was cautioned about the problems of discontinuing antipsychotic medications Following this appointment, the patient was lost to follow-up; when the family was contacted, they said that patient had left their house and that they did not know his whereabouts
Steps to the Diagnosis Schizophrenia A severe psychotic disorder that causes significant limitations in the ability to function It is found at a significantly higher rate in the homeless and indigent populations because of the inability of those affected to function in society Typically begins in late adolescence Diagnosis requires the presence of two or more significant psychotic symptoms (i.e., delusions, hallucinations, disorganized speech, disorganized or catatonic behavior, or negative symptoms), the presence of the symptoms for at least 1 month within a 6-month period, and impaired social function for >6 months Risk factors: family history, maternal malnutrition or illness during pregnancy History: Positive symptoms: delusions, hallucinations (usually auditory), disorganized thoughts and behavior, thought broadcasting (i.e., belief that others can read the patient's thoughts or that thoughts are being transmitted to others), ideas of reference (i.e., belief that hidden meanings are found in common items) P.313 Negative symptoms: social withdrawal, flat affect (i.e., displaying little emotional response to stimuli), apathy, anhedonia, lack of motivation Cognitive symptoms: attention deficits, inability to organize or form abstractions, poor memory Although the patient's baseline function is impaired, periodic psychotic exacerbations will occur with a further worsening of symptoms Physical examination: physical findings are consistent with the functional limitations of the disease and include poor hygiene, flat affect, bizarre behavior, interaction with perceived hallucinations, and inattentiveness Tests: typically noncontributory but useful to rule out other conditions Treatment: Antipsychotics are the mainstay of therapy (Table 13-4) Psychotherapy may be useful in teaching the patient how to recognize symptoms of the disease Psychotic exacerbations frequently require inpatient treatment Outcomes:
The prognosis is generally poor, with a gradual deterioration in the ability to function over several years; over this time period symptoms generally wax and wane Positive prognostic factors include a history of a comorbid mood disorder, predominantly positive symptoms, and good support systems Negative prognostic factors include predominantly negative symptoms, neurologic abnormalities, and poor support systems Many patients have a history of substance abuse, and this may cause other complications associated with the substance (e.g., HIV, cirrhosis)
Table 13-4 Antipsychotic Medications
Drug Ty pe
Mec hanism
Atypical antipsychotics
Block
(e.g., clozapine,
dopamine
risperidone, olanzapine,
and
sertindole, quetiapine,
serotonin
ziprasidone,
receptors
paliperidone)
Indic ations
First-line drugs for maintenance therapy for psychotic disorders Clozapine is the most effective neuroleptic but
Adverse Effec ts
Anticholinergic effects, weight gain, arrhythmias, seizures; c lozapine carries a risk of agranuloc ytosis; the frequency and severity of side effects is significantly less than seen with traditional neuroleptics
is reserved for refractory psychosis due to the risk of agranulocytosis
Traditional high-potency
Block D2
(e.g., haloperidol,
dopamine
droperidol,
receptors
fluphenazine,
Strong positive symptoms Emergenc y c ontrol of psychosis or agitation Frequently second-line
thiothixene)
drugs for maintenance therapy
Extrapyramidal effec ts (e.g., dystonia, parkinsonism), tardive dy skinesia, antic holinergic effec ts (e.g., sedation, constipation, urinary retention, hypotension), confusion, sexual dysfunction, hyperprolactinemia, neuroleptic malignant syndrome, seizures, arrhythmias; least anticholinergic effects of traditional antipsychotics but highest rate of extrapyramidal effects
Traditional medium-
Block D2
potency (e.g.,
dopamine
trifluoperazine,
receptors
perphenazine)
Strong positive symptoms Frequently second-line
Similar to other traditional drugs; mix of moderate extrapyramidal and anticholinergic effects
drugs for maintenance therapy May be used in patients exhibiting significant extrapyramidal and anticholinergic side effects with other traditional neuroleptics
Traditional low-potency
Block D2
(e.g., thioridazine,
dopamine
chlorpromazine)
receptors
Strong positive symptoms Frequently second-line drugs for maintenance therapy
Similar to other traditional drugs; less extrapyramidal effects than more potent traditional antipsychotics but more anticholinergic effects
P.314 Medicine noncompliance is a common problem and complicates treatment Ten percent of cases will end in suicide Clues to the diagnosis: History: bizarre behavior, history of homelessness and erratic behavior, loose associations, delusions,
disorganized thoughts, poor memory Physical: disheveled appearance, flat affect, interaction with hallucinations Tests: noncontributory
High-potency antipsychotics have more extrapyramidal side effects and fewer anticholinergic side effects. Low-potency antipsychotics have fewer extrapyramidal side effects and more anticholinergic side effects.
Tardive dyskinesia is a complication of antipsychotic medications that begins after several months of therapy and is characterized by repetitive facial movements (e.g., chewing, lip smacking). It may improve with cessation of the inciting medication but sometimes is irreversible.
Neuroleptic malignant syndrome is an uncommon complication of antipsychotic medications that starts within days of usage and carries a high mortality rate. It is characterized by high fever, muscle rigidity, decreased consciousness, and an increased blood pressure and heart rate. It is treated by immediately stopping use of the drug and administering dantrolene.
While neuroleptic malignant syndrome and malignant hyperthermia are both characterized by high fevers, the former is associated with antipsychotic use and the latter with inhaled anesthetic use. Brief psychotic disorder More thorough discussion in Table 13-5 Why eliminated from differential: the patient's reported history of prior erratic behavior makes this diagnosis unlikely Schizoaffective disorder More thorough discussion in Table 13-5 Why eliminated from differential: the predomination of psychotic features makes this diagnosis unlikely; in addition, the diagnostic criteria for schizophrenia are fulfilled, making this diagnosis unlikely Schizophreniform disorder More thorough discussion in Table 13-5 Why eliminated from differential: because the patient's complete psychiatric history is unknown, it is difficult to fully rule out this diagnosis, but the report of prior strange behavior and chronic social disability makes schizophrenia a better classification for his presentation Delusional disorder More thorough discussion in Table 13-5 Why eliminated from differential: the presence of multiple psychotic symptoms besides delusions rules out this diagnosis
Bipolar disorder More thorough discussion in prior case Why eliminated from differential: although the patient's suboptimal cooperation and his active psychosis make a full assessment of his mood difficult, the predominance of his psychotic symptoms and the significant functional disability they place on him make schizophrenia a better diagnosis
Table 13-5 Psychotic Disorders Not Classified as Schizophrenia
Disorder
Schizophreniform
Desc ription
Symptoms similar to schizophrenia but last >1 month and <6 months;
Treatment
Antipsychotics, psychotherapy
patients return to normal function following resolution of psychotic episode; two thirds of patients will go on to develop true schizophrenia in the future
Schizoaffective
Delusional
Presence of mood disorder and psyc hotic symptoms but not meeting
Combination of antipsychotics with mood
the criteria for either diagnosis alone; diagnosis requires the presence of
stabilizers and/or antidepressants;
psychotic symptoms during a normal mood for more than 2 weeks
psychotherapy is a useful adjunct
Presence of one or more distinct realistic delusions lasting more than a
Antipsychotics, psychotherapy; SSRIs are
month without any other psychotic symptoms; patient is able to function
helpful when delusions are of a somatic
normally; unrealistic delusions are classified as sc hizophreniform
nature
disorder or sc hizophrenia
Brief psychotic
Sudden onset of psychotic symptoms (possibly stress-related) that last 1
Psychotherapy or short-term
month
antipsychotics; hospitalization is necessary if symptoms affect the ability to function
Shared psychotic
A sec ond patient accepts and becomes involved in the preexisting
Group psychotherapy, antipsychotics; the
(i.e., Folie à deux)
delusions of the primary patient
second patient's acceptance of delusions often wanes if separated from primary patient
SSRI, selective serotonin-reuptake inhibitors.
P.315 Dementia More thorough discussion in later case Why eliminated from differential: because the patient's symptoms are not limited to cognitive dysfunction, this diagnosis is unlikely Substance abuse More thorough discussion in later case Why eliminated from differential: although this patient has been shown to use both alcohol and tobacco, the significant psychotic symptoms are better explained by a diagnosis of schizophrenia Stroke/brain abscess/brain tumor More thorough discussion in Chapter 7
Why eliminated from differential: the negative head CT rules out these diagnoses Vitamin B12 deficiency More thorough discussion in Chapter 6 Why eliminated from differential: the normal vitamin B12 level rules out this diagnosis Hyperthyroidism/hypothyroidism More thorough discussion in Chapter 5 Why eliminated from differential: the normal thyroid panel rules out these diagnoses Encephalitis More thorough discussion in Chapter 7 Why eliminated from differential: the absence of focal neurologic deficits and the absence of an effusion on the head CT make this diagnosis unlikely Acute intermittent porphyria More thorough discussion in Chapter 6 Why eliminated from differential: the absence of neurologic deficits makes this diagnosis unlikely Syphilis More thorough discussion in Chapter 11 Why eliminated from differential: the negative RPR rules out this diagnosis Human immunodeficiency virus More thorough discussion in Chapter 6 Why eliminated from differential: the negative ELISA rules out this diagnosis
Case 13-5: “I can't find a good relationship” A 40-year-old woman presents to a psychiatrist because she feels that she is unable to maintain any committed relationships. She says that she wants help recognizing people who are not a good match for her. She says that she has fallen “head-over-heels” in love with several men, and that she has thrown herself full force into these relationships. In each case, the man was “bad for [her]” and the relationship ended in a serious argument. She has been in a couple of romantic relationships with women that have ended in a similar explosive way. She says that she typically has multiple sexual partners at a given time because “that's what gets men interested in you.” She says that she “just can't find a good person.” She says that she also has a difficult time making friends because she “always [manages] to find bad people.” She has a few close friends that she says are “complete saints and are just like [her] in every way.” She says that she has a new boyfriend who is “perfect” and is “exactly what [she] needs.” She says that she is paying for private tennis and ski lessons because they are activities in which her new boyfriend is involved and “therefore, must be things [she] should be doing.” Despite her excitement for this relationship, she is worried because of her previous negative relationships and wants “to know how to recognize if something is wrong.” She is afraid that she may never find anyone to be in a long-term relationship because of her past experiences. She says that she has encountered a similar pattern in her work. She has changed jobs four times in the past 2 years because she cannot find “a good place to work.” In her current job in an advertising agency she initially enjoyed it and spent P.316
long hours at work but now knows “that it is not a good place to work forever.” She has called out of work several days in the past few months because she does not want to be at her current job. She feels that her difficulty with commitments stems from her childhood. She says that both of her parents traveled often and were not around much to take care of her brother and her. She feels that her parents never really knew her well. She feels that because of this upbringing she was “never taught to recognize the bad people in life.” She denies being unhappy but is concerned about her love life. She denies having difficulty sleeping or eating. She adamantly denies ever having had auditory or visual hallucinations. She admits that she took a large amount of diazepam with considerable alcohol after the end of two different relationships and required treatment in an emergency department on each occasion. She says that she was not trying to commit suicide but wanted to “punish” her ex-lover in each case. She denies any current thought of suicide. She says that she has a medical history of irritable bowel syndrome that has responded well to eating small meals. She takes a multivitamin and uses a contraceptive vaginal ring for birth control. She says that she has two alcohol drinks each night to “help deal with the day.” She denies other substance use. On the mental status examination, she is well groomed and dressed well. She is able to cooperate well with the entire interview. Her speech is articulate, and she is frequently expansive in her answers to questions. Occasionally she must be guided back to the direction of the question to get a clear answer. Her affect is variable, and she is both occasionally calm and emotional when talking about some past relationships. She remains alert during the entire interview. She is oriented to her name, the current place, and the time. She is able to repeat the names of three objects listed to her and is able to recall all three names later in the conversation. The following vital signs are measured: T: 98.7○F, HR: 70 bpm, BP: 126/78 mm Hg, RR: 18 breaths/min
Differential Diagnosis Attention deficit hyperactivity disorder, borderline personality disorder, bipolar disorder, major depressive disorder, dysthymia, paranoid personality disorder, schizotypal personality disorder, histrionic personality disorder, narcissistic personality disorder, dependent personality disorder, substance abuse
Laboratory Data and Other Study Results None performed
Diagnosis Borderline personality disorder
Treatment Administered The patient was enrolled in psychotherapy designed to initially explore the patient's beliefs of why she was having difficulties in relationships and to help the patient gradually evaluate her actions and thought processes in relationships The patient was prescribed olanzapine
Follow-up The patient was initially suspicious of the pharmacologic prescription and refused to take the medication but agreed to participate in psychotherapy After some time in therapy, the patient became more agreeable to taking the prescribed medication The patient continued to have relatively tumultuous relationships with people but was better able to accept others' actions as not always being “bad” and was better able to realize not all of her actions were “good” P.317
Steps to the Diagnosis Borderline personality disorder A disorder of persistent abnormal behavior characterized by intense relationships with others, poor impulse control, and a skewed self-perception Personality disorders In general, these are all disorders of persistent behaviors that deviate significantly from what is considered the societal norm They are typically manifested through the interaction with others Behaviors tend to be inflexible and lead to an impairment of function The abnormal behaviors begin in late adolescence and cannot be attributed to a medical comorbidity, substance use, or another psychiatric condition History: findings are described in Table 13-6 Physical examination: a variable or emotional affect may be observed Tests: typically noncontributory Treatment: psychotherapy directed at addressing relationships with others is the primary treatment; antipsychotics, mood stabilizers, or SSRIs are also utilized in treatment Outcomes: Suicide and substance abuse are the most common complications The response to medication is not as good as in full mood disorders or psychosis Symptoms tend to be lifelong but become somewhat milder with age Clues to the diagnosis: History: unstable and intense relationships, splitting people into all good or all bad, impulsivity (e.g., sexual promiscuity, sudden commitments of time to learning tennis and skiing, daily alcohol use), dissociation from problems in relationships, multiple quasisuicidal attempts Physical: variable emotional affect Tests: none performed
A patient who exhibits mild signs of a personality disorder but is able to function normally in society is said to have a personality trait and may not require treatment.
Alcohol and tobacco abuse are very common in patients with primary psychiatric diagnoses. Paranoid personality disorder More thorough discussion in Table 13-6 Why eliminated from differential: the multiple self-destructive activities and constant fear of being alone
in this case are less consistent with this diagnosis Schizotypal personality disorder More thorough discussion in Table 13-6 Why eliminated from differential: this diagnosis would be characterized less by a concern over relationships and more by a greater degree of quasipsychotic behaviors Histrionic personality disorder More thorough discussion in Table 13-6 Why eliminated from differential: the self-destructive activities and explosive conclusions to relationships in this case are not typical of this diagnosis Narcissistic personality disorder More thorough discussion in Table 13-6 Why eliminated from differential: the self-destructive activities and fear of being alone in this case are not typical of this diagnosis Dependent personality disorder More thorough discussion in Table 13-6 Why eliminated from differential: the intensity of relationships would be much less in this diagnosis, and the patient would be expected to be resistant to ending any relationships Attention deficit hyperactivity disorder More thorough discussion in later case Why eliminated from differential: although the patient in this case has made some extremely impulsive decisions, impulsivity does not appear to be a problem with every single activity; likewise, inattentiveness and hyperactivity do not appear to be key features of her presentation Bipolar disorder More thorough discussion in prior case P.318
Table 13-6 Personality Disorders
Disorder
Charac teristic s
Treatment
CLUSTER A
Paranoid
Persistent distrust of others' loyalty, others' actions are consistently interpreted
Supportive and nonjudgmental
as harmful or deceptive, reluctant to share information, frequent
psychotherapy, low-dose
misinterpretation of comments, frequently bears grudges, perceives attacks on
antipsychotics
own reputation with an associated angry response, suspicions of partner infidelity
Schizoi
Inability to enjoy close relationships, social detachment, emotionally restricted,
Antipsychotics to initially resolve
anhedonia, flat affect, lack of sexual interests, lack of close friends, indifferent to
behavior, supportive psychotherapy
praise or criticism
focusing on achieving comfortable interactions with others
Schizotypal
Paranoia, ideas of reference, eccentric and inappropriate behavior, social
Supportive psychotherapy focusing on
anxiety, disorganized or vague speech, odd beliefs, relationships only with close
recognition of reality, low-dose
relatives
antipsychotics or anxiolytics
Aggressive behavior toward people and animals, disregard for others' safety,
Structured environment,
destruction of property, illegal activity, pathological lying, irritability, impulsivity,
psychotherapy with defined limit
risk-taking behavior, lack of responsibility, lack of remorse for actions; patients
setting may be helpful in controlling
are typically older than 18 years and have a history of conduct disorder prior to 15
behavior
CLUSTER B
Antisocial
years of age; more common in men
Borderline
Histrionic
Unstable relationships, feelings of emptiness, fear of abandonment, poor
Extensive psychotherapy employing
self-esteem, impulsivity in self-damaging behaviors, mood lability, suicidal ideation
multiple techniques combined with
or self-mutilating behavior, inappropriate irritability, paranoia, splitting (i.e.,
low-dose antipsychotics, SSRIs, or
seeing others as either all good or all bad); much more common in women
mood stabilizers
Dire need for attention, inappropriate seductive or theatrical behavior, emotional
Long-term psychotherapy focusing on
lability, exaggerated expression of emotions, dramatic speech, uses appearance to
relationship development and limit
draw attention to self, easily influenced by others, believes relationships are more
setting
intimate than in reality
Narcissistic
Grandiosity, fantasies of success, manipulation of others, expectation of
Psychotherapy focusing on acceptance
admiration, arrogance, sense of entitlement, believes self to be “special”, lacks
of shortcomings
empathy, envious of others
CLUSTER C
Avoidant
Dependent
Fear of criticism and embarrassment, social withdrawal, unwillingness to interact
Psychotherapy (initially individualized
with others without a certainty of being liked, fear of intimacy, poor self-esteem,
then group therapy later) focusing on
reluctance to try new activities, preoccupied by fear of rejection, inhibited by
self-confidence combined with
feelings of inadequacy
antidepressants or anxiolytics
Difficulty making decisions without the advice of others, fear of responsibility,
Psychotherapy focusing on developing
difficulty expressing disagreement, difficulty in doing activities by self, going to
social skills and development of
excessive lengths to receive others' support, fear of being alone, requiring
decisive behavior
constant close relationships
Obsessive-
Preoccupied with details to the extent that the point of the activity is lost,
Psychotherapy focusing on accepting
compulsive
perfectionistic, excessively devoted to work, inflexible in beliefs, miserly, difficulty
alternative ideas and working with
working with others, hoarding of worthless objects, stubbornness
others
SSRIs, selective serotonin reuptake inhibitors.
Why eliminated from differential: the patient does not meet the criteria for major depression, and her extreme actions are more relationship-related and not independently periodic Major depressive disorder/dysthymia More thorough discussion in prior case P.319
Why eliminated from differential: the patient does not meet the depressive criteria for either of these diagnoses Substance abuse More thorough discussion in later case Why eliminated from differential: the patient's daily use of alcohol is somewhat concerning but appears to be more of a reaction to her personality disorder than a cause for her difficulties in relationships; toxicology screening should be seriously considered to rule out other substance use
Case 13-6: “This man tried to kill his sister” A 28-year-old man has recently been arrested for attacking his sister and is awaiting trial for assault and battery. Because of his odd behavior, a psychiatrist is called to evaluate the patient for a mental illness. Police records show that the patient lived in a basement apartment at his parents' house. He has been unemployed for 4 months but prior to that time he had worked as a dishwasher at a local restaurant. He was fired from that job because of getting into a fight with the manager over people secretly saying things about the patient. On the night of the assault, the patient's sister came to their parents' house for dinner. She tried to convince her brother to see fireworks that evening outside of town. When he refused to accompany her, she teased him about never leaving the house and being afraid of people. He became upset at these remarks. When the patient's sister grabbed the patient's arm and tried to drag him out of the house, the patient began punching his sister and continued to hit her after she let go of his arm. Their parents called the police, who had to restrain the patient from further attacking his sister. The patient was very agitated upon being removed from the house, but calmed down upon entering the holding cell at the police station. Since that time he has kept to himself, and has avoided speaking to other people unless necessary. In a conversation with the patient's sister, she confirms the details of the police report. She says that she thought it was best for the patient to “get out of the house for once” and never expected his violent response. She says that her brother started to become aloof during his last 2 years of high school. She says that he became extremely concerned with other people's perceptions of him. He attended a local college because he did not want to be far from home, but failed out in his second semester because he did not attend any classes. His sister says that he told his parents that everyone in class would look at him and would laugh at him behind his back. Since that time he has lived in his parents' basement. He has held low-level jobs on occasion, but these tend to be short-lived because of his perception that others are “looking down” on him. Much of the time he has been unemployed and supported by his parents. They tend to be overly protective of their son because of his eccentricities but feel powerless in how to help him feel more comfortable around people. His sister says that he is normally very tolerant of their parents and talks to them daily. She says that he is more hesitant to speak with her since she moved out of her parents' house, but that she is far better tolerated than nonrelatives. She says that her brother has no friends or relationships and spends much of his time watching television or playing on the computer. She is unaware of any medical problems that he has or medications that he takes on a regular basis. She is aware of a paternal uncle who was diagnosed with schizophrenia, but he had committed suicide prior to her birth. She says that her brother smokes cigarettes constantly, but she is unaware of any other substance use. When the patient is interviewed, he refuses to speak to the psychiatrist unless no one else is in the room. He is reluctant to speak until after the physician has reassured him that he is there to help the patient. The patient says that “everyone is out to get him” because “they know [his] secret.” He refuses to explain what he means because “it would be too dangerous” for him. He says that he has “figured out this secret from watching television”. He says that he refuses to watch news reports because “they spy on you” via the television. He feels that his parents were the only people he could trust prior to this incident. He also admits that he would become very anxious when leaving the house, so he tries to remain indoors as much as possible. He explains his attack on his sister only by saying “she became one of them.” He denies wanting to hurt anyone else or himself, and says that he realizes that he could be incarcerated for his P.320 actions. He refuses to talk about any personal or family medical history. He admits to smoking cigarettes but denies other substance use. On the mental status examination, he appears clean but somewhat disheveled. His tone of
speech is low, and he speaks quietly. Although he answers most questions, his answers tend to be rather vague and incomplete. He has no other abnormal patterns of speech. His affect is slightly flattened, but he does become more worried-appearing when talking about his “secret.” He remains alert during the interview. He is oriented to his name, location, and the time. He is able to repeat the names of three listed objects and recalls all three objects several minutes later. He refuses physical examination. The following vital signs are measured: T: 98.6○F, HR: 86 bpm, BP: 135/83 mm Hg, RR: 19 breaths/min
Differential Diagnosis Social phobia, delusional disorder, autism, schizotypal personality disorder, paranoid personality disorder, schizoid personality disorder, narcissistic personality disorder, avoidant personality disorder, major depressive disorder, substance abuse
Laboratory Data and Other Study Results Urine toxicology screen: positive for nicotine only
Diagnosis Schizotypal personality disorder
Treatment Administered The patient was enrolled in psychotherapy that focused on helping him discern reality from internal beliefs and on teaching him a means to cope with his perception of others Quetiapine was prescribed for the patient
Follow-up Although the patient agreed to participate in psychotherapy, he refused to use the antipsychotic medication Although charges were not pressed by his family members for the attack, the court ruled that the patient should receive inpatient psychiatric treatment Despite therapy, the patient did not significantly improve in his ability to interact with others and remained in the psychiatric facility
Steps to the Diagnosis Schizotypal personality disorder More thorough discussion in Table 13-6 Clues to the diagnosis: History: paranoia (e.g., worry of others' perceptions of him), ideas of reference (e.g., messages from the television), odd beliefs (e.g., “secret” ability), social anxiety and withdrawal, vague answers to questions, close relationships only with immediate family, family history of schizophrenia Physical: flattened affect Tests: noncontributory Paranoid personality disorder
More thorough discussion in Table 13-6 Why eliminated from differential: despite the important role of paranoia in this patient, the ideas of reference, odd beliefs, and social anxiety seen in this case are not characteristic of this disorder P.321 Schizoid personality disorder More thorough discussion in Table 13-6 Why eliminated from differential: although social withdrawal and flattened affect are characteristic of this diagnosis, ideas of reference and odd beliefs are not typically demonstrated Narcissistic personality disorder More thorough discussion in Table 13-6 Why eliminated from differential: grandiosity and personal fantasies may be seen in this diagnosis, but the social anxiety and withdrawal are not typical Avoidant personality disorder More thorough discussion in Table 13-6 Why eliminated from differential: the ideas of reference, odd beliefs, and paranoia are not typically seen in this diagnosis Social phobia More thorough discussion in prior case Why eliminated from differential: the ideas of reference, odd beliefs, and paranoia are not typically seen in this diagnosis Delusional disorder More thorough discussion in prior case Why eliminated from differential: because isolated delusions are the main characteristic of this diagnosis, the other odd behaviors in this case rule it out Autism A condition of severe impairment in interpersonal interactions and communication and unusual inflexible behaviors Mental retardation is a common comorbid condition Disease findings are usually exhibited prior to the age of 3 years Diagnosis requires six abnormal patterns of interpersonal interactions including at least two types of impaired social interactions, one type of impaired communication, and one type of restricted behavior History: Impaired social interactions: impaired use of nonverbal behaviors, failure to develop peer relationships, failure to seek social interaction, lack of social reciprocity to others' attempts at interaction Impaired communication: developmental language delays, poor initiation or sustenance of
conversation, repetitive language, lack of imaginative or imitative play considered appropriate for age Restricted behavior: inflexible routines, preoccupation with a restricted pattern of interest, repetitive motor mannerisms, preoccupation with parts of objects Physical examination: findings are typically consistent with the abnormal behaviors (e.g., poor eye contact, mannerisms) Tests: typically noncontributory, but appropriate testing is useful in ruling out other conditions Treatment: Behavioral, speech, and social psychotherapy involving peers and family members may help improve social interactions Antipsychotics may be required to treat aggressive behaviors Long-term supervision is usually required for these patients Outcomes: the prognosis is generally poor with the majority of these patients never developing skills of social interaction and requiring lifelong care; patients with a milder form of the disease (i.e., Asperger disease) may be much more capable of functioning in society Why eliminated from differential: although the social withdrawal and poor social interaction are consistent with this disease, the odd beliefs and ideas of reference make it an unlikely diagnosis Major depressive disorder More thorough discussion in prior case P.322 Why eliminated from differential: some psychotic features may be seen in mood disorders, but the degree of social withdrawal and apparent lack of depressive characteristics make this diagnosis unlikely Substance abuse More thorough discussion in later case Why eliminated from differential: the denial of substance use beyond tobacco and the negative toxicology screen make this diagnosis unlikely
Case 13-7: “It's so hot that I'm having a heart attack!” A 46-year-old man presents to an emergency department with complaints of palpitations and feeling extremely warm. The patient says that he also feels short of breath and nauseous. He says that he vomited once at home before coming to the hospital but has not had any emesis or diarrhea since that time. He said that he was watching television when these symptoms began to develop, and that the first thing he noticed was some increasingly blurry vision and a mild headache. He says that his arms now feel weak and heavy. His symptoms started about 2 hours ago. He denies any paresthesias, hematemesis or hematochezia, dizziness, syncope, or extremity pain. He says that he has never had this experience in the past and that nothing he has done since his symptoms started, including lying down, drinking water, and walking around, has made his symptoms improve. He has never had any form of chest pain prior to tonight. He denies any trauma, prolonged sun exposure, or consumption of any unusual foods. He says that he knew it was important to come to the hospital because all of his family members want him to die so that they can get his money, and he is convinced that if he would have stayed at home they would not have helped him. He says that he has a past medical history of asthma, hypercholesterolemia, and gout for which he takes allopurinol, atorvastatin, and as needed albuterol. He denies having had a previous heart attack or stroke. He says that he drinks alcohol occasionally but denies other substance use. On examination, the patient is very agitated and constantly paces back and forth across the room during the interview. He is extremely anxious-appearing. He is diaphoretic,
and his skin feels clammy. His eyes are symmetric and extremely dilated despite the bright lighting in the examination room. They are minimally reactive to additional light. There are infrequent beats of vertical nystagmus. There is a small collection of dried blood at the end of his right nostril. Examination of the nasal cavity finds the right side to be clogged with clotted blood. Examination of his ear canals is normal. He has no lymphadenopathy. Auscultation of his lungs detects diffuse rhonchi, wheezing, and tachypnea. Auscultation of his heart detects tachycardia and no extra heart sounds. His abdomen has slight diffuse tenderness, no masses, and hyperactive bowel sounds. A neurovascular examination detects full motor strength and sensation in all extremities. He has no rashes or wounds on his skin. The following vital signs are measured: T: 102.9°F, HR: 118 bpm, BP: 143/90 mm Hg, RR: 24 breaths/min
Differential Diagnosis Asthma, pulmonary embolism, chronic obstructive pulmonary disease, pneumonia, congestive heart failure, myocardial infarction, heat emergency, somatoform disorder, substance abuse, schizophrenia, cerebral vascular accident
Laboratory Data and Other Study Results Pulse oximetry: 94% on room air, 99% on 2L O2 via nasal cannula CBC: WBC: 10.6, Hgb: 13.2, Plt: 391 Chem10: Na: 144 mEq/L, K: 3.0 mEq/L, Cl: 97 mEq/L, CO2: 20 mEq/L, BUN: 28 mg/dL, Cr: 1.4 mg/dL, Glu: 93 mg/dL, Mg: 1.7 mg/dL, Ca: 9.5 mg/dL, Phos: 3.0 mg/dL LFTs: AlkPhos: 157 U/L, ALT: 83 U/L, AST: 84 U/L, TBili: 1.1 mg/dL, DBili: 0.7 mg/dL Cardiac enzymes: creatine kinase (CK): 652 U/L, creatine kinase myocardial component (CK-MB): 4.8 ng/mL, troponin-I: 0.3 ng/mL UA: light cola colored, pH: 6.7, specific gravity: 1.045, mild hematuria, no glucose/ketones/nitrites/leukocyte esterase/proteinuria Urine toxicity screen: positive for alcohol and cocaine Electrocardiogram (ECG): sinus tachycardia; multiple leads with T wave abnormalities; no leads with ST segment elevation or depression; no Q waves Chest x-ray (CXR): diffuse moderate bronchoconstriction and diffuse slight infiltrates; no focal effusions, masses, or lymphadenopathy Head CT: no cerebral masses or ischemia; normal cerebral and ventricular volumes; no evidence of trauma or hematoma Chest CT: no focal areas of ischemia; no effusions, masses, or lymphadenopathy; mild diffuse infiltrates Repeat cardiac enzymes are ordered eight hours later: Cardiac enzymes: CK 483 U/L, CK-MB 4.1 ng/mL, troponin-I 0.3 ng/mL
Diagnosis Substance abuse—cocaine toxicity with secondary pneumonitis, renal insufficiency, hypokalemia, hepatic irritation, and rhabdomyolysis
Treatment Administered
The patient was given supplemental oxygen to optimize his arterial oxygenation Intravenous (IV) lorazepam was administered to decrease the patient's agitation Sublingual nitroglycerin was administered to decrease the patient's chest pain and hypertension (HTN) Ice packs were placed on the patient to help decrease his fever IV fluids were infused to help maintain renal blood flow and prevent permanent renal damage secondary to rhabdomyolysis The patient was admitted for observation and serial cardiac enzymes
Follow-up Several hours after admission the patient felt more comfortable and less agitated Repeat cardiac enzymes demonstrated no evidence of a coronary insult; his creatine kinase gradually decreased over the course of several days The patient was seen by a social worker to discuss the dangers of cocaine use and options for drug counseling and cessation The patient was discharged after 5 days but never followed-up with any physicians involved in his care or with a recommended drug counselor
Steps to the Diagnosis Substance abuse Repetitive use of a substance that results in negative psychiatric or physical consequences Substance use: occasional use of a substance with infrequent negative effects Intoxication: reversible central nervous system (CNS) effects due to substance use Physical dependence: physical adaptation to repetitive substance use in which abrupt cessation or antagonist use causes a withdrawal syndrome Psychologic dependence: the perceived need for a given substance because of its associated positive effects or because of the fear of effects from a lack of use Addiction: chronic substance use leading to physical and psychologic dependence, the development of tolerance (i.e., increasing doses required for the desired effect), feelings of loss of control over use, and a significant amount of time devoted to procuring the substance History: the particular symptoms of use are substance dependent (Table 13-7) Physical examination: physical findings are substance-dependent (Table 13-7) Tests: urine or serum toxicity screens are useful for demonstrating recent use Treatment: The effects of intoxication will self-resolve in most cases, and supportive care is sufficient Toxicity following substance use may require more aggressive intervention that is substance dependent (Table 13-7)
Table 13-7 Characteristics of Substance Abuse
Complic ations of Substanc e
Alcohol
Intoxic ation
W ithdraw al
Chronic Use
Treatment
Dec reased
Diaphoresis,
Malnutrition (e.g.,
Supplemental nutrition,
inhibition, slurred
tachycardia,
vitamin B12,
supportive psychotherapy
speech, impaired
anxiety, nausea,
thiamine),
or group c ounseling (e.g.,
c oordination,
vomiting, tremor,
encephalopathy
Alcoholics Anonymous, etc.),
inattentiveness,
delirium tremens
(i.e., Wernicke-
naltrexone to decrease
decreased
(i.e., seizures,
Korsakoff),
cravings, disulfiram causes
consciousness,
delirium)
ac c idents, suicide,
unpleasant nausea and
c irrhosis, GI
vomiting if taken before
bleeding
alcohol consumption,
retrograde amnesia
benzodiazepines prevent delirium tremens during withdrawal
Amphetamines (e.g.,
Hyperac tivity ,
Anxiety,
Psychosis,
Rehabilitative counseling,
methamphetamine,
psychomotor
depression,
depression, fatigue,
antipsychotics,
methylphenidate,
agitation, pupillary
increased
parkinsonian
benzodiazepines
etc.)
dilation,
appetite, fatigue
symptoms
Memory loss
tachycardia, HTN, psychosis
Benzodiazepines
Sedation, amnesia,
Anxiety, insomnia,
(e.g., alprazolam,
slurred speech,
tremor, seizures
etc.)
decreased
Rehabilitative counseling, anticonvulsants
coordination
Caffeine
Cocaine
Insomnia,
Headaches,
GI irritation,
restlessness, tremor,
fatigue,
fatigue,
anxiety, tachycardia
inattentiveness
inattentiveness
Euphoria,
Sedation,
Arrhythmias,
Reduction of hypertension,
diaphoresis,
depression,
sudden c ardiac
antipsychotics,
tachycardia,
psychomotor
death, stroke,
benzodiazepines,
agitation,
retardation,
suicidal ideation,
rehabilitative counseling
restlessness,
fatigue, anhedonia
inattentiveness
Minimal
Psychosis,
Gradual reduction in usage
mydriasis, nystagmus, chest pain, hypertension, paranoia, grandiosity
Remove patient from dangerous environment until
Hallucinogens (e.g.,
Halluc inations,
LSD, mescaline,
delusions, anxiety,
ketamine)
paranoia,
intoxication resolves,
tachycardia,
antipsychotics
“flashbacks”
pupillary dilation, tremors
Marijuana
Euphoria, paranoia,
Irritability,
Motivational
Rehabilitative counseling,
psychomotor
depression,
syndrome,
antipsychotics
retardation,
insomnia, nausea,
infertility,
impaired judgment,
tremor
depression,
increased appetite,
psychosis
c onjunc tival injec tion, dry mouth
Nicotine (and other
Restlessness,
Insomnia, weight
Canc er (many
Rehabilitative counseling,
substances found in
nausea, vomiting,
gain, irritability,
different forms),
cutaneous (i.e., patch) or
tobacco and
abdominal pain
inability to
COPD, increased
mucosal (i.e., gum) nic otine
cigarettes)
Opioids
PCP
concentrate,
respiratory
administration to reduce
nervousness,
infections, ischemic
cravings for cigarettes,
headaches
heart disease
hypnosis, bupropion
Euphoria, slurred
Depression,
Constipation,
Methadone therapy,
speech, pupillary
anxiety, stomac h
increased risk of
inpatient rehabilitative
c onstric tion,
c ramps, nausea,
blood-borne
counseling, naltrexone may
inattentiveness,
vomiting,
infec tion with IV
prevent euphoria with use,
decreased
diarrhea, myalgias
drug use
naloxone is an opioid
consciousness,
antagonist used for acute
respiratory
overdose with significant
depression
respiratory depression
Euphoria,
Sudden violent
Psychosis, memory
Isolated containment until
impulsiveness,
behavior, variable
deficits, impaired
after resolution of
aggressive
levels of
cognitive function,
intoxication,
behavior,
consciousness
inability to retrieve
benzodiazepines,
words
antipsychotics, ascorbic
ny stagmus (vertic al and horizontal),
acid
hyperreflexia
COPD, chronic obstructive pulmonary disease; GI, gastrointestinal; HTN, hypertension; IV, intravenous; LSD, lysergic acid diethylamide; PCP, phencyclidine.
Because of the addictive nature of many substances and the complications associated with chronic use, referral of the patient to a detoxification and rehabilitation program is important Outcomes: complications from chronic use are substance dependent; people who abuse one substance are at a much higher risk of acquiring a transmissible disease, becoming addicted to a second substance, or dying from suicide or homicide Clues to the diagnosis: History: acute multisystem symptoms without a clear explanation, paranoia Physical: mydriasis, nystagmus, agitation, epistaxis, diaphoresis, hyperthermia, HTN, tachycardia Tests: urine toxicology results, increased creatine kinase, hypokalemia
Use the CAGE questionnaire to screen for drug abuse (especially alcohol). More than one “yes” response to any of these conditions should raise the suspicion for excessive use: desire to Cut down on usage; Annoyance over others' suggestions to stop usage; Guilt over usage; drug use upon waking (i.e., Eye-opener).
Pupillary responses to drug intoxication may be remembered by the mnemonic “All Crack Heads Dance, Never Clothed”: Amphetamine, Cocaine, Hallucinogens —Dilation of pupils; Narcotics—Constriction of pupils. Asthma More thorough discussion in Chapter 2 Why eliminated from differential: given the patient's history of asthma, it is quite possible that his respiratory symptoms are related to an exacerbation (probably cocaine related), but this diagnosis alone cannot explain the constellation of symptoms and findings in the case
Chronic obstructive pulmonary disease/pneumonia More thorough discussion in Chapter 2 Why eliminated from differential: the appearance of the CXR and chest CT makes these diagnoses unlikely Pulmonary embolism More thorough discussion in Chapter 2 Why eliminated from differential: the appearance of the chest CT and the good oximetry response to supplemental oxygen make this diagnosis unlikely Congestive heart failure More thorough discussion in Chapter 1 Why eliminated from differential: the mild pulmonary infiltrates and renal and hepatic lab abnormalities could be seen in the setting of congestive heart failure (CHF); however, a greater degree of pulmonary edema would be expected given the significant onset of symptoms; the positive toxicology screen is an additional clue that another underlying cause may explain the patient's constellation of findings Myocardial infarction More thorough discussion in Chapter 1 Why eliminated from differential: the negative serial cardiac enzymes rule out this diagnosis Heat emergency More through discussion in Chapter 5 Why eliminated from differential: the lab abnormalities would not be seen in heat exhaustion but may be possible in heat stroke; the patient's positive toxicology screen suggests a more likely cause of these findings Somatoform disorder More thorough discussion in later case Why eliminated from differential: the multiple legitimate physical findings and lab abnormalities rule out this diagnosis Schizophrenia More thorough discussion in prior case Why eliminated from differential: the multiple legitimate physical findings and lab abnormalities accompanying the patient's odd behavior make a primary psychiatric cause less likely Cerebral vascular accident More thorough discussion in Chapter 7 Why eliminated from differential: the normal head CT rules out this diagnosis
Case 13-8: “Our daughter has gotten so thin” A 15-year-old adolescent girl is seen by her pediatrician because her parents are concerned about her weight. Her parents, who accompany her to the appointment, say that their daughter was formally an avid distance runner and was very active. Last year she broke her ankle while training for competition and has never resumed the mileage
that she was formerly running on a weekly basis. They say that she has progressively lost weight over the past 8 months despite her decreased level of exercise. She was always a thin girl when she competed because of the amount of running that she did, but now they are concerned that she is becoming too thin. The patient does not share her parents concern and is annoyed by being forced to come to this appointment. She says that she is healthy and that there is no reason for them to be concerned. She says that she was formerly an “unhealthy eater” and that she merely decided to eat “healthier” after she was unable to run. She says that she was “a little chubby in the past” and that it was the best thing for her to lose a couple of pounds. She says that she has no problems eating and that she eats all of the time; she just does not eat unhealthy foods. Her parents disagree with her; her mother says that she never sees her eat breakfast or lunch and that she does not eat much food at dinner. Her mother also says that on occasions when the family goes to functions at which the patient eats a greater amount of food, her daughter goes running as soon as she returns home. The patient says that she is doing nothing different than she had before her injury except for being more selective about her food choices. She says that she is much better now compared to “when [she] used to be so fat.” She says that the only unusual thing that she has noticed is that she has not had a menstrual period in 4 months. She is not bothered by this fact because she says that she knows she cannot be pregnant. She denies any new injuries since her ankle injury a year ago. She denies any bingeing behavior, being disturbed by her appearance, or actively trying to lose additional weight. She also denies feeling sad, fatigued, or unable to sleep. She denies polyuria, polydipsia, nausea, vomiting, diarrhea, or abdominal pain. She does not think that she is too thin and says that her parents are overly concerned. She denies any past medical history aside from her ankle fracture. She takes a multivitamin but no other medications. She denies any substance use. On examination, she is a very thin girl in no acute distress. She must remove a jacket and two sweat shirts to be examined. Her face is symmetric without any swelling. Her oropharynx is clear, and her teeth appear normal. She does not have any lymphadenopathy. Auscultation of her lungs and heart detects clear breath sounds, bradycardia, and a soft, blowing systolic murmur. Her abdomen is nontender with no masses and hypoactive bowel sounds. Her extremities are extremely thin and nontender to palpation. She has light hair covering her forearms, and her skin is dry and slightly scaly. A neurovascular examination is normal. The following vital signs are measured: T: 98.2°F, HR: 54 bpm, BP: 102/73 mm Hg, RR: 16 breaths/min
Differential Diagnosis Anorexia nervosa, bulimia, body dysmorphic disorder, major depressive disorder, diabetes mellitus type I, hyperthyroidism, pregnancy
Laboratory Data and Other Study Results Height and weight: 5﹐ 8", 101 lb, body mass index (BMI): 15.4 (ideal body weight 140 lb) Urine pregnancy test: negative CBC: WBC: 5.4, Hgb: 12.1, Plt: 173 Chem10: Na: 134 mEq/L, K: 3.2 mEq/L, Cl: 95 mEq/L, CO2: 34 mEq/L, BUN: 36 mg/dL, Cr: 1.4 mg/dL, Glu: 84 mg/dL, Mg: 1.2 mg/dL, Ca: 8.5 mg/dL, Phos: 2.7 mg/dL Thyroid panel: TSH: 3.78 µU/mL, T4: 4.9 µg/dL, free T4 index: 5.0, T3: 0.5 ng/mL, T3 reuptake: 0.77 UA: dark amber colored, pH: 7.4, specific gravity: 1.060, no glucose/ketones/nitrites/leukocyte esterase/hematuria/proteinuria ECG: sinus bradycardia; no abnormal wave morphology and intervals; no ST segment elevation or depression
Diagnosis Anorexia nervosa
Treatment Administered The patient was admitted to the hospital for IV hydration and correction of her multiple electrolyte abnormalities; the patient was initially highly resistant to this plan but was agreeable when it was explained that her electrolyte abnormalities were nearing concerning levels The patient was given a regular diet and appeared to accept it; however, on the second day after admission one of the nurses realized that the patient had been hiding all of her food underneath her mattress Following the above finding, the patient was given all meals under supervision to confirm that she was eating food The patient was entered into intensive psychotherapy addressing introspection into her disorder, body image, dangers of eating disorders, and healthy eating decisions
Follow-up The patient was able to be discharged to home once her electrolyte levels remained stable within the normal ranges She continued outpatient psychotherapy for her eating disorder The patient's parents supervised her consumption of food closely and confirmed that she was gaining weight The patient exercised to avoid gaining excessive weight but maintained her weight close to the ideal body weight
Steps to the Diagnosis Anorexia nervosa An eating disorder in which patients refuse to maintain a normal body weight Patients have a distorted body image and believe that they are overweight Fasting, excessive exercise, and purging are means by which weight loss is achieved The patient's body weight is <85% of the ideal body weight Risk factors: adolescence, high socioeconomic status, female gender History: fixation on the prevention of weight gain, distortion of self-body image, amenorrhea, cold intolerance Physical examination: body weight below 85% of the ideal weight, hypothermia, dry skin, lanugo hair growth (i.e., fine, short hair growth similar to that of newborns) Tests: Anemia, leukopenia, and thrombocytopenia may be seen on the CBC Electrolyte abnormalities include hyponatremia, hypokalemia, hypomagnesemia, hypocalcemia, and hypophosphatemia; metabolic alkalosis and prerenal azotemia are also possible Estrogen may be decreased ECG will frequently show bradycardia and rarely prolonged QT intervals A dual energy x-ray absorptiometry (DEXA) scan will show osteopenia or osteoporosis
Treatment: Psychotherapy is the main treatment and should focus on body image, weight gain, and how to maintain long-term control Inpatient treatment is indicated for a weight below 75% of the ideal body weight or for concerning electrolyte and ECG findings Medications have not been shown to be beneficial, but antidepressants or anxiolytics do play a role in patients with a comorbid mood or anxiety disorders Outcomes: Complications include those associated with electrolyte abnormalities (e.g., arrhythmias), abnormal gastrointestinal (GI) mobility, nephrolithiasis, infertility, peripheral neuropathy, and osteopenic fractures Refeeding syndrome is the sudden increase in blood volume and change in electrolyte levels following the initiation of a regular diet and may lead to cardiovascular collapse, rhabdomyolysis, mental status changes, and seizures Half of the patients with the condition will recover fully; the mortality rate is 10% and occurs due to arrhythmias, starvation, or suicide Clues to the diagnosis: History: excessive weight loss, decreased food intake, compensatory exercise, amenorrhea, poor body image Physical: hypothermia, bradycardia, excessive clothing use because of cold intolerance, lanugo, dry skin Tests: body weight below 85% of ideal weight, multiple electrolyte abnormalities
Ninety percent of the cases of anorexia nervosa are diagnosed in women.
Tell parents and coaches of young female athletes to be on the lookout for the “female athlete triad” (i.e., eating disorders, amenorrhea, and osteoporosis) that may occur in girls involved in competitive sports.
Patients with anorexia nervosa should be screened for depression, and SSRIs should be included in treatment if depression is diagnosed. Bulimia nervosa An eating disorder in which patients feel a lack of control over their eating behavior, engage in binge eating, and maintain a normal body weight through some type of compensatory activity (e.g., purging, excessive exercise, laxative use) Diagnosis requires the occurrence of bingeing-compensation episodes at least twice per week for more than 3 months History: binge eating accompanied by a sense of loss of control, compensatory behavior after binges, dissatisfaction with body weight and shape, oligomenorrhea Physical examination: weight is >85% of the ideal weight, dental enamel erosion and scars on hands (both
from induced vomiting), parotid enlargement, dry skin Tests: Hypokalemic metabolic alkalosis occurs from repeated vomiting Normokalemic metabolic acidosis occurs from laxative use Hyponatremia, hypomagnesemia, hypocalcemia, and hypophosphatemia may occur but are less common than in anorexia nervosa ECG and DEXA should be considered but are abnormal less frequently than in anorexia nervosa Treatment: psychotherapy is the first-line treatment and is directed at improving patient body image and reducing binge-compensation cycles; SSRIs or TCAs have shown benefit in helping to improve behaviors Outcomes: Complications from weight loss are not as common as they are in anorexia nervosa Esophageal rupture, Mallory-Weiss tears, and chronic reflux are risks of repeated vomiting episodes Half of patients will recover, but this is frequently a chronic diagnosis with general improvement and periodic exacerbations Why eliminated from differential: the patient's weight below 85% of her ideal body weight rules out this diagnosis Body dysmorphic disorder A preoccupation with an imagined defect in one's appearance that limits the ability to function normally Typically begins in adolescence History: imagined defect in a distinct body area, seeking out of a means to improve the perceived defect (e.g., dermatologists, plastic surgeons), continued dissatisfaction with body area following treatments Physical examination: typically normal Tests: typically noncontributory Treatment: psychotherapy directed at self-body image; SSRIs have been shown to improve the misperceptions of appearance Outcomes: the prognosis is frequently good with the introduction of psychotherapy Why eliminated from differential: the patient is this case is focused on her entire body weight and not just one part of her body Major depressive disorder More thorough discussion in prior case Why eliminated from differential: the patient does not meet the criteria for this diagnosis; however, given her lack of full honesty about her self-perception, she should be carefully observed for signs of depression even though she has denied such symptoms Diabetes mellitus More thorough discussion in Chapter 5 Why eliminated from differential: the patient's normal glucose level and lack of glucosuria rule out this
diagnosis Hyperthyroidism More thorough discussion in Chapter 5 Why eliminated from differential: the presence of low or low-normal thyroid hormones (a result of her poor nutrition) rules out this diagnosis Pregnancy More thorough discussion in Chapter 12 Why eliminated from differential: the negative pregnancy test rules out this diagnosis
A plastic surgeon or dermatologist should be wary of the patient with possible body dysmorphic disorder and should consider a psychiatric referral before proceeding with any significant procedures or treatments.
Case 13-9: “My whole left leg is numb” A 46-year-old woman presents to a neurologist with the complaints that she is unable to move or feel any sensation in her left leg. She also says that her entire left leg aches and hurts when it is moved. She arrives to her appointment using a wheelchair because she says that her left leg prevents her from walking. She says that her symptoms started 2 days ago after she was in a fight with her husband. She had caught him having an affair and confronted him. After the argument she developed the sudden onset of left leg weakness, numbness, and pain. She says that unfortunately she has caught her husband cheating on their marriage two previous times after suspecting it for several years. She had a similar onset of symptoms after confronting him both times. She was seen in the emergency department and by her primary care provider (PCP) both previous episodes, but no one was able to explain her symptoms. In both cases, her symptoms self-resolved within a week. She is directly consulting a neurologist on this occasion because of her frustration with a lack of a diagnosis. She is very worried that she is “giving [herself] strokes” by becoming so upset with her marriage problems. She says that the onset of her current symptoms was sudden and began immediately after her argument with her husband. She denies similar symptoms in any other extremity. She also denies headaches, dizziness, vision changes, chest pain, or nausea. She denies any prior history of trauma or neurologic diagnoses. She says that she has a history of mild depression that has been well treated with fluoxetine. She smokes one pack of cigarettes per day and occasionally drinks alcohol. On examination, she is a well-appearing woman in no acute distress. Her face is symmetric in appearance and movement. Funduscopic examination of her eyes detects no abnormalities. She has no lymphadenopathy. Auscultation of her lungs and heart is normal. Her abdomen is nontender with no masses and normal bowel sounds. On neurovascular examination both of her arms have full strength, normal sensation, and strong pulses. Her right leg is also neurovascularly intact. She is unable to voluntarily move her left leg at the hip, knee, ankle, or foot. On fine filament light touch testing of her left leg she denies any sensation from the thigh to the foot. When she is poked with a needle, she reflexively withdraws, although she insists that she can barely feel the needle. When she lies supine and is asked to perform a straight leg raises for each side, the neurologist can feel each of her contralateral legs pushing down into the table. Her pulses are palpable in both feet. The following vital signs are measured: T: 98.5°F, HR: 73 bpm, BP: 127/82 mm Hg, RR: 16 breaths/min
Differential Diagnosis Cerebral vascular accident, multiple sclerosis, polymyositis, Guillain-Barré syndrome, conversion disorder, somatization disorder, hypochondriasis, pain disorder, factitious disorder, malingering
Laboratory Data and Other Study Results
Chem10: Na: 140 mEq/L, K: 4.1 mEq/L, Cl: 106 mEq/L, CO2: 28 mEq/L, BUN: 16 mg/dL, Cr: 0.7 mg/dL, Glu: 100 mg/dL, Mg: 1.8 mg/dL, Ca: 10.1 mg/dL, Phos: 3.9 mg/dL ANA: negative Head MRI: no intracranial lesions or hemorrhages; normal parenchymal and ventricular size Cervical/lumbar MRI: normal cord appearance without lesions; minimal degenerative changes; no sites of increased signal Left leg electromyography/nerve conduction study (EMG/NCS): normal conduction latency and action potential amplitudes; no abnormal signal patterns Lumbar puncture: clear appearance, opening pressure 73 mm Hg, WBC: 1/µL, Glu: 54 mg/dL, protein: 34 mg/dL
Diagnosis Conversion disorder
Treatment Administered The patient was referred to a psychiatrist for further care The patient was enrolled in psychotherapy to help apply coping skills to stressful situations
Follow-up The patient's symptoms resolved in a few days Shortly after the patient entered psychotherapy, she divorced her husband After the divorce the patient did not experience any additional episodes
Steps to the Diagnosis Conversion disorder The development of sensory or motor deficits or pseudoseizures following periods of significant stress There is no identifiable associated medical condition History: patients report the onset of sensory deficits (e.g., paresthesias, blindness), motor deficits (e.g., paralysis, loss of voice), or pseudoseizures (i.e., seizure-like movements without any detectable abnormal cerebral activity) following periods of stress Physical examination: patients will insist they are unable to perform voluntary movement or detect subjective sensation, but reflex motor and sensory function (e.g., withdrawal to pain, compensatory motor activity) will be intact Tests: noncontributory, but testing to rule out the more likely diagnoses is almost always performed Treatment: psychotherapy is useful for helping patients identify stressors and learn to develop healthy responses to stress Outcomes: the majority of cases self-resolve; recurrence is higher in patients experiencing pseudoseizures Clues to the diagnosis: History: onset of deficits after stressful events, prior self-resolution of symptoms
Physical: subjective deficits with intact reflex sensory and motor activity Tests: noncontributory (except for ruling out other diagnoses) Somatization disorder A somatoform disorder in which multiple unintentional physical symptoms occur in a patient The symptoms cannot be explained by any medical condition Typically begins in young adulthood, and patients must be younger than 30 years at the onset of symptoms Diagnosis requires pain at four unrelated body sites, two GI symptoms, one sexual symptom, and one pseudoneurologic symptom History: GI symptoms: nausea, vomiting, diarrhea, indigestion Sexual symptoms: decreased libido, erectile dysfunction, menorrhagia Pseudoneurologic symptoms: ataxia, weakness, urinary retention, paresthesias, hallucinations Pain symptoms: multiple distinct sites on the body Physical examination: despite the multiple complaints, the examination may be normal and will show retained reflex ability Tests: typically noncontributory but performed to rule out other diagnoses Treatment: psychotherapy focusing on coping skills and the maintenance of function is the primary treatment Outcomes: the prognosis is variable, but a greater extent of symptoms upon presentation can be associated with a greater likelihood of chronicity Why eliminated from differential: the isolation of the patient's complaints to just one leg and just pseudoneurologic symptoms rules out this diagnosis Hypochondriasis Excessive fear that a minor symptom represents a serious illness Fear limits the ability to function Typically begins in middle aged patients Diagnosis requires the fear of an illness and the portrayal of symptoms for at least 6 months History: preoccupation with the fear of having a serious illness, despite normal medical evaluations and symptoms consistent with the patient's perception of the disease (not necessarily the accurate symptomatology) Physical examination: often normal, although the patient will frequently try to re-create the findings that are felt to be indicative of a disease Tests: noncontributory to the diagnosis but useful to rule out other diagnoses Treatment: Regular physician visits to confirm the absence of a particular disease will help to allay patient fears
Group psychotherapy for support and for focusing on realistic beliefs about a disease Treatment of comorbid depression also frequently helps to improve patient fears Outcomes: the prognosis is variable, but the condition tends to improve to some degree while still remaining a chronic condition Why eliminated from differential: although the patient is worried about her symptoms and is particularly concerned about a stroke, the distinct relationship of her symptoms to periods of significant stress makes conversion disorder a better diagnosis Pain disorder The development of a specific pain complaint that cannot be explained entirely by a medical condition The pain is not intentionally induced History: pain at a particular isolated body area that is exacerbated during periods of stress Physical examination: typically normal Tests: noncontributory Treatment: psychotherapy with biofeedback can help reduce the degree of pain; hypnosis may also be beneficial for pain reduction Outcomes: the prognosis is variable, with some patients experiencing full resolution and others having chronic complaints Why eliminated from differential: although the patient does complain of leg pain, this diagnosis would not explain her sensory and motor complaints Factitious disorder (a.k.a. Münchhausen syndrome) The intentional induction of disease findings by a patient that carries no clear benefit to the patient May involve any body system Diagnosis requires the intentional production of disease findings by the patient, denial of intention, wandering of the patient from one physician to another, and no clear incentive for the patient's actions History: the patient reports the symptoms of a given disease and attempts to induce the disease process (e.g., self-injections of insulin or excrement, attempts to become infected by a pathogen, self-wounding) Physical examination: findings are appropriate for the induced condition Tests: tests will reflect those associated with the induced condition Treatment: Treat any induced conditions appropriately Do not institute any unnecessary therapies Attempt to limit the patient to one physician and one hospital Psychotherapy including family members may be helpful but is difficult due to the high rate of patient resistance Outcomes: the condition tends to be a chronic problem despite the attempts at treatment Why eliminated from differential: it does not appear that the patient has done anything to induce her
symptoms, so this diagnosis is unlikely Malingering The intentional induction of disease or the complaint of symptoms by a patient who will directly benefit from appearing ill History: the patient reports symptoms for a certain disease in order to realize a personal gain (e.g., time off from work, financial compensation, a particular therapy), patients will frequently leave the site of care if they are confronted or realize that a goal is unattainable Physical examination: if an actual disease has been induced, the findings will be consistent with the disease Tests: normal unless an actual disease has been induced Treatment: avoid unnecessary treatments and report all suspicious activities to a higher authority (e.g., psychiatrist, hospital patient data base, ethics committee) Outcomes: because an actual disease does not exist (or did not until induction), the patient will likely continue to act in such a way as long as he or she stands to reap benefits from his or her actions Why eliminated from differential: although this diagnosis must always be considered for unusual or suspicious presentations, the relationship to stress in this case and the patient's seemingly legitimate concerns make this diagnosis less likely Cerebral vascular accident More thorough discussion in Chapter 7 Why eliminated from differential: the negative head MRI makes this diagnosis unlikely Multiple sclerosis More thorough discussion in Chapter 7 Why eliminated from differential: the negative MRIs, normal cerebral spinal fluid (CSF) analysis, and isolated occurrences make this diagnosis unlikely Polymyositis More thorough discussion in Chapter 9 Why eliminated from differential: the normal EMG and negative ANA make this diagnosis unlikely Guillain-Barré syndrome More thorough discussion in Chapter 7 Why eliminated from differential: the normal CSF analysis makes this diagnosis unlikely
Many patients with somatization disorder and hypochondriasis will resist psychiatric treatment because they believe that there are nonpsychiatric explanations for their multiple symptoms.
Unlike fibromyalgia, the symptoms of pain disorder are related strongly to stressful events.
Analgesic medications will not relieve the pain symptoms with genuine pain
disorder.
Münchhausen syndrome by proxy is a disorder in which parents try to make their children appear to have a certain disease. It is considered child abuse and must be reported to the appropriate authorities.
Case 13-10: “My father isn't himself” A 78-year-old man is an inpatient 1 day after the completion of a successful left hip replacement surgery. A psychiatrist is called to evaluate the patient because of acute mental status changes. The patient's daughter, who is present in the patient's room, says that after the surgery her father was sleepy but otherwise normal and conversive with his family. He was having postoperative hip pain and was using the prescribed morphine patient-controlledanalgesia devise to decrease his pain level. His daughter, who stayed overnight with him, was hitting his dose button throughout the night because she did not want him to wake up in pain. She says that he awoke early in the morning and has been very confused and agitated since that time. He has been asking repeatedly to speak to his brother, who passed away 5 years ago and keeps telling people that he needs to go to work in the factory, at which he worked 10 years ago. Later in the morning he got out of bed without any supervision or assistance and tried to walk down the hall. He fell and had to be returned to his bed by the nursing staff. X-rays of his operative leg and pelvis were negative, and the primary team ordered that he be put in soft restraints as long as he remained agitated. When approached, the patient is mildly agitated in bed. He asks the psychiatrist if he is a police officer because “[he has] been kidnapped and [needs] to get to work.” He repeats this question two other times during the assessment. When asked if he is in pain, the patient says that “these people held [him] down and kicked [him] in the hip.” His daughter says that he has a history of HTN and coronary artery disease (CAD) and normally takes lisinopril, atenolol, simvastatin, and low-dose aspirin. Postoperatively he has been started on short-term prophylactic IV cephazolin, IV morphine for pain control, and enoxaparin for deep-vein thrombosis (DVT) prophylaxis. He had no prior history of mental illness and had been living independently with his wife of 50 years prior to the surgery. He has a history of smoking tobacco for 60 pack-years but quit 2 years ago. He drinks a beer with dinner on a few nights of the week. On the mental status examination, he appears agitated and disheveled. He is in soft restraints in bed. He is intermittently cooperative with the examination, but most questions must be asked repeatedly until the patient provides an answer. The tone of his voice varies during the interview and is even at times and excessively loud at other times. He repeats certain questions and statements multiple times during the assessment. Some statements, particularly those involving people and places from the past, he is unable to expand upon when questioned further or challenged. His affect is labile and irritable. He remains awake throughout the conversation, but his level of interaction is variable. He can provide his name but he thinks that he is at home and that it is 1988. He can identify the season of the year correctly as winter, but is incorrect in naming the day, month, and date. Likewise he can correctly name the country, state, and city that he is in but not his exact location. When provided the names of three objects, he is able to repeat two of them and cannot remember any of the objects a few minutes later. When asked to spell “world” backward, he is unable to perform this task. When showed a pen, he is able to name it correctly. He is able to take a piece of paper fold it once and return it to the psychiatrist when asked to do so. He cannot repeat the phrase “no ifs, ands, or buts” when asked to do so. He is able to close his eyes on command but cannot write a sentence. He is unable to copy a picture of two shapes. For these tests his minimental status examination score is 11. On physical examination, his face and body appear symmetric in shape and movement. Auscultation of his lungs and heart is normal. He is neurovascularly intact in all extremities. The following vital signs are measured: T: 98.9°F, HR: 92 bpm, BP: 130/85 mm Hg, RR: 20 breaths/min
Differential Diagnosis Delirium, dementia, schizophrenia, cerebrovascular accident, meningitis, substance withdrawal, hypothyroidism, hyperthyroidism, folate deficiency, vitamin B12 deficiency
Laboratory Data and Other Study Results Pulse oximetry: 99% on room air CBC: WBC: 9.7, Hgb: 11.5, Plt: 327 Chem10: Na: 141 mEq/L, K: 3.9 mEq/L, Cl: 103 mEq/L, CO2: 26 mEq/L, BUN: 21 mg/dL, Cr: 1.0 mg/dL, Glu: 101 mg/dL, Mg: 1.8 mg/dL, Ca: 9.9 mg/dL, Phos: 3.8 mg/dL LFTs: AlkPhos: 112 U/L, ALT: 28 U/L, AST: 22 U/L, TBili: 1.0 mg/dL, DBili: 0.7 mg/dL UA: straw colored, pH: 6.9, specific gravity: 1.015, no glucose/ketones/nitrites/leukocyte esterase/hematuria /proteinuria Thyroid panel: TSH: 2.11 µU/mL, T4: 6.8 µg/dL, free T4 index: 7.0, T3: 1.3 ng/mL, T3 reuptake: 0.99 Vitamin B12: 591 pg/mL Folate: 10.1 ng/mL Head CT: normal cerebral and ventricular volume; no focal lesions or areas of ischemia; no sites of hematoma or hemorrhage
Diagnosis Delirium
Treatment Administered All narcotic medications were stopped The restraints were removed The patient was given a low dose of haloperidol to control his acute agitation A nursing assistant was assigned to be in the patient's room at all time to monitor his activity and prevent him from getting out of bed unassisted The window blinds of the room were kept open, and the lights were kept on during the day At night all lights were turned off, and the blinds were closed
Follow-up The patient's agitation decreased over the course of the subsequent two days, and he became more directable By 48 hours after the psychiatric assessment, the patient was oriented to his name, his location, and the time Pain control focused on nonsedating medications, with the occasional use of low-dose narcotics The patient was able to be discharged to a rehabilitation facility in stable condition on the fifth postoperative day
Steps to the Diagnosis Delirium An acute altered state of consciousness and orientation most commonly due to medications (e.g., narcotics, benzodiazepines, corticosteroids, antipsychotics, anticholinergics, alcohol), substance withdrawal,
infection, hypoxia, hypoglycemia, or sleep deprivation Generally reversible once the underlying cause is identified and treated History: acute development of an altered level of consciousness (e.g., inattentiveness, confusion), disturbance of sleep patterns, emotional lability, psychomotor agitation, or retardation Physical examination: the minimental status examination (MMSE) may be used to test cognitive function (a score below 25 is considered dysfunctional) (Figure 13-1)
Figure 13-1 The minimental status examination used to assess cognitive function.
Tests: typically noncontributory, but other causes of mental status changes should be ruled out with the appropriate testing Treatment: Any precipitating medications should be stopped or the dose should be decreased to the minimum required amount Infections should be treated with the necessary antibiotics and any additional required therapies
Supplemental oxygen should be supplied as needed Low-dose benzodiazepines should be administered in cases of substance withdrawal as a seizure prophylaxis and to calm rebound agitation Reorientation through observation, reassurance, normalization of sleep-wake cycles, and decreasing excess stimuli improves behavior Restraints should be avoided because they may actually increase agitation (unless the patient is in danger of harming him- or herself) Antipsychotics have a role in decreasing acute agitation but should be discontinued as soon as possible Outcomes: although most patients recover fully, there is a 25% mortality rate associated with patients with a significant delay in identifying the causative factor; complications arise from the cognitive impairments and include malnutrition, injuries from falls, aspiration, and pressure sores Clues to the diagnosis: History: prior normal cognitive function, excessive narcotic administration, acute mental status changes, disorientation to time and place, lack of insight, agitation, impoverished thought processes Physical: noncontributory Tests: noncontributory
Table 13-8 Comparison of Delirium and Dementia
Charac teristic
Delirium
Dementia
Onset
Acute
Gradual
Daily course
Fluctuating cognitive function and behavior
Generally consistent; sun-downing common
Level of consciousness
Decreased
Normal
Orientation
Aware of self; impaired for time and place
Generally impaired
Thought production
Disorganized, flight of ideas
Impoverished
Psychotic features
Delusions, hallucinations
Minimal
Memory
Short-term impairment
Short- and long-term impairment
Prognosis
Reversible
Usually nonreversible
Dementia A chronic progressive cognitive impairment that involves memory and at least one other cognitive function, does not involve changes in consciousness, and significantly limits the ability to function (Table
13-8) Multiple etiologies exist Alzheimer disease is a possibility (discussed more thoroughly in Chapter 7) Vascular dementia: due to multiple cerebral infarcts that also features neurologic deficits in addition to the cognitive impairment Parkinson dementia: commonly associated with Parkinson disease Alcohol-induced: dementia and aphasias associated with chronic alcohol use Less common causes include Huntington disease, normal pressure hydrocephalus, endocrine diseases, neoplasms, and chronic infections History: progressive memory impairments and a decreased ability to perform daily functions, possible aphasia (i.e., impairments of speech), apraxia (i.e., impaired purposeful movement), or agnosia (i.e., impaired recognition of objects and people) Physical examination: the MMSE will demonstrate cognitive dysfunction Tests: Endocrine and metabolic causes can be detected by measuring electrolytes, glucose, nutrient levels, and hormone levels CT or MRI of the brain may detect cortical atrophy, infarcts, or abnormal cerebral anatomy Positron emission tomography (PET) scans may be useful to demonstrate decreases in vascular flow or dopamine uptake Treatment: Purely endocrine or infectious causes may be reversible to some extent by treating the infection or normalizing the abnormal hormone function Cholinesterase inhibitors (e.g., tacrine, donepezil, rivastigmine), memantine, and vitamin E help to optimize the remaining cognitive function Antipsychotics and antidepressants may be used to treat comorbid psychosis and depression Supervised care is frequently needed in all patients who develop significant cognitive impairments that prevent independent living Outcomes: in the vast majority of cases dementia is a permanent and progressive decline in cognitive function that leads indirectly to mortality Why eliminated from differential: the report that the patient was living independently prior to the hospital admission suggests an acute change in mental status and makes this diagnosis unlikely
Elderly patients are particularly susceptible to delirium during inpatient stays.
Delirium differs from “sundowning,” the deterioration of behavior during evening hours in patients with dementia, in that it occurs in patients without a history of dementia and may be linked to a medical or substance-related cause.
Causes of dementia that may be reversible can be remembered by the mnemonic MIND HATS: Metabolic (electrolytes, endocrine disorders), Infection, Nutrition (poor), Drugs, Hydrocephalus (normal pressure), Atherosclerosis, Tumors, Sensory deficits (vision, hearing).
More than 70% of the cases of dementia are due to Alzheimer disease; vascular disease is the second most common cause of dementia and causes 15% of cases. Schizophrenia More thorough discussion in prior case Why eliminated from differential: the acute nature of the patient's changes in behavior makes this diagnosis unlikely Cerebrovascular accident More thorough discussion in Chapter 7 Why eliminated from differential: the normal head CT rules out this diagnosis Meningitis More thorough discussion in Chapter 7 Why eliminated from differential: the normal WBC level decreases the suspicion for this diagnosis, but continued cognitive impairments should prompt the performance of a lumbar puncture Substance withdrawal More thorough discussion in prior case Why eliminated from differential: the patient's prior use of alcohol should raise the concern for this diagnosis, but the absence of physical signs of withdrawal (e.g., tachycardia, diaphoresis, emesis, delirium tremens) makes it less likely Hypothyroidism/hyperthyroidism More thorough discussion in Chapter 5 Why eliminated from differential: the normal thyroid panel rules out these diagnoses Folate/vitamin B12 deficiency More thorough discussion in Chapter 6 Why eliminated from differential: the normal folate and vitamin B12 levels rules out these diagnoses
Case 13-11: “My son is having problems in school” An 8-year-old boy is brought to a pediatric psychiatrist by his mother because he is having difficulties in school. The primary school's psychologist and his teachers are concerned about his disruptive hyperactivity and have suggested to his patients that he be evaluated by a psychiatrist. The boy's mother says that her son has always been hyperactive and that from the time he was 3 years old he had difficulty sitting still and following directions. It was difficult to bring her son to any event or function that required him to sit still because he was always talking loudly, leaving his seat, and trying to distract other people. She says that she and her husband thought that the structure of
school may help to improve his behavior. He started kindergarten when he was 5 years old and did fairly well, but his teachers reported the same problems that his parents were having at home. When he was in first grade, he was forced to repeat the grade because he did not meet the milestones required for advancement. Mainly, his teachers reported he had difficulty finishing projects, did not follow directions, caused distractions in class, and was unable to pay attention to teaching. When he started second grade at 8 years of age, he was placed in special learning groups to help with his work. He has had the same problems of following instructions, paying attention to his teachers, and becoming easily distracted by outside stimuli. In addition, he constantly fidgets in his seat, frequently talks when he is supposed to be paying attention, and leaves his seat to move around the room when he should be seated. His mother says that she sees these behaviors at home. When asked to help his father take out the garbage, he must be told several times before following instructions. He tends to wander away from conversations before they are finished. He has lost three backpacks this school year and forgets his lunch at home at least twice per week. It is always a struggle to get the boy to eat dinner with the family because he constantly leaves the table before anyone else is finished. When he is told to wait until the end of the meal, he constantly fidgets in his seat and plays with objects on the dinner table. When other family members are talking, he frequently interrupts them or tries to finish their story. His mother says that he has never intentionally hurt someone or purposefully tried to break anything. He has never stolen anything while they are food shopping or played with fire. She says that when she works with him on his homework, he seems to be able to do the work but will not sit still long enough to finish an assignment in one sitting. He was the product of a full-term, uncomplicated vaginal birth. He met all of the appropriate developmental milestones before starting school. He is up to date on all of his vaccinations and has never had any serious illness. He has one older brother and one older sister who are in good health and do not display any of the behavioral symptoms of the patient. Both siblings have done well in school without any difficulties. On the mental status examination, the patient is overly active and is running back and forth across the examination room when the psychiatrist enters. His mother must frequently ask him to sit still while the physician is speaking with her. He cooperates with the examination when doing activities but becomes fidgety when asked multiple questions. His speech is rapid, and he frequently tries to interrupt his mother or correct her while she is talking. When asked about school, he provides short answers without much expansion. He has tangential speech when asked some questions about school and must be reined in by his mother to return to the original topic. He appears happy, and his affect is euthymic. He is able to tell the physician his name, the current day of the week, and his general location. He can repeat back three listed words to the psychiatrist, but only remembers one word a few minutes later. When asked to perform three-part tasks, he occasionally leaves out one of the steps. He drops a word when he attempts to repeat offered multiword phrases. He is able to follow simple single-command instructions and is able to reproduce a line drawing. On a basic visual acuity test he scores a 20/30 for each eye. A hearing assessment is normal. The following vital signs are measured: T: 98.6°F, HR: 70 bpm, BP: 110/75 mm Hg, RR: 18 breaths/min
Differential Diagnosis Attention-deficit/hyperactivity disorder, conduct disorder, learning disability, hyperthyroidism
Laboratory Data and Other Study Results Thyroid panel: TSH: 3.18 µU/mL, T4: 8.6 µg/dL, free T4 index: 8.4, T3: 1.1 ng/mL, T3 reuptake: 0.92
Diagnosis Attention-deficit/hyperactivity disorder
Treatment Administered The patient was prescribed methylphenidate to improve his ability to focus Behavioral psychotherapy was initiated to help improve the child's ability to perform goal-oriented activities
Follow-up Following the initiation of methylphenidate, the child's performance improved significantly at school Although the patient remained an “on-the-go” individual and his grades were average, he was able to progress through school without any further difficulties The patient's behavior continued to improve into young adulthood, he attended a vocational school and did well, and he was able to be weaned from methylphenidate by the time he was 21 years old
Steps to the Diagnosis Attention-deficit/hyperactivity disorder (ADHD) A disorder of inattention and hyperactivity in school-aged children that causes problems in multiple settings Diagnosis requires the child to have either six inattention symptoms or six hyperactivity and impulsivity symptoms before the age of 7 years that limit the ability to function in social, educational, and organized settings History: Inattention: decreased attention span, difficulty following instructions, carelessness in tasks, easily losing items, forgetfulness, poor listening skills, easy distractibility, difficulty organizing activities, avoidance of tasks requiring prolonged focus Hyperactivity: fidgetiness, inappropriate activity, excessive talking, inability to remain quiet, inability to remain seated at times where prolonged sitting is required, constantly “on the go” Impulsivity: difficulty waiting one's turn to speak, frequent interruptions of others, answering of questions before they are completed Physical examination: patient activity during the examination will reflect the classic symptoms Tests: typically noncontributory Treatment: Psychostimulants (e.g., methylphenidate, pemoline) or atomoxetine improve the ability to focus and help control behaviors Bupropion, α-agonists, and TCAs may serve a role in refractory cases Behavioral psychotherapy is used to address a child's self-esteem and to help modify inappropriate behaviors The child's educational setting may need to be adjusted to optimize the ability to participate and learn Outcomes: The severity of the disease improves with age, and most patients function well by the time of young adulthood Despite the improvements in symptoms in most children, up to 50% of children will continue to exhibit some signs of the condition into adulthood, including 15% of cases that show minimal improvement The rate of substance use later in life is higher than in unaffected children Tourette syndrome is a chronic tic disorder (motor and vocal) that is seen with higher frequency in
patients with ADHD or OCD Clues to the diagnosis: History: decreased attention span, difficulty following instructions, forgetfulness, recurrent loss of possessions, easy distractibility, inappropriate talking, inability to remain quiet, difficulty in not speaking out of turn, interrupting of others, prematurely answering questions Physical: fidgetiness Tests: noncontributory
ADHD is four times more common in males than in females. Conduct disorder A behavioral disorder of repetitive disruptive and antisocial activities that violate others' rights and the social norms Diagnosis requires the presence of a problematic behavior before the age of 10 years and three such behaviors after the age of 10 years History: aggressive behavior toward people or animals, intentional destruction of property, deceitfulness or theft from others, violation of serious rules Physical examination: noncontributory Tests: noncontributory Treatment: Psychotherapy involving the entire family and parent management training may help to control conduct Psychostimulants are prescribed in cases of comorbid ADHD Mood stabilizers may be required in severe cases Outcomes: One or more comorbid psychiatric disorders are found in up to two thirds of patients and include ADHD, anxiety disorders, mood disorders, and learning disabilities Substance use and risky behaviors are much more common in these patients than in the general population Patients frequently will progress to antisocial personality disorder if not appropriately treated Why eliminated from differential: the lack of hurtful activities in this case rules out this diagnosis
Oppositional defiant disorder is similar to conduct disorder in that patients exhibit aggressive behavior, but illegal and destructive activities do not occur.
ADHD is a comorbid condition in 50% of cases of conduct disorder. Learning disability Impairment in the educational development in a healthy child with no other psychiatric diagnoses or
cognitive pathologies (e.g., Down syndrome, fragile X syndrome) Impairments may be specific to reading, calculations, or the expression of thoughts History: language delays, impaired coordination, memory impairments, inattentiveness, poor spatial or temporal ordering skills, or poor reading ability in a child with an otherwise normal intelligence Physical examination: typically noncontributory Tests: scores on standardized knowledge tests are consistently lower than the normal range Treatment: Placing the child in a special education group and initiating therapy focusing on the specific learning disorder can help the child improve his or her ability to learn Parent education is important to help optimize the learning environment for the child Each child's strengths should be recognized and encouraged to improve overall education and self-esteem Outcomes: early recognition and institution of a controlled learning environment will optimize a child's educational potential, while failure to do such will precipitate further difficulties in school and social function Why eliminated from differential: the child's difficulties in school in this case seem to stem more from his hyperactivity than a specific difficulty in learning material Hyperthyroidism More thorough discussion in Chapter 5 Why eliminated from differential: the normal thyroid panel rules out this diagnosis
Auditory and visual pathologies must be ruled out in a patient suspected for having a learning disorder.
Authors: Van Kleunen, Jonathan P. Title: Step-Up to USMLE Step 3, 1st Edition Copyright ©2009 Lippincott Williams & Wilkins > Back of Book > Appendices
Appendices
Appendix A: Epidemiology Research Studies Basic study requirements Subjects must be representative of the population examined The number of subjects must be sufficient to detect statistical significance (i.e., adequate power) Subjects must be able to give their informed consent to participation Proper control groups are important to a study examining treatment efficacy The interests of the patient take precedence over those of the study Investigators must track data to look for potential risks to the subjects An excessive amount of risk should lead to an early termination of the study Subject confidentiality must be maintained Human studies must be approved by an institutional review board (IRB), and animal studies must be approved by an analogous animal review committee Study design The chosen study design is based on the type of question proposed, the size of the affected patient population, the incidence of the disease or exposure in question, and the limitations of time and funding upon the study (Table A-1) Study bias Biases in the design of a study or data interpretation will detract from the accuracy of the results (Table A-2)
Investigator and observational bias may be avoided by double-blinding a study.
Confounding variables are factors that affect both the experimental and control groups and interfere with the relationship between these groups.
The relative risk is determined through cohort studies.
Biostatistics Disease rates Incidence: the number of new cases that occurs within a certain time period in a population (i.e., the likelihood of developing that condition in that period of time)
Prevalence: the number of individuals with a certain diagnosis at a given moment of time
Case fatality rate: the percentage of individuals with a certain disease who die within a certain amount of time
Disease-exposure associations Relative risk (RR): the risk of contracting a certain disease in a population with a common type of exposure (Table A-3)
P.324
Table A-1 Study Designs Used in Clinical Research
Study Type
Randomized clinical trial
Desc ription
Conc lusions
Effectiveness of
Prospec tive c omparison of experimental treatment to plac ebo controls and existing therapies Double-blinded to avoid bias
experimental treatment compared to controls and existing therapies
Advantages
Gold standard for testing therapies May be controlled for several confounders
Disadvantages
Often c ostly and time
Gold standard for
Often
c onsuming, and patients
c ostly
may not be willing to undergo
and time
randomization
Patients randomized into study groups
Cohort study
Examines a group of subjects exposed to a given situation or
Examines a group of
Relative risk
Able to examine rare
subjects
exposures
factor
Can study multiple
May be prospec tive (exposed group
effects of exposure
May be costly and time consuming, and it is difficult to study rare diseases
identified and followed over time) or retrospec tive (examines exposed group in whom disease has already occurred)
Case-control study
Retrospective comparison of patients
Odds ratio
May examine rare
with a disease to healthy controls;
diseases or those with
frequency of certain exposures in
long c ourse in short
both groups is considered
amount of time
Susceptible to rec all and selec tion bias and cannot determine disease incidence
Can study multiple types of exposure May examine small group size
Crosssectional survey
Survey of large number of people at one time to assess exposure and disease prevalence
Disease prevalence and a hypothesis for risk factors
Can be used as an estimate for disease
Cannot be used to test hypotheses
prevalence following exposure
Case series
Report of characteristics of a disease by examining multiple cases
Hypothesis for risk factors
May be easy to complete
Cannot be used to test hypotheses
Provides insight into poorly understood conditions
Meta-analysis
Pooling of multiple studies examining a given disease or exposure
Depends on original study type
Larger study size Can resolve conflicts in
Unable to eliminate limiting factors in original studies
literature
An RR >1 suggests a positive relationship between the exposure and the disease (i.e., exposure increases the likelihood of developing the disease) An RR <1 suggests a negative relationship between the exposure and the disease (i.e., exposure decreases the likelihood of developing the disease) An RR equal to 1 suggests no relationship between the exposure and the disease Odds ratio (OR): the odds of having a history of a certain exposure among patients with a disease compared to those without the disease (Table A-3) It is considered an estimate of the relative risk for that exposure
Attributable risk (AR): the difference in the rates of a disease between exposed and unexposed populations AR = (rate of disease in exposed population) - (rate of disease in unexposed population) Statistics of diagnostic tests P.325
Table A-2 Types of Bias in Clinical Studies
Ty pe of Bias
Enrollment (selection)
Desc ription
Nonrandom assignment of subjects to study groups
Consequenc es
Results of the study may not be applicable to the general population
Investigator
Subjective interpretation of the data by investigator deviates toward
Results of study will incorrectly resemble the proposed hypothesis
“desired” conclusions
Lead-time
A screening test provides an earlier diagnosis in the studied group when
The time from diagnosis to outcome increases because the earlier diagnosis causes the false appearance of an
compared to controls but has no effec t on time of survival
increased time of survival; the time from the beginning of the disease process to the outcome actually remains the same regardless of screening
Length
A screening test detec ts several slow ly progressive cases of a disease and
Effectiveness of screening test is overstated
misses rapidly progressive cases
Observational
Subjects may respond to subjective questions in a different way than normal
Effectiveness of therapy is not accurately depicted by study group
because their aw areness of the study changes their perception of the examined issue
Publication
Studies that show a difference between groups are more likely to be
Data available for meta-analyses may not include studies that support the null hypothesis
published than studies that do not show a difference
Recall
Errors of memory within the subjects occurs due to prior c onfounding
Patients with negative experiences are more likely to recall negative details
experienc es
Self-selection
Patients with a certain past medic al history may be more likely to
Subjects are not representative of the general population and introduce confounding variables
participate in a study related to their condition
Sensitivity: the probability that a screening test will be positive in patients with a disease (Table A-4)
Most acceptable screening tests are >80% sensitive A false negative is a negative test in a patient with the disease The false negative rate is approximated by (1 - sensitivity) Specificity: the probability that a test will be negative in patients without a disease (Table A-4)
Most acceptable confirmatory tests are >85% specific A false positive is a positive test in a patient without the disease The false-positive rate is approximated by (1 - specificity)
Table A-3 Calculation of Disease Risk
Disease
Exposure
Yes
No
Yes
A
B
No
C
D
P.326
Table A-4 Analysis of Diagnostic Tests
Disease
Yes
No
Test
Positive
A
B
Negative
C
D
Positive predictive value (PPV): the probability that a patient with a positive test has the disease of interest (Table A-4)
Negative predictive value (NPV): the probability that a patient with a negative test does not have the disease of interest (Table A-4)
Likelihood ratios: the odds of having a positive test result in patients with a disease compared to those without it (i.e., positive likelihood ratio; PLR) or the odds of having a negative test result in patients with a disease compared to those without it (i.e., negative likelihood ratio; NLR) Used to measure the performance of a diagnostic test while eliminating the test's dependence on the disease prevalence
Accuracy: the performance of diagnostic tests, considering only the number of true positive and true negative results
The odds ratio is determined through case-control studies.
The odds ratio is the most accurate estimate of relative risk in cases of rare diseases.
Screening tests seek to reliably detect a disease in a patient without incorrectly diagnosing people without the disease (i.e., ideally both high sensitivity and high specificity).
Confirmatory tests are used to validate that a patient with a positive test truly has a disease.
A disease with a high prevalence will be associated with a high positive predictive value in a screening test.
A disease with a low prevalence will be associated with a high negative predictive value in a screening test. Error in studies Null hypothesis: the hypothesis that there is no association between the exposure and a disease or the treatment and a response Alternative hypothesis: the hypothesis that there is an association between the exposure and a disease or the treatment and a response Type I error: the null hypothesis is rejected although it is true (i.e., false positive) Type II error: the null hypothesis is not rejected although it is false (i.e., false negative) The risk of an error occurring decreases with an increasing sample size (i.e., increasing study power) Significance: the statistically detectable difference between two sample groups Probability value (P value) The chance of a type I error occurring for a given result If P <0.05, the null hypothesis can be rejected (i.e., there is a significant relationship between the groups)
Confidence interval (CI) A range of values (e.g., relative risk, odds ratio, objective data) calculated in a study and the likelihood that the true result falls within that range P.327 Most commonly listed as a 95% CI (i.e., there is a 95% chance that the true result falls within the study-derived range) If the CIs for a measure in two sample groups overlap, the likelihood that the measure is statistically different between the groups is low If the CI for a calculated relative risk or odds ratio include 1.0 within its range, the risk of exposure is not significant Power: the ability of a study to detect an actual difference between two groups Studies with insufficient power may state two groups are equal when they are actually significantly different (i.e., occurrence of a type II error) P.328
Appendix B: Ethics Patient Rights Confidentiality All information regarding the patient must be kept private between the physician and the patient All patient records and billing must be designed to maintain confidentiality The exchange of patient information may only occur between providers involved with the care of that patient Confidentiality may be forgone in some circumstances The patient gives the physician permission to share information with designated parties (e.g., family members) The patient carries a legally reportable disease (e.g., human immunodeficiency virus [HIV], tuberculosis [TB], syphilis) The patient is homicidal or suicidal The patient has sustained a penetrating wound from an assault The patient is an adolescent with a condition that is potentially harmful to him or herself or others
Confidentiality should be maintained in adolescents seeking treatment for sexually transmitted diseases (STDs) or pregnancy (a point that may need to be clarified with parents).
The patient should be made aware that certain diseases or conditions will be reported.
A competent patient may change his or her mind regarding accepting therapy at any time. Public reporting By law, a public health department must be notified of cases of certain STDs or other transmissible diseases (e.g., HIV, hepatitis, Lyme disease, food-borne illness, meningitis, rabies, and TB) Impairments in driving ability, child abuse, and elder abuse must be reported to authorities (exact legal requirements vary from state to state) Informed consent Prior to any procedure or therapy, the patient must be made aware of the indications, risks, and potential benefits of a proposed treatment, any alternative treatments and their risks, and the risks of refusing treatment Informed consent (including parental consent in the case of a minor) is not required for emergent therapy (i.e., implied consent) If a patient is not capable of making a decision for a nonemergent procedure, a designated surrogate decision maker is required Full disclosure Patients have the right to be made aware of their medical status, prognosis, treatment options, and medical errors in their care If a family requests that a physician withhold information from the patient, physicians must deny the request unless it is determined that disclosing information would significantly harm the patient
Medical Decision Making Competency A competent patient is entitled to make all decisions regarding his or her medical care
Requirements for competency The patient cannot be currently psychotic or intoxicated P.329 The patient must have an understanding of his or her medical situation The patient must be capable of making decisions that are in agreement with his or her history of values Medical decisions for nonemancipated minors (i.e., younger than 18 years) are made by a minor's parents unless legally ruled to be not in the best interests of the child Emancipated minors (e.g., pregnant, married, financially independent) may make their own decisions
Parents' decisions regarding their children may be legally overruled if they are considered harmful to the children. Durable power of attorney Legal documentation that designates a second party (e.g., family member) as a surrogate decision maker for medical issues (not valid in all states) The designated individual should be able to make decisions consistent with the patient's values Living will A written document that details a patient's wishes in specific medical situations (e.g., resuscitation, ventilation, extraordinary maintenance of life) The patient is responsible for creating the definitions of his or her own living will It may be more stringent than a durable power of attorney to solidly define the patient's wishes
End-of-Life Issues Do-not-resuscitate orders (DNR) A type of advanced directive document that details care in cases of coma, cardiac arrest, severe dementia, and terminal illness A DNR may object to all nonpalliative therapies or may only restrict use of specific therapies (e.g., ventilation, cardiopulmonary resuscitation, feeding tubes, antibiotics) A physician writes a DNR in correspondence with the patient's wishes (may be outlined in a living will) Life support Competent patients may request to have supportive measures withdrawn at any time (including through a living will) Physicians are able to remove respiratory care in cases in which there is no living will and the patient is incapable of voicing a decision if both the family and the physician believe that removal of care is consistent with what the patient would want Physician-aided death Physician-assisted suicide occurs when a physician supplies a patient with a means of ending his or her life but is not directly responsible for the death Euthanasia is the active administration by a physician of a lethal agent to a patient in order to cause death and eliminate suffering from a condition
Physicians are not required to supply therapies that are irrational for the current condition or when the maximum reasonable therapy has already failed.
The absence of electroencephalogram (EEG) activity does not define brain death but may help prompt a brain death work-up
Conditions that may mimic brain death include metabolic encephalopathy, hypothermia, intoxication, locked-in syndrome, and Guillain-Barre syndrome. Death Brain death is defined as the irreversible absence of all brain activity (including the brainstem) in a patient lasting more than 6 hours The absence of cranial nerve reflexes (e.g., gag, corneal, and caloric reflexes) Apnea off of a ventilator for a duration considered sufficient to produce a normal hypercarbic drive The absence of hypothermia or intoxication The absence of brainstem-evoked responses, a persistent isoelectric EEG, or absent cerebral circulation on vascular imaging studies A patient's appearance cannot be explained by a medical condition that mimics death
Cardiac death is considered the inability to restore a spontaneous heartbeat in an asystolic patient Either brain death or heart death may be used to define formal patient death, but both are not required Hypothermic patients must be warmed to a normal body temperature before death can be declared P.330 Organ donation Patients may declare themselves as organ donors prior to death (e.g., living will, designated on driver's license) Patients and their families may define exactly which organs may be donated Hospitals receiving payments from Medicare are required to approach the family of the deceased regarding organ donation Organs may be judged unsuitable for donation in cases of widespread or uncured neoplasm, sepsis or blood-borne infection, compromised organ function, organ-specific infection or disease, hypothermia, age >80 years, hemoglobinopathy, or prolonged ischemia P.331
Appendix C: Pediatric Development Pediatric Physical Growth Weight Rate of weight gain Body weight decreases for the first few days after birth The birth weight is regained by 2 weeks after delivery The child's weight should double by 4 months, triple by 1 year, and quadruple by 2 years Annual weight gain should be approximately 5 pounds between 2 and 13 years of age Inadequate weight gain may be due to poor food intake, chronic vomiting or diarrhea, malabsorption, neoplasm, or congenital diseases (e.g., cardiac, endocrine) Weight below the 5th percentile suggests failure to thrive Childhood obesity (weight over the 95th percentile) is associated with increased rates of sleep apnea, hypertension (HTN), slipped capital femoral epiphysis, precocious puberty, skin infections, social dysfunction, and diabetes mellitus Height Rate of height gain The child's height should increase by 50% by 1 year after birth, should double by 4 years, and should triple by 13 years of age The annual increase in height should be 2 inches per year between 2 and 13 years of age Greater-than-normal height may be associated with familial tall stature, precocious puberty, acromegaly, hyperthyroidism, Klinefelter syndrome, Marfan syndrome, or obesity Lower-than-normal height may be associated with familial short stature, neglect, Turner syndrome, constitutional growth delay, chronic renal failure, asthma, cystic fibrosis, inflammatory bowel disease, immunologic disease, growth hormone deficiency, hypothyroidism, glucocorticoid excess, skeletal dysplasias, or neoplasm
Causes of short stature may be remembered by the mnemonic ABCDEFGHIJKL: Abuse, Bad cancers, Chromosomal (Turner), Delayed growth (constitutional), Endocrine (growth hormone deficiency, hypothyroidism), Familial short stature, GI disease (IBD, celiac), Heart (congenital disease), Immune disorders, Joint and bone dysplasias, Kidney failure, Lung disease (asthma, cystic fibrosis). Head circumference Rate of increase Approximately 5 cm of growth occurs by 3 months after birth, an additional 4 cm by 6 months, an additional 2 cm by 9 months, and an additional 1 cm by 1 year Macrocephaly may be associated with cerebral metabolic diseases (e.g., Tay-Sachs, maple syrup urine disease), neurocutaneous syndromes (e.g., neurofibromatosis, tuberous sclerosis), hydrocephalus, increased intracranial pressure, skeletal dysplasia, acromegaly, or intracranial hemorrhage Microcephaly may be associated with fetal toxin exposure (e.g., fetal alcohol syndrome), chromosomal trisomies, congenital infection, cranial anatomical abnormalities, metabolic disorders, or neural tube defects Cumulative growth abnormalities
A child with normal fetal growth, an initially normal growth rate, and a delayed onset of inadequate growth following birth has a postnatal cause of the growth abnormality Growth that is abnormal from the time of birth suggests a prenatal onset (e.g., genetic abnormalities, intrauterine pathology) P.332 Growth that is low-normal in range but eventually becomes closer to the mean suggests constitutional growth delay Growth that is consistently low-normal suggests genetic short stature A comparison to the parents' growth histories will help make this diagnosis Other symptoms and physical signs are useful for determining the cause of the growth abnormality Growth should be assessed and recorded on a growth chart at each health visit to confirm normal patterns and to catch abnormalities early in their process
Ages for developmental milestones are guidelines. It is normal for milestones to occur at an appropriate range of ages, and parents should be reassured that milestones occur within such a range and not at concrete ages.
Be sure to collect a thorough family history while assessing growth or developmental delays in order to help distinguish a hereditary cause from an environmental one.
Pediatric Developmental Milestones Schedule of milestones Social, physical, and intellectual achievements are reached by children at characteristic ages (Table A-5) The absence of delay of milestones may suggest developmental delays Some delays are hereditary, but the presence of multiple or persistent delays is concerning for pathologic process (e.g., mental retardation, genetic disorders, language or hearing impairments, child abuse, psychiatric disease) The persistence of infantile reflexes beyond 6 months of age may suggest a central nervous system abnormality Adolescence A period of rapid physical, psychosocial, and sexual growth during the second decade of life Psychosocial issues Early adolescence (i.e., 10 to 13 years old) is typified by concrete thinking and early independent behavior
Table A-5 Important Developmental Milestones During Childhood
A ge
Soc ial/Cognitive
Gross Motor
Fine Motor
2 months
Social smile
Lifts head 45°
Eyes follow object to midline
4 months
Laughs
Lifts head 90°
Eyes follow object past midline
Differentiates parents from others
Rolls over
Grasps/rakes
Separation anxiety
Holds self up with hands
Attempts to feed self
Language
Coos
Aware of caregiver Localizes sound
6 months
Babbles
Sits
9 months
Crawls
Grasps with thumb
First words
12
Pulls to stand
Pincer grasp
~5- to 10-word
months
Walks with help
Makes tower of 2 blocks
vocabulary
Walks well
Makes tower of 4 blocks
10- to 50-word
Walks backwards
Uses cup or spoon
vocabulary
18
Interactive games
Parallel play
months
2-word sentences
2 years
3 years
Dresses self with help
Magical thinking
Runs
Makes tower of 6 blocks
50- to 75-word
Climbs stairs (initially 2 feet/step, then 1
vocabulary
foot/step)
3-word sentences
Climbs/descends stairs
Makes tower of 9 blocks Able to draw circle (O)
4 years
Plays with others
6 years
Hops on 1 foot
Able to distinguish fantasy from
Skips
Able to draw line image (+), later able to draw closed line drawing
250+ word vocabulary
(▲)
4-word sentences
Draws a person
Fluent speech
reality
P.333 Middle adolescence (i.e., 14 to 16 years old) is typified by the emergence of sexuality (i.e., sexual identity, sexual activity), an increased desire for independence (i.e., conflict with parents, need for guidance, self-absorption), and abstract thought Late adolescence (i.e., 17 to 21 years old) is typified by an increased self-awareness, an increased confidence in one's own abilities, a more open relationship with one's parents, and cognitive maturity Adolescents are at an increased risk for risk-taking behaviors (e.g., drug use, unprotected sexual activity, violence), depression, suicidal ideation, homicide, and eating disorders It is important to consider both the psychological and emotional issues of these patients in addition to their medical issues during routine office visits Patient issues should be approached in a nonjudgmental fashion
Some teaching methods that maximize the communication of important information during a visit are repetition, positive reinforcement, limitation of the number of topics discussed, and not focusing on minor points.
Childhood Health Maintenance Well-child visits Periodic physician visits are important during childhood to assess growth and developmental milestones, provide vaccinations, screen for certain disease processes, and provide anticipatory guidance (Table A-6) Screening tests should be performed to address common medical concerns (e.g., vision, hearing, dentition, diseases in high-risk populations) Anticipatory guidance should address nutrition, development, daily care, accident prevention, and behavioral issues
Table A-6 Screening Performed and Anticipatory Guidance Discussed during Regular Childhood Health Visits
V isit
Sc reening
Newborn/1
Phenylketonuria, hypothyroidism, genetic
week
metabolic disorders (maple syrup urine
Nutrition
Breast or bottle feeding
Daily Care
Ac c ident Prevention
Crying, sleep position,
Smoke detectors, baby
bathing
furniture, car seats
Breast or bottle feeding,
Sleep, bowel and
Sun exposure
fluoride supplements
bladder habits
Behavioral Issues
Parent-child interaction
disease, cystic fibrosis, etc.) in highrisk patients, hearing, visual mobility and reflexes
1 month
2 months
Visual mobility and reflexes
Visual mobility and reflexes
Sleep, bowel habits
Importance of close contact
Close supervision, risks due to ability to roll over
4 months
Visual mobility and reflexes
Solid foods, fruits and
Teething
vegetables
6 months
9 months
12 months
15 months
Visual mobility and reflexes
Hgb/Hct; visual mobility and reflexes
Keeping small objects out of
Vocal interaction
reach
Preparation for increased
Stranger anxiety, separation
needs, finger foods,
mobility (“child-proofing” the
anxiety
avoidance of milk or juice at
house), electrical socket
bedtime
covers, stair and door gates
Cup training, daily caloric
Shoes
Iron supplementation,
Tooth care, favorite
self-feeding, spoon training
toys
Aspiration risks
Communication, discipline
Visual mobility and reflexes, lead exposure,
Bottle weaning, eating at
Poisoning risks, stair safety,
Speech, rules, positive
PPD in high-risk areas
table, whole cow's milk
burns
reinforcement
Hgb/Hct, visual mobility and reflexes
Family meals
Toilet training, temper tantrums, punishment, listening to parent read
18 months
Visual mobility and reflexes
Reinforcement of utensil use
Nightmares, bedtime
Supervised play, dangerous toys
regimens
2 years
3 years
Lead exposure, visual mobility and reflexes
Visual acuity, cholesterol (if family history of
Discipline, “terrible 2s”, toilet training, games
Avoiding unhealthy snacks,
Transition from crib
Toddler independence, explanation
encouraging eating during
to bed, toothbrush
of body parts, early play with other
meals
training
children
Healthy diet
Regular sleep
high cholesterol or CAD)
Water safety, animal safety
Day care or babysitters (earlier if
schedule, television
both parents work), play with other
limitation
children, reinforce consistent toileting, conversation
4 years
Hearing, lead exposure, visual acuity, PPD (if
Meals as time for family
high-risk group), urinalysis
bonding
Self dental care
Pedestrian and bicycle safety,
Chores, interactions at day care or
car seat or seat belt, dangers
preschool, school preparation
of strangers, guns, fires, poisons, teach phone number
6 years
Lead exposure, visual acuity
Avoidance of excess weight,
Exercise, hygiene,
obesity prevention
school activities
Swimming
Allowance, encourage learning and development, reading
counseling
10 years
12 years
Hearing, visual acuity
Hearing, visual acuity, PPD (if high-risk
Adequate sleep,
group), Pap smear only if sexually active
school and
(girls)
extracurricular
Hazardous activities, drug use
Friends, sexual education, puberty,
(including alcohol and tobacco)
responsibility
Sexual responsibility
Body image, privacy issues
Risk-taking behavior, driving,
Dating, sexuality, goals, careers,
sexual responsibility
independence issues
activities
14 years
Hearing, Hgb/Hct (girls), visual acuity, STD
and older
screening (if sexually active)
Weight maintenance
School and activities
Note: Anticipatory guidance from prior visits should be review ed when appropriate. CAD, coronary artery disease; Hgb/Hct, hemoglobin and hematocrit; PPD, purified protein derivative of tuberculin (TB test); STD, sexually transmitted disease.
P.334 P.335
Table A-7 Vaccination Schedule and Contraindications During Well-Child Health Visits (2008 Recommendations)
Vaccinations should be administered at the appropriate visits according to the recommended schedule (Table A-7)
The first dose of hepatitis B vaccine is typically administered shortly after birth before the infant leaves the hospital.
Haemophilus influenzae type b (Hib) vaccine is unnecessary in previously unvaccinated children older than 5 years because of the low risk of severe infection at this age and older. Asplenic children should always receive Hib and pneumococcal vaccines regardless of their age. P.336
Appendix D: Traumatology Mechanisms of Injury Blunt trauma: contusion of local structures and induction of a significant local inflammatory response Penetrating trauma: the penetrating object (e.g., bullet, sharp object) damages tissue in path of its trajectory and causes indirect damage from fragmented bone and external objects Acceleration-deceleration injuries (e.g., falls, motor vehicle crashes): injury occurs secondary to shearing forces in tissues due to sudden changes in momentum and sudden forces applied to tethered portions of organs (e.g., aortic arch, mesentery)
Count and pair all entrance and exit gunshot wounds to suggest a number of insulting bullets and to deduce a path for each bullet.
Loss of consciousness is considered due to head trauma until ruled out.
Hypertension with bradycardia is suggestive of increased intracranial pressure (i.e., Cushing phenomenon).
Key points of the neurologic examination for suspected head trauma may be remembered by the 12 Ps: Paired ocular movement, Papilledema, Paralysis, Paresthesias, Patellar and other reflexes, Pee (incontinence), Pressure (blood and intracranial), Psychological (mental) status, Ptosis, Pulse rate, Pupils (size, symmetry, reflexes), Pyramidal signs (abnormal movements).
Rule out cervical fracture and spinal cord injury before performing any examination requiring head movement.
Assessment of the Trauma Patient The primary assessment must be performed in an organized manner to detect all life-threatening injuries and to judge their severity The initial assessment focuses on the ABCs A secure airway must be established Breathing is confirmed, and adequate oxygenation (e.g., supplemental oxygen, ventilation) is secured Adequate circulation is confirmed, adequate vascular access is secured, and bleeding is controlled The secondary assessment consists of a highly detailed examination performed to detect all wounds, fractures, signs of internal injury, and neurologic insult The Glasgow coma scale (GCS) and the revised trauma score (RTS) are used to objectify injury severity (Tables A-8 and A-9) Head trauma Trauma may cause direct injury or secondary injuries though intracranial bleeding Cerebral damage may be at the point of insult (i.e., coup) or on the opposite side of the head (i.e., contrecoup) The physical examination should include assessments of the level of consciousness, papillary light reflex, and oculocephalic reflex A head and cervical spine computed tomography (CT) should be performed on every patient with suspected head trauma Management should focus on maintaining cerebral perfusion, decreasing excessive intracranial pressure, and stabilizing spinal instability Neck trauma Because of the multiple vascular, neurologic, gastrointestinal (GI), and respiratory structures, injuries can quickly become life-threatening The physical examination should include assessments of perfusion, airway patency, and neurologic injury X-ray and CT of the neck, Doppler ultrasound (US), esophagogastroduodenoscopy (EGD), bronchoscopy, and angiography are tests that should be considered in the proper clinical scenarios P.337
Table A-8 Glasgow Coma Scale1
Category
Eye opening
Verbal response
Motor response
Condition
Points
Spontaneous
4
To voice
3
To pain
2
None
1
Oriented
5
Confused
4
Inappropriate words
3
Incomprehensible
2
None
1
Obeys commands
6
1
Localizes pain
5
Withdraws from pain
4
Flexion with pain
3
Extension with pain
2
None
1
Total score is calculated by adding the component score for each category.
12+: minor brain injury with probable recovery 9–11: moderate severity requiring close observation for changes 8 or less: coma; 8 after 6 hours associated with 50% mortality
Management should focus on maintaining cerebral perfusion, maintaining airway patency, stabilizing spinal injuries, and treating any contamination of surrounding structures by esophageal perforation
Sites of significant (>1,500 mL) blood loss frequently not found by physical examination include blood left at the injury scene, pleural cavity bleeding (seen with CXR), intra-abdominal bleeding (seen with CT or US), pelvic bleeding (seen with CT), and bleeding into the thighs (seen on x-ray). Chest trauma Injuries can occur to the lungs, heart, and GI system Aortic injury, pneumothorax, and cardiac injury (e.g., perforation, tamponade) are the most emergently life-threatening injuries The physical examination should focus on clinical signs of injury to the major vascular structures, heart, and lungs Chest x-ray (CXR), chest CT, EGD, bronchoscopy, and angiography are potential useful tests to be considered if the patient is stable Management should focus on immediate stabilization of life-threatening injuries Abdominal trauma Injuries can occur to the abdominal viscera and abdominal vascular structures
Table A-9 Revised Trauma Score1
Coded Value 2
Glasgow Coma Sc ale Sc ore
Systolic Blood Pressure
Respiratory Rate
4
13–15
>89
10–29
3
9–12
76–89
>29
2
6–8
50–75
6–9
1
4–5
1–49
1–5
0
3
0
0
RTS = 0.9368 (GCS coded value) + 0.7326 (SBP coded value) + 0.2908 (RR coded value)
GCS, Glasgow coma scale; RR, respiratory rate; RTS, revised trauma score; SBP, systolic blood pressure.
1
RTS of 4 or greater is associated with 60% survival.
2
Coded value is assigned to each measurement individually.
P.338 Physical examination should look for signs of peritoneal irritation and peritoneal perforation Abdominal CT and the focused abdominal sonography for trauma (FAST) examination are the best means of detecting injury
Exploratory laparotomy is required for any penetrating injury, retroperitoneal vascular injury, or in an unstable patient with an intraperitoneal vascular injury
The hemodynamically unstable blunt trauma patient should be taken to the operating room and not to radiology.
A Foley catheter should never be placed in a patient with a suspected urethral rupture (e.g., blood seen at the urethral meatus, high-riding prostate on rectal examination) to avoid further urologic injury unless performed under cystoscopic guidance.
Serial neurovascular examinations should be performed following any type of treatment for an extremity to detect an evolving or iatrogenic neurologic injury. Genitourinary and pelvic trauma Injury may occur to the kidneys or lower urinary system Physical examination should look for signs of urologic bleeding (e.g., blood at the urethral meatus) and should include a pelvic examination in women Pelvic CT, retrograde urethrogram, or intravenous pyelogram should be considered to rule out urologic disruptions Disruption of the urologic tract must be treated surgically Extremity trauma Injury may involve bones, vasculature, soft tissues, or nerves in the extremities The physical examination should focus on the detection of gross instability and neurovascular deficits X-ray, CT, and MRI are the principal tools for detecting injuries Immediate management should focus on the stabilization of bony injuries and the maintenance of perfusion Trauma in expectant mothers Trauma is the leading cause of nonobstetric maternal death Inferior vena cava (IVC) compression by the uterus makes pregnant women more susceptible to poor cardiac output following injury Superior displacement of the bowel by the uterus decreases the risk of GI injury from lower abdominal trauma but increases the risk of GI injury from upper abdominal or chest trauma Although the risk of fetal death is low with minor injuries, it increases significantly in high-energy trauma Any trauma increases the risk of abruptio placentae The assessment must consider both the mother and the fetus Management should prioritize the needs of the mother
Criteria that should be met in posttraumatic pregnant females prior to discharge are contractions no more frequent than every ten minutes, no vaginal bleeding, no abdominal pain, and a normal fetal heart tracing.
A physician who has due cause to suspect child abuse but does not report it or act to protect the child may be held liable for subsequent injury or mortality.
Abuse and Sexual Assault Abuse of anyone may be physical, emotional, sexual, or exploitative Child abuse Neglect is the most prevalent form of child abuse and constitutes the failure to provide the physical, emotional, educational, and medical needs of a child Concerning findings during the assessment of an injured child include an injury inconsistent with the history, vague details, discrepancies in the history, delays in seeking treatment, a lack of parental concern, evidence of multiple injuries in various stages of healing, and injuries pathognomonic for abuse (e.g., rib fractures, femur fractures in nonambulatory children, burns in the shapes of objects) The treating physician has an obligation to report any suspected cases of child abuse to the legal child-protective services All suspected cases should be well documented Spousal/partner abuse Patients may present with vague complaints (e.g., chronic headaches, abdominal pains) Concerning findings during the assessment include an injury inconsistent with the history, vague details, inconsistencies in the history, delays in seeking
treatment, evidence of multiple injuries in various stages of healing, a hypervigilant partner, and recurrent sexually transmitted diseases (STDs) The patient should be interviewed without the partner present The initial approach should focus on the immediate safety of the patient P.339 Patients should be provided with information on safety plans, escape strategies, legal rights, and shelter locations, but the victim should not be forced into any action Reporting of this type of abuse is nonmandatory
The greatest risk of mortality in abused women is when they try to leave their abusers. Elder abuse Considered to be abuse in a patient >60 years old that occurs at the hands of a caregiver The concerning findings are similar to those seen in child and partner abuse The physician should contact social services to facilitate placement of the patient in a safe environment Reporting of this type of abuse is mandatory
Another health care worker (chaperone) must be present when a sexual examination is performed, and the patient should be made to feel as comfortable as possible with the history and physical examination. Sexual assault Nonconsensual sexual activity with physical contact Nonconsensual, forced intercourse is rape Victims may be children or adults A detailed history must be collected and thoroughly documented The examination should focus on the entire body with particular attention paid to the genitals, anus, and mouth to look for signs of assault In cases of rape, all injuries must be well documented and vaginal fluid and pubic hair should be collected for evidence (i.e., rape kit) Oral, vaginal, and penile fluid specimens should be tested for STDs Pregnancy testing should be performed to look for incidental conception occurring during assault The careful and well-documented collection of all details and evidence is important to future follow-up and legal action Referral to social support systems and counseling is very important
Toxicology Treatment principles The initial evaluation must focus on determining the type of poison involved Patient history, witness input, and clues found near the patient (e.g., empty bottles of medications, other medications) will help in making diagnosis Therapies for poisoning Induced vomiting: chemical induction of vomiting to empty the stomach of its contents; it is rarely performed because it is only useful in the initial 1 to 2 hours after ingestion and is contraindicated in caustic agents Activated charcoal: absorbs substances and blocks their systemic absorption but is not as useful in cases of ingested metals or alcohol Gastric lavage: suction drainage and saline lavage of the gastric contents via a nasogastric tube; it carries similar limitations to induced vomiting and must be performed within the initial hour of ingestion for much benefit Antidotes: chemicals that reverse or inhibit an ingested poison's activity Diuretics: may help in cases where increased urination helps remove toxin (e.g., salicylates, phenobarbital) Dialysis or exchange transfusion: used in cases of severe symptoms or when other treatments are unsuccessful Supportive care is routinely administered including airway stabilization, IV hydration, cardiac support, and anticonvulsant therapy when appropriate
Beware of the alcohol abuser that comes into the emergency department fictitiously saying that he has ingested ethylene glycol and needs ethanol for treatment; check for sweet breath and perform a toxicology screen before administering ethanol.
Organophosphates may also be absorbed through the skin, so all contaminated clothing must be removed from patients with
this type of poisoning. Ingested poisons May occur in children from the accidental ingestion of cleaning products, medications, or personal care products May occur in elderly patients from an accidental repeat dosing of usual medications May be intentional (i.e., suicide attempt) Identification of the ingested poison is important to dictating treatment (Table A-10) P.340
Table A-10 Common Poisons and Their Antidotes
Substanc e
Symptoms
Treatment
DRUGS
Acetaminophen
Nausea, hepatic insufficiency
N-acetylcysteine
Anticholinergics
Dry mouth, urinary retention, QRS widening on the ECG
Physostigmine
Benzodiazepines
Sedation, respiratory depression
Flumazenil
β-blockers
Bradycardia, hypotension, hypoglycemia, pulmonary edema
Glucagon, calcium, insulin, and dextrose
Calcium channel blockers
Bradycardia, hypotension
Glucagon, calcium, insulin, and dextrose
Cocaine
Tachycardia, agitation
Supportive care
Cyanide
Headache, nausea, vomiting, altered mental status
Nitrates, hydroxocobalamin
Digoxin
Nausea, vomiting, visual changes, arrhythmias
Digoxin antibodies
Heparin
Excessive bleeding, easy bruising
Protamine sulfate
Isoniazid
Neuropathy, hepatotoxicity
Vitamin B6
Isopropyl alcohol
Decreased consciousness, nausea, abdominal pain
Supportive care
Methanol
Headache, visual changes, dizziness
Ethanol, dialysis
Opioids
Pinpoint pupils, respiratory depression
Naloxone
Salicylates
Nausea, vomiting, tinnitus, hyperventilation, anion gap metabolic acidosis
Charcoal, dialysis, sodium bicarbonate
Sulfonylureas
Hypoglycemia
Octreotide and dextrose
Tricyclic antidepressants
Tachycardia, dry mouth, urinary retention, QRS widening on the ECG
Sodium bicarbonate, diazepam
Warfarin
Excessive bleeding, easy bruising
Vitamin K, FFP
Severe oropharyngeal and gastric irritation or burns, drooling, odynophagia, abdominal pain,
Copious irrigation (do not induce emesis or attempt neutralization),
symptoms of gastric perforation
activated charcoal
Ethylene glycol
Ataxia, hallucinations, seizures, sweet breath
Ethanol, dialysis
Organophosphates (e.g., insecticides,
Salivation, lacrimation, miosis, vomiting, diarrhea, increased urination, paralysis, decreased
Atropine, pralidoxime, supportive care
fertilizers)
consciousness
INDUSTRIAL CHEMICALS
Caustics (e.g., acids, alkali)
METALS
Iron
Nausea, constipation, hepatotoxicity
Deferoxamine
Lead
Peripheral neuropathy, anemia
EDTA, dimercaprol
Mercury
Renal insufficiency, tremor, mental status changes
Dimercaprol
ECG, electrocardiogram; EDTA, ethylenediamine tetraacetic acid; FFP, fresh frozen plasma.
P.341
Table A-11 Common Types of Venomous Bites and Stings
Type of Bite
Symptoms
Treatment
Snake (e.g., rattlesnake, copperhead, water
Pain and swelling at the bite, progressive dyspnea,
Immobilize the extremity and cleanse the wound
Effects are more severe in children
moccasin, coral snake)
toxin-induced DIC
A ntivenin is likely required
Increased mortality without prompt treatment
Scorpion
Severe pain and swelling at bite, increased
Antivenin, atropine, phenobarbital
Acute pancreatitis, myocardial toxicity,
sweating, vomiting, diarrhea
Spider (e.g., black widow, brown recluse)
Mammals
Human
Complic ations
respiratory paralysis
Abdominal pain, wound pain, vomiting, jaundice,
Black widow: calcium gluconate, methocarbamol
DIC
DIC
Brown recluse: dexamethasone, colchicine, dapsone
Brown recluse bites are more severe
Pain and swelling at bite, penetrating trauma
Saline irrigation, debridement, tetanus and rabies
Infection (e.g., staphylococci, Pasteurella
depending on the size of bite
prophylaxis, antibiotics for infection
multocida, rabies virus)
Pain and swelling at bite, tender local
Saline irrigation, broad coverage antibiotics, surgical
High incidence of infection with primary closure
lymphadenopathy
debridement, thorough documentation
or delayed presentation
DIC, disseminated intravascular coagulation.
Venomous bites and stings Poisonous bites can come from snakes (e.g., rattlesnake, copperhead, coral snake), spiders (e.g., black widow, brown recluse), or other less frequently encountered animals (e.g., scorpion) The venom in these bites may contain neurotoxins, cardiotoxins, or proteolytic enzymes that can potentially be fatal The treatment of a poisonous bite depends on the causative animal (Table A-11)
The symptoms of organophosphate poisoning can be remembered by the mnemonic DUDE SLOP: Diarrhea, Urination, Decreased consciousness, Emesis, Salivation, Lacrimation, Ocular miosis, Paralysis.
Body packing involves ingesting a large number of drug-filled latex and/or wax packets for the purpose of smuggling them; rupture of a packet may cause a massive overdose effect from a large quantity of the drug. P.342
Appendix E: Normal Lab Values
Hematology Test
Normal Range
Units
CD4 count
800–1,500
cells/mL
Free erythrocyte protoporphyrin
16–36
µg/dL
Haptoglobin
83–267
mg/dL
Hematocrit Female
34–48
Male
40–52
%
Hemoglobin Female
12.0–16.0
Male
13.5–17.5
g/dL
Hemoglobin A 2
2.0–3.3
%
Hemoglobin F
<2.0
%
IgA
80–350
mg/dL
IgD
0.3–3.0
mg/dL
IgE
0.002–0.2
mg/dL
IgG
620–1,400
mg/dL
IgM
45–250
mg/dL
Mean corpuscular hemoglobin concentration
31–36
g/dL
Mean corpuscular volume
80–100
fL
Platelets
150–400
1,000 units/µL
Reticulocyte count
0.5–2
%
White blood cells
4.0–11.0
1,000 cells/µL
Immunoglobulins
Serum Chemistry Test
Normal Range
Adrenocorticotropin hormone
Units
pg/mL
Morning (0800)
25–100
Afternoon (1800)
<50
Alanine aminotransferase
9–52
U/L
Albumin
3.5–5.8
g/dL
Aldosterone
ng/dL
Female
5–30
Male
6–22
Alkaline phosphatase
35–125
U/L
Ammonia
9–33
µmol/L
Amylase
0–140
U/L
Aspartate aminotransferase
14–36
U/L
Bilirubin, direct
0.0–0.4
mg/dL
Bilirubin, indirect
Total bili.–direct bili.
mg/dL
Bilirubin, total
0.0–1.2
mg/dL
Brain natriuretic peptide
0–99
pg/mL
Minimal heart failure
100–299
Mild heart failure
300–599
Moderate heart failure
600–899
Severe heart failure
900+
CA125
<35
U/mL
Calcium
8.5–10.5
mg/dL
Calcium, ionized
1.00–1.25
mmol/L
Carbon dioxide
22–30
mEq/L
Ceruloplasmin
27–37
mg/dL
Chloride
97–107
mEq/L
Cholesterol, total
<200
mg/dL
High density lipoprotein
>40
Low density lipoprotein
<130
Cortisol
µg/dL
Morning (0800)
5–23
Afternoon (1600)
3–15
C-reactive protein
0.1–0.9
mg/dL
Creatine kinase
20–315
U/L
0.0–5.0
ng/mL
Creatinine
0.8–1.3
mg/dL
D-dimer
0–300
ng/mL
Erythrocyte sedimentation rate
0–25
mm/hr
Myocardium fraction
DHEA
ng/mL
Female
1.4–8.0
Female (post-menopausal)
0.3–4.5
Male
0.5–5.5
Estradiol
pg/mL
Pre/post cycle
20–60
Midcycle
100–500
Post-menopausal
10–30
Ferritin
9–120
ng/mL
Fibrinogen
200–400
mg/dL
Folate
3.0–17.0
ng/mL
Follicle-stimulating hormone
U/L
Prepubertal
<5
Pre-/postcycle
4.6–22.4
Midcycle
13–41
Postmenopausal
30–170
Glucose
70–99
mg/dL
Growth hormone
0–4
ng/mL
Hemoglobin A1c
4.0–6.0
%
Human chorionic gonadotropin
U/L
Nonpregnant
<5
3 weeks pregnant
5–50
6 weeks pregnant
1,080–56,500
9–12 weeks pregnant
25,700–288,000
13–16 weeks pregnant
13,300–254,000
17–24 weeks pregnant
4,060–165,400
25–40 weeks pregnant
3,640–117,000
Postmenopausal
<9.5
Iron
37–170
µg/dL
Lactate
0.3–2.3
mEq/L
Lactate dehydrogenase
313–618
U/L
Lead
<10
µg/dL
Lipase
23–300
U/L
Luteinizing hormone
U/L
Prepubertal
<5
Pre-/postcycle
3–30
Midcycle
75–150
Postmenopausal
30–130
Magnesium
1.3–2.5
Osmolality
mg/dL mOsm/kg
Serum
275–295
Plasma
285–295
Parathyroid hormone
10–55
pg/mL
Phenylalanine
<3
mg/dL
Phosphorus
2.5–5.0
mg/dL
Potassium
3.5–5.3
mEq/L
Prolactin
ng/mL
Male
2–18
Female (nonpregnant)
2–29
Female (pregnant)
10–300
Prostate-specific antigen
0.0–4.0
ng/mL
Protein, total serum
6.1–8.5
g/dL
Sodium
133–143
mEq/L
Thyroid stimulating hormone
0.40–4.00
µU/mL
Thyroxine
4.5–12.5
µg/dL
Free T4 index
3.5–14.6
Transferrin
200–400
mg/dL
Triglycerides
<150
mg/dL
Triiodothyronine
0.7–1.7
ng/mL
T3 reuptake
0.77–1.17
Troponin-I
0.0–0.4
ng/mL
Urea nitrogen, blood
10–20
mg/dL
Vitamin B12
193–982
pg/mL
Vitamin D 1, 25-dihydroxycholecalciferol
25–45
pg/mL
Vitamin D 25-hydroxyvitamin D 3
15–80
ng/mL
P.343 P.344 P.345
Arterial Blood Gas Test
Normal Range
Units
pH
7.35–7.45
Oxygen
75–100
mmHg
Carbon dioxide
35–45
mmHg
Bicarbonate
22.0–26.0
mEq/L
Base excess
–3.0–3.0
mEq/L
Oxygen saturation
94.0–99.0
%
Carboxyhemoglobin
0–2.3
%
Toxicity
>20
Lethality
>50
Sweat Chemistry Test
Normal Range
Units
Chloride
3–35
mEq/L
Sodium
10–40
mEq/L
Urine Chemistry Test
Normal Range
Units
Calcium
100–250
mg/day
Chloride
<8
mEq/L
110–250
mEq/day
Copper
15–30
µg/day
Cortisol
10–100
µg/day
Creatinine
800–2,800
mg/day
Osmolality
80–1,300
mOsm/kg
pH
4.5–8.0
Porphobilinogen
0–2.0
mg/day
Potassium
Protein
Sodium
Specific gravity
<8
mEq/L
35–85
mEq/day
1–15
mg/dL
0–150
mg/day
10–40
mEq/L
15–25
mEq/day
<1.030
P.346
Stool Chemistry Test
Normal Range
Osmolality
~ 290
pH
6.1–7.9
Units
mOsm/kg
Coagulation Test
Normal Range
Units
Prothrombin time
10.4–13.2
sec
Partial thromboplastin time
21.8–32.5
sec
INR
Dependent on PT
Cerebral Spinal Fluid Test
Normal Range
Units
Glucose
40–70
mg/dL
Opening pressure
50–100
mm H2O
Protein
20–45
mg/dL
White blood cells
<5
cells/µL
P.347
Appendix F: Abbreviations
Abbreviations %
Percent °F
Degrees Fahrenheit AAA
Abdominal aortic aneurysm A-a gradient
Alveolar-arterial gradient ABC
Airway, Breathing, Circulation ABG
Arterial blood gas ABI
Ankle-brachial index AC
Assist-control ventilation
ACA
Anterior cerebral artery ACE-I
Angiotensin-converting enzyme inhibitor ACL
Anterior cruciate ligament ACTH
Adrenocorticotropic hormone ADH
Antidiuretic hormone ADHD
Attention-deficit/hyperactivity disorder ADP
Adenosine diphosphate Afib
Atrial fibrillation Aflutter
Atrial flutter AI
Apnea Index AICA
Anterior inferior cerebellar artery AIDS
Acquired immune deficiency syndrome α-IFN
α-interferon AlkPhos
Alkaline phosphatase ALL
Acute lymphocytic leukemia ALS
Amyotrophic lateral sclerosis ALT
Alanine aminotransferase AML
Acute myelogenous leukemia ANA
Antinuclear antibodies APUD
Amine-precursor-uptake and decarboxylation AR
Aortic regurgitation ARB
Angiotensin receptor blocker ARDS
Acute respiratory distress syndrome ARF
Acute renal failure AS
Aortic stenosis ASA
Aspirin ASCA
Anti-saccharomyces cerevisiae antibodies ASCH
Atypical squamous cells, cannot exclude HSIL ASCUS
Atypical squamous cells of undetermined significance ASD
Atrial septal defect AST
Aspartate aminotransferase ATN
Acute tubular necrosis AV
Arteriovenous or atrioventricular AVM
Arteriovenous malformation AXR
Abdominal x-ray Bicarb
Bicarbonate P.348 BMI
Body mass index BNP
Brain natriuretic peptide BP
Blood pressure BPH
Benign prostatic hypertrophy bpm
Beats per minute BPPV
Benign paroxysmal positional vertigo BSA
Body surface area BUN
Blood urea nitrogen Ca
Calcium CABG
Coronary artery bypass grafting CAD
Coronary artery disease CAH
Congenital adrenal hyperplasia cANCA
Cytoplasmic antineutrophil cytoplasmic antibodies CBC
Complete blood cell count CEA
Carcinoembryonic antigen CF
Cystic fibrosis CFTR
Cystic fibrosis transmembrane conductance regulator Chem10
10-electrolyte chemistry panel Chem7
7-electrolyte chemistry panel CHF
Congestive heart failure CI
Confidence interval CICU
Cardiac intensive care unit CIN
Cervical intraepithelial neoplasia CK
Creatine kinase (a.k.a. creatine phosphokinase) CKD
Chronic kidney disease CK-MB
Creatine kinase myocardial component Cl
Chloride CLL
Chronic lymphocytic leukemia cm
Centimeter CML
Chronic myelogenous leukemia CMT
Charcot-Marie-Tooth disease CMV
Controlled mechanical ventilation CN
Cranial nerve CNS
Central nervous system CO
Cardiac output CO2
Carbon dioxide Coags
Coagulation panel COPD
Chronic obstructive pulmonary disease CPAP
Continuous positive airway pressure CPPD
calcium pyrophosphate dehydrate deposition CPRS
Complex regional pain syndrome Cr
Creatinine CRP
C-reactive protein CSF
Cerebral spinal fluid CT
Computed tomography CVA
Cerebrovascular accident CXR
Chest x-ray DBili
Direct bilirubin DCIS
Ductal carcinoma in–situ DDAVP
Demopressin acelate DDH
Developmental dysplasia of the hip DES
Diffuse esophageal spasm DEXA
Dual energy x-ray absorptiometry DHEA
Dehydroepiandrosterone DI
Diabetes insipidus DIC
Disseminated intravascular coagulation DIP
Distal interphalangeal P.349 DKA
Diabetic ketoacidosis dL
Deciliters DLco
Diffusing capacity of lungs DM
Diabetes mellitus DMSA
2,3-dimercaptosuccinic acid DNR
Do-not-resuscitate DTR
deep tendon reflex DVT
Deep vein thrombosis EBV
Epstein-Barr virus ECG
Electrocardiogram ECMO
Extracorporeal membrane oxygenation ECT
Electroconvulsive therapy EDTA
Ethylenediamine tetraacetic acid EEG
Electroencephalogram EF
Ejection fraction EGD
Esophagogastroduodenoscopy ELISA
Enzyme-linked immunosorbent assay EMG
Electromyography EMT
Emergency medical technician EPO
Erythropoietin ERCP
Endoscopic retrograde cholangiopancreatography ERV
Expiratory reserve volume ESR
Erythrocyte sedimentation rate EtOH
Ethyl alcohol FAP
Familial adenomatous polyposis FAST
Focused abdominal sonography for trauma FDG-PET
Fluorodeoxyglucose positron emission tomography Fe
Iron FEF25%–75%
Forced expiratory flow rate from 25%–75% of functional vital capacity FENa
Fractional excretion of sodium FEV 1
Forced expiratory volume in 1 second FFP
Fresh frozen plasma FH
Familial hypercholesterolemia Fio2
Fraction of inspired oxygen fL
Femtoliters FNA
Fine needle aspiration FRC
Functional reserve capacity FSH
Follicle-stimulating hormone FTA-ABS
Fluorescent treponemal antibody absorption FVC
Functional vital capacity g
Grams G6PD
Glucose 6-phosphate dehydrogenase GABA
ϓ-aminobutyric acid GAD
Generalized anxiety disorder GBM
Glioblastoma multiforme GBS
Group B streptococcus GCS
Glasgow coma scale GERD
Gastroesophageal reflux disease GGT
Gamma glutamyl transpeptidase GH
Growth hormone GHB
Gamma hydroxybutyrate GI
Gastrointestinal Glu
Glucose GnRH
Gonadotropin-releasing hormone H2O
Water HAART
Highly active antiretroviral treatment HAV
Hepatitis A virus P.350 HbA1c
Hemoglobin A1c HBV
Hepatitis B virus hCG
Human chorionic gonadotropin HCO32
Bicarbonate HCTZ
Hydrochlorothiazide HCV
Hepatitis C virus HDL
High-density lipoprotein HDV
Hepatitis D virus HEV
Hepatitis E virus Hgb
Hemoglobin HgbCO
Carboxyhemoglobin HHNC
Hyperosmolar hyperglycemic nonketotic coma
HIDA
Hepatic iminodiacetic HIT
Heparin-induced thrombocytopenia HIV
Human immunodeficiency virus HLA
Human leukocyte antigen HNPCC
Hereditary nonpolyposis colorectal cancer HOCM
Hypertrophic obstructive cardiomyopathy HPV
Human papilloma virus hr
Hours HR
Heart rate HSIL
High grade squamous intraepithelial lesion HSV
Herpes simplex virus HTN
Hypertension HUS
Hemolytic uremic syndrome IBD
Inflammatory bowel disease Ibili
Indirect bilirubin IBS
Irritable bowel syndrome IC
Inspiratory capacity ICP
Intracranial pressure ICU
Intensive care unit I+D
Incision and drainage Ig
Immunoglubulin IMV
Intermittent mandatory ventilation INH
Isoniazid INR
International normalized ratio IPF
Idiopathic pulmonary fibrosis IRB
Institutional review board IRV
Inspiratory reserve volume ITP
Idiopathic thrombocytopenia purpura IUD
Intrauterine device IUGR
Intrauterine growth restriction IV
Intravenous IVC
Inferior vena cava IVDA
Intravenous drug abuse IVDU
Intravenous drug use IVF
Intravenous fluids IVIG
Intravenous immunoglobulin IVP
Intravenous pyelogram JRA
Juvenile rheumatoid arthritis JVD
Jugular venous distention K
Potassium kg
Kilograms KOH
Potassium hydroxide L
Liters LAD
Left anterior descending artery LBBB
Left bundle branch block P.351 LBO
Large bowel obstruction lb
Pounds LCIS
Lobular carcinoma in situ LDH
Lactate dehydrogenase LDL
Low-density lipoprotein LES
Lower esophageal sphincter LFT
Liver function test LH
Luteinizing hormone LMWH
Low molecular weight heparin LSD
Lysergic acid diethylamide LSIL
Low grade squamous intraepithelial lesion LVH
Left ventricular hypertrophy MAC
Mycobacterium avis complex MAOI
Monoamine oxidase inhibitors MAP
Mean arterial pressure MAT
Multifocal atrial tachycardia MCA
Middle cerebral artery MCL
Medial collateral ligament MCP
Metacarpophalangeal MCV
Mean corpuscular volume MEN
Multiple endocrine neoplasia mEq
Milliequivalents mg
Milligrams Mg
Magnesium MGUS
Monoclonal gammopathy of undetermined significance MHA-TP
Microhemagglutination assay for antibodies to treponemes MI
Myocardial infarction MICU
Medical intensive care unit min
Minutes mL
Milliliters MLF
Medial longitudinal fasciculus mm
Millimeters mm Hg
Millimeters of mercury mmol
Millimoles MMR
measles, mumps rubella vaccine MMSE
Minimental status examination mOsm
Milliosmoles MR
Mitral regurgitation MRA
Magnetic resonance angiography MRI
Magnetic resonance imaging MRSA
Methicillin-resistent S. aureus MS
Mitral stenosis MSH
Melanocyte-stimulating hormone MSSA
Methicillin-sensitive S. aureus Na
Sodium NCS
Nerve conduction study ng
Nanograms NICU
Neonatal intensive care unit NMDA
N-methyl-D-aspartate NPH
Neutral protamine Hagedorn (insulin) NPO
Nulla per orem (i.e., nothing by mouth) NPV
Negative predictive value NSAID
Nonsteroidal anti-inflammatory drug O2
Oxygen OA
Osteoarthritis OCD
Obsessive-compulsive disorder OCP
Oral contraceptive pill P.352 OR
Odds ratio pANCA
Perinuclear antineutrophil cytoplasmic antibodies PAo2
Alveolar oxygen pressure Pao2
Arterial oxygen pressure PAPP-A
Pregnancy-associated plasma protein A PBC
Primary biliary cirrhosis PCA
Patient-controlled analgesia PCA
Posterior cerebral artery PCO2
Partial pressure of carbon dioxide PCOS
Polycystic ovary syndrome PCP
Primary care provider PCR
Polymerase chain reaction PDA
Patent ductus arteriosus PE
Pulmonary embolism PEA
Pulseless electrical activity PEEP
Positive end-expiratory pressure PEFR
Peak expiratory flow rate PET
Positron emission tomography PFT
Pulmonary function test pg
Picograms Phos
Phosphorus PICA
Posterior inferior cerebellar artery PICC
Peripheral indwelling central catheter PID
Pelvic inflammatory disease PIP
Proximal interphalangeal Plt
Platelets PMDD
Premenstrual dystrophic disorder PMN
Polymorphonuclear cells PMR
Polymyalgia rheumatica PMS
Premenstrual syndrome PO2
Partial pressure of oxygen PPD
Purified protein derivative PPI
Proton pump inhibitor PPV
Positive predictive value PRBC
Packed red blood cells PROM
Premature rupture of membranes PSA
Prostate-specific antigen PSC
Primary sclerosing cholangitis PSVT
Paroxysmal supraventricular tachycardia PT
Protime PTA
Percutaneous transluminal angioplasty PTCA
Percutaneous transluminal coronary angioplasty PTH
Parathyroid hormone PTSD
Posttraumatic stress disorder PTT
Partial thromboplastin time PTU
Propylthiouracil PTX
Pneumothorax PUD
Peptic ulcer disease PVC
Premature ventricular complex PVD
Peripheral vascular disease RA
Rheumatoid arthritis RAST
Radioallergosorbent testing RBBB
Right bindle branch block RBC
Red blood cell REM
Rapid eye movement RF
Rheumatoid factor RHD
Rheumatic heart disease P.353 RPR
Rapid plasma reagin RR
Relative risk RR
Respiratory rate RSD
Reflex sympathetic dystrophy RSV
Respiratory syncytial virus RTS
Revised trauma score RV
Residual volume RVH
Right ventricular hypertrophy SBO
Small bowel obstruction SCFE
Slipped capital femoral epiphysis SCID
Severe combined immunodeficiency syndrome sec
Seconds SIADH
Syndrome of inappropriate antidiuretic hormone secretion SIMV
Synchronized intermittent mandatory ventilation SLE
Systemic lupus erythematosus SNF
Skilled nursing facility SNRI
Serotonin–norepinephrine reuptake inhibitor SPECT
Single-positron emission computed tomography SPEP
Serum protein electrophoresis SSRI
Selective serotonin reuptake inhibitor STD
Sexually transmitted disease SV
Stroke volume T
Temperature TAH-BSO
Total abdominal hysterectomy and bilateral salpingo-oophorectomy TB
Tuberculosis TBG
Thyroid-binding globulin TBili
Total bilirubin TCA
Tricyclic antidepressant TEE
Transesophageal echocardiogram TEN
Toxic epidermal necrosis TENS
Transcutaneous electrical nerve stimulation TIA
Transient ischemic attack TIPS
Transjugular intrahepatic portal-caval shunting TLC
Total lung capacity TMP-SMX
Trimethoprim-sulfamethoxazole TNF
Tumor necrosis factor TPN
Total parenteral nutrition TRH
Thyrotropin-releasing hormone Trig
Triglycerides Trop-I
Troponin I TSH
Thyroid-stimulating hormone TSIs
Thyroid-stimulating immunoglobulins TTP-HUS
Thrombotic thrombocytopenic purpura-hemolytic uremic syndrome TURP
Transurethral resection of the prostate TV
Tidal volume U
Units UA
Urinalysis UC
Ulcerative colitis UPEP
Urine protein electrophoresis URI
Upper respiratory infection US
Ultrasound UTI
Urinary tract infection VDRL
Venereal Disease Research Laboratory test Vfib
Ventricular fibrillation VIP
Vasoactive intestinal peptide V/Q scan
Ventilation-perfusion scan VSD
Ventricular septal defect P.354 Vtach
Ventricular tachycardia vWF
von Willebrand factor VZV
varicella zoster vaccine WBC
White blood cells WPW
Wolff-Parkinson-White syndrome XRT
X-ray therapy µg
Micrograms µL
Microliters µmol
Micromoles µU
Microunits
Color Plates
COLOR FIGURE 1-1 Four examples of patterns detected during technetium-99m-sestamibi nuclear stress testing with single-positron emission computed tomography imaging. (A) Normal patient. Homogenous thickening of the myocardium occurs during systole. Less uptake (arrows) is noted in the inferior wall in the end-diastolic short axis and vertical long axis views that is likely due to the spatial relationship of the heart to the diaphragm. (B) Patient with a septal infarct. Note the perfusion defects (arrows) in the ventricular septum in the short and horizontal long axis views in both the end-systolic and end-diastolic phases. (C) Patient with inferior wall ischemia. End-diastolic perfusion defects (arrows) are noted on the short and vertical long axis views. Slight signal increase occurs in the end-systolic views indicating reversible ischemia. (D) Patient with both ischemia and infarction. Septal and anteroapical perfusion defects are noted in the end-diastolic images. Only the septal region shows signal increase in the end-systolic images indicating reversible septal ischemia and anteroapical infarction. ED, end-diastolic; ES, end-systolic; HLA, horizontal long axis; SA, short axis; VLA, vertical long axis. (Taken from Sandler MP, Gottschalk A, Patton JA, et al., eds. Diagnostic nuclear medicine. 4th ed. Philadelphia: Lippincott Williams & Wilkins, 2002. Fig. 15-16. Used with permission of Lippincott Williams & Wilkins.) (See black and white image.)
COLOR FIGURE 2-1 Paired Gram-positive cocci seen in sputum consistent with Streptococcus pneumoniae pneumonia. (Taken from McClatchey KD. Clinical laboratory medicine. 2nd ed. Philadelphia: Lippincott Williams & Wilkins, 2002. Used with permission of Lippincott Williams & Wilkins.) (See black and white image.)
COLOR FIGURE 2-2 Numerous acid-fast bacilli seen in sputum specimen consistent with Mycobacterium tuberculosis infection. (Taken from Rubin E, Farber JL. Pathology. 3rd ed. Philadelphia: Lippincott Williams & Wilkins, 1999. Used with permission of Lippincott Williams & Wilkins.) (See black and white image.)
COLOR FIGURE 3-1 Duodenal ulcer in a tissue specimen. The ulcer is located distal to the musculature of the pyloric sphincter and demonstrates penetration of the mucosal surface. (Taken from Rubin E, Farber JL. Pathology. 3rd ed. Philadelphia: Lippincott Williams & Wilkins, 1999. Fig. 13.19. Used with permission of Lippincott Williams & Wilkins.) (See black and white image.)
COLOR FIGURE 3-2 Giardiasis; several trophozoites are seen with characteristic pear shape and paired nuclei resembling owls' eyes. (Taken from Fenoglio-Preiser CM, Lantz PE, Listrom MB, et al. Gastrointestinal pathology. an atlas and text. 2nd ed. Philadelphia: Lippincott Williams & Wilkins, 1999. Used with permission of Lippincott Williams & Wilkins.) (See black and white image.)
COLOR FIGURE 3-3 Hepatic steatosis due to chronic alcohol consumption. Numerous hepatocytes are distended due to cytoplasmic fat collections. (Taken from Rubin E, Farber JL. Pathology. 3rd ed. Philadelphia: Lippincott Williams & Wilkins, 1999. Fig. 14.31. Used with permission of Lippincott Williams & Wilkins.) (See black and white image.)
COLOR FIGURE 3-4 Jaundice in a patient with hyperbilirubinemia. Note the yellow sclera and skin compared to the normal hue of the examiner's hand. (Taken from Bickley LS, Szilagyi P. Bate's guide to physical examination and history taking. 8th ed. Philadelphia: Lippincott Williams & Wilkins, 2003. Used with permission of Lippincott Williams & Wilkins.) (See black and white image.)
COLOR FIGURE 3-5 Photograph of a Kayser-Fleischer ring in a patient with Wilson disease. Note the brownish ring obscuring details of the peripheral iris. (Taken from Gold DH, Weingeist TA. Color atlas of the eye in systemic disease. Baltimore: Lippincott Williams & Wilkins, 2001. Fig. 89.1. Used with permission of Lippincott Williams & Wilkins.) (See black and white image.)
COLOR FIGURE 4-2 Immunofluorescence microscopy of a renal biopsy in a patient with Goodpasture syndrome. Note the linear deposition of anti-basement membrane IgG antibodies along the glomerular basement membrane. Ig, immunoglobulin. (Taken from Rubin E, Farber JL. Pathology. 3rd ed. Philadelphia: Lippincott Williams & Wilkins, 1999. Fig. 16-50. Used with permission of Lippincott Williams & Wilkins.) (See black and white image.)
COLOR FIGURE 4-1 Autosomal dominant polycystic kidney disease; note enlargement of the kidney with many cysts of various sizes. (Taken from Rubin E, Farber JL. Pathology. 3rd ed. Philadelphia: Lippincott Williams & Wilkins, 1999. Used with permission of Lippincott Williams & Wilkins.) (See black and white image.)
COLOR FIGURE 4-3 Renal biopsy with silver stain from a patient with membranous glomerulonephritis. Note the numerous “spikes” of immune complex deposition along the glomerular basement membrane. (Taken from Rubin E, Farber JL. Pathology. 3rd ed. Philadelphia: Lippincott Williams & Wilkins, 1999. Fig. 16-20. Used with permission of Lippincott Williams & Wilkins.) (See black and white image.)
COLOR FIGURE 4-4 Immunofluorescence microscopy of a renal biopsy in a patient with membranoproliferative glomerulonephritis. Note the granular deposition of antibasement membrane IgG antibodies along the glomerular basement membrane. This granular appearance is more consistent with deposition of immune complexes along the basement membrane and may be contrasted with the linear appearance seen in Goodpasture syndrome. Ig, immunoglobulin. (Taken from Rubin E, Farber JL. Pathology. 3rd ed. Philadelphia: Lippincott Williams & Wilkins, 1999. Fig. 16-39. Used with permission of Lippincott Williams & Wilkins.) (See black and white image.)
COLOR FIGURE 5-1 Photograph of a female patient with Graves disease. There is noticeable exophthalmos of both eyes, and a goiter is seen in the neck. (Taken from Rubin E, Farber JL. Pathology. 3rd ed. Philadelphia: Lippincott Williams & Wilkins, 1999. Fig. 21-13. Used with permission of Lippincott Williams & Wilkins.) (See black and white image.)
COLOR FIGURE 5-2 Diagram of the common physical characteristics seen in Cushing syndrome. (Taken from Smeltzer SC, Bare BG. Textbook of medical-surgical nursing. 9th ed. Philadelphia: Lippincott Williams & Wilkins, 2000. Fig. 38-03.01. Used with permission of Lippincott Williams & Wilkins.) (See black and white image.)
COLOR FIGURE 6-1 Lead poisoning anemia; note the hypochromic red blood cells and basophilic stippling seen in some cells. (Taken from Anderson SC, Poulsen KB. Anderson's atlas of hematology. Philadelphia: Lippincott Williams & Wilkins, 2003. Fig. IIA2-18. Used with permission of Lippincott Williams & Wilkins.) (See black and white image.)
COLOR FIGURE 6-2 Hereditary spherocytosis; blood smear shows numerous spherocytes with decreased diameter, increased staining, and absence of central pallor. (Taken from Rubin E, Farber JL. Pathology. 3rd ed. Philadelphia: Lippincott Williams & Wilkins, 1999. Fig. 20-27. Used with permission of Lippincott Williams & Wilkins.) (See black and white image.)
COLOR FIGURE 6-3 Microangiopathic hemolytic anemia demonstrating multiple schistocytes (fragmented red blood cells). (Taken from Rubin E, Farber JL. Pathology. 3rd ed. Philadelphia: Lippincott Williams & Wilkins, 1999. Fig. 20-31. Used with permission of Lippincott Williams & Wilkins.) (See black and white image.)
COLOR FIGURE 6-4 Microcytic hypochromic red blood cells characteristic of iron deficiency anemia. (Taken from Rubin E, Farber JL. Pathology. 3rd ed. Philadelphia: Lippincott Williams & Wilkins, 1999. Fig. 20-22. Used with permission of Lippincott Williams & Wilkins.) (See black and white image.)
COLOR FIGURE 6-5 Blood smear in a patient with sideroblastic anemia; note several red blood cells surrounded by rings of iron granules (ring sideroblasts). (Taken from Handin RI, Lux SE, Stossel TP. Blood: principles and practice of hematology. 2nd ed. Philadelphia: Lippincott Williams & Wilkins, 2003. Color Fig. 3-6D. Used with permission of Lippincott Williams & Wilkins.) (See black and white image.)
COLOR FIGURE 6-6 α-thalassemia, hemoglobin H type; note hypochromic red blood cells and occasional target cells. (Taken from Anderson SC, Poulsen KB. Anderson's atlas of hematology. Philadelphia: Lippincott Williams & Wilkins, 2003. Fig. IIA2-2. Used with permission of Lippincott Williams & Wilkins.) (See black and white image.)
COLOR FIGURE 6-7 Anemia due to vitamin B12 deficiency; note the macrocytic red blood cells and presence of a hypersegmented neutrophil. (Taken from Anderson SC, Poulsen KB. Anderson's atlas of hematology. Philadelphia: Lippincott Williams & Wilkins, 2003. Fig. IIA3-3. Used with permission of Lippincott Williams & Wilkins.)
COLOR FIGURE 6-8 Sickle cell anemia; note multiple sickle cells and occasional target cells. (Taken from Rubin E, Farber JL. Pathology. 3rd ed. Philadelphia: Lippincott Williams & Wilkins, 1999. Fig. 20-26. Used with permission of Lippincott Williams & Wilkins.)
COLOR FIGURE 6-9 Blood smear from a patient with Plasmodium infection showing two microgametes with large nuclei and loose chromatin (arrow) and several red blood cells with intracellular rings and eosinophilic granules signifying infiltration. (Taken from Sun T. Parasitic disorders: pathology, diagnosis, and management. 2nd ed. Baltimore: Lippincott Williams & Wilkins, 1999. Fig. 16.24. Used with permission of Lippincott Williams & Wilkins.)
COLOR FIGURE 6-10 Acute myelogenous leukemia with monocytic differentiation. Note the prominent waxy nucleoli, large size of blasts, and presence of Auer rods. (Taken from Handin RI, Lux SE, Stossel TP. Blood: principles and practice of hematology. 2nd ed. Philadelphia: Lippincott Williams & Wilkins, 2003. Color Fig. 15-1A. Used with permission of Lippincott Williams & Wilkins.).
COLOR FIGURE 6-11 Acute lymphocytic leukemia. Note the lymphoblasts with irregular nuclei and prominent nucleoli. (Taken from Rubin E, Farber JL. Pathology. 3rd ed. Philadelphia: Lippincott Williams & Wilkins, 1999. Fig. 20-59. Used with permission of Lippincott Williams & Wilkins.)
COLOR FIGURE 6-12 Chronic lymphocytic leukemia. Note the small lymphocytes of a comparable size to nearby red blood cells and the presence of smudge cells (fragile lymphocytes disrupted during smear preparation) in upper portion of image. (Taken from Rubin E, Farber JL. Pathology. 3rd ed. Philadelphia: Lippincott Williams & Wilkins, 1999. Fig. 20-57. Used with permission of Lippincott Williams & Wilkins.)
COLOR FIGURE 6-13 Hodgkin lymphoma. A histologic section of a lymph node demonstrates pathognomonic binucleated Reed-Sternberg cells that resemble owls' eyes. (Taken from Rubin E, Farber JL. Pathology. 3rd ed. Philadelphia: Lippincott Williams & Wilkins, 1999. Fig. 20-72. Used with permission of Lippincott Williams & Wilkins.)
COLOR FIGURE 8-1 Patient with an age-related cataract, demonstrating complete opacification of the lens. (Taken from Rubin E, Farber JL. Pathology. 3rd ed. Philadelphia: Lippincott Williams & Wilkins, 1999. Fig. 29-3. Used with permission of Lippincott Williams & Wilkins.) (See black and white image.)
COLOR FIGURE 7-1 Patient with multiple neurofibromas on the face and chest consistent with neurofibromatosis type I. (Taken from Rubin E, Farber JL. Pathology. 3rd ed. Philadelphia: Lippincott Williams & Wilkins, 1999. Fig. 6-19. Used with permission of Lippincott Williams & Wilkins.) (See black and white image.)
COLOR FIGURE 8-3 Fundoscopic view in a patient with retinal artery occlusion. Note the generalized retinal edema and the presence of a cherry-red spot. (Taken from Gold DH, Weingeist TA. Color atlas of the eye in systemic disease. Philadelphia: Lippincott Williams & Wilkins, 2001. Fig. 75-2. Used with permission of Lippincott Williams & Wilkins.)
COLOR FIGURE 8-2 Fundoscopic view in a patient with diabetic retinopathy. Note the yellowish lipid exudates and multiple small retinal hemorrhages. (Taken from Rubin E, Farber JL. Pathology. 3rd ed. Philadelphia: Lippincott Williams & Wilkins, 1999. Fig. 29-11A. Used with permission of Lippincott Williams & Wilkins.)
COLOR FIGURE 8-4 Fundoscopic view in a patient with retinal vein occlusion. Note the edematous retina, retinal hemorrhages, cotton wool spots, and venous dilation. (Taken from Gold DH, Weingeist TA. Color atlas of the eye in systemic disease. Philadelphia: Lippincott Williams & Wilkins, 2001. Fig. 29-1. Used with permission of Lippincott Williams & Wilkins.)
COLOR FIGURE 8-5 Leukokoria in a child with a left-eye retinoblastoma. (Taken from Rubin E, Farber JL. Pathology. 3rd ed. Philadelphia: Lippincott Williams & Wilkins, 1999. Fig. 29-23A. Used with permission of Lippincott Williams & Wilkins.)
COLOR FIGURE 9-1 Photograph of the hands in a patient with rheumatoid arthritis. Note the ulnar deviation of the fingers and metacarpophalangeal hypertrophy. (Taken from Smeltzer SC, Bare BG. Textbook of medical-surgical nursing. 9th ed. Philadelphia: Lippincott Williams & Wilkins, 2000. Fig, 50-4B. Used with permission of Lippincott Williams & Wilkins.) (See black and white image.)
COLOR FIGURE 9-2 Patient with Lyme disease exhibiting erythema chronicum migrans (bull's eye rash). (Taken from Goodheart HP. Goodheart's photoguide of common skin disorders. 2nd ed. Philadelphia: Lippincott Williams & Wilkins, 2003. Fig. 7-19. Used with permission of Lippincott Williams & Wilkins.) (See black and white image.)
COLOR FIGURE 9-3 Synovial aspirate from a patient with gout; note the needle-shaped negatively birefringent sodium urate crystals that are visible under polarized light microscopy. (Taken from McClatchey KD. Clinical laboratory medicine. 2nd ed. Philadelphia: Lippincott Williams & Wilkins, 2002. Fig. 27-17. Used with permission of Lippincott Williams & Wilkins.) (See black and white image.)
COLOR FIGURE 9-4 Synovial aspirate from patient with calcium pyrophosphate dehydrate deposition disease; under polarized light microscopy rhomboid-shaped calcium pyrophosphate dehydrate crystals appear positively birefringent. (Taken from McClatchey KD. Clinical laboratory medicine. 2nd ed. Philadelphia: Lippincott Williams & Wilkins, 2002. Fig. 27-22. Used with permission of Lippincott Williams & Wilkins.) (See black and white image.)
COLOR FIGURE 9-5 Patient exhibiting the classic malar rash of systemic lupus erythematosus. (Taken from Goodheart HP. Goodheart's photoguide of common skin disorders. 2nd ed. Philadelphia: Lippincott Williams & Wilkins, 2003. Fig. 2-63. Used with permission of Lippincott Williams & Wilkins.) (See black and white image.)
COLOR FIGURE 10-1 Allergic contact dermatitis due to exposure to poison ivy; note the linearity of the rash consistent with an outside-of-body cause. (Taken from Goodheart HP. Goodheart's photoguide of common skin disorders. 2nd ed. Philadelphia: Lippincott Williams & Wilkins, 2003. Fig. 2-48. Used with permission of Lippincott Williams & Wilkins.) (See black and white image.)
COLOR FIGURE 10-2 Cellulitis of the right pretibial region; note the erythematous, swollen skin with mild desquamation. (Taken from Goodheart HP. Goodheart's photoguide of common skin disorders. 2nd ed. Philadelphia: Lippincott Williams & Wilkins, 2003. Fig. 2-69. Used with permission of Lippincott Williams & Wilkins.)
COLOR FIGURE 10-3 Adult atrophic dermatitis (eczema) characterized by erythematous patches of dry skin. (Taken from Goodheart HP. Goodheart's photoguide of common skin disorders. 2nd ed. Philadelphia: Lippincott Williams & Wilkins, 2003. Fig. 2-8. Used with permission of Lippincott Williams & Wilkins.)
COLOR FIGURE 10-4 Impetigo involving left nostril due to Staphylococcus aureus infection; note presence of greasy yellow scales within lesion. (Taken from Smeltzer SC, Bare BG. Textbook of medical-surgical nursing. 9th ed. Philadelphia: Lippincott Williams & Wilkins, 2000. Fig. 52-1. Used with permission of Lippincott Williams & Wilkins.)
COLOR FIGURE 10-5 Multiple erythematous pustules on the face of an adolescent patient consistent with acne vulgaris. (Taken from Sauer GC, Hall JC. Manual of skin diseases. 7th ed. Philadelphia: Lippincott-Raven Publishers, 1996, Fig. 32-6. Used with permission of Lippincott Williams & Wilkins.)
COLOR FIGURE 10-6 Herpes simplex; these perioral vesicles are more indicative of infection with herpes simplex virus type 1 than of type 2. (Taken from Weber J, Kelley J. Health assessment in nursing. 2nd ed. Philadelphia: Lippincott Williams & Wilkins, 2003. Display 13-1a. Used with permission of Lippincott Williams & Wilkins.)
COLOR FIGURE 10-7 Chickenpox in a child due to varicella zoster infection. Although the small crusted vesicles are distributed across the body in the childhood form, reactivated infection in adults (shingles) occurs in a single dermatome. (Taken from Goodheart HP. Goodheart's photoguide of common skin disorders. 2nd ed. Philadelphia: Lippincott Williams & Wilkins, 2003. Fig. 8-2. Used with permission of Lippincott Williams & Wilkins.)
COLOR FIGURE 10-8 Tinea corporis; fungal infection of skin characterized by scaly rash on the body with central clearing and a papular border. (Taken from Goodheart HP. Goodheart's photoguide of common skin disorders. 2nd ed. Philadelphia: Lippincott Williams & Wilkins, 2003. Fig. 4-12. Used with permission of Lippincott Williams & Wilkins.)
COLOR FIGURE 10-9 Palm of a patient with scabies. Note the multiple red papules and a visible mite burrow (white arrow). (Taken from Goodheart HP. Goodheart's photoguide of common skin disorders. 2nd ed. Philadelphia: Lippincott Williams & Wilkins, 2003. Fig. 20-14. Used with permission of Lippincott Williams & Wilkins.)
COLOR FIGURE 10-10 Toxic epidermal necrolysis. This severe dermatologic condition begins as a generalized erythematous rash that progresses into widespread desquamation and erosion formation. (Taken from Elder D, Johnson B Jr, Ioffreda M, et al. Synopsis and atlas of Lever's histopathology of the skin. Philadelphia: Lippincott Williams & Wilkins, 1999. Clin. Fig. IVC2.c. Used with permission of Lippincott Williams & Wilkins.)
COLOR FIGURE 10-11 Pemphigus vulgaris. Fragile bullae develop which rupture easily leading to widespread erosions and desquamation. (Taken from Elder D, Johnson B Jr, Ioffreda M, et al. Synopsis and atlas of Lever's histopathology of the skin. Philadelphia: Lippincott Williams & Wilkins, 1999. Clin. Fig. IVD3.b. Used with permission of Lippincott Williams & Wilkins.)
COLOR FIGURE 10-12 Bullous pemphigoid. Multiple large bullae form on an erythematous base leading to severe erosions. (Taken from Elder D, Johnson B Jr, Ioffreda M, et al. Synopsis and atlas of Lever's histopathology of the skin. Philadelphia: Lippincott Williams & Wilkins, 1999. Clin. Fig. IVE3. Used with permission of Lippincott Williams & Wilkins.)
COLOR FIGURE 10-13 Red plaques with silver scales on the extensor forearm surface of a patient with psoriasis; similar lesions may also be seen on the extensor surfaces of the knee. (Taken from Goodheart HP. Goodheart's photoguide of common skin disorders. 2nd ed. Philadelphia: Lippincott Williams & Wilkins, 2003. Fig. 2-23. Used with permission of Lippincott Williams & Wilkins.)
COLOR FIGURE 10-14 Pityriasis rosea. These scaled papules fan out across the chest or back to give the overall appearance of a Christmas tree pattern. (Taken from Barankin B, Lin AN, Metelitsa AI. Stedman's illustrated dictionary of dermatology eponyms. Baltimore: Lippincott Williams & Wilkins, 2004. Used with permission of Lippincott Williams & Wilkins.)
COLOR FIGURE 10-15 Melanoma, superficial spreading type; note the ABCDs of the lesion—asymmetry, irregular border, inconsistent color, and large diameter (>20 mm). (Taken from Rubin E, Farber JL. Pathology. 3rd ed. Philadelphia: Lippincott Williams & Wilkins, 1999. Fig. 24-51. Used with permission of Lippincott Williams & Wilkins.)
COLOR FIGURE 10-17 Squamous cell carcinoma with erythematous base and ulceration. (Taken from Rubin E, Farber JL. Pathology. 3rd ed. Philadelphia: Lippincott Williams & Wilkins, 1999. Fig. 24-76A. Used with permission of Lippincott Williams & Wilkins.)
COLOR FIGURE 10-16 Melanocytic nevus; unlike melanoma, this lesion is near symmetrical, has better border regularity, is a more consistent color, and is a smaller diameter. (Taken from Goodheart HP. Goodheart's photoguide of common skin disorders. 2nd ed. Philadelphia: Lippincott Williams & Wilkins, 2003. Fig. 21-1. Used with permission of Lippincott Williams & Wilkins.)
COLOR FIGURE 10-18 Basal cell carcinoma; note the pearly appearance of a papule with central ulceration. (Taken from Goodheart HP. Goodheart's photoguide of common skin disorders. 2nd ed. Philadelphia: Lippincott Williams & Wilkins, 2003. Fig. 22-17. Used with permission of Lippincott Williams & Wilkins.)
COLOR FIGURE 10-19 Actinic keratosis; these lesions are superficial papules covered by dry scales and are a result of sun exposure. (Taken from Sauer GC. Manual of skin diseases. 5th ed. Philadelphia: JB Lippincott, 1985, Table 4-4. Used with permission of Lippincott Williams & Wilkins.)
COLOR FIGURE 11-1 Pathology specimen of an ovary demonstrating the characteristic powder-burn lesions of endometriosis. (Taken from Rubin E, Farber JL. Pathology. 3rd ed. Philadelphia: Lippincott Williams & Wilkins, 1999. Fig. 18-27A. Used with permission of Lippincott Williams & Wilkins.) (See black and white image.)
COLOR FIGURE 11-2 Colposcopy view of the cervix with multiple lesions consistent with human papilloma virus infection. Application of acetic acid to the cervix during the examination has made these lesions more apparent. (Taken from Goodheart HP. Goodheart's photoguide of common skin disorders. 2nd ed. Philadelphia: Lippincott Williams & Wilkins, 2003. Fig. 19.6. Used with permission of Lippincott Williams & Wilkins.) (See black and white image.)